Вы находитесь на странице: 1из 753

The following items, Copyright © by the American Institute of Certified Public Accountants, Inc.

, are reprinted with 
per­mission:
1. Material from Uniform CPA Examination Questions and Answers, 1978 through 2001. 
2. Information for Uniform CPA Examination Candidates, Board of Examiners, 2000.
3. Material from the Certified Internal Auditor Examination, Copyright  © 1994 through 1997 by the Institute of In­
ternal Auditors, Inc., are reprinted and/or adapted with permission. 
4. Material from the Certified Management Accountant Examinations, Copyright  © 1993 through 1997 by the In­
stitute of Certified Management Accountants, are reprinted and/or adapted with permission. 
From a declaration of principles jointly adopted by a Committee of the American Bar Association and a Committee of
Publishers.

This book is printed on acid­free paper.
Copyright © 2011 by John Wiley & Sons, Inc.  All rights reserved.
Published by John Wiley & Sons, Inc., Hoboken, New Jersey
Published simultaneously in Canada.
No part of this publication may be reproduced, stored in a retrieval system, or transmitted in any form or by any means, 
electronic, mechanical, photocopying, recording, scanning, or otherwise, except as permitted under Section 107 or 108 of
the 1976 United States Copyright Act, without either the prior written permission of the publisher, or authorization 
through payment of the appropriate per­copy fee to the Copyright Clearance Center, 222 Rosewood Drive, Danvers, MA
01923, 978­750­8400, fax 978­646­8600, or on the web at www.copyright.com. Requests to the publisher for permis­
sion should be addressed to the Permissions Department, John Wiley & Sons, Inc., 111 River Street, Hoboken, NJ 
07030, 201­748­6011, fax 201­748­6008, or online at http:/www.wiley.com/go/permissions
Limit of Liability/Disclaimer of Warranty: While the publisher and author have used their best efforts in preparing this 
book, they make no representations or warranties with respect to the accuracy or completeness of the contents of this 
book and specifically disclaim any implied warranties of merchantability or fitness for a particular purpose. No warranty 
may be created or extended by sales representatives or written sales materials. The advice and strategies contained herein
may not be suitable for your situation. You should consult with a professional where appropriate. Neither the publisher 
nor author shall be liable for any loss of profit or any other commercial damages, including but not limited to special, 
incidental, consequential, or other damages.
For general information on our other products and services or for technical support, please contact our Customer Care
Department within the US at 800­762­2974, outside the US at 317­572­3993 or fax 317­572­4002.
Wiley also publishes its books in a variety of electronic formats. Some content that appears in print may not be available
in electronic books. For more information about Wiley products, visit our Web site at www.wiley.com
ISBN: 978­0­470­92384­9 (Volume 2 paperback); 978­1­118­10714­0 (ebk); 978­1­118­10715­7 (ebk);
978­1­118­10716­4 (ebk)
ISBN: 978­0­470­93241­4 (Set)
Printed in the United States of America.
10 9 8 7 6 5 4 3 2 1
PREFACE
Passing the CPA exam upon your first attempt is possible! TheWiley CPA Examination Reviewpreparation ma­
terials provide you with the necessary materials (visit our Web site at www.wiley.com/cpa for more information). It’s up 
to you to add the hard work and commitment. Together we can beat the first­time pass rate of less than 50%. All Wiley 
CPA products are continuously updated to provide you with the most comprehensive and complete knowledge base. 
Choose your products from the Wiley preparation materials and you can proceed confidently. You can select support 
materials that are exam­based and user­friendly. You can select products that will help you pass!
The first purpose of Volume 2 is to provide CPA candidates with sample examination problems/questions organized
by topic (e.g., internal control, consolidations, etc.). This text includes over 2,600 multiple­choice questions. These 
questions provide an effective means of studying the material tested on past exams; however, it is also necessary to work
with task­based simulations to develop the solutions approach (the ability to solve CPA questions and problems 
efficiently).
The second objective of this volume is to explain the AICPA unofficial answers and author question answers to the 
examination problems/questions included in this text. The AICPA published all CPA examinations and unofficial an­
swers through the November 1995 exam and selected questions and answers since then. No explanation is made, how­
ever, of the procedures that should have been applied to the examination problem to obtain the unofficial answers. Relat­
edly, the unofficial answers to multiple­choice questions provide no justification and/or explanation. This text provides 
explanations of both how to work problems and the unofficial answers to multiple­choice questions.
This text is designed to be used in conjunction with Volume 1, Outlines and Study Guides, but may be used with or
without any other study source. Both volumes are organized into 47 manageable study units (modules) to assist candi­
dates in organizing their study programs. The multiple­choice questions in this volume are grouped into topical catego­
ries (submodules) that correspond to the sequencing of material as it appears in Volume 1.
New author­constructed questions have been added to this Thirty­Eighth Edition. As new questions and problems 
are added, older ones are deleted. New problems have been added to address the new content of the exam that was 
adopted January 1, 2011.
A Sample Examination for each of the four parts of the exam is included in the Appendix at the end of this volume.
The CPA exam is one of the toughest exams you will ever take. It will not be easy. But if you follow our guidelines 
and focus on your goal, you will be thrilled with what you can accomplish.

Ray Whittington
April 2011

Don’t forget to visit our Web site at www.wiley.com/cpa
for supplements and updates.
This page intentionally left blank
CONTENTS
1   HOW TO USE THIS BOOK 1

2 AUDITING AND ATTESTATION 7
*
Module
Abbreviations Numbers
Professional Responsibilities RESP 1 8
Engagement Planning, Obtaining an Understanding
of the Client and Assessing Risks ENPL 2 24
Understanding Internal Control and Assessing
Control Risk IC 3 67
Responding to Risk Assessment:  Evidence
Accumulation and Evaluation EVID 4 113
Reporting REPT 5 163
Accounting and Review Services AARS 6 202
Audit Sampling AUDS 7 214
Auditing with Technology ATEC 8 230

3   FINANCIAL ACCOUNTING AND REPORTING 239

Basic Theory and Financial Reporting TREP 9 241


A.  Basic Concepts 241
B.  Error Correction 268
C.  Accounting Changes 280
D.  Financial Statements 295
Inventory INVY 10 333
Fixed Assets FA 11 361
Monetary Current Assets and Current Liabilities CACL 12 392
Present Value PV 13 426
A.  Fundamentals 426
B.  Bonds 437
C.  Debt Restructure 456
D.  Pensions 460
E.  Leases 473
Deferred Taxes DETX 14 494
Stockholders’ Equity STK 15 513
Investments IVES 16 546
Statement of Cash Flows SCF 17 573
Business Combinations and Consolidations BCC 18 592
Derivative Instruments and Hedging Activities DIHA 19 617
Miscellaneous MISC 20 634
A.  Personal Financial Statements 634
B.  Interim Reporting 638
C.  Segment Reporting 642
D.  Partnership Accounting 649
E.  Foreign Currency Translation 661
Governmental (State and Local) Accounting GOV 21 670
Not­for­Profit Accounting NFP 22 695

4 REGULATION 711

*
As explained in Chapter 1, this volume is organized into 47 modules (manageable study units). Volume 1 is organized in a parallel
fashion. 
5 PROFESSIONAL RESPONSIBILITIES AND BUSINESS LAW 713

Module
Abbreviations Numbers
Professional and Legal Responsibilities PLR 23 714
Federal Securities Acts FEDE 24 730
Business Structure BSTR 25 742
Contracts CONT 26 761
Sales SALE 27 778
Commercial Paper CPAP 28 793
Secured Transactions SECU 29 810
Bankruptcy BANK 30 818
Debtor­Creditor Relationships DBCR 31 829
Agency AGEN 32 837
Regulation of Employment and Environment EMEN 33 845
Property PROP 34 856

6   FEDERAL TAXATION 861

Individual Taxation ITAX 35 862


Transactions in Property TPRO 36 931
Partnerships Taxation PTAX 37 948
Corporate Taxation CTAX 38 974
Gift and Estate Taxation GETX 39 1031

7   BUSINESS ENVIRONMENT AND CONCEPTS 1055

Corporate Governance, Internal Control, and
Enterprise Risk Management CGIC 40 1056
Information Technology IFTC 41 1066
Economics, Strategy, and Globalization ECON 42 1093
Financial Risk Management and Capital Budgeting RMCB 43 1114
Financial Management FINM 44 1132
Performance Measures PERM 45 1158
Cost Measurement COST 46 1171
Planning, Control, and Analysis PLAN 47 1186

APPENDIX A: SAMPLE EXAMINATIONS 1207

APPENDIX B: SAMPLE TESTLETS RELEASED FROM THE AICPA 1299

APPENDIX C: 2011 RELEASED AICPA QUESTIONS 1323
ABOUT THE AUTHORS
Ray Whittington, PhD, CPA, CMA, CIA, is the dean of the College of Commerce at DePaul University. Prior 
tojoining the faculty at DePaul, Professor Whittington was the Director of Accountancy at San Diego State University. 
From 1989 through 1991, he was the Director of Auditing Research for the American Institute of Certified Public Ac­
countants (AICPA), and he previously was on the audit staff of KPMG. He previously served as a member of the Audit­
ing Standards Board of the AICPA and as a member of the Accounting and Review Services Committee and the Board of
Regents of the Institute of Internal Auditors. Professor Whittington has published numerous textbooks, articles, mono­
graphs, and continuing education courses.

Patrick R. Delaney, deceased, was the dedicated author and editor of theWiley CPA Exam Reviewbooks for 
twentyyears. He was the Arthur Andersen LLP Alumni Professor of Accountancy and Department Chair at Northern 
Illinois University. He received his PhD in Accountancy from the University of Illinois. He had public accounting 
experience with Arthur Andersen LLP and was coauthor of GAAP: Interpretation and Application, also published by 
John Wiley & Sons, Inc. He served as Vice President and a member of the Illinois CPA Society’s Board of Directors, and
was Chair­man of its Accounting Principles Committee; was a past president of the Rockford Chapter, Institute of 
Management Ac­countants; and had served on numerous other professional committees. He was a member of the 
American Accounting Association, American Institute of Certified Public Accountants, and Institute of Management 
Accountants. Professor Delaney was published in The Accounting Review and was a recipient of the Illinois CPA 
Society’s Outstanding Educator Award, NIU’s Excellence in Teaching Award, and Lewis University’s Distinguished 
Alumnus Award. He was involved in NIU’s CPA Review Course as director and instructor.

ABOUT THE CONTRIBUTORS
Anita L. Feller, PhD, CPA, is a Lecturer in Accountancy at the University of Illinois at Urbana­Champaign. She 
isthe recipient of the Commerce Council Excellence in Teaching Award. Formerly a staff accountant in the tax 
department at KPMG Peat Marwick, she teaches courses in financial accounting and applied professional research. 
Professor Feller also teaches financial and managerial topics in the University of Illinois CPA Review Course. She is 
also the author of
Mastering Accounting Research for the CPA Exam and Wiley CPA Exam Review Impact Audios (both published 
byWiley).
Edward C. Foth, PhD, CPA, is the Administrator of the Master of Science in Taxation Program at DePaul Univer­
sity. He is a member of the American Accounting Association, the American Institute of Certified Public Accountants, 
the Illinois CPA Society, and the American Taxation Association. He has been a recipient of the Ledger & Quill Faculty 
Excellence Award, and is the author of CCH Incorporated’s Federal Tax Study Manual, Federal Taxation 
RefresherCourse, and is coauthor of their S Corporations Guide. He prepared Chapter 6.
Mark L. Frigo, PhD, CPA, CMA, is Director of The Center for Strategy, Execution and Valuation in 
KellstadtGraduate School of Business and Eichenbaum Foundation Distinguished Professor of Strategy and 
Leadership in the School of Accountancy at DePaul University. Professor Frigo authored the Performance Measures
module for the Business Environment and Concepts section of this manual.
Kurt Pany, PhD, CPA, is a Professor of Accounting at Arizona State University. Prior to entering academe,
heworked   as  a  staff  auditor   for  Deloitte   and  Touche   LLP.  He  is  a  former   member   of  the  AICPA’s  Auditing
Standards Board and has taught in the Arizona State University CPA Review Course.
Patricia L. Smith, MBA, CPA, is an instructor in the School of Accountancy at DePaul University. She has 
abackground in auditing and financial accounting. She has audit experience with KPMG. She contributed to the auditing
modules.
This page intentionally left blank
HOW TO USE THIS BOOK

This volume is a collection of CPA questions, task­based simulations and solutions. The text is designed 
and organized to be used in conjunction with Volume 1, CPA Examination Review: Outlines and StudyGuides,
but may be used with or without any other study source. Each module in this volume corresponds to amodule 
in Volume 1. In this volume, a module consists of
1. Multiple­choice questions 
2. Task­based simulations 
3. Written communication tasks 
4. Unofficial answers for the multiple­choice questions with the author’s explanations 
5. Answers for task­based simulations 
The sources of the questions and task­based simulations material in each module are, of course, the four
sections of the CPA Exam. The modules are grouped together into chapters, which correspond with the four
sections of the exam. The following table shows how the modules are organized in Volume 2.
This manual has been updated for the major changes that occurred in the CPA exam effective January 1,
2011.
CPA EXAM SECTION AND CORRESPONDING MODULES
Auditing and Attestation CH 2 Modules 1­6
Financial Accounting and Reporting CH 3 Modules 7­20
Regulation CH 5 & 6 Modules 21­37
Business Environment and Concepts CH 7 Modules 38­45
The material in Volume 2 allows candidates to work CPA exam questions, some of which have appeared on
previous examinations. This provides candidates with an effective method of studying the material tested on the
exam. However, candidates should also realize that the CPA has new material. Therefore, candidates should use
the study guides and outlines included in Volume 1 as a means of bridging this gap.
Also included at the end of this volume are sample exams for all four sections. They are included to en­
able candidates to gain experience in taking a “realistic” exam. While working through the modules, the can­
didate can become accustomed to concentrating on a fairly narrow range of topics. By taking the sample ex­
aminations near the end of their study program, candidates will be better prepared for taking the actual 
examination.
Before you begin working the CPA questions in this volume, peruse the table of contents and scan through
the book, noting the manner in which the chapters and modules are organized. The schedule at the beginning 
of each chapter provides an index to the questions appearing in each module. Some questions have been 
modified to reflect recent changes in law or practice. For a complete analysis of recent examinations and the 
AICPA Uniform CPA Examination Content Specification Outlines of future examinations, see the beginning 
of each chapter starting with Chapter 5 in Volume 1, Outlines and Study Guides.
Multiple­Choice Questions
The multiple­choice questions and answer explanations can be used in many ways. First, they may be 
used as a diagnostic evaluation of your knowledge. For example, before beginning to review audit sampling 
you may wish to answer every fourth multiple­choice question to determine your ability to answer CPA ex­
amination questions on audit sampling. The apparent difficulty of the questions and the correctness of your 
answers will allow you to determine the necessary breadth and depth of your review. Additionally, exposure 
to examination questions prior to review and study of the material should provide motivation. You will de­
velop a feel for your level of proficiency and an understanding of the scope and difficulty of past examination
questions.
Second, the multiple­choice questions can be used as a poststudy or postreview evaluation. You should 
attempt to understand all concepts mentioned (even in incorrect answers) as you answer the questions. Refer 
to the explanation of the answer for discussion of the alternatives even though you selected the correct re­
2 CH 1   HOW TO USE THIS BOOK

sponse. Thus, you should read the explanation of the unofficial answer unless you completely understand the
question and all of the alternative answers.
Third, you may wish to use the multiple­choice questions as a primary study vehicle. This is probably the 
quickest, but least thorough, approach. Make a sincere effort to understand the question and to select the cor­
rect reply before referring to the answer and explanation. In many cases the explanations will appear inade­
quate because of your unfamiliarity with the topic.
The multiple­choice questions in Volume 2 are grouped into study sets. The study sets include a smaller 
amount of related material than the study modules, which provides greater flexibility in the individual candi­
date’s study strategy. The answer explanations for the multiple­choice questions in Volume 2 also include 
headings which provide cross­references to the text material in Volume 1. For example, in Module 11, Fixed 
Assets, the heading “F. Depreciation” appears above the answers to those questions dealing with depreciation.
The topical coverage of depreciation in Volume 1 can then be found by referring to the corresponding 
heading within Module 11.
One of the benefits of working through multiple­choice questions is that it helps you to identify your 
weak areas. Once you have graded your answers, your strong areas and weak areas should be clearly evident. 
Yet, the important point here is that you should not stop at a simple percentage evaluation. The percentage 
only provides general feedback about your knowledge of the material contained within that particular module.
The percentage does not give you any specific feedback regarding the concepts which were tested. In order to
get this feedback, you should look at the questions missed on an individual basis because this will help you 
gain a better understanding of why you missed the question. This feedback process has been facilitated by the 
fact that within each module where the multiple­choice answer key appears, two blank lines have been 
inserted next to the multiple­choice answers. As you grade the multiple­choice questions, mark those 
questions which you have missed. However, instead of just marking the questions right and wrong, you 
should now focus on marking the questions in a manner which identifies why you missed the question. As an 
example, a candidate could mark the questions in the following manner: for math mistakes, x for conceptual 
mistakes, and ? for areas which the candidate was unfamiliar with. The candidate should then correct these 
mistakes by rework­ing through the marked questions. The objective of this marking technique is to help you 
identify your weak areas and thus, the concepts which you should be focusing on. While it is still important 
for you to get be­tween 75% and 80% correct when working multiple­choice questions, it is more important 
for you to under­stand the concepts. This understanding applies to both the questions answered correctly and 
those answered incorrectly. Remember, most of the questions on the CPA exam will be different from the 
questions in the book; however, the concepts will be the same. Therefore, your preparation should focus on 
understanding concepts, not just getting the correct answer.
The multiple­choice questions substantially outnumber the task­based simulations in this book. This is
similar to what can be expected on the CPA exam. The multiple­choice questions make up about between
60% and 85% (depending on the section) of the total examination.
One difficulty with so many multiple­choice questions is that you may overemphasize them. Candidates 
generally prefer to work multiple­choice questions because they are shorter and less time consuming and 
solv­able with less effort.
Another difficulty with the large number of multiple­choice questions is that you may tend to become 
overly familiar with the questions. The result may be that you may begin reading the facts and assumptions of 
previously studied questions into the questions on your examination. Guard against this potential problem by 
reading each multiple­choice question with extra care.
Although not as critical as for task­based simulations, the solutions approach (a systematic problem­
solving methodology) is relevant to multiple­choice questions.
Multiple­Choice Screen Layout
The following is a computer screenshot that illustrates the manner in which multiple­choice questions will
be presented:
CH 1   HOW TO USE THIS BOOK 3
As indicated previously, multiple­choice questions will be presented in three individual testlets of 24 to 
30 questions each. Characteristics of the computerized testlets of multiple­choice questions include the 
follow­ing:
1. You may move freely within a particular testlet from one question to the next or back to previous 
questions until you click the “Exit” button. Once you have indicated that you have finished the 
testlet by clicking on the “Exit” button and reconfirmed, you can never return to that set of 
questions. 
2. A button on the screen will allow you to “flag” a question for review if you wish to come back to
it later. 
3. A four­function computer calculator with an electronic tape is available as a tool. 
4. The time remaining for the entire exam section is shown on the screen. 
5. The questions will be shown at the bottom of the screen. You may navigate between questions
by simply clicking on the question number. 
6. The “Help” button will provide you with help in navigating and completing the testlet. 
The screenshot above was obtained from the AICPA’s sample exam at www.cpa­exam.org. Candidates
are   urged   to   complete   the   tutorial   and   other   example   questions   on   the   AICPA’s   Web   site   to   obtain
additional experience with the computer­based testing.
Multiple­Choice Questions Solutions Approach
1. Work individual questions in order. 
a. If a question appears lengthy or difficult, skip it until you can determine that extra time is
avail­able. Mark it for review to remind you to return to it later. 
2. Read the stem of the question without looking at the answers. 
a. The   answers   are   sometimes   misleading   and   may   cause   you   to   misread   or   misinterpret   the
question. 
3. Read each question carefully to determine the topical area. 
a. Study the requirements first so you know which data are important 
b. Note keywords and important data 
c. Identify pertinent information 
d. Be especially careful to note when the requirement is an  exception  (e.g., “Which of the
following is not an effective disclaimer of the implied warranty of merchantability?”). 
e. If a set of data is the basis for two or more questions, read the requirements of each of the 
ques­tions before beginning to work the first question (sometimes it is more efficient to 
work the ques­tions out of order) 
4 CH 1   HOW TO USE THIS BOOK

f. Be alert to read questions as they are, not as you would like them to be. You may encounter a
fa­miliar   looking  item;   don’t   jump  to  the   conclusion  that   you  know  what   the   answer   is
without read­ing the question completely. 
4. Anticipate the answer before looking at the alternative answers. 
a. Recall the applicable principle (e.g., offer and acceptance, requisites of negotiability, etc.)
and the respective applications thereof. 
b. If a question deals with a complex area, it may be very useful to set up a timeline or diagram
using abbreviations. 
5. Read the answers and select the best alternative. 
6. Click on the correct answer (or your educated guess). 
7. After completing all of the questions including the ones marked for review click on the “Done”
button to close out the testlet. Remember once you have closed out the testlet you can never re­
turn to it. 
Currently, all multiple­choice questions are scored based on the number correct, weighted by a difficulty 
rating (i.e., there is no penalty for guessing). The rationale is that a “good guess” indicates knowledge. Thus, 
you should answer all multiple­choice questions.
Task­Based Simulations
Simulations are case­based problems designed to
• Test integrated knowledge 
• More closely replicate real­world problems 
• Assess research and other skills 
Any of the following types of responses might be required on task­based simulations:
• Drop­down selection 
• Numeric and monetary inputs 
• Formula answers 
• Check box response 
• Enter spreadsheet formulas 
• Research results 
To complete the simulations, candidates are provided with a number of tools, including
• A four­function computer calculator with an electronic tape 
• Scratch spreadsheet 
• The ability to split windows horizontally or vertically to show two tabs on the screen (e.g., you can
examine the situation tab in one window and a requirement tab in a second window) 
• Access to professional literature databases to answer research requirements 
• Copy and paste functions 
In addition, the resource tab provides other resources that may be needed to complete the simulation. For 
example, a resource tab might contain a present value table for use in answering a lease problem. A window 
on the screen shows the time remaining for the entire exam, and the “Help” button provides instructions for 
navigating the simulation and completing the requirements.
Task­Based Simulations Solutions Approach
The following solutions approach is suggested for answering simulations:
1. Review the entire background and problem. Get a feel for the topical area and related concepts 
thatare being tested. Even though the format of the question may vary, the exam continues to test your 
understanding of applicable principles or concepts. Relax, take a deep breath, and determine your strategy for 
conquering the simulation. 
2. Identify the requirements of the simulation.  This step will help you focus in more quickly on
thesolution(s) without wasting time reading irrelevant material. 
3. Study the items to be answered. As you do this and become familiar with the topical area 
beingtested, you should review the concepts of that area. This will help you organize your thoughts 
so that you can relate logically the requirements of the simulation with the applicable concepts. 
CH 1   HOW TO USE THIS BOOK 5

4. Use the scratch paper (which will be provided) and the spreadsheet and calculator tools to
assist you in answering the simulation. 
You are urged to complete the tutorial and other sample tests that are on the AICPA’s Web site 
(www.cpa­exam.org) to obtain additional experience with the interface and computer­based testing.
Task­Based Research Simulations
One research simulation will be included on the Auditing and Attestation, Financial Accounting and 
Reporting, and Regulation sections of the exam. Research simulations require candidates to search the 
professional literature and income tax code in electronic format and interpret the results. The table below
describes the research material that will be available for each section of the exam that includes research 
simulations.
Section Potential research resources
Auditing and Attestation • AICPA Statements on Auditing Standards and
Interpretations (AU)
•  PCAOB Auditing Standards and Interpretations
(PCAOB)
•  AICPA Statements on Attestation Standards and
Interpretations (AT)
•  AICPA Statements on Standards for Accounting and
Review Services and Interpretations (AR)
•  AICPA Code of Professional Conduct (ET)
• AICPA Bylaws (BL)
•  AICPA Statements on Standards for Valuation Services
(VS)
•  AICPA Statement on Standards for Consulting Services
(CS)
•  AICPA Statements on Standards for Quality Control
(QC)
•  AICPA Peer Review Standards (PR)
•  AICPA Statement on Standards for Tax Services (TS)
•  AICPA Statement on Responsibilities in Personal
Financial Planning Practice (PFP)
•  AICPA Statement on Standards for Continuing
Professional Education Programs (CPE)
Financial Accounting and Reporting • FASB Accounting Standards Codification
Regulation • Internal Revenue Code
The research material may be searched using the table of contents or a keyword search. Therefore, 
knowing important code sections, FASB codification sections, auditing standards section numbers, etc. 
may speed up your search.
If possible, it is important to get experience using an electronic version of the research databases to 
sharpen your skills. If that is not available, you should use the printed copy of the professional standards and
the IRS code and regulations to answer the simulations in the manual. Remember, the AICPA offers an 
electronic version of professional standards to registered candidates. Refer to the AICPA Web site at 
www.cpa­exam.org.
Written Communication Tasks
The Business Environment and Concepts section of the exam will require the completion of three 
written communication questions. Communication questions will involve some real­world writing 
assignment that a CPA might have to perform, such as a memorandum to a client explaining a management 
technique. The subject of the communication will be a Business Environment and Concepts topic.
It is essential for the communication to be in your own words. In addition, the communication will not be
graded for technical accuracy. If it is on point, it will only be graded for usefulness to the intended user and 
writing skills. The following screenshot illustrates a task requiring the composition of a memorandum to a 
company president.
6 CH 1   HOW TO USE THIS BOOK

Diagnose Your Weaknesses prior to the Exam
This volume of questions, task­based simulations and solutions provides you with an opportunity to 
diagnose and correct any exam­taking weaknesses prior to your taking the examination. Continuously analyze
the contributing factors to incomplete or incorrect solutions to CPA questions prepared during your study 
program. General categories of candidates’ weaknesses include
1. Failure to understand the exam question requirements 
2. Misunderstanding the supporting text of the question 
3. Lack of knowledge of material tested, especially recently issued pronouncements 
4. Failure to develop proficiency with practice tools such as electronic research databases and spread­
sheets 
5. Inability to apply the solutions approach 
6. Lack of an exam strategy (e.g., allocation of time) 
7. Sloppiness and computational errors 
8. Failure to proofread and edit 
Time Management
Each section of the CPA exam will contain a number of multiple­choice testlets, and all of the sections ex­
cept Business Environment and Concepts will contain two simulations. As you complete each testlet keep 
track of how you performed in relation to the AICPA suggested times. After you finish the multiple­choice 
testlets, budget your time for the simulations based on your remaining time and the AICPA suggested times. 
For example, if you have two hours remaining to complete two simulations that each have the same AICPA 
suggested time, budget one hour for each simulation. Remember that you alone control watching your prog­
ress towards successfully completing this exam.
Additional Study Aids
A more complete discussion of the solutions approach, including illustrations thereof, appears in Chapter 3
of Volume 1, Outlines and Study Guides. Additionally, use of “note cards” as an integral part of your study 
program is discussed and illustrated in Chapter 1 of Volume 1. Chapter 4 of Volume 1 includes a detailed 
checklist to assist candidates with their last­minute preparation and to provide guidance concerning the actual 
taking of the exam.

NOW IS THE TIME
TO MAKE A COMMITMENT
AUDITING AND ATTESTATION

The Auditing and Attestation Exam is scheduled for four and one­half hours.  Based on information released by the
AICPA, candidates should expect three multiple­choice testlets of approximately thirty questions each, and two 
simulations.The Uniform CPA Examination Content Specifications appear in Volume 1, Outlines and Study 
Guides.

Module 1/Professional Responsibilities (RESP)
39 Multiple­Choice 8 17
7 Task­Based Simulations 12 21

Module 2/Engagement Planning, Obtaining an Understanding of the Client and
Assessing Risks (ENPL)
113 Multiple­Choice 24 47
14 Task­Based Simulations 34 59

Module 3/Understanding Internal Control and Assessing Control Risk (IC)
166 Multiple­Choice 67 91
9 Task­Based Simulations 82 109

Module 4/Responding to Risk Assessment: Evidence Accumulation and Evaluation
(EVID)
169 Multiple­Choice 113 137
13 Task­Based Simulations 128 156

Module 5/Reporting (REPT)
164 Multiple­Choice 163 183
4 Task­Based Simulations 179 200

Module 6/Accounting and Review Services (AARS)
49 Multiple­Choice 202 208
1 Task­Based Simulations 207 213

Module 7/Audit Sampling (AUDS)
58 Multiple­Choice 214 222
3 Task­Based Simulations 220 229

Module 8/Auditing with Technology (ATEC)
32 Multiple­Choice 230 234
1 Task­Based Simulation 233 238

Auditing and Attestation Sample Examination 1207

Auditing and Attestation Regulation AICPA Sample Testlets 1299

Auditing and Attestation 2011 Released AICPA Questions 1323
7
8 MODULE 1   PROFESSIONAL RESPONSIBILITIES

PROFESSIONAL RESPONSIBILITIES
MULTIPLE­CHOICE QUESTIONS (1­39)
attest
engage
1. Which   of   the   following   best ment. 
describes   what   is   meant   by   the d. Management selection of the CPA
term generally accepted auditing firm. 
standards? 
a. Rules   acknowledged 4. Which of the following is a
by   the   accounting “self   review”   threat   to
profession   because   of member independence? 
their   universal a. An
application.  engagemen
b. Pronouncements issued t   team
by   the   Auditing member
Standards Board.  has   a
c. Measures   of   the spouse that
quality   of   the serves   as
auditor’s   perfor­ CFO of the
mance.  attest
client. 
d. Procedures   to   be   used   to   gather
b. A   second
evidence   to   support   financial
partner
statements. 
review   is
2. For which of the following can a required on
member of the AICPA receive an all   attest
automatic   expulsion   from   the engagemen
AICPA?  ts. 
c. An
I.  Member is convicted of a felony.
engagement
II.  Member files his own fraudulent tax 
team
return.
member
BI. Member   files
fraudulent   tax   return
prepares
for   a   client   knowing invoices for
that it is fraudulent.  the   attest
client. 
a. I only.  d. An
b. I and II only.  engagement
c. I and III only.  team   member
d. I, II, and III.  has   a   direct
financial
3. Which of the following is an
interest   in   the
example   of   a   safeguard attest client. 
implemented by the client that
might   mitigate   a   threat   to 5. According to the standards of the
independence?  profession,   which   of   the
a. Requir following   circumstances   will
ed prevent a CPA performing audit
contin engagements   from   being
uing independent? 
educati a. Obtaining   a
on   for collateralized
all automobile
attest loan   from   a
engage financial
ment institution
team client. 
membe b. Litigation
rs.  with   a   client
b. An   effective   corporate relating   to
governance structure.  billing   for
c. Requir con­sulting
ed services   for
second which   the
partner amount   is
review immate­rial. 
of   an c. Employment
of the CPA’s b. Actual   fee   would   be
spouse   as   a substantially   lower
client’s   di­ than the fees charged
rector   of by   other   CPAs   for
internal comparable ser­vices.
audit.  c. CPA would not be independent. 
d. Acting as an d. Fee was a competitive bid. 
honorary
trustee for a 7. According   to   the   ethical
not­for­ standards   of   the   profession,
profit which  of  the  following  acts  is
organization generally prohibited? 
client.  a. Issuing   a   modified
report   explaining   a
6. The profession’s ethical 
failure   to   follow   a
standards most likely would be 
governmental
considered to have been violated 
regulatory   agency’s
when a CPA represents that 
stan­dards   when
specific consulting services will 
conducting   an   attest
be performed for a stated fee and 
service for a cli­ent. 
it is apparent at the time of the 
representation that the  b. Revealing   confidential
client   information
a. Actual fee would be substantially
during a quality review
higher. 
of   a   professional
practice by a team from
the state CPA society. 
c. Accepting a contingent 
fee for representing a 
client in an examination 
of the client’s federal 
tax return by an IRS 
agent. 
d. Retaining client records 
after an engagement is 
ter­minated prior to 
completion and the 
client has de­manded 
their return. 
8. According to the profession’s 
ethical standards, which of the 
following events may justify a 
departure from a Statement of 
the Governmental Accounting 
Standards Board? 
New Evolution of a new form
legislation of business transaction
a. No Yes
b. Yes No
c. Yes Yes
d. No No
9. May a CPA hire for the CPA’s 
public accounting firm a non­
CPA systems analyst who 
specializes in developing 
computer systems? 
a. Yes,   provided   the   CPA   is
qualified  to   perform   each   of  the
specialist’s tasks. 
b. Yes,   provided   the   CPA   is   able   to
supervise   the   spe­cialist   and
evaluate   the   specialist’s   end
product. 
c. No, because non­CPA
professionals   are  not
per­mitted   to   be
associated   with   CPA
firms   in   public
practice. 
d. No, because 
developing computer 
systems is not 
recognized as a 
service performed by 
public ac­countants. 
10. Stephanie Seals is a CPA who is working 
as a controller for Brentwood Corporation. 
She is not in public practice. Which 
statement is true? 
a. She may use the CPA 
designation on her 
business cards if she 
also puts her 
employment title on 
them. 
b. She may use the CPA
designation   on   her
business cards as long
as   she   does   not
mention   Brentwood
Corporation or her title
as controller. 
c. She   may   use   the
CPA   designation   on
company transmittals
but   not   on   her
business cards. 
d. She   may   not   use   the
CPA   designation
because   she   is   not   in
public practice. 
11. According to the standards of 
the profession, which of the 
following activities would most 
likely not impair a CPA’s 
independence? 
a. Providing advisory services for a
client. 
MODULE 1   PROFESSIONAL RESPONSIBILITIES 9
a publishing
company. 
b. Contracting   with   a c. Accepting
client to supervise the a
client’s   office commissi
personnel.  on   for
c. Signing   a   client’s   checks   in recommen
emergency situations.  ding   a
d. Accepting a luxurious gift from a product to
client.  an   audit
client. 
12. Which of the following reports d. Accepting
may   be   issued   only   by   an engagements
accountant who is independent obtained
of a client?  through   the
a. Standard report on an ef­forts   of
examination   of   a third parties. 
financial forecast. 
b. Report on consulting services.  16. To   exercise   due   professional  care   an
c. Compilation   report auditor should 
on   historical a. Critically
financial   state­ review   the
ments.  judgment
d. Compilation report on a financial exercised   by
projection.  those assisting
in the audit. 
13. According to the standards of  b. Examine   all
the profession, which of the  available
following activities may be  corroborating
required in exercising due care?  evidence   sup­
Consulting Obtaining porting
with experts specialty accreditation managements
a. Yes Yes assertions. 
b. Yes No c. Design   the
c. No Yes audit   to
d. No No detect   all
instances   of
14. Larry Sampson is a CPA and is  illegal acts. 
serving as an expertwitness in a trial  d. Attain   the
concerning a corporation’s financial state­ proper
ments. Which of the following is(are)  balance   of
true? professional
I. Sampson’s status as an expert witness experi­ence
is based upon his specialized  and   formal
knowledge, experience, and training. education. 
AI. Sampson   is   required   by
AICPA ruling to present his 17. Kar,   CPA,   is   a   staff   auditor
position objectively.  participating   in   the   audit
BI. Sampson may regard himself as acting engagement of Fort, Inc. Which
as an advocate.  of the following circum­stances
impairs Kar’s independence? 
a. I only. 
a. During the 
b. I and II only. 
period of the 
c. I and III only. 
professional 
d. III only.  engagement, 
15. According   to   the   ethical Fort gives Kar 
standards   of   the   profession, tickets to a 
which  of  the  following  acts  is football game 
worth $75. 
generally prohibited? 
b. Kar   owns
a. Purchasing   a   product   from   a
stock   in   a
third party and resell­ing it to a
corporation
client.  that   Fort’s
b. Writing   a 401(k)   plan
financial also invests in.
managemen
t  newsletter
pro­moted
and sold by
19. Which of the following 
c. Kar’s statements is(are) correct re­
friend, garding a CPA employee of a 
an CPA firm taking copies of 
employe information contained in client
e   of files when the CPA leaves the 
another firm? 
local   ac­ I. A CPA leaving a firm 
counting may take copies of 
firm, information contained 
prepares in client files to assist 
Fort’s another firm in serving 
tax that client.
returns.  AI. A CPA leaving a firm 
d. Kar’s   sibling   is   director   of may take copies of 
internal audit at Fort.  information contained 
in client files as a 
18. On June 1, 2008, a CPA 
method of gaining 
obtained a $100,000 personal  technical expertise. 
loan from a financial institution 
client for whom the CPA  a. I only. 
provided compilation services.  b. II only. 
The loan was fully secured and  c. Both I and II. 
considered material to the  d. Neither I nor II. 
CPA’s net worth. The CPA paid
the loan in full on December 31, 20. Which   of   the   following
2009. On April 3, 2009, the  statements   is   correct   regarding
client asked the CPA to audit  an   accountant’s   working
the client’s financial statements  papers? 
for the year ended December 31, a. The accountant
2009. Is the CPA considered  owns   the
independent with respect to the  working papers
audit of the client’s December  and   gen­erally
31, 2009 financial statements?  may   disclose
a. Yes, because the loan was fully them   as   the
secured.  accountant sees
b. Yes,   because fit. 
the   CPA   was b. The client owns the working 
not  required papers but the ac­countant has 
to   be   inde­ custody of them until the 
pendent at the accountant’s bill is paid in full. 
time   the   loan c. The
was granted.  accountant
c. No, owns   the
because the working
CPA had a papers   but
loan   with gen­erally
the   client may  not
during   the disclose them
period of a without   the
professiona client’s
l consent   or   a
engagemen
court order. 
t.  d. The client 
owns the 
d. No,
working 
because
the   CPA papers but, in
had   a  loan the ab­sence 
with   the of the 
client accountant’s 
during   the consent, may 
period not dis­close 
covered by them without 
the a court order. 
financial
21. Which of the following is
state­
an   authoritative   body
ments. 
designated   to   promulgate
attestation standards? 
a. Auditing Standards Board. 
b. Governmental   Accounting
Standards Board. 
c. Financial   Accounting   Standards
Board. 
d. General Accounting Office. 
22. According to the profession’s
standards,   which   of   the
following would be considered
consulting services? 
Advisory Implementation Product
services services services
a. Yes Yes Yes
b. Yes Yes No
c. Yes No Yes
d. No Yes Yes
10 MODULE 1   PROFESSIONAL RESPONSIBILITIES
e
cli
23. According to the standards of the  en
profession, which ofthe following events  t. 
would require a CPA performing a  b. Not perform any attest services for
consulting services engagement for a  the client. 
nonaudit client to withdraw from the  c. I
engagement? n
f
I. The CPA has a conflict of 
o
interest that is disclosed to the 
r
client and the client consents to 
the CPA continuing the  m
engagement.   t
AI. The CPA fails to obtain h
a written understanding e
from   the   client cl
concerning the scope of ie
the engagement.  n
t
a. I only. 
o
b. II only.  f
c. Both I and II.  si
d. Neither I nor II.  g
24. Which of the following services n
may a CPA perform in carrying if
out   a   consulting   service   for   a ic
client?  a
n
I. Analysis   of   the   client’s   accounting t
system.  r
AI. Review of the client’s prepared business e
plan.  s
III.  Preparation of information for obtaining e
financing.  r
a. I and II only.  v
b. I and III only.  at
c. II and III only.  i
d. I, II, and III.  o
n
25. Under the Statements on Standards for  s
Consulting Ser­vices, which of the  c
following statements best reflects a CPA’s  o
responsibility when undertaking a  n
consulting services engagement? The CPA  ­
must  c
a. N e
ot r
se n
ek i
to n
m g
od t
if h
y e
an b
y e
ag n
re e
e fi
m ts
en o
t f
m t
ad h
e e
wi e
th n
th g
a plann
g ing
e deci­
m sions.
e c. Monitoring progress in achieving
nt goals. 
.  d. Upd
d. Obta atin
in   a g
writt reco
en mm
unde end
rstan atio
ding ns
with and
the revi
sing
clien
plan
t
ning
con­ deci
cerni sion
ng s. 
the
time 27. In relation to the AICPA Code of
for Professional   Conduct,  the   IFAC
com Code   of   Ethics   for   Professional
pleti Accountants
on a. Has more outright prohibitions. 
of b. Has fewer outright prohibitions. 
the c. Has no outright prohibitions. 
enga d.
gem Applies
ent.  onl
y
26. Which of the following  to
services is a CPA generally  pro
required to perform when  fes
conducting a personal  sio
financial planning  nal
engagement?  acc
a. Assi ou
sting nta
the nts
clien in
t   to bus
ident ine
ify ss. 
tasks 28. Based on the IFAC Code of 
that
Ethics for 
are
esse ProfessionalAccountants, 
n­tial threats to independence arise 
in from all of thefollowing 
order except: 
to a. Self interest. 
act b. Advocacy. 
on c. The audit relationship. 
plan d. Intimidation. 
ning
decis 29. If an audit firm discovers threats 
ions. to independence with respect to 
b. Assis an audit engagement, the IFAC 
ting Code of Ethics forProfessional 
the Accountants indicates that the 
client firm should
to
take
actio
n   on
a. International   auditing
a. Immediately   resign   from   the standards   do   not
engagement.  require   an   audit   of
b. Notify the appropriate regulatory internal control. 
body.  b. International auditing 
c. Document the issue.  standards do not allow 
d. Evaluate the  refer­ence to division of
significance of the  responsibilities in the 
threats and apply  audit re­port. 
appropriate safeguards c. International   auditing
to reduce them to an  standards   require
acceptable level.  obtaining   an
attorney’s letter. 
30. With respect to the acceptance of  d. International   auditing
contingent fees for professional  standards are based on a
services, the IFAC Code of Ethics  risk   assessment
forProfessional Accountants  approach. 
indicates that the accounting firm
a. Should not accept contingent fees. 34. Which of the following is not
b. Should   establish true   about   international
appropriate   safeguards auditing standards? 
around ac­ceptance of a a. Audit   report   modification   for
contingent fee.  consistency in the ap­plication of
c. Should   accept accounting principles is required. 
contingent fees only for b. Confirmation   of
assurance   services accounts
receivable   is   not
other   than   audits   of
re­quired. 
financial statements. 
c. The   location   in   which
d. Should   accept the   auditor   practices
contingent fees if it is must be disclosed in the
customary   in   the audit report. 
country.  d. International   auditing
31. With regard to marketing  standards   do   not
professional services, the IFAC  require   an   audit   of
Code of Ethics for Professional  internal control. 
Accountants indicates that 
35. Independence standards of the 
a. Direct marketing is prohibited. 
GAO for audits in accordance 
b. Marketing is allowed if lawful.  with generally accepted 
c. Marketing   should   be   honest   and government auditing standards 
truthful.  describe three types of 
d. Marketing   of   audit   services   is impairments of indepen­dence. 
prohibited.  Which of the following is not 
32. What   body   establishes   international one of these types of 
impairments? 
auditing standards? 
a. Personal. 
a. The   Public   Company   Accounting
b. Organizational. 
Oversight Board. 
c. External. 
b. The   International   Federation   of
d. Unusual. 
Accountants. 
c. The World Bank.  36. In accordance with the 
d. The   International   Assurance independence standards of the 
Body.  GAO for performing audits in 
accordance with generally 
33. Which of the following is not
accepted government auditing 
true   about   international
standards, which of the fol­ 
auditing standards? 
MODULE 1   PROFESSIONAL RESPONSIBILITIES 11

lowing is not an 
example of an 
external impairment
of independence?
a. Redu
cing
the
exten
t   of
audit
work
due
to
press
ure
from
mana
geme
nt   to
reduc
e
audit
fees. 
b. Sel
ecti
ng
aud
it
ite
ms
bas
ed
on
the
wis
hes
of
an
em
plo
yee
of
the
org
ani
zati
on
bei
ng
aud
ite
d. 
c. Bias
in
the
item
s the
audit
ors
deci
de to
selec
t   for
testi
ng. 
d.
Influenc
e
by
ma
nag
eme
nt
on
the
pers
onn
el
assi
gne
d to
the
aud
it. 
37. Under the 
independence 
standards of the
GAO for per­
forming audits 
in accordance 
with generally 
accepted gov­
ernment 
auditing 
standards, 
which of the 
following are 
overreaching 
principles for 
determining 
whether a 
nonaudit 
service impairs 
independence? 
I.
Audi
t
o
r
s
 
m
u
s
t
 
n
o
t
 
p
e
r
f
o
r
m
 
n
o
n
a
u
d
i
t
 
s
e
r
v
i
c
e
s
 
t
h
a
t
 
i
n
­
v
o
l
v
e
 
p
e
r
f
o
r
m
i
n
g
 
m
a
n
a
g
e
m
e
n
t
 
f
u
n
c
t
i
o
n
s
 
o
r
 
m
a
k
i
n
g
 
m
a
n
a
g
e
m
e
n
t
 
d
e
c
i
s
i
o
n
s
.
 
AI.
Audit
o
r
s
m
u
s

n
o

a
u
d
it
t
h
e
i

o
w
n
w
o
r
k
o

p
r
o
v
i
d
e
n
o
n
a
u
d
it
s
e
r
v
i
c
e
s
i
n
s
it
u
a
ti
o
n
s
i
n
w
h
i
c
h
t
h
e
n
o
n
a
u
d
it
s
e
r
v
i
c
e
s
a
r
e
s
i
g
n
i
f
i
c
a
n

o

m
a
t
e
r
i
a

t
o
t
h
e
s
u
b
j
e
c

matter of the audit.
BI.
Audi
t
o
r
s
 
m
u
s
t
 
n
o
t
 
p
e
r
f
o
r
m
 
n
o
n
a
u
d
i
t
 
s
e
r
v
i
c
e
s
 
w
h
i
c
h
 
r
e
q
u
i
r
e
 
i
n
d
e
p
e
n
d
e
n
c
e
.
 
a. I only. 
b. I and II only. 
c. I, II and III. 
d. II and III only. 
38. Which of the
following
bodies
enforce   the
audit   re­
quirements
of   the
Employee
Retirement
Security  Act
of   1974
(ERISA)
with   respect
to   employee
benefit
plans? 
a. The Department of Labor. 
b. The Department of Pension Management. 
c. The Securities and Exchange Commission. 
d. The Public Company Accounting Oversight Board. 
39. The requirement
for 
independence by
the auditor re­
garding audits of
employee 
benefit plans 
apply to the plan
as well as 
a. In
ve
st
m
en
t
co
m
pa
ni
es
do
in
g
bu
si
ne
ss
wi
th
th
e
pl
an

b. Members of the plan. 
c. The plan sponsor. 
d. The actuary firm doing services for the plan. 
12 MODULE 1   PROFESSIONAL RESPONSIBILITIES

SIMULATIONS

Task­Based Simulation 1
Independence
Issues
Authoritative
Literature Help

Assume that you are analyzing relationships for your firm to identify situations in which an auditor’s independence may 
be impaired. For each of the following numbered situations, determine whether the auditor (a covered member in the situation) 
is considered to be independent. If the auditor’s independence would not be impaired select No. If the auditor’s independence 
would be impaired select Yes.

1. T

2. T

3. T

4. T
h
5. T

6. T

7. T

8. T

I
n
V
a
L
i
The director of the audit committee of Hanmei Corp., a nonissuer (nonpublic) company, has indicated that the company
may be interested in engaging your firm to perform various professional services. Consider each of the following potential
services by itself, and determine whether a CPA firm may provide such a service. If a CPA firm may provide the service, fill in
the circle under the first or second column of replies based upon whether independence is required. If the service may not be
provided, fill in the circle under “May Not Provide.” For each service you should have only one reply.

Service
1. Provide an opinion on whether financial statements are
pre­pared following the cash basis of accounting. 
2. Compile a forecast for the coming year. 
3. Compile   the   financial   statements   for   the   past   year   and
issue a publicly available report. 
4. Apply certain agreed­upon procedures to accounts receivable 
for purposes of obtaining a loan, and express a summary of 
findings relating to those procedures. 
5. Review quarterly information and issue a report that includes
limited assurance. 
6. Perform an audit of the financial statements on whether
they   are   prepared   following   generally   accepted
accounting princi­ples. 
7. Perform a review of a forecast the company has prepared
for the coming year. 
May provide, May provide,
independence independence is May
is required  not  required not provide
Service
8. Compile the financial statements for the past year, but not is­sue
a report since the financial statements are only for the 
company’s use. 

MODULE 1   PROFESSIONAL RESPONSIBILITIES 13
May provide, May provide,
independence independence is May
is required  not  required not provide

9.  Calculate the client’s taxes and fill out the appropriate tax
forms.
10.  Design a new payroll system for Hanmei and base billings on
Hanmei’s actual savings for the next three years.

Task­Based Simulation 3
Research
Authoritative
Literature Help
3. Regardless of what you find, he would like you to determine whether a covered member 
may have such an immaterial financial investment in an audit client. What title, 
section, and paragraph addresses this issue?

Task­Based Simulation 4
Providing Various
Services
Authoritative
Literature Help

The firm of Willingham and Whiting, CPAs, has had requests from a number of clients and prospective clients as to
providing various types of services. Please reply as to whether the appropriate independence rules (AICPA and/or PCAOB)
allow the following engagements with
A—Allowable, given these facts. N
—Not allowable, given these facts.
(If both AICPA and PCAOB rules apply and one of them does not allow the services answer N.)
14 MODULE 1   PROFESSIONAL RESPONSIBILITIES

Allowable (A)
Public or or not
nonpublic allowable
Case Request client (NA)?
1. Provide internal audit outsourcing as well as perform the audit. Public
2. Prepare the corporate tax return as well as perform the audit. Public
3. Prepare the corporate tax return as well as perform the audit. Nonpublic
4. Provide bookkeeping services as well as perform the audit; WW will not Nonpublic
determine journal entries, authorize transactions, prepare or modify source
documents.
5. Provide financial information systems design and implementation assistance; Public
WW provides no attest services for that company.
6. Serve on the board of directors of the company; WW provides no attest Public
services for that company.
7. Implement an off­the­shelf accounting package as well as perform the audit. Nonpublic
8. Provide actuarial services related to certain liabilities as well as perform the Nonpublic
audit; the subjectively determined liabilities relate to a material portion of the
financial statements.
9. Provide actuarial services related to certain liabilities as well as perform the Public
audit; the subjectively determined liabilities relate to material portion of the
financial statements.
10. Corporate executives of an audit client want to have WW provide tax planning Public
for themselves (not the company).

Task­Based Simulation 5
Research
Authoritative
Literature Help
Payroll System Engagement
Michael Edlinger is president of Edlinger Corporation, a nonpublic manufacturer of kitchen cabinets. He has been 
approached by Marla Wong, a partner with Wong and Co., CPAs, who suggests that her firm can design a payroll system for 
Edlinger that will either save his corporation money or be free. More specifically, Ms. Wong proposes to design a payroll 
system for Edlinger on a contingent fee basis. She suggests that her firm’s fee will be 25 % of the savings in payroll for each 
of the next four years. After four years Edlinger will be able to keep all future savings. Edlinger Corporation’s payroll system 
costs currently are approximately $200,000 annually, and the corporation has not previously been a client of Wong. Edlinger 
Corporation is audited by another CPA firm and Wong & Co. provides no other services to Edlinger Corporation. Select one 
of the following topics to answer question 1.
D.
Selections
A. AU
B. PCAOB AR
C. AT E.
ET
F. BL
G. CS
H. QC

1. Which topic of the Professional Standards addresses this issue and will be helpful
in determining whether Wong & Co. may perform this engagement under these 
terms without violating professional requirements? 
(A) (B) (C) (D) (E)  (F) (G) (H)

2. Provide the appropriate paragraph citation that addresses this issue. 
MODULE 1   PROFESSIONAL RESPONSIBILITIES 15

3. Interpret your findings in parts 1. and 2. and conclude on whether Wong & Co. may perform this service without
violating professional standards. 
___Yes, this service may be performed without violating professional standards.
___No, this service may not be performed without violating professional standards.

Task­Based Simulation 6
Research
Authoritative
Literature Help

Professional Standards
You have worked with James & Co. CPAs for approximately 4 months. Jen Jefferson, who has just started with James
& Co., has asked you to explain the nature of various professional standards to her. More specifically, she would like to
have a better understanding of which standards to address, in which circumstances.
Select the appropriate title of standards for 1. through 10. below.  Standards may be selected once, more than once, or not 
at
all.
Title of Standards
A. AICPA Bylaws (BL) 
B. Code of Professional Conduct (ET) 
C. PCAOB Auditing Standards 
D. Standards for Performing and Reporting on Peer Reviews (PR) 
E. Statements on Auditing Standards (AU) 
F. Statements on Quality Control Standards (SQCS) 
G. Statements on Standards for Accounting and Review Services (SSARS) 
H. Statements on Standards for Attestation Engagements (SSAE) 
I. Statements on Standards for Consulting Services (CS) 
J. Statements on Standards for Tax Services (TS) 
8. For   reporting   on   client   pro   forma
Standards that provide guidance financial information. 
1. For   performance   of   a   review   of   a 9. On   whether   an   investment   of   a   CPA
nonpublic   company’s   annual   financial impairs her independence with respect
statements.  to a client. 
2. On whether a contingent fee may be billed to a client.  10. On performing a compilation of a
3. Related to firm requirements of CPA firms nonpublic   company’s   quarterly
that   are   enrolled   in   an   AICPA­approved statements. 
practice­monitoring system. 
4. For an examination of a client’s financial forecast. 
5. Relating   to   overall   requirements   when
providing   services   for   an   advisory
services engagement. 
6. For the audit of a public company. 
7. For the performance of an interim review of
the   quarterly   financial   statements   of   a
nonpublic audit client. 
(A) (B) (C) (D) (E)  (F) (G) (H)  (I) (J)
16 MODULE 1   PROFESSIONAL RESPONSIBILITIES

Task­Based Simulation 7
Research
Authoritative
Literature Help

Code of Professional Conduct
Assume that you are employed by DFW, CPAs. One of the partners has asked you to research the professional standards
for the section that identifies the requirements regarding the acceptance of contingent fees for engagements.
Title choices
A. AU
B. PCAOB
C. AT
D. AR
E. ET
F. BL
G. CS
H. QC
(A) (B) (C) (D) (E)  (F) (G) (H)

1. Which title of the Professional Standards addresses this issue and will be
helpful in responding to the partner? 

2. Enter the exact section and paragraphs with helpful information. 
MODULE 1 PROFESSIONAL RESPONSIBILITIES 17

*
MULTIPLE­CHOICE ANSWERS
1. C __ __ 10. a __ __ 19. d __ __ 28. c __ __ 37. b __ __
2. D __ __ 11. a __ __ 20. c __ __ 29. d __ __ 38. a __ __
3. B __ __ 12. a __ __ 21. a __ __ 30. b __ __ 39. c __ __
4. C __ __ 13. b __ __ 22. a __ __ 31. c __ __
5. C __ __ 14. b __ __ 23. d __ __ 32. b __ __
6. A __ __ 15. c __ __ 24. d __ __ 33. c __ __
7. D __ __ 16. a __ __ 25. c __ __ 34. a __ __
8. C __ __ 17. d __ __ 26. a __ __ 35. d __ __ 1st: __/39 = __%
9. B __ __ 18. b __ __ 27. b __ __ 36. c __ __ 2nd: __/39 = __%

MULTIPLE­CHOICE ANSWER EXPLANATIONS
because although the conviction of a felony can result in 
A.1.–3.  Code of Professional Conduct automatic expulsion, likewise can the other two. An­ 
1. (c) The requirement is to identify the statement thatbest  swers (b) and (c) are incorrect because all three can result
describes the meaning of generally accepted auditing  in automatic expulsion from the AICPA. 
standards. Answer (c) is correct because generally  3. (b) Answer (b) is correct because an effective 
accepted auditing standards deal with measures of the  corporate governance structure is a control that 
quality of the performance of audit procedures (AU 150).  can be implemented by a client that increases 
Answer (d) is incorrect because procedures relate to acts to independence of the attest team. Answer (a) is 
be performed, not directly to the standards. Answer (b) is  incorrect because it is a safeguard that is 
incorrect be­cause generally accepted auditing standards  implemented by regulation or the CPA firm. 
have been is­sued by predecessor groups, as well as by the Answer 
Auditing Standards Board. Answer (a) is incorrect because (c) is incorrect because it is a safeguard that is required by 
there may or may not be universal compliance with the  regulation or the CPA firm. Answer (d) is incorrect 
standards. because it represents a threat rather than a safeguard. 
2. (d) All of these can result in the automatic expul­sion of  4. (c) Answer (c) is correct because the team 
the member from the AICPA. Answer (a) is incor­rect  member would be reviewing his or her own 
work. Answer (a) is incorrect because this is an  are not intentional misstatements. Answer (c) is incorrect 
example of a familiarity threat. Answer (b) is  because while one must remain objective while 
incorrect because this is an example of a  performing consulting services, independence is not 
safeguard to threats to independence. Answer  required unless the CPA also performs attest services for 
(d) is incorrect because this represents a  that client.
financial self­interest threat to independence. 
7. (d) The requirement is to determine which act is generally 
5. (c) According to the Code of Professional  prohibited. Answer (d) is correct because “If an 
Conduct, Rule 101 regarding independence, a  engagement is terminated prior to completion, the member
spouse may be employed by a client if s/he does  is required to return only client records” (ET 501). Answer
not exert significant influence over the contents  (a) is incorrect because issuing a modified report 
of the client’s financial statements. This is a key  explaining a failure to follow a governmental regulatory 
position as defined by the Interpretation of Rule  agency’s stan­dards when conducting an attest service is 
101.  not prohibited. Answer (c) is incorrect because accepting a
contingent fee is allowable when representing a client in 
6. (a) According to Rule 102 of the Code of Profes­ an examination by a revenue agent of the client’s federal 
sional Conduct, in performing any professional  or state income tax re­turn (ET 302). Answer (b) is 
service, a member shall maintain objectivity and  incorrect because revealing confidential client information 
integrity, avoid con­flicts of interest, and not  during a quality review of a professional practice by a 
knowingly misrepresent facts. An­swer (a) is  team from the state CPA society is not prohibited (ET 
correct as this would be knowingly misrepre­ 301). 
senting the facts. Answers (b) and (d) are 
incorrect as these  8. (c) According to Rule 203 of the Code of 
Profes­sional Conduct, CPAs are allowed to 
depart from a GASB Statement only when 
results of the Standard would be misleading. 
Examples of possible circumstances justifying 
departure are new legislation and a new form of
business transaction. 
9. (b) The requirement is to determine whether a 
CPA may hire a non­CPA systems analyst and, 
if so, under what conditions. Answer (b) is 
correct because ET 291 allows such a situation 
when the CPA is qualified to supervise and 
evaluate the work of the specialist. Answer (a) is
incorrect because the CPA need not be qualified 
to perform the spe­cialist’s tasks. Answer (c) is 
incorrect because non­CPA professionals are 
permitted to be associated with CPA firms in 
public practice. Answer (d) is incorrect because 
nonpro­fessionals may be hired, and because 
developing computer systems is recognized as a 
service performed by public ac­countants. 
10. (a) She may use the CPA designation on her 
busi­ness cards when she does not imply 
independence but shows her title and her 
employer. Therefore, answer (b) is incor­rect. 
Answer (c) is incorrect because she may use the 
CPA designation on her business cards or 
company transmittals if she does not imply 
independence. Answer (d) is incorrect because 
under the above situations, she can use the CPA 
designation. 
11. (a) The requirement is to determine the activity
that   would   most   likely  not  impair   a   CPA’s
independence. Ac­ 

*
  Explanation of how to use this performance record appears on page 2.
18 MODULE 1   PROFESSIONAL RESPONSIBILITIES
special­ized knowledge, 
training, and experience. 
counting and consulting services do not 
normally impair independence because  15. (c) The requirement is to determine which 
the member’s role is advisory in na­ture  act is generally prohibited. Answer (c) is 
(ET 191). Answers (b) and (c) are  correct because “a member in public 
incorrect because management functions  practice shall not for a commission rec­
are being performed (ET 191). An­swer  ommend or refer to a client any product or 
(d) is incorrect because accepting a  service, or for a commission recommend or
luxurious gift im­pairs a CPA’s  refer any product or service to be supplied 
independence (ET 191). by a client, or receive a commission when 
the member or the member’s firm perform 
12. (a) The requirement is to identify the type  for that client: (1) an audit of a financial 
of report that may be issued only by an  statement; or (2) a compilation of a fi­
independent accountant. An­swer (a) is  nancial statement when the member 
correct because AT 101 requires an  expects that a third party will use the 
accountant be independent for all  financial statement and the member’s 
attestation engagements. An attestation  compilation report does not disclose a lack 
engagement is one in which the accountant  of independence; or (3) an examination of 
expresses a conclusion about the reliability  prospective financial information.” Answer
of assertions which are the responsibility of (a) is incorrect because a member may 
another party. A standard report on an  purchase a product and resell it to a client. 
examination of a financial forecast requires  Any profit on sale would not constitute a 
the auditor to express an opinion, which  commission (ET 591). 
requires an accountant to be inde­pendent. 
Answer (b) is incorrect because CS 100  16. (a) The principle of due care requires the 
indicates that consulting services are  member to observe the profession’s 
fundamentally different from the attestation technical and ethical standards, strive 
function, and therefore do not require  continually to improve competence and the 
indepen­dence of the accountant. Answers  quality of services, and discharge 
(c) and (d) are incorrect because AR 100  responsibility to the best of the member’s 
indicates that an accountant who is not  ability. Answer (b) is incorrect as the 
independent is not precluded from issuing a auditor is not required to examine all 
report on a compilation of financial  corroborating evidence support­ing 
statements.  management’s assertions, but rather to 
examine evidence 
13. (b) Per ET 56, due care requires 
a member to dis­charge 
professional responsibilities with
competence and diligence. 
Competence represents the 
attainment and main­tenance of a
level of understanding and 
knowledge that en­ables a 
member to render services with 
facility and acumen. It also 
establishes the limitations of a 
member’s capabilities by 
dictating that consultation or 
referral may be required when a 
professional engagement exceeds
the personal com­petence of a 
member or a member’s firm. 
Accordingly, answer (b) is 
correct as it may be required to 
consult with experts in 
exercising due care. Due care 
does not require obtaining 
specialty accreditation. 
14. (b) Under ruling 101 under Rule 
of Conduct 102, when a CPA is 
acting as an expert witness, s/he 
should not act as an advocate 
but should give his/her position 
based on objectivity. The expert 
witness does this based on 
incorrect because statement II 
on a scope basis based on his/her  also is incor­rect. Answer (c) is 
consideration of material­ity and level of  incorrect because statements I 
risk assessed. Answer (c) is incorrect as the and II are both incorrect. 
auditor should be aware of the possibility 
20. (c) Information in the CPA’s 
of illegal acts, but an audit provides no 
assurance that all or any illegal acts will be  working papers is confidential 
detected. Answer (d) is not the best answer  and may not be disclosed except
because competence is derived from both  with the cli­ent’s consent or by 
education and experience. The principle of  court order. Answer (a) is 
due care requires the member to strive to  incorrect be­cause disclosure of 
improve competence, however, attaining  the information would generally
the proper balance of professional  violate Rule 301 on confidential 
experience and formal education is not a  client information. Answers (b) 
criterion for exercising due care. and (d) are incorrect because the
CPA owns the working papers. 
17. (d) The fact that a close relative 
of Kar works for Fort impairs  21. (a) The requirement is to identify 
Kar’s independence. Answer (a)  the listed authoritative body 
is incorrect because the gift is of a designated to promulgate 
token amount which does not  attestation stan­dards. Answer (a)
impair Kar’s independence.  is correct because only the 
Answer (b) is incorrect because a  Auditing standards Board, the 
joint financial investment must be Accounting and Review Services 
material to impair indepen­dence, Committee, and the Management 
and this would generally not  Advisory Services Execu­tive 
occur with respect to a retirement  Committee have been authorized 
plan. Answer (c) is incorrect  to promulgate attesta­tion 
because preparation of the client’s standards. 
tax return is not a service that  A.4.  Responsibilities in Consulting 
impairs inde­pendence. 
Services
18. (b) Independence was not required at the 
time the loan was obtained, and because it  22. (a)   Types   of   consulting
is fully secured it is grand­fathered by 101­ services include consulta­tions,
5. Answer (a) is incorrect because if the  advisory   services,
CPA is required to be independent, a  implementation   services,
mortgage loan would not be permitted even transac­tion services, staff and
if it was fully secured. Answer (c) is  other   support   services,   and
incorrect because the CPA was not required product services. 
to be indepen­dent of the client. Answer (d) 23. (d) According to the Statements on Standards
is incorrect because the CPA was not  for Consulting Services, independence is not 
required to be independent of the client.  required for per­formance of consulting 
19. (d) Both of the statements are  services unless the CPA also per­forms attest 
incorrect; either would violate  services for that client. However, the CPA 
Rule 301 on confidential client  must remain objective in performing the 
information. Answer (a) is  consulting services. Furthermore, the 
incorrect because statement I also understanding with the client for perform­ing 
is incorrect. Answer (b) is  the services can be established either in 
writing or orally. 
MODULE 1   PROFESSIONAL RESPONSIBILITIES 19
identify the characteristicthat 
differs between the two sets of 
24. (d) CS 100 indicates that the  ethical standards. Answer (b) 
nature and scope of consulting  is correct because the IFAC 
services is determined solely by Code has fewer outright 
the practitioner and the client,  prohibitions than the AICPA 
typically in which the  Code. Answers (a) and (c) are 
practitioner develops findings,  incorrect because the IFAC 
conclusions, and  Code has fewer outright 
recommendations for the client.  prohibitions. Answer (d) is 
All three services listed would  incorrect because the IFAC 
fall under the definition of  Code applies to all professional
consulting services.  accountants. 
25. (c) The AICPA Statement on  28. (c)  The   requirement   is   to
Standards for Consult­ing  identify   the   item   that   is   nota
Services, Section 100, describes  threat to  independence.  Answer
general standards for all  (c)   is   correct  because   the   audit
consulting services, in addition to  relationship,   in   itself,   is   not   a
those established under the  threat to independence. 
AICPA Code of Professional  Answers   (a),   (b),   and   (d)   are
Conduct. Section 100 ad­dresses  incorrect   because   they  all  represent
the areas of client interest,  types of threats to independence. 
understanding with the client, and
communication with the client.  29. (d) The requirement is to identify the 
Specifically, this section states  appropriatecourse of action when threats to 
that the accountant should inform  independence are discov­ered. Answer (d) 
the client of significant  is correct because the firm should evaluate 
reservations concerning the scope the significance of the threats and apply 
or benefits of the engagement.  safeguards, if neces­sary, to reduce them to 
an acceptable level. Answer (a) is incorrect 
A.5.  Responsibilities in Personal  because the firm would only resign if 
Financial Planning appropriate safeguards could not reduce the
threats to an acceptable level, or it is 
26. (a) Personal financial planning  required based on a prohibition. Answer (b)
engagements are onlythose that involve  is incorrect because the firm would not 
developing strategies and making  notify a regulatory body at this point. 
recommendations to assist a client in  Answer (c) is incorrect because the firm 
defining and achieving personal financial  would document the issue, but only after it 
goals. Personal financial engagements  is resolved. 
involve all of the following:
20. Defining engagement objectives 
21. Planning   specific   procedures
appropriate to en­gagement 
22. Developing   basis   for
recommendations 
23. Communicating recommendations
to client 
24. Identifying   tasks   for   taking
action on planning de­cisions. 
Other engagements may also include, but
generally are not required to perform, 
the following:
1. Assisting   client   to
take   action   on
planning deci­sions 
2. Monitoring   client’s   progress   in
achieving goals 
3. Updating
recommendations   and
helping   client   re­vise
planning decisions. 
D. International Standards—Ethical 
27. (b) The requirement is to 
risk of material misstatement. 
54. (b)    The   requirement   is   to   identify Answers (a), (b) and (d) are 
what the IFAC incorrect because they are all 
Code of Ethics for Professional  true about international auditing 
Accountants provides withrespect to  standards. 
contingent fees. Answer (b) is correct 
34. (a) The requirement is to identify
because the IFAC Code indicates that if the
contingent fee presents a threat to apply  the item that is nottrue about 
fundamental principles, the firm should  international auditing standards. 
establish appropriate safeguards. Answer  Answer (a) is correct because 
(a) is incorrect because a contingent fee  international auditing standards 
may be accepted if threats can be reduced  do not require a modification of 
to an acceptable level. Answers (c) and (d)  the audit report for consistency 
are incorrect because the IFAC Code does  in the application of accounting 
not contain these provisions.  principles. Answers (b), (c), and 
(d) are incorrect because they are
55. (c) The requirement is to  true about international auditing 
identify the IFAC  standards. 
Codeprovision regarding 
marketing. Answer (c) is  F.2.  Government Accountability Office 
correct because the IFAC Code (GAO)
indicates the marketing must 
35. (d) The requirement is to identify 
be honest and truthful. 
Answers (a) and (d) are  the impairment thatis not one of 
incorrect because no particular  the three types of impairments 
form of marketing is  described in the GAO standards. 
prohibited. Answer (b) is  Answer (d) is correct because an 
incorrect because marketing  unusual impairment is not one of 
must be honest and truthful as  the types of impairments 
well as legal.  described in the GAO standards. 
Answers (a), (b) and (c) are 
E. International   Standards­­ incorrect because they are the 
Auditing/Assurance  three types of impairments 
described in the GAO standards. 
32. (b) The requirement is to identify the body 
that estab­lishes international auditing  36. (c) The requirement is to identify
standards. Answer (b) is correct because  the example thatdoes not 
the International Auditing and Assurance  represent an external impairment 
Standards Board of the International  of independence. Answer (c) is 
Federation of Accountants establishes  correct because this item is an 
international auditing standards. Answer  example of a personal 
(a) is incorrect because the Public  impairment of independence. 
Company Accounting Oversight Board  Answers (a), (b) and 
establishes standards for the audit of public (d)   are   incorrect   because   they   are   all
companies in the US. Answers (c) and (d)  examples   of   external   impairments   of
are incorrect because these bodies do not  independence. 
establish auditing standards.  37. (b) The requirement is to identify 
33. (c) The requirement is to identify the overreachingprinciples for 
the item that is nottrue about  identifying whether nonaudit 
international auditing standards.  services impair independence. 
Answer (c) is correct because  Answer (b) is correct because I 
international auditing standards  and II are the two principles. 
require obtaining an attorney’s  Answer (a) is incorrect because II
letter only if the auditors assess a is also an 
20 MODULE 1   PROFESSIONAL RESPONSIBILITIES

overreaching principle. Answer (c) is incorrect because III is not an overreaching principle. 
Answer (d) is incorrect because I is an overreaching principle and III is not.
F.3.  Department of Labor (DOL)
38. (a) The requirement is to identify the body that en­forces the audit requirements of ERISA. 
Answer (a) is cor­rect because the Department of Labor is responsible for en­forcing the audit 
requirements. Answer (b) is incorrect because the Department of Pension Management does 
not exist. Answers (c) and (d) are incorrect because the SEC and the PCAOB deal with 
auditing requirements for entities with publicly traded securities (issuers). 
39. (c) The requirement is to identify the party that in­dependence standards also apply 
to when performing an audit of an employee benefit plan. Answer (c) is correct 
because the Department of Labor rules also apply to independence from the plan and
the plan sponsor. Answers (a), (b) and (d) are incorrect because the independence 
standards do not apply to these parties. 
MODULE 1   PROFESSIONAL RESPONSIBILITIES 21

SOLUTIONS TO SIMULATIONS

Task­Based Simulation 1
Independence
Issues Authoritative
Literature Help

Yes No
1. The auditor is a cosigner of a client’s checks.
2. The auditor is a member of a country club which is a client.
3. The auditor owns a large block of stock in a client but has placed it in a blind trust.
4. The auditor placed her checking account in a bank which is her client.  The account is fully
insured by a federal agency.
5. The client has not paid the auditor for services for the past two years.
6. The auditor is leasing part of his building to a client.
7. The auditor joins, as an ordinary member, a trade association which is also a client.
8. The auditor has an immaterial, indirect financial interest in the client.

Explanation of solutions
1. (Y) Since the auditor is a cosigner on a client’s check, the auditor could become
liable   if   the   client   defaults.   This   rela­tionship   impairs   the   auditor’s
independence. 
2. (N)  Independence is not impaired because membership in the country club is essentially
a social matter. 
3. (Y) An auditor may not hold a direct financial interest in a client. Putting it in a
blind trust does not solve the impairment of independence. 
4. (N) If the auditor places his/her account in a client bank, this does not impair
independence if the accounts are state or federally insured. If the accounts are
not insured, independence is not impaired if the amounts are immaterial. 
5. (Y) The auditor’s independence is impaired when prior years’ fees for professional
services remain unpaid for more than one year. 
6. (Y)   The auditor’s independence is impaired when s/he leases space out of a building
s/he owns to a client. 
7. (N)  When the auditor does not serve in management, s/he may join a trade association
who is a client. 
8. (N) Independence is impaired for direct financial interests and material, indirect
financial interests but not for immaterial, indirect financial interests. 

Task­Based Simulation 2
Independence and
Various Services Authoritative
Literature

Help

Service
1. Provide   an   opinion   on   whether
financial   statements   are   pre­pared
following the cash basis of accounting 
2. Compile a forecast for the coming year. 
3. Compile the financial statements for the
past year and issue a publicly available
report. 
4. Apply certain agreed­upon procedures to 
accounts receivable for purposes of 
obtaining a loan, and express a summary of 
findings relating to those procedures. 
5. Review   quarterly   information   and   issue   a
report that includes limited assurance. 
May provide, May provide,
independence independence is May
is required  not  required not provide
22 MODULE 1   PROFESSIONAL RESPONSIBILITIES

Service
6. Perform   an   audit   of   the   financial
statements   on   whether   they   are
prepared following generally accepted
accounting princi­ples. 
7. Perform   a   review   of   a   forecast   the
company has prepared for the coming
year. 
8. Compile the financial statements for the past 
year, but not is­sue a report since the 
financial statements are only for the 
company’s use. 
May provide, May provide,
independence independence is May
is required  not  required not provide

9.  Calculate the client’s taxes and fill out the appropriate tax
forms.
10. Design a new payroll system for Hanmei and base billings on
Hanmei’s actual savings for the next three years.

Task­Based Simulation 3
Research
Authoritative
Literature Help
(A) (B) (C) (D) (E) (F) (G) (H)
1. He thinks that he recalls the issue relates to whether you are or are not a
“covered member.”  He would like you to find the definition of a covered
member in the professional standards.  Which title of the Professional Stan­
dards addresses this issue and will be helpful in responding to the senior?

2. Enter the exact section and paragraphs with helpful information. 92 06

3. Regardless of what you find, he would like you to determine whether a covered member 
may have such an immaterial financial investment in an audit client.  What title, section, ET 101 02
and paragraph addresses this issue?

Task­Based Simulation 4
Providing Various
Services
Authoritative
Literature Help

1. Not allowable (PCAOB requirements prohibit) 
2. Allowable 
3. Allowable 
4. Allowable 
5. Allowable (Because no attest services are provided, the PCAOP allows this) 
6. Not allowable 
7. Allowable 
8. Not allowable (AICPA rules prohibit this when amounts are subjectively determined
and material) 
9. Not   allowable   (Both   AICPA   nor   PCAOB   rules   prohibit   this   when   amounts   are
subjectively determined and material) 
10. Not allowable 
MODULE 1   PROFESSIONAL RESPONSIBILITIES 23
Task­Based Simulation 5
Research
Authoritative
Literature Help

(A) (B) (C) (D) (E) (F) (G) (H)


1. Which topic of the Professional Standards addresses this issue and will be
helpful in determining whether Wong & Co. may perform this engagement
under these terms without violating professional requirements?

2. Provide the appropriate paragraph citation that addresses this issue. 302 01
3. Interpret your findings in parts 1 and 2 and conclude on whether Wong & Co.
may perform this service without violating professional standards. 
_x_   Yes, this service may be performed without violating professional standards.
___ No, this service may not be performed without violating professional 
standards.

Task­Based Simulation 6
Research
Authoritative
Literature Help

Standards that provide guidance (A) (B) (C) (D) (E) (F) (G) (H) (I) (J)
1. For performance of a review of a nonpublic company’s annual financial
statements.
2. On whether a contingent fee may be billed to a client.
3. Related to firm requirements of CPA firms that are enrolled in an
AICPA­approved practice­monitoring system.
4. For an examination of a client’s financial forecast.
5. Relating to overall requirements when providing services for an
advisory services engagement.
6. For the audit of a public company.
7. For the performance of an interim review of the quarterly financial
statements of a nonpublic audit client.
8. For reporting on client pro forma financial information.
9. On whether an investment of a CPA impairs her independence with
respect to a client.
10. On performing a compilation of a nonpublic company’s quarterly
statements.

Task­Based Simulation 7
Research
Authoritative
Literature Help

(A) (B) (C) (D) (E)  (F) (G) (H)
1. Which title of the Professional Standards addresses this issue and will be
helpful in responding to the partner?
2. Enter the exact section and paragraphs with helpful information. 302 1
24 MODULE 2   ENGAGEMENT PLANNING AND ASSESSING RISKS

ENGAGEMENT PLANNING, OBTAINING AN UNDERSTANDING
OF THE CLIENT AND ASSESSING RISKS
MULTIPLE­CHOICE QUESTIONS (1­113)
6. Financial statement assertions are 
established foraccount balances,
1.  Which   of   the   following   is   a   conceptual Classes of transactions Disclosures
difference be­tween the attestation standards a. Yes Yes
and generally accepted au­diting standards? b. Yes No
a. The attestation standards do not apply c. No Yes
to   audits   of   historical   financial d. No No
statements,   while   the   generally
accepted auditing standards do.  7. Which   of   the   following   is  not  a   financial
b. The requirement that the practitioner statement   as­sertion   relating   to   account
be indepen­dent in mental attitude is balances? 
omitted   from   the   attesta­tion a. Completeness. 
standards.  b. Existence. 
c. The   attestation   standards   do  not c. Rights and obligations. 
permit  an  attest engagement  to  be d. Valuation and competence. 
part of a business acquisition study
or a feasibility study.  8. As the acceptable level of detection
d. None of the standards of fieldwork in  risk decreases, an auditor may 
generally ac­cepted auditing standards are 
included in the at­testation standards. 
2. Which   of   the   following   is  not  an indications of employee
attestation standard?  fraud and illegal acts. 
a. Sufficient evidence shall be c. Objective   review   of   the
obtained to provide a  adequacy   of   the   technical
reasonable basis for the  training and proficiency of
conclusion that is ex­ firm personnel. 
pressed in the report.  d. Critical   review   of   the
b. The report shall identify the judgment   exercised   at
subject   matter   on   the every level of supervision. 
assertion being reported on
5. After fieldwork audit procedures are
and   state   the   character   of
completed, a partner of the CPA 
the engagement. 
firm who has not been involved in 
c. The   work   shall   be the audit performs a second or wrap­
adequately   planned   and up working paper review. This 
assis­tants, if any, shall be second review usually focuses on 
properly supervised. 
a. The fair presentation of the
d. A   sufficient   understanding
financial   statements   in
of   internal   control   shall   be
conformity with GAAP. 
obtained   to   plan   the
b. Fraud   involving   the
engagement. 
client’s   management   and
3. Which of the following is most likely its employees. 
to be unique to the audit work of  c. The   materiality   of   the
CPAs as compared to work  adjusting entries proposed
performed by practitioners of other  by the audit staff. 
professions?  d. The   communication   of   internal
a. Due professional care.  control   weaknesses   to   the   client’s
b. Competence.  audit committee. 
c. Independence. 
d. Complex body of knowledge. 
4. The   third   general   standard   states
that   due care  is  to  be  exercised  in
the   performance   of   an   audit.   This
standard is ordinarily interpreted to
require 
a. Thorough   review   of   the
existing   safeguards   over
access   to   assets   and
records. 
b. Limited   review   of   the
a. Reduce   substantive   testing   by
relying   on   the   as­sessments   of
inherent risk and control risk. 
b. Postpone   the   planned
timing of substantive tests
from   interim  dates  to   the
year­end. 
c. Eliminate the assessed level
of   inherent   risk   from
consideration as a planning
factor. 
d. Lower the assessed level of
control   risk   from   the
maximum   level   to   below
the maximum. 
9. The risk that an auditor will conclude,
based on sub­stantive tests, that a 
material misstatement does not exist 
in an account balance when, in fact, 
such misstatement does exist is 
referred to as 
a. Sampling risk. 
b. Detection risk. 
c. Nonsampling risk. 
d. Inherent risk. 
10. As the acceptable level of detection
risk decreases, the assurance directly
provided from 
a. Substantive tests should increase. 
b. Substantive tests should decrease. 
c. Tests of controls should increase. 
d. Tests of controls should decrease. 
11. Which   of   the   following   audit   risk
components   may   be   assessed   in
nonquantitative terms? 
Control risk Detection risk Inherent risk
a. Yes Yes No
b. Yes No Yes
c. Yes Yes Yes
d. No Yes Yes
12. Inherent   risk   and   control   risk   differ
from detection risk in that they 
a. Arise   from   the
misapplication   of   auditing
proce­dures. 
b. May   be   assessed   in
either   quantitative   or
non­quantitative terms. 
c. Exist   independently   of   the
financial statement au­dit. 
d. Can   be   changed   at   the   auditor’s
discretion. 
13. On the basis of the audit evidence 
gathered and evalu­ated, an auditor 
decides to increase the assessed level of
control risk from that originally 
planned. To achieve an 
MODULE 2   ENGAGEMENT PLANNING AND ASSESSING RISKS 25
b. Corroborating evidence. 
c. Quality control. 
overall audit risk level that is substantially  d. Materiality and relative risk. 
the same as the planned audit risk level, the 
auditor would 18. In considering materiality for 
a. Decrease substantive testing.  planning purposes, an auditor 
b. Decrease detection risk.  believes that misstatements 
c. Increase inherent risk.  aggregating $10,000 would have a 
d. Increase materiality levels.  material effect on an entity’s income 
state­ment, but that misstatements 
14. Relationship   between   control   risk would have to aggregate $20,000 to 
and detection risk is ordinarily  materially affect the balance sheet. 
a. Parallel.  Ordinarily, it would be appropriate to 
b. Inverse.  design auditing procedures that 
c. Direct.  would be expected to detect 
d. Equal.  misstatements that aggregate 
a. $10,000 
15. Which of the following would an  b. $15,000 
auditor most likely use in determining c. $20,000 
the auditor’s preliminary judgment  d. $30,000 
about materiality? 
a. The   anticipated   sample   size   of   the 19. Which of the following would an 
planned substan­tive tests.  auditor most likely use in 
b. The   entity’s   annualized determining the auditor’s preliminary
interim   financial   state­ judgment about materiality? 
ments.  a. The results of the initial assessment of
c. The   results   of   the   internal   control control risk. 
questionnaire. 
d. The   contents   of   the
management   representation
let­ter. 
16. Which of the following statements is
not correct about materiality? 
a. The concept of materiality 
recognizes that some matters
are important for fair 
presentation of finan­cial 
statements in conformity 
with GAAP, while other 
matters are not important. 
b. An auditor considers 
materiality for planning 
pur­poses in terms of the 
largest aggregate level of 
misstatements that could be
material to any one of the 
financial statements. 
c. Materiality judgments are 
made in light of sur­
rounding circumstances 
and necessarily involve 
both quantitative and 
qualitative judgments. 
d. An auditor’s consideration 
of materiality is influ­enced 
by the auditor’s perception 
of the needs of a reasonable 
person who will rely on the 
financial statements. 
17. Which of the following elements 
underlies the applica­tion of 
generally accepted auditing 
standards, particularly the standards 
of fieldwork and reporting? 
a. Internal control. 
c. Reasonable assurance. 
b. The   anticipated   sample d. Supervision. 
size   for   planned   substan­
tive tests.  23. Professional skepticism  requires that
c. The   entity’s   financial   statements   of an auditor assume that management is
the prior year.  a. Honest,  in   the   absence   of  fraud  risk
d. The   assertions   that   are factors. 
embodied in the financial b. Dishonest   until   completion   of   audit
statements.  tests. 
c. Neither honest nor dishonest. 
20. Holding other planning considerations equal, a  d. Offering   reasonable   assurance   of
decrease in the amount of misstatement in a  honesty. 
class of transactions that an auditor could 
tolerate most likely would cause the auditor to  24. Which   of   the   following   is   an
a. Apply   the   planned example   of   fraudulent   financial
substantive tests prior to reporting? 
the balance sheet date.  a. Company   management
b. Perform   the   planned changes   inventory   count
auditing   procedures   closer tags and overstates ending
to the balance sheet date.  inventory,   while   un­
c. Increase   the   assessed   level derstating   cost   of   goods
of control risk for rele­vant sold. 
financial   statement b. The treasurer diverts 
assertions.  customer payments to his 
d. Decrease   the   extent   of personal due, concealing his 
auditing procedures to be actions by debiting an 
applied   to   the   class   of expense account, thus 
transactions.  overstating expenses. 
c. An   employee   steals
21. When issuing an unqualified opinion, inventory   and   the
the   auditor   who   evaluates   the   audit “shrinkage”   is   recorded   in
findings should be satisfied that the  cost of goods sold. 
a. Amount   of   known d. An   employee   steals   small
misstatement   is tools from the company and
documented   in   the neglects to return them; the
management   representation cost   is   reported   as   a
letter.  miscellaneous   operating
b. Estimate   of   the   total expense. 
likely   misstatement   is
less   than   a   material 25. Which of the following best describes
amount.  what is meant by the term “fraud risk
c. Amount   of   known factor?” 
misstatement   is a. Factors   whose   presence
acknowledged   and indicates   that   the   risk   of
recorded by the client.  fraud is high. 
d. Estimate of the total likely misstatement  b. Factors   whose   presence
includes the adjusting entries already  often have been observed in
recorded by the client.  circumstances   where   frauds
have occurred. 
22. An   attitude   that   includes   a c. Factors   whose   presence
questioning   mind   and   a   critical requires   modification   of
assessment   of   audit   evidence   is planned audit procedures. 
referred to as  d. Material weaknesses identified during
a. Due professional care.  an audit. 
b. Professional skepticism. 
26 MODULE 2   ENGAGEMENT PLANNING AND ASSESSING RISKS
tofraud? 
a. Failure   to   correct   known   reportable
26. Which   of   the   following   is   correct conditions on a timely basis. 
concerning   require­ments   about b. Nonfinancial
auditor   communications   about management’s
fraud?  preoccupation   with   the
a. Fraud that involves senior  selection   of   accounting
management should be reported  principles. 
directly to the audit committee  c. Significant portion of management’s 
regardless of the amount involved.  compensation represented by bonuses 
b. Fraud with a material effect based upon achieving unduly 
on the financial state­ments aggressive operating results. 
should  be  reported   directly d. Use   of   unusually
by   the   auditor   to   the conservative   accounting
Securities   and   Exchange prac­tices. 
Commission. 
32. Which   of   the   following   conditions
c. Fraud with a material effect on the 
identified   during   fieldwork   of   an
financial state­ments should ordinarily
audit   is   most   likely   to   affect   the
be disclosed by the auditor through 
use of an “emphasis of a matter”  auditor’s   assessment   of   the   risk   of
paragraph added to the audit report.  misstatement due to fraud? 
d. The   auditor   has   no a. Checks   for   significant
responsibility   to   disclose amounts outstanding at
fraud   outside   the   entity year­end. 
under any circumstances. 
27. When performing a financial 
statement audit, auditors are 
required to explicitly assess the risk 
of material misstatement due to 
a. Errors. 
b. Fraud. 
c. Illegal acts. 
d. Business risk. 
28. Audits of financial statements are
designed   to   obtain   assurance   of
detecting misstatement due to 
Fraudulent Misappropriation
Errors financial reporting of assets
a. Yes Yes Yes
b. Yes Yes No
c. Yes No Yes
d. No Yes No
29. An auditor is unable to obtain absolute 
assurance thatmisstatements due to fraud will
be detected for all of the following except
a. Employee collusion. 
b. Falsified documentation. 
c. Need  to  apply  professional
judgment   in   evaluating
fraud risk factors. 
d. Professional skepticism. 
30. The   most   difficult   type   of
misstatement   to   detect   is   fraud
based on 
a. The overrecording of transactions. 
b. The nonrecording of transactions. 
c. Recorded transactions in subsidiaries. 
d. Related­party receivables. 
31. When considering fraud risk 
factors relating to man­agement’s 
characteristics, which of the 
following is leastlikely to indicate 
a risk of possible misstatement due
personnel is low. 
b. Computer generated documents.  b. Insiders   recently   purchased
c. Missing documents.  additional   shares   of   the
d. Year­end adjusting journal entries.  entity’s stock. 
c. Management   places
33. Which of the following is most  substantial   emphasis
likely to be a response to the  on   meeting   earnings
auditor’s assessment that the risk of  projections. 
material misstatement due to fraud  d. The   rate   of   change   in   the   entity’s
for the existence of inventory is  industry is slow. 
high? 
a. Observe   test   counts   of 36. Which   of   the   following   statements
inventory at certain loca­ reflects an auditor’s responsibility for
tions on an unannounced detecting misstatements due to errors
basis.  and fraud? 
b. Perform   analytical a. An auditor is responsible 
procedures   rather   than for detecting employee 
taking test counts.  errors and simple fraud, 
c. Request that inventories be but not for discovering 
counted prior to year­end.  fraud involving employee 
d. Request   that   inventory collusion or manage­ment
counts   at   the   various   loca­ override. 
tions be counted on different b. An auditor should plan the
dates so as to allow the same audit to detect mis­
auditor   to   be   present   at statements due to errors 
every count.  and fraud that are caused 
by departures from GAAP.
34. Which of the following is most likely c. An auditor is not responsible
to be an example of fraud?  for detecting mis­statements 
a. Defalcations   occurring due to errors and fraud 
due to invalid electronic unless the ap­plication of 
approvals. 
GAAS would result in such 
b. Mistakes   in   the
detection. 
application of accounting
princi­ples.  d. An auditor should design 
c. Mistakes in processing data.  the audit to provide rea­
sonable assurance of 
d. Unreasonable   accounting
detecting misstatements due
estimates   arising   from
to errors and fraud that are 
oversight. 
material to the financial 
35. Which of the following  statements. 
characteristics most likely would 
37. Disclosure of fraud to parties other than a 
heighten an auditor’s concern 
client’s senior management and its audit 
about the risk of in­tentional 
committee or board of directors ordinarily is 
manipulation of financial 
not part of an auditor’s responsibility. 
statements? 
However, to which of the following outside 
a. Turnover   of   senior   accounting
parties may a duty to disclose fraud exist? 
MODULE 2   ENGAGEMENT PLANNING AND ASSESSING RISKS 27
conditions relating to finan­cial stress of 
employees or adverse relationships between a 
To a governmentcompany and its employees? 
To the SEC To a successor funding agency a. The auditor is required to
when the auditor when the from which the plan   the   audit   to   detect
client reports successor makes client receives these   conditions   on   all
an auditor appropriate financial audits. 
change inquiries assistance b. These conditions relate to 
a. Yes Yes No fraudulent financial re­
b. Yes No Yes porting, and an auditor is 
c. No Yes Yes required to plan the audit to 
d. Yes Yes Yes detect these conditions 
38. Under   Statements   on   Auditing when the client is ex­posed 
Standards,   which   of   the   following to a risk of misappropriation
would be classified as an error?  of assets. 
a. Misappropriation   of   assets c. The auditor is required to 
for   the   benefit   of   man­ plan the audit to detect 
agement.  these conditions whenever 
b. Misinterpretation   by they may result in mis­
management   of   facts   that statements. 
ex­isted  when the  financial d. The   auditor   is   not   required
statements were prepared.  to plan the audit to dis­cover
c. Preparation   of   records these conditions, but should
by employees to cover a consider   them   if   he   or   she
fraudulent scheme.  becomes   aware   of   them
d. Intentional omission of the during the audit. 
recording   of  a  transac­tion
to benefit a third party. 
39. What   assurance   does   the   auditor
provide   that   misstate­ments   due   to
errors, fraud, and direct effect illegal
acts that are material to the financial
statements will be detected? 
Direct effect
Errors Fraud illegal acts
a. Limited Negative Limited
b. Limited Limited Reasonable
c. Reasonable Limited Limited
d. Reasonable Reasonable Reasonable
40. Because of the risk of material 
misstatement, an audit of financial 
statements in accordance with 
generally ac­cepted auditing 
standards should be planned and 
performed with an attitude of 
a. Objective judgment. 
b. Independent integrity. 
c. Professional skepticism. 
d. Impartial conservatism. 
41. Which   of   the   following   most
accurately   summarizes   what   is
meant   by   the   term   “material
misstatement?” 
a. Fraud and direct­effect illegal acts. 
b. Fraud   involving   senior
management   and   material
fraud. 
c. Material   error,   material
fraud,   and   certain   illegal
acts. 
d. Material   error   and   material   illegal
acts. 

42. Which of the following statements best 
describes the auditor’s responsibility to detect 
46. At  which   stage(s)   of   the   audit   may
43. When the auditor believes a 
fraud risk factors be identified? 
misstatement is or may be the result 
of fraud but that the effect of the  Obtaining Conducting
misstatement is not material to the  Planning Understanding fieldwork
financial statements, which of the  a. Yes Yes Yes
fol­lowing steps is required?  b. Yes Yes No
a. Consider   the   implications c. Yes No No
for   other   aspects   of   the d. No Yes Yes
audit.  47. Management’s attitude toward 
b. Resign from the audit.  aggressive financial reporting and its 
c. Commence a fraud examination.  emphasis on meeting projected profit 
d. Contact regulatory authorities.  goals most likely would significantly 
influence an entity’s control 
44. Which of the following statements is
environment when 
correct   relating   to   the   auditor’s
a. External policies established
consideration of fraud? 
by parties outside the entity
a. The auditor’s interest in fraud 
affect   its   accounting
consideration relates to fraudulent acts
practices. 
that cause a material misstate­ment of 
b. Management   is   dominated
financial statements. 
by   one   individual   who   is
b. A   primary   factor   that
also a shareholder. 
distinguishes fraud from er­
c. Internal auditors have direct
ror   is   that  fraud  is   always
access   to   the   board   of
intentional, while errors are
directors   and   the   entity’s
generally,   but   not   always,
management. 
intentional. 
c. Fraud   always   involves   a
d. The   audit   committee   is   active   in
overseeing   the   en­tity’s   financial
pressure   or   incentive   to
reporting policies. 
commit   fraud,   and   a
misappropriation of assets.  48. Which of the following is least likely
d. While an auditor should be aware of  to be required on an audit? 
the possibility of fraud, management,  a. Test   appropriateness   of
and not the auditor, is re­sponsible for journal   entries   and   adjust­
detecting fraud.  ment. 
b. Review   accounting   estimates   for
45. Which of the following factors
biases. 
or conditions is an auditor least
c. Evaluate   the   business
likely   to   plan   an   audit   to
rationale for significant un­
discover? 
usual transactions. 
a. Financial   pressures   affecting
d. Make   a   legal
employees. 
determination   of   whether
b. High turnover of senior management.  fraud has occurred. 
c. Inadequate   monitoring   of   significant
controls.  49. Which   of   the   following   is   most
d. Inability   to   generate likely to be an overall response to
positive   cash   flows   from fraud risks identified in an audit? 
op­erations. 
28 MODULE 2   ENGAGEMENT PLANNING AND ASSESSING RISKS
d. Small high­dollar inventory items. 

a. Supervise members of the 54. Which of the following is most likely


audit team less closely and to be presumed to represent fraud risk
rely more upon judgment.  on an audit? 
b. Use less predictable audit procedures.  a. Capitalization   of   repairs
c. Only use certified public and   maintenance   into   the
accountants   on   the   en­ property,   plant,   and
gagement.  equipment asset account. 
d. Place increased emphasis on b. Improper revenue recognition. 
the   audit   of   objective c. Improper interest expense accrual. 
transactions   rather   than d. Introduction   of   significant   new
subjective transactions.  products. 

50. Which of the following is least  55. An auditor who discovers that a 


likely to be included in an auditor’s  client’s employees paid small 
inquiry of management while  bribes to municipal officials most 
obtaining infor­mation to identify the likely would withdraw from the 
risks of material misstatement due to engagement if 
fraud?  a. The   payments   violated   the
a. Are   financial   reporting client’s   policies   regard­ing
operations   controlled   by the   prevention   of   illegal
and   limited   to   one acts. 
location?  b. The client receives financial
b. Does   it   have   knowledge   of   fraud   or assistance   from   a   fed­eral
suspect fraud?  government agency. 
c. Does   it   have   programs   to   mitigate c. Documentation   that   is
fraud risks?  necessary to prove that the
d. Has it reported to the audit bribes were paid does  not
committee the nature of the exist. 
company’s internal control?  d. Management   fails   to   take
the   appropriate   remedial
51. Individuals who commit fraud are action. 
ordinarily   able   to   rationalize   the
act and also have an  56. Which of the following factors most
Incentive Opportunity likely   would   cause   a   CPA   to  not
a. Yes Yes accept a new audit engagement? 
b. Yes No
c. No Yes
d. No No
52. What   is   an   auditor’s   responsibility
who   discovers   man­agement
involved   in   what   is   financially
immaterial fraud? 
a. Report   the   fraud   to   the   audit
committee. 
b. Report   the   fraud   to   the
Public   Company   Oversight
Board. 
c. Report the fraud to a level
of management at least one
below those involved in the
fraud. 
d. Determine   that   the
amounts   involved   are
immate­rial,   and   if   so,
there   is   no   reporting
responsibility. 
53. Which   of   the   following   is   most   likely   to   be
considered   a  risk   factor   relating   to   fraudulent
financial reporting? 
a. Domination   of   management   by   top
executives. 
b. Large amounts of cash processed. 
c. Negative cash flows from operations. 
c. Violations of antitrust laws. 
a. The   prospective   client d. Price fixing. 
has already completed its
physical inventory count. 60. Which of the following relatively 
b. The   CPA   lacks   an small misstatements most likely could
understanding   of   the have a material effect on an entity’s 
prospective   client’s finan­cial statements? 
operation and industry.  a. An   illegal   payment   to   a
c. The   CPA   is   unable   to foreign   official   that   was
review   the   predecessor not recorded.
audi­tor’s working papers.  b. A piece of obsolete office
d. The   prospective   client   is equipment   that   was  not
unwilling   to   make   all   fi­ retired. 
nancial   records   available c. A   petty   cash   fund
to the CPA.  disbursement   that   was  not
57. Which of the following factors  prop­erly authorized. 
would most likely heighten an  d. An   uncollectible   account
auditor’s concern about the risk of  receivable   that   was  not
fraudulent financial reporting?  written off. 
a. Large   amounts   of   liquid
61. During the annual audit of Ajax Corp., a 
assets   that   are   easily   con­
vertible into cash.  publicly held company, Jones, CPA, a 
b. Low   growth   and continuing auditor, determined that illegal 
profitability as compared to political contributions had been made during 
other   entities   in   the   same each of the past seven years, including the year 
industry.  under audit. Jones notified the board of 
directors about the illegal contributions, but 
c. Financial   management’s
they refused to take any action because the 
participation   in   the   initial
amounts in­volved were immaterial to the 
selection   of   accounting
financial statements. Jones should reconsider 
principles. 
the intended degree of reliance to be placed on 
d. An   overly   complex the 
organizational   structure
a. Letter of audit inquiry to the client’s
involv­ing   unusual   lines   of
authority.  attorney. 
b. Prior years’ audit programs. 
58. An auditor who discovers that a 
client’s employees have paid small 
bribes to public officials most likely 
would withdraw from the 
engagement if the 
a. Client   receives   financial
assistance   from   a   federal
government agency. 
b. Evidence  that is necessary
to prove that the illegal acts
were   committed   does   not
exist. 
c. Employees’ actions affect
the   auditor’s   ability   to
rely   on   management’s
representations. 
d. Notes   to   the   financial
statements   fail   to   disclose
the employees’ actions. 
59. Which of the following illegal acts
should   an   audit   be   designed   to
obtain   reasonable   assurance   of
detecting? 
a. Securities   purchased   by
relatives   of   management
based   on   knowledge   of
inside information. 
b. Accrual   and   billing   of   an
improper   amount   of   reve­
nue   under   government
contracts. 
MODULE 2   ENGAGEMENT PLANNING AND ASSESSING RISKS 29
necessary. 
c. The   illegal   act   was
c. Management representation letter.  committed   during   a   prior
d. Preliminary   judgment   about year that was not audited. 
materiality levels.  d. The   auditor   has   already
assessed   control   risk   at   the
62. The most likely explanation why the  maximum level. 
auditor’s ex­amination cannot 
reasonably be expected to bring all  65. Under the Private Securities 
illegal acts by the client to the  Litigation Reform Act of 1995, 
auditor’s attention is that  Baker, CPA, reported certain 
a. Illegal acts are perpetrated uncorrected illegal acts to 
by   management   over­ride Supermart’s board of directors. Baker
of internal control.  believed that fail­ure to take remedial
b. Illegal acts by clients often action would warrant a qualified 
relate to operating as­pects audit opinion because the illegal acts 
rather   than   accounting had a material effect on Su­permart’s 
aspects.  financial statements. Supermart failed
c. The client’s internal control to take appropriate remedial action 
may be so strong that the  and the board of directors re­fused to 
auditor performs only  inform the SEC that it had received 
minimal substantive  such notifica­tion from Baker. Under 
testing.  these circumstances, Baker is re­
d. Illegal   acts   may   be quired to 
perpetrated   by   the   only a. Resign   from   the   audit
person   in   the   client’s engagement within ten busi­
organization with access to ness days. 
both   as­sets   and   the b. Deliver   a   report
accounting records.  concerning the illegal acts
to   the   SEC   within   one
63. If specific information comes to an  business day. 
auditor’s attention that implies the  c. Notify   the   stockholders
existence of possible illegal acts that that   the   financial   state­
could have a material, but indirect  ments   are   materially
effect on the financial state­ments,  misstated. 
the auditor should next  d. Withhold an audit opinion
a. Apply   audit   procedures until   Supermart   takes
specifically   directed   to   as­ appropriate   remedial
certaining   whether   an action. 
illegal act has occurred. 
66. Which of the following would be least likely to
b. Seek   the   advice   of   an
informed expert qualified to be con­sidered an audit planning procedure? 
practice  law  as to possible
contingent liabilities. 
c. Report   the   matter   to   an
appropriate   level   of   man­
agement   at   least   one   level
above those involved. 
d. Discuss   the   evidence   with
the   client’s   audit   com­
mittee,   or   others   with
equivalent   authority   and
re­sponsibility. 
64. An auditor who discovers that 
client employees have committed 
an illegal act that has a material 
effect on the client’s financial 
statements most likely would 
withdraw from the engagement if 
a. The   illegal   act   is   a
violation   of   generally
accepted   accounting
principles. 
b. The   client  does   not  take
the   remedial   action   that
the   auditor   considers
predecessor had with the 
a. Use an engagement letter.  client concerning auditing 
b. Develop the overall audit strategy.  procedures and accounting 
c. Perform risk assessment.  principles. 
d. Develop the audit plan.  b. The   predecessor’s   evaluation   of
matters   of   continu­ing   accounting
67. Which of the following factors would significance. 
most likely cause a CPA to decide not c. The   degree   of   cooperation   the
to accept a new audit engagement?  predecessor   received   concerning   the
a. The   CPA’s   lack   of inquiry of the client’s lawyer. 
understanding   of   the d. The   predecessor’s
prospec­tive   client’s assessments of inherent risk
internal   auditor’s and   judgments   about
computer­assisted   audit materiality. 
techniques. 
b. Management’s disregard of  70. Before accepting an audit 
its responsibility to maintain  engagement, a successor auditor 
an adequate internal control  should make specific inquiries of 
environment.  the predecessor auditor regarding 
the predecessor’s 
c. The CPA’s inability to 
determine whether related­ a. Opinion of any subsequent
party transactions were  events   occurring   since   the
consummated on terms  predecessor’s   audit   report
equivalent to arm’s­length  was issued. 
transactions.  b. Understanding   as   to   the
d. Management’s refusal to reasons  for  the  change   of
permit   the   CPA   to   per­ auditors. 
form   substantive   tests c. Awareness   of   the
before the year­end.  consistency   in   the
application   of   GAAP
68. Before   accepting   an   engagement   to between periods. 
audit a new client, a CPA is required d. Evaluation of all matters of
to obtain  continuing   accounting
a. An   understanding   of   the significance. 
prospective   client’s   indus­
try and business.  71. An auditor is required to establish an 
b. The   prospective   client’s understanding with a client regarding 
signature   to   the   engage­ the services to be performed for each 
ment letter.  engagement. This understanding 
generally includes 
c. A   preliminary   understanding   of   the
prospective   cli­ent’s   control a. Management’s
environment.  responsibility for errors and
the   il­legal   activities   of
d. The   prospective   client’s
employees   that   may   cause
consent   to   make   inquiries
mate­rial misstatement. 
of the predecessor auditor,
if any.  b. The   auditor’s
responsibility for ensuring
69. Before accepting an audit  that the audit committee is
engagement, a successor auditor  aware   of   any   significant
should make specific inquiries of  defi­ciencies   in   internal
the predecessor auditor regarding  control   that   come   to   the
a. Disagreements the  audi­tor’s attention. 
30 MODULE 2   ENGAGEMENT PLANNING AND ASSESSING RISKS
they exist, will not be 
detected. 
c. Management’s  d. After performing our 
responsibility for  preliminary analytical 
providing the auditor with  proce­dures we will discuss 
an assessment of the risk  with you the other proce­
of material misstatement  dures we consider necessary
due to fraud.  to complete the en­
d. The auditor’s responsibility for  gagement. 
determining pre­liminary judgments 
about materiality and audit risk  75. Which   of   the   following
factors.  documentation is not required for an
audit   in   accordance   with   generally
72. Which   of   the   following   matters   is accepted auditing standards? 
generally   included   in   an   auditor’s a. A   written   audit   plan
engagement letter?  setting   forth   the
a. Management’s procedures   necessary   to
responsibility   for   the accomplish   the   audit’s
entity’s   com­pliance   with objectives. 
laws and regulations.  b. An   indication   that   the
b. The factors to be considered accounting records agree or
in   setting   preliminary reconcile with the financial
judgments   about statements. 
materiality.  c. A client engagement 
c. Management’s   vicarious letter that summarizes the
liability   for   illegal   acts timing and details of the 
committed   by   its auditor’s planned field­
employees.  work. 
d. The   auditor’s d. The   assessment   of   the
responsibility to search for risks of material misstate­
signifi­cant   internal ment. 
control deficiencies. 
76. An   engagement   letter   should
73. During  the  initial  planning  phase ordinarily   include   infor­mation   on
of   an   audit,   a   CPA   most   likely the   objectives   of   the   engagement
would  and 
a. Identify   specific   internal
CPA Client Limitation of
control   activities   that   are responsibilities responsibilities engagement
likely to prevent fraud.  a. Yes Yes Yes
b. Evaluate   the b. Yes No Yes
reasonableness   of   the c. Yes No No
client’s   ac­counting d. No No No
estimates. 
c. Discuss   the   timing   of   the
audit   procedures   with   the
client’s management. 
d. Inquire   of   the   client’s
attorney as to whether any
unrecorded   claims   are
probable of assertion. 
74. Which of the following statements
would  least  likely   appear   in   an
auditor’s engagement letter? 
a. Fees   for   our   services   are
based   on   our   regular   per
diem   rates,   plus  travel  and
other   out­of­pocket   ex­
penses. 
b. During the course of our 
audit we may observe op­
portunities for economy in, 
or improved controls over, 
your operations. 
c. Our engagement is subject 
to the risk that material 
misstatements or fraud, if 
factory equipment. 
77. Arrangements concerning which of
81. A CPA wishes to determine how various 
the following are  least  likely to be
publicly held companies have complied with 
included in engagement letter? the disclosure requirements of a new financial 
a. A predecessor auditor.  accounting standard. Which of the fol­lowing 
b. Fees and billing.  information sources would the CPA most likely
c. CPA investment in client securities.  con­sult for information? 
d. Other   services   to   be   provided   in a. AICPA   Codification   of
addition to the au­dit.  Statements   on   Auditing
Standards. 
78. The auditor should document the
b. AICPA  Accounting   Trends   and
understanding es­tablished with a
Techniques. 
client through a(n) 
a. Oral communication with the client.  c. SEC Quality Control Review. 
b. Written   communication   with   the d. SEC Statement 10­K Guide. 
client.  82. An auditor should design the written audit
c. Written   or   oral   communication   with program so 
the client.  that 
d. Completely detailed audit plan. 
a. All   material   transactions
79. Which of the following factors most will   be   selected   for   sub­
likely would in­fluence an auditor’s stantive testing. 
determination  of  the  auditability  of b. Substantive tests prior to
an entity’s financial statements?  the   balance   sheet   date
a. The   complexity   of   the   accounting will be minimized. 
system.  c. The   audit   procedures   selected   will
b. The   existence   of   related­party achieve specific audit objectives. 
transactions.  d. Each account balance will
c. The   adequacy   of   the   accounting be   tested   under   either
records.  tests of controls or tests of
transactions. 
d. The operating effectiveness of control
procedures.  83. The   audit   program   usually  cannot  be
finalized until the 
80. Which of the following is most likely
a. Consideration   of   the
to   require   special   planning
entity’s   internal   control
considerations   related   to   asset
has been completed. 
valuation? 
b. Engagement   letter   has
a. Inventory   is   comprised   of   diamond
been signed by the auditor
rings. 
and the client. 
b. The   client   has   recently
c. Reportable   conditions   have
purchased   an   expensive
been   communicated   to   the
copy machine. 
audit   committee   of   the
c. Assets   costing   less   than board of directors. 
$250   are   expensed   even
d. Search   for   unrecorded
when   the   expected   life
liabilities has been per­
exceeds one year. 
formed   and
d. Accelerated depreciation methods are documented. 
used   for   am­ortizing   the   costs   of
MODULE 2   ENGAGEMENT PLANNING AND ASSESSING RISKS 31
c. Read current issues
of   specialized
84. Audit programs should be designed so that  industry journals. 
a. Most   of   the   required d. Reevaluate   the   client’s
procedures   can   be internal   control   environ­
performed as interim work.  ment. 
b. Inherent   risk   is   assessed   at   a
89. On an audit engagement performed 
sufficiently low level. 
by a CPA firm with one office, at the 
c. The   auditor   can   make minimum, knowledge of the relevant 
constructive suggestions to professional accounting and auditing 
management. 
standards should be held by 
d. The audit evidence gathered
a. The   auditor   with   final   responsibility
supports   the   auditor’s
for the audit. 
conclusions. 
b. All   professionals   working   upon   the
85. In designing written audit programs,  audit. 
an auditor should establish specific  c. All professionals working
audit objectives that relate primarily  upon   the   audit   and   the
to the  partner   in   charge   of   the
a. Timing of audit procedures.  CPA firm. 
b. Cost­benefit of gathering evidence.  d. All   professionals   working   in   the
c. Selected audit techniques.  office. 
d. Financial statement assertions.  90. An   auditor   obtains   knowledge
86. With   respect   to   planning   an   audit, about a new client’s business and
which of the follow­ing statements is its industry to 
always true?  a. Make   constructive
a. It is acceptable to perform a suggestions   concerning
portion   of   the   audit   of   a im­provements   to   the
continuing   audit   client   at client’s internal control.
interim dates. 
b. An   engagement   should
not be accepted after the
client’s year­end. 
c. An inventory count must be observed
at year­end. 
d. Final staffing decisions
must be  made  prior  to
completion   of   the
planning stage. 
87. The element of the audit planning process most 
likely to be agreed upon with the client before 
implementation of the audit strategy is the 
determination of the 
a. Evidence to be gathered to
provide   a   sufficient   ba­sis
for the auditor’s opinion. 
b. Procedures   to   be
undertaken   to   discover
litigation,   claims,   and
assessments. 
c. Pending legal matters to be
included   in   the   inquiry   of
the client’s attorney. 
d. Timing   of   inventory
observation   procedures   to
be performed. 
88. To obtain an understanding of a
continuing   client’s   business,   an
auditor most likely would 
a. Perform   tests   of   details
of   transactions   and   bal­
ances. 
b. Review prior year working
papers and the perma­nent
file for the client. 
that Star’s prior year’s financial 
c. Evaluate whether the  statements, which were audited by 
aggregation of known mis­ Tell, require substantial revisions. 
statements causes the  Under these circum­stances, Land 
financial statements taken as should 
a whole to be materially  a. Notify Star’s audit 
misstated.  committee and stockholders 
d. Understand the events and  that the prior year’s 
transactions that may have an financial statements cannot 
effect on the client’s  be relied on. 
financial statements.  b. Request Star to reissue the
prior   year’s   financial
91. Which of the following procedures 
statements   with   the
would an auditor least likely  appropriate revisions. 
perform while obtaining an  c. Notify   Tell   about   the   information   and
understanding of aclient in a 
make   inquir­ies   about   the   integrity   of
financial statement audit? 
Star’s management. 
a. Coordinating the assistance
d. Request   Star   to   arrange   a
of entity personnel in data
meeting   among   the   three
preparation. 
parties to resolve the matter.
b. Discussing matters that may 
affect the audit with firm  94. A successor auditor should request
personnel responsible for  the   new   client   to   authorize   the
nonaudit services to the  predecessor   auditor   to   allow   a
entity.  review of the predecessor’s 
c. Selecting   a   sample   of Engagement letter Working papers
vendors’ invoices for com­ a. Yes Yes
parison   to   receiving b. Yes No
reports.  c. No Yes
d. Reading  the current year’s d. No No
interim   financial   state­
ments.  95. Which of the following procedures would an 
auditormost likely perform in planning a 
92. Ordinarily, the predecessor auditor permits the  financial statement audit?
succes­sor auditor to review the predecessor’s 
a. Inquiring   of   the   client’s   legal
working paper analy­ses relating to 
counsel   concerning   pending
Contingencies Balance sheet accounts litigation. 
a. Yes Yes b. Comparing   the   financial   statements
b. Yes No to anticipated results. 
c. No Yes c. Examining   computer   generated
d. No No exception   reports   to   verify   the
93. In auditing the financial statements of effectiveness of internal control. 
Star Corp., Land discovered  d. Searching   for   unauthorized
information leading Land to believe  transactions   that   may   aid   in
detecting unrecorded liabilities. 
b. Develop an attitude of professional skepticism con­ 96.  Analytical procedures used in planning an
audit should
cerning management’s financial statement  focus on
asser­tions. a. Reducing the scope of tests of controls
and sub­stantive tests. 
32 MODULE 2   ENGAGEMENT PLANNING AND ASSESSING RISKS
with the conclusions to be
presented in the auditor’s
b. Providing   assurance   that report. 
potential   material   mis­ c. Audit   procedures
statements   will   be performed   are   approved
identified.  in   the   professional
c. Enhancing   the   auditor’s standards. 
understanding   of   the   cli­ d. Audit has been performed 
ent’s business.  by persons having ade­
d. Assessing   the   adequacy   of   the quate technical training 
available evidence.  and proficiency as audi­
tors. 
46. The   objective   of   performing
analytical   procedures   in   planning 50. While assessing the risks of 
an audit is to identify the existence material misstatement auditors 
of  identify risks, relate risk to what 
a. Unusual transactions and events.  could go wrong, consider the 
b. Illegal   acts   that   went magnitude of risks and 
undetected   because   of a. Assess the risk of misstatements due
inter­nal   control to illegal acts. 
weaknesses.  b. Consider the complexity
c. Related­party transactions.  of   the   transactions   in­
d. Recorded transactions that volved. 
were  not  properly   au­ c. Consider   the   likelihood
thorized.  that the risks could result
in material misstatements. 
47. Which of the following nonfinancial  d. Determine materiality levels. 
information would an auditor most 
likely consider in performing  51. Which of the following are 
analytical procedures during the  considered further audit procedures 
planning phase of an audit?  that may be designed after assessing 
a. Turnover of personnel in the risks of material misstatement? 
the   accounting   depart­ Substantive Risk assessment
ment.  tests of details procedures
b. Objectivity   of   audit   committee a. Yes Yes
members.  b. Yes No
c. Square footage of selling space.  c. No Yes
d. Management’s   plans   to   repurchase d. No No
stock. 
48. The   in­charge   auditor   most   likely
would   have   a   super­visory
responsibility to explain to the staff
assistants 
a. That   immaterial   fraud   is
not  to   be   reported   to   the
client’s audit committee. 
b. How   the   results   of   various   auditing
procedures   per­formed   by   the
assistants should be evaluated. 
c. What   benefits   may   be
attained by the assistants’
adherence   to   established
time budgets. 
d. Why certain documents are
being   transferred   from   the
current   file   to   the
permanent file. 
49. The   audit   work   performed   by   each
assistant   should   be   reviewed   to
determine whether it was adequately
performed   and   to   evaluate   whether
the 
a. Auditor’s system of quality
control   has   been   main­
tained at a high level. 
b. Results   are   consistent
The nature
103. Which of the following is least likely The CPA of the CPA Cost­benefit
to   be   considered   a   risk   assessment firm’s size firm’s practice considerations
procedure?  a. Yes Yes Yes
b. Yes Yes No
a. Analytical procedures.  c. Yes No Yes
b. Confirmation   of   ending   accounts d. No Yes Yes
receivable. 
c. Inspection of documents.  108. A CPA firm may communicate its quality 
d. Observation   of   the controlpolicies and procedures to its personnel in 
performance   of   certain which manner(s):
account­ing procedures.  Orally Written
104. In   an   audit   of   a   nonissuer a. No No
(nonpublic) company, the auditors b. No Yes
identify   significant   risks.   These c. Yes No
d. Yes Yes
risks often 
a. Involve   routine,   high­volume 109. Which of the following is not an 
transactions.  element of qualitycontrol?
b. Do not require special audit attention.  a. Acceptance and continuance of client
c. Involve   items   with   lower   levels   of relationships   and   specific
inherent risk.  engagements. 
d. Involve judgmental matters.  b. Human resources. 
105. The auditor with final responsibility  c. Internal control. 
for an engage­ment and one of the  d. Monitoring. 
assistants have a difference of 
opinion about the results of an 
auditing procedure. If the assistant 
believes it is necessary to be 
disassociated from the matter’s 
resolution, the CPA firm’s 
procedures should enable the 
assistant to 
a. Refer   the   disagreement
to   the   AICPA’s   Quality
Review Committee. 
b. Document the details of the
disagreement   with   the
conclusion reached. 
c. Discuss   the   disagreement   with   the
entity’s   manage­ment   or   its   audit
committee. 
d. Report   the   disagreement
to   an   impartial   peer   re­
view monitoring team. 
106. An   examination   of   a   financial
forecast is a profes­sional service
that involves 
a. Compiling   or   assembling   a   financial
forecast   that   is   based   on
management’s assumptions. 
b. Limiting   the   distribution   of
the   accountant’s   report   to
management   and   the   board
of directors. 
c. Assuming   responsibility   to
update management on key
events for one year after the
report’s date. 
d. Evaluating the preparation 
of a financial forecast and 
the support underlying 
management’s assump­
tions. 
107. The   nature   and   extent   of   a   CPA
firm’s   quality   control   policies   and
procedures depend on 
MODULE 2   ENGAGEMENT PLANNING AND ASSESSING RISKS 33

110. Quality   control   for   a


CPA firm, as referred to
in   State­ments   on
Quality   Control
Standards, applies to 
a. Auditing services only. 
b. Auditing and management advisory services. 
c. Auditing and tax services. 
d. Auditing and accounting and review services. 
111. One of a CPA firm’s 
basic objectives is to 
provide professional 
services that conform 
with professional stan­
dards. Reasonable 
assurance of achieving 
this basic objec­tive is 
provided through 
a. A system of quality control. 
b. A system of peer review. 
c. Continuing professional education. 
d. Complianc
e   with
generally
accepted
reporting
standards. 
112. Which of the following
is   correct   concerning
PCAOB   guidance   that
uses the term “should”? 
a. The auditor must fulfill the responsibilities. 
b. The auditor 
must comply 
with 
requirements 
unless s/he 
demonstrates 
that alternative 
actions were 
sufficient to 
achieve the 
objectives of 
the standard. 
c. The auditor 
should 
consider 
performing 
the 
procedure; 
whether the 
auditor 
performs 
depends on 
the exercise 
of 
professional 
judgment in 
the 
circumstance
s. 
d. The   auditor
has   complete
discretion   as
to   whether   to
perform   the
procedure. 
113. Which of the following 
sets of standards does the
Pub­lic Company 
Accounting Oversight 
Board not have the au­
thority to establish for 
audits of public 
companies? 
a. Auditing standards. 
b. Quality control standards. 
c. Accounting standards. 
d. Independence standards. 
34 MODULE 2   ENGAGEMENT PLANNING AND ASSESSING RISKS

SIMULATIONS

Task­Based Simulation 1
Audit Risk
Application
Authoritative
Literature Help

Green, CPA, is considering audit risk, including fraud risk, at the financial statement level in planning the audit of National
Federal Bank (NFB) Company’s financial statements for the year ended December 31, 2005. Audit risk at the financial state­
ment level is influenced by the risk of material misstatements, which may be indicated by a combination of factors related to 
management, the industry, and the entity. In assessing such factors Green has gathered the following information concerning 
NFB’s environment.
Company profile
NFB is a federally insured bank that has been consistently more profitable than the industry average by marketing mort­
gages on properties in a prosperous rural area, which has experienced considerable growth in recent years. NFB packages its
mortgages and sells them to large mortgage investment trusts. Despite recent volatility of interest rates, NFB has been able to
continue selling its mortgages as a source of new lendable funds.
NFB’s board of directors is controlled by Smith, the majority stockholder, who also acts as the chief executive officer. 
Management at the bank’s branch offices has authority for directing and controlling NFB’s operations and is compensated based
on branch profitability. The internal auditor reports directly to Harris, a minority shareholder, who also acts as chairman of the 
board’s audit committee.
The accounting department has experienced little turnover in personnel during the five years Green has audited NFB. 
NFB’s formula consistently underestimates the allowance for loan losses, but its controller has always been receptive to 
Green’s suggestions to increase the allowance during each engagement.
Recent developments
During 2005, NFB opened a branch office in a suburban town thirty miles from its principal place of business. Although 
this branch is not yet profitable due to competition from several well­established regional banks, management believes that the
branch will be profitable by 2007. Also, during 2005, NFB increased the efficiency of its accounting operations by installing a 
new, sophisticated computer system.
Based only on the information above, indicate by marking the appropriate button whether the following factors indicate an
increased or decreased audit risk. Also, indicate whether the factor is a fraud risk factor.
Factor Increased audit risk Decreased audit risk Fraud risk factor
1. Branch management authority
2. Government regulation
3. Company profitability
4. Demand for product
5. Interest rates
6. Availability of mortgage funds
7. Involvement of principal shareholder in
Management
8. Branch manager compensation
9. Internal audit reporting relationship
10. Accounting department turnover
11. Continuing audit relationship
12.  Internal controls over accounting estimates
13.  Response to proposed accounting
Adjustments
14. New unprofitable branch
15. New computer system

MODULE 2   ENGAGEMENT PLANNING AND ASSESSING RISKS 35
Task­Based Simulation 2
Risk of Material Authoritative
Misstatement Literature Help

Green, CPA, is considering audit risk, including fraud risk, at the financial statement level in planning the audit of National
Federal Bank (NFB) Company’s financial statements for the year ended December 31, 2005. Audit risk at the financial state­
ment level is influenced by the risk of material misstatements, which may be indicated by a combination of factors related to 
management, the industry, and the entity. In assessing such factors Green has gathered the following information concerning 
NFB’s environment.
Company profile
NFB is a federally insured bank that has been consistently more profitable than the industry average by marketing mort­
gages on properties in a prosperous rural area, which has experienced considerable growth in recent years. NFB packages its
mortgages and sells them to large mortgage investment trusts. Despite recent volatility of interest rates, NFB has been able to
continue selling its mortgages as a source of new lendable funds.
NFB’s board of directors is controlled by Smith, the majority stockholder, who also acts as the chief executive officer. 
Management at the bank’s branch offices has authority for directing and controlling NFB’s operations and is compensated based
on branch profitability. The internal auditor reports directly to Harris, a minority shareholder, who also acts as chairman of the 
board’s audit committee.
The accounting department has experienced little turnover in personnel during the five years Green has audited NFB. 
NFB’s formula consistently underestimates the allowance for loan losses, but its controller has always been receptive to 
Green’s suggestions to increase the allowance during each engagement.
Recent developments
During 2005, NFB opened a branch office in a suburban town thirty miles from its principal place of business. Although 
this branch is not yet profitable due to competition from several well­established regional banks, management believes that the
branch will be profitable by 2007. Also, during 2005, NFB increased the efficiency of its accounting operations by installing a 
new, sophisticated computer system.
Assume you are preparing for the audit personnel discussion of potential risks of material misstatement due to fraud for 
the NFB audit. While any matters below might be discussed, indicate by marking the appropriate four highest risks based on 
the information contained in the simulation description and requirements of professional standards.
Risk High­risk item
1. Computer fraud risk
2. Risk related to management override of internal control
3. Fraud by branch management
4. Fraud by accounting personnel
5. Misstatement of accounting estimates
6. Fraud by loan processing clerks
7. Fraud by internal auditors
8. The risk of fraudulent misstatement of revenues

Task­Based Simulation 3
Research
Topics
Authoritative
Literature Help

Assume that you have been hired to perform the audit of Hanmei’s financial statements. When planning such an audit, you 
often may need to refer to various of the profession’s auditing standards. For each of the following circumstances in Column A, 
select the topic from the Professional Standards that is likely to provide the most guidance in the planning of the audit. A topic 
may be selected once, more than once, or not at all.
Topic
A. Analytical Procedures F. Illegal Acts by Clients
B. Audit Risk and Materiality G. Management Representations
C. Communications between Predecessor and Successor H. Part of the Audit Performed by Other Independent
Auditors Auditors
D. Consideration of Fraud in a Financial Statement Audit I. Related Parties
E. Understanding the Entity and Its Environment and
Assessing the Risks of Material Misstatement 
36 MODULE 2   ENGAGEMENT PLANNING AND ASSESSING RISKS

T
r
1. P

2. T
h
3. H

4. C
o
5. A
u
6. R
e
7. A
u
8. T
h
9. D

10. T
h
Task­Based Simulation 4
Risk Authoritative
Analysis Literature Help

DietWeb Inc. (hereafter DietWeb) was incorporated and began business in March of 20X1, 
seven years ago. You are working on the 20X8 audit—your CPA firm’s fifth audit of DietWeb.
The company’s mission is to provide solutions that help individuals to realize their full potential 
through better eating hab­its and lifestyles. Much of 20X1 and 20X2 was spent in developing a unique 
software platform that facilitates the production of individualized meal plans and shopping lists using 
a specific mathematical algorithm, which considers the user’s physical con­dition, proclivity to 
exercise, food preferences, cooking preferences, desire to use prepackaged meals or dine out, among 
others. DietWeb sold its first online diet program in 20X2 and has continued to market memberships 
through increasing online adver­tising arrangements through the years. The company has continued to 
develop this program throughout the years and finally became profitable in 20X6.
DietWeb is executing a strategy to be a leading online provider of services, information and 
products related to nutrition, fitness and motivation. In 20X8, the company derived approximately 86%
of its total revenues from the sale of approximately 203,000 personalized subscription­based online 
nutrition plans related to weight management, to dietary regimens such as vegetarianism and to 
specific medical conditions such as Type 2 diabetes. Given the personal nature of dieting, DietWeb 
assures customers of complete privacy of the information they provide. To this point DietWeb’s 
management is proud of its success in assuring the privacy of information supplied by its customers—
this is a constant battle given the variety of intrusion attempts by various Internet hackers.
DietWeb nutrition plans are paid in advance by customers and offered in increments of thirteen
weeks with the customers having the ability to cancel and receive a refund of the unused portion of the
subscription—this   results   in   a   significant   level   of   “deferred   revenue”   each   period.   Although   some
DietWeb members are billed through use of the postal system, most DietWeb members currently purchase
programs and products using credit cards, with renewals billed automatically, until cancellation.
One week of a basic DietWeb membership costs less than one­half the cost of a weekly visit to the 
leading classroom­based diet program. The president, Mr. William Readings, suggests that in addition 
to its superior cost­effectiveness, the DietWeb online diet program is successful relative to classroom­
based programs due to its customization, ease of use, expert support, privacy, constant availability, and
breadth of choice. The basic DietWeb membership includes
• Customized   meal   plans   and   workout   schedules   and   related   tools   such   as   shopping   lists,
journals, and weight and exercise tracking. 
• Interactive online support and education including approximately 100 message boards on 
various topics of interest to members and a library of dozens of multimedia educational 
segments presented by experts including psychologists, mental health counselors, 
dietitians, fitness trainers, a spiritual advisor and a physician. 
• 24/7/365   telephone   support   from   a   staff   of   approximately   30   customer   service
representatives, nutritionists and fitness personnel. 
Throughout its nine­year history, Mr. William Readings has served as chief executive officer. The
other three founders of the company are also officers. A fifth individual, Willingsley Williamson, also 
a founder, served as Chief Financial Officer until mid­20X8 when he left the company due to a 
difference of opinion with Mr. Readings. The four founders purchased Mr.
MODULE 2   ENGAGEMENT PLANNING AND ASSESSING RISKS 37

Williamson’s stock and invested an additional approximately $1.2 million in common stock during 
20X8 so as to limit the use of long­term debt.
The company’s board of directors is currently composed of the four individuals who remain 
active in the company; these four individuals also serve as the company’s audit committee; Mr. 
Readings chairs both the board and the audit committee. Previously, Mr. Readings had also served 
on the board and the audit committee. With Mr. Williamson’s departure, Ms. Jane Jennings, 
another of the founders, became the company’s CFO.
The nutrition and diet industry in many ways thrives because individuals are becoming more 
aware of the negative health and financial consequences of being overweight, and consider important 
both weight loss and healthy weight maintenance. A study by two respected researchers concluded 
that obesity was linked to higher rates of chronic illness than living in poverty, than smoking, or than 
drinking. In addition, the American Cancer Society reported that as many as 14% of cancer deaths in 
men and 20% of cancer deaths in women could be related to being overweight.
The financial costs of excess weight are also high. A 20X8 study based on data from a major 
automobile manufacturer’s health­care plan showed that an overweight adult has annual health­care 
costs that are 7.3% higher than a person in a healthy weight range, while obese individuals have 
annual health­care costs that are 69% higher than a person of a healthy weight. With health­care cost 
inflation running in the double digits in the United States since 20X4, supporters of the industry 
believe that the implementation of effective weight management tools will attract more attention from 
insurers, employers, consumers, and the government. As of January 20X9 five nutrition­ or fitness­
related bills were being considered in Congress, and several states had enacted or were considering 
enacting legislation relating to the sale of “junk” food in public schools. In addition, the US Food and 
Drug Administration, Department of Health and Human Services, and Federal Trade Commission are 
contemplating new labeling requirements for packaged food and restaurant food, new educational and 
motivational programs related to healthy eating and exercise, and increased regulation of advertising 
claims for food.
In response to consumers’ growing demand for more healthful eating options, quick­service 
and full­service restaurants have introduced new offerings including salads, sandwiches, burgers, 
and other food items designed for the weight­conscious person. At the retail level, sales of natural 
and organic foods have been growing more rapidly than the overall food and over­the­counter drug
market for the last several years. Nutritional supplement sales in the US, for instance, are estimated
to have grown 34% between 20X4 and 20X8, while natural and organic foods are estimated to be 
growing at a rate of approximately 15% annually. Also, the industry has a tendency to change 
quickly as “dieting fads” regularly are introduced; some remain popular for years, some for only 
months.
Approximately 60% of the US adult population, or 120 million adults, are overweight and, of 
those, the Calorie Control Council estimates only about 50 million are dieting in a given year. 
About 15% of these dieters are using a commercial weight loss center, generating revenues of 
approximately $1.5 billion annually. DietWeb targets dieters who are online, which repre­sents 
about two­thirds of the total universe at current Internet penetration rates, or 34 million adults, about
5 million of whom are spending approximately $1 billion at weight loss centers.
At the same time, the online dieting segment of the market is growing rapidly. The online diet 
industry in the US generated in excess of $100 million in 20X8, compared to revenues of 
approximately $75 million in 20X2. The industry includes other online nutrition and diet­oriented 
Web sites.
Another group of competitors to DietWeb are commercial weight loss centers, an industry that 
has shown marked decline in the last decade. According to Market Analysis Enterprises, the number 
of commercial weight loss centers in the US declined approximately 50% between 20X2 and 20X8, 
from over 8,600 to approximately 4,400. DietWeb competes against this segment on the basis of 
lower price, superior value, convenience, availability, the ability to personalize a meal plan on an 
ongoing basis, its extensive support capabilities, and the breadth of its meal plan options.

1. O
f
A. M
B. a
M
C. Ia
D. In
n
2. W

A. I
B. In
C. n
E
D. nM
i
38 MODULE 2   ENGAGEMENT PLANNING AND ASSESSING RISKS

(A) (B) (C) 
(D)
3. Which of the following is most accurate concerning DietWeb’s
audit committee? 

A. It should be considered very independent in that the company’s founders serve on it. 
B. Mr. Readings’ chairmanship of the audit committee creates a situation in which the audit
committee serves a strong independent role due to his closeness to company operations. 
C. Because   all   committee   members   are   members   of   management,   the   audit   committee   lacks
independence. 
D. The audit committee does not have enough members in that the size of the board of directors
should be less than that of the audit committee. 
(A) (
B
)
(
C
)
(
D

4. Which   of   the   following   indicates   an   increased   risk   of
misstatement due to fraud? 
A. Resignation of Mr. Williamson. 
B. Domination of management by company founders. 
C. Issuance of debt during 20X8. 
D. Competition with non­Internet weight loss organizations. 
(A) (B) (C) 
(D)
5.   Which of the following is the most significant risk facing DietWeb that might cause it to not be
able to continue increasing sales?
A. A decreasing market in the United States for dietary products.
B. DietWeb must respond to “dieting fads” on a timely basis.
C. The constantly decreasing number of individuals in the United States.
D. Obsolescence of the Internet.

Task­Based Simulation 5
Financial Statement
Analysis
Authoritative
Literature Help

DietWeb Inc. (hereafter DietWeb) was incorporated and began business in March of 20X1, seven 
years ago. You are working on the 20X8 audit—your CPA firm’s fifth audit of DietWeb. Analyze the 
following financial statements and reply to each of the questions that follow.
DietWeb, Inc.
BALANCE SHEET
December 31, 20X8 and 20X7
(in thousands)
20X8 20X7
Assets
Current assets
Cash and cash equivalents $3,032 $1,072
Trade receivables 485 450
Prepaid advertising expenses 59 609
Prepaid expenses and other current assets 175 230
Total current assets 3,751 2,361
Fixed assets, net 3,321 3,926
Total assets $7,072 $6,287

Liabilities and shareholders’ equity
Current liabilities
Accounts payable $1,070 $  909
Current maturities of notes payable 42 316
Deferred revenue 1,973 1,396
Other current liabilities 171 12
Total current liabilities 3,256 2,633
Long­term debt, less current maturity 34 176
Accrued liabilities 792 690
Deferred tax liability 15 145
Total liabilities 4,097 3,644
Shareholders’ equity
Common stock 6,040 4,854
Retained earnings (3,065) (2,211)
Total shareholders’ equity 2,975 2,643
Total liabilities plus shareholders’ equity $7,072 $6,287
MODULE 2   ENGAGEMENT PLANNING AND ASSESSING RISKS 39

DietWeb, Inc.
INCOME STATEMENT
Two Years Ended December 31, 20X8 and 20X7
(in thousands)
20X8 20X7
Revenue $19,166 $14,814
Costs and expenses
Cost of revenue 2,326 1,528
Product development 725 653
Sales and marketing 13,903 8,710
General and administrative 2,531 2,575
Depreciation and amortization 629 661
Impairment of intangible assets 35 ­
Total costs and expenses 20,149 14,127
Net income before taxes (983) 687
Income tax benefit 129 125
Net income (loss) $(854) $812

DietWeb, Inc.
STATEMENT OF CASH FLOWS
Year Ended December 31, 20X8
20X8 20X7
Cash flows from operations
Net income (loss) $(854) 812
Adjustments to net income
Depreciation 629 660
Increase in receivables (35) (47)
Decrease (Increase) in prepaid advertising 550 (650)
Decrease in other current assets 55 74
Increase (Decrease) in accounts payable 161 (540)
Increase in accrued liabilities 102 43
Increase (Decrease) in deferred revenue 432 (665)
Increase in common stock issued 1,186 ­
Increase in other current liabilities 159 43
Net cash provided (used) by operations 2,385 (270)
Cash flows from investing activities
Purchase of property and equipment (320) 2,016
Cash flows from financing activities
New debt 613 40
Debt payments (718) (918)
Net cash provided (used) by financing activities (105) (878)
Net increase in cash and cash equivalents $1,960 868
Cash and equivalents at beginning of year $1,072 204
Cash and equivalents at end of year $3,032 1,072

Required
(A) (B) (C) (D)
1.   The most likely misstatement in the financial statements is
A. The increase in cash in 20X8. 
B. Treatment of impaired intangible assets as an expense in 20X8. 
C. Treatment of common stock issued as an adjustment to net income (loss) under cash flow
from operations. 
D. An income tax benefit on the income statement as contrasted to income tax expense. 
(A) (B
)
(C
)
(D

2.   Which of the following is the most unexpected change on the balance sheet, if one assumes the
revenue increase in 20X8 is correct?
A. Decrease in prepaid advertising expenses.
B. Increase in accounts payable.
C. Decrease in deferred revenues.
D. Increase in common stock.
4
0
C. I
D. nM
r
4. W

A. C
B. l
C
C. lA
D. R
 
e
5. W

A. T
B. h
T
C. hA
D. n
A
n
(

Task­Based Simulation 6
Risk of Material
Misstatement Analysis Authoritative
Literature Help

You are working with William Bond, CPA, and you are considering the risk of material 
misstatement in planning the audit of Toxic Waste Disposal (TWD) Company’s financial statements 
for the year ended December 31, 20X0. TWD is a privately owned entity that contracts with municipal 
governments to remove environmental waste. Based only on the information below, indicate whether 
each of the following factors would most likely increase (I), decrease (D), or have no effect on the risk 
of material misstatement (N).
3. The
internal
1. Because municipalities have received increased federal and state funding auditor
for   environmental   purposes,   TWD  returned  to  profitability  for  the  first reports
year following three years with losses.  to   the
2. TWD’s Board of Directors is controlled by Mead, the majority stockholder, controll
who also acts as the chief executive officer.  er   and
the
controller reports to Mead. 
4. The   accounting   department   has   experienced   a   high   rate   of   turnover   of   key
personnel. 
5. TWD’s bank has a loan officer who meets regularly with TWD’s CEO and
controller to monitor TWD’s financial performance. 
6. TWD’s employees are paid biweekly. 
7. TWD has such a strong financial presence in its history so as to allow
it   often   to   dictate   the   terms   or   conditions   of   transactions   with   its
suppliers. 
8. During 20X1, TWD changed its method of preparing its financial statements
from the cash basis to generally accepted accounting principles. 
9. During   20X1,   TWD   sold   one­half   of   its   controlling   interest   in   United
Equipment Leasing (UEL) Co. TWD retained significant influence over UEL. 
10. During 20X1, litigation filed against TWD from an action ten years ago that
alleged that TWD discharged pollutants into state waterways was dropped by
the state. Loss contingency disclo­ 
sures that TWD included in prior years’ financial statements are being 
removed from the 20X1 financial statements.
11. During December 20X1, TWD signed a contract to lease disposal 
equipment from an entity owned by Mead’s parents. This related­party 
transaction is not disclosed in TWD’s notes to the 20X1 financial 
statements. 
(I) (D) (N)

MODULE 2   ENGAGEMENT PLANNING AND ASSESSING RISKS 41

(I) (D) (N)


12. During December 20X1, TWD completed a barter transaction with a 
municipality. TWD re­moved waste from the municipally owned site 
and acquired title to another contaminated site at below market price. 
TWD intends to service this new site in 20X2. 
13. During December 20X1, TWD increased its casualty insurance coverage on several pieces of
sophisticated machinery from historical cost to replacement cost.
14. Inquiries about the substantial increase in revenue TWD recorded in the fourth quarter of 20X1
disclosed a new policy.  TWD guaranteed to several municipalities that it would refund the
federal and state funding paid to TWD if any municipality fails a federal or state site clean­up
inspection in 20X2.
15. An initial public offering of TWD’s stock is planned for late 20X2.

Task­Based Simulation 7
Identifying
Risks
Authoritative
Literature Help

Planning
You are beginning the 20X8 audit of Toastco, a nonpublic company that manufactures various
kitchen products. This is your firm’s sixth annual audit of Toastco.
Toastco’s most profitable product is a deluxe toaster that is designed for those who want “a truly 
outstanding toaster.” The toaster was first marketed through Sky Mall Magazine, which offers 
specialty­type products to airplane passengers. Demand has grown so that at this point the company 
advertises the product much more broadly through a variety of media. Sales have been outstanding 
from the start and continue to increase. The recorded level of sales for 20X8 indicates that the 
company now con­trols approximately 5% of the toaster market, which represents 55% of company 
sales. In addition to the toaster, the company manufactures other products, including toaster ovens, 
coffee makers, food blenders, and electric can openers. Most of the manufacturing process is 
performed in several plants in Asia.
The highly successful toaster is the product of Bill Williams, who became Toastco’s chief 
executive officer in 20X3. Bill and several other officers invested heavily in the company at that time 
with the intent of ultimately selling common stock to the public. His and those individuals’ lives are 
centered about the firm and making it a success. Consistently, Toastco is of critical importance to 
each of these individuals financially.
Recently you discussed the coming audit with Bill. You quickly found that he was elated that the 
company had earned $1.21 per share, one cent more than he had assured the bank that had provided 
Toastco with extensive financing during 20X8. He suggested that some had feared that Toastco 
wouldn’t make it after somewhat weak second and third quarters, but that he knew all along that a 
strong fourth quarter was ahead and that it would “bring us through.” He also pointed out that sales are
up about 38% compared to the previous year and income by 54%. Furthermore, he indicated that an 
initial public offering of secu­rities was planned for approximately eighteen months from now.
Toastco’s outstanding performance for the past several years is in large part due to the highly 
profitable toaster, with other products gaining sales at approximately the industry’s growth rate. In 
fact, Bill indicated that the toaster has become even more profitable this year due to reengineering of 
its production process and components, which brought costs of production down and decreased sales 
returns and allowances.
Bill also suggested to you that the $42,216,000 bank loan received during the year has been 
primarily used to increase in­ventories and fixed assets to support the rapidly rising demand for the 
toaster, as well as Toastco’s other products. He pointed out that the company could never have 
achieved its projected earnings per share goal without the bank’s support. This bank support has also
allowed Toastco to work on developing what Bill refers to as “our next super product… an 
improved can opener that works well with all sizes of cans and doesn’t leave a mess.”
Finally, Bill said that there was some sad news. The Chairman of the Board of Directors, John 
Whing, an independent di­rector, had been forced to step down for health reasons. Bill had replaced 
Mr. Whing as Chairman of the Board, and Toastco Vice President Sam Adamson filled the empty seat 
on the board of directors. The seven­member board is now composed of Bill (who will begin serving 
as Chairman), Sam Adamson, two others from management, and the three independent directors. The 
audit committee has not been affected by the change, as the three independent directors continue to 
serve in that role.
Industry Information
In the portions of the kitchen product industry in which Toastco operates, demand is influenced 
by economic trends such as increases or decreases in consumer disposable income, availability of 
credit, and housing construction. Competition is very active in all products and comes from a number 
of principal manufacturers and suppliers. An important factor is the degree of product differentiation 
achieved through innovation and new product features. Other significant factors include product 
quality and cost, brand recognition, customer responsiveness, and appliance service capability.
Overall, for the industry, sales have been and are expected to remain relatively stable with a slight 
increase—increases have approximated 3% industry­wide per year during the past five years, and are 
expected to continue at that rate of increase for the
42 MODULE 2   ENGAGEMENT PLANNING AND ASSESSING RISKS

next five years. Much of the manufacturing of these products takes place in Asia, and to a lesser extent in South America. In­
creasingly, Asian companies are becoming directly involved in marketing their own household products.
Required
(A) (B) (C) (D)
1. Which of the following identifies an aspect of the company’s business model, strategies, and/or
business environment that is most likely to increase Toastco’s inherent risk? 
A. Manufacturing in Asia. 
B. An expected industry growth rate in sales of approximately 3%. 
C. Product obsolescence or loss of product differentiation advantages. 
D. Overstated accounts receivable due to internal control deficiencies. 
(A) (B) (C) (D)
2.  Which of the following identifies a situation most likely to increase the risk of misstatement arising
from fraudulent financial reporting?
A. The household products industry seems stable, and is not rapidly growing. 
B. Toastco reported a strong fourth quarter that brought it up to expectations for earnings per share. 
C. Toastco controls approximately 5% of the market for toasters. 
D. Toastco has one relatively profitable product, and a number of products that are not as profitable. 
(A) (B) (C) (D)

3. Which of the following identifies an aspect of the company’s business model, strategies, and/or 
business environment that is most likely to increase the risk of misstatement arising from fraudulent 
financial reporting? 
A. Toastco sells more than one type of product. 
B. The president’s wealth is based on the success of Toastco. 
C. The audit committee is composed entirely of independent directors. 
D. Toastco continues to hire your firm for its fifth year. 
Ratio Analysis

Below are two sets of ratios that were identified as significant in the current and prior years’ audits of Toastco. For each 
pair, compare the values of each ratio. Then select an audit finding that is consistent with these metrics. Each of the audit 
findings may be used once, more than once, or not at all. Ratios using balance sheet numbers are based on end of year 
balances.
Ratio 20X8 20X7
Gross margin percentage 0.154 0.166
Current ratio 2.619 3.688

Substantive procedure
A. Increases in costs of purchases were not completely passed on to customers through higher selling prices. 
B. Increases in trade receivables. 
C. Owners’ equity increased due to retention of profits. 
D. A larger percentage of sales occurred during the last month of 20X8, as compared to 20X7. 
E. Interest expense decreased during 20X8. 
F. The percentage tax included in the provision for income taxes for 20X8 was less than the percentage in 20X7. 
G. A significant amount of long­term debt became current. 
(A) (B) (C) (D) (E) (F) (G)
1. An audit finding most consistent with the change in the gross margin percentage
2. An audit finding most consistent with the change in the current ratio

Ratio 20X8 20X7


Inventory turnover 10.52 7.95
Return on equity 0.40 0.67

Audit findings
H. Increases in costs of purchases were not completely passed on to customers through higher selling prices. 
I. Increases in trade receivables. 
J. Owners’ equity increased due to retention of profits. 
K. A larger percentage of sales occurred during the last month of 20X8, as compared to 20X7. 
L. Interest expense decreased during 20X8. 
M. The percentage tax included in the provision for income taxes for 20X8 was less than the percentage in 20X7. 
MODULE 2   ENGAGEMENT PLANNING AND ASSESSING RISKS 43

(H) (I) (J) (K) (L) (M)


3. An audit finding consistent with the change in inventory turnover for Toastco
4. An audit finding consistent with the change in the return on equity

Task­Based Simulation 8
Observed Ratio
Changes Authoritative
Literature Help

Items 1 through 6 represent an auditor’s observed changes in certain financial statement ratios or amounts from the 
prioryear’s ratios or amounts. For each observed change, select the most likely explanation or explanations from List B. 
Select only the number of explanations as indicated. Answers on the list may be selected once, more than once, or not at all.
List B
A. Items shipped on consignment during the last month of the year were recorded as sales. 
B. A significant number of credit memos for returned merchandise that were issued during the last month of the year
were not recorded. 
C. Year­end purchases of inventory were overstated by incorrectly including items received in the first month of
the subsequent year. 
D. Year­end purchases of inventory were understated by incorrectly excluding items received before the year­end. 
E. A larger percentage of sales occurred during the last month of the year, as compared to the prior year. 
F. A smaller percentage of sales occurred during the last month of the year, as compared to the prior year. 
G. The same percentage of sales occurred during the last month of the year, as compared to the prior year. 
H. Sales increased at the same percentage as cost of goods sold, as compared to the prior year. 
I. Sales increased at a greater percentage than cost of goods sold increased, as compared to the prior year. 
J. Sales increased at a lower percentage than cost of goods sold increased, as compared to the prior year. 
K. Interest expense decreased, as compared to the prior year. 
L. The effective income tax rate increased, as compared to the prior year. 
M. The effective income tax rate decreased, as compared to the prior year. 
N. Short­term borrowing was refinanced on a long­term basis at the same interest rate. 
O. Short­term borrowing was refinanced on a long­term basis at lower interest rates. 
P. Short­term borrowing was refinanced on a long­term basis at higher interest rates. 
Auditor’s observed changes (A) (B) (C) (D) (E) (F) (G) (H) (I)  (J) (K) (L) (M) (N) (O) (P)
1. Inventory turnover increased substantially from
the prior year.  (Select 3 explanations)
2. Accounts receivable turnover decreased substan­
tially from the prior year.  (Select 3 explanations)
3. Allowance for doubtful accounts increased from
the prior year, but allowance for doubtful accounts
as a percentage of accounts receivable decreased
from the prior year.  (Select 3 explanations)
4. Long­term debt increased from the prior year, but 
interest expense increased a larger­than­
proportionate amount than long­term debt.  (Select
one explanation)
5. Operating income increased from the prior year
although the entity was less profitable than in the
prior year.  (Select two explanations)
6. Gross margin percentage was unchanged from the
prior year although gross margin increased from
the prior year.  (Select one explanation)
44 MODULE 2   ENGAGEMENT PLANNING AND ASSESSING RISKS

Task­Based Simulation 9
Assertions and Authoritative
Audit Procedures Literature Help

You are a staff auditor with Williams and Co. CPAs. Bill Jones, a new hire, has come to you with
questions concerning “assertions” and “audit procedures.” For 1. through 6. match each assertion with 
the statement that most closely approximates its meaning. Each statement may be used only once.
Statement
A. There is such an asset 
B. The company legally owns the assets 
C. All assets have been recorded 
D. Transactions are recorded in the correct accounting period 
E. Assets are recorded at proper amounts 
F. Assets are properly classified 
Assertion (A) (B) (C) (D) (E)  (F)
1. Completeness
2. Cutoff
3. Existence and occurrence
4. Presentation and disclosure
5. Rights and obligations
6. Valuation
Auditors perform audit procedures to obtain audit evidence that will allow them to draw 
reasonable conclusions as to whether the client’s financial statements follow generally accepted 
accounting principles. Match each audit procedure with its type. Each type of audit procedure is used, 
one of them twice.
Type of audit procedure
A. Analytical procedures 
B. Tests of controls 
C. Risk assessment procedures (other than analytical procedures) 
D. Test of details of account balances, transactions, or disclosures 
Audit procedures (A) (B) (C) (D)
1. Prepare a flowchart of internal control over sales.
2. Calculate the ratio of bad debt expense to credit sales.
3. Determine whether disbursements are properly approved.
4. Confirm accounts receivable.
5. Compare current financial information with comparable prior periods.

Task­Based Simulation 10
Audit Risk and
Its Components   Authoritative
Literature Help

You recently graduated from college and joined a CPA firm as a junior assistant. You have been 
assigned to audit Wiglo, Inc. At lunch one of Wiglo’s accountants said to you that he is having some 
trouble with basic audit concepts, and this bothers him since he thinks he may want to take the CPA 
exam at some point in the future. He has the following questions for you to which you should reply 
with a “yes” if it is correct, and a “no” if it is incorrect.
Yes

No
1. Am I right that the risk of material misstatement is composed of the three components of
audit risk? 
2. Is inherent risk the possibility of material misstatement before considering the client’s
internal control? 
3. In our company I think our control risk is lower this year than it was last year; does that mean that
there is an increase in the risk of material misstatement? 
M

4. I

5. A

6. A
m
7. D
o
8. D

T
Research:
Supervision
Authoritative
Literature Help

You graduated from college recently and joined a CPA firm as an assistant. You have been 
assigned to the audit of PJ Wholesale, Inc. The CEO of the nonpublic company cannot understand why
a supervisor from your firm needs to review your audit work, given that you have a formal accounting 
education and took auditing courses in college.
Title choices
A. AU
B.

PCAOB
C. AT
D. AR
E. ET
F. BL (A) (B) (C) (D) (E) (F) (G)
G. CS (H)
H. QC

1. Which title of the Professional Standards addresses this issue and
will be helpful in responding to the CEO? 
2. Enter the exact section and paragraph with helpful information. 

Task­Based Simulation 12
Research: Tone
at the Top
Authoritative
Literature Help
Bill Jensen, CPA, has just become partner of Wigg & Co. CPAs. He is concerned that the “tone at 
the top” of the firm could use some improvement in the messages it sends to employees concerning the
firm’s culture. He has asked you to find any professional standards that address the need for a CPA 
firm to have a proper tone at the top.
Title choices
A. AU
B.

PCAOB
C. AT
D. AR
E. ET
F. BL (A) (B) (C) (D) (E) (F) (G)
G. CS (H)
H. QC

1. Which title of the Professional Standards addresses this issue and
will be helpful in responding to the partner? 
2. Enter the exact section and paragraphs with helpful information. 
46 MODULE 2   ENGAGEMENT PLANNING AND ASSESSING RISKS

Task­Based Simulation 13
Effects on Audit Risk Components Authoritative
Literature Help

Bestwood Furniture, Inc., a private company that produces wood furniture, is undergoing a year 2 
audit. The situations in the table below describe changes Bestwood made during year 2 that may or 
may not contribute to audit risk. For each situation, select from the list provided the impact, if any, that
the situation has on a specific component of audit risk for the year 2 audit. A selection may be used 
once, more than once, or not at all.

I
m
A
.B
.C
.D
.E
.F
.G
.
S
i
1. D
u
2. I
n
3. E
a
Task­Based Simulation 14
Research
Authoritative
Literature Help

Quality Control Standards
Assume that you are employed by Wilson & Wilson CPAs. One of the partners has asked you to 
research the professional standards for the reference that identifies the requirements for documentation 
of a firm’s quality control policies and procedures.
Title choices
A. AU
B.

PCAOB
C. AT
D. AR
E. ET
F. BL
G. CS (A) (B) (C) (D) (E) (F) (G)
H. QC (H)

1. Which title of the Professional Standards addresses this issue and
will be helpful in responding to the partner? 
2. Enter the exact section and paragraphs with helpful information. 
MODULE 2 ENGAGEMENT PLANNING AND ASSESSING RISKS 47

MULTIPLE­CHOICE ANSWERS

1. a __ __ 21. b __ __ 41. c __ __ 61. c __ __ 81. b __ __ 101. c __ __


2. d __ __ 22. b __ __ 42. d __ __ 62. b __ __ 82. c __ __ 102. b __ __
3. c __ __ 23. c __ __ 43. a __ __ 63. a __ __ 83. a __ __ 103. b __ __
4. d __ __ 24. a __ __ 44. a __ __ 64. b __ __ 84. d __ __ 104. d __ __
5. a __ __ 25. b __ __ 45. a __ __ 65. b __ __ 85. d __ __ 105. b __ __
6. a __ __ 26. a __ __ 46. a __ __ 66. c __ __ 86. a __ __ 106. d __ __
7. d __ __ 27. b __ __ 47. b __ __ 67. b __ __ 87. d __ __ 107. a __ __
8. b __ __ 28. a __ __ 48. d __ __ 68. d __ __ 88. b __ __ 108. d __ __
9. b __ __ 29. d __ __ 49. b __ __ 69. a __ __ 89. a __ __ 109. c __ __
10. a __ __ 30. b __ __ 50. a __ __ 70. b __ __ 90. d __ __ 110. d __ __
11. c __ __ 31. d __ __ 51. a __ __ 71. b __ __ 91. c __ __ 111. a __ __
12. c __ __ 32. c __ __ 52. a __ __ 72. a __ __ 92. a __ __ 112. b __ __
13. b __ __ 33. a __ __ 53. c __ __ 73. c __ __ 93. d __ __ 113. c __ __
14. b __ __ 34. a __ __ 54. b __ __ 74. d __ __ 94. c __ __
15. b __ __ 35. c __ __ 55. d __ __ 75. c __ __ 95. b __ __
16. b __ __ 36. d __ __ 56. d __ __ 76. a __ __ 96. c __ __
17. d __ __ 37. d __ __ 57. d __ __ 77. c __ __ 97. a __ __
18. a __ __ 38. b __ __ 58. c __ __ 78. b __ __ 98. c __ __
19. c __ __ 39. d __ __ 59. b __ __ 79. c __ __ 99. b __ __ 1st:  __/113 = __%
20. b __ __ 40. c __ __ 60. a __ __ 80. a __ __ 100. b __ __ 2nd: __/113 = __%
MULTIPLE­CHOICE ANSWER EXPLANATIONS
compared to work performed by 
A. Overview practitioners of other pro­fessions. 
1. (a) The requirement is to identify a  Answer (c) is correct because 
conceptual difference between the  independence is absolutely required 
attestation standards and generally  for the performance of audits; other 
accepted auditing standards. Answer  pro­fessions do not in general require
(a) is correct because AT 101 states  such independence. An­swers (a), (b),
that the attestation standards do not  and (d) are incorrect because the 
apply to audits of historical financial  various pro­fessions require due 
statements. Answer (b) is in­correct  professional care and competence 
because an independent mental  and have a complex body of 
attitude is required for attestation  knowledge. 
engagements. Answer (c) is incorrect  4. (d) The requirement is to determine what is 
because an attest engagement may be  meant by the third general standard’s 
related to a business acquisi­tion  requirement of due care in the performance of 
study or a feasibility study. Answer  an audit. Answer (d) is correct because due care
(d) is incorrect because while there is  requires critical review at every level of 
no internal control fieldwork standard  supervision of the work done and the judgment 
under the attestation standards, both a  exercised by those assisting 
planning and an evi­dence standard of
fieldwork are included. 
2. (d) The requirement is to identify the 
reply which is not an attestation 
standard. Answer (d) is correct 
becausethe attestation standards do 
not include a requirement that a 
sufficient understanding of internal 
control be obtained to plan the 
engagement. There is no internal 
control standard because the concept 
of internal control may not be 
relevant to certain assertions on 
which a CPA may be engaged to 
report (e.g., aspects of information 
about computer soft­ware). Answers 
(a), (b), and (c) are all incorrect 
because standards exist for evidence, 
reporting on the assertion or subject 
matter, and proper planning. 
3. (c) The requirement is to identify the 
characteristic that is most likely to be 
unique to the audit work of CPAs as 
in the audit. Answer (a) is incorrect because the many matters the review may address in an 
due care standard does not directly address  effort to address fairness of presentation of the 
safeguards over access to assets and records.  financial statements.
Answer (b) is incorrect because due care does 
not relate to a limited review of employee fraud A.1. Financial Statement Assertions
and illegal acts. Answer (c) is incorrect because
6. (a) The requirement is to identify the categories
the first general standard addresses technical 
training and proficiency as an auditor. of financial statement assertions. Answer (a) is 
correct because the professional standards 
5. (a) The requirement is to identify the focus  establish financial statement assertions for 
of afinal wrap­up review performed by a second account balances, classes of transactions and 
partner who has not been involved in the audit.  disclosures. Answer (b) is incorrect because 
Answer (a) is correct be­cause this second or  financial statement assertions are established 
“cold” review aims at determining whether the  for disclosures. An­swer (c) is incorrect 
financial statements result in fair presentation in because financial statement assertions are 
conformity with GAAP and with whether  established for classes of transactions. Answer 
sufficient appro­priate evidence has been  (d) is incorrect because financial statement 
obtained. Answer (b) is incorrect because most  assertions are estab­lished for both classes of 
frequently fraud involving the client’s man­ transactions and disclosures. 
agement and its employees have not been 
7. (d) The requirement is to identify the
discovered and, even if they have been, the 
focus of the review is still on the fairness of  item  that is  not  a  financial statement
presentation of the financial statements. An­ assertion relating to account bal­ances.
swers (c) and (d) are incorrect because  Answer   (d)   is   correct   because
decisions on mate­riality and communications  valuation and alloca­tion is an account
with the audit committee are only two of the  balance   assertion,   not   valuation   and
com­
48 MODULE 2   ENGAGEMENT PLANNING AND ASSESSING RISKS
misstatement,   assuming   that   there   are   no
related controls.
petence. Answer (a) is incorrect because 
completeness is anassertion relating to account  10. (a) The requirement is to identify an effect 
balances. Answer (b) is incor­rect because  of a de­crease in the acceptable level of 
existence is an assertion relating to account  detection risk. Answer (a) is correct because as 
balances. Answer (c) is incorrect because rights the acceptable level of detection risk decreases, 
and obliga­tions is an assertion relating to  the assurance provided from substantive tests 
account balances. should increase. To gain this increased 
assurance the audi­tors may (1) change the 
A.2. Audit Risk nature of substantive tests to more effective 
procedures (e.g., use independent parties 
8. (b) The requirement is to determine a
outside the entity rather than those within the 
likely auditor reaction to a decreased 
entity), (2) change the timing of substantive 
acceptable level of detection risk. 
Answer (b) is correct because  tests (e.g., perform them at year­endrather than 
postponement of interim sub­stantive at an interim date), and (3) change the extent of
tests to year­end decreases detection  substantive tests (e.g., take a larger sample). 
risk by reduc­ing the risk for the  Answer (b) is incorrect because the assurance 
period subsequent to the  provided from substantive tests increases, it 
performance of those tests; other  does not decrease. Answers (c) and (d) are 
approaches to decreasing detection  incorrect because the acceptable level of 
risk include changing to more  detection risk is based largely on the assessed 
effective substantive tests and  levels of control risk and in­herent risk. 
increasing their extent. Answer (a) is Accordingly, any tests of controls will already 
incorrect because in­creased, not  have been performed.
reduced, substantive testing is 
required. An­swer (c) is incorrect 
because inherent risk must be 
consid­ered in planning, either by 
itself or in combination with control 
risk. Answer (d) is incorrect because 
tests of con­trols must be performed 
to reduce the assessed level of con­
trol risk. 
9. (b) The requirement is to identify the 
risk that an auditor will conclude, 
based on substantive tests, that a ma­ 
terial error does not exist in an account balance
when, in fact, such error does exist. Answer (b)
is correct because detection risk is the risk that 
the auditor will not detect a material 
misstatement that exists in an assertion. 
Detection risk may be viewed in terms of two 
components (1) the risk that analytical 
procedures and other relevant substantive tests 
would fail to detect misstatements equal to 
tolerable misstatement, and (2) the allowable 
risk of incorrect accep­tance for the substantive
tests of details. Answer (a) is in­correct 
because sampling risk arises from the 
possibility that, when a test of controls or a 
substantive test is restricted to a sample, the 
auditor’s conclusions may be different from the
conclusions he or she would reach if the tests 
were ap­plied in the same way to all items in 
the account balance or class of transactions. 
When related to substantive tests sam­pling 
risk is only a part of the risk that the auditor’s 
substan­tive tests will not detect a material 
misstatement. An­
swer (c) is incorrect because nonsampling risk
includes only those aspects  of audit risk that
are   not   due   to   sampling.   An­swer   (d)   is
incorrect   because   inherent   risk   is   the
susceptibil­ity   of   an   assertion   to   a   material
14. (b) The requirement is to determine 
11. (c) The requirement is to determine 
the relationship between control risk 
whether inher­ent risk, control risk, 
and detection risk. Inverse is correct 
and detection risk may be assessed 
because as control risk increases 
in nonquantitative terms. Answer (c)
(decreases) detection risk must 
is correct because all of these risks 
decrease (increase). 
may be assessed in either 
quantitative terms such as  A.3. Materiality
percentages, or nonquantitative 
terms such as a range from a  15. (b) The requirement is to identify the 
minimum to a maximum.  information that an auditor would 
most likely use in determining a pre­
12. (c) The requirement is to determine a manner  liminary judgment about materiality. 
in which inherent risk and control risk differ  Answer (b) is correct because many 
from detection risk. Answer (c) is correct  materiality measures relate to an 
because inherent risk and control risk exist  annual figure (e.g., net income, sales).
independently of the audit of the financial  Answer (a) is incorrect because the 
state­ments as functions of the client and its  preliminary judgment about 
environment, whereas detection risk relates to  materiality is a factor used in 
the auditor’s procedures and can be changed at  determining the anticipated sample 
his or her discretion. Answer (a) is incorrect  size, not the reverse as suggested by 
because inherent risk and control risk are  the reply. Answers (c) and (d) are 
functions of the client and its environment and  incorrect because materiality will not 
do not arise from misapplica­tion of auditing  normally be affected by the re­sults of
procedures. Answer (b) is incorrect because  the internal control questionnaire or 
inherent risk, control risk and detection risk  the contents of the management 
may each be assessed in either quantitative or  representation letters. 
nonquantitative terms. Answer (d) is incorrect 
because inherent risk and control risk are  16. (b) The requirement is to identify the
functions of the client and its environment,  statement that is not correct 
they cannot be changed at the auditor’s  concerning materiality. Answer (b) is
discretion. However, the assessed levels of  the proper reply because the auditor 
inherent and control risk (not addressed in this  considers materiality for planning 
question) may be affected by auditor decisions  purposes in terms of the smallest, not
relating to the cost of gathering evidence to  the largest, aggregate amount of 
substantiate assessed levels below the  misstatement that could be material 
maximum.  to any one of the financial 
13. (b) The requirement is to determine  statements. Answers (a), (c), 
the best way for an auditor to achieve and   (d)   all   represent  correct  statements   about
an overall audit risk level when the  materiality. 
audit evidence relating to control risk
indicates the need to increase its 
assessed level. Answer (b) is correct 
because a decrease in detection risk 
will allow the auditor achieve an 
overall audit risk level substantially 
the same as planned. Answer (a) is 
incorrect because a decrease in 
substantive testing will increase, not 
decrease, detection risk and thereby 
increase audit risk. Answer (c) is 
incorrect because an in­crease in 
inherent risk will also increase audit 
risk. An­swer (d) is incorrect because
there appears to be no justifi­cation 
for increasing materiality levels 
beyond those used in planning the 
audit. 
MODULE 2   ENGAGEMENT PLANNING AND ASSESSING RISKS 49
judgment will affect the anticipated sample size
for planned substantive tests, sample size does 
17. (d) The requirement is to identify the  not affect the materiality judgment. Answer (d) 
elements which underlie the  is incorrect because the assertions embodied in 
application of generally accepted  the financial statements remain the same from 
audit­ing standards, particularly the  one audit to another. 
standards of fieldwork and reporting. 
Answer (d) is correct because AU  20. (b) The requirement is to identify the 
150 states that materiality and  most likely effect of a decrease in the 
relative risk underlie the application  tolerable amount of misstatement 
of all the standards. Answer (a) is  (tolerable misstatement) in a class of 
incorrect because a considera­tion of  transactions. An­swer (b) is correct 
internal control is one of the field  because auditing standards state that 
standards, not an element underlying  decreasing the tolerable amount of 
the standards. Answer (b) is incorrect misstatement will require the auditor 
because the second fieldwork  to do one or more of the following: 
standard, on evidence, relates most  (1) perform auditing procedures 
directly to corroborating evidence.  closer to the balance sheet date 
Answer (c) is in­correct because  (answer [b]); (2) select a more 
while it is accurate that quality  effective auditing procedure; or (3) 
control stan­dards encompass the  increase the extent of a particular 
firm’s policies and procedures to pro­ auditing procedure. Answer (a) is 
vide reasonable assurance of  incorrect because in such a 
conforming with professional  circumstance substantive tests are 
standards, the standards are not  more likely to be performed at or 
related more directly to the fieldwork after the balance sheet date than prior 
and reporting standards than to the  to the balance sheet date. Answer (c) 
general group of generally accepted  is incorrect because decreasing the 
auditing standards.  tolerable amount of misstatement will
not necessarily lead to an increase in 
18. (a) The requirement is to determine  the assessed level of control risk. 
the appropriate level of materiality for Answer (d) is incorrect because the 
planning purposes when $10,000  extent of auditing procedures will be 
would have a material effect on an  increased, not decreased. 
entity’s income state­ment, but 
$20,000 would materially affect the 
balance sheet. AU 312 states that the 
audit should be designed to obtain 
reasonable assurance about whether 
the financial statements are free of 
material misstatement. Because it will
ordinarily be difficult to anticipate 
during the planning stage of an au­dit 
whether all misstatements will affect 
only one financial statement, the 
auditor is generally required to use 
the lower financial statement figure 
for most portions of planning. 
Therefore, answer (a), $10,000, is 
correct. Answers (b), (c), and (d) are 
all incorrect because they are dollar 
amounts which exceed the lowest 
level of materiality. 
19. (c) The requirement is to identify the 
information that an auditor would be most 
likely to use in making a pre­liminary judgment
about materiality. Answer (c) is correct because
auditors often choose to use a measure relating 
to the prior year’s financial statements (e.g., a 
percentage of total assets, net income, or 
revenue) to arrive at a prelimi­nary judgment 
about materiality. Answer (a) is incorrect 
because materiality is based on the magnitude 
of an omis­sion or misstatement and not on the 
initial assessment of control risk. Answer (b) is 
incorrect because while an audi­tor’s materiality
what presump­tion concerning 
21. (b) The requirement is to identify the  management’s honesty that 
necessary con­dition for an auditor to  professional skepticism requires. 
be able to issue an unqualified opin­ Answer (c) is correct because profes­
ion. Answer (b) is correct because if  sional skepticism requires that an 
the estimate of likely misstatement is  auditor neither assume dishonesty nor
equal to or greater than a material  unquestioned honesty. Answers (a) 
amount a material departure from  and (b) are incorrect because neither 
generally accepted accounting prin­ honesty in the absence of fraud risk 
ciples exists and thus AU 508 requires factor nor dishonesty are assumed. 
either a qualified or adverse opinion  Answer (d) is incor­rect because the 
in such circumstances. Answer (a) is  concept of reasonable assurance is not
incor­rect because the amount of  di­rected towards management’s 
known misstatement (if any) need not  honesty. 
be documented in the management 
representation letter. Answer (c) is  A.4. Errors and Fraud
incorrect because it ordinarily is not  24. (a) The requirement is to identify the example 
necessary for the client to  of fraudulent financial reporting. Answer (a) is 
acknowledge and record immate­rial  correct because fraudulent financial reporting 
known misstatements. Answer (d) is  involves intentional misstate­ments or 
incorrect because the total likely  omissions of amounts or disclosures in financial
misstatement need not include the  statements to deceive financial statement users 
adjusting entries already recorded by  and changing the inventory count tags results in
the client.  such a misstatement. An­swers (b), (c), and (d) 
22. (b) The requirement is to determine  are all incorrect because they repre­sent the 
which concept requires an attitude  misappropriation of assets. See AU 316 which 
that includes a questioning mind and a di­vides fraudulent activities into misstatement 
critical assessment of audit evidence.  arising from fraudulent financial reporting and 
Answer (b) is correct because AU 230 misstatements arising from misappropriation of 
states that professional skepticism  assets (sometimes referred to as defalcation). 
includes these qualities. Answer (a) is  25. (b) The requirement is to identify the 
incorrect because due profes­sional  best descrip­tion of what is meant by 
care is a broader concept that  a “fraud risk factor.” Answer (b) is 
concerns what the inde­pendent  correct because AU 316 suggests that 
auditor does and how well he or she  while fraud risk fac­tors do not 
does it. An­swer (c) is incorrect  necessarily indicate the existence of 
because reasonable assurance is based fraud, they often have been observed 
on the concept that an auditor is not  in circumstances where frauds have 
an insurer and his or her report does  occurred. Answer (a) is incorrect 
not provide absolute assurance.  because the risk of fraud may or may 
Answer (d) is incorrect because  not be high when a risk factor is 
supervision involves the directing of  present. Answer (c) is incorrect 
the efforts of assistants who are  because the current audit program 
involved in accomplishing the  may in many circumstances 
objectives of the audit and  appropriately address a fraud risk 
determining whether those objec­tives factor. Answer (d) is incorrect 
were accomplished.  because a fraud risk factor may or 
23. (c) The requirement is to determine  may not represent a material 
weakness. 
50 MODULE 2   ENGAGEMENT PLANNING AND ASSESSING RISKS
incorrect because they represent factors 
considered in the professional literature for 
26. (a) The requirement is to identify the  providing reasonable, and not absolute assur­
reply which represents an auditor  ance. 
communication responsibility relating
to fraud. Answer (a) is correct  30. (b) The requirement is to identify the type of 
because all fraud involving senior  fraudu­lent misstatement that is most difficult to
management should be reported  detect. Answer (b) is correct because 
directly to the audit committee.  transactions that have not been recorded are 
Answer (b) is incorrect because  generally considered most difficult because 
auditors are only required to report  there is no general starting point for the auditor 
fraud to the Securities and Exchange  in the consideration of the transaction. Answers 
Commission under particular  (a), (c), and (d) all represent re­corded 
circumstances. Answer (c) is incorrect transactions which, when audited, are in general
because auditors do not ordinarily  easier to detect. 
disclose fraud through use of an  31. (d) The requirement is to identify the least 
“emphasis of a matter” paragraph  likely indicator of a risk of possible 
added to their report. Answer (d) is  misstatement due to fraud. Answer (d) is correct
incorrect because under certain  because one would expect unusually aggressive,
circumstances auditors must disclose  rather than unusually conservative accounting 
fraud outside the en­tity.  practices to indicate a risk of misstatement due 
27. (b) The requirement is to identify the risk  to fraud. Answers (a), (b), and (c) are all 
relating to material misstatement that auditors  incorrect because they repre­sent risk factors 
are required to assess. Answer (b) is correct  explicitly included in AU 316, which pro­vides 
because SAS 109 and AU 316 require auditors  guidance on fraud. 
to specifically assess the risk of material 
misstate­ments due to fraud and consider that 
assessment in designing the audit procedures to 
be performed. Answer (a) is incor­rect because 
while SAS 109 also requires an assessment of 
the overall risk of material misstatement 
(whether caused by error or fraud) there is no 
requirement to explicitly assess the risk of 
material misstatement due to errors. Answer (c) 
is incorrect because the auditor need not 
explicitly assess the risk of misstatement due to 
illegal acts (see AU 317 for in­formation on 
illegal acts). Answer (d) is incorrect because no 
assessment of business risk is required. 
28. (a) The requirement is to determine 
whether audits are designed to 
provide reasonable assurance of 
detecting misstatements due to 
errors, fraudulent financial 
reporting, and/or misappropriation 
of assets. Answer (a) is correct 
because AU 110 and AU 316 
require that an audit obtain 
reasonable assurance that material 
misstatements, whether caused by 
error or fraud, be detected. 
Fraudulent financial reporting and 
the misappropriation of assets are 
the two major types of fraud with 
which an audit is relevant. 
29. (d) The requirement is to identify the reply 
which is not a reason why auditors are unable to
obtain absolute as­surance that misstatements 
due to fraud will be detected. Answer (d) is 
correct because while an auditor must exercise 
professional skepticism when performing an 
audit it does not represent a limitation that 
makes is impossible to obtain ab­solute 
assurance. Answers (a), (b), and (c) are all 
(d) are all incorrect because mistakes 
32. (c) The requirement is to determine  in the application of accounting 
the reply which represents  principles or in processing data, and 
information most likely to affect the  unreasonable accounting estimates 
auditor’s assessment of the risk of  arising from oversight are examples 
misstatement due to fraud. An­swer  of misstatements rather than fraud. 
(c) is correct because AU 316 states 
35. (c) The requirement is to identify the 
that missing documents may be 
indicative of fraud. Answer (a) is  characteristic most likely to heighten an 
incor­rect because checks for  auditor’s concern about the risk of intentional 
significant amounts are normally  manipulation of financial statements. Answer 
expected to be outstanding at year­ (c) is correct because the placement of 
end. Answer (b) is incor­rect because  substantial emphasis on meeting earnings 
almost all audits involve computer  projections is considered a risk factor. Answer 
generated documents and their  (a) is incorrect because high turnover, not low 
existence is not considered a  turn­over, is considered a risk factor. Answer 
condition indicating possible fraud.  (b) is incorrect because insider purchases of 
Answer (d) is incorrect because while  additional shares of stock are less likely to be 
last­minute adjustments that  indicative of intentional manipulation of the 
significantly affect finan­cial results  financial statements than is undue emphasis on 
may be considered indicative of  meeting earnings projections. Answer (d) is 
possible fraud, year­end adjusting  incorrect because a rapid rate of change in an 
journal entries alone are to be  industry, not a slow rate, is con­sidered a risk 
expected.  factor. 

33. (a) The requirement is to identify the  36. (d) The requirement is to identify an 


most likely response to the auditor’s  auditor’s responsibility for detecting 
assessment that the risk of material  errors and fraud. Answer (d) is 
misstatement due to fraud for the  correct because AU 110 requires that 
existence of inventory is high.  an auditor design the audit to provide 
Answer (a) is correct because  reasonable assurance of detecting 
observing test counts of inventory on  misstate­ments due to errors and 
an unannounced basis will provide  fraud that are material to the finan­
evidence as to whether record  cial statements. Answer (a) is 
inventory exists. Answer (b) is  incorrect because audits pro­vide 
incorrect because replacing test  reasonable assurance of detecting 
counts with analytical procedures is  material errors and fraud. Answer (b) 
not likely to be particularly effective.  is incorrect because it doesn’t restrict 
Answers (c) and (d) are incorrect  the responsibility to material errors 
because the inventories might well be  and fraud. Answer (c) is incorrect 
counted at year­end, all on the same  because it is less precise than answer 
date, rather than prior to year­end and (d), which includes the AU 110 
at differing dates.  responsibility on errors and fraud. 

34. (a) The requirement is to identify the  37. (d) The requirement is to identify the 


reply that is most likely to be an  circumstances in which an auditor 
example of fraud. Answer (a) is most may have a responsibility to disclose 
likely, since “defalcation” is another  fraud to parties other than a client’s 
term for misstatements arising from  senior management and its audit 
misappropriation of assets, a major  committee or board of directors. 
type of fraud. Answers (b), (c), and  Answer (d) is cor­ 
MODULE 2   ENGAGEMENT PLANNING AND ASSESSING RISKS 51
adverse relationships between a 
company and its employees. Answer 
rect because AU 316 states that such a  (d) is correct because AU 316 states 
responsibility may exist to the SEC when there that, while the auditor is not required 
has been an auditor change to a successor  to plan the audit to discover informa­
auditor or to comply with SEC 1995 Private  tion that is indicative of financial 
Se­curities Reform Act communication  stress of employees or adverse 
requirement, when the successor auditor makes relationships between the company 
inquiries, and to a government agency from  and its employ­ees, such conditions 
which the client receives financial assistance.  must be considered when an auditor 
In addition, that section states that an auditor  becomes aware of them. Answers (a), 
may have such a disclosure responsibility in  (b), and (c) are all incorrect because 
response to a subpoena, a cir­cumstance not  the auditor does not plan the audit to 
considered in this question. de­tect these conditions. 
38. (b) Errors refer to unintentional  43. (a) The requirement is to identify an 
mistakes in finan­cial statements  auditor’s responsibility when he or 
such as misinterpretation of facts.  she believes that a misstatement is or 
An­swers (a), (c), and (d) all  may be the result of fraud, but that 
represent fraud which are defined as the effect of the mis­statements is 
intentional distortions of financial  immaterial to the financial 
statements.  statements. An­swer (a) is correct 
39. (d) The requirement is to identify the  because AU 316 states that in such 
level of assur­ance an auditor  cir­cumstances the auditor should 
provides with respect to detection of  evaluate the implications of the fraud,
mate­rial errors, fraud, and direct  especially those dealing with the 
effect illegal acts. Answer (d) is  organizational position of the 
correct because AU 110 requires the  person(s) involved. 
auditor to design the audit to provide 
reasonable assurance of detecting 
material errors, fraud and direct effect
illegal acts. (A “direct effect” illegal 
act is one that would have an effect 
on the determina­tion of financial 
statement amounts.) 
40. (c) The requirement is to identify the proper 
attitude of an auditor who is performing an 
audit in accordance with generally accepted 
auditing standards. Answer (c) is correct 
because the auditor should plan and perform the
audit with an attitude of professional 
skepticism, recognizing that the application of 
the auditing procedures may produce evidence 
indicating the possibility of misstatements due 
to errors or fraud. Answer (a) is incorrect 
because while the CPA must exhibit objective 
judgment, “professional skepticism” more 
accurately summarizes the proper attitude 
during an audit. Answer (b) is incorrect because
while a CPA must be independent and have 
integrity, this is not the “attitude” used to plan 
and perform the audit. Answer (d) is incorrect 
because the audit is not planned and performed 
with impartial conservatism. 
41. (c) The requirement is to identify the meaning 
of the term “material misstatement” when used 
in the professional standards. Answer (c) is 
correct because SAS 107 and 
AU 316 state that a material misstatement may
occur due to errors, fraud, and illegal acts with
a direct effect on financial statement amounts. 
42. (d) The requirement is to identify an 
auditor’s responsibility for detecting 
financial stress of employees or 
likely that management’s attitude 
44. (a) The requirement is to identify the toward aggressive financial reporting 
correct state­ments relating to the  and toward meeting projected profit 
auditor’s consideration of fraud. An­ goals would most likely significantly 
swer (a) is correct because AU 316  influence an entity’s control 
states that the auditor’s interest  environment. Answer (b) is correct 
relates to fraudulent acts that cause a because when management is 
material mis­statement of financial  dominated by one individual, that in­
statements. Answer (b) is incorrect  dividual may be able to follow overly 
because errors are unintentional.  aggressive accounting principles. 
Answer (c) is incorrect because 
48. (d) The requirement is to identify the 
fraud does not necessarily involve 
procedure least likely to be required 
the misappro­priation of assets (it 
may involve fraudulent financial re­ on an audit. Answer (d) is 
porting). Answer (d) is incorrect  correctbecause fraud is a broad legal 
because an auditor must design an  concept and auditors do not make 
audit to obtain reasonable assurance  legal determinations of whether fraud 
of detecting misstatements,  has occurred. Answers (a), (b), and 
regardless of whether they are  (c) are incorrect because considering 
caused by errors or fraud.  journal entries, estimates, and unusual
transactions are ordi­narily required 
45. (a) The requirement is to identify the factor or  audit procedures to address the risk of
condition that an audit is least likely to be  man­agement override of controls. 
planned to dis­cover. Answer (a) is correct  See AU 316 for information on the 
because it represents a financial stress, and  auditor’s responsibility for the 
auditors are not required to plan audits to dis­ consideration of fraud in a financial 
cover information that is indicative of financial  statement audit. 
stress of employees or adverse relationships  49. (b) The requirement is to identify the most 
between the entity and its employees. Answers 
likely response when a risk of fraud has been 
(b), (c), and (d) are all incorrect because they 
identified on an au­dit. Answer (b) is correct 
represent examples of risk factors that should 
because AU 316 indicates that overall 
be considered in an audit and are included in 
responses to the risk of material misstatements 
AU 316. 
due to fraud include (1) assigning personnel 
46. (a) The requirement is to determine  with particular skills relating to the area and 
when audit risk factors may be  considering the necessary extent of supervision 
identified. Answer (a) is correct  to the audit, (2) increasing the consideration of 
because AU 316 states that fraud risk  management’s selection and application of 
factors may be identified dur­ing  accounting prin­ciples, and (3) making audit 
planning, obtaining an understanding, procedures less predictable. Answer (a) is 
or while conduct­ing fieldwork; in  incorrect because closer supervision, not less 
addition, they may be identified while close supervision, is more likely to be 
con­sidering acceptance or  appropriate. An­swer (c) is incorrect because 
continuance of clients and  individuals with specialized skills may be 
engagements.  needed who are not CPAs. Answer (d) is 
incorrect because subjective transactions (e.g., 
47. (b) The requirement is to identify the  accounting estimates) often provide more risk 
circumstance in which it is most  than objective transactions. 
52 MODULE 2   ENGAGEMENT PLANNING AND ASSESSING RISKS
fraudulent financial re­porting. 

50. (a) The requirement is to identify  54. (b) The requirement is to identify the


the least likely inquiry of  most likely fraud risk factor on an 
management relating to identifying  audit. Answer (b) is correct because 
the risk of material misstatement  the possibility of improper revenue 
due to fraud. Answer (a) is correct  recognition is ordinarily presumed on
because financial operations of  audits. Answers (a), (c), and (d) all 
many companies are not ordinarily  represent potential risks, but risks 
controlled by and limited to one  that are not ordinarily presumed on 
location. An­swers (b), (c), and (d)  an audit. See AU 316 for information
are all incorrect because they are in­ on the auditor’s re­sponsibility for 
cluded in AU 316 as inquiries that  the consideration of fraud in a 
should be made of man­agement.  financial statement audit. 

51. (a) The requirement is to identify the  55. (d) The requirement is to identify the 


attributes ordi­narily present when  circumstances relating to the discovery of the 
individuals commit fraud. Answer (a)  payment of small bribes to municipal officials 
is correct because AU 316 suggests  that is most likely to cause an auditor to 
that the three conditions generally  withdraw from an engagement. Answer (d) is 
present when fraud occurs are that  correct be­cause AU 317 states that 
individuals have an (1) incentive or  management failure to take the appropriate 
pressure, (2) opportunity, and (3)  remedial action is particularly problematical 
ability to rationalize. Answers (b), (c), since it may affect the auditor’s ability to rely 
and (d) are all incor­rect because they  on manage­ment representation and may 
suggest that one of the three elements  therefore lead to withdrawal. Answers (a), (b), 
is not ordinarily present.  and (c) all represent circumstances which the 
auditor will consider, but are not ordinarily 
52. (a) The requirement is to determine  considered as serious as failure to take the 
an auditor’s reporting responsibility  appropriate remedial action. 
when he or she has discovered that 
management is involved in a 
financially immaterial fraud. Answer 
(a) is correct because AU 316 
requires that all man­agement fraud, 
regardless of materiality, be reported 
to the audit committee. Answer (b) is 
incorrect because fraud is not directly
reported to the Public Company 
Accounting Oversight Board. Answer
(c) is incorrect because if anything, in
addition to the audit committee, the 
fraud is reported to a level of 
management at least one level above 
those involved in a fraud. Answer (d) 
is incorrect because there is a 
reporting responsibility for 
financially immaterial management 
fraud. 
53. (c) The requirement is to identify the 
most likely risk factor relating to 
fraudulent financial reporting. An­
swer (c) is correct because negative 
cash flows from opera­tions may 
result in pressure upon management 
to overstate the results of operations. 
Answer (a) is incorrect because one 
would expect a company’s top 
executives to dominate management
—domination by one or a few might 
be consid­ered a risk factor. Answers 
(b) and (d) are incorrect because large
amounts of cash being processed and 
small high­dollar inventory items are 
more directly related to the misappro­
priation of assets than they are to 
discovered that a client’s employees 
56. (d) The requirement is to identify the factor most  have paid small bribes to public 
likely to cause a CPA not to accept a new audit  officials. An­swer (c) is correct 
engagement. Answer (d) is correct because a part  because AU 317 states that 
of the understanding an auditor must obtain with a resignation should be considered 
client is that management is re­sponsible for  when an illegal act does not receive 
making all financial records and related infor­ proper remedial action, because such 
mation available (see SAS 108). Accordingly, if  inaction may affect the auditor’s 
the client refuses to make such information  ability to rely on management 
representations and the effects of 
available the auditor is unlikely to accept the audit
continued association with the client. 
client. Answer (a) is incorrect because a 
An­swer (a) is incorrect because the 
circumstance­imposed scope limitations such as 
receipt of federal funds in such a 
completion of the physical inventory count results 
situation is not as likely to result in 
in a situation in which the auditor may consider  auditor with­drawal as is answer (c). 
using alternative procedures (including making  Answer (b) is incorrect because it 
some test counts) to deter­mine whether inventory  seems inconsistent with the premise 
counts are proper. Answer (b) is incorrect because  of the question in that, if no evidence 
an auditor may obtain an understanding of the  exists, the auditor is unlikely to know 
client’s operations and industry while performing  that bribes have been paid. Answer 
the audit. Answer (c) is incorrect because while a  (d) is incorrect because such small 
review of the predecessor auditor’s working  bribes will not ordinarily need to be 
papers is ordinarily desirable, it is not required.  disclosed. Alternatively, if the auditor 
believes that there is such a need, the 
lack of such disclosure represents a 
57. (d) The requirement is to identify the factor 
departure from generally accepted 
most likely to heighten an auditor’s concern  accounting principles and either a 
about the risk of fraudulent financial reporting.  quali­fied or adverse opinion is 
Answer (d) is correct because AU 316, which  appropriate. 
presents a variety of risk factors, suggests that 
an overly complex organizational structure is  59. (b) The requirement is to identify the 
such a risk factor. Answer (a) is incorrect  illegal act that an audit should be 
because large amounts of liquid assets that are  designed to obtain reasonable assurance
easily convertible into cash represent more of a  of detecting. Answer (b) is correct 
risk relating to misappropriation of assets rather because the accrual and billing of an 
than to fraudulent financial reporting. Answer  improper amount of revenue under 
(b) is incor­rect because high growth, rather  government contracts is an illegal act 
than low growth, is consid­ered a risk factor.  with a direct effect on the determi­
Answer (c) is incorrect because one would  nation of financial statement amounts, 
expect financial management’s participation in  and audits are de­signed to detect such 
the initial selection of accounting principles.  illegal acts. Answers (a), (c), and (d) 
A.5. Illegal Acts are all incorrect because they represent 
illegal acts with an indirect financial 
58. (c) The requirement is to identify the  statement effect and an audit provides 
situation in which an auditor would be no assurance that such acts will be 
most likely to withdraw from an  detected or that any contin­ 
engagement when he or she has 
MODULE 2   ENGAGEMENT PLANNING AND ASSESSING RISKS 53
terial, but indirect effect on the 
financial statements. An­swer (a) is 
gent liabilities that may result will be disclosed. correct because AU 317 requires the 
See auditor to apply audit procedures 
AU 317 for detailed guidance on auditor  specifically designed to determine 
responsibility with respect to direct and  whether an illegal act has occurred 
indirect illegal acts. when such information comes to 
his/her attention. Answers (b), (c), 
60. (a) The requirement is to identify the  and (d) are all incorrect because they
small misstatement that is most likely represent procedures the auditor 
to have a material effect on an  would perform after initial 
entity’s financial statements. Answer  procedures had confirmed the 
(a) is correct be­cause an illegal  existence of the possible illegal 
payment of an otherwise immaterial  act(s). 
amount may be material if there is a 
reasonable possibility that it may lead 64. (b) The requirement is to determine 
to a material contingent liability or a  the circum­stance in which it is most 
material loss of revenue.  likely that a CPA would withdraw 
from an audit engagement after 
61. (c) The requirement is to determine what an  having discovered that client 
auditor might reconsider when a client’s board  employees have committed an illegal 
of directors has re­fused to take any action  act. Answer (b) is correct because the 
relating to an auditor’s disclosure that the  auditor may conclude that withdrawal 
company has made immaterial illegal  is necessary when the client does not 
contributions. Answer (c) is correct because in  take the remedial action, even when 
such a circumstance the failure to take remedial  the illegal act is not material to the 
action may cause an auditor to de­crease  financial statements. Answers (a) and 
reliance on management representations.  (c) are incorrect because whether 
Answer (a) is incorrect because the reply by the  generally accepted accounting 
attorney is likely to disclose any claims,  principles have been violated and 
litigation or assessments that the client has  whether the illegal act occurred 
improperly omitted from the letter of audit  during a prior year that was not 
inquiry. An­swer (b) is incorrect because the  audited may or may not have an effect
prior years’ audit programs are not being relied  on 
upon for this year’s audit. Answer (d) is 
incorrect because the preliminary judgment 
about materiality levels would not be expected 
to change. 
62. (b) The requirement is to identify a 
reason why au­dits cannot reasonably
be expected to bring all illegal acts to 
the auditor’s attention. Answer (b) is 
correct because illegal acts relating to
the operating aspects of an entity are 
often highly specialized and complex 
and often are far removed from the 
events and transactions reflected in 
financial state­ments. Answer (a) is 
partially correct since management 
override represents a limitation of the
effectiveness of inter­nal control. Yet,
auditors are more likely to identify 
such transactions because they relate 
to events and transactions reflected in
the financial statements. Answer (c) 
is incorrect because many illegal acts 
are not subject to the client’s in­ternal
control. Answer (d) is incorrect 
because illegal acts may be 
perpetrated without access to both 
assets and ac­counting records. 
63. (a) The requirement is to determine 
an auditor’s responsibility when 
information comes to his/her 
attention that implies the existence 
of possible illegal acts with a ma­
reporting, and management’s disregard in this 
the decision to withdraw from the engagement.  area may lead the CPA to reject the 
Answer (d) is incorrect because the assessed  engagement. Answer (a) is incorrect both be­
level of control risk will not have a direct  cause a CPA may not need an understanding of 
relationship on the decision to withdraw from  the prospec­tive client’s internal auditor’s 
the engagement. computer­assisted audit tech­nique to form an 
opinion on the financial statements, and 
65. (b) The requirement is to identify a CPA’s  because if such understanding is necessary, it 
responsi­bility under the Securities Litigation  can be ob­tained subsequent to engagement 
Reform Act of 1995 for uncorrected illegal acts  acceptance. Answer (c) is incorrect because AU
which have been communicated to the board of  334 indicates that a CPA often will be unable to
directors which refuses to inform the SEC of  determine whether related­party transactions 
their existence. Answer (b) is correct because  were consummated on terms equivalent to 
CPAs are required under the law to deliver a  arm’s­length transac­tions. Answer (d) is 
report on those illegal acts to the SEC within  incorrect because while management’s refusal 
one business day in such circum­stances.  to permit the performance of substantive tests 
Answer (a) in incorrect because there is no re­ before the year­end may present a problem, the 
quirement to resign, although the auditor may  auditor may be able to effectively perform such 
decide to do so. Answer (c) is incorrect because tests after year­end. 
the Act sets up reporting to the SEC, not to the 
stockholders. Answer (d) is incorrect because  B.1. Communicate with Predecessor 
withholding of the audit opinion is not  Auditors (Prior to Engagement 
suggested in the Act. Acceptance)
B. Audit Planning  68. (d) The requirement is to identify a 
66. (c) The requirement is to identify the procedure  requirement prior to accepting an 
least likely to be considered an audit planning  engagement to audit a new client. 
procedure.Answer (c) is correct because  An­swer (d) is correct because AU 
performing the risk assess­ment occurs  315 requires that an auditor attempt 
subsequent to audit planning. Answer (a) is  to obtain client permission to contact 
incorrect because an engagement letter is used  the predeces­sor prior to accepting a 
to establish an understanding with the client,  new engagement. Answers (a), (b), 
and this is a planning pro­cedure. Answer (b) is  and (c) are incorrect because they 
incorrect because auditors develop the overall  may all be obtained sub­sequent to 
audit strategy during audit planning. Answer (d) accepting an engagement. 
is incorrect because the audit plan is developed  69. (a) The requirement is to determine the nature 
during plan­ning.  of the inquiries that a successor auditor should 
67. (b) The requirement is to identify the factor  make of the prede­cessor auditor prior to 
accepting an audit engagement. An­swer (a) is 
most likely to cause a CPA to decide not to 
correct because the inquiries should include 
accept a new audit engagement. Answer (b) is 
spe­cific questions to management on (1) 
correct because a certain level of internal 
disagreements with 
control is essential for financial statement 
54 MODULE 2   ENGAGEMENT PLANNING AND ASSESSING RISKS
incorrect because management 
liability (if any) for illegal acts 
management as to auditing procedures and  committed by employees is not gener­
accounting prin­ciples (reply [a]), (2) facts that  ally included in an engagement letter. 
might bear on the integrity of management and  Answer (d) is incor­rect because 
(3) the predecessor’s understanding as to the  while an auditor is required to obtain 
reasons for the change of auditors. Answers  an under­standing of internal control, 
(b), (c), and (d) are incorrect because, if made  he or she is not required to search for 
at all, they will be after the engagement has  significant internal control 
been accepted. deficiencies. 
70. (b) The requirement is to identify the  73. (c) The requirement is to identify the 
correct state­ment regarding a successor  most likely procedure during the 
auditor’s inquiries of the prede­cessor auditor.  initial planning phase of an audit. An­
Answer (b) is correct because the successor  swer (c) is correct because during 
should request information such as (1) facts  initial planning the timing of 
that might bear on the integrity of  procedures will be discussed due to 
management, (2) disagreements with  the need for client assistance with 
management as to accounting principles,  many of these procedures. Answer 
auditing proce­dures, or other significant  (a) is incorrect because the timing of 
matters, and (3) the predecessor’s  audit procedures will occur 
understanding of the reasons for the change of  subsequent to the initial planning 
auditors. Answers (a), (c), and (d) all relate to  stage of an audit. An­swer (b) is 
matters not required to be discussed prior to  incorrect because the evaluation of 
accepting an audit engagement. reasonableness of the client’s 
accounting estimates will occur after 
B.2. Establishing an Understanding with
planning (see AU 342). Answer (d) is
the Client (Engagement Letters) incorrect because the inquiry of a 
71. (b) The requirement is to identify the item  client’s attorney will occur 
ordinarily included when an auditor establishes 
an understanding with a client regarding the 
services to be performed. Answer (b) is correct 
because auditing standards require that an 
auditor ensure that the audit committee is aware
of any significant deficiencies which come to 
the CPA’s attention. Answer (a) is incorrect 
because while an understanding will include a 
statement that management is responsible for 
the entity’s financial statements, an explicit 
statement about errors and illegal activities of 
employees is not ordinarily included. Answer 
(c) in incorrect because management does not 
pro­vide the auditor with an assessment of the 
risk of material misstatement due to fraud. 
Answer (d) is incorrect because no such 
statement about an auditor’s responsibility for 
de­termining preliminary judgments about 
materiality and audit risk factors is ordinarily 
included in establishing an under­standing. See 
SAS 108 for information on establishing an 
understanding with a client. 
72. (a) The requirement is to identify the 
matter gener­ally included in an 
auditor’s engagement letter. Answer 
(a) is correct because AU 310, which 
outlines requirements for engagement 
letters, indicates that an engagement 
letter should include an indication that
management is responsible for 
identifying and ensuring that the 
company complies with the laws and 
regulations applicable to its activities. 
An­swer (b) is incorrect because such 
detailed information on materiality is 
not generally included in an 
engagement letter. Answer (c) is 
swer (c) is correct because AU 311, 
subsequently to initial planning (see AU 337).  which provides infor­mation on 
See SAS 108 for information on planning. obtaining an understanding with the 
client, does not suggest any 
74. (d) The requirement is to identify the 
arrangement concerning CPA 
statement thatis least likely to appear in an  investment in client securities; indeed
auditor’s engagement letter. Answer (d) is  such investments are prohibited by 
correct because auditors ordinarily will not  the Code of Professional Conduct. 
discuss with management the details of  Answers (a), (b), and (d) all represent 
procedures that are necessary to perform the  arrangements which AU 311 suggests
audit. Answers (a), (b), and (c) are incorrect  may be included in an engagement 
because engagement letters will include a state­ letter (or other form of under­standing
ment on the risk of not detecting material  with a client). 
errors and fraud, and may include information 
on fees and observed opportu­nities for  B.3. Preliminary Engagement Activities
economy. 78. (b) The requirement is to identify form(s) of 
75. (c) The requirement is to identify the item for  docu­mentation of the understanding obtained 
which the generally accepted auditing standards with a client. An­swer (b) is correct because the
do not require documentation. Answer (c) is  professional standards require that an auditor 
correct because while a CPA firm will include  document the understanding through a writ­ten 
an engagement letter in the working papers, it  communication with the client. Answer (a) is 
will not detail the auditor’s planned fieldwork.  incorrect because an oral communication is not 
Answer (a) is incorrect because SAS 108  sufficient. Answer (c) is incorrect because the 
requires a written audit plan. Answer (b) is  communication should be in writing. Answer 
incorrect because SAS 103 requires that the  (d) is incorrect because the understanding is not
working papers document the agreement or  documented in a completely detailed audit plan
reconciliation of the accounting records with  —a term of questionable meaning. 
the financial statements. Answer (d) is incorrect 79. (c) The requirement is to identify the 
because SAS 109 requires the auditor to  factor that most likely would 
document the assessment of the risks of  influence an auditor’s determination 
material misstatement.  of the auditability of an entity’s 
76. (a) The requirement is to determine  financial statements. An­swer (c) is 
what types of items are ordinarily  correct because inadequate 
included in an engagement letter in  accounting records may cause an 
ad­dition to the objectives of the  auditor to conclude that it is unlikely 
engagement. Answer (a) is correct  that suf­ficient appropriate evidence 
because SAS 108 also requires  will be available to support an 
inclusion of informa­tion on CPA  opinion on the financial statements; 
responsibilities, client  accordingly, an auditor may 
responsibilities, and limitations of the determine that the financial 
engagement.  statements are not audit­able. Answer
(a) is incorrect because an auditor 
77. (c) The requirement is to determine  should be able to obtain the 
the reply which is least likely to be  knowledge necessary to audit a 
included in an engagement letter. An­ complex 
MODULE 2   ENGAGEMENT PLANNING AND ASSESSING RISKS 55
B.5. The Audit Plan

accounting system. Answer (b) is incorrect  82. (c) The requirement is to determine why an 


because while related­party transactions may  auditor should design a written audit program. 
raise transaction valuation issues due to the  Answer (c) is correct because an audit program 
lack of an “arm’s­length transaction,” the  sets forth in detail the audit proce­dures that are 
problem is normally not so severe as to make  necessary to accomplish the objectives of the 
the entity not auditable. Answer (d) is  audit. Answer (a) is incorrect because audit 
incorrect because a lack of op­erating  programs ad­dress topics beyond selecting 
effectiveness of controls may often be  material transactions and this is not their primary 
overcome through an increase in the scope of  focus. Answer (b) is incorrect because a program 
substantive tests. may include numerous substantive tests to be per­
B.4. Developing an Overall Strategy formed prior to the balance sheet date. Answer (d) 
is incor­rect because immaterial accounts often are
80. (a) The requirement is to identify the  not tested and because tests of transactions, tests 
area that is most likely to require  of balances, and analytical procedures are used to 
special audit planning considerations. test account balances; account bal­ances are not 
Answer (a) is correct because an  directly tested through tests of controls. 
inventory comprised of diamond 
rings is likely to require that the  83. (a) The requirement is to determine a 
auditor plan ahead to involve a  point at which an audit program may 
specialist to assist in valuation issues. be finalized. Answer (a) is correct 
An­swer (b) is incorrect because  because the consideration of internal 
valuation of an asset such as a new  control helps the audi­tor to assess 
copy machine is not ordinarily  control risk and to plan the audit: 
expected to provide valuation  accordingly, the audit program is not 
difficulties. Answer (c) is incorrect  generally finalized prior to the con­
because the expensing purchases of  sideration of internal control. Answer 
such small assets is ordinarily ac­ (b) is incorrect be­cause, while 
ceptable due to the immateriality of  generally desirable, engagement 
the transactions. An­swer (d) is  letters are not required on audits. 
incorrect because accelerated  Answer (c) is incorrect because 
depreciation meth­ods are ordinarily  report­ 
acceptable. 
81. (b) The requirement is to identify the 
information source that a CPA would 
most likely consult for information on 
how various publicly held companies 
have complied with the disclosure 
requirements of a new financial 
accounting standard. Answer (b) is 
correct because AICPA 
AccountingTrends and Techniques, 
which is issued annually, summa­rizes 
such disclosures of 600 industrial and 
merchandising corporations. Answer (a)
is incorrect because the AICPA 
Codification of Statements on Auditing 
Standards codifies the various 
Statements on Auditing Standards and 
does not include information on 
individual company compliance with 
disclosure requirements. Answer (c) is 
incorrect because Quality Control 
Review standards are established by the 
AICPA and because they do not include 
information on in­dividual company 
compliance with disclosure 
requirements. Answer (d) is incorrect 
because Form 10­K itself provides 
information on preparing Form10­K and
this form does not include information 
on individual company compliance with
disclosure requirements. 
acceptable for a continuing client. 
able conditions may be communicated at various  Also, when a new client has engaged 
times sub­sequent to finalization of the audit  an auditor prior to year­end, a portion
program. Answer (d) is incorrect because audit  of the audit may be conducted prior to
programs are often finalized prior to the  year­end. Answer (b) is incorrect 
performance of the search for unrecorded  because an engagement may be 
liabilities. accepted after the client’s year­end. 
Answer (c) is incorrect because 
84. (d) The requirement is to determine the manner  alterna­tive procedures may be 
in which audit programs should be designed.  possible when an inventory count was
Answer (d) is correct because an audit program  not observed at year­end. Answer (d) 
should be designed so that the audit evidence  is incorrect be­cause final staffing 
gathered is sufficient to support the audi­tor’s  decisions need not be made prior to 
conclusions. Answer (a) is incorrect because,  completion of the planning stage of 
often, most audit procedures will not be  audits. 
performed as interim work. Answer (b) is 
incorrect because inherent risk need not be  87. (d) The requirement is to identify the element 
assessed at a low level. Answer (c) is incorrect  of the audit planning process most likely to be 
because while providing constructive  agreed upon with the client before 
suggestions to management is desirable, the  implementation of the audit strategy. An­swer 
audit program is not based on developing con­ (d) is correct because the auditor will ordinarily
structive suggestions.  ob­serve the counting of inventory and this will
require a degree of coordination between the 
85. (d) The requirement is to determine  performance of audit proce­dures and client 
what specific audit objectives are  count procedures. Answer (a) is incorrect 
addressed when designing an audit  because the client will not determine the 
pro­gram. Answer (d) is correct  evidence to be gathered to provide a sufficient 
because in obtaining evidence in  basis for the auditor’s opin­ion. Answers (b) 
support of financial statement  and (c) are incorrect because these proce­dures 
assertions, the auditor de­velops  will be determined subsequent to 
specific audit objectives in the light of implementation of the audit strategy. 
those asser­tions. Answers (a), (b), 
and (c) are all incorrect because these  C. Obtain   an   Understanding   of
replies do not relate specifically to the the   Entity   and   Its
audit objectives as do the financial  Environment 
statement assertions. 
88. (b) The requirement is to determine the 
B.7. Timing of Audit Procedures manner inwhich an auditor plans an audit of a 
continuing client. An­swer (b) is correct 
86. (a) The requirement is to identify the  because a review of prior year working papers 
statement that is always true with  and the permanent file may provide useful 
respect to planning an audit. Answer  informa­tion about the nature of the business, 
(a) is correct because it is acceptable  organizational struc­ture, operating 
for an auditor to perform a certain  characteristics, and transactions that may 
portion of the audit at an interim date; require special attention. Answer (a) is 
for example, performing a portion of  incorrect because tests of details of transactions
planning prior to year­end is always  and balances occur subsequent
56 MODULE 2   ENGAGEMENT PLANNING AND ASSESSING RISKS
because while planning the audit, 
CPAs may discuss matters that affect 
to planning. Answer (c) is incorrect because  the audit with firm personnel 
while reading specialized industry journals will responsible for providing nonaudit 
help the auditor to obtain a better  services to the entity. Answer (d) is 
understanding of the client’s industry, it is  incorrect because any available 
likely to be less helpful than reviewing the  current year interim financial 
working papers. Answer (d) is incorrect  statements will be read during the 
because a reevaluation of the client’s internal  planning stage. 
control environment occurs subsequent to the 
ordinal plan­ning of the audit. C.1. Communicate with Predecessor 
Auditors (Subsequent to 
89. (a) The requirement is to determine who, at a 
Engagement Acceptance)
mini­mum, must have knowledge of the 
relevant professional accounting and auditing  92. (a) The requirement is to identify 
standards when an audit is being performed.  whether a predecessor auditor should
Answer (a) is correct because AU 230 requires  permit a successor auditor to review 
that, at a minimum, the auditor with final  working paper analyses relating to 
responsibility have such knowledge. Answers  contingencies, balance sheet 
(b), (c), and (d) are all incorrect because they  accounts, or both. Answer (a) is 
suggest a higher minimum requirement.  correct be­cause AU 315 states that a
predecessor auditor should ordi­
90. (d) The requirement is to determine 
narily permit the successor to review 
why an auditor obtains knowledge 
working papers, in­cluding 
about a new client’s business and its 
documentation of planning, internal 
in­dustry. Answer (d) is correct 
control, audit results, and other 
because obtaining a level of 
matters of continuing accounting and
knowledge of the client’s business 
au­diting significance, such as the 
and industry enables the CPA to 
working paper analysis of balance 
obtain an understanding of the events,
sheet accounts and those relating to 
transactions, and practices that, in the 
contingencies. 
CPA’s judgment, may have a sig­
nificant effect on the financial  93. (d) The requirement is to determine a successor 
statements. Answer (a) is incorrect  auditor’s responsibility when financial statements 
because providing constructive  audited by a predecessor auditor are found to 
suggestions is a secondary, and not  require substantial revi­ 
the primary, reason for obtaining 
knowl­edge about a client’s business 
and industry. Answer (b) is incorrect 
because while a CPA must develop 
an attitude of professional skepticism 
concerning a client, this attitude is not
obtained by obtaining knowledge 
about the client’s business and 
industry. Answer (c) is incorrect 
because in­formation on the business 
and industry of a client will pro­vide 
only limited information in 
determining whether finan­cial 
statements are materially misstated, 
and numerous other factors are 
considered in evaluating audit 
findings. 
91. (c) The requirement is to identify the 
least likely procedure to be 
performed in planning a financial 
statement audit. Answer (c) is correct 
because selecting a sample of 
vendors’ invoices for comparison to 
receiving reports will occur normally 
as a part of the evidence 
accumulation pro­cess, not as a part 
of the planning of an audit. Answer 
(a) is incorrect because coordination 
of the assistance of entity personnel 
in data preparation occurs during 
planning. An­swer (b) is incorrect 
96. (c) The requirement is to determine the proper 
sions. Answer (d) is correct because when a 
successor auditor becomes aware of  focus of analytical procedures used in planning an 
information that indicates that financial  audit. An­swer (c) is correct because analytical 
statements reported on by the predecessor may  procedures used in planning should focus on (1) 
require revision, the successor should request  enhancing the auditor’s under­standing of the 
that the client arrange a meeting among the  client’s business and the transactions and events 
three parties to discuss and attempt to resolve  that have occurred since the last audit date, and (2)
the matter. Answer (a) is incorrect be­cause the identifying areas that may represent specific risks 
successor is not required to notify the audit  relevant to the audit. Answer (a) is incorrect 
com­mittee and stockholders. Answer (b) is  because while analytical procedures performed as 
incorrect because the client should first  substantive tests may affect the scope of other 
communicate with the predecessor before  substantive tests and of tests of controls, analytical
revising the financial statements. Answer (c) is  procedures used in planning generally do not. 
incorrect because a meeting of the three parties Answer (b) is incorrect because the general nature 
is arranged by the client and because the  of ana­lytical procedures used in planning provide 
situation may or may not have any­thing to do  only very limited assurance that potential 
with the integrity of management. misstatements will be identified; analytical 
94. (c) The requirement is to determine  procedures used as substantive tests provide a 
whether asuccessor auditor should request a  level of assurance that potential misstatements 
new client to authorize the predecessor auditor will be identi­fied. Answer (d) is incorrect because
to allow a review of the predeces­sor’s  analytical procedures performed at the review 
engagement letter, working papers, or both.  stage of audits more directly relate to assessing the
An­ adequacy of the available evidence. 
swer (c) is correct because AU 315 states that  97. (a) The requirement is to identify the 
it is advisable that a successor auditor request 
objective of performing analytical 
to be allowed to review the predecessor’s 
procedures in planning an audit. An­
working papers.
swer (a) is correct because AU 329 
C.2.  Perform Analytical Procedures states that the objective of such 
procedures during planning is to identify
95. (b) The requirement is to identify  such things as the existence of unusual 
the audit proce­dure that an auditor  transactions and events, amounts, ratios 
will most likely perform in  and trends that might indicate matters 
planning a financial statement  that have finan­cial statement and audit 
audit. Answer (b) is correct 
planning ramifications. An­ 
because AU 329 requires that an 
swers (b), (c), and (d) are all incorrect 
auditor perform analytical proce­
because while ana­lytical procedures may 
dures such as comparing the 
lead to the discovery of illegal acts, related­
financial statements to antici­pated 
party transactions, and unauthorized 
results during the planning stage of 
transactions, this is not the primary objective.
an audit. An­swers (a), (c), and (d) 
are all incorrect because these  98. (c)   The   requirement   is   to   identify   the
procedures will all occur  type   of   nonfi­nancial   information   an
subsequent to planning.  auditor would most likely consider in 
MODULE 2   ENGAGEMENT PLANNING AND ASSESSING RISKS 57
101. (c) The requirement is to identify the next step 
in assessing the risks of material misstatement 
performing analytical procedures during the  after auditors identify risks, relate risks to what 
planning phase of an audit. Answer (c) is  could go wrong, and con­sider the magnitude of
correct because the square footage of selling  risks. Answer (c) is correct because the 
space may be used in considering the overall  professional standards suggest that auditors 
rea­sonableness of sales. Answer (a) is incorrect should then consider the likelihood that risks 
because while the turnover of personnel in the  involved could result in material misstatements.
accounting department may provide a measure  Answer (a) is incorrect because the assessment 
of risk relating to the accounting function, it is  is not limited to illegal acts. Answer (b) is 
not ordinarily used in performing analytical  incor­rect because the complexity of 
procedures. Similarly, answer (b) is incorrect  transactions is not next to be considered. 
because while the objec­tivity of audit  Answer (d) is incorrect because determining 
committee members is an important consid­ materiality levels occurs prior to this stage of 
eration, it is not ordinarily used in performing  the audit. 
analytical procedures. Answer (d) is also 
incorrect because manage­ment’s plans to  102. (b) The requirement is to identify 
repurchase stock is not directly related to  whether substan­tive tests of details 
analytical procedures. See AU 329 for  and/or risk assessment procedures 
information on ana­lytical procedures. are considered further audit 
procedures that may be designed 
C.4.  Supervision Requirements
after assessing the risks of material 
99. (b) The requirement is to identify the  misstatement. Further audit 
information that is most likely to be  procedures are composed of 
communicated by a supervisor to staff substantive procedures 
assistants. Answer (b) is correct 
because staff assis­tants must be 
aware of how their procedures should
be evaluated in order to perform these
procedures effectively. Answer (a) is 
incorrect because some immaterial 
fraud may be reported to the client’s 
audit committee. Answer (c) is 
incorrect because the emphasis in an 
audit must be on per­forming the 
audit effectively and not merely on 
adhering to time budgets. Answer (d) 
is incorrect because decisions 
regarding transferring documents 
from the current file to the permanent 
file are generally of less importance 
than the pro­cedure suggested by 
answer (b). 
100. (b) The requirement is to determine why the 
work of each assistant should be reviewed. 
Answer (b) is correct because AU 311 suggests 
that the work performed by each assistant 
should be reviewed to determine whether it was
adequately performed and to evaluate whether 
the results are consistent with the conclusions to
be presented in the audi­tor’s report. Answer (a)
is incorrect because CPA firms, not individual 
auditors within the firms, have systems of 
quality control. Answer (c) is incorrect because 
the professional standards do not in general 
approve specific audit proce­dures. Answer (d) 
is incorrect because while determining that the 
audit has been performed by persons having 
ade­quate technical training and proficiency as 
auditors is im­portant, it should be addressed 
prior to the commencement of fieldwork. 
D. Assess   the   Risks   of   Material
Misstatement   and   Design
Further Audit Procedures 
quality control standards require 
(tests of details and analytical procedures) and documentation of the considerations in­volved 
tests of con­trols. Answer (b) is correct  in the resolution of differences of opinion. An­
because while substantive tests of details are  swer (a) is incorrect because the AICPA does 
further audit procedures, risk assessment pro­ not, in general, rule on disagreements of this 
cedures are not. Answer (a) is incorrect  nature. Answer (c) is incorrect because the 
because risk as­sessment procedures are not  disagreement relates to an auditing procedure 
further audit procedures. An­swer (c) is  and therefore in most such circumstances the 
incorrect because substantive tests of details  entity’s man­agement or its audit committee 
are further audit procedures and because risk  will have no particular ex­pertise. Answer (d) is
assessment proce­dures are not. Answer (d) is  incorrect because the disagreement need not 
incorrect because substantive tests of details  necessarily be reported to a peer review 
are further audit procedures. “monitor­ing” team.
103. (b) The requirement is to identify the  E.4.  Prospective Financial Statements
procedure that is least likely to be 
considered a risk assessment  106. (d) The requirement is to identify what is 
procedure. Answer (b) is correct  includedin the examination of a financial 
because confirmation is a substantive  forecast. Answer (d) is correct because an 
test, rather than a risk assessment  examination of a forecast includes an 
procedure. Answers (a), (b), and (c)  evaluation of its preparation and the support 
are all risk assessment procedures, as  underlying management’s assumptions. As 
are certain inquiries of others outside  discussed in AT 301, an examination also 
the entity.  includes evaluating the representation of the 
prospective financial statements for conformity
104. (d) Answer (d) is correct because  with AICPA presentation guidelines and the 
significant risks often involve  issuance of an ex­amination report. Answer (a) 
accounting estimates or complex  is incorrect because the ser­vice need not 
accounting that involves significant  include the compiling or assembling of the 
judgments. Answer (a) is incorrect  financial forecast. Answer (b) is incorrect 
because routine, high­volume  because distribu­tion of financial forecasts 
transactions typically have lower  need not be limited. Answer (c) is incorrect 
risk. Answer (b) is incorrect because  because the CPA assumes no responsibility to 
significant risks do require special  up­date management on key events. See AT 
audit attention. Answer (c) is  301 for informa­tion on prospective financial 
incorrect because significant risks  information.
involve items with high levels of 
E.5.  Quality Control
inherent risk. 
107. (a) The requirement is to determine the 
E. Quality Control
factors thataffect the nature and extent of a CPA 
105. (b) The requirement is to determine the  firm’s quality control policies and procedures. 
propermethod for handling a difference of  Answer (a) is correct because the nature and 
opinion between audi­tors concerning  extent of a firm’s quality control policies and 
interpretation of the results of an auditing  procedures depend on a number of factors, 
procedure. Answer (b) is correct because the  including its size,
58 MODULE 2   ENGAGEMENT PLANNING AND ASSESSING RISKS
professional standards. 

the degree of operating autonomy allowed to E.6.  Public Company Accounting 
its personnel and practice offices, the nature Oversight Board
of   its   practice,   its   organiza­tion,   and Requirements
appropriate cost­benefit considerations.
112. (b) The requirement is to identify the 
108. (d) The requirement is to identify the  correct state­ment concerning PCAOB 
manners in which a CPA firm may  guidance that uses the term “should.” Answer 
communicate its quality control  (a) is correct because the term “should” means 
policies and procedures to its  that the auditor must comply with the 
personnel. Answer (d) is correct  requirements unless he or she can demonstrate 
because the requirements relating to  that alternative actions were sufficient to 
quality control standard  achieve the objectives of the standards. 
documentation state that either orally  Answer (a) is incorrect because terms such as 
or written are acceptable. Answers  “must,” “shall,” and “is required to” are used 
(a), (b) and (c) all include one or  to indicate that the auditor must fulfill the 
more inappropriate “no” replies.  responsibilities. Answer (c) is incor­rect 
109. (c) The requirement is to identify the because terms such as “may,” “might,” and 
“could” are used when the auditor should 
reply that is not one of the elements 
consider performing the audit procedure. 
of quality control. Answer (c) 
Answer (d) is incorrect because no particular 
iscorrect because there is no quality 
terms are used for the situation in which the 
control element on internal control. 
auditor has complete discretion whether to 
Acceptance of client relationships 
perform the procedure.
and specific engagements (a), human
resources (b), and monitoring (d) are
all elements of quality control. In 
addition, the following also are 
elements of quality control: 
leadership responsibilities for 
engagement performance, quality 
within the firm, and relevant ethical 
requirements. 
110. (d) The requirement is to determine the types of
services to which Statements on Quality 
Control Standards apply. Answer (d) is correct 
because the standards explicitly limit 
application to auditing and accounting and 
review services. Although the quality control 
standards may be applied to other segments of a
firm’s practice (e.g., manage­ment advisory 
services and tax), the standards do not require 
it. 
111. (a) The requirement is to determine how a CPA
firm obtains reasonable assurance of providing 
professional ser­vices that conform with 
professional standards. Answer (a) is correct 
because a system of quality control is designed 
to provide a CPA firm with reasonable 
assurance of meeting its responsibility to 
provide professional services that conform with
professional standards. Answer (b) is incorrect 
because a peer review provides information on 
whether a CPA firm is following an appropriate
system of quality control. An­swer (c) is 
incorrect because it is less complete than an­
swer (a) since continuing professional 
education helps achieve the specific quality 
control element of professional development. 
Answer (d) is incorrect because complying with
generally accepted reporting standards is only 
one part of the basic objective of providing 
professional services that conform with 
companies. Answer (a) is incorrect because the 
113. (c) The requirement is to determine the set  PCAOB has authority to issue auditing 
of stan­dards that the PCAOB does not have  standards. Answer (b) is incorrect because the 
authority to establish. Answer (c) is correct  PCAOB has the authority to issue quality 
because the FASB establishes ac­counting  control standards. Answer (d) is incorrect 
standards for both public and nonpublic  because the PCAOB has the authority to issue 
independence standards.
MODULE 2   ENGAGEMENT PLANNING AND ASSESSING RISKS 59

SOLUTION TO SIMULATIONS

Task­Based Simulation 1
Audit Risk
Application Authoritative
Literature Help

Factor Increased audit riskDecreased audit risk Fraud risk factor


1. Branch management authority
2. Government regulation
3. Company profitability
4. Demand for product
5. Interest rates
6. Availability of mortgage funds
7. Involvement of principal shareholder in
management
8. Branch manager compensation
9. Internal audit reporting relationship
10. Accounting department turnover
11. Continuing audit relationship
12. Internal controls over accounting estimates
13. Response to proposed accounting adjustments
14. New unprofitable branch
15. New computer system

Task­Based Simulation 2
Risk of Material
Misstatement Authoritative
Literature Help

Risk High risk item
1. Computer fraud risk
2. Risk related to management override of internal control
3. Fraud by branch management
4. Fraud by accounting personnel
5. Misstatement of accounting estimates
6. Fraud by loan processing clerks
7. Fraud by internal auditors
8. The risk of fraudulent misstatement of revenues

Task­Based Simulation 3
Research
Topics Authoritative
Literature Help

T
r
1. P

2. T
h
(
60 MODULE 2   ENGAGEMENT PLANNING AND ASSESSING RISKS

Transactions (A) (B) (C) (D) (E) (F) (G) (H) (I)
4. Comparing a client’s unaudited results for the year with last year’s audited results.
5. Auditing and reporting guidance on the possible need to reaudit previous year
results due to the disbanding of the firm that performed last year’s audit.
6. Requirements relating to identifying violations of occupational safety and health
regulations.
7. Audit report considerations when audit of a subsidiary of the client will be
performed by Williams & Co., CPAs.
8. The need to “brainstorm” among audit team members about how accounts could
be intentionally misstated.
9. Details on considering design effectiveness of controls.
10. The importance of considering the possibility of overstated revenues (for example,
through premature revenue recognition).

Task­Based Simulation 4
Risk Analysis
Authoritative
Literature Help

(A

(B

(C

(D
)
1. Of the following, which is likely to be one of DietWeb’s major risks of doing business on
the Internet in the future? 
2. Which of the following is likely to be the most significant business risk for DietWeb? 
3. Which of the following is most accurate concerning DietWeb’s audit committee? 
4. Which of the following indicates an increased risk of misstatement due to fraud? 
5. Which of the following is the most significant risk facing DietWeb that might cause it to not be
able to continue increasing sales? 
Explanation of solutions
1. (A) The requirement is to identify DietWeb’s major listed risk of doing business on the 
Internet. Answer (A) is correct because DietWeb must carefully maintain the privacy of 
their customers’ information—both due to law and due to DietWeb’s assurance provided to 
its customers. Answer (B) is incorrect because there are no major Federal Communications 
Commission Internet use fees. Answer (C) is incorrect because the case indicates no 
particular problem in providing 24/7/365 support. Answer (D) is incorrect because the 
Internet is able to reach customers beyond the United States. 
2. (C) The requirement is to identify the most significant business risk listed for DietWeb. 
Answer (C) is correct because barriers to entrance on the Internet are ordinarily not high. 
Another organization might develop similar (or more accepted) software, and/or charge 
lower prices than those charged by DietWeb. Answer (A) is incorrect because internal 
control limitations need not necessarily be a major problem, and because internal control 
relates to control risk more directly than to the company’s business risk. Answer (B) is 
incorrect because while Internet instability may cause difficulties, few would consider it as 
significant a problem as new competitors. Answer (D) is incorrect because determining 
appropriate year­end cutoffs is not likely to create major difficulties. 
3. (C) The requirement is to identify the most accurate statement concerning DietWeb’s audit 
committee. Answer (C) is correct because the audit committee has no members who are 
independent of management. Answer (A) is incorrect because the audit committee has no 
members who are independent of management. Answer (A) is incorrect because the 
company founders are not independent. Answer (B) is incorrect because Mr. Readings’ 
chairmanship of the audit committee is likely to result in a weak, not a strong audit 
committee. Answer (D) is incorrect because the size of the board of directors virtually 
always exceeds that of the audit committee—is not equal to or less than in size. 
4. (A) The requirement is to identify a factor that indicates an increased risk of misstatement due to fraud. 
Answer (A) is correct because the simulation provides no explanation of the nature of the disagreement 
that led to Mr. Williamson’s resignation from an apparently very desirable job. Answer (B) is incorrect 
because many small companies are dominated by company founders and those companies do not 
generally misstate earnings due to fraud. Answer (C) is incorrect because the issuance of debt need not 
indicate fraud. Answer (D) is incorrect because competition itself need not lead to an increased risk of 
misstatement due to fraud. 
5. (B) The requirement is to identify the most significant risk facing DietWeb that might
cause it not to be able to continue increasing sales. Answer (B) is correct because the
industry information makes clear that the market changes rapidly—only 
MODULE 2   ENGAGEMENT PLANNING AND ASSESSING RISKS 61

companies that can respond in a timely basis are likely to be able to maintain and improve sales. 
Answers (A) and (C) are incorrect because there is no indication that the United States will face a 
decreasing market for dietary products or that the population is decreasing. Answer (D) is 
incorrect because at this point there is no indication of obsolescence of the Internet.

F
iA
n

L
i
1. T

2. W
h
3. W
h
4. W

5. W

E
1. (C) The requirement is to identify a likely misstatement in the financial statements. Answer 
(C) is correct because common stock issued should be treated under financing rather than 
operations. Answer (A) is incorrect because an increase in cash may well occur—even 
during a year in which the company encounters a loss. Answer (B) is incorrect because there 
is no indication that the impairment expense is inappropriate. Answer (D) is incorrect 
because previous years’ pattern of income and losses may create a situation in which a net 
income tax benefit occurs. 
2. (A) The requirement is to identify, of the balance sheet changes listed, the most unexpected 
one. Answer (A) is unexpected in that prepaid advertising expenses decreased by more than 
ninety percent—at a time when the company increased its sales and marketing expenses so 
significantly. Answer (B) is incorrect because the relatively small increase in accounts 
payable may be expected given the increase in revenues. Answer (C) is incorrect because one
would expect such an increase in deferred revenues as revenues increase. Answer (D) is 
incorrect since the company simply issued more stock—as indicated in the company profile. 
3. (A) The requirement is to identify the factor that might lead the auditors to question 
whether DietWeb has the ability to continue as a going concern. Answer (A) is correct 
because the current net loss may raise a question as to future profitability. Answer (B) is 
incorrect because there was an increase in cash, not a decrease. Answer (C) is incorrect 
because fixed assets decreased rather than increased during 20X8. Answer (D) is incorrect 
because Mr. Reading’s serving as CEO indicates no particular problem. 
4. (B) The requirement is to identify an incorrect classification in the financial statements. 
Answer (B) is correct because accrued liabilities are in general current, not noncurrent, 
liabilities. Answer (A) is incorrect because prepaid expenses are ordinarily assets. Answer 
(C) is incorrect because one would expect the deferred tax liability to have a positive 
balance. Answer D is incorrect because the retained earnings negative balance may be 
explained by early year losses. 
5. (A) The requirement is to identify the most unexpected change. Answer (A) is most unexpected because
the expanded scale of operations would lead one to expect an increase in fixed assets, not a decrease. 
Answer (B) is incorrect because small changes in prepaid expense and other current assets are expected.
Answer (C) is incorrect because it is not surprising that cash may increase when there is a net loss—
particularly in a year when common stock has been issued. Answer (C) is incorrect because an increase 
in accrued liabilities is consistent with an increase in the scale of operations as indicated by a large 
increase in sales. 

Task­Based Simulation 6
Risk of Material Authoritative
Misstatement Analysis Literature Help
AU 110, 312, and 316 all require that auditors consider factors influencing the risk of material 
misstatement and that the risk of material misstatement be considered during the planning phase of 
an audit engagement.
1. (D) Since TWD returned to profitable operation, its healthier financial condition leads to a decrease in
the risk of material misstatement. 
2. (I) The risk of material misstatement increases when management is dominated by a single
person. Since Mead controls the Board of Directors, is a majority stockholder, and is the
CEO, it would appear that Mead dominates management. 
62 MODULE 2   ENGAGEMENT PLANNING AND ASSESSING RISKS

3. (I) The risk of material misstatement increases when the internal auditor reports to top management rather than to the
audit committee because it is less likely that the internal auditor will be able to objectively perform the function. 
4. (I) The risk of material misstatement increases when the key management positions (particularly senior accounting
per­sonnel) encounter turnover. 
5. (D)  The loan officer’s continual monitoring of TWD decreases the risk of material misstatement. 
6. (N)  Timing of payroll cycles would normally have no impact on the risk of material misstatement. 
7. (I) A strong financial presence or ability to dominate a certain industry sector that allows a company to dictate
terms or conditions to suppliers or customers may result in inappropriate or non­arm’s­length transactions. 
8. (I) A change to generally accepted accounting principles will increase the risk of material misstatement because the change in 
basis requires management to prepare a number of entries that have not been made in the past; these entries may be made 
improperly. Also, difficulties in determining beginning accrual basis balances increases the risk of misstatement. 
9. (I) The sale of one­half of the company’s controlling interest in United Equipment Leasing is an entry that is out
of the ordinary course of business, and accordingly, increases the risk of material misstatement. 
10. (D) Litigation results in contentious and difficult accounting valuation issues because an accountant must attempt
to de­termine the likelihood of loss and the amount. 
11. (I) The risk of material misstatement increases when significant related­party transactions occur and management
has an aggressive attitude towards reporting of the transactions. 
12. (I) The risk of material misstatement increases in situations where there are unusual and difficult accounting issues present. It
would appear that the barter transaction with a below­market purchase would be considered an unusual transaction. 
13. (N)  The amount of insurance coverage would have little impact on the risk of material misstatement. 
14. (I) The risk of material misstatement increases as it appears that management has taken an aggressive attitude 
toward reporting this transaction. In addition, this appears to be an unusual and difficult accounting issue involving 
revenue recogni­tion. 
15. (I) Experience has shown that a number of entities have intentionally misstated reported financial condition and
operating results in situations in which a public (or private) placement of securities is planned. Accordingly, an initial
public offering of stock increases the risk of material misstatement. 

Task­Based Simulation 7

I
d
R
i

L
i
1. W
h
2. W
h
3. W
h
E
1. (C) The requirement is to identify the factor that is most likely to increase Toastco’s inherent risk. Answer (C) is correct because 
Toastco relies significantly upon the toaster, and since obsolescence of that product could cause very major difficulties for the 
company. Answer (A) is incorrect because the manufacturing continues to be in Asia and there is no indication of a related 
problem. Answer (B) is incorrect because the growth of 3%, while high, does not increase inherent risk. Answer (D) is incorrect 
because overstated accounts receivable due to internal control deficiencies relates to control risk, not inherent risk. 
2. (B) The requirement is to determine the aspect of Toastco that is most likely to increase the risk of misstatement 
arising from fraudulent financial reporting. Answer (B) is correct because the president, Bill Williams, felt pressure to
meet the earn­ings per share number he had promised the bank due to weak second and third quarters. Therefore the 
high earnings in the fourth quarter represent a risk. Answer (A) is incorrect because the stability of the industry 
presents no particular risk. An­swer (C) is incorrect because Toastco’s market share by itself does not seem to pose a 
risk of misstatement. Answer (D) is in­correct because while Toastco would certainly be in a safer position with more
profitable products, this is not as significant a fraud risk as is the pressure to achieve the earnings per share target. 
3. (B) The requirement is to determine an aspect of a company that is most likely to increase the risk of misstatement
arising  from  fraudulent  financial  reporting.  Answer   (B)  is   correct  because   the  president’s  investment in   Toastco
creates a situation in 
MODULE 2   ENGAGEMENT PLANNING AND ASSESSING RISKS 63

which losses in value of the company will profoundly affect him—particularly given his desire to take the company public. 
Answer (A) is incorrect because selling more than one type of product may or may not affect the risk of misstatement. An­
swer (C) is incorrect because it is desirable to have the audit committee composed entirely of independent directors. Answer
(D) is incorrect because the fact that this is a continuing engagement has no direct tie to the risk of misstatement.
Ratio Analysis
(A) (B) (C) (D) (E)  (F) (G)
1. An audit finding most consistent with the change in the gross margin percentage
2. An audit finding most consistent with the change in the current ratio
Explanation of solutions
1. (A) The requirement is to identify an audit finding most consistent with a decrease in the gross margin percentage—
gross margin/sales. Answer (A) is correct because the inability to pass on increases in costs will decrease the gross 
margin because cost of goods sold as a percentage of sales will increase. 
2. (G) The requirement is to identify an audit finding that is consistent with a decrease in the current ratio—current 
assets/current liabilities. Answer (G) is correct because the long­term debt becoming current increases the 
denominator of the current ratio, which decreases the ratio. 
(H) (I) (J) (K) (L) (M)
3. An audit finding consistent with the change in inventory turnover for Toastco
4. An audit finding consistent with the change in the return on equity
Explanation of solutions
3. (K) The requirement is to identify the finding that is consistent with an increase in the inventory turnover ratio—
cost of goods sold/inventory. Answer (D) is correct because a larger percentage of sales in the last month is likely to
result in a lower ending inventory, thus increasing the inventory turnover ratio since the denominator of the fraction
becomes smaller. 
4. (J) The requirement is to identify the reply that is consistent with a decrease in the return on equity ratio—net 
income/ shareholders’ equity. Answer (C) is correct because retention of profits increases the shareholders’ equity, 
thereby decreasing the ratio. 

Task­Based Simulation 8
Observed Ratio Authoritative
Changes Literature Help

A
u
1. I
n
2. A
c
3. A
l
4. L
o
(

6.  Gross margin percentage was unchanged from the
prior year although gross margin increased from
the prior year.  (Select one explanation)
Explanation of solutions
1. (A, B, D) The requirement is to identify three explanations for an increase in the inventory turnover when compared to 
theprior year. The inventory turnover is calculated by dividing the cost of goods sold by the inventory. An increase may occur
64 MODULE 2   ENGAGEMENT PLANNING AND ASSESSING RISKS

either through (1) an overstatement of the cost of goods sold (the numerator), (2) an understatement of inventory (the denomi­
nator), or (3) a combination of changes. Answer (A) is correct because the recording of the consignment shipment as a sale will 
overstate cost of goods sold and understate the ending inventory. Answer (B) is correct because not recording the credit memos 
will result in understatement of inventory. Answer (D) is correct because the understatement of purchases of inventory will 
understate the ending inventory.
Answer (C) is incorrect because overstating the year­end purchases will result in overstatement of inventory, and thereby
decrease the inventory turnover. Answers (E) through (J) are all incorrect because such changes in sales will not affect the in­
ventory turnover ratio. Answers (K) through (P) are all incorrect because the interest expense, income tax rate, and the short­
term borrowing do not affect the inventory turnover. Note that this question relies upon an unstated assumption that the year­
end inventory is not adjusted to a year­end physical count.
2. (A, B, E) The requirement is to identify three explanations for a decrease in the accounts receivable turnover when com­
pared to the prior year. The accounts receivable turnover is calculated by dividing sales by accounts receivable. A decrease 
may occur through (1) an understatement of sales (the numerator), (2) an overstatement of accounts receivable (the 
denomina­tor), or (3) a combination of misstatements of sales and accounts receivable that decrease the ratio. Answer (A) 
is correct be­cause recording the consignment as sales overstates both sales and accounts receivable by an identical amount,
thus decreasing the ratio; the decrease is due to the entry debiting accounts receivable and crediting sales for the same 
amount. Answer (B) is correct because not recording the credit memo overstates both sales and accounts receivable by an 
identical amount, thus de­creasing the ratio; this identical amount of decrease is due to the lack of a debit to sales returns 
and allowances and a credit to accounts receivable. Note that answers (A) and (B) are correct in any situation in which the 
ratio is greater than 1.0; when the ratio is less than 1.0 they result in an increase in the ratio. Answer (E) is correct because 
while the sales for the year remain at the expected level, accounts receivable at year­end will be at a higher than average 
level due to the year­end sales. 
Answers (C) and (D) are incorrect because the level of inventory does not affect the ratio. Answers (F) and (G) are incor­
rect because a larger, not a smaller or the same, percentage of sales near year­end decreases the ratio. Answers (H), (I) and (J) 
are incorrect because one would expect accounts receivable to increase at the same rate as the increase in sales. Answers (K) 
through (P) are all incorrect because interest expense, income tax rate, and the short­term borrowing do not affect the accounts
receivable turnover. 
3. (A, B, E) The requirement is to identify three explanations for an increase in the allowance for doubtful accounts, but 
a decrease in the allowance for doubtful accounts as a percentage of accounts receivable. The allowance for doubtful 
accounts as a percentage of accounts receivable is calculated by dividing the allowance for doubtful accounts by 
accounts receivable. The percentage may decrease due to (1) a decrease in the allowance for doubtful accounts, (2) an 
increase in the accounts receivable, or (3) a combination of misstatements that decrease the ratio; here, however, we 
are told that reason (1), a decrease in the allow­ance, has not occurred. Answer (A) is correct because recording the 
consignment as a sale results in an increase in accounts receivable which decreases the ratio. Answer (B) is correct 
because not recording the credit memos overstates accounts receiv­able, thereby decreasing the ratio. Answer (E) is 
correct because the larger percentage of sales occurring during the last month of the year results in accounts receivable
at year­end that will be at a higher than average level due to the year­end sales. 
Answers (C) and (D) are incorrect because the level of inventory does not affect the ratio. Answers (F) and (G) are incor­
rect because a larger, not a smaller or the same, percentage of sales near year­end decreases the ratio. Answers (H), (I), and (J) 
are incorrect because one would expect accounts receivable and the allowance for doubtful accounts to increase at approxi­
mately the same rate as the increase in sales. Answers (K) through (P) are all incorrect because interest expense, income tax 
rate, and the short­term borrowing do not affect the accounts receivable turnover.
4. (P) The requirement is to identify a reason why long­term debt increased, but interest expense increased a larger­
than­proportionate amount than long­term debt. Answer (P) is correct because the higher interest rates on long­term 
debt will result in higher interest expense. Answers (A) through (M) are all incorrect because they relate neither to 
long­term debt nor interest expense. Answers (N) and (O) are incorrect because refinancing at the same or a lower 
interest rate will result in smaller­than­proportionate amounts of interest expense. 
5. (L, P) The requirement is to identify two reasons why operating income might increase, yet the company would be 
less profitable. Since operating income increased and net income decreased, the explanation must be items that are 
listed on the income statement between operating income and net income—interest expense and federal income taxes.
The net of these two expenses must have increased to result in a situation in which the entity was less profitable. 
Answer (L) is correct because an increase in the effective income tax rate could decrease the profit when compared to
the prior year. Answer (P) is correct be­cause higher interest rates decrease profits. Answers (A) through (J) are all 
incorrect because they pertain to details of operating income. Answers (K) and (M) are incorrect because a decrease 
in interest expense or the income tax rate would increase net income. Answer (N) is incorrect because refinancing at 
the same rate will not affect net income. Answer (O) is incorrect be­cause refinancing at a lower interest rate will 
increase profits. 
6. (H) The requirement is to identify one reason why the gross margin percentage may remain unchanged, despite an
increase   in   gross   margin   from   the   prior   year.   Answer   (H)   is   correct   because   when   sales   increase   at   the   same
percentage as cost of goods sold, the gross margin percentage remains unchanged, and yet the increased sales will
result in an increase in the gross margin. 
Answers (A) through (D) are all incorrect because they will result in a change in the gross margin percentage. Answers (E), (F),
and (G) are all incorrect because no increase in sales is indicated and no information on the gross margin is provided. An­swers 
(I) and (J) are incorrect because they suggest a decrease and an increase in the gross margin, respectively. Answers (K) through 
(P) are all incorrect because interest expense, income tax rate, and debt do not affect gross margin. 
MODULE 2 ENGAGEMENT PLANNING AND ASSESSING RISKS 65

Task­Based Simulation 9
Assertions and
Audit Procedures Authoritative
Literature Help

Assertion (A) (B) (C) (D) (E)  (F)
1. Completeness
2. Cutoff
3. Existence and occurrence
4. Presentation and disclosure
5. Rights and obligations
6. Valuation

Audit procedures (A) (B) (C) (D)


1. Prepare a flowchart of internal control over sales
2. Calculate the ratio of bad debt expense to credit sales.
3. Determine whether disbursements are properly approved.
4. Confirm accounts receivable.
5. Compare current financial information with comparable prior periods.

Task­Based Simulation 10
Audit Risk and
Its Components   Authoritative
Literature Help

1. (No)  The risk of material misstatement is composed of inherent risk and detection risk. 
2. (Yes)
3. (No)  A decrease in control risk, absent other changes, results in a decrease in the risk of material misstatement. 
4. (Yes) Detection risk is a function of the audit and its procedures.  If there is no audit there is no measure of detection risk. 
5. (No) This is backwards. Auditors restrict detection risk through the performance of more substantive procedures.
Audi­tors assess inherent risk and control risk. 
6. (Yes)
7. (No)  The error is the “or immaterially.”  Audit risk deals with material misstatements. 
8. (Yes)
2. Enter   the   exact   section   and
Task­Based Simulation 11 paragraph   with   helpful
information 
Research:
Supervision
Authoritative
Literature Help

1. Which title of the Professional Standards addresses this issue and will be helpful in
responding to the CEO? 
(A) (B) (C) (D) (E) (F) (G) (H)

210 03
66 MODULE 2   ENGAGEMENT PLANNING AND ASSESSING RISKS

Task­Based Simulation 12
Research: Tone
at the Top Authoritative
Literature Help
(A) (B) (C) (D) (E) (F) (G) (H)
1. Which title of the Professional Standards addresses this issue and will be helpful
in responding to the partner?
2. Enter the exact section and paragraphs with helpful information. 10 15­18

Task­Based Simulation 13
Effects on Audit
Risk Components Authoritative
Literature Help

S
i
1. D
u
2. I
n
3. E
a
Task­Based Simulation 14 (A) (B) (C) (D) (E) (F) (G) (H)
Research
Authoritative
10 12
Literature Help

1. Which title of the Professional Standards addresses this issue and will be helpful in
responding to the partner? 
2. Enter the exact section and paragraphs with helpful information. 
MODULE 3  UNDERSTANDING INTERNAL CONTROL AND ASSESSING CONTROL RISK 67

UNDERSTANDING INTERNAL CONTROL
AND ASSESSING CONTROL RISK
Audit Integrity and
MULTIPLE­CHOICE QUESTIONS (1­166)
committee ethical values Organizational
1. Which of the following most likely would not be a. Yes Yes No
con­sidered   an   inherent   limitation   of   the b. Yes No Yes
potential   effectiveness   of   an   entity’s   internal c. No Yes Yes
control?  d. Yes Yes Yes
a. Incompatible duties. 
b. Management override. 
c. Mistakes in judgment. 
d. Collusion among employees. 
2. When considering internal control, an auditor 
should be aware of the concept of reasonable 
assurance, which recog­nizes that 
a. Internal control may be ineffective due
to   mistakes   in   judgment   and   personal
carelessness. 
b. Adequate safeguards over access to 
assets and rec­ords should permit an 
entity to maintain proper accountability. 
c. Establishing   and   maintaining   internal
control is an important responsibility of
management. 
d. The cost of an entity’s internal control
should not exceed the benefits expected
to be derived. 
3. Proper segregation of functional responsibilities
calls for separation of the functions of 
a. Authorization, execution, and payment. 
b. Authorization, recording, and custody. 
c. Custody, execution, and reporting. 
d. Authorization, payment, and recording. 
4. An entity’s ongoing monitoring activities often include 
a. Periodic audits by the audit committee. 
b. Reviewing the purchasing function. 
c. The audit of the annual financial statements. 
d. Control risk assessment in conjunction
with quar­terly reviews. 
5. The overall attitude and awareness of an entity’s 
board of directors concerning the importance of 
internal control usually is reflected in its 
a. Computer­based controls. 
b. System of segregation of duties. 
c. Control environment. 
d. Safeguards over access to assets. 
6. Management philosophy and operating style most likely 
would have a significant influence on an entity’s control 
environment when 
a. The   internal   auditor   reports   directly
to manage­ment. 
b. Management is dominated by one individual. 
c. Accurate management job descriptions
delineate specific duties. 
d. The audit committee actively oversees the financial
reporting process. 
7. Which of the following factors are included in
an en­tity’s control environment? 
8. Which of the following is  not  a component of out recording the purchase or receipt of the merchandise. 
an en­tity’s internal control?  When vendors’ invoices arrive, one of the employees ap­
a. Control risk.  proves the invoices for payment. After the invoices are paid,
b. Control activities.  the employee destroys the invoices and the related vouchers.
c. Monitoring.  In gathering evidence regarding the fraud, the auditor most 
d. Control environment.  likely would select items for testing from the file of all 
a. Cash disbursements. 
9. Which of the following is a provision of the b. Approved vouchers. 
Foreign Corrupt Practices Act?  c. Receiving reports. 
a. It is a criminal offense for an auditor to d. Vendors’ invoices. 
fail to de­tect and report a bribe paid by
an   American   busi­ness   entity   to   a 11. Which of the following procedures most likely 
foreign   official   for   the   purpose   of would provide an auditor with evidence about 
obtaining business.  whether an entity’s internal control activities are 
b. The auditor’s detection of illegal acts  suitably designed to prevent or detect material 
committed by officials of the auditor’s  misstatements? 
publicly held client in conjunction with  a. Reperforming   the   activities   for   a   sample   of
foreign officials should be re­ported to  transac­tions. 
the Enforcement Division of the Securi­ b. Performing   analytical   procedures   using
ties and Exchange Commission.  data aggre­gated at a high level. 
c. If the auditor of a publicly held  c. Vouching   a   sample   of   transactions
company con­cludes that the effects on  directly related to the activities. 
the financial statements of a bribe given  d. Observing   the   entity’s   personnel
to a foreign official are not sus­ceptible  applying the ac­tivities. 
of reasonable estimation, the auditor’s  12. Which   statement   is   correct   concerning   the
re­port should be modified. 
relevance   of   various   types   of   controls   to   a
d. Every publicly held company must  financial audit? 
devise, docu­ment, and maintain internal a. An   auditor   may   ordinarily   ignore   a
control sufficient to provide reasonable  consideration   of   controls   when   a
assurances that internal control  substantive audit approach is taken. 
objectives are met. 
b. Controls over the reliability of financial reporting 
10. An auditor suspects that certain client employees are  are ordinarily most directly relevant to an audit, 
ordering merchandise for themselves over the Internet with­ but other controls may also be relevant. 
68 MODULE 3  UNDERSTANDING INTERNAL CONTROL AND ASSESSING CONTROL RISK
17. Decision   tables   differ   from   program
c. Controls over safeguarding of  flowcharts   in   that   decision   tables
assets and liabilities are of primary emphasize 
importance, while controls over  a. Ease of manageability for complex programs.
the reliability of financial reporting b. Logical   relationships   among
may also be rele­vant.  conditions and ac­tions. 
d. All   controls   are   ordinarily   relevant   to   an c. Cost benefit factors justifying the program. 
audit.  d. The   sequence   in   which   operations   are
performed. 
13. In an audit of financial statements in 
accordance with generally accepted auditing  18. During the consideration of internal control
standards, an auditor is required to  in a finan­cial statement audit, an auditor is
a. Document   the   auditor’s not obligated to 
understanding   of   the   en­tity’s a. Search for significant deficiencies
internal control.  in   the   operation   of   the   internal
b. Search for significant deficiencies control. 
in the operation of internal control. b. Understand   the   internal   control   and   the
c. Perform   tests   of   controls   to information system. 
evaluate  the effective­ness of  the
entity’s internal control. 
d. Determine   whether   controls   are
suitably   designed   to   prevent   or
detect material misstatements. 
14. In obtaining an understanding of an 
entity’s internal control relevant to audit 
planning, an auditor is required to obtain 
knowledge about the 
a. Design of the controls pertaining to internal
control components. 
b. Effectiveness   of   controls   that
have been imple­mented. 
c. Consistency   with   which   controls
are currently be­ing applied. 
d. Controls   related   to   each  principal
transaction   class   and   account
balance. 
15. An   auditor   should   obtain   sufficient
knowledge   of   an   entity’s   information
system to understand the 
a. Safeguards used to limit access to
computer facili­ties. 
b. Process used to prepare significant
accounting esti­mates. 
c. Controls used to assure proper
authorization of transactions. 
d. Controls   used   to   detect   the   concealment   of
fraud. 
16. When obtaining an understanding of an 
entity’s internal control, an auditor should 
concentrate on the substance of controls 
rather than their form because 
a. The   controls   may   be   operating
effectively   but   may  not  be
documented.
b. Management   may   establish
appropriate   controls   but  not
enforce compliance with them. 
c. The   controls   may   be   so
inappropriate  that  no  reli­ance is
contemplated by the auditor. 
d. Management   may   implement
controls whose costs exceed their
benefits. 
22. Which   of   the   following   may   not   be
c. Determine   whether   the   control   activities required   on   a   particular   audit   of   a
relevant   to   audit   planning   have   been nonissuer (nonpublic) company? 
implemented.  a. Risk assessment procedures. 
d. Perform   procedures   to   understand b. Tests of controls. 
the design of in­ternal control.  c. Substantive procedures. 
19. A   primary   objective   of   procedures d. Analytical procedures. 
performed   to   obtain   an   understanding   of 23. Control risk should be assessed in terms of 
internal control is to provide an auditor with a. Specific controls. 
a. Knowledge   necessary   to b. Types of potential fraud. 
assess the risks of material c. Financial statement assertions.
misstatements.  d. Control environment factors. 
b. Evidence to use in assessing inherent risk. 
c. A basis for modifying tests of controls.  24. After assessing control risk, an auditor desires to seek
d. An  evaluation   of  the  consistency a further reduction in the assessed level of control risk.
of   application   of   management’s At this time, the auditor would consider whether 
policies.  a. It would be efficient to obtain an
understanding   of   the   entity’s
20. Which   of   the   following   statements information system. 
regarding   auditor   documentation   of   the b. The entity’s controls have been implemented.
client’s internal control is correct?  c. The entity’s controls pertain to any
a. Documentation must include flowcharts.  financial state­ment assertions. 
b. Documentation   must   include   procedural d. Additional   audit   evidence
write­ups.  sufficient   to   support   a   further
c. No   documentation   is   necessary reduction   is   likely   to   be
although it is desir­able.  available. 
d. No   one   particular   form   of
documentation is neces­sary, and the 25. Assessing control risk at a low level most
extent of documentation may vary.  likely would involve 
a. Performing   more   extensive
21. In obtaining an understanding of an entity’s  substantive   tests   with   larger
internal control, an auditor is required to obtain sample   sizes   than   originally
knowledge about the  planned. 
b. Reducing inherent risk for most of
Operating effectiveness Design
of controls of controls the   assertions   relevant   to
a. Yes Yes significant account balances. 
b. No Yes c. Changing the timing of substantive 
c. Yes No tests by omit­ting interim­date 
d. No No testing and performing the tests at 
year­end. 
MODULE 3  UNDERSTANDING INTERNAL CONTROL AND ASSESSING CONTROL RISK 69
docu­mented unless control risk is assessed 
at the maxi­mum level. 
a. Identifying   specific   controls   relevant   to d. The lower the assessed level of 
specific as­sertions.  control risk, the less assurance 
the evidence must provide that 
26. An auditor assesses control risk because it  the control procedures are 
a. Is   relevant   to   the   auditor’s operating effectively. 
understanding   of   the   control
environment.  31. Regardless   of   the   assessed   level   of   control   risk,   an
b. Provides   assurance   that   the audi­tor would perform some 
auditor’s   materiality   levels   are a. Tests of controls to determine the
appropriate.  effectiveness   of   internal   control
c. Indicates   to   the   auditor   where policies. 
inherent risk may be the greatest.  b. Analytical procedures to verify the
d. Affects the level of detection risk design of inter­nal control. 
that the auditor may accept.  c. Substantive   tests   to   restrict
detection   risk   for   sig­nificant
27. When an auditor increases the assessed level transaction classes. 
of   control   risk   because   certain   control
activities were determined to be ineffective,
the auditor would most likely increase the 
a. Extent of tests of controls. 
b. Level of detection risk. 
c. Extent of tests of details. 
d. Level of inherent risk. 
28. An auditor uses the knowledge provided by 
the under­standing of internal control and 
the assessed level of the risk of material 
misstatement primarily to 
a. Determine   whether   procedures
and   records   con­cerning   the
safeguarding   of   assets   are
reliable. 
b. Ascertain whether the 
opportunities to allow any person
to both perpetrate and conceal 
fraud are minimized. 
c. Modify   the   initial   assessments   of
inherent   risk   and   preliminary
judgments about materiality levels.
d. Determine   the   nature,   timing,
and   extent   of   sub­stantive   tests
for   financial   statement
assertions. 
29. An auditor may compensate for a weakness
in internal control by increasing the 
a. Level of detection risk. 
b. Extent of tests of controls. 
c. Preliminary judgment about audit risk. 
d. Extent of analytical procedures. 
30. Which   of   the   following   statements   is   correct
concerning an auditor’s assessment of control risk? 
a. Assessing control risk may be 
performed concur­rently during 
an audit with obtaining an under­
standing of the entity’s internal 
control. 
b. Evidence   about   the   operation   of
internal control in prior audits may
not be considered during the cur­
rent   year’s   assessment   of   control
risk. 
c. The basis for an auditor’s conclusions about 
the as­sessed level of control risk need not be
implemented. 
d. Dual­purpose tests to evaluate  c. Detect   material   misstatements   in
both the risk of monetary  the   account   bal­ances   of   the
misstatement and preliminary  financial statements. 
control risk.  d. Evaluate   whether   controls   operated
effectively. 
32. How frequently must an auditor test operating 
effectiveness of controls that appear to function as  36. After obtaining an understanding of internal control 
they have in past years and on which the auditor  and assessing the risk of material misstatement, an 
wishes to rely in the current year?  auditor decided to perform tests of controls. The 
a. Monthly.  auditor most likely decided that 
b. Each audit.  a. It would be efficient to perform
c. At least every second audit.  tests   of   controls   that   would
d. At least every third audit. result in a reduction in planned
sub­stantive tests. 
33. Before assessing control risk at a level lower  b. Additional evidence to support a 
than the maximum, the auditor obtains  further reduction in the risk of 
reasonable assurance that controls are in use  material misstatement is not 
and operating effectively. This assurance is  available. 
most likely obtained in part by 
c. An increase in the assessed 
a. Preparing flowcharts.  level of the risk of material 
b. Performing substantive tests.  misstatement is justified for 
c. Analyzing tests of trends and ratios.  certain financial statement 
d. Inspection of documents.  assertions. 
34. An  auditor  generally  tests   the  segregation d. There were many internal control
of duties re­lated to inventory by  weaknesses   that   could   allow
misstatements   to   enter   the
a. Personal inquiry and observation. 
accounting system. 
b. Test counts and cutoff procedures. 
c. Analytical   procedures   and   invoice 37. In   assessing   control   risk,   an   auditor
recomputation.  ordinarily   selects   from   a   variety   of
d. Document inspection and reconciliation.  techniques, including 
a. Inquiry and analytical procedures. 
35. The   objective   of   tests   of   details   of
b. Reperformance and observation.
transactions   per­formed   as   tests   of
c. Comparison and confirmation. 
controls is to 
d. Inspection and verification. 
a. Monitor   the   design   and   use   of
entity   documents   such   as 38. Which of the following types of evidence 
prenumbered shipping forms.  would an auditor most likely examine to 
b. Determine   whether determine whether controls are operating 
controls   have   been as designed? 
70 MODULE 3  UNDERSTANDING INTERNAL CONTROL AND ASSESSING CONTROL RISK
(nonpublic) companies with 
more than $100,000,000 of 
a. Confirmations   of   receivables net worth. 
verifying account balances.  d. All nonissuer companies. 
b. Letters   of   representations
corroborating inventory pricing.  44. The framework most likely to be used by
c. Attorneys’   responses   to   the   auditor’s management   in   its   internal   control
inquiries.  assessment   under   requirements   of   the
d. Client  records   documenting  the Sarbanes­Oxley Act of 2002 is the 
use of computer programs.  a. COSO internal framework. 
b. COSO   enterprise   risk   management
39. Which of the following is  not  a step in framework. 
an auditor’s assessment of control risk?  c. FASB   37   internal   control   definitional
a. Evaluate   the   effectiveness   of framework. 
internal   control   with   tests   of d. AICPA internal control analysis manager. 
controls. 
b. Obtain   an   understanding   of   the   entity’s
information system and control environment. 
c. Perform tests of details of 
transactions to detect material 
misstatements in the financial 
statements. 
d. Consider whether controls can have
a   pervasive   ef­fect   on   financial
statement assertions. 
40. To obtain audit evidence about control risk,
an   auditor   selects   tests   from   a   variety   of
techniques including 
a. Inquiry. 
b. Analytical procedures. 
c. Calculation. 
d. Confirmation. 
41. Which of the following is least likely to be evidence 
the auditor examines to determine whether controls are
operat­ing effectively? 
a. Records   documenting   usage
of computer pro­grams. 
b. Canceled supporting documents. 
c. Confirmations of accounts receivable.
d. Signatures on authorization forms. 
42. Which of the following procedures 
concerning accounts receivable would an 
auditor most likely perform to obtain 
evidence in support of an assessed level of 
control risk below the maximum? 
a. Observing   an   entity’s   employee
prepare   the   sched­ule  of  past  due
accounts receivable. 
b. Sending confirmation requests to an entity’s 
princi­pal customers to verify the existence 
of accounts receivable. 
c. Inspecting an entity’s analysis  of
accounts   receiv­able   for   unusual
balances. 
d. Comparing an entity’s uncollectible
accounts   ex­pense   to   actual
uncollectible accounts receivable. 
43. The internal control provisions of the 
Sarbanes­Oxley Act of 2002 apply to which 
companies in the United States? 
a. All companies. 
b. SEC registrants.
c. All issuer (public) 
companies and nonissuer 
a. Remote. 
45. Which   of   the   following   is   correct b. More than remote. 
concerning   the  level  of  assistance  auditors c. Probable. 
may provide in assisting management with d. Not explicitly considered. 
its assessment of internal control? 
a. No assistance of any type may be provided.  51. Assume that a company has a control deficiency 
b. No limitations on assistance exist.  regard­ing the processing of cash receipts. 
c. Only   very   limited   assistance   may   be Reconciliation of cash accounts by a competent 
provided. individual otherwise independent of the cash function 
might make the likelihood of a signifi­cant 
d. As   less   risk   is   assumed   by   the
misstatement due to the control deficiency remote. In 
auditors,   a   higher   level   of
this situation, reconciliation may be referred to as what
assistance is appropriate. 
type of control? 
46. Which of the following need not be included a. Compensating. 
in management’s report on internal control  b. Preventive. 
under Section 404a of the Sarbanes­Oxley 
Act of 2002? 
a. A   statement   that   the   company’s
auditor   has   issued   an   attestation
report on management’s assertion. 
b. Identification   of   the
framework   for   evaluating
internal control. 
c. Management’s assessment of the
effectiveness of internal control. 
d. Management’s statement of 
responsibility to estab­lish and 
maintain internal control that has 
no sig­nificant deficiencies. 
47. Which   of   the   following   is   an   accurate
statement   about   internal   control
weaknesses? 
a. Material   weaknesses   are   also   control
deficiencies.
b. Significant   deficiencies
are   also   material
weaknesses. 
c. Control   deficiencies   are   also   material
weaknesses. 
d. All   control   deficiencies   must   be
communicated   to   the   audit
committee. 
48. In an integrated audit, which of the following
is defined as a weakness in internal control 
that is less severe than a material weakness 
but important enough to warrant attention by 
those responsible for oversight of the 
financial reporting function? 
a. Control deficiency. 
b. Unusual weakness. 
c. Unusual deficiency. 
d. Significant deficiency. 
49. A material weakness is a significant 
deficiency (or combination of significant 
deficiencies) that results in a reasonable 
possibility that a misstatement of at least 
what amount will not be prevented or 
detected? 
a. An amount greater than zero. 
b. An amount greater than zero, but
at least inconse­quential. 
c. An amount greater than inconsequential. 
d. A material amount. 
50. The minimum likelihood of loss involved in
the consid­eration of a control deficiency is 
MODULE 3 UNDERSTANDING INTERNAL CONTROL AND ASSESSING CONTROL RISK 71

c. Adjustive. c. No Yes
d. Nonroutine. d. No No
b. Do you believe that you are underpaid? 
52. According to Public Company Accounting c. What do you do when you
Oversight Board Standard 5, what type of  find   a   fraudulent
transaction involves establishing a loan  transaction? 
loss reserve?  d. Who trained you for this job? 
a. Substantive transaction. 
b. Routine transaction.  58. How large must the actual loss identified by
c. Nonroutine transaction.  the   auditor  be   for   a   control   deficiency   to
d. Estimation transaction.  possibly be considered a mate­rial weakness?
53. A procedure that involves tracing a transaction from Immaterial Material
a. Yes Yes
its   origination   through   the   company’s   information
b. Yes No
systems until it is reflected in the company’s financial
report is referred to as a(n) 
a. Analytical analysis. 
b. Substantive procedure. 
c. Test of a control. 
d. Walk­through. 
54. For purposes of an audit of internal control
performed   under   Public   Company
Accounting   Oversight   Board   stan­dards,
the “as of date” is ordinarily 
a. The first day of the year. 
b. The last day of the fiscal period. 
c. The last day of the auditor’s fieldwork. 
d. The average date for the entire fiscal period. 
55. Consider   an   issuer   (public)   company
whose   purchases   are   made   through   the
Internet   and   by   telephone.   Which   of   the
following is correct? 
a. These   types   of   purchases
represent   control   objec­tives   for
the audit of internal control. 
b. These   purchases   are   the
assertions   related   to   the
purchase class of transactions. 
c. These types of purchases 
represent two major classes of 
transactions within the 
purchases pro­cess. 
d. These two types of transactions 
represent routine transactions 
that must always be investigated 
in extreme detail. 
56. For an issuer (public) company audit of internal control,
walkthroughs provide the auditor with  primary evidence
to 
Evaluate the Confirm whether
effectiveness of the controls have been
design of controls placed in operation
a. Yes Yes
b. Yes No
c. No Yes
d. No No
57. Which is most likely to be a question 
asked of em­ployee personnel during a 
walk­through in an audit of the internal 
control of an issuer (public) company? 
a. Have   you   ever   been   asked   to   override   the
process? 
59. For purposes of an audit of internal control  a circumstance­caused scope limitation 
performed under Public Company  relating to a signifi­cant account in a 
Accounting Oversight Board re­quirements,  Sarbanes­Oxley 404 internal control audit 
an account is significant if there is more than  is most likely to result in a(n) 
a  a. Adverse opinion. 
a. Reasonably   possible   likelihood b. Qualified opinion. 
that it could con­tain immaterial c. Unqualified   opinion   with   explanatory
or material misstatements.  language. 
b. Reasonably   possible d. All of the above are equally likely. 
likelihood   that   it   could
63. Which   of   the   following   is   most   likely   to
contain   material
misstatements.  indicate a sig­nificant deficiency relating to a
c. Remote likelihood that it could client’s antifraud programs? 
contain material misstatements.  a. A broad scope of internal audit activities. 
d. Remote   likelihood   that   it   could b. A  “whistle­blower”  program
contain more than inconsequential that   encourages   anonymous
misstatements.  submissions. 
c. Audit   committee   passivity   when
60. A control deficiency that is more than a significant  conducting over­sight functions. 
defi­ciency is most likely to result in what form of  d. Lack   of   performance   of
audit opinion relating to internal control?  criminal   background
a. Adverse.  investigations   for   likely
b. Qualified.  customers. 
c. Unqualified. 
d. Unqualified with explanatory language.  64. An auditor identified a material weakness in 
December. The client was informed and corrected it 
61. Which of the following is most likely to be considered shortly after the “as of date” (December 31); the 
a material weakness in internal control for purposes of auditor agrees that the correc­tion eliminates the 
an in­ternal control audit of an issuer (public)  material weakness as of January 31. The appropriate 
company?  report under a PCAOB Standard 5 audit of internal 
a. An   ineffective   oversight   of control is 
financial   reporting   by   the   audit a. Adverse. 
committee.  b. Unqualified. 
b. Restatement   of   previously   issued c. Unqualified   with   explanatory
financial   state­ments   due   to   a language   relating   to   the   material
change in accounting principles.  weakness. 
c.Inadequate   segregation   of d. Qualified. 
recordkeeping from ac­counting. 
d. Weaknesses in control activities.  65. In   an   integrated   audit,   which   of   the
following lead(s) to an adverse opinion on
62. Inability to evaluate internal control due to  internal control? 
72 MODULE 3  UNDERSTANDING INTERNAL CONTROL AND ASSESSING CONTROL RISK
testing beyond those controls. 
b. Starting   at   the   top—controls
Material Significant over   specific   asser­tions—the
weaknesses deficiencies auditor   should   link   to   major
a. Yes Yes accounts and reporting items. 
b. Yes No c. The goal is to focus on details of 
c. No Yes accounting con­trols, while 
d. No No
avoiding consideration of overall 
66. In an integrated audit, what must the auditor  entity­level controls. 
communi­cate to the audit committee? d. The goal is to focus on all controls 
related to asser­tions, omitting 
Known material All control
consideration of controls related to 
weaknesses deficiencies
the financial statements. 
a. Yes Yes
b. Yes No
c. No Yes
d. No No
67. In which manner are significant deficiencies 
communi­cated by the auditors to the audit 
committee under Public Company 
Accounting Oversight Board Standard 5? 
a. The communication may either be orally or in
writ­ten form. 
b. The communication must be oral,
and not in writ­ten form. 
c. The communication must be in written form. 
d. No   such   communication   is
required   as   only   mate­rial
weaknesses   must   be
communicated. 
68. Which is correct concerning the external 
auditors’ use of the work of others in an 
audit of internal control per­formed for a 
public company? 
a. It is not allowed. 
b. The work of internal auditors may 
be used, but only when those 
internal auditors report directly to 
the audit committee. 
c. Ordinarily   the   work   of   internal
auditors   and   others   is   used
primarily in low­risk areas. 
d. There is no limitation and is likely
to   reduce   audi­tor   liability   since
the   auditors   will   then   share   legal
responsibility with those who have
performed the service. 
69. In an integrated audit, which must
the auditor communicate in writing
to management? 
a. Only material weaknesses. 
b. Material   weaknesses   and   significant
deficiencies. 
c. Material   weaknesses,   significant
deficiencies   and   other   control
deficiencies. 
d. Material weaknesses, significant deficiencies,
other control deficiencies,  and all suspected
and possible employee law violations. 
70. Which of the following is correct when 
applying a top­down approach to identify 
controls to test in an integrated audit? 
a. For certain assertions, strong 
entity­level controls may allow 
the auditor to omit additional 
were accounted for. 
71. Which of the following is not included in a
76. Which of the following controls most likely would re­
standard   unqualified   opinion   on   internal
duce the risk of diversion of customer receipts by an 
control over financial re­porting performed
entity’s employees? 
under PCAOB requirements? 
a. A bank lockbox system. 
a. Because of inherent limitations, 
b. Prenumbered remittance advices. 
internal control over financial 
c. Monthly bank reconciliations. 
reporting may not prevent or detect
misstatements.  d. Daily deposit of cash receipts. 
b. In our opinion, [company name] maintained, 77. An auditor suspects that a client’s cashier is
in   all   material   respects,   effective   internal misappro­priating   cash   receipts   for   personal
control over fi­nancial reporting.  use   by   lapping   customer   checks   received   in
c. Our   audit   included   obtaining   an the mail. In attempting to uncover this 
understanding   of   internal   control
over financial reporting. 
d. The [company name] management 
and audit com­mittee is responsible 
for maintaining effective in­ternal 
control over financial reporting. 

72. Walk­throughs ordinarily provide evidence
that helps the auditor to 
Confirm whether
Evaluate design controls have been
effectiveness of placed in
controls Operation
a. Yes Yes
b. Yes No
c. No Yes
d. No No
73. Which of the following procedures would an 
auditor most likely perform to test controls 
relating to manage­ment’s assertion about the
completeness of cash receipts for cash sales 
at a retail outlet? 
a. Observe   the   consistency   of   the
employees’   use   of   cash   registers
and tapes. 
b. Inquire about employees’  access
to recorded but undeposited cash.
c. Trace   deposits   in   the   cash
receipts   journal   to   the   cash
balance in the general ledger. 
d. Compare the cash balance in the
general   ledger   with   the   bank
confirmation request. 
74. Sound internal control dictates that 
immediately upon receiving checks from 
customers by mail, a responsible em­ployee 
should 
a. Add the checks to the daily cash summary. 
b. Verify   that   each   check   is
supported by a prenum­bered sales
invoice. 
c. Prepare a duplicate listing of checks received.
d. Record   the   checks   in   the   cash   receipts
journal. 
75. Tracing   shipping   documents   to
prenumbered   sales   in­voices   provides
evidence that 
a. No duplicate shipments or billings occurred. 
b. Shipments   to   customers   were   properly
invoiced. 
c. All   goods   ordered   by   customers   were
shipped. 
d. All   prenumbered   sales   invoices
MODULE 3  UNDERSTANDING INTERNAL CONTROL AND ASSESSING CONTROL RISK 73
entity’s authorized price list. 
d. Inquire about the  entity’s  credit
embezzlement scheme, the auditor most likely would granting   policies   and   the
com­pare the consistent   application   of   credit
a. Dates checks are deposited per checks. 
bank  statements  with   the  dates
remittance credits are recorded. 81. Which of the following controls most likely
b. Daily cash summaries with the sums of the would   as­sure   that   all   billed   sales   are
cash re­ceipts journal entries.  correctly posted to the accounts receivable
c. Individual   bank   deposit  slips   with ledger? 
the   details   of   the   monthly   bank a. Daily   sales   summaries   are
statements.  compared to daily post­ings to the
d. Dates uncollectible accounts are authorized to accounts receivable ledger. 
be written off with the dates the write­offs are b. Each sales invoice is supported by
actually recorded.  a   prenumbered   shipping
document. 
78. Upon receipt of customers’ checks in the  c. The accounts receivable ledger is
mailroom, a responsible employee should  reconciled   daily   to   the   control
prepare a remittance listing that is forwarded  account in the general ledger. 
to the cashier. A copy of the listing should be d. Each   shipment   on   credit   is
sent to the  supported by a prenum­bered sales
a. Internal   auditor   to   investigate invoice. 
the   listing   for   un­usual
transactions. 
b. Treasurer   to   compare   the   listing
with the monthly bank statement. 
c. Accounts receivable bookkeeper to
update   the   sub­sidiary   accounts
receivable records. 
d. Entity’s   bank   to   compare   the
listing   with   the   cash­ier’s   deposit
slip. 
79. Which of the following procedures most 
likely would not be a control designed to 
reduce the risk of misstatementsin the billing 
process? 
a. Comparing   control   totals   for
shipping   documents   with
corresponding   totals   for   sales
invoices. 
b. Using   computer   programmed
controls   on   the   pric­ing   and
mathematical   accuracy   of   sales
invoices. 
c. Matching shipping documents with
approved   sales   orders   before
invoice preparation. 
d. Reconciling   the   control   totals
for   sales   invoices   with   the
accounts   receivable   subsidiary
ledger.
80. Which of the following audit procedures would an 
audi­tor most likely perform to test controls relating to 
manage­ment’s assertion concerning the completeness 
of sales trans­actions? 
a. Verify that extensions and 
footings on the entity’s sales 
invoices and monthly customer 
statements have been recomputed. 
b. Inspect the entity’s reports of 
prenumbered ship­ping documents 
that have not been recorded in the 
sales journal. 
c. Compare   the   invoiced   prices   on
prenumbered   sales   invoices   to   the
invoices are matched by an 
82. An   auditor   tests   an   entity’s   policy   of employee who does not have au­
obtaining   credit   approval   before   shipping thority to write off bad debts. 
goods   to   customers   in   support   of c. Employees   involved   in   the   credit­granting
management’s   financial   statement function   are   separated   from   the   sales
assertion of  function. 
a. Valuation or allocation.  d. Subsidiary   accounts   receivable
b. Completeness.  records   are   recon­ciled   to   the
c. Existence or occurrence.  control   account   by   an   employee
d. Rights and obligations.  inde­pendent of the authorization of
credit. 
83. Which of the following controls most likely 
would help ensure that all credit sales  85. Proper authorization of write­offs of 
transactions of an entity are re­corded?  uncollectible ac­counts should be 
a. The billing department supervisor approved in which of the following de­
sends   copies   of   approved   sales partments? 
orders to the credit department for a. Accounts receivable. 
comparison   to   authorized   credit b. Credit. 
limits   and   current   customer c. Accounts payable. 
account balances.  d. Treasurer. 
b. The accounting department 
supervisor indepen­dently  86. Employers   bond   employees   who   handle
reconciles the accounts receivable  cash receipts because fidelity bonds reduce
subsidi­ary ledger to the accounts  the   possibility   of   employing   dishonest
receivable control ac­count  individuals and 
monthly.  a. Protect   employees   who   make
c. The accounting department supervisor controls  unintentional   mis­statements
the mailing of monthly statements to customers  from possible monetary damages
and investigates any differences reported by  re­sulting   from   their
customers.  misstatements. 
d. The billing department supervisor  b. Deter dishonesty by making 
matches pre­numbered shipping  employees aware that insurance 
documents with entries in the  companies may investigate and 
sales journal.  prose­cute dishonest acts. 
c. Facilitate   an   independent
84. Which of the following controls most likely monitoring   of   the   receiv­ing   and
would be effective in offsetting the  depositing of cash receipts. 
tendency of sales personnel to maximize  d. Force   employees   in   positions   of
sales volume at the expense of high bad  trust to take peri­odic vacations and
debt write­offs?  rotate their assigned duties. 
a. Employees   responsible   for
authorizing   sales   and   bad   debt 87. During the consideration of a small business
write­offs   are   denied   access   to client’s internal control, the auditor 
cash.  discovered that the accounts receivable clerk
b. Shipping documents and sales  approves credit memos and has access to 
74 MODULE 3  UNDERSTANDING INTERNAL CONTROL AND ASSESSING CONTROL RISK
and receiving reports. 

cash. Which of the following controls would be most  93. With properly designed internal control, the


effec­tive in offsetting this weakness? same em­ployee most likely would match
a. The   owner   reviews   errors   in vendors’   invoices   with   re­ceiving   reports
billings to customers and postings and also 
to the subsidiary ledger.  a. Post the detailed accounts payable records. 
b. The   controller   receives   the   monthly   bank b. Recompute   the   calculations   on   vendors’
statement directly and reconciles the checking invoices. 
accounts.  c. Reconcile the accounts payable ledger. 
c. The owner reviews  credit memos d. Cancel vendors’ invoices after payment. 
after they are re­corded.  94. An entity’s internal control requires for 
d. The controller reconciles the total every check request that there be an 
of   the   detail   ac­counts   receivable approved voucher, supported by a 
accounts   to   the   amount  shown   in
the ledger. 
88. When a customer fails to include a remittance advice 
with a payment, it is common practice for the person 
open­ing the mail to prepare one. Consequently, mail 
should be opened by which of the following four 
company employees? 
a. Credit manager. 
b. Receptionist.
c. Sales manager. 
d. Accounts receivable clerk. 
89. To provide assurance that each voucher is 
submitted and paid only once, an auditor 
most likely would examine a sample of paid
vouchers and determine whether each 
voucher is 
a. Supported by a vendor’s invoice. 
b. Stamped “paid” by the check signer. 
c. Prenumbered and accounted for. 
d. Approved for authorized purchases. 
90. In testing controls over cash disbursements,
an auditor most likely would determine that
the person who signs checks also 
a. Reviews the monthly bank reconciliation. 
b. Returns the checks to accounts payable. 
c. Is   denied   access   to   the   supporting
documents. 
d. Is responsible for mailing the checks. 
91. In assessing control risk for purchases, an 
auditor vouches a sample of entries in the 
voucher register to the supporting 
documents. Which assertion would this 
test of controls most likely support? 
a. Completeness. 
b. Existence or occurrence. 
c. Valuation or allocation. 
d. Rights and obligations. 
92. Which of the following controls is  not
usually   per­formed   in   the   vouchers
payable department? 
a. Matching   the   vendor’s   invoice
with the related re­ceiving report. 
b. Approving   vouchers   for   payment
by having an au­thorized employee
sign the vouchers. 
c. Indicating   the   asset   and
expense accounts to be debited.
d. Accounting   for   unused
prenumbered   purchase   or­ders
accounts. 
prenumbered purchase order and a prenumbered  d. Sending   written   quarterly
receiving report. To determine whether checks are  confirmations to all ven­dors. 
being issued for unauthorized expenditures, an auditor
most likely would select items for testing from the 
99. With well­designed internal control, 
population of all
employees in the same department most 
likely would approve purchase or­ders, 
a. Purchase orders. 
and also 
b. Canceled checks. 
a. Reconcile the open invoice file. 
c. Receiving reports. 
b. Inspect goods upon receipt. 
d. Approved vouchers. 
c. Authorize requisitions of goods. 
95. Which   of   the   following   questions   would d. Negotiate terms with vendors. 
most   likely   be   included   in   an   internal
control   questionnaire   concerning   the 100. In obtaining an understanding of a 
completeness assertion for purchases?  manufacturing en­tity’s internal control 
a. Is an authorized purchase order  over inventory balances, an auditor most 
required before the receiving  likely would 
department can accept a shipment 
or the vouchers payable 
department can record a voucher? 
b. Are   purchase   requisitions
prenumbered and inde­pendently
matched with vendor invoices? 
c. Is   the   unpaid   voucher   file
periodically   reconciled   with
inventory records by an employee
who   does   not   have   access   to
purchase requisitions? 
d. Are   purchase   orders,   receiving
reports, and vouch­ers prenumbered
and periodically accounted for? 

96. For effective internal control, the accounts
payable de­partment generally should 
a. Stamp,   perforate,   or   otherwise
cancel   supporting   documentation
after payment is mailed. 
b. Ascertain that each requisition is
approved   as   to   price,   quantity,
and   quality   by   an   authorized
em­ployee. 
c. Obliterate the quantity ordered on
the receiving de­partment copy of
the purchase order. 
d. Establish   the   agreement   of   the
vendor’s   invoice   with   the
receiving   report   and   purchase
order. 

97. Internal control is strengthened when the
quantity   of   merchandise   ordered   is
omitted from the copy of the pur­chase
order sent to the 
a. Department that initiated the requisition. 
b. Receiving department. 
c. Purchasing agent. 
d. Accounts payable department. 
98. A client erroneously recorded a large 
purchase twice. Which of the following 
internal control measures would be most 
likely to detect this error in a timely and 
efficient man­ner? 
a. Footing the purchases journal. 
b. Reconciling   vendors’   monthly   statements
with sub­sidiary payable ledger accounts. 
c. Tracing   totals   from   the
purchases journal to the ledger
MODULE 3  UNDERSTANDING INTERNAL CONTROL AND ASSESSING CONTROL RISK 75
b. Purchasing clerk. 
c. Receiving clerk. 
a. Analyze the liquidity and turnover d. Inventory control clerk. 
ratios of the in­ventory. 
b. Perform   analytical   procedures 105. Alpha Company uses its sales invoices for posting per­
designed to identify cost variances. petual inventory records. Inadequate controls over the 
c. Review the entity’s descriptions of in­voicing function allow goods to be shipped that are 
inventory   poli­cies   and not in­voiced. The inadequate controls could cause an 
procedures.  a. Understatement   of   revenues,
d. Perform   test   counts   of   inventory receivables, and in­ventory. 
during the entity’s physical count.  b. Overstatement   of   revenues   and
receivables, and an understatement
101. Which of the following controls most likely of inventory. 
would   be   used   to   maintain   accurate c. Understatement  of revenues  and
inventory records?  receivables, and an overstatement
a. Perpetual   inventory   records   are of inventory. 
periodically   com­pared   with   the d. Overstatement   of   revenues,
current   cost   of   individual receivables, and inven­tory. 
inventory items. 
b. A   just­in­time   inventory   ordering
system keeps in­ventory levels to a
desired minimum. 
c. Requisitions, receiving reports, 
and purchase or­ders are 
independently matched before 
payment is approved. 
d. Periodic   inventory   counts   are
used   to   adjust   the   perpetual
inventory records. 
102. A   client   maintains   perpetual   inventory
records in both quantities and dollars. If the
assessed   level   of   control   risk   is   high,   an
auditor would probably 
a. Insist   that   the   client   perform
physical   counts   of   in­ventory
items   several   times   during   the
year. 
b. Apply gross profit tests to ascertain
the reasonable­ness of the physical
counts. 
c. Increase   the   extent   of   tests   of
controls of the inven­tory cycle. 
d. Request the client to schedule the
physical   inven­tory   count   at   the
end of the year. 
103. Which of the following controls most likely
addresses   the   completeness   assertion   for
inventory? 
a. Work   in   process   account   is
periodically   reconciled   with
subsidiary records. 
b. Employees   responsible   for
custody   of   finished   goods   do
not  perform   the   receiving
function. 
c. Receiving   reports   are
prenumbered   and   periodi­cally
reconciled. 
d. There   is   a   separation   of   duties
between   payroll   de­partment   and
inventory accounting personnel. 
104. Sound internal control dictates that defective 
merchan­dise returned by customers should 
be presented initially to the 
a. Salesclerk. 
computed at authorized rates. 
106. Which of the following is a question that b. Employees work the number of
the   auditor   would   expect   to   find   on   the hours for which they are paid. 
production   cycle   section   of   an   internal c. Segregation of duties exist between
control questionnaire?  the preparation and distribution of
a. Are vendors’ invoices for raw  the payroll. 
materials approved for payment by  d. Controls   relating   to   unclaimed
an employee who is independent of payroll   checks   are   operating
the cash disbursements function?  effectively. 
b. Are signed checks for the 
purchase of raw materi­als mailed 109. Which of the following is a control that
directly after signing without  most likely could help prevent employee
being re­turned to the person who  payroll fraud? 
authorized the invoice  a. The personnel department 
processing?  promptly sends em­ployee 
c. Are all releases by storekeepers  termination notices to the payroll 
of raw materials from storage  supervi­sor. 
based on approved requisition  b. Employees who distribute payroll 
docu­ments?  checks forward unclaimed payroll 
d. Are details of individual  checks to the absent employees’ 
disbursements for raw materials  supervisors. 
balanced with the total to be posted c. Salary   rates   resulting   from   new
to the appropriate general ledger  hires are approved by the payroll
account?  supervisor. 
d. Total hours used for determination
107. The objectives of internal control for a  of gross pay are calculated by the
production cycle are to provide assurance  payroll supervisor. 
that transactions are properly executed and 
recorded, and that  110. In determining the effectiveness of an entity’s 
a. Production   orders   are   prenumbered   and controls relating to the existence or occurrence 
signed by a supervisor.  assertion for payroll transactions, an auditor 
b. Custody of  work in process and most likely would inquire about and 
of   finished   goods   is   properly a. Observe   the   segregation   of   duties
maintained.  concerning   per­sonnel
c. Independent internal verification of responsibilities   and   payroll
activity reports is established.  disbursement. 
d. Transfers to finished goods are  b. Inspect   evidence   of   accounting
documented by a completed  for prenumbered payroll checks.
production report and a quality  c. Recompute   the   payroll
control report.  deductions for employee  fringe
benefits. 
108. An   auditor   vouched   data   for   a   sample   of d. Verify   the   preparation   of   the
employees in a payroll register to approved monthly   payroll   ac­count   bank
clock card data to provide as­surance that  reconciliation. 
a. Payments   to   employees   are
76 MODULE 3  UNDERSTANDING INTERNAL CONTROL AND ASSESSING CONTROL RISK
unclaimed cash payroll should be 
a. Deposited in a safe­deposit box. 
111. An auditor most likely would assess control  b. Held by the payroll custodian. 
risk at a high level if the payroll department  c. Deposited in a special bank account. 
supervisor is responsible for  d. Held by the controller. 
a. Examining   authorization   forms   for   new
employees.  117. The auditor may observe the distribution of
b. Comparing   payroll   registers paychecks to ascertain whether 
with   original   batch   transmittal a. Pay rate authorization is properly
data.  separated   from   the   operating
c. Authorizing   payroll   rate   changes   for   all function. 
employees.  b. Deductions   from   gross   pay   are
d. Hiring   all   subordinate   payroll calculated   correctly   and   are
department employ­ees.  properly authorized. 
c. Employees   of   record   actually
112. Which of the following controls most  exist and are em­ployed by the
likely would prevent direct labor hours  client. 
from being charged to manufacturing 
overhead? 
a. Periodic   independent   counts   of
work in process for comparison to
recorded amounts. 
b. Comparison   of   daily   journal
entries   with   approved   production
orders. 
c. Use   of   time   tickets   to   record
actual   labor   worked   on
production orders. 
d. Reconciliation of work­in­process
inventory   with   periodic   cost
budgets. 
113. In   meeting   the   control   objective   of
safeguarding   of   as­sets,   which   department
should be responsible for 
Distribution Custody of
of paychecks unclaimed paychecks
a. Treasurer Treasurer
b. Payroll Treasurer
c. Treasurer Payroll
d. Payroll Payroll
114. Proper   internal   control   over   the   cash
payroll function would mandate which of
the following? 
a. The payroll clerk should fill the
envelopes   with   cash   and   a
computation of the net wages. 
b. Unclaimed pay envelopes should be retained
by the paymaster. 
c. Each   employee   should   be   asked   to   sign   a
receipt. 
d. A   separate   checking   account  for
payroll be main­tained. 
115. The   purpose   of   segregating   the   duties   of
hiring   person­nel   and   distributing   payroll
checks is to separate the 
a. Authorization   of   transactions
from the custody of related assets.
b. Operational   responsibility   from
the recordkeeping responsibility. 
c. Human   resources   function   from
the controllership function. 
d. Administrative   controls   from   the
internal account­ing controls. 
116. To minimize the opportunities for fraud,
mergers, acquisitions, and the sale 
d. Paychecks agree with the payroll of treasury stock. 
register and the time cards.  d. Verify that stock is issued in 
accordance with the authorization 
118. Which of the following departments most likely would of the board of directors and the 
approve   changes   in   pay   rates   and   deductions   from arti­cles of incorporation. 
employee salaries? 
a. Personnel.  121. Where no independent stock transfer agents 
b. Treasurer.  are em­ployed and the corporation issues its 
c. Controller.  own stocks and main­tains stock records, 
d. Payroll.  canceled stock certificates should 
a. Be defaced to prevent reissuance
119. Which of the following questions would an and   attached   to   their
auditor most likely include on an internal  corresponding stubs. 
control questionnaire for notes payable?  b. Not be defaced but segregated 
a. Are   assets   that   collateralize   notes from other stock certificates and 
payable   critically   needed   for   the retained in a canceled certificates 
entity’s continued existence?  file. 
b. Are   two   or   more   authorized c. Be   destroyed   to   prevent   fraudulent
signatures required on checks that reissuance. 
repay notes payable?  d. Be defaced and sent to the secretary of state. 
c. Are   the   proceeds   from   notes
payable used for the purchase of 122. Which   of   the   following   is  not  a
noncurrent assets?  control   that   is   designed   to   protect
d. Are   direct   borrowings   on   notes investment securities? 
payable authorized by the board of a. Custody over securities should be
directors?  limited to indi­viduals who have 
recordkeeping responsibility over
120. The primary responsibility of a bank acting as registrar the securities. 
of capital stock is to  b. Securities   should   be   properly
a. Ascertain that dividends declared controlled   physically   in   order   to
do  not  exceed   the   statutory prevent unauthorized usage. 
amount allowable in the state of c. Access   to   securities   should   be
in­corporation.  vested in more than one individual.
b. Account   for   stock   certificates   by d. Securities should be registered in
comparing   the   to­tal   shares the name of the owner. 
outstanding   to   the   total   in   the
sharehold­ers subsidiary ledger.  123. Which of the following controls would a 
c. Act as an independent third party  company most likely use to safeguard 
between the board of directors and  marketable securities when an independent 
outside investors concerning  trust agent is not employed? 
MODULE 3  UNDERSTANDING INTERNAL CONTROL AND ASSESSING CONTROL RISK 77

a. The investment committee of the board of directors 128.  Which of the following controls would an entity most
periodically reviews the investment  investments   are   executed   on   the
decisions dele­gated to the treasurer. specific authorization of the board
b. Two company officials have joint of direc­tors. 
control of mar­ketable securities,  d. The   recorded   balances   in   the
which are kept in a bank safe­ investment   subsidiary   ledger   are
deposit box.  periodically   compared   with   the
c. The internal auditor and the controller  contents of the safe­deposit box by
indepen­dently trace all purchases and sales  independent personnel. 
of marketable securities from the subsidiary 
ledgers to the general ledger.  127. A company holds bearer bonds as a short­
d. The chairman of the board verifies  term invest­ment. Responsibility for custody 
the marketable securities, which are  of these bonds and sub­mission of coupons 
kept in a bank safe­deposit box,  for periodic interest collections probably 
should be delegated to the 
each year on the balance sheet date. 
a. Chief Accountant. 
124. A   weakness   in   internal   control   over b. Internal Auditor. 
recording   retire­ments   of   equipment  may c. Cashier. 
cause an auditor to  d. Treasurer. 
a. Inspect certain items of equipment
in the plant and trace those items to
the accounting records. 
b. Review   the   subsidiary   ledger   to
ascertain whether depreciation was
taken on each item of equipment
during the year. 
c. Trace   additions   to   the   “other
assets”   account   to   search   for
equipment that is  still on  hand
but no longer being used. 
d. Select   certain   items   of
equipment   from   the   ac­
counting   records   and   locate
them in the plant. 
125. Which of the following questions would an 
auditor least likely include on an internal 
control questionnaire con­cerning the 
initiation and execution of equipment 
transac­tions? 
a. Are   requests   for   major   repairs
approved   at   a   higher   level   than   the
department initiating the request? 
b. Are prenumbered purchase orders
used   for   equip­ment   and
periodically accounted for? 
c. Are   requests   for   purchases   of
equipment   reviewed   for
consideration   of   soliciting
competitive bids? 
d. Are   procedures   in   place   to
monitor   and   properly   restrict
access to equipment? 
126. Which of the following controls would be 
most effec­tive in assuring that the proper 
custody of assets in the in­vesting cycle is 
maintained? 
a. Direct access to securities in the safe­deposit
box is limited to only one corporate officer. 
b. Personnel   who   post   investment
transactions to the general ledger
are not permitted to update the in­
vestment subsidiary ledger. 
c. The   purchase   and   sale   of
likely use to assist in satisfying the completeness  tests   of   property   and   equipment
assertion related to long­term investments? balances at the end of the year. 
a. Senior management verifies that  b. Analytical   procedures   for   current   year
securities in the bank safe­deposit  property and equipment transactions. 
box are registered in the entity’s  c. Tests of controls and limited tests
name.  of   current   year   property   and
b. The   internal   auditor   compares   the equipment transactions. 
securities   in   the   bank   safe­deposit d. Analytical procedures for property
box with recorded investments.  and equipment balances at the end
c. The treasurer vouches the  of the year. 
acquisition of securities by 
comparing brokers’ advices with  131. In   general,   material   fraud   perpetrated   by
canceled checks.  which of the following are most difficult to
detect? 
d. The   controller   compares   the
current market prices of recorded a. Cashier. 
investments   with   the   brokers’ b. Keypunch operator. 
advices on file.  c. Internal auditor. 
d. Controller. 
129. Which of the following controls would an 
entity most likely use in safeguarding  132. Which of the following is not an accurate
against the loss of marketable securities?  statement about communication of internal
a. An independent trust company that has no  control related matters to management on
direct contact with the employees who have  a nonissuer (nonpublic) company? 
recordkeep­ing responsibilities has possession  a. The auditor must communicate
of the securities.  both   material   weaknesses   and
b. The internal auditor verifies the  significant deficiencies. 
marketable securi­ties in the  b. The auditor must communicate in writing. 
entity’s safe each year on the  c. Previously   communicated
balance sheet date.  weaknesses   that   have   not   been
c. The independent auditor traces all  corrected   need   not   be
purchases and sales of marketable  recommunicated. 
securities through the subsidi­ary  d. A communication indicating that 
ledgers to the general ledger.  no significant deficiencies were 
d. A designated member of the board  identified should not be issued. 
of directors controls the securities 
133. Which of the following matters would an 
in a bank safe­deposit box. 
auditor most likely consider to be a material 
130. When there are numerous property and  weakness to be communi­cated to those 
equipment transactions during the year, an  charged with governance of an audit client? 
auditor who plans to assess control risk at a a. Management’s   failure   to
low level usually performs  renegotiate   unfavorable   long­
a. Tests   of   controls   and   extensive term purchase commitments. 
78 MODULE 3  UNDERSTANDING INTERNAL CONTROL AND ASSESSING CONTROL RISK

b. Recurring operating losses that may indicate going 138.  Which of the following representations should not be
concern problems. a. Significant   deficiencies   are   material   weaknesses
c. Ineffective   oversight   of   financial in   the   design   or   operation   of   specific   internal
reporting   by   those   charged   with control components. 
governance.  b. The auditor is obligated to search for 
d. Management’s current plans to reduce significant deficiencies that could 
its owner­ship equity in the entity.  adversely affect the entity’s ability to 
record and report financial data. 
134. Which   of   the   following   statements   is   correct c. Significant deficiencies need not be 
concern­ing significant deficiencies in an audit?  recommunicated each year if 
a. An auditor is required to search for management has acknowledged its 
significant   deficiencies   during   an understanding of such deficiencies. 
audit.  d. The   auditor   should   separately
b. All   significant   deficiencies   are   also communicate   those   significant
considered to be material weaknesses.  deficiencies   considered   to   be   material
c. An auditor may communicate  weaknesses. 
significant deficiencies during an 
audit or after the audit’s 
completion. 
d. An auditor may report that  no
significant   deficiencies   were
noted during an audit. 

135. An auditor’s letter issued on significant 
deficiencies relating to an entity’s internal 
control observed during a financial statement 
audit should 
a. Include a brief description of the tests 
of controls performed in searching for 
significant deficiencies and material 
weaknesses. 
b. Indicate that the significant deficiencies
should be disclosed in the annual report
to the entity’s share­holders. 
c. Include a paragraph describing 
management’s as­sertion concerning 
the effectiveness of internal control. 
d. Indicate that the audit’s purpose was 
to report on the financial statements 
and not to express an opinion on 
internal control. 
136. Which of the following statements is correct 
concern­ing an auditor’s required 
communication of significant deficiencies? 
a. A significant deficiency previously 
communicated during the prior year’s 
audit that remains uncor­rected causes a
scope limitation. 
b. An auditor should perform tests of
controls on significant deficiencies
before communicating them to the
client. 
c. An auditor’s report on significant 
deficiencies should include a 
restriction on the distribution of the 
report. 
d. An auditor should communicate 
significant deficiencies after tests 
of controls, but before 
commencing substantive tests. 
137. Which   of   the   following   statements   is   correct
concern­ing significant deficiencies noted in an
audit? 
included in a report on internal control related matters  a. Consider   the   weakness   a   scope
noted in an audit? limitation   and   therefore   disclaim
a. Significant   deficiencies   related   to an opinion. 
internal control exist.  b. Consider the effects of the condition on the audit. 
b. There   are   no   significant   deficiencies c. Suspend   all   audit   activities   pending
in the design or operation of internal directions   from   the   client’s   audit
control.  committee. 
c. Corrective follow­up action is  d. Withdraw from the engagement. 
recommended due to the relative 
significance of material weaknesses  141. In identifying matters for communication with 
discovered during the audit.  those charged with governance of an audit 
d. The auditor’s consideration of  client, an auditor most likely would ask 
internal control would not  management whether 
necessarily disclose all significant  a. The   turnover   in   the   accounting
deficiencies that exist.  department was un­usually high. 
b. It   consulted   with   another   CPA   firm
139. Which   of   the   following   statements   concerning   material about account­ing matters. 
weaknesses and significant deficiencies is correct?  c. There were any subsequent events of
a. An   auditor   should   not   identify   and which the auditor was unaware. 
communicate   material   weaknesses d. It   agreed   with   the   auditor’s   assessed
separately   from   significant level of con­trol risk. 
deficiencies. 
142. Which   of   the   following   statements   is   correct
b. Compensating   controls   may   limit   the
severity   of   a   material   weakness   or concern­ing an auditor’s required communication
significant deficiency.  with those charged with governance of an audit
c. Upon discovery an auditor should  client? 
immediately report all material  a. This communication is required to occur
before the auditor’s report on the 
weaknesses and significant 
financial statements is is­sued. 
deficiencies identified during an 
b. This communication should include 
audit. 
discussion of any significant 
d. All   significant   deficiencies   are disagreements with management 
material weak­nesses.  concerning the financial statements.
140. During the audit the independent auditor  c. Any significant matter communicated
identified the existence of a weakness in the  to the audit committee also should be
client’s internal control and communicated this  communicated to man­agement. 
finding in writing to the client’s senior  d. Significant audit adjustments proposed 
management and those charged with governance. by the audi­tor and recorded by 
The auditor should  management need not be communicated
to those charged with governance. 
MODULE 3  UNDERSTANDING INTERNAL CONTROL AND ASSESSING CONTROL RISK 79
that describes the
a. Documentary   evidence   regarding   the   control
143. An   auditor   would  least  likely   initiate   a   discussion   with envi­ronment factors. 
those   charged   with   governance   of   an   audit   client b. Changes in internal control since the prior report. 
concerning  c. Potential   benefits   from   the   practitioner’s
a. The   methods   used   to   account   for suggested improvements. 
significant un­usual transactions.  d. Inherent limitations of any internal control. 
b. The maximum dollar amount of 
misstatements that could exist without 
causing the financial state­ments to be 
materially misstated. 
c. Indications of fraud and illegal acts 
committed by a corporate officer that 
were discovered by the auditor. 
d. Disagreements with management as to 
accounting principles that were 
resolved during the current year’s audit.
144. Which   of   the   following   statements   is   correct
about an auditor’s required communication with
those   charged   with   governance   of   an   audit
client? 
a. Any matters communicated to the entity’s audit 
committee also are required to be communicated 
to the entity’s management. 
b. The auditor is required to inform 
those charged with governance about 
significant misstatements discovered 
by the auditor and subsequently cor­
rected by management. 
c. Disagreements with management about
the ap­plication of accounting 
principles are required to be 
communicated in writing to those 
charged with governance. 
d. Weaknesses in internal control 
previously reported to those charged 
with governance need not be re­
communicated. 
145. Which   of   the   following   matters   is   an   auditor
required   to   communicate   to   an   entity’s   audit
committee? 
I. Disagreements with management about matters 
significant to the entity’s financial statements that 
have been satisfactorily resolved. 
AI. Initial   selection   of   significant   accounting
policies   in   emerging   areas   that   lack
authoritative guidance. 
a. I only. 
b. II only. 
c. Both I and II. 
d. Neither I nor II. 
146.  Should   an   auditor   communicate   the   following
mattersto   those   charged   with   governance   of   an   audit
client?
Management’s consulta­
Significant audit tion with other accoun­
adjustments recorded tants about significant
by the entity accounting matters
a. Yes Yes
b. Yes No
c. No Yes
d. No No
147. In reporting on an entity’s internal control over finan­
cial   reporting,   a   practitioner   should   include   a   paragraph
audit? 
148. Which of the following best describes a CPA’s  Scope Procedures Purpose
en­gagement to report on an entity’s internal  a. Similar Different Similar
control over finan­cial reporting?  b. Different Similar Similar
a. An attestation engagement to form an c. Different Different Different
opinion   on   the   effectiveness   of   its d. Different Similar Different
internal control. 
b. An   audit   engagement   to   provide 151. When an independent auditor reports on internal
negative   assur­ance   on   the   entity’s con­trol   based   on   criteria   established   by
internal control.  governmental agencies, the report should 
c. A prospective engagement to project, for a. Not include the agency’s name in the report. 
a period of time not to exceed one year, b. Indicate matters covered by the study and
whether the auditor’s study included tests
and   report   on   the   expected   benefits   of
of   controls   with   the   procedures   covered
the entity’s internal control. 
by the study. 
d. A   consulting   engagement   to   provide
constructive  advice to  the entity  on its c. Not   express   a   conclusion   based   on
the agency’s criteria. 
internal control. 
d. Assume responsibility for the 
149. An   engagement   to   examine   internal   control   will comprehensiveness of the criteria 
gener­  established by the agency and in­clude 
ally  recommendations for corrective action. 
a. Require procedures that duplicate those already 
152. When an examination has been performed on the
ap­plied in assessing control risk during a 
financial statement audit.  ef­fectiveness of entity’s internal control over 
financial report­ing and a material weakness has 
b. Increase the reliability of the financial
been noted, the practition­er’s report should 
statements   that   have   already   been
express an opinion on 
audited. 
a. The assertion. 
c. Be   more   extensive   in   scope   than   the
assessment of control risk made during b. The subject matter to which the assertion relates.
a financial statement au­dit.  c. Neither of the above. 
d. Be more limited in scope than the  d. Both of the above. 
assessment of control risk made during 153. In assessing the competence of an internal 
a financial statement au­dit.  auditor, an independent CPA most likely would 
150. How do the scope, procedures, and purpose of an obtain information about the 
ex­amination of internal control compare to  a. Quality   of   the   internal   auditor’s
those for obtaining an understanding of internal  working paper documentation. 
control and assessing control risk as part of an  b. Organization’s commitment to integrity
and ethical values. 
80 MODULE 3  UNDERSTANDING INTERNAL CONTROL AND ASSESSING CONTROL RISK

c. Influence of management on the scope of the inter­ 160.  If the independent auditors decide that the work per­
nal auditor’s duties. formed by the internal auditor may have a bearing on their 
d. Organizational   level   to   which   the own procedures, they should consider the internal auditor’s
internal auditor reports.  a. Competence and objectivity. 
b. Efficiency and experience. 
154. For which of the following judgments may an inde­pendent
c. Independence and review skills. 
auditor share responsibility with an entity’s internal auditor
d. Training and supervisory skills. 
who is assessed to be both competent and objective? 
Assessment of Assessment of obtained from 
inherent risk control risk a. Discussions with management personnel. 
a. Yes Yes b. External   quality   reviews   of   the
b. Yes No internal auditor’s activities. 
c. No Yes c. Previous experience with the internal auditor. 
d. No No d. The results of analytical procedures. 
155. The work of internal auditors may affect the 
indepen­dent auditor’s
I. Procedures   performed   in   obtaining   an
understanding of internal control. 
AI. Procedures performed in assessing the risk of
material misstatement. 
BI. Substantive   procedures   performed   in
gathering direct evidence. 
a. I and II only. 
b. I and III only. 
c. II and III only. 
d. I, II, and III. 
156. An internal auditor’s work would most likely 
affect the nature, timing, and extent of an 
independent CPA’s au­diting procedures when 
the internal auditor’s work relates to assertions 
about the 
a. Existence of contingencies. 
b. Valuation of intangible assets. 
c. Existence of fixed asset additions. 
d. Valuation of related­party transactions. 
157. During an audit an internal auditor may provide
direct assistance to an independent CPA in 
Obtaining an Performing Performing
understanding of tests of substantive
internal control controls tests
a. No No No
b. Yes No No
c. Yes Yes No
d. Yes Yes Yes
158. When   assessing   the   internal   auditor’s
competence, the independent CPA should obtain
information about the 
a. Organizational   level   to   which   the
internal auditors report. 
b. Educational   background   and
professional   certifica­tion   of   the
internal auditors. 
c. Policies   prohibiting   the   internal
auditors   from   au­diting   areas   where
relatives are employed. 
d. Internal   auditors’   access   to   records   and
information that is considered sensitive. 
159. In assessing the competence and objectivity of 
an en­tity’s internal auditor, an independent 
auditor would least likely consider information 
161. In   assessing   the   objectivity   of   internal retailer. Cook, CPA, is engaged to express an opinion on a 
auditors, an independent auditor should  description of PDC’s internal controls implemented as of a 
a. Evaluate the quality control program specific date. These controls are relevant to the retailer’s 
in effect for the internal auditors.  internal control, so Cook’s report may be useful in providing 
b. Examine   documentary   evidence   of the retailer’s independent auditor with information necessary 
the work per­formed by the internal to plan a financial statement audit. Cook’s report should 
auditors.  a. Contain   a   disclaimer   of   opinion   on
c. Test a sample of the transactions and the operating effectiveness of PDC’s
balances   that   the   internal   auditors controls.
examined.  b. State whether PDC’s controls were
d. Determine   the   organizational   level   to suitably   de­signed   to   achieve   the
which the in­ternal auditors report.  retailer’s objectives. 
c. Identify   PDC’s   controls   relevant   to
162. Dunn, CPA, is auditing the financial statements of Taft Co. specific finan­cial statement assertions.
Taft uses Quick Service Center (QSC) to process its  d. Disclose   Cook’s   assessed   level   of
payroll. Price, CPA, is expressing an opinion on a descrip­ control risk for PDC. 
tion of the controls implemented at QSC regarding the 
processing of its customers’ payroll transactions. Dunn ex­ 164. The   auditor   who   audits   the   processing   of
pects to consider the effects of Price’s report on the Taft  transactions by a service organization may issue a
engagement. Price’s report should contain a(n)  report on controls 
a. Description of the scope and nature of
Price’s pro­cedures.  Implemented Operating effectiveness
b. Statement that Dunn may assess control a. Yes Yes
b. Yes No
risk based on Price’s report. 
c. No Yes
c. Assertion that Price assumes no  d. No No
responsibility to determine whether 
QSC’s controls are suitably de­signed.  165. Computer Services Company (CSC) processes 
d. Opinion   on   the   operating payrolltransactions for schools. Drake, CPA, is engaged to 
effectiveness   of   QSC’s   internal report on CSC’s policies and procedures implemented as of a 
controls.  specific date. These policies and procedures are relevant to 
the schools’ internal control, so Drake’s report will be useful 
163. Payroll Data Co. (PDC) processes payroll transactions for a 
in providing the schools’ independent auditors with infor­
MODULE 3  UNDERSTANDING INTERNAL CONTROL AND ASSESSING CONTROL RISK 81

mation necessary to plan their audits. Drake’s report 
ex­pressing an opinion on CSC’s policies and 
procedures implemented as of a specific date should 
contain a(n)
a. Description of the scope and nature of
Drake’s pro­cedures. 
b. Statement that CSC’s management has
disclosed   to   Drake   all   design
deficiencies of which it is aware. 
c. Opinion   on   the   operating
effectiveness of CSC’s policies and
procedures. 
d. Paragraph   indicating   the   basis   for
Drake’s assess­ment of control risk. 
166. Lake, CPA, is auditing the financial statements
of Gill Co. Gill uses the EDP Service Center, 
Inc. to process its payroll transactions. EDP’s 
financial statements are audited by Cope, CPA,
who recently issued a report on EDP’s inter­nal
control. Lake is considering Cope’s report on 
EDP’s internal control in assessing control risk
on the Gill engage­ment. What is Lake’s 
responsibility concerning making reference to 
Cope as a basis, in part, for Lake’s own opin­
ion? 
a. Lake may refer to Cope only if Lake 
is satisfied as to Cope’s professional 
reputation and indepen­dence. 
b. Lake may refer to Cope only if 
Lake relies on Cope’s report in 
restricting the extent of substan­tive
tests. 
c. Lake   may   refer   to   Cope   only   if
Lake’s report indi­cates the division
of responsibility. 
d. Lake   may  not  refer   to   Cope
under the circum­stances above. 
82 MODULE 3  UNDERSTANDING INTERNAL CONTROL AND ASSESSING CONTROL RISK

SIMULATIONS

Task­Based Simulation 1
Cash Receipts
and Billing
Authoritative
Literature Help
An auditor’s working papers include the narrative description of the cash receipts and billing portions of Southwest 
Medical Center’s internal control. Evaluate the information in the situation as being either (1) a strength, (2) a weakness, (3) not
a strength or a weakness.
Southwest is a health­care provider that is owned by a partnership of five physicians. It employs eleven physicians, in­
cluding the five owners, twenty nurses, five laboratory and X­ray technicians, and four clerical workers. The clerical workers 
perform such tasks as reception, correspondence, cash receipts, billing, accounts receivable, bank deposits, and appointment 
scheduling. These clerical workers are referred to in the situation as office manager, clerk #1, clerk #2, and clerk #3. Assume 
that the narrative is a complete description of the system.
About two­thirds of Southwest’s patients receive medical services only after insurance coverage is verified by the office
manager and communicated to the clerks. Most of the other patients pay for services by cash or check when services are 
rendered, although the office manger extends credit on a case­by­case basis to about 5% of the patients.
When services are rendered, the attending physician prepares a prenumbered service slip for each patient and gives the 
slip to clerk #1 for pricing. Clerk #1 completes the slip and gives the completed slip to clerk #2 and a copy to the patient.
Using the information on the completed slip, clerk #2 performs one of the following three procedures for each patient:
• Clerk #2 files an insurance claim and records a receivable from the insurance company if the office manager has
verified the patient’s coverage, or 
• Clerk #2 posts a receivable from the patient on clerk #2’s PC if the office manager has approved the patient’s credit, or 
• Clerk #2 receives cash or a check from the patient as the patient leaves the medical center, and clerk #2 records the
cash receipt. 
At the end of each day, clerk #2 prepares a revenue summary.
Clerk #1 performs correspondence functions and opens the incoming mail. Clerk #1 gives checks from insurance compa­
nies and patients to clerk #2 for deposit. Clerk #2 posts the receipt of patients’ checks on clerk #2’s PC patient receivable rec­
ords and insurance companies’ checks to the receivables from the applicable insurance companies. Clerk #1 gives mail re­
quiring correspondence to clerk #3.
Clerk #2 stamps all checks “for deposit only” and each day prepares a list of checks and cash to be deposited in the bank.
(This list also includes the cash and checks personally given to clerk #2 by patients.) Clerk #2 keeps a copy of the deposit list
and gives the original to clerk #3.
Clerk #3 personally makes the daily bank deposit and maintains a file of the daily bank deposits. Clerk #3 also performs
appointment scheduling for all of the doctors and various correspondence functions. Clerk #3 also maintains a list of patients
whose insurance coverage the office manager has verified.
When insurance claims or patient receivables are not settled within sixty days, clerk #2 notifies the office manager. The of­
fice manager personally inspects the details of each instance of nonpayment. The office manger converts insurance claims that 
have been rejected by insurance companies into patient receivables. Clerk #2 records these patient receivables on clerk #2’s PC 
and deletes these receivables from the applicable insurance companies. Clerk #2 deletes the patient receivables that appear to be
uncollectible from clerk #2’s PC when authorized by the office manager. Clerk #2 prepares a list of patients with uncollectible 
balances and gives a copy of the list to clerk #3, who will not allow these patients to make appointments for future services.
Once a month an outside accountant posts clerk #2’s daily revenue summaries to the general ledger, prepares a monthly 
trial balance and monthly financial statements, accounts for prenumbered service slips, files payroll forms and tax returns, and 
reconciles the monthly bank statements to the general ledger. This accountant reports directly to the physician who is the man­
aging partner.
All four clerical employees perform their tasks on PCs that are connected through a local area network. Each PC is acces­
sible with a password that is known only to the individual employee and the managing partner. Southwest uses a standard soft­
ware package that was acquired from a software company and that cannot be modified by Southwest’s employees. None of the
clerical employees have access to Southwest’s check writing abilities.
For each of the following conditions indicate whether it is a strength, weakness, or neither.
(checks)   from   patients
Condition
and   records   the   cash
1. Southwest   is   involved   only   in   medical   services   and   has   not receipt. 
diversified its operations. 
2. Insurance coverage for patients is verified and communicated to the
clerks by the office manager before medical services are rendered. 
3. The physician who renders the medical services documents the services on a 
prenumbered slip that is used for recording revenue and as a receipt for the 
patient. 
4. Cash  collection  is   centralized  in  that  Clerk  #2 receives  the  cash
Strength Weakness Neither


MODULE 3  UNDERSTANDING INTERNAL CONTROL AND ASSESSING CONTROL RISK 83

Condition
5. Southwest extends credit rather than requiring cash or insurance in all cases. 
6. The office manager extends credit on a case­by­case basis rather than
using a formal credit search and established credit limits. 
7. The office manager approves the extension of credit to patients and
also approves the write­offs of uncollectible patient receivables. 
8. Clerk #2 receives cash and checks and prepares the daily bank deposit. 
9. Clerk #2 maintains the accounts receivable records and can add or
delete information on the PC. 
10. Prenumbered service slips are accounted for on a monthly basis by
the outside accountant who is independent of the revenue 
generating and revenue recording functions. 
Strength Weakness Neither

11. The bank reconciliation is prepared monthly by the outside accountant who is
independent of the revenue generating and revenue recording functions. 
12. Computer passwords are only known to the individual employees and the
managing partner who has no duties in the revenue recording functions. 
13. Computer software cannot be modified by Southwest’s employees. 
14. None of the employees who perform duties in the revenue generating
and revenue recording are able to write checks. 
Task­Based Simulation 2
Purchases and
Disbursements Authoritative
Literature Help

The following flowchart depicts part of a client’s purchases and cash disbursements cycle. Some of the flowchart symbols
are labeled to indicate operations, controls, and records. For each symbol numbered 1 through 12, select one response from the
answer lists below. Each response in the lists may be selected once or not at all.
Connectors, documents, departments, and files
Operations and controls
J.  Accounts payable O. Purchase order No. 5
A. Approve receiving report 
K. Canceled voucher P. Receiving report No. 1
B. Prepare and approve voucher 
C. Prepare purchase order  package Q. Stores
D. Prepare purchase requisition  L. From purchasing R. To vendor
E. Prepare purchases journal  M. From receiving S. Treasurer
F. Prepare receiving report  N. From vouchers T. Unpaid voucher file,
G. Prepare sales journal  payable filed by due date
H. Prepare voucher 
I. Sign checks and cancel voucher package documents 
RECEIV VOUCHERS 10
PURCHASING ING PAYABLE

From From From


Stores Purch. 4 5 Ven- From
dor Vouchers
Payable

Pur. Order 4
Requisition 1
Approved
Requisition
1 7 Invoice Voucher Package
6 Requisition 1
Pur. Order 5
Receive Rec. Report 1
Goods Invoice
1 Match with By Approved 1
Pur. Order File Pending Voucher
Name
Arrival of all
Documents

Requisition 1
Pur. Order 5
Pur. Order 4
3
Match
Pur. Order 3 Review
Pur. Order 2
Purchase 1
Order

To
Stores

By
Num-
ber

1.

2.

3.
4.

5.

6.

To
Receiv-
ing

To
Vouchers
Payable

7.

8.

9.

10.

11.

12.

Pur. Order 4
Rec. Report 4
Rec. Report 3
Rec. Report 2
Receiving 1
Report

To Vouchers
Payable

For Receipts and


Returns To
Stores

By
Num-
ber
Docu-
ments

Vouc
her 2
R
e
q
ui
si
ti
o
n 1
Pur.
Ord
er 5
R
e
c.
R
e
p
o
rt 1
In
voi
ce
A
pp
ro
ve
d 1
Vouche
r

General
Accoun
By
9 ing

To Trea- On Due
surer Date
Documents,
Prepare Check &
Remittance Advice

11

Check Copy
12 Remittance 2
Signed Check
Remittance 1
Advice
By
Num-
ber

To
Cancelled To General
Voucher Ven- Account-
dor
Package File ing
MODULE 3  UNDERSTANDING INTERNAL CONTROL AND ASSESSING CONTROL RISK 85

Task­Based Simulation 3
Research
Authoritative
Literature Help

Internal Control Limitations
In a discussion with the controller of Gemcon, the topic of limitations of internal control arose. Relatedly, the Professional
Standards acknowledge that internal control has certain limitations that affect financial statement audits. Search the 
Professional Standards to find the location at which a number of limitations are discussed together.
Selections
A. AU 
B. PCAOB 
C. AT 
D. AR 
E. ET 
F. BL 
G. CS 
H. QC 

(A) (B) (C) (D) (E)  (F) (G) (H)
1. Which title of the Professional Standards addresses this issue?
2. Enter the exact section and paragraph numbers that describe the limitations.

Task­Based Simulation 4
Sales/Shipping
Process
Authoritative
Literature Help

You are working for Smith & Co. CPAs. The following partially completed flowchart depicts part of Welcore Inc., your 
client’s revenue cycle. Some of the flowchart symbols are labeled to indicate controls and records. For each symbol numbered 
1 though 13, select one response from all the answer lists below. Each response in the lists may be selected once or not at all.
86 MODULE 3  UNDERSTANDING INTERNAL CONTROL AND ASSESSING CONTROL RISK

WAREHOUSE
COMPUTER
&
PROCESSING
SHIPPING
DEPARTMENT
SALES DEPT. DEPARTMENT

From From
Custome Computer
r COMPUTERIZED SHIPPING Processing
COMPUTERIZED ORDER PROGRAM:
Cust. Dept.
PROGRAM:
Credit 2 Retrieve Open Orders;
1  and perform edit checks Add Shipping Data;
File
and prepare sales order Transfer to Shipping File; and
Prepare Shipping Documents

Sales Order
Customer
Purchase
3
Order
Sales Order To
Sales Order Warehouse Shipping Doc.
3 and Shipping
Sales Order Shipping
Dept. 2
Transmit 2 Document
Customer 1 1
Data to
Computer

Customer
Purchase
Order

Customer P.O.
Sales Order
Sales Order
2
1

To
Customer
4
Accounts
Rec. Shipping 6
Master File
File

COMPUTERIZED BILLING
Sales PROGRAM;
Inventory Retrieve Shipping Data;
Trans­ Enter Price Data;
Master 10 action 5
Prepare Sales
File File Transaction File; and

2
COMPUTERIZED UPDATE
8 1
PROGRAM;
Update master files;
Prepare G/L Transaction
Summary, Prepare Accounts To Transmit Shipping
Receivable Ledger, Prepare Information
Customer 9 to Computer
Aged T/B, and 11

Sales Order
Shipping Doc. 3
General Shipping
Ledger Accounts Document 2

Transaction 12 Receivable 13 1
Summary Ledger

To
Customer
To To To To with
Accounts
Accounting Accounting Receivable Customer Goods
Credit

1. 8.
2. 9.
3. 10.
4. 11.
5. 12.
6. 13.
7.
MODULE 3  UNDERSTANDING INTERNAL CONTROL AND ASSESSING CONTROL RISK 87

Operations and controls Documents, journals, ledgers, and files
A. Enter shipping data P. Shipping document
B. Verify agreement of sales order and shipping document Q. General ledger master file
C. Write off accounts receivable R. General journal
D. To warehouse and shipping department S. Master price file
E. Authorize account receivable write­off T. Sales journal
F. Prepare aged trial balance U. Sales invoice
G. To sales department V. Cash receipts journal
H. Release goods for shipment W. Uncollectible accounts file
I. To accounts receivable department X. Shipping file
J. Enter price data Y. Aged trial balance
K. Determine that customer exists Z. Open order file
L. Match customer purchase order with sales order 
M. Perform customer credit check 
N. Prepare sales journal 
O. Prepare sales invoice 

Task­Based Simulation 5
Research
Authoritative
Literature Help

Internal Control Reporting
The president of Welcore Inc., a nonpublic audit client of your firm, has indicated to you that he believes that
he may “take the company public in a year or two.” He asked you whether, in addition to your report on the
financial staments, your firm can issue a report on internal control that he could make available to any individual or
organization he so desired. He indicated that he would like to have two reports—as do public companies—one on
the financial statements and one on the company’s internal control.
Selections
A. AU 
B. PCAOB 
C. AT 
D. AR 
E. ET 
F. BL 
G. CS 
H. QC 

(A) (B) (C) (D) (E)  (F) (G) (H)
1. Which title of the Professional Standards addresses this issue and will be
helpful in responding to him?
2. Enter the exact section number that provides the appropriate guidance.

Task­Based Simulation 6
Purchasing Receiving,
and Accounts Payable Authoritative
Literature Help

The following flowchart depicts the activities relating to the purchasing, receiving, and accounts payable 
departments of Model Company, Inc. Assume that you are a supervising assistant assigned to the Model 
Company audit.
88 MODULE 3  UNDERSTANDING INTERNAL CONTROL AND ASSESSING CONTROL RISK

PURCHASIN RECEIVIN
G G ACCOUNTS PAYABLE
From From
Fro
Dept. From From m Vendor
Head Purchasing Purchasing Receiving

4
Purchase Order
1 2
with Quantity Requisition Rec. Report
Approved P.O.3 Invoice
Blacked­Out
Requisition
Form in

Duplicate

Goods Rec'd
Matches 4
Assures Best & Counted By Vendor Doc
Price is Independently Pending &
Recomputes
Obtained & in Secure Invoice
Receipt Math
Request is Facility Accuracy
of
within Budget Invoices
Limits

Prepares Approves & Compares Invoice


Verifies Adequacy
Prepares 3­ Quantity
4­Copy Copy &
of Vendor's Past
Price to
Receiving Voucher P.O.
Record &
& Rec.
Prepares 5­ Report (Prenumbered) Report
Copy Purchase Quantity
Order

Rec. 
4
6 Report
2
Requisition 4 Rec. Report
P.O. 5 P.O. Vo 3 1
uch
er Vouche Requisition
3
P. Rec. Report r
2
O. P.O.
2
P.O.3 Rec. Report Invoice
1
Requisition Receiving
1
Report

Vou
cher
2 1
P.O. (Prenumbered) To
Purchase Order1 Purchasing
(Prenumbered) By P.O.# To
Recorded in Voucher
General
Register &
Acctg. Independently
To Reconciled Monthly
Accounts to Control Accounts
Payable
To
Vendor
To Dept. By P.O.#
To To Head for
Accounts Count & By
Receiving Quality
Payable Dept. Check
Voucher#

To
Dept.
Head

Joe Werell, a beginning assistant, analyzed the flowchart and has supplemented the flowchart by making certain inquiries 
of the controller. He has concluded that the internal control over purchasing, receiving, and accounts payable is strong and has 
provided the following list of what he refers to as internal control strengths. Review his list and for each internal control 
strength indicate whether you agree or disagree that each represents a strength.

I
nP
r
P

1. T
h
2. P
r
3. P

4. P
u
5. T

6. S

7. R
e
8. A

9. T
h
10. R
A
g
MODULE 3  UNDERSTANDING INTERNAL CONTROL AND ASSESSING CONTROL RISK 89

13. A
l
14. A
l
15. T
Task­Based Simulation 7
Research
Authoritative
Literature Help

Internal Control Communications
On the Adams audit your firm discovered certain immaterial audit judgments that the client’s management chose 
not to record. Management has now asked you not to communicate anything regarding these omitted entries to the audit 
committee—after all, they are immaterial.
Selections
A. AU 
B. PCAOB 
C. AT 
D. AR 
E. ET 
F. BL 
G. CS 
H. QC 

(A) (B) (C) (D) (E)  (F) (G) (H)
1. Which title of the Professional Standards addresses this issue?
2. Enter the exact section number and paragraphs that address communication 
of these uncorrected misstatements to the audit committee.

Task­Based Simulation 8
Internal Control Related
Matters Communication Authoritative
Literature Help

You have been asked to the audit partner to draft a letter to the client on internal control related matters. You were 
informed that the written communication regarding significant deficiencies and material weaknesses indentified during an 
audit of financial statements should include certain statements.
For each of the significant deficiencies and material weaknesses reflected in the table below, double­click on each of
the associated shaded cells and select from the list provided the appropriate disposition of each statement in regard to the
letter to the client on internal control related matters. Each selection may be used once, more than once, or not at all.
Selection List
• Included 
• Excluded 
• Included, but only with client management’s approval 
• Communicated orally with no need to document the communication 
90 MODULE 3  UNDERSTANDING INTERNAL CONTROL AND ASSESSING CONTROL RISK

Internal controls Related matters
State that the purpose of the audit was to express an
opinion on the financial statements, and to express an
opinion on the effectiveness of the entity’s internal
control over financial reporting.
Identify, if applicable, items that are considered to be
material weaknesses.
State that the author is not expressing an opinion on the
effectiveness of internal control.
Include the definition of the term significant deficiency.
Include the definition of the term material weakness,
where relevant.
State that the author is expressing an unqualified opinion
on the effectiveness of internal control.
State that the communication is intended solely for
management and external parties.
Identify the matters that are considered to be significant
deficiencies.

Task­Based Simulation 9
Research
Authoritative
Literature Help

Integrated audits
Assume that you are assigned to the audit of Regis Corporation, an issuer company. Your firm is performing its first 
integrated audit for the company, and the partner on the engagement has asked you to research professional standards to 
identify the controls that address the risk of fraud.
Selections
A. AU 
B. PCAOB 
C. AT 
D. AR 
E. ET 
F. BL 
G. CS 
H. QC 
(A) (B) (C) (D) (E)  (F) (G) (H)
1. Which title of the Professional Standards addresses this issue?
2. Enter the exact section and paragraph with helpful information.
MODULE 3 UNDERSTANDING INTERNAL CONTROL AND ASSESSING CONTROL RISK 91

MULTIPLE­CHOICE ANSWERS

1. a __ __ 35. d __ __ 69. c __ __ 103. c __ __ 137. d __ __


2. d __ __ 36. a __ __ 70. a __ __ 104. c __ __ 138. b __ __
3. b __ __ 37. b __ __ 71. d __ __ 105. c __ __ 139. b __ __
4. b __ __ 38. d __ __ 72. a __ __ 106. c __ __ 140. b __ __
5. c __ __ 39. c __ __ 73. a __ __ 107. b __ __ 141. b __ __
6. b __ __ 40. a __ __ 74. c __ __ 108. b __ __ 142. b __ __
7. d __ __ 41. c __ __ 75. b __ __ 109. a __ __ 143. b __ __
8. a __ __ 42. a __ __ 76. a __ __ 110. a __ __ 144. b __ __
9. d __ __ 43. b __ __ 77. a __ __ 111. c __ __ 145. a __ __
10. a __ __ 44. a __ __ 78. c __ __ 112. c __ __ 146. a __ __
11. d __ __ 45. c __ __ 79. d __ __ 113. a __ __ 147. d __ __
12. b __ __ 46. d __ __ 80. b __ __ 114. c __ __ 148. a __ __
13. a __ __ 47. a __ __ 81. a __ __ 115. a __ __ 149. c __ __
14. a __ __ 48. d __ __ 82. a __ __ 116. c __ __ 150. d __ __
15. b __ __ 49. d __ __ 83. d __ __ 117. c __ __ 151. b __ __
16. b __ __ 50. d __ __ 84. c __ __ 118. a __ __ 152. b __ __
17. b __ __ 51. a __ __ 85. d __ __ 119. d __ __ 153. a __ __
18. a __ __ 52. d __ __ 86. b __ __ 120. d __ __ 154. d __ __
19. a __ __ 53. d __ __ 87. c __ __ 121. a __ __ 155. d __ __
20. d __ __ 54. b __ __ 88. b __ __ 122. a __ __ 156. c __ __
21. b __ __ 55. c __ __ 89. b __ __ 123. b __ __ 157. d __ __
22. b __ __ 56. a __ __ 90. d __ __ 124. d __ __ 158. b __ __
23. c __ __ 57. a __ __ 91. b __ __ 125. d __ __ 159. d __ __
24. d __ __ 58. a __ __ 92. d __ __ 126. d __ __ 160. a __ __
25. d __ __ 59. c __ __ 93. b __ __ 127. d __ __ 161. d __ __
26. d __ __ 60. a __ __ 94. b __ __ 128. b __ __ 162. a __ __
27. c __ __ 61. a __ __ 95. d __ __ 129. a __ __ 163. a __ __
28. d __ __ 62. b __ __ 96. d __ __ 130. c __ __ 164. a __ __
29. d __ __ 63. c __ __ 97. b __ __ 131. d __ __ 165. a __ __
30. a __ __ 64. a __ __ 98. b __ __ 132. c __ __ 166. d __ __
31. c __ __ 65. b __ __ 99. d __ __ 133. c __ __
32. d __ __ 66. b __ __ 100. c __ __ 134. c __ __
33. d __ __ 67. c __ __ 101. d __ __ 135. d __ __ 1st:  __/166= __%
34. a __ __ 68. c __ __ 102. d __ __ 136. c __ __ 2nd: __/166= __%

MULTIPLE­CHOICE ANSWER EXPLANATIONS
because ongoing monitoring involves assessing the design 
A.1.  Definition of Internal Control and operation of controls on a timely basis and taking 
1. (a) The requirement is to identify the reply that most likely  neces­sary corrective actions and such an approach may be 
fol­lowed in reviewing the purchasing function. Answer (a) 
would not be considered an inherent limitation of the 
is incorrect because periodic audits are not ordinarily per­
potential effectiveness of an entity’s internal control. An­
formed by the audit committee, a subcommittee of the 
swer (a) is correct because incompatible duties may gener­ Board of Directors. Answer (c) is incorrect because the 
ally be divided among individuals in such a manner as to  audit of the annual financial statements is not ordinarily 
control the problem. Answers (b), (c), and (d) are all incor­ considered monitoring as presented in the professional 
rect because management override, mistakes of judgment,  standards. An­swer (d) is incorrect because the meaning of 
and collusion among employees are all inherent limitations  the reply, con­trol risk assessment in conjunction with 
of internal control.  quarterly reviews, is uncertain.
2. (d) The requirement is to identify the meaning of the  5. (c) The requirement is to identify where the 
concept of reasonable assurance. Answer (d) is correct be­ overall attitude and awareness of an entity’s board
cause reasonable assurance recognizes that the cost of inter­ of directors con­cerning the importance of internal
nal control should not exceed the benefits expected to be  control is normally re­flected. Answer (c) is 
derived.  correct because the control environ­ment reflects 
3. (b) The requirement is to identify the functions  the overall attitude, awareness, and actions of the 
that should be segregated for effective internal  board of directors, management, owners, and 
control. An­swer (b) is correct because  others con­cerning the importance of control and 
authorizing transactions, re­cording transactions,  its emphasis in the entity. 
and maintaining custody of assets should be  6. (b) The requirement is to identify the 
segregated.  circumstance in which management philosophy 
A.2.  Major Components of Internal Control and operating style would have a significant 
influence on an entity’s control environ­ment. 
4. (b) The requirement is to identify the most likelytype  Answer (b) is correct because management 
of ongoing monitoring activity. Answer (b) is correct philoso­phy and operating style, while always 
important, is particu­ 
92 MODULE 3  UNDERSTANDING INTERNAL CONTROL AND ASSESSING CONTROL RISK
have been implemented. Answer (a) is 
incorrect because reperforming the activities 
larly so when management is dominated by one or a  is a test of control to help assess the 
few individuals because it may impact numerous  operating effectiveness of a con­trol. Answer
other factors. Answer (a) is incorrect because the  (b) is incorrect because analytical procedures
impact of the internal auditor reporting directly to  are not performed to determine whether 
management is likely to be less than that of answer  controls are suitably designed. Answer (c) is 
(a). Answer (c) is incorrect because while accurate  incorrect because vouching a sam­ple of 
management job descriptions are desirable, they do  transactions is a substantive test not directly 
not have as significant of an effect on management  aimed at determining whether controls are 
philosophy and operating style as does domination by suitably designed. See SAS 109 for a 
an individual. Answer (d) is incorrect because an  discussion of an auditor’s responsibility for 
active audit committee might temper rather than lead  determining whether controls have been 
to a more signifi­cant influence of management  implemented vs. their operating 
philosophy and operating style. effectiveness. 
7. (d) The requirement is to determine which of the 
factors listed are included in an entity’s control 
environment. Answer (d) is correct because the audit 
committee, integrity and ethical values, and 
organization structure are all in­cluded. 
8. (a) The requirement is to identify the reply 
that is not a component of an entity’s 
internal control. Answer (a)is correct 
because while auditors assess control risk as 
a part of their consideration of internal 
control, it is not a compo­nent of an entity’s 
internal control. Answers (b), (c), and (d) are
incorrect because the control environment, 
risk assess­ment, control activities, 
information and communication, and 
monitoring are the five components of an 
entity’s inter­nal control (SAS 109). 
A.3.  Related Topics
9. (d) The Foreign Corrupt Practices Act makes pay­
ment of bribes to foreign officials illegal and requires
pub­licly held companies to maintain systems of 
internal control sufficient to provide reasonable 
assurances that internal control objectives are met.
B.1.  Obtain Understanding of the Client and Its
Internal Control
10. (a) The requirement is to determine the most 
likely population from which an auditor would 
sample when ven­dors’ invoices and related 
vouchers relating to purchases made by 
employees have been destroyed. Answer (a) is 
correct because the disbursement will be 
recorded and the auditor may thus sample from 
that population. Answers (b) and (d) are 
incorrect because the related vouchers and ven­
dors’ invoices are destroyed. Answer (c) is 
incorrect be­cause there is no recording of the 
receipt of the merchandise. 
11. (d) The requirement is to identify the 
procedure an auditor would perform to 
provide evidence about whether an entity’s 
internal control activities are suitably 
designed to prevent or detect material 
misstatements. Answer (d) is correct because
SAS 109 indicates an auditor will observe 
the entity’s personnel applying the 
procedures to determine whether controls 
whether the client has placed those controls in 
• (b) The requirement is to identify the correct state­ operation. Answers (b) and (c) are incorrect 
ment relating to the relevance of various types of  because auditors may choose not to obtain 
controls to a financial statement audit. Answer (b) is  infor­mation on operating effectiveness of 
correct because, generally, controls that are relevant to controls and their con­sistency of application. 
an audit pertain to the entity’s objective of preparing  Answer (d) is incorrect because there is no 
financial statements for ex­ternal purposes. Answer (a) such explicit requirement relating to controls; 
is incorrect because SAS 109 makes clear that an  see SAS 109 for the necessary understanding of
auditor may not ignore consideration of controls under internal control. 
any audit approach. Answer (c) is incorrect because 
control over financial reporting are of primary im­ • (b) SAS 109 states that the auditor should 
portance. Answer (d) is incorrect because many  obtain sufficient knowledge of the 
operational and compliance related controls are not  information (including account­ing) system 
ordinarily relevant to an audit.  to understand the financial reporting process
used to prepare the entity’s financial 
• (a) The requirement is to identify the statement  statements, including significant accounting
that represents a requirement when an audit  estimates and disclosures. It also states that 
of financial state­ments in accordance with  this knowledge is obtained to help the 
generally accepted accounting principles is  auditor to understand (1) the entity’s classes
performed. Answer (a) is correct because  of transactions, (2) how transactions are 
SAS 109 requires that the auditor document  initiated, (3) the accounting records and 
the understand­ing of the entity’s internal  support, and (4) the accounting processing 
control. Answer (b) is incorrect because  involved from initiation of a transaction to 
while an auditor might find significant  its inclusion in the financial statements. 
deficiencies in the operation of internal 
control, no such search is re­  • (b) The requirement is to determine why an 
quired. Answer (c) is incorrect because an auditor  auditor should concentrate on the substance of 
might use a substantive approach in performing an  procedures rather than their form when 
audit and thereby perform few (if any) tests of  obtaining an understanding of an en­tity’s 
controls. Answer (d) is incor­rect because while  controls. Answer (b) is correct because 
auditors must obtain knowledge of inter­nal control  management may establish appropriate 
sufficiently to identify types of potential mis­ controls but not act on them, thus creating a 
statements, they are not required to obtain the detailed  situation in which the form differs from the 
knowledge of internal control suggested by this reply.  sub­stance. Answer (a) is incorrect because 
documentation is not directly related to the 
• (a) The requirement is to identify the knowledge 
issue of substance over form. An­swer (c) is 
that an auditor must obtain when obtaining an 
incorrect because inappropriate controls is 
understanding of an entity’s internal control 
only a part of an auditor concern; for example, 
sufficient for audit planning. Answer (a) is 
a control may be appropriate, but it may not be
correct because an auditor must obtain an un­
operating effectively. An­swer (d) is incorrect 
derstanding that includes knowledge about the 
because while an auditor might suggest 
design of relevant controls and records and 
MODULE 3  UNDERSTANDING INTERNAL CONTROL AND ASSESSING CONTROL RISK 93
client’s internal control. An auditor may 
docu­ment his/her understanding of the 
to   management   that   the   cost   of   certain   controls structure and his/her con­clusions about the 
seems to exceed their likely benefit, this is not the design of that structure in the form of 
primary   reason   auditors   are   concerned   with   the answers to a questionnaire, narrative 
substance of controls. memorandums, flow­charts, decision tables,
or any other form that the auditor considers 
17. (b) Decision tables include various  appropriate in the circumstances. Answers 
combinations of conditions that are matched to  (a) and (b) are, thus, incorrect because they 
one of several actions. In an internal control  suggest restrictions which do not exist in 
setting, the various important controls are  practice. Answer (c) is incorrect since at a 
reviewed and, based on the combination of  minimum a list of reasons for nonreliance 
answers re­ceived, an action such as a decision  must be pro­vided. 
on whether to perform tests of controls is 
determined. Program flowcharts simply  21. (b) In obtaining an understanding of internal
summarize the steps involved in a program.  con­trol, the auditor should perform 
Answer (a) is incorrect because decision tables  procedures to provide suffi­cient knowledge
do not emphasize the ease of manageability for  of the design of the relevant controls and 
complex programs. Answer (c) is in­correct  whether they have been implemented. 
because while decision tables may be designed  Information on 
using various cost benefit factors relating to the
various conditions and actions, they do not 
justify the program. Answer (d) is incorrect 
because program flowcharts, not decision 
tables, emphasize the sequence in which 
operations are performed. 

18. (a) The requirement is to identify the procedure that is 
not required to be included in an auditor’s 
consideration of internal control. Answer (a) is correct 
because the auditor need not obtain evidence relating 
to operating effectiveness when control risk is to be 
assessed at the maximum level. Answer (b) is incorrect
because an auditor must obtain an understanding of the
internal control environment and the information 
system. Answers (c) and (d) are incorrect be­cause an 
auditor is obligated to obtain information on the design
of internal control and on whether control activities 
have been implemented. 
19. (a) The requirement is to identify the primary
objec­tive of procedures performed to obtain 
an understanding of internal control. Answer 
(a) is correct because the auditor obtains a 
sufficient understanding of internal control to
assess the risks of material misstatement and 
to design the nature, timing, and extent of 
further audit procedures. Answer (b) 
addresses inherent risk, the susceptibility of 
an assertion to material misstatement, 
assuming that there are no related controls. 
Answer (b) is incorrect since the concept of 
inherent risk assumes no internal control and 
is therefore not the primary objective. 
Answer (c) is incorrect because answer (a) is 
more complete and because decisions on 
modifying tests of controls are often made at 
a later point in the audit. Answer (d) is 
incorrect because the consistency of 
application of management’s policies relates 
more directly to tests of controls than to 
obtaining an understanding of internal 
control. 
20. (d) The requirement is to determine the 
correct statement with respect to the 
auditor’s required documenta­tion of the 
(1) identifying specific controls relevant to specific
operating effectiveness need not be obtained unless  asser­tions   that   are   likely   to   prevent   or   detect
control risk is to be assessed at a level below the  material misstate­ments in those assertions, and (2)
maximum. performing   tests   of   con­trols   to   evaluate   the
effectiveness of such controls.
B.2. Assess the Risk of Material  Answer (a) is incorrect because assessing control risk 
Misstatement and Design Further Audit at a low level may lead to less extensive, not more 
Procedures extensive substantive tests. Answer (b) is incorrect 
because the actual level of inherent risk is not 
22. (b)   Answer   (b)   is   correct   because   tests   of affected by the level of control risk. Also, one would 
controls are only required when the auditor not expect a change in the assessed level of control 
relies   on   the   controls   or   substantive   tests risk to result in a change in the assessed level of 
alone   are   not   sufficient   to   audit   particular inherent risk. Answer (c) is incorrect because as­
assertions.   Answers   (a),   (c),   and   (d)   are sessing control risk at a low level may lead to interim­
incorrect   because   these   procedures   are date substantive testing rather than year­end testing.
required on every audit. 
26. (d) The requirement is to determine why an 
23. (c) The requirement is to identify the terms in which  auditor assesses control risk. Answer (d) is 
control risk should be assessed. Answer (c) is correct  correct because the as­sessed levels of 
be­cause SAS 109 requires that control risk be  control risk and inherent risk are used to 
assessed in terms of financial statement assertions.  determine the acceptable level of detection 
risk for financial statement assertions. 
24. (d) The requirement is to identify a situation
in which an auditor may desire to seek a  27. (c) Increases in the assessed level of the risk 
further reduction in the assessed level of  of material misstatement lead to decreases in
control risk. Answer (d) is correct be­cause  the acceptable level of detection risk. 
such a reduction is only possible when  Accordingly, the auditor will need to 
additional evi­dence, evaluated by  increase the extent of substantive tests such 
performing additional tests of controls, is  as tests of details. Answer (a) is incorrect 
available. Answer (a) is incorrect because  because tests of controls are performed to 
auditors at this point will ordinarily already  reduce the assessed level of control risk only
have obtained the understand­ing of the  when controls are believed to be effective. 
information system to plan the audit.  Answer (b) is incorrect because the level of 
Further­more, an understanding of internal  detection risk must be decreased, not 
control is needed on all audits. Answer (b) is increased. Answer (d) is incorrect because 
incorrect because auditors must de­termine  the level of inherent risk pertains to the 
that controls have been implemented in all  susceptibility of an account to material 
audits. Answer (c) is incorrect because a  misstatement independent of related 
significant number of controls always  controls. 
pertain to financial statement assertions. 
28. (d) The requirement is to determine the 
25. (d)       Assessing   control   risk   at   a   low   level primary purpose for which an auditor uses 
involves  the knowledge provided by the understanding
of internal control and the assessed 
94 MODULE 3  UNDERSTANDING INTERNAL CONTROL AND ASSESSING CONTROL RISK
control risk is assessed at the maximum level. Answer 
(b) is incorrect because analytical procedures are not 
level of the risk of material misstatement. Answer (d)  designed to verify the design of internal control. 
is correct because the auditor uses such knowledge in  Answer (d) is incorrect because dual­purpose tests 
determining the nature, timing, and extent of  (i.e., those that serve as both substantive tests and tests
substantive tests for financial statement assertions.  of controls) are not required to be performed, and 
Answer (a) is incorrect because it is incomplete. For  because the term “preliminary control risk” is unclear.
example, while auditors are concerned with the 
safeguarding of assets, they also need to determine 
whether the financial statement information is 
accurate. Answer (b) is incorrect for reasons similar 
to (a) in that determining whether opportunities are 
available for committing and concealing fraud is 
incomplete since this knowledge is also used to 
ascertain whether the chance of errors is minimized. 
Answer (c) is incorrect because knowledge provided 
by the understanding of internal control and the 
assessed level of control risk is not used to modify 
initial assessments of inherent risk and preliminary 
judgments about materiality levels. This knowledge is
unrelated to those processes.
29. (d) The requirement is to identify a way that anauditor 
may compensate for a weakness in internal control. 
Answer (d) is correct because increasing analytical 
proce­dures decreases detection risk in a manner 
which may counterbalance the condition in internal 
control. In effect, the weakness in internal control is 
compensated for by in­creased substantive testing. See 
the outline of SAS 107 for the relationships among 
audit risk and its component risks— inherent risk, 
control risk, and detection risk. Answer (a) is incorrect
because increasing both control risk (through a 
weakness in internal control) and detection risk 
increases audit risks. In addition, control risk and 
detection risk do not compensate for one another. 
Answer (b) is incorrect because increasing the extent 
of tests of controls is unlikely to be effective since the 
condition is known to exist. An­swer (c) is incorrect 
because it is not generally appropriate to increase the 
judgment as to audit risk based on the results obtained.
30. (a) The requirement is to identify the correct statement
concerning an auditor’s assessment of control risk. 
Answer (a) is correct because SAS 109 indicates that 
assessing control risk may be performed concurrently 
during an audit with obtaining an understanding of 
internal control. Answer (b) is incorrect because 
evidence about the operation of internal control 
obtained in prior audits may be considered during the 
current year’s assessment of control risk. Answer (c) is
incorrect because the basis for an auditor’s conclusions
about the assessed level of control risk needs to be 
documented when control risk is assessed at 
levels other than the maximum level. Answer (d) is 
incor­rect because a lower level of control risk 
requires more as­surance that the control procedures 
are operating effectively.
31. (c) The requirement is to determine the 
correctstatement concerning the assessed level of 
control risk. An­swer (c) is correct because ordinarily 
the assessed level of control risk cannot be 
sufficiently low to eliminate the need to perform any 
substantive tests to restrict detection risk for 
significant transaction classes. Answer (a) is incorrect 
be­cause tests of controls are unnecessary when 
duties) an auditor should use the 
B.3.  Perform Further Audit Procedures—Tests of observation and inquiry techniques. 
Controls
35. (d) The requirement is to identify the objective of tests
32. (d) The requirement is to identify how  of details of transactions performed as tests of 
frequently controls that the auditor wishes to  controls. Answer (d) is correct because the purpose of 
rely upon must be tested. Answer (d) is  tests of controls is to evaluate whether internal control 
correct because the professional standards  operates effectively. Answer (a) is incorrect because 
require auditors to test controls at least every while monitoring the design and use of entity 
third year. Answer (a) is incorrect because  documents may be viewed as a test of con­trols, it is 
controls need not be tested monthly. Answer  not the objective. Answer (b) is incorrect because 
(b) is incorrect because ordinarily controls  determining whether internal control is implemented is
need not be tested with each audit. Answer  not directly related to tests of controls; see SAS 109 
(c) is incorrect because auditors may test  for the distinction between “implemented” and 
such controls every third year, not every  “operating effec­tiveness.” Answer (c) is incorrect 
second year.  because substantive tests, not tests of controls, are 
focused on detection of material misstatements in the 
33. (d) The requirement is to identify the type of account balances of the financial statements. 
audit test most likely to provide assurance 
that controls are in use and operating  36. (a) The requirement is to identify a 
effectively. Answer (d) is correct because  circumstance in which an auditor may 
inspection of documents is a form of a test of decide to perform tests of controls. Answer 
controls, and such tests are used to obtain  (a) is correct because tests of controls will 
reasonable assurance that con­trols are in use be per­formed when they are expected to 
and operating effectively. Answer (a) is in­ result in a cost effective reduction in planned
correct because auditors prepare flowcharts  substantive tests. Answer (b) is incor­rect 
to document a company’s internal control,  because tests of controls are only performed 
not to obtain assurance that controls are in  when they are likely to support a further 
use and operating effectively. Answer (b) is  reduction in the assessed level of the risk of 
incorrect because substantive tests relate to  material misstatement. Answer (c) is 
the accuracy of accounts and assertions  incorrect because tests of controls are 
rather than testing controls directly. Answer  designed to decrease the assessed level of 
(c) is incorrect because analyzing tests of  the risk of material misstatement, not 
trends and ratios is an analytical procedure  increase it. Answer (d) is incorrect because 
that does not directly test controls.  internal control weaknesses normally result 
in more substantive testing and less tests of 
34. (a) The requirement is to identify the  controls. 
appropriate procedures for testing the 
segregation of duties related to inventory.  37. (b) The requirement is to identify the most 
Answer (a) is correct because SAS 109  appropri­ate procedures for assessing control 
suggests that when no audit trail exists (as  risk. Auditors perform tests of controls to 
is often the case for the segregation of  obtain evidence on the operating effec­ 
MODULE 3  UNDERSTANDING INTERNAL CONTROL AND ASSESSING CONTROL RISK 95
operating effectively. Answer (c) is correct 
because confirmation of accounts receivable
tiveness of controls to assess control risk. Answer (b)  is a substantive test, not a test of a control. 
is correct because tests of controls include inquiries of Answer (a) is incor­rect because records 
appro­priate entity personnel, inspection of documents documenting the usage of computer 
and reports, observation of the application of the  programs may be tested to determine 
policy or procedure, and reperformance of the  whether access is ap­propriately controlled. 
application of the policy or proce­dure. Answer (b) is incorrect because ex­amining 
38. (d) The requirement is to identify the type of evi­dence canceled supporting documents may help 
an auditor most likely would examine to determine  the audi­tor to determine that the structure 
whether controls are operating as designed. Answer (d) will not allow duplicate billing to result in 
is correct because inspection of client records  multiple payments. Answer (d) is incor­rect 
documenting the use of computer programs will  because proper signatures will help the 
provide evidence to help the auditor evaluate the  auditor to de­termine whether the 
effectiveness of the design and opera­tion of internal  authorization controls are functioning 
control; the client’s control over use of its computer  adequately. 
programs in this case is documentation of the use of 
the programs. In order to test this control, the auditor 
will inspect the documentation records. See SAS 110 
for infor­mation on the nature of tests of controls. 
Answer (a) is in­correct because the confirmation 
process is most frequently considered a substantive 
test, not a test of a control. An­swer (b) is incorrect 
because letters of representations pro­vide 
corroborating information on various management rep­
resentations obtained throughout the audit and 
therefore only provides limited evidence on internal 
control. Answer (c) is incorrect because attorneys’ 
responses to auditor inquiries most frequently pertain 
to litigation, claims, and assess­ments. 
39. (c) The requirement is to identify the 
procedure that is not a step in an auditor’s 
assessment of control risk. An­swer (c) is 
correct because performing tests of details of
transactions to detect material misstatements 
pertains more directly to detection risk rather
than inherent or control risk. Answer (a) is 
incorrect because auditors evaluate the effec­
tiveness of internal control with tests of 
controls. An­ 
swer (b) is incorrect because obtaining an 
understanding of the entity’s information system and 
control environment is a preliminary step for 
considering control risk. Answer (d) is incorrect 
because auditors will consider the effect of internal 
control on the various financial statement assertions. 
40. (a) The requirement is to identify an 
approach that auditors use to obtain audit 
evidence about control risk. Answer (a) is 
correct because auditors test controls to pro­
vide evidence for their assessment of control
risk through inquiries of appropriate 
personnel, inspection of documents and 
records, observation of the application of 
controls, and reperformance of the 
application of the policy or procedure. 
Answers (b), (c), and (d) are incorrect 
because analytical procedures, calculation, 
and confirmation relate more di­rectly to 
substantive testing and are not primary 
methods to test controls for purposes of 
assessing control risk. 
41. (c) The requirement is to identify the least 
likely type of evidence the auditor will 
examine to determine whether controls are 
its internal control assessment. Answer (a) is 
42. (a) The requirement is to identify the accounts re­ correct as the COSO internal control 
ceivable auditing procedure that an auditor would most framework is by far the most frequently used 
likely perform to obtain support for an assessed level  one. Answer (b) is incorrect because while a 
of control risk below the maximum. Since an auditor  COSO enterprise risk management framework 
uses the results of tests of controls to support an  does exist, it is not ordinarily used by 
assessed level of control risk below the maximum, we  management in its internal control assessment. 
are attempting to iden­tify a test of a control. Answer  An­swers (c) and (d) are incorrect because 
(a) is correct because ob­serving an entity’s employee  there is no such thing as a “FASB 37 internal 
prepare the schedule of past due accounts receivable is control definitional framework” or an “AICPA 
a test of a control to evaluate the effectiveness the  internal control analysis manager.” 
process of preparing an accurate schedule of past due 
accounts; if the process is found to be effective it may  C.2.  Management’s Assessment
lead to a reduction in the assessed level of control risk.
Answer (b) is incorrect because the confirmation of  45. (c) The requirement is to identify the correct
accounts receivable is a substantive test, not a test of a  state­ment concerning the level of assistance
control. An­swer (c) is incorrect because the inspection that auditors may provide in assisting 
of accounts re­ceivable for unusual balances and  management with its assessment of in­ternal
comparing uncollectible accounts expense to actual  control. Answer (c) is correct since only 
accounts expense is ordinarily an analytical procedure  limited as­sistance may be provided so as 
performed as a substantive test. An­swer (d) is  not to create a situation in which the 
incorrect because comparing uncollectible ac­counts  auditors are auditing their own work. 
expense to actual uncollectible accounts is ordinarily a  Answer (a) is incorrect since some 
substantive test. assistance may be provided. An­swer (b) is 
incorrect because there are limitations on 
C.1.  Management’s Responsibility the level of assistance. Answer (d) is 
incorrect because the tie between risk and 
43. (b) The requirement is to identify the type of  assistance seems inappropriate and in the 
companies to which the internal control provisions of  wrong direction; also, this type of tradeoff 
the Sarbanes­Oxley Act of 2002 apply. Answer (b) is  between risk and assistance is not included 
correct because the provisions apply to public  in PCAOB Standard 5. 
companies that are registered with the Securities and 
Exchange Commission. Answer (a) is incorrect  46. (d) The requirement is to identify which of 
because nonissuer companies are not directly affected  the fol­lowing need not be included in 
by the control provisions. Answer (c) is incorrect;  management’s report on in­ternal control 
there is no $100,000,000 requirement. Answer (d) is  under Section 404a of the Sarbanes­Oxley 
incorrect because nonissuer companies are not directly Act of 2002. Answer (d) is correct because, 
affected by the internal control provisions of the Act.  while the report must indicate that it is 
management’s responsibility to es­tablish 
44. (a) The requirement is to identify the most  and maintain adequate internal control, it 
likely framework to be used by management in  need not 
96 MODULE 3  UNDERSTANDING INTERNAL CONTROL AND ASSESSING CONTROL RISK
prevents errors or fraud from occurring. Answer (c) is 
incorrect because the term “adjustive” control is not 
also indicate that such control has no significant  ordinarily used. Answer (d) is incorrect because 
deficien­cies. Answers (a), (b), and (c) are all incorrect “nonroutine” is ordinarily considered a type of 
because they include information that must be  transaction (e.g., the year­end close process), not a 
contained in management’s report. type of control. 
C.3.  Control Definitions 52. (d) The requirement is to identify the type 
of transaction that establishing loan loss 
47. (a) Answer (a) is correct because all material
reserves is. An­swer (d) is correct because 
weaknesses are control deficiencies. Answer  estimation transactions are ac­tivities 
(b) is incorrect because a significant  involving management’s judgments or 
deficiency may or may not be a material  assumptions, such as determining the 
weakness. Answer (c) is incorrect because  allowance for doubtful accounts, 
not all control deficiencies are material 
weaknesses. Answer (d) is incorrect because 
only significant deficiencies and material 
weaknesses must be communicated. 
48. (d) The requirement is to identify the term 
that is defined as a weakness in internal 
control that is less severe than a material 
weakness but important enough to warrant 
attention by those responsible for oversight 
of the financial reporting function. Answer 
(d) is correct because this is the definition of
a significant deficiency. Answer (a) is incor­
rect because a control deficiency exists 
when the design or operation of a control 
does not allow management, or em­ployees, 
in the normal course of performing their 
functions to prevent or detect misstatements 
on a timely basis. An­swer (b) is incorrect 
because an unusual weakness is not used in 
the standards for integrated audits. Answer 
(c) is incorrect because the term unusual 
deficiency is not used in the standards for 
integrated audits. 
49. (d) The requirement is to identify the 
amount in­volved with a material weakness.
Answer (d) is correct be­cause a material 
amount is involved. Answers (a), (b), and 
(c)   are   all   incorrect   because   they   suggest   smaller
amounts. 
50. (d) The requirement is to identify the minimum 
likelihood of loss involved in the consideration of a 
control deficiency. Answer (d) is correct because a 
control defi­ciency is a condition in which the 
operation of a control does not allow management, or 
employees, in the normal course of performing their 
functions to prevent or detect misstate­ments on a 
timely basis—it does not explicitly consider likelihood
of loss. Answer (a) is incorrect because the minimum 
likelihood of loss is not considered. Answer (b) is 
incorrect because the control deficiency occurrence of 
loss need not be more than remote. Answer (c) is 
incorrect be­cause whether the minimum likelihood of 
loss is probable is not considered. 
C.4.  Evaluating Internal Control
51. (a) The requirement is to identify the type of control 
that reconciliation of cash accounts represents. Answer
(a) is correct in that it is a compensating control which 
supple­ments a basic underlying control, in this case 
basic informa­tion processing controls related to cash. 
Answer (b) is incor­rect because a preventive control 
appropriate date on which to evaluate inter­
establishing warranty reserves, and assessing assets  nal control. Answer (d) is incorrect because 
for im­pairment. Answer (a) is incorrect because the  the “as of date” is a particular date, not an 
term sub­stantive transaction is not used in PCAOB  average. 
standards. An­swer (b) is incorrect because routine 
transactions are those for recurring activities, such as  55. (c) The requirement is to identify the correct state­
sales, purchases, cash re­ceipts and disbursements, and ment concerning a company that makes purchases 
payroll. Answer (c) is incor­rect because nonroutine  both through the Internet and by telephone. Answer (c)
transactions occur only periodically, such as the taking is correct because both types of purchases are a part of
of physical inventory, calculating depre­ciation  the purchases process and represent major classes of 
expense, or adjusting for foreign currencies; non­ transactions, as per PCAOB Standard 5. Answer (a) is 
routine transactions generally are not a part of the  incorrect because the purchase types themselves are 
routine flow of transactions. not control objectives for internal control (control 
objectives address issues such as the completeness of 
C.5.  The Audit of Internal Control the recording of sales). Answer (b) is in­correct 
53. (d) The requirement is to identify the  because purchases are not assertions. Answer (d) is 
incorrect because purchase transactions may or may 
procedure that involves tracing a transaction 
not be investigated in extreme detail. 
from origination through the company’s 
information systems until it is reflected in the 56. (a) The requirement is to identify the circum­
company’s financial report. Answer (d) is  stance(s) in which walk­throughs provide the
correct because this is the approach followed  auditor with primary evidence. A walk­
in a walk­through. Answer (a) is incorrect  through involves literally tracing a 
because analytical analysis is a general term  transaction from its origination through the 
that simply suggests a general analysis.  company’s information systems until it is 
Answer (b) is incorrect because a substantive reflected in the financial re­ports. Answer (a)
procedure addresses the correctness of a  is correct because a walk­through pro­vides 
particular financial statement amount or  evidence to (1) confirm the auditor’s 
disclosure. Answer (c) is incorrect because a  understanding of the flow of transactions and
test of a control addresses the operating  the design of controls, (2) eval­uate the 
effectiveness of a control.  effectiveness of the design of controls, and 
(3) to confirm whether controls have been 
54. (b) The requirement is to identify the “as of  placed in operation. Answer (b) is incorrect 
date” for purposes of an audit of internal  because walk­throughs provide the auditors 
control performed under PCAOB standards.  with primary evidence to confirm whether 
Answer (b) is correct because the “as of  controls have been placed in operation. 
date” is the last day of the fiscal period; it is  Answer (c) is incorrect be­cause walk­
this date on which the auditor concludes as  throughs provide primary evidence to 
to the effectiveness of inter­nal control.  evaluate the effectiveness of design in 
Answers (a) and (c) are incorrect because  internal control. Answer (d) is incorrect both 
nei­ther the first day of the year nor the last  because walk­throughs provide primary evi­
day of the auditor’s fieldwork is the  dence to (1) evaluate the effectiveness of the 
design of con­ 
MODULE 3  UNDERSTANDING INTERNAL CONTROL AND ASSESSING CONTROL RISK 97
material weakness exists. Answer (d) is incorrect 
because explanatory lan­guage added to an unqualified
trols and (2) confirm whether the controls have been  report is not appropriate when a material weakness 
placed in operation. exists. 
57. (a) The requirement is to identify the most likely  61. (a) The requirement is to identify the 
question to be asked of employee personnel during a  deficiency that is most likely to be 
walk­through. Answer (a) is correct because a question considered a material weakness in inter­nal 
on whether an employee has ever been asked to  control for purposes of an internal control 
override the process is included in the example  audit of a public company. Answer (a) is 
questions to be asked by the auditor. Answer (b) is  correct because ineffective oversight of 
incorrect because auditors do not in general ask  financial reporting by the audit committee is 
whether the employee believes he or she is underpaid.  among the list of circumstances that PCAOB
Answer (c) is incorrect because a direct question on  Standard 5 suggests are strong indicators of 
fraudulent transactions like this, while possible,  the existence of a material weakness. 
ordinar­ily is not suggested. Answer (d) is incorrect  Restatement of previously issued financial 
because the auditor will not usually ask who trained  statements as a result of a change in 
the person. Note that all four questions might be asked, accounting principles is 
but only one is among those recommended in Standard
5. 
58. (a) The requirement is to identify how large 
the actual loss identified must be for a 
control deficiency to pos­sibly be considered 
a material weakness. Answer (a) is cor­rect 
because a material weakness is determined 
by whether there is more than a remote 
likelihood of a material loss oc­curring due 
to the control deficiency; the actual loss 
identi­fied need not be material. Answer (b) 
is incorrect because it suggests that a 
material amount identified will not be con­
sidered a material weakness. Answer (c) is 
incorrect be­cause it states that when the 
identified amount is immaterial it is never a 
material weakness. Answer (d) is incorrect 
be­cause it suggests that when an immaterial 
or material actual loss is discovered, the 
situation would not be assessed as a possible 
material weakness. 
59. (c) The requirement is to identify the 
circumstance that makes an account 
significant for purposes of a PCAOB audit of
internal control. Answer (c) is correct 
because Standard 5 requires only more than a
remote likelihood of material misstatement. 
Answer (a) is incorrect because the standard 
requires only a remote likelihood and 
because it is limited to material 
misstatements. Answer (b) is incorrect 
because the standard requires more than a 
remote likelihood, not more than a 
reasonably possible likelihood. Answer (d) is
incorrect because material misstatements are 
involved, not misstatements that are more 
than inconsequential. 
60. (a) The requirement is to identify the appropriate 
report when a control deficiency that is more than a 
signifi­cant deficiency is identified. Answer (a) is 
correct because a control deficiency that is more than a
significant deficiency is a material weakness, and 
because a material weakness leads to an adverse 
opinion on internal control. Answer (b) is incorrect 
because qualified opinions are not issued when a 
material weakness exists. Answer (c) is incorrect 
because an unqualified opinion is not issued when a 
end, but before the audit report is issued. 
ordinarily not considered even a significant deficiency.  An­swer (a) is correct because PCAOB 
Answer (c) is incorrect because the reply “inadequate  Standard 5 requires an adverse audit report 
segre­gation of recordkeeping from accounting” makes  when a material weakness exists at year­
no real sense because accounting is involved with  end, the “as of date.” Answer (b) is incorrect
recordkeeping. Answer (d) is incorrect because control  because an unqualified opinion is not 
activity weaknesses often do not represent material  appropriate. Answer (c) is incorrect because
weaknesses. an unqualified opinion with explanatory 
language is not adequate. Answer (d) is 
62. (b) The requirement is to identify the most appropri­ incorrect because a qualified opinion is not 
ate report when a circumstance­caused scope  appropriate when a material weak­ness 
limitation re­sults in inability to evaluate internal  exists at year­end. 
control for a significant account involved in the audit. 
Answer (b) is correct because PCAOB Standard 5  65. (b) The requirement is to specify whether material 
indicates that either a qualified opinion or a disclaimer  weaknesses and/or significant deficiencies lead to an 
is appropriate, and because the disclaimer is not listed  adverse opinion on internal control in an integrated audit. 
as an option. Answer (a) is incorrect because an  An­ 
adverse opinion is not appropriate. Answer (c) is  swer (b) is correct because only material weaknesses 
incorrect because an unqualified opinion with  lead to an adverse opinion. Answer (a) is incorrect 
explanatory language is not appropriate when the  because significant deficiencies do not result in an 
auditor is unable to evaluate internal control for a  adverse opinion. Answer (c) is incorrect because 
significant account. Answer (d) is incorrect because  material weaknesses do result in adverse opinions but 
answers (a) and (c) are not appropriate.  significant deficiencies do not. Answer (d) is incorrect
because material weaknesses do result in adverse 
63. (c) The requirement is to identify the most  opinions. 
likely significant deficiency relating to a 
client’s antifraud pro­grams. Answer (c) is  66. (b) The requirement is to specify what types 
correct because an active audit com­mittee,  of defi­ciencies must be communicated by 
not a passive audit committee is needed.  the auditor to the audit committee. Answer 
Answer (a) is incorrect because a broad  (b) is correct because the auditor must 
scope of internal audit activities is ordinarily  communicate material weaknesses and other 
a strength, not a deficiency. Answer (b) is in­ significant de­ficiencies, but not all control 
correct because a whistle­blower program  deficiencies. Answer (a) is incorrect because 
that encourages anonymous submissions is  control deficiencies that are not significant 
required. Answer (d) is incorrect because it is need not be communicated to the audit 
not ordinarily necessary to perform criminal  committee (unless the auditor has made an 
background investigations for likely  agreement to communicate them). Answers 
customers.  (c) and (d) are incorrect because known 
material weaknesses must be communicated 
64. (a) The requirement is to identify the  to the audit committee and control 
appropriate audit report when a material  deficiencies that are not significant 
weakness is corrected subse­quent to year­ deficiencies need not be communicated. 
98 MODULE 3  UNDERSTANDING INTERNAL CONTROL AND ASSESSING CONTROL RISK
over financial reporting, not management 
and the audit commit­tee. Answers (a), (b) 
67. (c) The requirement is to determine the manner in  and (c) are all incorrect because they 
which significant deficiencies are communicated by  represent statements included in the audit 
the auditor to the audit committee under PCAOB  report. 
Standard 5. Answer (c) is correct because the Standard
requires a written communication. Answer (a) is  72. (a) The requirement is to identify the correct
incorrect because a written communication is required. state­ment relating to walk­throughs. 
Answer (b) is incorrect because the communication  Answer (a) is correct be­cause walk­
must be in a written form, not in an oral form. Answer  throughs—literally tracing transactions from
(d) is incorrect because both material weak­nesses and  their origination through the company’s 
significant deficiencies must be communicated.  information system until they are reflected 
in the company’s financial reports— are 
68. (c) The requirement is to identify the correct often effective procedures for evaluating 
state­ment concerning the external auditors’  both the design effectiveness of controls and
use of the work of others when performing  whether those controls have been placed in 
an audit of internal control of a public  operation. Answer (b) is incorrect because 
company. Answer (c) is correct because,  walk­throughs help auditors to confirm 
after assur­ing themselves as to the  whether controls have been placed in 
competence and objectivity of the internal  operation. Answer (c) is incorrect 
auditors and others, the external auditors 
may use their work—particularly in low­risk
areas and when that work is supervised 
and/or reviewed. Answer (a) is incorrect 
because using the work of internal auditors 
and others is allowed. Answer (b) is 
incorrect because there is no such 
requirement of reporting to the audit 
committee, although this is one indication of
internal auditor objective. An­swer (d) is 
incorrect because there are limitations, and 
because it is uncertain whether liability will 
be shared. 
69. (c) The requirement is to identify the 
information that must be communicated in 
writing to management. An­swer (c) is 
correct because in an integrated audit all 
material weaknesses, significant 
deficiencies, and other control defi­ciencies 
must be reported to management. Answers 
(a) and 
(b) are incorrect because they are incomplete. Answer
(d)   is   incorrect   because   “all   suspected   and   possible
employee law violations” need not be communicated. 
70. (a) The requirement is to identify a correct state­ment 
about applying a top­down approach to identify con­
trols to test in an integrated audit. Answer (a) is correct
because certain effective entity­level controls may 
allow the auditor to omit additional testing beyond 
those controls. Answer (b) is incorrect because starting
with assertions does not represent starting at the top 
(starting at the top includes consideration of the 
financial statements and entity­level controls first). 
Answer (c) is incorrect because consideration of 
entity­level controls cannot be avoided. Answer (d) is 
incorrect because not all controls related to assertions 
need to be focused upon, and because one may not 
omit controls related to the financial statements. 
71. (d) The requirement is to identify the 
statement not included in a standard 
unqualified opinion on internal con­trol 
performed under PCAOB requirements. 
Answer (d) is correct because the report 
indicates that management is re­sponsible 
for maintaining effective internal control 
(a) is incorrect because the daily cash summary
because walk­throughs help auditors to evaluate the  will ordinarily be prepared at the end of the day
design effectiveness of controls. Answer (d) is  when all checks have been received. Answer 
incorrect because walk­throughs help auditors  (b) is in­correct because checks need not be 
evaluate the design effective­ness of controls and to  compared to a sales in­voice. Answer (d) is 
confirm whether controls have been placed in  incorrect because the employee open­ing the 
operation. mail should not also perform the recordkeeping
D.1.  Sales, Receivables, and Cash Receipts function of recording the checks in the cash 
receipts journal. 
73. (a) The requirement is to identify the procedure an 
75. (b) The requirement is to determine the type of evi­
auditor most likely would perform to test controls relating
dence obtained when tracing shipping documents to 
to management’s assertion about the completeness of 
pre­numbered sales invoices. Answer (b) is correct 
cash receipts for cash sales at a retail outlet. Answer (a) is
because the shipping documents relate to shipments to 
correct because the use of cash registers and tapes helps  customers, and tracing them to sales invoices will 
assure that all such sales are recorded. Answer (b) is  provide evidence on whether sales invoices were 
incorrect because the cash has already been recorded.  prepared. Answer (a) is incor­rect because duplicate 
Answer (c) is incorrect because the procedure only deals  shipments or billings will not in gen­eral be detected 
with recorded deposits, and therefore the completeness  by tracing individual shipping documents to 
assertion is not addressed as directly as in answer (a).  prenumbered sales invoices. Answer (c) is incorrect 
Answer (d) is incorrect because one would not expect the  because an auditor will trace from customer orders to 
cash balance in the general ledger to agree with the bank  shipping doc­uments to determine whether all goods 
confirmation request amount due to items in transit and  ordered were shipped. Answer (d) is incorrect because 
outstanding at the point of reconciliation.  an auditor will account for the sequence of sales 
invoices to determine whether all sale invoices were 
74. (c) The requirement is to identify the proper  accounted for. 
proce­dure to be performed immediately upon 
receipt of checks by mail. Sound internal  76. (a) The requirement is to identify the control 
control requires the use of adequate  most likely to reduce the risk of diversion of 
documentation to ensure that all transactions  customer receipts by an entity’s employees. 
are properly recorded. This helps the company  Answer (a) is correct because a bank lockbox
attain the financial state­ment assertion of  system eliminates employee contact with 
cash re­ceipts, and thereby greatly reduces 
completeness. Answer (c) is correct be­cause 
the risk of diversion by employees. Answer 
the preparation of a duplicate listing of checks 
(b) is incorrect because remittance ad­vices 
re­ceived provides the company with a source  are ordinarily prenumbered using the 
document of all the checks received that day.  numbering schemes of the various customers
One list is then forwarded to the employee  and not of the client; also, even if a 
responsible for depositing the checks at the end prenumbering system is instituted, 
of the day and the other list is sent to the  difficulties re­main in assuring that all 
accounting department so that they can post the receipts are recorded. Answer (c) is 
amount to the cash re­ceipts journal. Answer 
MODULE 3  UNDERSTANDING INTERNAL CONTROL AND ASSESSING CONTROL RISK 99
not been recorded in the sales journal will possibly 
reveal items that have been sold (as evidenced by a 
incorrect because a monthly bank reconciliation is  shipping document) but for some reason have not 
only likely to be effective when receipts are deposited been recorded as sales. An­swers (a), (c), and (d) are
and then abstracted. Answer (d) is incorrect because  all incorrect because verification of extension and 
while the daily deposit of cash receipts may reduce  footings, comparing invoiced prices, and inquiring 
the risk of employee diversion of receipts, the  about credit granting policies all relate more di­
procedure is not as effective as the bank lockbox  rectly to the valuation assertion than to the 
system, which eliminates employee contact with the  completeness assertion.
receipts.
77. (a) The requirement is to identify the best listed 
procedure for detecting the lapping of cash receipts by 
the client’s cashier through use of customer checks 
received in the mail. Answer (a) is correct because 
lapping will result in a delay in the recording of 
specific remittance credits on the financial records, but
the checks will be deposited in the bank as they are 
received. Answer (b) is incorrect because the daily 
cash summaries will include the same sums as the cash
receipts journal entries. Answer (c) is incorrect be­
cause the bank deposit slips will be identical to any 
details included in the monthly bank statements. 
Answer (d) is incorrect because while the write­off of 
a receivable may help the individual involved in the 
lapping to avoid repay­ment, no lag is to be expected 
between authorization of the write­off and the date it is
actually recorded. 
78. (c) The requirement is to determine the 
individual or organization to which a list of 
remittances should be for­warded to in 
addition to the cashier. Answer (c) is 
correct because the accounts receivable 
bookkeeper will use the listing to update the
subsidiary accounts receivable records. 
Answer (a) is incorrect because internal 
auditors will not normally investigate each 
day’s listing of remittances for unusual 
transactions. Answer (b) is incorrect 
because the treasurer will not in general 
compare daily listings to the monthly bank 
statement. Answer (d) is incorrect because 
the list will not be sent to the bank. 
79. (d) The requirement is to identify the procedure that 
most likely would not be a control designed to reduce 
the risk of errors in the billing process. Answer (d) is 
correct because the reconciliation of the control totals 
for sales in­voices with the accounts receivable 
subsidiary ledger will follow billing; thus billing errors
will have already occurred. Answer (a) is incorrect 
because identification of differences in shipping 
documents and sales invoices will allow for a 
correction of any errors and proper billing. Answer (b) 
is incorrect because computer programmed controls 
will assure the accuracy of the sales invoice. Answer 
(c) is incorrect because the matching of shipping 
documents with the ap­proved sales orders will allow 
the preparation of a correct invoice. 
80. (b) The requirement is to identify the audit
proce­dure an auditor most likely would 
perform to test controls relating to 
management’s assertion concerning the 
com­pleteness of sales transactions. 
Answer (b) is correct be­ 
cause inspection of shipping documents that have 
liabilities are the obligations of the entity at 
81. (a) The requirement is to identify the control  a given date. 
that most likely would assure that all billed 
83. (d) The requirement is to identify the control
sales are correctly posted to the accounts 
receivable ledger. Answer (a) is cor­rect  that most likely would help ensure that all 
because the daily sales summary will include credit sales trans­actions are recorded. 
all “billed” sales for a particular day.  Answer (d) is correct because the matching 
Comparing this summary to the postings to  of shipping documents with entries in the 
the accounts receivable ledger will provide  sales journal will provide assurance that all 
evidence on whether billed sales are  shipped items (sales) have been completely 
correctly posted. An­swer (b) is incorrect  recorded. Answer (a) is incorrect because 
because comparing sales invoices to shipping comparison of approved sales orders to 
documents provides evidence on whether  authorized credit limits and balances will 
invoiced sales have been shipped. Answer (c) help ensure that customer credit limits are 
is incorrect because reconciling the accounts  not exceeded, rather than the complete re­
receivable ledger to the control account will  cording of credit sales. Answer (b) is 
not provide assurance that all billed sales  incorrect because rec­onciliation of the 
were posted in that both the receivable ledger accounts receivable subsidiary ledger to the 
and the control ac­count may have omitted  accounts receivable control account will 
the sales. Answer (d) is incorrect because  provide only a limited amount of control 
comparing shipments with sales invoices  over the complete recording of sales. The 
provides evidence on whether all shipments  control is incomplete since, for example, a 
have been invoiced, not on whether all billed  sale that has not been recorded in either the 
sales are correctly posted.  subsidiary or control accounts will not be 
detected. Answer (c) is incorrect be­cause 
82. (a) The requirement is to determine the  monthly statements generally will not be 
financial statement assertion being most  sent to cus­tomers to whom no sales have 
directly tested when an auditor tests an  been recorded. 
entity’s policy of obtaining credit approval 
before shipping goods to customers. Answer 84. (c) The requirement is to identify the control that will be 
(a) is correct because testing credit approval  most effective in offsetting the tendency of sales 
helps assure that goods are shipped to  personnel to maximize sales volume at the expense of 
customers who are likely to be able to pay;  high bad debt write­offs. Answer (c) is correct because 
ac­cordingly the valuation assertion for  segrega­tion of the authorization of credit from the sales 
receivables is being directly tested. Answer  function will allow an independent review of the 
(b) is incorrect because complete­ness deals  creditworthiness of customers. Answer (a) is incorrect 
with whether all transactions and accounts  because while denying access to cash by employees 
are recorded. Answer (c) is incorrect  responsible for sales and bad debt write­offs may deter 
because existence deals with whether assets  embezzlements, the problem of high bad debt write­offs 
exist at a given date and whether re­corded  is likely to remain. Answer (b) is incorrect because while 
transactions have occurred during a given  so segregating the matching of ship­ping documents and 
period. Answer (d) is incorrect because  sales invoices may help assure that items are shipped 
rights and obligations deal with whether  properly and subsequently recorded, it will 
assets are the rights of the entity and 
100 MODULE 3  UNDERSTANDING INTERNAL CONTROL AND ASSESSING CONTROL RISK
reconciliation of checking accounts with the monthly 
bank statement will not reflect cash that has been 
not significantly affect bad debts. Answer (d) is  improperly diverted by the accounts receivable clerk. 
incorrect because while independent reconciliation  Answer (d) is incorrect because the accounts 
of control and subsidiary accounts receivable  receivable clerk should be able to maintain the detail 
records may defer embez­zlements, bad debt write­ and ledger amount in balance and thereby avoid 
offs will not be affected. detection. 
85. (d) The requirement is to determine the  88. (b) The requirement is to determine who should 
department that should approve bad debt  prepare a remittance advice when the customer fails to 
write­offs. The department responsible for  in­clude one with a remittance. Remittances should be 
bad debt write­offs should be independent of  opened by an individual such as a receptionist who is 
the sales, credit, and the recordkeeping for  independent of the sales function. That individual will 
that function, and should have knowledge  prepare any needed remittance advices. The credit 
relating to the accounts. Answer (d) is  manager [answer (a)], and the sales manager [answer 
correct because, in addition to being  (c)], are incorrect because they 
independent of the various functions, the 
treasurer’s department is likely to have 
knowledge to help make proper decisions of 
this na­ture. Answers (a) and (b), accounts 
receivable and the credit department, are 
incorrect because neither department is in­
dependent of this function. Answer (c) is 
incorrect because while accounts payable is 
independent of the function, its personnel are
less likely than those of the treasurer’s de­
partment to have the necessary information 
relating to the accounts that should be 
written off. 
86. (b) The requirement is to identify a reason 
that em­ployers bond employees who 
handle cash receipts. An­swer (b) is correct 
because employee knowledge that bond­ing
companies often prosecute those accused of 
dishonest acts may deter employees’ 
dishonest acts. Answer (a) is incorrect 
because bonding protects the employer 
from dis­honest acts, not the employee from
unintentional errors. Answer (c) is incorrect
because the bonding company does not 
serve the role of independent monitoring of 
cash. An­swer (d) is incorrect because while
rotation of positions and forcing employees 
to take periodic vacations are effective 
controls for preventing fraud, they are not 
accomplished through bonding. 
87. (c) The requirement is to identify the control which 
would offset a weakness that allowed the accounts 
receiv­able clerk to approve credit memos and to have 
access to cash. Note that such a weakness may lead to 
a fraud in which the accounts receivable clerk receives
and keeps cash payments while issuing a fraudulent 
credit memo as the ba­sis for a credit to the customer’s
account. Answer (c) is correct because the owner’s 
review of credit memos could help establish that 
fraudulent memos had not been issued for receivables 
which had in actuality been collected by the clerk; for 
example, when reviewing credit memos the owner 
would expect to see a receiving report for sales returns 
for which credit memos have been generated. Answer 
(a) is incorrect because the bookkeeper may be able to 
maintain the records in such a manner as to avoid 
billing errors re­lated to the fraud, and therefore a 
review of billings and postings may not reveal such 
fraud. Answer (b) is incorrect because month­end 
ments most directly addresses. Answer (b) is
perform an authorization function related to sales.  correct be­cause the existence or occurrence 
An­swer (d) is incorrect because the accounts  assertion addresses whether recorded entries 
receivable clerk performs a recordkeeping function  are valid and the direction of this test is from
for sales. the recorded entry in the voucher register to 
the supporting documents. Answer (a) is 
D.2.  Purchases, Payables, and Cash Disbursements incorrect because completeness addresses 
89. (b) The requirement is to identify the audit  whether all transactions and ac­counts are 
proce­dure relating to paid vouchers that  included and would involve tests tracing 
will provide assurance that each voucher is  from support for purchases to the recorded 
submitted and paid only once. An­swer (b)  entry. Answers (c) and (d) are incorrect 
is correct because when the check signer  because vouching the entries provides only 
stamps vouchers “paid” it is unlikely to be  limited evidence on valuation and rights. 
paid a second time since that individual will 92. (d) The requirement is to determine the 
notice the stamp on the voucher the sec­ond control that is not usually performed in the 
time it is submitted for payment. 
vouchers payable depart­ment. Answer (d) 
90. (d) The requirement is to determine the  is correct because the vouchers payable 
appropriate responsibility for the person who department will not in general have access 
signs checks. Answer (d) is correct because  to unused pre­numbered purchase orders 
the individual who signs checks should be  and receiving reports. An­ 
responsible for mailing them so as to avoid a  swer (a) is incorrect because the vouchers payable 
variety of fraud in which the checks are  depart­ment will match the vendor’s invoice with 
improperly converted into cash by company  the related receiving report to determine that the 
employees. Answer (a) is incorrect because  item for which the company has been billed has 
the individual who reconciles the bank  been received. Answer (b) is incorrect because the 
accounts should have no other  vouchers payable department will approve vouchers
responsibilities with respect to cash. Answer  for payment when all documentation is proper and 
(b) is incorrect because accounts payable  present. Answer (c) is incorrect because the 
does not need the checks and, if they receive  vouchers payable department will code the voucher 
them, those checks may be converted into  with accounts to be debited. 
cash which is then stolen. Answer (c) is  93. (b) The requirement is to identify the function that is 
incorrect be­cause the person who signs the  consistent with matching vendors’ invoices with 
checks should determine that proper  receiving reports. Answer (b) is correct because while 
supporting documents exist and should  matching in­voices and receiving reports, the 
cancel that documentation after the payment  employee might effectively recompute the calculations
is made.  on the vendors’ invoices to de­termine that the 
91. (b) The requirement is to determine the  amounts are proper. Answers (a) and (c) are incorrect. 
financial statement assertion that a test of  The individual who matches the invoices and receiving
controls of vouching a sam­ple of entries in  reports will often also approve them for payment. 
the voucher register to the supporting docu­ Therefore, this individual should not also post 
accounts pay­ 
MODULE 3  UNDERSTANDING INTERNAL CONTROL AND ASSESSING CONTROL RISK 101
of merchandise ordered. Answer (b) is correct because 
if the receiving department personnel are unaware of 
able records or reconcile the accounts payable ledger.  the quantities ordered, they will provide an 
An­swer (d) is incorrect because the individual who  independent count of quantities received. Answer (a) is
controls the signing of the checks should cancel the  incorrect because the department that initiated the 
invoices after pay­ment. requisition needs the merchandise, and therefore, 
should know what has been ordered. Answer (c) is 
94. (b) The requirement is to identify the 
incorrect because the purchasing agent is involved 
population from which items should be  with purchasing the items and therefore must be aware 
selected to determine whether checks are  of the quantity involved. Answer (d) is incorrect 
being issued for unauthorized expenditures.  because the ac­ 
An­swer (b) is correct because a sample of 
canceled checks should be selected and 
compared with the approved vouch­ers, a 
prenumbered purchase order and 
prenumbered re­ceiving reports. A canceled 
check that does not have such support may 
have been unauthorized. Answers (a), (c), 
and (d) are all incorrect because selecting 
items from pur­chase orders, receiving 
reports, or approved vouchers will not 
reveal circumstances in which a check was 
issued with­ 
out that supporting document. For example, when 
selecting a sample from purchase orders, one would 
not discover a situation in which a check had been 
issued without a pur­chase order. 
95. (d) The requirement is to determine which question 
would most likely be included in an internal control 
ques­tionnaire concerning the completeness assertion 
for pur­chases. Answer (d) is correct because 
prenumbering and accounting for purchase orders, 
receiving reports, and vouchers will allow a company 
to determine that purchases are completely recorded. 
For example, in examining a re­ceiving report the 
client might discover that the purchase was not 
recorded. Answer (a) is incorrect because requiring a 
purchase order before a shipment is accepted will 
address whether or not the shipment has been ordered. 
Answer (b) is incorrect because matching purchase 
requisitions with ven­dor invoices does not directly 
address whether the purchase is recorded. Answer (c) 
is incorrect because the unpaid voucher file represents 
items that have already been re­corded, and thus will 
not directly address the completeness of the recording 
of purchases. 
96. (d) The requirement is to identify the proper control 
pertaining to the accounts payable department. Answer
(d) is correct because the accounts payable department 
should establish the agreement of the vendor’s invoice 
with the purchase order and receiving report to provide
assurance that the item was both ordered and received. 
Answer (a) is in­correct because the individual signing
the check (e.g., the treasurer), not the accounts payable
department, should stamp, perforate, or otherwise 
cancel supporting documen­tation after a check is 
signed. Answer (b) is incorrect be­cause purchase 
requisitions will not normally include de­tailed price 
information. Answer (c) is incorrect because the 
quantity ordered on the receiving department copy of 
the purchase order will already be obliterated when the
purchase order is completed by the purchasing 
department. 
97. (b) The requirement is to determine which copy of the 
purchase order should omit indication of the quantity 
authorization, and custodial functions should all be 
counts   payable   department   must   reconcile   the segregated. 
quantity   re­ceived   and   the   quantity   billed   to   the
quantity that was au­thorized to be purchased per D.3.  Inventories and Production
the purchase order. 100. (c) The requirement is to identify the most 
98. (b) The requirement is to identify the audit  likely procedure an auditor would perform 
proce­dure which would most likely detect a in obtaining an under­standing of a 
client error in re­cording a large purchase.  manufacturing entity’s internal control for 
Answer (b) is correct because reconciling  inventory balances. Answer (c) is correct 
the vendors’ monthly statements with the  because a review of the entity’s descriptions 
sub­sidiary ledger for payables should  of inventory policies and proce­dures will 
disclose a difference in the month following help the auditor to obtain the necessary 
the error. Answer (a), footing the pur­chases under­standing about the design of relevant 
journal, is unlikely to detect the error since  policies, procedures, and records, and 
the jour­nal’s totals will have been  whether they have been implemented by the 
mathematically accumulated properly.  entity. Answers (a) and (b) are incorrect 
Answer (c) is incorrect because the incorrect because analyses of liquidity and turnover 
total will be reflected in both the purchases  ratios are analytical pro­cedures designed to 
journal and in the ledger accounts. Answer  identify cost variances that will help the 
(d) is incorrect because such con­firmations  auditor primarily to determine the nature, 
will only detect the error quarterly which is  timing and extent of auditing procedures that
nei­ther timely nor efficient.  will be used to obtain evidence for specific 
account balances or classes of transactions. 
99. (d) The requirement is to identify a function  Answer (d) is incorrect because test counts 
that is compatible with the approval of  of inventory are generally obtained as a 
purchase orders. An­swer (d) is correct  substantive procedure. 
because the purchases department will 
normally approve purchase orders (generated  101. (d) The requirement is to identify the control that most
from user de­partments or stores) and  likely would be used to maintain accurate inventory 
negotiate terms of purchase with vendors.  records. Answer (d) is correct because periodic 
Answer (a) is incorrect because while the pur­ inventory counts will assure that perpetual inventory 
chases department may reconcile the open 
records are accu­rate and, because employees will 
know that inventory differ­ences are investigated, they
invoice file, this is primarily a recordkeeping 
will be less likely to steal any inventory. Answer (a) is 
function that will often be per­formed by the 
incorrect because comparing the perpetual inventory 
accounting department. Answer (b) is incor­
records with current costs of items will reveal 
rect because most frequently the receiving 
situations in which costs have changed, but is unlikely 
department will  to help in the maintenance of accurate inventory 
inspect goods upon receipt. Answer (c) is incorrect  records. Answer (b) is incorrect because while a just­
because user groups or stores will authorize  in­time inven­tory ordering system may help assure 
requisitions of goods. Keep in mind the principle of  that inventory records are accurate (as well as kept at 
segregation of functions. Recordkeeping,  low levels), such a system 
102 MODULE 3  UNDERSTANDING INTERNAL CONTROL AND ASSESSING CONTROL RISK
typically involved in the receiving function and thus 
involve a higher risk relating to establishing 
generally also requires periodic inventory counts to  accountability. 
maintain accurate inventory records. Answer (c) is 
incorrect because matching requisitions, receiving  105. (c) The requirement is to identify the effect 
reports, and purchase orders only helps assure that items  on reve­nues, receivables, and inventory of 
received are paid for; the matching process does not  inadequate controls over the invoicing 
assure accurate inventory records. function that allows goods to be shipped 
with­out being invoiced. Items shipped 
102. (d) The requirement is to determine a likely effect on  without invoicing will result in a situation in 
the audit of inventory if the assessed level of control  which the accounting department is unaware 
risk is high. Answer (d) is correct because a high  of the sale. Therefore, debits to accounts 
assessed level of control risk may result in changing  receiv­able and credits to sales will not be 
the timing of substan­tive tests to year­end rather than  recorded, resulting in an understatement of 
at an interim date. If the assessed level of control risk  both revenues and receivables. Similarly, 
is low, the auditor could per­form interim substantive 
tests and rely upon internal control to provide valid 
year­end records. However, because the assessed level 
of control risk is high, the controls cannot be relied 
upon. Also, the nature of substantive tests may change 
from less effective to more effective procedures (e.g., 
use of independent parties outside the entity rather 
than internal) and an increase in the extent of 
procedures (e.g., larger sample sizes). Answer (a) is 
incorrect because, as indicated, an auditor will 
generally seek a year­end count of inventory. Answer 
(b) is incorrect because gross profit tests will not in 
general have the required precision when control risk 
is high. Answer (c) is incorrect because tests of 
controls are likely to substantiate an auditor’s view 
that control risk is high, and it is therefore unlikely that
their performance will be cost­effective. 
103. (c) The requirement is to identify the control
which is most likely to address the 
completeness assertion for in­ventory. 
Answer (c) is correct because by 
prenumbering receiving reports and by 
reconciling them with inventory records, one
is able to test completeness by determining 
whether all receipts have been recorded. 
Answer (a) is in­correct because reconciling 
the subsidiary records with the work in 
process will only identify discrepancies 
between the records, it will not identify 
whether all transactions that should be in the
inventory records are represented in the 
records. Answer (b) is incorrect because 
while the segrega­tion of receiving from 
custody of finished goods is impor­tant, it 
less directly addresses completeness than 
does an­swer (a). Answer (d) is incorrect 
because separating the duties between the 
payroll department and inventory ac­
counting personnel does not directly address 
completeness of inventory. 
104. (c) The requirement is to determine the proper inter­
nal control for handling customer returns of defective 
mer­chandise. Answer (c) is correct because the 
receiving de­partment can count the goods, and list 
them on a sales return notice to determine that all such 
returns are properly re­corded. This serves as a control 
because the normal proce­dures of the receiving 
function include establishing the original 
accountability and recordkeeping for items received. 
Answers (a), (b), and (d) all represent functions not 
swer (a) is incorrect because clock card data often does 
because accounting is unaware of the sale, no entry  not include authorized pay rates. Answer (c) is incorrect 
to re­duce inventory will be made, resulting in an  be­cause the procedure does not directly address the 
overstatement of inventory. segregation of duties since no information is provided 
concerning the distribution of the payroll. Answer (d) is 
106. (c) The requirement is to identify the 
incorrect because unclaimed payroll checks are not being 
question that an auditor would expect to find
analyzed. 
on the production cycle section of an 
internal control questionnaire. Answer (c) is  109. (a) The requirement is to identify the control 
correct because approved requisitions will  that most likely could help prevent employee 
help maintain control over raw materials  payroll fraud. An­swer (a) is correct because 
released to be used in the produc­tion cycle.  prompt notification of the payroll supervisor 
Answers (a), (b), and (d) are all incorrect be­ concerning terminations will lead to timely re­
cause approval of vendors’ invoices for  moval of terminated employees from the 
payment, mailing of checks after signing,  payroll. Accord­ingly, no payroll checks will be
and comparing individual disburse­ments to  prepared for such terminated employees. 
totals all pertain more directly to the  Answer (b) is incorrect because unclaimed pay­
disbursement cycle.  roll checks should not be returned to the 
107. (b) The requirement is to determine an  supervisors who might inappropriately cash 
objective of internal control for a production  them. Answer (c) is incorrect because since the 
cycle in addition to provid­ing assurance that payroll department is involved in record­
transactions are properly executed and  keeping, it should not approve salary rates. 
recorded. Answer (b) is correct because, in  Answer (d) is incorrect because calculation of 
addition to pro­viding assurance as to proper total hours by the payroll supervisor is unlikely 
execution and recording, an objective for the to prevent employee payroll fraud. 
production cycle (as well as other cycles) is 
110. (a) The requirement is to determine the best 
the safeguarding of assets, here work in 
process and finished goods. Answers (a), (c), proce­dure for determining the effectiveness 
and (d) are incorrect because they represent  of an entity’s controls relating to the 
detailed controls established to help achieve  existence or occurrence assertion for payroll 
the overall objectives. They are all much  transactions. Answer (a) is correct because 
more specific than the overall objectives.  proper segrega­tion of duties between 
personnel and payroll disbursement 
D.4.  Personnel and Payroll eliminates many frauds in which “phantom” 
employees are being paid. Answer (b) is 
108. (b) The requirement is to identify a purpose of vouching  incorrect because accounting for the 
data for a sample of employees in a payroll regis­ter to  prenumbered payroll checks addresses 
approved clock card data. Answer (b) is correct be­cause  completeness more directly than it does 
the clock card data provides the auditor with evidence on  existence or occurrence. An­swer (c) is 
whether employees worked the number of hours for  incorrect because recomputing payroll 
which the payroll register indicates they were paid. An­ deductions for employee fringe benefits, 
without additional analysis, 
MODULE 3  UNDERSTANDING INTERNAL CONTROL AND ASSESSING CONTROL RISK 103
required to sign a receipt for the amount of 
pay received. Thus, there would be control 
provides only a very limited test of existence or  over the total amount disbursed as well as 
occurrence. Answer (d) is incorrect because verifying  amounts disbursed to each individual 
the preparation of the monthly payroll account bank  employee. Answer (a) is incorrect because if 
reconciliation is unlikely to provide evidence on the  a signed receipt is not received from each 
existence or occurrence assertion, although some  employee paid, there would be no proof of 
possibility does exist if signatures on checks are  payment. Even though the pay envelopes 
analyzed in detail. include both cash and a computation of net 
wages, the employees should have the 
111. (c) The requirement is to identify the 
opportunity to count the cash received before
situation in which it is most likely that an 
signing a pay­roll receipt. Answer (b) is 
auditor would assess control risk for payroll
incorrect because unclaimed pay envelopes 
at a high level. Answer (c) is correct be­
should not be retained by the paymaster, but 
cause the payroll department, which is 
rather deposited in a bank account by the 
essentially a record­keeping function, 
cashier. An­ 
should not also authorize payroll rate 
changes. Under strong internal control 
recordkeeping, au­thorization, and custody 
over assets should be segregated. Answer 
(a) is incorrect because examining 
authorization forms for new employees is 
consistent with the payroll de­partment’s 
recordkeeping function. Answer (b) is 
incorrect because comparing payroll 
registers with original batch transmittal data
is a control relating to recordkeeping. An­
swer (d) is incorrect because while the 
actual hiring of em­ployees is normally 
done in the personnel department, al­lowing
the payroll department supervisor to hire 
subordinates, with proper approval, is not as
inconsistent with payroll’s recordkeeping 
function as is authorizing rate changes for 
all employees. 
112. (c) The requirement is to identify the control most 
likely to prevent direct labor hours from being charged
to manufacturing overhead. Answer (c) is correct 
because time tickets may be coded as to whether direct
labor on various projects was involved. Accordingly, 
using time tickets will help identify direct labor costs. 
Answer (a) is incorrect be­cause while periodic counts 
of work in process may provide a control over physical
units of production, the counts will not in general 
provide assurance that direct labor hours are properly 
charged to the product rather than to manufacturing 
overhead. Answer (b) is incorrect because comparing 
daily journal entries with production orders will not in 
general identify costs that have been omitted from 
direct labor due to the level of aggregation of the 
entries. Answer (d) is incor­rect because the 
reconciliation of work in process inventory with 
budgets will provide only very limited detection ability
relating to the charging of direct labor to 
manufacturing overhead. 
113. (a) The requirement is to determine the individual(s) 
who should distribute paychecks and have custody of 
un­claimed paychecks. Answer (a) is correct because 
these custody functions should not be performed by 
the payroll department which is a recordkeeping 
function. Under proper internal control recordkeeping, 
custody, and authori­zation of transactions should be 
segregated. 
114. (c) If payment of wages were to be in cash, 
each employee receiving payment should be 
auditor may observe the distribution of paychecks. An­
swer (d) is incorrect because the wage payment will be swer (c) is correct because an employee’s presence to 
made in cash and not by check. Accordingly, a receipt  collect the paycheck provides evidence that the 
must be obtained for each cash payment. employee actually exists and is currently employed by 
the client. Answer (a) is incorrect because the 
115. (a) The requirement is to identify the purpose 
distributions of payroll checks would not reveal 
of segregating the duties of hiring personnel  whether payrate authorization is properly sepa­rated 
and of distributing payroll checks. Answer (a)  from the operating function. Answer (b) is incorrect 
is correct because the hiring of personnel is an  because the paycheck distribution does not provide 
authorization function while the distribution of  informa­tion on whether deductions from gross pay 
checks is a custody function. Thus, in order to  have been cal­culated properly. Answer (d) is incorrect
properly segregate authorization from custody,  because observa­tion of the paycheck distribution 
these duties should not be performed by the  process does not of itself provide assurance that the 
same individual. The combination of these two  paychecks agree with the related payroll register and 
functions in the same position would create the  time cards. 
possibility of the addition of a fictitious 
employee to the payroll and subsequent  118. (a) The requirement is to determine the 
misappropriation of paychecks. Answer (b) is  department most likely to approve change in 
incorrect because the functions involved are  pay rates and deductions from employee 
not primarily operational or recordkeeping.  salaries. Answer (a) is correct because the 
Answer (c) is incorrect because the treasury  personnel department, which has the primary
function, and not the control­lership function,  objective of planning, controlling and 
will normally be responsible for distribut­ing  coordinating employees, will de­termine that
payroll checks. Answer (d) is incorrect because proposed salary increases (often 
recommended by supervisors of employees) 
segre­gation of duties does not directly address 
are consistent with the com­pany’s salary 
administrative controls vs. internal accounting 
guidelines and will approve changes in 
controls. 
deduc­tions. Answers (b) and (c) are 
116. (c) The requirement is to determine the best incorrect because the trea­surer and 
method   to   minimize   the   opportunities   for controller will in general initiate the pay rate 
fraud for unclaimed cash  change process for only those employees 
in a cash payroll system. For a cash payroll the best  within their de­partments and will not 
control is to get the unclaimed cash out of the firm’s  generally approve changes for em­ployees 
physical con­trol and into the bank. Answer (a) is  outside their departments. Answer (d) is 
incorrect because maintaining the accountability for  incorrect because the payroll functions is a 
cash which is in a safe­deposit box is difficult.  recordkeeping function which will modify 
Answers (b) and (d) are incorrect because the cash  employee pay rates based on approved 
need not be kept by the firm. changes from personnel. Payroll should not 
have authority regarding pay rates and 
117. (c) The requirement is to identify a reason why an  deductions. 
104 MODULE 3  UNDERSTANDING INTERNAL CONTROL AND ASSESSING CONTROL RISK
controlled physically in order to prevent 
unautho­rized usage. Answer (c) is incorrect 
D.5.  Financing because access to secu­rities should 
ordinarily be vested in two individuals so as 
119. (d) The requirement is to determine the most likely  to assure their safekeeping. Answer (d) is 
question that would be included on an internal control  incorrect because securities should be 
ques­tionnaire for notes payable. Answer (d) is correct  registered in the name of the owner. 
because companies frequently require that direct 
borrowings on notes payable be authorized by the  123. (b) The requirement is to identify the best 
board of directors; accordingly, auditors will  control for safeguarding marketable 
determine whether proper policy has been fol­lowed.  securities when an independent trust agent is
Answer (a) is incorrect because internal control  not employed. Answer (b) is correct because
questionnaires do not in general include questions on  requiring joint control over securities 
whether assets that collateralize notes payable are  maintained in a safe­deposit box assures 
critically needed. Answer (b) is incorrect because the  that, absent collusion, assets are safe­
internal control questionnaire for disbursements is  guarded. Answer (a) is incorrect because a 
more likely to address the required authorized  review of in­ 
signatures on checks than will the inter­nal control 
questionnaire for notes payable. Answer (c) is 
incorrect because while it is often good business 
practice to use proceeds from long­term notes to 
purchase noncurrent assets, this is not required and is 
not included on an internal control questionnaire. 
120. (d) The requirement is to identify the 
primary re­sponsibility of a bank acting as 
registrar for capital stock. Answer (d) is 
correct because the primary responsibility of
the stock registrar is to prevent any 
overissuance of stock, and thereby verify 
that the stock is issued properly. An­swer (a)
is incorrect because registrar will not in 
general determine that the dividend amounts 
are proper. Answer (b) is incorrect because 
the transfer agent will maintain records of 
total shares outstanding as well as detailed 
stockholder records, and carry out transfers 
of stock ownership. An­swer (c) is incorrect 
because registrars do not perform the role 
described relating to mergers, acquisitions, 
and the sale of treasury stock. 
121. (a) Canceled stock certificates should be defaced and 
attached to corresponding stubs as is done with voided 
checks. The objective of the control is to prevent 
reissuance. Answer (b) is incorrect because failure to 
deface permits reissuance. Answer (c) is incorrect 
because destruction of the certificates would preclude 
their control (i.e., their exis­tence after defacing 
provides assurance that they cannot be reissued). If the
certificates were destroyed, one or more might be 
reissued without any proof that such occurred. Answer
(d) is incorrect because the Secretary of State has no 
interest in receiving defaced and canceled stock 
certificates. 
D.6.  Investing
122. (a) The requirement is to identify the reply 
which is not a control that is designed to 
protect investment securi­ties. Answer (a) is 
not a control since the custody of securi­ties 
should be assigned to individuals who do 
not have ac­counting responsibility for 
securities; as with other assets, 
authorization, recordkeeping, and custody 
should be sepa­rated. Answer (b) is incorrect
because securities should be properly 
Answer (b) is incorrect because pre­
vestment decisions by the investment committee will  numbered purchase orders may be used to 
have a very limited effect on safeguarding  control all pur­chases, including purchases of
marketable securities. Answer (c) is incorrect because equipment. Answer (c) is incorrect because 
the simple tracing of mar­ketable securities from the  the significant amounts of money involved 
subsidiary ledgers to the general ledger does not  with purchases of equipment suggest the 
directly safeguard marketable securities since, for  need for the solici­tation of competitive bids.
example, unrecorded transactions may occur. Answer
(d) is incorrect because, even if the chairman of the  126. (d) The requirement is to identify the control that 
board did verify marketable securities on the balance  would be most effective in assuring that the proper 
sheet date, the control will only be effective at that  custody of assets in the investing cycle is maintained. 
point in time. Answer (d) is correct because comparing recorded 
balances in the invest­ment subsidiary ledger with 
124. (d) The requirement is to identify the best  physical counts will help assure that recorded assets 
audit pro­cedure when a weakness in internal are those over which the company has custody. This is 
control over reporting of retirements exists.  an example of the control activity of com­parison of 
Answer (d) is correct because selecting  assets with recorded accountability. Answer (a) is 
certain items of equipment from the  incorrect because internal control is improved when 
accounting records and attempting to locate  two individuals, not one, must be present for entry to 
them will reveal situations in which the  the safe­deposit box. Answer (b) is incorrect because 
accounting records still have them recorded  while the seg­regation of duties within the 
subsequent to their retirement. Answer (a) is  recordkeeping may in certain circumstances be 
incorrect because inspecting items that still  desirable, it does not directly address cus­tody over 
exist is not likely to lead to discovery of un­ assets. Answer (c) is incorrect because only ex­tremely
recorded retirements. Answer (b) is incorrect major investments generally need be authorized by the 
because de­preciation may continue to be  board of directors. 
taken on equipment that has been retired, but
not recorded. Answer (c) is incorrect be­ 127. (d) The requirement is to determine who 
cause it is doubtful that such retirements  should have responsibility for custody of 
have been reclassi­fied as “other assets.”  short­term bearer bond investments and the 
submission of coupons for periodic 
125. (d) The requirement is to identify the  collections of interest. The treasurer 
question that is least likely to be included on  authorizes such trans­actions. Answer (a) is 
an internal control question­naire concerning incorrect because the chief accoun­tant, who 
the initiation and execution of equipment  is in charge of the recordkeeping function, 
transactions. Answer (d) is correct because  should not also maintain custody of the 
procedures to monitor and properly restrict  bonds. Answer (b) is incor­rect because the 
access to equipment do not relate directly to  internal auditor should not be directly in­
the initiation and execution of equipment  volved as such involvement would make an 
transactions. Answer (a) is incorrect because  independent review of the system 
requests for major repairs relate to initiation  impossible. Answer (c) is incorrect 
of a transaction which should be controlled. 
MODULE 3  UNDERSTANDING INTERNAL CONTROL AND ASSESSING CONTROL RISK 105
131. (d) The requirement is to determine the type 
offraud which is most difficult to detect. Answer (d), 
because   the   cashier   function   is   more   directly a fraud committed by the controller, is most difficult 
involved with details such as endorsing, depositing, to detect be­cause the controller is in control of the 
and maintaining rec­ords of cash receipts. recordkeeping func­tion and thus may be able to 
commit a fraud and then ma­nipulate the accounting 
128. (b) The requirement is to identify the control that 
records so as to make its discovery unlikely. Answer 
would be most likely to assist an entity in satisfying 
(a) is incorrect because while a cashier may be able 
the completeness assertion related to long­term 
to embezzle funds, s/he will not have access to the 
investments. Answer (b) is correct because 
accounting records and thus discovery of the 
completeness deals with whether all transactions are 
embezzle­ment will be likely. Answer (b) is incorrect
recorded, and the comparison of securities in the bank 
because a key­
safe­deposit box with recorded in­vestments may 
reveal securities which are in the safe­deposit box but 
are not recorded. Answer (a) is incorrect because 
verification of security registration helps establish the 
rights assertion not the completeness assertion. Answer
(c) is in­correct because vouching the acquisition of 
securities by comparing brokers’ advices with 
canceled checks helps to establish the existence 
assertion not the completeness asser­tion. Answer (d) 
is incorrect because a comparison of the current 
market prices of recorded investments with brokers’ 
advices addresses the valuation assertion not the 
complete­ness assertion. 
129. (a) The requirement is to identify the best control for 
safeguarding marketable securities against loss. An­
swer (a) is correct because use of an independent trust 
com­pany allows the effective separation of custody 
and record­keeping for the securities. Answer (b) is 
incorrect because a verification of marketable 
securities at the balance sheet date may have only a 
limited effect on safeguarding the securities 
throughout the year. Answer (c) is incorrect because 
tracing all purchases and sales of marketable securities
will not af­fect securities that have disappeared for 
which no entries have been made. Also, it is unlikely 
that an entity will rely upon the independent auditor in 
this manner. Answer (d) is incorrect because 
maintenance of control over custody by a member of 
the board of directors may provide less complete 
control than the use of an independent trust company. 
130. (c) The requirement is to determine the 
appropriate combination of audit tests when 
there are numerous property and equipment 
transactions during the year and the auditor 
plans to assess control risk at a low level. 
Answer (c) is correct because, to justify an 
assessment of control risk at a low level, tests
of controls will be required. This will allow 
auditors to perform only limited tests of 
current year prop­erty and equipment 
transactions. Answer (a) is incorrect because 
tests of controls will be performed to allow 
the auditor to perform limited, not extensive, 
tests of property and equipment balances at 
the end of the year. Answers (b) and (d) are 
incorrect because analytical procedures on 
either year­end balances or transactions will 
not justify a low as­sessed level of control 
risk. 
D.7.  Overall Internal Control Questionnaires 
(Check­
lists)
135. (d) The requirement is to identify the 
punch operator will not in general have access to  statement that should be included in an 
assets. Answer (c) is incorrect because an internal  auditor’s letter on significant defi­ciencies. 
auditor will not generally be able to manipulate the  Answer (d) is correct because AU 325 
accounting records and generally has limited access  indicates that such a letter to the audit 
to assets. committee should (1) indicate that the 
audit’s purpose was to report on the 
E.1.a. Communication of Internal Control Related
financial state­ments and not to express an 
Matters
opinion on internal control, 
132. (c) Answer (c) is correct because weaknesses that were (2) include the definition of a significant deficiency,
communicated in the past and not corrected must be  and 
re­communicated to management and those charged  (3) restrict distribution of the report. 
with gov­ernance. Answers (a), (b), and (d) are all  136. (c) The requirement is to identify the correct state­ment 
accurate state­ments about audit requirements. 
concerning an auditor’s required communication of 
133. (c) The requirement is to identify the matter an auditor  significant deficiencies. Answer (c) is correct because 
would most likely consider to be a material weakness  dis­tribution of an auditor’s report on significant 
to be communicated to those charged with govern­ deficiencies should be restricted to management and the 
ance. Answer (c) is correct because ineffective  audit committee. Answer (a) is incorrect because lack of 
oversight of financial reporting by those charged with  correction of a sig­nificant deficiency does not 
governance is an indicator of a material weakness (AU necessarily result in a scope limitation. Answer (b) is 
325). Answers (a), (b), and (d) are all incorrect  incorrect because tests of controls need not be performed 
because a failure to renegotiate unfavorable long­term  relating to significant deficiencies. Answer (d) is 
purchase commitments, recurring operating losses, and incorrect because although timely communi­cation of 
plans to reduce ownership equity do not fall within the significant deficiencies may be important, de­pending 
definition of a material weakness.  upon the nature of the significant deficiency identi­fied, 
the auditor may choose to communicate it either after the 
134. (c) The requirement is to identify the correct state­
audit is concluded or during the course of the audit. 
ment concerning significant deficiencies identified in 
an audit. Answer (c) is correct because an auditor may  137. (d) The requirement is to identify the correct 
com­municate significant deficiencies either during an  state­ment about significant deficiencies noted 
audit or after the audit’s completion. Answer (a) is  in an audit. An­swer (d) is correct because the 
incorrect because an auditor need not search for  auditor should separately identify and 
significant deficiencies. An­swer (b) is incorrect  communicate material weaknesses and 
because all significant deficiencies are not also  significant deficiencies (AU 325). Answer (a) 
material weaknesses. Answer (d) is incorrect be­cause  is incorrect because significant deficiencies are 
an auditor may not issue a written report that no sig­ less severe than material weaknesses. Answer 
nificant deficiencies were noted during an audit.  (b) is incorrect because the auditor is 
106 MODULE 3  UNDERSTANDING INTERNAL CONTROL AND ASSESSING CONTROL RISK
with the audit committee his/her views about 
significant matters that were the subject of such 
not obligated to search for significant deficiencies. consultation; accordingly, such a discussion with 
Answer (c) is incorrect because such deficiencies  manage­ment is to be expected. While the information 
should be recommunicated. suggested in answers (a), (c), and (d) may all be 
communicated to the audit committee, they are not 
138. (b) The requirement is to determine the  included as required disclo­sures under AU 380. See 
representa­tion that should not be included  AU 380 for the various matters that must be 
in a report on internal con­trol related  communicated to those charged with govern­ance. 
matters noted in an audit. Answer (b) is 
correct because the auditors should not issue  142. (b) The requirement is to identify the 
a report on internal control stating that no  correct state­ment concerning an auditor’s 
significant deficiencies were identi­fied  required communication with those charged
during the audit. Answer (a) is incorrect  with governance. Answer (b) is correct be­
because significant deficiencies should be  cause the communication should include 
disclosed. Answer (c) is incorrect because an such information on disagreements. See AU
auditor may recommend corrective follow­ 380 for this and other required 
up action. Answer (d) is incorrect because an communications with those charged with 
auditor may disclose the fact that the  governance. An­ 
consideration of internal control would not 
necessarily disclose all significant 
deficiencies that exist. 
139. (b) The requirement is to identify the 
correct state­ment regarding material 
weaknesses and significant defi­ciencies. 
Answer (b) is correct because a 
compensating control is a control that 
lessens the severity of a deficiency (AU 
325). Answer (a) is incorrect because 
material weaknesses should be reported 
separately from significant deficiencies. 
Answer (c) is incorrect because while the 
auditor may choose to communicate 
material weakness and significant 
deficiencies immediately, the 
communication may occur at other times. 
Answer (d) is incorrect because significant 
deficiencies are less severe than material 
weaknesses. 
140. (b) The requirement is to determine an 
auditor’s responsibility after s/he has 
discovered and orally communi­cated 
information on a weakness in internal control
to the client’s senior management and those 
charged with corpo­rate governance. Answer
(b) is correct because the auditor, as outlined 
throughout AU 318, considers and 
documents his/her understanding of internal 
control to assist in planning and determining 
the proper nature, timing, and extent of 
substantive tests. Answer (a) is incorrect 
because no scope limitation is indicated 
although an internal control condition does 
exist. Similarly, answers (c) and (d) are 
incorrect be­cause audit activities need not 
be suspended and the auditor need not 
withdraw from the engagement. 
E.1.b The Auditor’s Communication with 
Those Charged with Governance
141. (b) The requirement is to determine the matter that an 
auditor would communicate to those charged with 
gov­ernance. Answer (b) is correct because AU 380 
requires that when the auditor is aware of such 
consultation with another CPA, s/he should discuss 
is incorrect because direct communication by the 
swer (a) is incorrect because the communication may  auditor is not required for selection of accounting 
occur before or after issuance of the auditor’s report.  principles. Management may engage in this 
Answer (c) is incorrect because not all matters need be communication with those charged with corporate 
communicated to management. Answer (d) is  governance. Answer (c) is incorrect because the 
incorrect because significant adjustments need to be  auditor only needs to make sure that management has 
communicated to those charged with governance. com­municated with the committee concerning initial 
selection of significant accounting policies in 
143. (b) The requirement is to identify the  emerging areas that lack authoritative guidance. 
discussion that it is least likely that an  Answer (d) is incorrect because disagreements should 
auditor will initiate with those charged with  be communicated directly to those charged with 
governance. Answer (b) is correct because  corporate governance. 
auditors do not generally initiate a discussion
on materiality, although they do occasionally 146. (a) The requirement is to determine the 
respond to such questions. See AU 380 for  information that an auditor should 
auditor communications with those charged  communicate to those charged with corporate 
with governance.  governance. Answer (a) is correct because both 
significant audit adjustments and 
144. (b) The requirement is to identify the correct
management’s consultation with other 
state­ment about an auditor’s required 
communication with those charged with  accountants about significant accounting 
governance. Answer (b) is correct because  matters should be communicated to an audit 
the communication must include significant  committee. See 
misstatements dis­covered, even if corrected AU 380 for these and other matters that should be
by management. Answer (a) is incorrect  so com­municated. 
because while such communications may be  E.2.  Reporting on Internal Control
com­municated to management, there is no 
such requirement. Answer (c) is incorrect  147. (d) The requirement is to identify the 
because disagreements with man­agement,  statement that should be included in a 
as well as the other required disclosures,  CPA’s report on a client’s internal control 
may be communicated either orally or in  over financial reporting. Answer (d) is 
writing. Answer (d) is incorrect because an  correct be­cause AT 501 requires that the 
auditor must recommunicate such  report include a comment on the inherent 
weaknesses in internal control. Also, see AU limitations of any internal control. 
380 for the matters communicated to those 
charged with governance.  148. (a) The requirement is to identify the 
statement that best describes a CPA’s 
145. (a) The requirement is to determine whether dis­ engagement to report on an entity’s internal 
agreements with management and initial selection of  control over financial reporting. Answer (a) 
signifi­cant accounting policies need to be  is cor­rect because the objective of an 
communicated to those charged with corporate  attestation engagement is to form an opinion
governance. Answer (a) is correct because  on the effectiveness of internal control. 
disagreements should be communicated directly to  Answer (b) is incorrect because no such 
those charged with corporate governance. Answer (b)  negative assurance 
MODULE 3  UNDERSTANDING INTERNAL CONTROL AND ASSESSING CONTROL RISK 107
exists (here a material weakness in internal 
control) the CPA should report directly upon
is provided based on an “audit” of the entity’s  the subject matter and not upon the 
internal control. Answer (c) is incorrect because such assertion. 
engagements do not project expected benefits of the 
entity’s internal control. Answer (d) is incorrect  E.3.  Effects of an Internal Audit Function
because such engagements are attestation 
153. (a) The requirement is to identify the most 
engagements, not consulting engagements.
likelyinformation a CPA would obtain in assessing the
149. (c) The requirement is to determine the correct statement  compe­tence of an internal auditor. Answer (a) is 
regarding an engagement to examine internal control.  correct because in assessing competence, an internal 
Answer (c) is correct because the procedures relat­ing to  auditor will consider the quality of working paper 
internal control will be more extensive when reporting on  documentation as well as a variety of other factors 
internal control as compared to procedures performed for  outlined in AU 322. Answer (b) is incorrect because 
a financial statement audit. This difference occurs  an organization’s commitment to integrity and ethi­cal
because during financial statement audits the auditor may  values, while important, does not bear as directly upon
decide not to perform tests of controls and may simply 
assess control risk at the maximum level. Conversely, in 
an engagement to report on internal control an auditor 
must perform additional tests of controls. Answer (a) is 
incorrect because such du­plication of procedures may 
not be necessary. Answer (b) is incorrect because a report
on internal control will not in general increase the 
reliability of the financial statements. Answer (d) is 
incorrect because, as indicated, the scope of procedures 
relating to internal control is more extensive, not more 
limited, than the assessment of control risk made dur­ing 
a financial statement audit. 

150. (d) The requirement is to identify the 
relationship between an examination of 
internal control and obtaining an 
understanding of internal control and 
assessing control risk as part of an audit. 
Answer (d) is correct because, while the 
scope and purpose differ between the two 
types of engage­ments, the procedures 
followed are similar. See AT 501 (or 
PCAOB Standard 5) for information on 
reporting on an audit of internal control. 
151. (b) The requirement is to describe the 
contents of a report on the study of internal 
control that is based on crite­ria established 
by governmental agencies. Answer (b) is 
correct because the report should indicate 
matters covered by the consideration and 
whether the auditor’s consideration included 
tests of controls with the procedures covered 
by his/her consideration. Additionally, the 
report should de­scribe the objectives and 
limitations of internal control and the 
accountant’s evaluation thereof; state the 
accountant’s conclusion, based on the 
agency’s criteria; and describe the purpose of
the report and state that it should not be used 
for any other purpose. Answer (a) is 
incorrect because the agency’s name should 
be included. Answer (c) is incorrect because 
a conclusion may be made relative to the 
agency’s criteria. Answer (d) is incorrect 
because the accountant should not assume 
responsibility for the comprehensiveness of 
the criteria. 
152. (b) AT 501 states that when a deviation from
the control criteria being reported upon 
degree of subjectivity involved in the 
internal auditor competence. Answers (c) and (d) are  evaluation of the audit evidence. Answer (c) 
incor­rect because the influence of management and  is correct because the existence of fixed 
the organiza­tional level to which the internal auditor  asset additions involves little subjectivity. 
reports are factors used to assess internal auditor  Answers (a) and (b) are incorrect because 
objectivity. AU 322 provides overall guidance on the  the existence of contingencies and the 
use of internal auditors. valuation of intangible assets are subjective 
and the risk of misstatement may be high. 
154. (d) The requirement is to determine whether
Answer (d) is incorrect because the 
the independent auditor may share  valuation of related­party transactions may 
responsibility with an en­tity’s internal  be very subjective due to the lack of an 
auditor for assessing inherent risk and  “arm’s­length” transaction. 
control risk. AU 322 requires that 
judgments about inherent and control risk  157. (d) AU 322 states that internal auditors may assist the 
always be those of the independent auditor.  CPA in obtaining an understanding of internal control,
It also requires that judgments about the  in performing tests of controls, and in performing 
materiality of mis­statements, the  substantive tests. 
sufficiency of tests performed, the valuation
of significant accounting estimates, and  158. (b) The requirement is to identify the type 
other matters af­fecting the auditor’s report  of infor­mation used by a CPA to assess the 
should always be those of the independent  competence of internal auditors. Answer (b)
auditor. See AU 322 for information on the  is correct because, along with various other 
external auditor’s consideration of the  factors, AU 322 indicates that the CPA 
internal audit func­tion.  should obtain evidence on the educational 
background and professional certification of
155. (d) The work of internal auditors may affect  the internal auditors when considering com­
the nature, timing and extent of the audit,  petence. Answers (a) and (c) are incorrect 
including (1) proce­dures the auditor  because analysis of organizational level to 
performs when obtaining an understanding  which the internal auditors report and 
of the entity’s internal control, (2)  policies on relatives are considered when 
procedures the auditor performs when  assessing in­ternal auditor objectivity. 
assessing risk, and (3) substantive proce­ Answer (d) is incorrect because access to 
dures the auditor performs.  sensitive records will not provide a CPA 
with in­formation on competence. 
156. (c) The requirement is to identify the 
circumstance in which an internal auditor’s  159. (d) The requirement is to identify the least 
work would most likely affect the nature,  likely source of information to the CPA on 
timing, and extent of a CPA’s auditing  an entity’s internal auditor’s competence and
proce­dures. When considering the effect of  objectivity. Answer (d) is correct because 
the internal auditors’ work, the CPA  analytical procedures do not ordinarily 
considers (1) the materiality of financial  provide in­formation on the internal auditor. 
statement amounts, (2) the risk of material  See AU 329 for informa­tion on analytical 
misstatement of the assertions, and (3) the  procedures. 
108 MODULE 3  UNDERSTANDING INTERNAL CONTROL AND ASSESSING CONTROL RISK
whether an auditor who audits the 
processing of transactions by a ser­vice 
160. (a) The requirement is to identify the  organization may issue a report on either,
characteristics of an internal auditor which  or both, of whether controls have been 
must be considered by an inde­pendent  implemented and control operating 
auditor who decides that the internal auditor’s effectiveness. Answer (a) is correct 
work might have a bearing on his/her  because AU 324 indicates that such 
procedures. Answer (a) is correct because  “service auditors” may issue either of the 
independent auditors must consider internal  two types of reports. 
auditor competence, objectivity, and work 
performance. Answer (b) is incorrect because 165. (a) The requirement is to identify the 
an independent auditor is less concerned  proper infor­mation to be included in a 
about internal auditor efficiency, although  service auditor’s report on whether a 
internal auditor experience will be considered client’s controls have been implemented. 
in the assess­ment of competence. Answers  Answer (a) is correct because such a report 
(c) and (d), while partially correct, are less  should include a description of the scope 
complete than answer (a).  and nature of the client’s proce­dures. 

161. (d) The requirement is to determine how a  166. (d)   The   requirement   is   to   determine   the


CPA should assess the objectivity of an  propriety   of   a   computer   “user”   auditor
internal auditor. An­swer (d) is correct  (Lake) making reference to a ser­ 
because when assessing the objectivity of an 
internal auditor the CPA should consider 
organizational status and policies for 
maintaining objectivity. Answers (a), (b), and
(c) are all incorrect because evaluating the 
quality control program, and examining and 
testing an internal auditor’s work all relate 
more directly to assessing internal auditor 
competence rather than to objectivity. See 
AU 322 for these and other factors 
considered when assessing inter­nal auditor 
competence. 
E.4. Reports on the Processing of Transactions by 
Ser­vice Organizations
162. (a) The requirement is to identify the 
information provided in a service auditor’s 
report which includes an opinion on a 
description of controls implemented. An­
swer (a) is correct since such a report 
includes a description of the scope and 
nature of the CPA’s procedures. Answers 
(b), (c), and (d) are all incorrect because they
suggest information not included in such a 
report. See AU 324 for information on the 
audit of service organizations. 
163. (a) The requirement is to identify a CPA’s 
reporting responsibility when reporting on 
internal control imple­mented for a service 
organization that processes payroll 
transactions. Answer (a) is correct because 
since the CPA is only expressing an opinion 
on whether controls have been implemented, 
a disclaimer should be provided on operating 
effectiveness. Answer (b) is incorrect because
no specific statement is made with respect to 
earlier objectives. Answer 
(c) is incorrect because controls relevant to financial 
state­ment assertions are not so identified. Answer (d) 
is incorrect because the assessed level of control risk is 
not disclosed. See AU 324 for information on 
processing of transactions by service organizations. 
164. (a) The requirement is to determine 
the report of the service auditor. See AU 324 for 
vice auditor’s (Cope) report. Answer (d) is correct  reports on the process­ing of transactions by service 
because the user auditor should not make reference to  organizations.
MODULE 3  UNDERSTANDING INTERNAL CONTROL AND ASSESSING CONTROL RISK 109

SOLUTIONS TO SIMULATIONS

Task­Based Simulation 1
Cash Receipts
and Billing
Authoritative
Literature

Help

Condition
1. Southwest is involved only in medical services and has not
diversified its operations. 
2. Insurance coverage for patients is verified and communicated to
the   clerks   by   the   office   manager   before   medical   services   are
rendered. 
3. The physician who renders the medical services documents the 
services on a prenumbered slip that is used for recording revenue
and as a receipt for the patient. 
4. Cash collection is centralized in that Clerk #2 receives the cash
(checks) from patients and records the cash receipt. 
5. Southwest extends credit rather than requiring cash or insurance in all
cases. 
6. The office manager extends credit on a case­by­case basis rather
than using a formal credit search and established credit limits. 
7. The office manager approves the extension of credit to patients
and   also   ap­proves   the   write­offs   of   uncollectible   patient
receivables. 
8. Clerk #2 receives cash and checks and prepares the daily bank deposit. 
9. Clerk #2 maintains the accounts receivable records and can
add or delete information on the PC. 
10. Prenumbered service slips are accounted for on a monthly basis by the 
outside accountant who is independent of the revenue generating and 
revenue recording functions. 
11. The   bank   reconciliation   is   prepared   monthly   by   the   outside
accountant   who   is   independent   of   the   revenue   generating   and
revenue recording functions. 
12. Computer passwords are only known to the individual employees
and   the   managing   partner   who   has   no   duties   in   the   revenue
recording functions. 
13. Computer software cannot be modified by Southwest’s employees. 
14. None of the employees who perform duties  in the revenue
generating and revenue recording are able to write checks. 
Strength Weakness Neither
Task­Based Simulation 2
Purchases and
Disbursements Authoritative
Literature Help

1. (C) Prepare purchase order—A trapezoid represents a manual operation. Here, a purchase order enters
the flowchart after this step; accordingly, a purchase order is being prepared. 
2. (R)  To vendor—A circle represents a connector, a symbol indicating that a document is entering or leaving that
portion of the flowchart. Here a copy of the purchase order is sent to the vendor to order the goods. This must be the
case, because other­wise the vendor would not be informed of the order. 
3. (F) Prepare receiving report—A trapezoid represents a manual operation. Here, a receiving report enters
the flowchart after this step; accordingly, a receiving report is being prepared. Also, note above this step
that goods are received, the point at which one would expect preparation of a receiving report. 
4. (L) From purchasing—A circle represents a connector, a symbol indicating that a document is entering or
leaving that portion of the flowchart. The document here is from purchasing because below the connector 
is requisition No. 1, which pur­chasing has sent with the purchase order No. 5 to vouchers payable, as 
evidenced by the connector in the bottom far right under purchasing. 
5. (M) From receiving—A circle represents a connector, a symbol indicating that a document is entering or leaving that
por­tion of the flowchart. The document here is from receiving because under the receiving portion of the flowchart,
approximately 
110 MODULE 3  UNDERSTANDING INTERNAL CONTROL AND ASSESSING CONTROL RISK

3/4 of the way down, we see a connector indicating that receiving report No. 1 is being sent to vouchers payable. Also, toward 
the bottom under the vouchers payable portion of the flowchart, we see that receiving report No. 1 is indeed in the system.
6. (O) Purchase order No. 5—We know from item 4 that this document was sent from purchasing, and we know that 
purchasing has sent to vouchers payable requisition No. 1 and purchase order No. 5. Since requisition No. 1 is 
labeled on the flowchart, this must be purchase order No. 5. 
7. (P) Receiving report No. 1—We know from item 5 that this document was sent from receiving, and since we know
that receiving has sent receiving report No. 1 to vouchers payable this must be that document. 
8. (B) Prepare and approve voucher—A trapezoid represents a manual operation. Here, an approved voucher enters
the flowchart after this step; accordingly, a voucher is being prepared and approved in this step. 
9. (T) Unpaid voucher file, filed by due date—The triangle symbol represents a file. Entering this file are the approved 
but unpaid vouchers with the support of their invoices, receiving reports, purchase orders and purchase requisitions. 
Because these vouchers are sent to the treasurer in order of due date (the bottom, right symbol under vouchers 
payable) this file is the unpaid voucher file, filed by due date. 
10. (S)  Treasurer—Because the  unpaid  vouchers (the  “voucher  package”) was  sent  from  vouchers  payable  to  the
treasurer, this is the treasurer. 
11. (I) Sign checks and cancel voucher package documents—A trapezoid represents a manual operation. Here, the 
operation prior to 11 involves a review of documents and preparation of a check and a remittance advice. After this 
operation the docu­ments changed are a “canceled voucher package file” and a “signed check”; accordingly, checks 
are being signed and the voucher package is being canceled. 
12. (K) Canceled voucher package—After step 11, the check copy, remittance advice No. 1, the signed check, and remittance 
advice No. 2 exit on the far right. Accordingly, item 12 is the voucher package, now canceled as evidenced by the description 
below the triangular file symbol. 

Task­Based Simulation 3
Research
Authoritative
Literature Help

(A) (B) (C) (D) (E)  (F) (G) (H)
1. Which title of the Professional Standards addresses this issue?
2. Enter the exact section and paragraph numbers that describe the limitations. 319 64­66

Task­Based Simulation 4
Sales/Shipping Authoritative
Process Literature Help

1. (M) Perform customer credit check—The customer credit file is being accessed, making it likely that a credit check is oc­curring.
2. (Z)  Open order file—The processing to the right of #2 begins with “open orders,” making this an open order file. 
3. (L) Match customer purchase order with sales order—Two copies of the sales order are being combined with the
cus­tomer purchase order through a manual operation (the trapezoid). 
4. (B) Verify agreement of sales order and shipping document—This manual operation (trapezoid) includes two copies
of the shipping document being combined with the sales order. 
5. (H)  Release   goods   for   shipment—The   department   is   the   warehouse   and   shipping   department,   and   out   of   this
operation is “shipping information”; accordingly goods are being released for shipment. 
6. (S)  Master price file—The operation below #6 includes entering price data; since the first two files being accessed are the
accounts receivable master file and the shipping file, this third file must include prices. 
7. (O) Prepare sales invoice—Since a document is being prepared through this computerized billing program, it is the
sales invoice. 
8. (U)  Sales invoice—A sales invoice is normally sent to the customer. 
9. (I) To accounts receivable department—This copy of the sales invoice informs accounts receivable that the sale has
been both processed and shipped. 
MODULE 3  UNDERSTANDING INTERNAL CONTROL AND ASSESSING CONTROL RISK 111

10. (Q) General ledger master file—Because the processing step below includes updating of master files, this is the
general ledger master file. 
11. (N)  Prepare sales journal—Sales transactions are being processed; accordingly a sales journal is prepared. 
12. (T)  Sales journal—From above, a sales journal was prepared; the accounting department will receive the sales journal. 
13. (Y) Aged trial balance—In the processing step above, an aged trial balance of accounts receivable is prepared; the
credit department will receive such a report. 

Task­Based Simulation 5
Research
Authoritative
Literature Help

(A) (B) (C) (D) (E) (F) (G) (H)


1. Which title of the Professional Standards addresses this issue and will be
helpful in responding to him?
2. Enter the exact section number that provides the appropriate guidance. 501

Task­Based Simulation 6
Purchasing Receiving,
and Accounts Payable Authoritative
Literature Help

1. Disagree.  Someone independent of requisitioning should select the supplier. 
2. Agree. 
3. Disagree.  Often, factors in addition to cost are considered (e.g., quality, dependability). 
4. Agree. 
5. Agree. 
6. Agree. 
7. Disagree.   A  comparison   of  quantities   is   not  possible   because   quantity   is   blacked   out  on   the   purchase   order
provided to receiving. 
8. No receiving report is ordinarily necessary for purchases of services. 
9. Agree. 
10. Agree. 
11. Agree. 
12. Disagree.  The reconciliation should be performed by an independent party. 
13. Disagree.  Documentation should be marked “paid” by the individual making the payment. 
14. Agree. 
15. Agree. 

R
e
A
u
L
i
(
112 MODULE 3  UNDERSTANDING INTERNAL CONTROL AND ASSESSING CONTROL RISK

Task­Based Simulation 8
Internal Control Related Authoritative
Matters Communication Literature Help

AU 325.25 includes the matters required to be included in the written communication regarding significant 
deficiencies and material weaknesses indentified during an audit as
• State that the purpose of the audit was to express an opinion on the financial statements, but not to express
an opinion on the effectiveness of the entity’s internal control over financial reporting. 
• State that the auditor is not expressing an opinion on the effectiveness of internal control. 
• Include the definition of the terms significant deficiency and, where relevant, material weakness. 
• Identify the matters that are considered to be significant deficiencies and, if applicable, those that are
considered to be material weaknesses. 
• State that the communication is intended solely for the information and use of management, those 
charged with governance, and others within the organization and is not intended to be and should not 
be used by anyone other than those specified parties. If an entity is required to furnish such auditor 
communications to a governmental authority, specific reference to such governmental authorities may 
be made. 
Internal controls Related matters Comment
State that the purpose of the audit was to express The latter part of the sentence is
an opinion on the financial statements, and to incorrect since no opinion on
express an opinion on the effectiveness of the Excluded internal control effectiveness is
entity’s internal control over financial reporting. issued.
Identify, if applicable, items that are considered to
be material weaknesses. Included Required
State that the author is not expressing an opinion
on the effectiveness of internal control. Included Required
Include the definition of the term significant
deficiency. Included Required
Include the definition of the term material
weakness, where relevant. Included Required
State that the author is expressing an unqualified The auditor expresses no opinion
opinion on the effectiveness of internal control. Excluded on internal control.
State that the communication is intended solely for The communication is not
management and external parties. Excluded intended for external parties.
Identify the matters that are considered to be Included Required
significant deficiencies.

Task­Based Simulation 9
Research
Authoritative
Literature Help

(A) (B) (C) (D) (E)  (F) (G) (H)
1.   Which title of the Professional Standards addresses this issue?
2.   Enter the exact section and paragraph with helpful information. 5 14
MODULE 4   RESPONDING TO RISK ASSESSMENT 113

RESPONDING TO RISK ASSESSMENT:
EVIDENCE ACCUMULATION AND EVALUATION
to   the   amount   of   corroborative
MULTIPLE­CHOICE QUESTIONS (1­169)
evidence obtained. 
1. Which   of   the   following   best   describes d. Effective   internal   control   provides   more
what   is   meant   by   the   term   generally assurance   about   the   reliability   of   audit
accepted auditing standards?  evidence. 
a. Procedures   to   be   used   to 6. Which of the following statements relating
gather   evidence   to   support
to the ap­propriateness of audit evidence is
financial statements. 
always true? 
b. Measures   of   the   quality   of   the
a. Audit   evidence   gathered   by   an
auditor’s perfor­mance. 
auditor   from   out­side   an
c. Pronouncements   issued   by   the   Auditing enterprise is reliable. 
Standards Board. 
b. Accounting data developed under 
d. Rules   acknowledged   by   the satisfactory conditions of internal 
accounting   profession   because   of control are more relevant than data
their universal application.  developed under unsatisfactory 
2. Which of the following is not an assertion internal control conditions. 
relating to classes of transactions?  c. Oral   representations   made   by
a. Accuracy.  management   are  not  valid
b. Consistency.  evidence.
c. Cutoff. 
d. Occurrence. 
3. Which   of   the   following   is   a   general
principle relating to the reliability of audit
evidence? 
a. Audit evidence obtained from 
indirect sources rather than directly
is more reliable than evidence 
obtained directly by the auditor. 
b. Audit   evidence   provided   by
copies   is   more   reliable   than
that provided by facsimiles. 
c. Audit evidence obtained from 
knowledgeable independent sources
outside the client company is more 
reliable than audit evidence 
obtained from nonindependent 
sources. 
d. Audit evidence provided by 
original documents is more reliable
than audit evidence generated 
through a system of effective 
controls. 
4. Which   of   the   following   types   of   audit
evidence is the most persuasive? 
a. Prenumbered client purchase order forms. 
b. Client   work   sheets   supporting   cost
allocations. 
c. Bank statements obtained from the client. 
d. Client representation letter. 
5. Which   of   the   following   presumptions   is
correct   about   the   reliability   of   audit
evidence? 
a. Information   obtained
indirectly   from   outside
sources   is   the   most   reliable
audit evidence. 
b. To be reliable, audit evidence
should   be   con­vincing   rather
than persuasive. 
c. Reliability of audit evidence refers
d. Evidence   gathered   by   auditors key factors or assumptions used in
must be both valid and relevant to preparing the estimates. 
be considered appropriate.  c. Test   the   calculations   used   by
management   in   de­veloping   the
7. Which   of   the   following   types   of   audit estimates. 
evidence is the least persuasive? d. Obtain   an   understanding   of   how
a. Prenumbered purchase order forms.  management   de­veloped   its
b. Bank statements obtained from the client.  estimates. 
c. Test   counts   of   inventory
performed by the audi­tor.  11. In evaluating the reasonableness of an 
d. Correspondence from the client’s accounting esti­mate, an auditor most likely 
attorney about litigation.  would concentrate on key fac­tors and 
assumptions that are 
8. In evaluating the reasonableness of an  a. Consistent with prior periods. 
entity’s ac­counting estimates, an auditor  b. Similar to industry guidelines. 
normally would be concerned about  c. Objective and not susceptible to bias. 
assumptions that are  d. Deviations from historical patterns. 
a. Susceptible to bias. 
b. Consistent with prior periods.  12. In evaluating an entity’s accounting 
c. Insensitive to variations.  estimates, one of an auditor’s objectives is to
d. Similar to industry guidelines.  determine whether the estimates are 
a. Not subject to bias. 
9. Which   of   the   following   is   not   a   basic b. Consistent with industry guidelines. 
procedure used in an audit?  c. Based on objective assumptions. 
a. Risk assessment procedures.  d. Reasonable in the circumstances. 
b. Substantive procedures. 
c. Tests of controls.  13. In   testing   the   existence   assertion   for   an
d. Tests of direct evidence.  asset, an audi­tor ordinarily works from the 
a. Financial   statements   to   the
10. Which of the following procedures would  potentially unrecorded items. 
an auditor ordinarily perform first in  b. Potentially unrecorded items to the financial
evaluating management’s ac­counting  state­ments. 
estimates for reasonableness?  c. Accounting   records   to   the   supporting
a. Develop   independent evidence. 
expectations   of   manage­ment’s d. Supporting   evidence   to   the   accounting
estimates.  records. 
b. Consider the appropriateness of the
114 MODULE 4   RESPONDING TO RISK ASSESSMENT
b. Adhering to a system of quality
control. 
14. A client uses a suspense account  c. Preparing   auditor   working
for unresolved ques­tions whose  papers. 
final accounting has not been  d. Increasing the level of detection
determined. If a balance remains  risk. 
in the suspense account at year­
end, the auditor would be most  18. An entity’s income statements 
concerned about  were misstated due to the 
a. Suspense   debits   that recording of journal entries that 
management believes involved debits and credits to 
will   benefit   future an unusual combination of 
operations.  expense and revenue accounts. 
b. Suspense debits that the The auditor most likely could 
auditor   verifies   will have detected this fraudulent 
have realizable value to financial reporting by 
the client.  a. Tracing   a   sample   of
c. Suspense   credits   that journal   entries   to   the
management   believes   should   be general ledger. 
classified as “Current liability.”  b. Evaluating   the   effectiveness   of
d. Suspense   credits   that internal control. 
the   auditor   determines c. Investigating   the
to be customer deposits. reconciliations   between
control­ling   accounts
15. Which of the following would and subsidiary records. 
not be considered an analytical d. Performing   analytical
procedure?  procedures   designed   to
a. Estimating payroll  dis­close   differences
expense by multiplying  from expectations. 
the number of 
19. Auditors try to identify 
employees by the 
average hourly wage  predictable relationships when 
rate and the total hours  using analytical procedures. 
worked.  Relationships involving trans­
actions from which of the 
b. Projecting an error rate
following accounts most likely 
by   comparing   the
would yield the highest level of 
results   of   a   statistical
evidence? 
sample with the actual
population a. Accounts receivable. 
characteristics.  b. Interest expense. 
c. Computing   accounts   receivable c. Accounts payable. 
turnover   by   divid­ing   credit   sales d. Travel   and   entertainment
by the average net receivables.  expense. 
d. Developing   the 20. Analytical procedures used in the
expected   current   year overall review stage of an audit
sales based on the sales generally include 
trend   of   the   prior   five
years. 
16. What type of analytical 
procedure would an auditor most
likely use in developing 
relationships among balance 
sheet accounts when reviewing 
the financial statements of a 
nonpublic entity? 
a. Trend analysis. 
b. Regression analysis. 
c. Ratio analysis. 
d. Risk analysis. 
17. An auditor may achieve audit
objectives   related   to
particular assertions by 
a. Performing   analytical
procedures. 
sence   of   known
a. Gathering   evidence conditions   to   the
concerning   account contrary. 
balances   that  have  not d. These procedures cannot replace
changed from the prior tests   of   balances   and
year.  transactions. 
b. Retesting   control
procedures   that 23. For all audits of financial 
appeared   to   be statements made in accor­dance 
ineffective   during   the with generally accepted auditing 
assessment   of   control standards, the use of analytical 
risk.  procedures is required to some 
c. Considering   unusual extent 
or unexpected account In the As a In the
bal­ances   that   were planning stage substantive test review stage
not  previously a. Yes No Yes
b. No Yes No
identified. 
c. No Yes Yes
d. Performing   tests   of
d. Yes No No
transactions   to
corroborate 24. An   auditor’s   analytical
management’s procedures most likely would be
financial   statement facilitated if the entity 
assertions.  a. Segregates   obsolete
inventory   before   the
21. Which of the following tends to  physical   inventory
be most predictable for purposes  count. 
of analytical procedures applied  b. Uses   a   standard   cost
as substantive tests?  system   that   produces
a. Relationships   involving   balance vari­ance reports. 
sheet accounts.  c. Corrects   material
b. Transactions   subject   to weaknesses in internal
management discretion.  control   before   the
c. Relationships beginning of the audit.
involving   income d. Develops its data from
statement ac­counts. sources   solely   within
d. Data   subject   to   audit   testing   in the entity. 
the prior year. 
25. Analytical procedures performed 
22. A   basic   premise   underlying   the in the overall review stage of an 
application   of   analyti­cal audit suggest that several 
procedures is that  accounts have unex­pected 
a. The study of financial relationships. The results of these
ratios is an acceptable procedures most likely would 
al­ternative   to   the indicate that 
investigation   of a. Irregularities   exist
unusual fluctua­tions.  among   the   relevant
b. Statistical   tests   of account balances. 
financial   information b. Internal   control
may   lead   to   the activities   are  not
discovery   of   material operating effec­tively. 
misstatements   in   the c. Additional   tests   of   details   are
financial statements.  required. 
c. Plausible   relationships d. The  communication
among   data   may with   the   audit
reason­ably   be committee   should
expected   to   exist   and be revised. 
continue   in   the   ab­
MODULE 4   RESPONDING TO RISK ASSESSMENT 115
to 
a. Comply   with
26. Which of the following  generally   accepted
comparisons would an auditor  auditing stan­dards. 
most likely make in evaluating an b. Attain   assurance
entity’s costs and ex­penses?  about   the   reliability
a. The   current   year’s of   the   ac­counting
accounts   receivable system. 
with   the   prior   year’s c. Detect   material
accounts receivable.  misstatements in the
b. The   current   year’s financial statements.
payroll   expense   with d. Evaluate   whether
the prior year’s payroll management’s   policies
expense.  and   pro­cedures
c. The   budgeted   current operated effectively. 
year’s   sales   with   the
prior year’s sales.  30. In   the   context   of   an   audit   of
d. The budgeted current financial statements, sub­stantive
year’s   warranty tests are audit procedures that 
expense   with   the a. May be eliminated under certain
conditions. 
current   year’s
b. Are   designed   to
contingent liabilities. 
discover   significant
27. To be effective, analytical procedures in the subsequent events. 
overall review stage of an audit engagement c. May   be   either   tests   of
should be performed by  transactions,   direct
a. The   staff   accountant tests   of   financial
who   performed   the balances,   or   analytical
substan­tive   auditing tests. 
procedures.  d. Will   increase
b. The   managing   partner proportionately   with
who   has   responsibility the   auditor’s   reliance
for   all   audit on internal control. 
engagements   at   that
31. The   auditor   will   most   likely
practice office. 
perform   extensive   tests   for
c. A   manager   or   partner possible understatement of 
who   has   a
a. Revenues. 
comprehensive
knowledge   of   the b. Assets. 
client’s   business   and c. Liabilities. 
industry.  d. Capital. 
d. The CPA firm’s quality 32. Which of the following 
control manager or  procedures would an auditor most
partner who has  likely perform in auditing the 
responsibility for the  statement of cash flows? 
firm’s peer review  a. Compare the amounts included 
program.  in the statement of cash flows to 
28. Which   of   the   following   is   the similar amounts in the prior 
best example of a sub­stantive year’s statement of cash flows. 
test? 
a. Examining   a   sample   of
cash disbursements to test
whether   expenses   have
been properly approved. 
b. Confirmation   of   balances   of
accounts receivable. 
c. Comparison   of
signatures   on   checks
to a list of authorized
signers. 
d. Flowcharting of the client’s cash
receipts system. 
29. The   objective   of   tests   of
details   of   transactions   per­
formed as substantive tests is
relevance   to
b. Reconcile   the   cutoff forthcoming
bank   statements   to audits. 
verify   the   accuracy   of c. Until 3 years after
the   year­end   bank the   client   selects
balances.  another auditor. 
c. Vouch   all   bank d. Working   papers   must   be
transfers   for   the   last maintained indefinitely. 
week  of  the   year  and
first   week   of   the 36. Which of the following pairs of
subsequent year.  accounts would an auditor most
d. Reconcile the amounts likely   analyze   on   the   same
included in the  working paper? 
statement of cash  a. Notes   receivable   and   interest
flows to the other  income. 
financial statements’  b. Accrued   interest
balances and amounts. receivable   and
accrued   interest
33. In determining whether  payable. 
transactions have been re­corded,  c. Notes   payable   and   notes
the direction of the audit testing  receivable. 
should be from the  d. Interest   income   and   interest
a. General ledger balances.  expense. 
b. Adjusted trial balance. 
c. Original source documents. 37. An   auditor’s   working   papers   serve
d. General journal entries.  mainly to 
a. Provide   the   principal
34. Which   statement   is   correct support   for   the
concerning the deletion of audit auditor’s re­port. 
documentation?  b. Satisfy   the   auditor’s
a. Superseded audit documentation responsibilities
should   always   be   deleted   from concerning   the   Code
the audit file.  of   Professional
b. After the audit  Conduct. 
file has been  c. Monitor   the
completed, the  effectiveness   of   the
auditor should  CPA   firm’s   qual­ity
not delete or  control procedures. 
discard audit  d. Document the level of
documentation.  independence
c. Auditors should use  maintained   by   the
professional skepticism  auditor. 
in determining which 
audit documentation  38. The   permanent   file   of   an
should be deleted.  auditor’s   working   papers
d. Audit generally would not include 
documentation a. Bond indenture agreements. 
should   never   be b. Lease agreements. 
deleted   from   the c. Working trial balance. 
audit file.  d. Flowchart of internal control. 
35. Ignoring any particular  39. An auditor ordinarily uses a 
legal or regulatory  working trial balance 
requirement, audit  resembling the financial 
documentation should be  statements without footnotes, 
retained  but containing columns for 
a. A minimum of five years.  a. Cash   flow   increases   and
b. As   long   as   lead decreases. 
schedules   have b. Audit objectives and assertions. 
116 MODULE 4   RESPONDING TO RISK ASSESSMENT
42. Which of the following factors 
c. Reclassifications   and most likely would af­fect an 
adjustments.  auditor’s judgment about the 
d. Reconciliations and tick marks.  quantity, type, and content of 
the auditor’s working papers? 
40. Which   of   the   following   is   least a. The   assessed   level   of   control
likely   to   be   a   factor   in   the risk. 
auditor’s   decision   about   the b. The   likelihood   of   a
extent of the documentation of a review   by   a
particular audit area?  concurring   (sec­ond)
a. The   risk   of   material partner. 
misstatement.  c. The   number   of   personnel
b. The   extent   of   the assigned to the audit. 
judgment   involved   in d. The content of the management
perform­ing   the representation let­ter. 
procedures. 
c. The   nature   and   extent   of 43. The   audit   working   paper   that
exceptions identified.  reflects   the   major   com­ponents
d. Whether   or   not   the of   an   amount   reported   in   the
client   has   an financial statements is the 
internal   audit a. Interbank transfer schedule. 
function.  b. Carryforward schedule. 
c. Supporting schedule. 
41. Which of the following is  d. Lead schedule. 
required documentation in an 
audit in accordance with  44. Which of the following 
generally accepted auditing  documentation is required for an 
standards?  audit in accordance with generally
a. A flowchart or narrative  accepted auditing standards? 
of the accounting system  a.
describing the recording  b.
and classification of  c.
transactions for financial 
reporting. 
b. An audit program 
setting forth in detail the
proce­dures necessary to
accomplish the 
engagement’s 
objectives. 
c. A planning 
memorandum 
establishing the timing 
of the audit procedures 
and coordinating the as­
sistance of entity 
personnel. 

A flowchart or an internal control questionnaire 
that evaluates the effectiveness of the entity’s 
controls.
A client engagement letter that summarizes the 
timing and details of the auditor’s planned field­
work.
An indication in the working papers that the ac­
counting records agree or reconcile with the fi­
nancial statements.
The  basis  for  the  auditor’s  conclusions  when  the
assessed  level   of  control   risk  is   at  the   maximum
level for all financial statement assertions.
d. An internal control 
questionnaire 
identifying con­trols that
assure specific 
objectives will be 
achieved. 
b. Cash   in   bank   and   collateral  for
45. No  deletions of audit  documentation loans. 
are allowed after  c. Inventory   on
the  consignment   and
a. Client’s year­end.  contingent liabili­ties. 
b. Documentation completion date.  d. Accounts   receivable
c. Last   date   of   significant and   accrued   interest
fieldwork.  receiv­able. 
d. Report release date.  49. The   usefulness   of   the   standard
46. Under the requirements of the PCAOB,  bank   confirmation  re­quest  may
audit documentation must contain sufficient be   limited   because   the   bank
information to allow what type of auditor to employee   who   com­pletes   the
understand the nature, timing, extent, and  form may 
results of procedures performed?  a. Not   believe   that   the
a. An   experienced   audit   team bank   is   obligated   to
member.  verify   confidential
information to a third
b. An   experienced
party. 
auditor  having  no
b. Sign   and   return   the
previous
connection   with form   without
the engagement.  inspecting the accuracy
c. Any certified public accountant.  of   the   client’s   bank
d. An   auditor   qualified   as   a   peer reconciliation. 
review specialist.  c. Not have access to the
client’s   cutoff   bank
47. Audit documentation for audits  state­ment. 
performed under the  d. Be   unaware   of   all   the
requirements of the Public  financial   relationships
Company Accounting Oversight  that the bank has with
Board should be retained for  the client. 
a. The   shorter   of   five
years,   or   the   period 50. An auditor most likely would 
required by law.  limit substantive audit tests of 
b. Seven years.  sales transactions when control 
risk is assessed as low for the 
c. The   longer   of   seven
occurrence assertion concerning
years,   or   the   period
sales transac­tions and the 
required by law. 
auditor has already gathered 
d. Indefinitely. 
evidence sup­porting 
48. Which of the following sets of a. Opening   and   closing   inventory
information   does   an   auditor balances. 
usually confirm on one form?  b. Cash   receipts   and   accounts
a. Accounts   payable   and   purchase receivable. 
commitments.  c. Shipping and receiving activities.
d. Cutoffs of sales and purchases. 

d.

Items 51 and 52 are based on the following:
The information below was taken from the bank transfer schedule prepared during the audit of 
Fox Co.’s financial statements for the year ended December 31, 2005. Assume all checks are 
dated and issued on December 30, 2005.
MODULE 4 RESPONDING TO RISK ASSESSMENT 117

Bank accounts Disbursement date Receipt date


Check Per Per Per Per
no. From To books bank books bank
101 National Federal Dec. 30 Jan. 4 Dec. 30 Jan. 3
202 County State Jan. 3 Jan. 2 Dec. 30 Dec. 31
303 Federal American Dec. 31 Jan. 3 Jan. 2 Jan. 2
404 State Republic Jan. 2 Jan. 2 Jan. 2 Dec. 31
indicate kiting? 
51. Which of the following checks might a. #101 and #303. 
b. #202 and #404.  52. Which of the following checks 
c. #101 and #404.  illustrate deposits/transfers in transit at 
d. #202 and #303.  December 31, 2005?
a. #101 and #202. 
b. #101 and #303. 
c. #202 and #404. 
d. #303 and #404. 

56. A cash shortage may be concealed
53. An   auditor   should   trace   bank by transporting funds from one 
transfers  for  the last part of  the location to another or by 
audit period and first part of the converting negotia­ble assets to 
subsequent   period   to   detect cash. Because of this, which of 
whether  the following is vital? 
a. The   cash   receipts a. Simultaneous confirmations. 
journal   was   held   open b. Simultaneous   bank
for a few days after the reconciliations. 
year­end.  c. Simultaneous verification. 
b. The last checks recorded before d. Simultaneous   surprise   cash
the   year­end   were   actually count. 
mailed by the year­end. 
c. Cash   balances   were   overstated 57. The primary purpose of sending a standard 
because of kiting.  confirma­tion request to financial 
d. Any   unusual   payments institutions with which the client has done 
to   or   receipts   from business during the year is to 
related parties occurred. a. Detect   kiting   activities
that may otherwise  not
54. To gather evidence regarding the  be discovered. 
balance per bank in a bank  b. Corroborate
reconciliation, an auditor would  information
examine all of the following  regarding deposit and
except loan balances. 
a. Cutoff bank statement.  c. Provide   the   data
b. Year­end bank statement.  necessary to prepare a
c. Bank confirmation.  proof of cash. 
d. General ledger.  d. Request   information
about   contingent
55. Which   of   the   following   cash liabilities and secured
transfers   results   in   a   mis­ transactions. 
statement   of   cash   at   December
31, 2005?  58. An auditor observes the mailing of monthly
Bank Transfer Schedule statements to a client’s customers and 
Disbursement Receipt reviews evidence of follow­up on errors 
reported by the customers. This test of 
Recorded Paid by Recorded controls most likely is performed to support
Transfer in books bank in books management’s financial statement 
a. 12/31/05 1/4/06 12/31/05 assertion(s) of 
b. 1/4/06 1/5/06 12/31/05
c. 12/31/05 1/5/06 12/31/05
d. 1/4/06 1/11/06 1/4/06
Presentation Existence
and disclosure occurrence
a. Yes Yes
b. Yes No
c. No Yes
d. No No

Items 59 and 60 are based on the following:
Miles Company
BANK TRANSFER SCHEDULE
December 31, 2005
Check Bank accounts Date disbursed per Date deposited per
no. From To Amount Books Bank Books Bank
2020 1st Natl. Suburban $32,000 12/31 1/5 12/31 1/3
2021 1st Natl. Capital 21,000 12/31 1/4 12/31 1/3
3217 2nd State Suburban 6,700 1/3 1/5 1/3 1/6
0659 Midtown Suburban 5,500 12/30 1/5 12/30 1/3
118 MODULE 4   RESPONDING TO RISK ASSESSMENT

60. The tick mark most likely indicates that
59. The tick mark most likely indicates that the amountwas traced to the
the amountwas traced to the a. Deposits   in   transit   of   the
a. December   cash   disbursements applicable bank recon­ciliation. 
journal.  b. December cash receipts journal. 
b. Outstanding   check   list   of   the c. January cash receipts journal. 
applicable bank rec­onciliation.  d. Year­end bank confirmations. 
c. January   cash   disbursements
journal. 
d. Year­end bank confirmations. 

misstatements, required 
substantial time to resolve. In 
61. Which   of   the   following defining the sam­pling unit for the
statements is correct concern­ing current year’s audit, the auditor 
the use of negative confirmation most likely would choose 
requests?  a. Individual overdue balances. 
a. Unreturned   negative b. Individual invoices. 
confirmation   requests c. Small account balances. 
rarely   provide d. Large account balances. 
significant   explicit
evidence.  64. Confirmation is most likely to be 
b. Negative   confirmation   requests a relevant form of evidence with 
are effective when detection risk regard to assertions about 
is low.  accounts receivable when the 
c. Unreturned   negative auditor has concerns about the 
confirmation   requests receivables’ 
indi­cate that alternative a. Valuation. 
procedures   are b. Classification. 
necessary.  c. Existence. 
d. Negative confirmation requests  d. Completeness. 
are effective when 
understatements of account  65. An   auditor   should   perform
balances are sus­pected.  alternative   procedures   to
substantiate   the   existence   of
62. When an auditor does not receive accounts receivable when 
replies to positive requests for  a. No  reply  to  a   positive
year­end accounts receivable  confirmation request is
confirmations, the auditor most  re­ceived. 
likely would  b. No reply  to  a  negative
a. Inspect   the   allowance confirmation   request   is
account   to   verify re­ceived. 
whether   the   accounts c. Collectibility   of   the   receivables
were   subsequently is in doubt. 
written off.  d. Pledging   of   the   receivables   is
b. Increase the assessed probable. 
level of detection risk
for the valuation and 66. Which of the following 
completeness procedures would an auditor most
assertions.  likely perform for year­end 
c. Ask   the   client   to accounts receivable con­
contact   the   customers firmations when the auditor did 
to   request   that   the not receive replies to second 
confirmations   be requests? 
returned. 
d. Increase   the   assessed
level of inherent risk for
the revenue cycle. 
63. In confirming a client’s accounts 
receivable in prior years, an 
auditor found that there were 
many differences between the 
recorded account balances and the
confirmation replies. These 
differences, which were not 
a. Review   the   cash confirma­tions   to   be
receipts   journal   for exceptions. 
the   month   prior   to d. Mail   second   requests   to   the   e­
the year­end.  mail respondents. 
b. Intensify   the   study   of
internal   control 69. To reduce the risks associated with 
concerning the revenue accepting fax re­sponses to requests for 
cycle.  confirmations of accounts receivable, an 
c. Increase the assessed auditor most likely would 
level of detection risk a. Examine the shipping documents
for   the   existence that   provide   evi­dence   for   the
assertion.  existence assertion. 
d. Inspect the shipping b. Verify   the   sources
records and   contents   of   the
documenting   the faxes   in   telephone
merchandise sold to calls to the senders. 
the debtors.  c. Consider the faxes to be
nonresponses   and
67. In which of the following  evalu­ate   them   as
circumstances would the use of  unadjusted differences. 
the negative form of accounts  d. Inspect the faxes for 
receivable confirmation most  forgeries or alterations 
likely be justified?  and consider them to be
a. A substantial number  acceptable if none are 
of accounts may be in  noted. 
dis­pute and the 
accounts receivable  70. In auditing accounts receivable,
balance arises from  the   negative   form   of
sales to a few major  confirmation   request   most
customers.  likely would be used when 
b. A substantial number  a. The total recorded amount of 
of accounts may be in  accounts receivable is immaterial
dis­pute and the  to the financial statements taken 
accounts receivable  as a whole. 
balance arises from  b. Response   rates   in   prior
sales to many  years   to   properly
customers with small  designed   positive
bal­ances.  confirmation   requests
c. A small number of  were inadequate. 
accounts may be in 
dispute and the 
accounts receivable 
balance arises from 
sales to a few major 
customers. 
d. A small number of 
accounts may be in 
dispute and the 
accounts receivable 
balance arises from 
sales to many 
customers with small 
balances. 
68. To   reduce   the   risks   associated
with accepting e­mail responses
to requests for confirmation of
accounts receiv­able, an auditor
most likely would 
a. Request the senders to
mail the original forms
to the auditor. 
b. Examine
subsequent   cash
receipts  for the ac­
counts in question. 
c. Consider   the   e­mail
responses   to   the
MODULE 4   RESPONDING TO RISK ASSESSMENT 119

c. Recipients are likely to return positive confirma­ 75.   An auditor most likely would make
inquiries of pro­
tion requests without verifying  duction and sales personnel concerning 
the accuracy of the information. possible obsolete or slow­moving 
d. The   combined   assessed inventory to support management’s 
level of inherent risk and financial statement assertion of
control   risk   relative   to a. Valuation. 
accounts   receivable   is b. Rights. 
low.  c. Existence. 
d. Presentation. 
71. Under which of the following circumstances
would the use of the blank form of 
confirmations of accounts receiv­able most 
likely be preferable to positive 
confirmations? 
a. The recipients are likely to sign the confirmations76.   While observing a client’s annual
physical inventory,
without devoting proper  receivable to encourage timely 
attention to them. consideration by the recipients? 
b. Subsequent   cash a. “This is not a request 
receipts   are   unusually for payment; 
difficult to verify.  remittances should not 
c. Analytical be sent to our auditors 
procedures   indicate in the enclosed 
that  few   excep­tions envelope.” 
are expected. 
b. “Report any differences 
d. The combined assessed on the enclosed 
level   of   inherent   risk
statement directly to our 
and control risk is low. 
auditors; no reply is 
72. In   confirming   accounts necessary if this amount 
receivable, an auditor decided to agrees with your 
confirm   customers’   account records.” 
balances   rather   than   individ­ual c. “If you do not report 
invoices. Which of the following any differences within 
most   likely   would   be   included fif­teen days, it will be 
with   the   client’s   confirmation assumed that this 
letter?  statement is correct.” 
a. An   auditor­prepared d. “The following invoices
letter  explaining  that a have been selected for 
non­response   may confirmation and 
cause an inference that represent amounts that 
the   account   balance   is are over­due.” 
correct. 
b. A client­prepared letter 74. Which   of   the   following
reminding the customer strategies   most   likely   could
that a nonresponse will  improve   the   response   rate   of
cause a second request  the   confirmation   of   accounts
to be sent.  receivable? 
c. An auditor­prepared  a. Including a list of items
letter requesting the cus­ or   invoices   that
tomer to supply missing  constitute   the   account
and incorrect  balance. 
information directly to  b. Restricting   the
the auditor.  selection  of accounts
d. A   client­prepared to   be   con­firmed   to
statement   of   account those customers with
showing   the   details   of relatively   large
the   customer’s   account balances. 
balance.  c. Requesting customers to
respond to the confirma­
73. Which of the following statements would  tion requests directly to 
an auditor most likely add to the negative  the auditor by fax or e­
form of confirmations of accounts  mail. 
d. Notifying the recipients an auditor recorded test counts for several 
that   second   requests items and noticed that certain test counts 
will   be   mailed   if   they were higher than the recorded quan­tities 
fail   to   respond   in   a in the client’s perpetual records. This 
timely man­ner.  situation could be the result of the client’s 
failure to record
a. Purchase discounts. 
b. Purchase returns. 
c. Sales. 
d. Sales returns. 
77. To gain assurance that all 
inventory items in a client’s 
inventory listing schedule are 
valid, an auditor most likely 
would trace 
a. Inventory tags noted 
during the auditor’s 
observa­tion to items 
listed in the inventory 
listing sched­ule. 
b. Inventory tags noted 
during the auditor’s 
observa­tion to items 
listed in receiving 
reports and ven­dors’ 
invoices. 
c. Items listed in the 
inventory listing 
schedule to in­ventory 
tags and the auditor’s 
recorded count sheets. 
d. Items   listed   in
receiving   reports   and
vendors’   in­voices   to
the   inventory   listing
schedule. 
78. To measure how effectively an 
entity employs its re­sources, an 
auditor calculates inventory 
turnover by dividing average 
inventory into 
a. Net sales. 
b. Cost of goods sold. 
c. Operating income. 
d. Gross sales. 
79. Which of the following 
auditing procedures most likely
would provide assurance about 
a manufacturing en­tity’s 
inventory valuation? 
a. Testing   the   entity’s
computation of standard
over­head rates. 
b. Obtaining   confirmation   of
inventories   pledged   un­der   loan
agreements. 
c. Reviewing   shipping
and   receiving   cutoff
proce­dures   for
inventories. 
d. Tracing test counts  to
the   entity’s   inventory
list­ing. 
80. A   client   maintains   perpetual
inventory   records   in   both c. Insist   that   the   client
quantities   and   dollars.   If   the perform   physical
assessed   level  of  control  risk   is counts   of   inventory
high, an auditor would probably  items   several   times
a. Increase the extent of during the year. 
tests of controls of the d. Apply gross profit
in­ventory cycle.  tests   to   ascertain
b. Request   the   client   to the   rea­
schedule   the   physical sonableness of the
inven­tory  count  at the physical counts. 
end of the year. 
120 MODULE 4   RESPONDING TO RISK ASSESSMENT
examines securities should insist
that   a   client   representative   be
81. An auditor concluded that no  present in order to 
excessive costs for idle plant  a. Detect fraudulent securities. 
were charged to inventory. This  b. Lend   authority   to   the   auditor’s
conclusion most likely related to  directives. 
the auditor’s objective to obtain  c. Acknowledge   the   receipt   of
evidence about the financial  securities returned.
statement assertions regarding  d. Coordinate   the   return
inventory, including presentation  of   securities   to   the
and disclosure and  proper locations. 
a. Valuation. 
b. Completeness.  87. In establishing the existence and 
c. Existence.  ownership of a long­term 
d. Rights.  investment in the form of 
publicly traded stock, an auditor 
82. An auditor selected items for test counts  should inspect the securities or 
while observ­ing a client’s physical  a. Correspond   with   the
inventory. The auditor then traced the test  investee   company   to
counts to the client’s inventory listing. This verify   the   number   of
procedure most likely obtained evidence  shares owned. 
concerning management’s assertion of  b. Inspect   the   audited
a. Rights.  financial   statements   of
b. Completeness. the in­vestee company. 
c. Existence.  c. Confirm   the   number   of   shares
d. Valuation.  owned   that   are   held   by   an
independent custodian. 
83. An   auditor   most   likely   would d. Determine   that   the
analyze   inventory   turn­over investment is carried
rates   to   obtain   evidence at   the   lower   of   cost
concerning   management’s   as­ or market. 
sertions about 
a. Existence.  88. When an auditor is unable to 
b. Rights.  inspect and count a cli­ent’s 
c. Presentation.  investment securities until after the 
d. Valuation. balance sheet date, 

84. An   auditor   usually   examines


receiving   reports   to   sup­port
entries in the 
a. Voucher   register   and   sales
returns journal. 
b. Sales   journal   and   sales   returns
journal. 
c. Voucher   register   and   sales
journal. 
d. Check register and sales journal. 
85. When   auditing   inventories,
an auditor would  least likely
verify that 
a. The   financial
statement  presentation
of   invento­ries   is
appropriate. 
b. Damaged goods and
obsolete   items   have
been   properly
accounted for. 
c. All   inventory   owned
by   the   client   is   on
hand   at   the   time   of
the count. 
d. The   client   has   used   proper
inventory pricing. 
86. An   auditor   who   physically
a. The   estimated
the bank where the securities are held in  remaining   useful   lives
a safe­deposit box should be asked to of   plant   as­sets   were
a. Verify any differences  revised upward. 
between the contents of  b. Plant   assets   were   retired   during
the box and the balances the year. 
in the client’s subsidiary c. The   prior   year’s
ledger.  depreciation   expense
b. Provide a list of  was   errone­ously
securities added and  understated. 
removed from the box  d. Overhead   allocations   were
between the balance  revised at year­end. 
sheet date and the 
security­count date.  92. In   testing   for   unrecorded
c. Confirm that there has retirements   of   equipment,   an
been  no  access to the auditor most likely would 
box   between   the a. Select items of equipment from 
balance sheet date and the accounting records and then 
the   security­count locate them during the plant tour.
date.  b. Compare depreciation 
journal entries with 
d. Count the securities in
similar prior year 
the   box so  the  auditor
will   have   an entries in search of fully
independent   direct depreciated equipment. 
verification.  c. Inspect   items   of
equipment   observed
89. In testing long­term investments, during   the   plant   tour
an  auditor   ordinarily   would  use and then trace them to
analytical procedures to ascertain the   equipment
the reasonable­ness of the  subsidiary ledger. 
a. Completeness   of   recorded d. Scan the general 
investment income.  journal for unusual 
b. Classification equipment additions 
between current and and excessive debits 
noncurrent to repairs and 
portfolios.  maintenance expense.
c. Valuation   of   marketable   equity
93. An   auditor   analyzes   repairs   and
securities. 
maintenance   accounts   primarily
d. Existence   of   unrealized
to obtain evidence in support of
gains   or   losses   in   the
the audit assertion that all 
port­folio. 
a. Noncapitalizable 
90. Analysis of which account is expenditures for repairs 
least  likely   to   reveal and maintenance have 
evidence relating to recorded been recorded in the 
retirement of equipment?  proper pe­riod. 
a. Accumulated depreciation.  b. Expenditures   for
b. Insurance expense.  property   and
c. Property, plant, and equipment.  equipment have been
recorded   in   the
d. Purchase returns and allowances.
proper period. 
91. Which of the following  c. Noncapitalizable 
explanations most likely would  expenditures for 
satisfy an auditor who questions  repairs and 
management about significant  maintenance have 
debits to the accumulated  been properly 
depreciation accounts?  charged to ex­pense. 
MODULE 4   RESPONDING TO RISK ASSESSMENT 121
possibility of a(n) 
a. Insurance   premium   due   but  not
d. Expenditures   for recorded. 
property   and b. Deficiency   in   the   coinsurance
equipment   have  not provision. 
been   charged   to c. Lien on the plant equipment. 
expense. d. Understatement   of   insurance
expense. 
94. The   auditor   is   most   likely   to   seek
information   from   the   plant   manager   with 99. Which   of   the   following
respect to the  procedures   would   an   auditor
a. Adequacy   of   the most likely perform in searching
provision   for for unrecorded liabilities? 
uncollectible   ac­ a. Trace a sample of 
counts.  accounts payable 
b. Appropriateness   of entries re­corded just 
physical   inventory before year­end to the 
observa­tion unmatched re­ceiving 
procedures.  report file. 
c. Existence of obsolete machinery. b. Compare a sample of 
d. Deferral   of purchase orders issued 
procurement of certain just after year­end with 
necessary   in­surance the year­end accounts 
coverage.  payable trial balance. 
c. Vouch a sample of cash 
95. Treetop Corporation acquired a  disbursements recorded just after
building and arranged mortgage  year­end to receiving reports and
financing during the year.  vendor invoices. 
Verification of the re­lated 
mortgage acquisition costs would
be least likely to include an 
examination of the related 
a. Deed. 
b. Canceled checks. 
c. Closing statement. 
d. Interest expense. 
96. In testing plant and equipment balances, an 
auditor may inspect new additions listed on 
the analysis of plant and equipment. This 
procedure is designed to obtain evidence 
concerning management’s assertions of 
Existence or Presentation and
occurrence disclosure
a. Yes Yes
b. Yes No
c. No Yes
d. No No
97. In auditing intangible assets, an 
auditor most likely would 
review or recompute 
amortization and determine 
whether the amortization period 
is reasonable in support of 
management’s financial 
statement assertion of 
a. Valuation or allocation. 
b. Existence or occurrence. 
c. Completeness. 
d. Rights and obligations. 
98. When auditing prepaid insurance,
an auditor discovers that the 
original insurance policy on plant 
equipment is not available for 
inspection. The policy’s absence 
most likely indicates the 
prenumbered purchase orders are
100. When   using   confirmations   to used and accounted for. 
provide   evidence   about   the b. Receiving the client’s mail, 
completeness   assertion   for unopened, for a rea­sonable 
accounts   payable,   the   appro­ period of time after the year­end 
priate   population   most   likely to search for unrecorded 
would be  vendors’ invoices. 
a. Vendors   with c. Vouching selected entries in the
whom the entity has accounts   payable   subsidiary
previously   done ledger   to   purchase   orders   and
business.  receiving reports. 
b. Amounts   recorded   in d. Confirming
the   accounts   payable accounts   payable
sub­sidiary ledger.  balances   with
c. Payees   of   checks known   suppliers
drawn in the month who   have   zero
after the year­end.  balances. 
d. Invoices   filed   in   the   entity’s
open invoice file.  103. In auditing accounts payable, an
auditor’s procedures most likely
101. Auditor   confirmation   of would   focus   primarily   on
accounts payable balances at the management’s asser­tion of 
balance   sheet   date   may   be a. Existence. 
unnecessary because  b. Presentation and disclosure. 
a. This   is   a   duplication   of   cutoff c. Completeness. 
tests.  d. Valuation. 
b. Accounts payable 
balances at the  104. When a CPA observes that the 
balance sheet date  recorded interest ex­pense seems 
may not be paid  to be excessive in relation to the 
before the audit is  balance in the bonds payable 
com­pleted.  account, the CPA might suspect 
c. Correspondence with  that 
the audit client’s  a. Discount   on   bonds   payable   is
attorney will reveal  understated. 
all legal action by  b. Bonds payable are understated. 
vendors for non­ c. Bonds payable are overstated. 
payment.  d. Premium   on   bonds   payable   is
d. There is  likely to  be overstated. 
other reliable external
evi­dence   to   support 105. An auditor most likely would 
the balances.  inspect loan agreements under 
which an entity’s inventories are 
102. Which   of   the   following   is   a pledged to support 
substantive test that an auditor management’s financial 
most likely would perform to statement assertion of 
verify   the   existence   and a. Presentation and disclosure. 
valuation of recorded accounts b. Valuation or allocation. 
payable?  c. Existence or occurrence. 
a. Investigating the open purchase  d. Completeness. 
order file to as­certain that 
d. Scan   the   cash   disbursements   entries   recorded   just  106.    In   auditing   long­term   bonds
payable, an auditor most
before year­end for indications of  likely would
unusual trans­actions. a. Perform analytical procedures on the
bond   pre­mium   and   discount
accounts. 
122 MODULE 4   RESPONDING TO RISK ASSESSMENT
are   approved   by
individual
b. Examine departmental
documentation of assets supervisors. 
purchased   with   bond d. A separate payroll bank
proceeds for liens.  account   is   maintained
c. Compare   interest on an imprest basis. 
expense with the bond
payable   amount   for 111. In   auditing   payroll,   an   auditor   most
reasonableness.  likely would 
d. Confirm   the   existence a. Verify   that   checks
of   individual   bond representing   unclaimed
holders at year­end.  wages are mailed. 
b. Trace   individual
107. The auditor can best verify a employee   deductions
client’s   bond   sinking   fund to   entity   journal
transactions   and   year­end entries. 
balance by  c. Observe   entity
a. Confirmation   with employees   during   a
individual   holders   of payroll dis­tribution. 
retired bonds.  d. Compare   payroll
b. Confirmation   with   the   bond costs   with   entity
trustee.  standards or budgets.
c. Recomputation of 
interest expense,  112. In performing tests concerning
interest pay­able, and  the   granting   of   stock   options,
amortization of bond  an auditor should 
discount or pre­mium.  a. Confirm   the
d. Examination   and   count   of   the transaction with the
bonds retired during the year.  Secretary   of   State
in   the   state   of
108. An   auditor   usually   obtains incorporation. 
evidence   of   stockholders’ b. Verify   the   existence
equity   transactions   by of   option   holders   in
reviewing the entity’s  the   en­tity’s   payroll
a. Minutes   of   board   of   directors records   or   stock
meetings.  ledgers. 
b. Transfer agent’s records.  c. Determine   that
c. Canceled stock certificates.  sufficient   treasury
d. Treasury stock certificate book.  stock   is   avail­able   to
cover   any   new   stock
109. When control risk is assessed as  issued. 
low for assertions related to  d. Trace   the
payroll, substantive tests of  authorization for the
payroll balances most likely  transaction to a vote
would be limited to applying  of   the   board   of
analytical procedures and  directors. 
a. Observing   the   distribution   of
paychecks. 
b. Footing   and   crossfooting   the
payroll register. 
c. Inspecting payroll tax returns. 
d. Recalculating payroll accruals. 
110. Which of the following 
circumstances most likely would 
cause an auditor to suspect an 
employee payroll fraud scheme? 
a. There   are   significant
unexplained variances
be­tween   standard
and actual labor cost. 
b. Payroll   checks   are
disbursed   by   the
same   em­ployee
each payday. 
c. Employee   time   cards
a. Cutoff tests indicate a
113. During an audit of an entity’s  substantial   amount   of
stockholders’ equity accounts,  ac­crued   payroll
the auditor determines whether  expense. 
there are restric­tions on retained  b. The   assessed   level   of
earnings resulting from loans,  control   risk   relative   to
agreements, or state law. This  pay­roll   transactions   is
audit procedure most likely is  low. 
intended to verify management’s  c. Analytical procedures
assertion of  indicate   unusual
a. Existence or occurrence.  fluctua­tions   in
b. Completeness.  recurring   payroll
c. Valuation or allocation.  entries. 
d. Presentation and disclosure.  d. Accrued payroll expense
consists primarily of un­
114. When a client company does not  paid commissions. 
maintain its own stock records, the 
auditor should obtain written  117. An auditor usually tests the reasonableness 
confirmation from the transfer agent of dividend income from investments in 
and registrar concerning  publicly held companies by computing the 
amounts that should have been received by 
a. Restrictions   on   the   payment   of
referring to 
dividends. 
a. Dividend   record
b. The number of shares issued and
books   produced   by
outstanding.  investment   advisory
c. Guarantees   of   preferred   stock services. 
liquidation value.  b. Stock   indentures
d. The   number   of   shares published by corporate
subject to agreements to transfer agents. 
re­purchase.  c. Stock   ledgers
115. An   audit   program   for   the maintained   by
examination   of   the   retained independent   regis­
earnings account should include trars. 
a step that requires verifi­cation d. Annual   audited
of the  financial   statements
issued   by   the   investee
a. Market   value   used   to
charge   retained companies. 
earnings to account for 118. The   most   likely   risk
a   two­for­one   stock involved with a bill and hold
split.  transaction   at   year­end   is
b. Approval   of   the a(n) 
adjustment   to   the a. Accrued liability may
beginning bal­ance as a be   overstated   as   of
result   of   a   write­down year­end. 
of   an   account   re­ b. Buyer   may   have   made
ceivable.  an   absolute   purchase
c. Authorization for both cash and com­mitment. 
stock dividends.  c. Sale   may
d. Gain   or   loss   resulting inappropriately   have
from   disposition   of been   recorded   as   of
treasury shares.  year­end. 
116. An   auditor   most   likely   would   perform d. Buyer   may   have
substantive   tests   of   details   on   payroll assumed   the   risk   and
transactions and balances when  reward   of   the
purchased product. 
MODULE 4   RESPONDING TO RISK ASSESSMENT 123
d. Reductions   of   obsolete
inventory   to   net
119. Which of the following  realizable value. 
accounts is the practice of 
“channel stuffing” for sales  123. The date of the management
most likely to most directly af­ representation   letter   should
fect, and thereby result in  coincide with the date of the
additional audit procedures?  a. Balance sheet. 
a. Accrued liabilities.  b. Latest   interim   financial
b. Allowance for sales returns.  information. 
c. Cash.  c. Auditor’s report. 
d. Marketable investments.  d. Latest related­party transaction. 

120. Recorded entries in which of  124. Which   of   the   following   matters


the following accounts are  would   an   auditor   most   likely
most likely to relate to the  include   in   a   management
property, plant, and equip­ment representation letter? 
completeness assertion?  a. Communications   with
a. Allowance for doubtful accounts. the   audit   committee
con­cerning
b. Marketable securities. 
weaknesses in internal
c. Property, plant, and equipment.  control. 
d. Repairs   and   maintenance b. The completeness and
expense.  availability of minutes
121. For   which   of   the   following of   stockholders’   and
matters should an auditor obtain directors’ meetings. 
written   management c. Plans   to   acquire   or   merge   with
representations?  other   entities   in   the   subsequent
a. Management’s   cost­ year. 
benefit   justifications d. Management’s
for  not  correcting acknowledgment   of
internal   control its respon­sibility for
weaknesses.  the   detection   of
employee fraud. 
b. Management’s
knowledge of future 125. The current chief executive and 
plans   that   may financial officers have only been 
affect   the   price   of employed by ABC Company for 
the entity’s stock.  the past five months of year 2. 
c. Management’s  ABC Company is presenting 
compliance with  comparative financial statements 
contractual  on Years 1 and 2, both of which 
agreements that may were audited by William Jones, 
affect the financial  CPA. For which year(s) should 
state­ments.  Jones obtain written 
d. Management’s representations from these two 
acknowledgment   of indi­viduals? 
its respon­sibility for
employees’
violations of laws. 
122. To which of the following 
matters would materiality limits 
not apply in obtaining written 
management represen­tations? 
a. The   availability   of
minutes   of
stockholders’   and
directors’ meetings. 
b. Losses from purchase
commitments   at
prices   in   excess   of
market value. 
c. The   disclosure   of
compensating   balance
arrange­ments
involving   related
parties. 
128. A written representation from a client’s 
Year 1 Year 2 management which, among other matters, 
a. No No acknowledges responsibility for the fair 
b. No Yes presentation of financial statements, should 
c. Yes No normally be signed by the 
d. Yes Yes a. Chief   executive   officer
126. Which of the following  and   the   chief   financial
statements ordinarily is in­cluded of­ficer. 
among the written client  b. Chief   financial
representations obtained by the  officer   and   the
auditor?  chairman   of   the
a. Compensating balances  board of directors. 
and other arrangements  c. Chairman   of  the   audit
involving restrictions on committee of the board
cash balances have been of directors. 
disclosed.  d. Chief   executive   officer,
b. Management   acknowledges the   chairman   of   the
responsibility for ille­gal actions board   of   directors,   and
committed by employees.  the client’s lawyer. 
c. Sufficient audit evidence 129. A limitation on the scope of the 
has been made available auditor’s examination sufficient 
to permit the issuance of to preclude an unqualified 
an unqualified opinion.  opinion will always result when 
d. Management management 
acknowledges   that a. Prevents   the   auditor
there are no material from   reviewing   the
weaknesses   in   the working   papers   of   the
internal control.  predecessor auditor. 
b. Engages   the   auditor   after   the
127. When considering the use of  year­end   physical   in­ventory
management’s written  count is completed. 
representations as audit evidence  c. Fails to correct a 
about the completeness assertion,  significant deficiency of
an auditor should understand that  internal control that had 
such representa­tions  been identified during 
a. Complement, but do not the prior year’s audit. 
replace, substantive tests d. Refuses   to   furnish   a
designed   to   support   the management
assertion.  representation letter to
b. Constitute sufficient  the auditor. 
evidence to support the 
as­sertion when  130. A   purpose   of   a   management
considered in  representation letter is to reduce 
combination with reli­ a. Audit   risk   to   an
ance on internal control. aggregate   level   of
c. Are  not  part   of   the misstatement   that
could   be   considered
audit   evidence
material. 
considered   to   support
b. An auditor’s 
the assertion. 
responsibility to detect 
d. Replace   reliance   on   internal material mis­statements 
control   as   evidence   to   support only to the extent that 
the assertion.  the letter is re­lied on. 
124 MODULE 4   RESPONDING TO RISK ASSESSMENT
the entity’s ability to 
continue as a going 
c. The possibility of a  concern. 
misunderstanding  b. Desires to disclose the specialist’s
concerning  findings, which imply that a more
management’s  thorough audit was performed. 
responsibility for the  c. Is   able   to   corroborate
financial statements.  another   specialist’s
d. The   scope   of   an earlier   findings   that
auditor’s   procedures were   consistent   with
concerning  related­party management’s
transactions   and representations. 
subsequent events.  d. Discovers   significant
deficiencies   in   the
131. “There   have   been   no design   of   the   entity’s
communications   from   regulatory internal   control   that
agencies   concerning management   does  not
noncompliance   with,   or
correct.
deficiencies in, financial reporting
practices   that   could   have   a 134. Which   of   the   following
material   effect   on   the   financial statements   is   correct   about   the
statements.”   The   foregoing auditor’s   use   of   the   work   of   a
passage is most likely from a  specialist? 
a. Report on internal control.  a. The   specialist   should
b. Special report.  not  have   an
c. Management   representation understanding   of   the
letter.  auditor’s corroborative
d. Letter for underwriters.  use of the special­ist’s
132. Which   of   the   following findings. 
statements is correct concern­ing b. The auditor is required 
an auditor’s use of the work of a to perform substantive 
specialist?  procedures to verify 
a. The work of a  the specialist’s 
specialist who is  assumptions and 
related to the cli­ent  findings. 
may be acceptable  c. The client should not 
under certain circum­ have an 
stances.  understanding of the 
b. If   an   auditor nature of the work to 
believes   that   the be performed by the 
determinations specialist. 
made   by   a d. The auditor should obtain
specialist   are an   understanding   of   the
unreasonable,   only methods and assumptions
a   qualified   opinion used by the specialist. 
may be issued. 
c. If there is a material  135. In using the work of a specialist, 
difference between a  an auditor referred to the 
special­ist’s findings  specialist’s findings in the 
and the assertions in the  auditor’s report. This would be an
financial statements,  appropriate reporting practice if 
only an adverse opinion  the 
may be is­sued.  a. Client is not familiar 
d. An auditor may not use  with the professional 
a specialist in the deter­ certifi­cation, personal 
mination of physical  reputation, or particular 
characteristics relating  compe­tence of the 
to in­ventories.  specialist. 

133. In using the work of a specialist,
an auditor may refer to the 
specialist in the auditor’s report 
if, as a result of the specialist’s 
findings, the auditor 
a. Becomes aware of 
conditions causing 
substantial doubt about 
collectively material to the 
b. Auditor, as a result of  client’s financial statements. 
the specialist’s  Which parties should reach an 
findings, adds an  understanding on the limits of 
explanatory paragraph materiality for this pur­pose? 
emphasizing a matter  a. The   auditor   and   the   client’s
regarding the  management. 
financial statements.  b. The client’s audit committee and
c. Client   understands   the the lawyer. 
auditor’s   corroborative c. The client’s management and the
use   of   the   specialist’s lawyer. 
findings   in   relation   to d. The lawyer and the auditor. 
the   repre­sentations   in
139. The refusal of a client’s attorney
the financial statements.
to provide informa­tion requested
d. Auditor, as a result of 
in   an   inquiry   letter   generally   is
the specialist’s findings, 
de­cides to indicate a  considered 
division of  a. Grounds for an adverse opinion. 
responsibility with the  b. A limitation on the scope of the
specialist.  audit. 
c. Reason   to   withdraw   from   the
136. In using the work of a specialist,  engagement. 
an understanding should exist  d. Equivalent   to   a   significant
among the auditor, the client, and deficiency. 
the specialist as to the nature of 
the specialist’s work. The  140. Which of the following is an 
documentation of this  audit procedure that an auditor 
understanding should cover  most likely would perform 
a. A statement that the  concerning litigation, claims, 
specialist assumes no  and assessments? 
respon­sibility to update  a. Request the client’s 
the specialist’s report for lawyer to evaluate 
future events or  whether the client’s 
circumstances.  pending litigation, 
b. The   conditions   under claims, and as­
which   a   division   of sessments indicate a 
respon­sibility   may   be going concern 
necessary.  problem. 
c. The   specialist’s b. Examine the legal 
understanding of the documents in the client’s
auditor’s law­yer’s possession 
corroborative use of concerning litigation, 
the   specialist’s claims, and assessments 
findings.  to which the lawyer has 
d. The auditor’s disclaimer as to  devoted sub­stantive 
whether the special­ist’s findings attention. 
corroborate the representations in c. Discuss   with
the financial statements.  management   its
policies   and   proce­
137. Which of the following is  not  a dures   adopted   for
specialist   upon   whose   work   an evaluating   and
auditor may rely?  accounting   for
a. Actuary.  litigation,   claims,   and
b. Appraiser.  assessments. 
c. Internal auditor.  d. Confirm   directly   with
d. Engineer.  the   client’s   lawyer   that
all   litigation,   claims,
138. A lawyer’s response to an  and   assessments   have
auditor’s inquiry concerning  been   re­corded   or
litigation, claims, and assessments disclosed   in   the
may be limited to matters that are  financial statements. 
considered individually or 
MODULE 4   RESPONDING TO RISK ASSESSMENT 125
representation. 
b. An   evaluation   of   the
141. The primary reason an auditor  likelihood   of   an
requests letters of in­quiry be  unfavorable   outcome   of
sent to a client’s attorneys is to  the   matters   disclosed   by
provide the auditor with  the entity. 
a. The   probable c. The   attorney’s
outcome of asserted opinion of the entity’s
claims  and pending historical   experience
or   threatened in   recent   similar
litigation.  litigation. 
b. Corroboration of the  d. The   probable
information furnished  outcome of asserted
by management about  claims  and pending
litigation, claims, and  or   threatened
assess­ments.  litigation. 
c. The attorneys’ opinions
of the client’s historical 144. A CPA has received an 
experiences   in   recent attorney’s letter in which no 
similar litigation.  significant disagreements with 
the client’s assessments of 
d. A description and 
contingent liabilities were noted. 
evaluation of litigation, 
The resignation of the client’s 
claims, and assessments
lawyer shortly after receipt of the
that existed at the 
letter should alert the auditor that
balance sheet date. 
a. Undisclosed   unasserted   claims
142. Which of the following is not an may have arisen. 
audit procedure that the  b. The attorney was 
independent auditor would  unable to form a 
perform concerning litiga­tion,  conclusion with 
claims, and assessments?  respect to the 
a. Obtain assurance from  significance of 
management that it has disclosed litigation, claims, 
all unasserted claims that the  and assessments. 
lawyer has advised are probable  c. The   auditor   must
of assertion and must be dis­ begin   a   completely
closed.  new   ex­amination
b. Confirm   directly   with of   contingent
the client’s lawyer that liabilities. 
all   claims   have   been d. An   adverse   opinion   will   be
recorded   in   the necessary. 
financial state­ments. 
145. Which of the following 
c. Inquire of and discuss 
statements extracted from a 
with management the 
client’s lawyer’s letter 
poli­cies and procedures
concerning litigation, claims, 
adopted for identifying, 
and assessments most likely 
evaluating, and 
would cause the auditor to 
accounting for 
request clarification? 
litigation, claims, and 
assessments.  a. “I   believe   that   the
possible   liability   to
d. Obtain from 
the   com­pany   is
management a 
nominal in amount.” 
description and 
evaluation of  b. “I   believe   that   the
litigation, claims, and  action   can   be   settled
for   less   than   the
assessments existing 
damages claimed.” 
at the balance sheet 
date. 
143. The   scope   of   an   audit   is  not
restricted   when   an   attor­ney’s
response to an auditor as a result
of   a   client’s   letter   of   audit
inquiry limits the response to 
a. Matters   to   which   the
attorney   has   given
substan­tive   attention
in   the   form   of   legal
indications  of  possible
6. “I believe that the plaintiff’s case financial difficulties. 
against the com­pany is without
merit.”  149. After determining that a related­
7. “I believe that the company will party   transaction   has,   in   fact,
be   able   to   defend   this   action occurred, an auditor should 
successfully.”  a. Add a separate paragraph to the
auditor’s   standard   report   to
146. When auditing the fair value of an  explain the transaction. 
asset or liability,valuation issues  b. Perform analytical 
ordinarily arise at the point of procedures to verify 
Initial Subsequent to whether similar 
recording initial recording transactions occurred, 
a. Yes Yes but were not re­
b. Yes No corded. 
c. No Yes c. Obtain   an
d. No No understanding   of   the
business purpose of the
147. Which of the following is least  transaction. 
likely to be an ap­proach  d. Substantiate that the
followed when auditing the fair  transaction   was
values of assets and liabilities?  consum­mated   on
a. Review   and   test terms   equivalent   to
management’s   process an   arm’s­length
of valua­tion.  transaction. 
b. Confirm   valuations
with   audit   committee 150. When   auditing   related­party
mem­bers.  transactions,   an   auditor   places
c. Independently   develop primary emphasis on 
an estimate of the value a. Ascertaining the rights
of the account.  and   obligations   of   the
d. Review   subsequent   events re­lated parties. 
relating to the account.  b. Confirming the existence of the
related parties. 
148. Which of the following  c. Verifying  the  valuation
auditing procedures most  of   the   related­party
likely would assist an auditor  trans­actions. 
in identifying related­party  d. Evaluating   the
transactions?  disclosure   of   the
a. Inspecting related­party
correspondence   with transactions. 
lawyers   for   evi­dence
of   unreported 151. Which   of   the   following
contingent liabilities.  statements is correct concern­ing
b. Vouching   accounting related­party transactions? 
records   for   recurring a. In the absence of evidence to the 
trans­actions   recorded contrary, related­party 
just   after   the   balance transactions should be assumed 
sheet date.  to be outside the ordinary course 
c. Reviewing of business. 
confirmations   of   loans b. An   auditor   should
receivable and payable determine   whether   a
for   indications   of particular   transaction
guarantees.  would   have   occurred   if
d. Performing   analytical the parties had not been
procedures   for related.
126 MODULE 4   RESPONDING TO RISK ASSESSMENT
d. Investigate changes in
long­term   debt
a. An auditor should  occurring   after   year­
substantiate that  end. 
related­party 
transactions were  154. Which of the following events occurring 
consummated on terms  after the issu­ance of an auditor’s report 
equiva­lent to those that most likely would cause the auditor to make
prevail in arm’s­length  further inquiries about the previously issued
transac­tions.  financial statements? 
b. The   audit   procedures a. An uninsured natural disaster 
directed   toward occurs that may af­fect the 
identifying   related­party entity’s ability to continue as a 
transactions   should going con­cern. 
include   consid­ering b. A   contingency   is
resolved   that   had   been
whether transactions are
disclosed in the  audited
occurring,   but   are  not financial statements. 
being   given   proper c. New   information   is
accounting recognition. discovered   concerning
152. An auditor most likely would modify an  undis­closed   lease
unqualified opinion if the entity’s financial  transactions   of   the
statements include a footnote on related­ audited period. 
party transactions  d. A   subsidiary   is   sold
a. Disclosing   loans   to that   accounts   for   25%
related parties at interest of   the   entity’s
rates significantly below consolidated   net
prevailing market rates.  income. 
b. Describing   an 155. Zero Corp. suffered a loss that would have 
exchange of real estate a material effect on its financial statements 
for similar property in on an uncollectible trade account receivable
a nonmonetary related­ due to a customer’s bankruptcy. This 
party transac­tion.  occurred suddenly due to a natural disaster 
c. Stating   that   a   particular ten days after Zero’s balance sheet date, but
related­party   transaction one month before the issuance of the 
occurred   on   terms financial statements and the auditor’s 
equivalent   to   those   that report. Under these circumstances, 
would have prevailed in
The The event The auditor’s
an   arm’s­length
financial requires financial report should
transaction.  statements statement be modified
d. Presenting the dollar  should be disclosure, but for a lack of
volume of related­party  adjusted no adjustment consistency
transactions and the  a. Yes No No
effects of any change in  b. Yes No Yes
the method of  c. No Yes Yes
establishing terms from  d. No Yes No
prior periods. 

153. Which of the following 
procedures would an auditor 
most likely perform in obtaining 
evidence about subsequent 
events? 
a. Determine  that changes
in   employee   pay   rates
af­ter   year­end   were
properly authorized. 
b. Recompute
depreciation   charges
for   plant   assets   sold
after year­end. 
c. Inquire   about   payroll
checks   that   were
recorded   before   year­
end   but   cashed   after
year­end. 
after year­end. 
156. After an audit report containing an  b. Comparing   the
unqualified opinion on a nonissuer  financial statements
(nonpublic) client’s financial statements  being   re­ported   on
was issued, the client decided to sell the  with   those   of   the
shares of a subsidiary that accounts for  prior period. 
30% of its revenue and 25% of its net  c. Investigating   personnel
income. The auditor should  changes   in   the
a. Determine whether the accounting   department
information is reliable  occurring after year­end.
and, if determined to  d. Inquiring as to whether
be reliable, request  any   unusual
that re­vised financial  adjustments   were
statements be issued.  made after year­end. 
b. Notify the entity that the  159. Which   of   the   following
auditor’s report may no longer  procedures   should   an   auditor
be associated with the financial  generally   perform   regarding
statements.  subsequent events? 
c. Describe the effects of  a. Compare the latest 
this subsequently  available interim 
discov­ered  financial statements 
information in a  with the financial 
communication with  statements being au­
per­sons known to be  dited. 
relying on the financial b. Send second requests 
state­ments.  to the client’s 
d. Take  no  action because customers who failed 
the   auditor   has  no to respond to initial 
obliga­tion to make any accounts receiv­able 
further inquiries.  confirmation requests.
c. Communicate
157. A   client   acquired   25%   of   its material   weaknesses
outstanding   capital   stock   after in internal control to
year­end and prior to completion the   client’s   audit
of  the auditor’s field­work. The committee. 
auditor should 
d. Review   the   cutoff
a. Advise   management   to bank   statements   for
adjust the balance sheet several months after
to   reflect   the the year­end. 
acquisition. 
b. Issue pro forma financial 160. On February 25, a CPA issued an 
statements   giving   effect auditor’s report ex­pressing an 
to the acquisition as if it unqualified opinion on financial 
had occurred at year­end.  statements for the year ended 
c. Advise   management   to January 31. On March 2, the CPA
disclose   the   acquisition learned that on February 11, the 
in   the   notes   to   the entity incurred a material loss on 
financial statements.  an uncollectible trade receivable 
d. Disclose   the as a result of the deteriorating 
acquisition   in   the financial condition of the entity’s 
opinion   paragraph   of principal customer that led to the 
the auditor’s report.  customer's bankruptcy. 
Management then refused to 
158. Which of the following  adjust the financial statements for 
procedures would an auditor  this subsequent event. The CPA 
most likely perform to obtain  determined that the information is
evidence about the occurrence of  reliable and that there are 
subsequent events?  creditors currently relying on the 
a. Confirming a sample of financial state­ments. The CPA’s 
material   accounts next course of action most likely 
receiv­able   established would be to 
MODULE 4   RESPONDING TO RISK ASSESSMENT 127
disclose   to   the
stockhold­ers   that
a. Notify the entity’s  Kent’s   unqualified
creditors that the  opinion   should  not  be
financial statements and relied on. 
the related auditor’s  b. Undertake to apply 
report should no longer  alternative procedures 
be relied on. that would provide a 
b. Notify each member of the  satisfactory basis for the 
entity’s board of directors about  unquali­fied opinion. 
management’s refusal to adjust  c. Reissue the auditor’s 
the financial statements.  report and add an 
c. Issue   revised  financial explana­tory paragraph 
statements   and describing the departure 
distribute them to each from gen­erally 
creditor   known   to   be accepted auditing 
relying on the financial standards. 
statements.  d. Compensate for the 
d. Issue a revised auditor’s omitted procedure by 
report   and   distribute   it per­forming tests of 
to   each   creditor   known controls to reduce audit
to   be   relying   on   the risk to a sufficiently 
financial statements.  low level. 
161. An auditor is considering whether 163. Six months after issuing an 
the omission of a substantive  unqualified opinion on audited 
procedure considered necessary  financial statements, an auditor 
at the time of an audit may impair discovered that the engagement 
the auditor’s present ability to  personnel failed to confirm 
support the previously expressed  several of the cli­ent’s material 
opinion. The auditor need not  accounts receivable balances. 
apply the omitted procedure if the The auditor should first 
a. Request   the   permission
a. Financial statements and
auditor’s report were not of the client to undertake
distributed beyond  the   confirmation   of
management and the  accounts receivable. 
board of directors.  b. Perform   alternative
b. Auditor’s   previously   expressed procedures to provide a
opinion   was   quali­fied   because satis­factory   basis   for
of a departure from GAAP.  the unqualified opinion.
c. Results   of   other c. Assess   the   importance
procedures   that   were of   the   omitted
applied   tend   to procedures   to   the
compensate   for   the auditor’s   ability   to
procedure omitted.  support   the   previously
d. Omission   is   due   to expressed opinion. 
unreasonable   delays   by d. Inquire   whether   there
client   personnel   in are   persons   currently
providing   data   on   a rely­ing,   or   likely   to
timely basis.  rely, on the unqualified
opinion. 
162. On March 15, 2002, Kent, CPA, 
164. Which   of   the   following
issued an unqualified opinion on 
a client’s audited financial  procedures   is  least  likely   to   be
statements for the year ended  performed   before   the   balance
December 31, 2001. On May 4,  sheet date? 
2002, Kent’s internal inspection 
program disclosed that 
engagement per­sonnel failed to 
observe the client’s physical 
inventory. Omission of this 
procedure impairs Kent’s present 
ability to support the unqualified 
opinion. If the stockholders are 
cur­rently relying on the opinion, 
Kent should first 
a. Advise   management   to
c. Determine   whether
a. Testing   of   internal   control   over inadequate   provisions
cash.  for the safeguarding of
b. Confirmation of receivables.  assets   have   been
c. Search for unrecorded liabilities.  corrected. 
d. Observation of inventory.  d. Consider   whether   the
results   of   audit
165. Which   of   the   following   most procedures   affect   the
likely   would   be   detected   by   an assessment of the  risk
auditor’s   review   of   a   client’s of   material   mis­
sales cutoff?  statement due to fraud.
a. Shipments   lacking
sales   invoices   and 168. Operational   auditing   is   primarily
shipping documents. oriented toward 
b. Excessive write­offs of accounts a. Future   improvements
receivable.  to   accomplish   the
c. Unrecorded sales at year­end.  goals of management. 
d. Lapping   of   year­end   accounts b. The   accuracy   of   data
receivable.  reflected   in
management’s
166. Cutoff tests designed to detect credit sales financial records. 
made before the end of the year that have c. The   verification   that   a
been   recorded   in   the   subsequent   year company’s   financial
provide   assurance   about   management’s state­ments   are   fairly
assertion of  presented. 
a. Presentation.  d. Past   protection
b. Completeness.  provided   by   existing
c. Rights.  internal con­trol. 
d. Existence. 
169. A   typical   objective   of   an
167. Which of the following  operational   audit   is   to   deter­
procedures would an auditor  mine whether an entity’s 
most likely perform during an  a. Internal   control   is
audit engagement’s overall  adequately
review stage in formulating an  operating   as   de­
opinion on an entity’s finan­cial  signed. 
statements?  b. Operational 
a. Obtain assurance  information is in 
from the entity’s  accordance with 
attorney that all  generally accepted 
material litigation has governmental 
been disclosed in the  auditing stan­dards. 
financial statements.  c. Financial   statements
b. Verify   the   clerical present   fairly   the
accuracy   of   the results of operations. 
entity’s   proof   of   cash d. Specific   operating   units   are
and   its   bank   cutoff functioning   efficiently   and
statement.  effectively. 
128 MODULE 4   RESPONDING TO RISK ASSESSMENT

SIMULATIONS

Task­Based Simulation 1
Audit Investments and
Accounts Receivable Authoritative
Literature Help

Items 1 through 7 represent audit objectives for the investments and accounts receivable. To the right of each set of 
auditobjectives is a listing of possible audit procedures for that account. For each audit objective, select the audit procedure 
that would primarily respond to the objective. Select only one procedure for each audit objective. A procedure may be selected
only once, or not at all.
Audit procedures for investments
A. Trace opening balances in the subsidiary ledger to prior year’s audit working papers. 
B. Determine that employees who are authorized to sell investments do not have access to cash. 
C. Examine supporting documents for a sample of investment transactions to verify that prenumbered documents
are used. 
D. Determine that any impairments in the price of investments have been properly recorded. 
E. Verify that transfers from the current to the noncurrent investment portfolio have been properly recorded. 
F. Obtain positive confirmations as of the balance sheet date of investments held by independent custodians. 
G. Trace investment transactions to minutes of the Board of Directors meetings to determine that transactions
were properly authorized. 

A
u
1. I

2. R

3. T

A
u
A. Analyze the relationship of accounts receivable and sales and compare it with relationships for preceding periods. 
B. Perform sales cutoff tests to obtain assurance that sales transactions and corresponding entries for inventories and
cost of goods sold are recorded in the same and proper period. 
C. Review the aged trial balance for significant past due accounts. 
D. Obtain an understanding of the business purpose of transactions that resulted in accounts receivable balances. 
E. Review loan agreements for indications of whether accounts receivable have been factored or pledged. 
F. Review the accounts receivable trial balance for amounts due from officers and employees. 
G. Analyze unusual relationships between monthly accounts receivable balances and monthly accounts payable
balances. 

A
u
4. A

5. T

6. A

7. A

Task­Based Simulation 2
Research
Authoritative
Literature Help

Confirmation of Accounts Receivable
Bill Smith, the president of Alex Inc., a nonpublic audit client, has suggested to you that his previous auditor did not 
confirm accounts receivable and he sees no reason why you should do so.
MODULE 4   RESPONDING TO RISK ASSESSMENT 129

Selections
A. AU
B. PCAOB
C. AT
D. AR
E. ET
F. BL
G. CS
H. QC

(A) (B) (C) (D) (E)  (F) (G) (H)
1. Which title of the Professional Standards addresses this issue and will be
helpful in responding to him?
2. Enter the exact section and paragraph with helpful information.

Task­Based Simulation 3
Illegal Acts and Related­
Party Transactions Authoritative
Literature Help

In   applying   audit   procedures   and   evaluating   the   results   of   those   procedures,   auditors   may   encounter   specific
information that may raise  a  question concerning the existence of illegal acts  and related­party transactions. Indicate
whether each of the following is more likely related to an illegal act (IA) or a related­party transaction (RP).

S
t
1. A
 
2. T
h
3. U

4. T

5. L
a
Task­Based Simulation 4
Accounts Receivable
Confirmations
Authoritative
Literature Help

An auditor may use confirmations of accounts receivable. Reply as to whether the following statements are correct or in­
correct with respect to the confirmation process when applied to accounts receivable.

S
1. t
T
h
2. A
 
3. S
e
4. C

5. C

6. A
b
7. A
u
8. A
u
130 MODULE 4   RESPONDING TO RISK ASSESSMENT

S
t
9. T
h
10. A
c
Task­Based Simulation 5
Auditing
Inventory
Authoritative
Literature Help
Auditors often observe the counting of their clients’ inventories.  Reply as to whether the 
following statements are correct
or incorrect with respect to the inventory observation.
Statement Correct Incorrect
1. With strong internal control, the inventory count may be at the end of the year or at other
times.
2. When a client has many inventory locations, auditors ordinarily need not be present at
each location.
3. All auditor test counts must be documented in the working papers.
4. Auditors’ observation of the counting of their clients’ inventories addresses the existence
of inventory, and not the completeness of the count.
5. When the client manufactures a product, direct labor and overhead ordinarily become a
part of inventory item costs.
6. Inventory is ordinarily valued at the lower of standard cost or market.
7. Inventory items present as “consigned in” should not be included in the clients’ inventory
value.
8. Auditor recording of test counts ordinarily replaces the need for client “tagging” of in­
ventory.
9. Ordinarily, an auditor need not count all items in the inventory.
10. At the completion of the count, an auditor will ordinarily provide the client with copies of
his or her inventory test counts to help assure inventory accuracy.

Task­Based Simulation 6
Research
Authoritative
Literature Help

Auditing Derivatives
The partner in charge of the audit you are currently working on is concerned about
overall risk involved with certain financial derivative transactions the client is involved
with. More specifically, she has asked you to find guidance in the Professional Standards
on determining that all of the client’s derivatives are properly reported.
Selections
A. AU 
B. PCAOB 
C. AT 
D. AR 
E. ET 
F. BL 
G. CS 
H. QC 

(A) 
(B) 
(C) 
(D) 
(E)  
(F) 
(G) 
(H)
1. Which title of the Professional Standards addresses this issue and will be 
helpful in responding to her?

2. Enter the exact section and paragraph with helpful information.
MODULE 4   RESPONDING TO RISK ASSESSMENT 131

Task­Based Simulation 7
Bank Authoritative
Reconciliation Literature Help

Items 1 through 6 represent the items that an auditor ordinarily would find on a client­
prepared bank reconciliation. Theaccompanying List of Auditing Procedures represents 
substantive auditing procedures. For each item, select one or more pro­cedures, as indicated, 
that the auditor most likely would perform to gather evidence in support of that item. The 
procedures on the List may be selected once, more than once, or not at all.
Assume
• The client prepared the bank reconciliation on 10/2/05. 
• The bank reconciliation is mathematically accurate. 
• The auditor received a cutoff bank statement dated 10/7/05 directly from the bank on
10/11/05. 
• The 9/30/05 deposit in transit, outstanding checks #1281, #1285, #1289, and #1292,
and the correction of the error re­garding check #1282 appeared on the cutoff bank
statement. 
• The auditor assessed control risk concerning the financial statement assertions related
to cash at the maximum. 
List of Auditing Procedures
A. Trace   to   cash   receipts F. Inspect bank debit memo. 
journal.  G. Ascertain reason for unusual delay. 
B. Trace to cash disbursements H. Inspect   supporting   documents   for   reconciling
journal.  item not appear­ing on cutoff statement. 
C. Compare to 9/30/01 general I. Trace items on the bank reconciliation to cutoff
ledger.  statement. 
D. Confirm directly with bank. J. Trace   items   on   the   cutoff   statement   to   bank
E. Inspect bank credit memo.  reconciliation. 

General Company
BANK RECONCILIATION
1ST NATIONAL BANK OF US BANK ACCOUNT
September 30, 2005
1. Select 2 Procedures — Balance per bank $ 
28,375
2. Select 5 Procedures — Deposits in transit 
9/29/05 $4,500
9/30/05 1,525 6,025
34,400
3. Select 5 Procedures — Outstanding checks
#  988 8/31/05 2,200
#1281 9/26/05 675
#1285 9/27/05 850
#1289 9/29/05 2,500
#1292 9/30/05 7,225 (13,450)
20,950
4. Select 1 Procedure — Customer note collected by bank (3,000)
5. Select 2 Procedures — Error:  Check #1282, written on 9/26/05
for $270 was erroneously charged by bank
as $720; bank was notified on 10/2/05 450
6. Select 1 Procedure — Balance per books $ 18,400

Task­Based Simulation 8
Audit
Procedures
Authoritative
Literature Help

Items 1 through 12 represent possible errors and fraud that you suspect may be present at
General Company. The accom­panying List of Auditing Procedures represents procedures that 
the auditor would consider performing to gather evidence con­cerning possible errors and 
fraud. For each item, select one or two procedures, as indicated, that the auditor most likely 
would perform to gather evidence in support of that item. The procedures on the list may be 
selected once, more than once, or not at all.
132 MODULE 4   RESPONDING TO RISK ASSESSMENT

List of Auditing Procedures
A. Compare the details of the cash  L. Examine the entity’s shipping 
receipts journal entries with the 
documents to verify that the 
merchandise that produced the 
details of the corresponding daily 
receivable was actually sent to the 
deposit slips. 
customer. 
B. Scan the debits to the fixed asset  M. Inspect the entity’s correspondence 
accounts and vouch selected amounts files for indications of customer 
to vendors’ invoices and  disputes for evidence that certain 
management’s authorization.  shipments were on consignment. 
C. Perform   analytical   procedures   that
N. Perform edit checks of data on the payroll
compare   documented   authorized   pay transaction tapes. 
rates   to   the   entity’s   budget   and
O. Inspect   payroll   check
forecast.  endorsements   for   similar
D. Obtain   the   cutoff   bank   statement   and handwriting. 
compare the cleared checks to the year­end P. Observe   payroll   check   distribution   on   a
bank reconciliation.  surprise basis. 
E. Prepare a bank transfer schedule.  Q. Vouch data in the payroll register to 
F. Inspect the entity’s deeds to its real estate.  documented authorized pay rates in 
G. Make   inquiries   of   the   entity’s the human resources department’s 
attorney concerning the details of files. 
real estate transactions.  R. Reconcile the payroll checking 
H. Confirm   the   terms   of   borrowing account and determine if there were
arrangements with the lender.  unusual time lags between the 
I. Examine   selected   equipment   repair issuance and payment of payroll 
orders and supporting documentation to checks. 
determine the propriety of the charges.  S. Inspect the file of prenumbered 
J. Send requests to confirm the entity’s vouchers for consecutive numbering
accounts   receivable   on   a   surprise and proper approval by an 
basis at an interim date.  appropriate employee. 
K. Send   a   second   request   for T. Determine  that  the  details  of
confirmation of the receivable to the selected   prenumbered
customer   and   make   inquiries   of   a vouchers   match   the   related
reputable   credit   agency   concerning vendors’ invoices. 
the customer’s creditworthiness.  U. Examine the supporting purchase
orders   and   receiving   reports   for
selected paid vouchers. 
Possible misstatements due to errors and fraud
1. The   auditor   suspects   that   a   kiting   scheme   exists   because   an   accounting
department employee who can issue and record checks seems to be leading an
unusually luxurious lifestyle. (Select only 1 procedure) 
2. An auditor suspects that the controller wrote several checks and recorded the cash
disbursements just before year­end but did not mail the checks until after the first
week of the subsequent year. (Select only 1 procedure) 
3. The entity borrowed funds from a financial institution. Although the transaction
was properly recorded, the auditor sus­pects that the loan created a lien on the
entity’s real estate that is not disclosed in its financial statements. (Select only
1procedure)
4. The auditor discovered an unusually large receivable from one of the entity’s new
customers. The auditor suspects that the receivable may be fictitious because the
auditor has never heard of the customer and because the auditor’s initial attempt to
confirm   the   receivable   has   been   ignored   by   the   customer.   (Select   only   2
procedures) 
5. The auditor suspects that fictitious employees have been placed on the payroll by
the   entity’s   payroll   supervisor,   who   has   access   to   payroll   records   and   to   the
paychecks. (Select only 1 procedure) 
6. The auditor  suspects that selected employees  of the  entity received unauthorized
raises from the entity’s payroll supervisor, who has access to payroll records. (Select
only 1 procedure) 
7. The entity’s cash receipts of the first few days of the subsequent year were properly 
deposited in its general operating ac­count after the year­end. However, the auditor 
suspects that the entity recorded the cash receipts in its books during the last week of the 
year under audit. (Select only 1 procedure) 
8. The auditor suspects that vouchers were prepared and processed by an accounting
department employee for merchandise that was neither ordered nor received by
the entity. (Select only 1 procedure) 
9. The details of invoices for equipment repairs were not clearly identified or explained to
the   accounting   department   employ­ees.   The   auditor   suspects   that   the   bookkeeper
incorrectly recorded the repairs as fixed assets. (Select only 1 procedure) 
10. The auditor suspects that a lapping scheme exists because an accounting 
department employee who has access to cash re­ceipts also maintains the accounts 
receivable ledger and refuses to take any vacation or sick days. (Select only 2 
proce­dures)
11. The auditor suspects that the entity is inappropriately increasing the cash reported
on its balance sheet by drawing a check on one account and not recording it as an
outstanding check on that account and simultaneously recording it as a deposit in a
second account. (Select only 1 procedure) 
12. The auditor suspects that the entity’s controller has overstated sales and accounts
receivable by recording fictitious sales to regular customers in the entity’s books.
(Select only 2 procedures) 
MODULE 4   RESPONDING TO RISK ASSESSMENT 133

Task­Based Simulation 9
Substantive Procedures for Property, Plant and  Authoritative
Equipment Literature Help

DietWeb Inc. (hereafter DietWeb) was incorporated and began business in March of 
20X1, seven years ago. You are working on the 20X8 audit—your CPA firm’s fifth audit of 
DietWeb. For each audit objective, select a substantive procedure that would help to achieve 
that objective. Each of the procedures may be used once, more than once, or not at all.
Substantive procedure
A. Trace opening balances in the summary schedules to the prior year’s audit working
papers. 
B. Review the provision for deprecation expense and determine that depreciable lives
and methods used in the current year are consistent with those used in the prior
year. 
C. Determine that responsibility for maintaining the property and equipment records is
segregated from the responsibil­ity for custody of property and equipment. 
D. Examine deeds and title insurance certificates. 
E. Perform cutoff test to verify that property and equipment additions are recorded in
the proper period. 
F. Determine that property and equipment is adequately insured. 
G. Physically examine all recorded major property and equipment additions. 
H. Analyze repairs and maintenance expense. 
Audit Objective
(A) (B) (C) 
(D) (E)  (F) 
(G) (H)
1. DietWeb has legal rights to property and equipment acquired during the year. 
2. DietWeb recorded property and equipment acquired during the year that did not
actually exist at the balance sheet date. 
3. DietWeb’s property and equipment was properly valued at the balance sheet date. 
4. DietWeb  recorded  all property and equipment assets  that  were  purchased
near year­end. 
5. DietWeb recorded all property retirements that occurred during the year. 
6. DietWeb capitalized all acquisitions that occurred during the period. 

Task­Based Simulation 10
Risk Analysis
Authoritative
Literature Help
You are working with William Bond, CPA, and you are considering the risk of material 
misstatement in planning the audit of Toxic Waste Disposal (TWD) Company’s financial 
statements for the year ended December 31, 20X0.
Assume that you have identified the following risks at the account level relating to 
TWD’s property and equipment. Iden­tify the most closely related financial statement 
assertion and the audit procedure that might be planned to most likely address the risk. 
Financial statement assertions and audit procedures may be used once, more than once, or not 
used at all.
Related financial statement assertion Audit procedures
A.  Existence or occurrence F. Trace opening balances in the summary schedules to the prior year’s
B.  Completeness audit working papers.
C.  Rights and obligations G. Review the provision for depreciation expense and determine that de­
D.  Valuation or allocation preciable lives and methods used in the current year are consistent with
E.  Presentation and disclosure those used in the prior year.
H.  Determine that the responsibility for maintaining the property and
equipment records is segregated from the responsibility for custody of
property and equipment.
I. Examine deeds and title insurance certificates.
J. Perform cutoffs tests to verify that property and equipment additions
are recorded in the proper period.
K. Determine that property and equipment are adequately insured.
L. Physically examine all major property and equipment additions.
134 MODULE 4   RESPONDING TO RISK ASSESSMENT

Risk identified

1. TWD   may   not   have   legal


title to certain prop­erty and
equipment   recorded   as
acquired dur­ing the year. 
2. Recorded   property   and
equipment   acquisitions   may
include nonexistent assets. 
3. Recorded net property and
equipment   are   for   proper
amounts. 
Related financial statement assertion Audit procedures
(A) (B) (C) (D) (E) (F) (G) (H) (I)  (J) (K) (L)

Task­Based Simulation 11

Audit Objectives Authoritative
and Procedures Literature Help

For each audit objective listed below select the most appropriate audit procedure for 
raw materials inventory (Items 1­3) and for Accounts Receivable (Items 4­6). Audit 
procedures may be used once, more than once, or not at all.
List of audit procedures for raw materials inventory
A. Compare standard costs of inventories with standardized market values. 
B. Determine that all direct labor and overhead has been expensed and not included in
inventory valuation. 
C. Examine vendors’ invoices. 
D. Perform analytical procedures comparing inventory to various industry averages. 
E. Review drafts of financial statement note disclosures. 
F. Select a sample of items during the physical count and determine that the client has
included items on inventory count sheets. 
G. Select a sample of recorded items on count sheets and determine that the items are on
hand. 

1. D

2. E

3. D

L
i
A. Analyze relationships between accounts receivable balances and changes in the current
portion of long­term debt. 
B. Compare  accounts  receivable  on  the   accounts   receivable   lead   schedule  with   those   on
supporting audit schedules. 
C. Compare total 20X8 annual sales with those of 20X7. 
D. Examine December 20X8 sales journal and determine that sales are properly recorded in
December. 
E. Examine January 20X9 sales journal and determine that sales are properly recorded in
January. 
F. Inquire of credit manager about the collectability of various receivables. 
G. Review   disclosure   checklist   for   recommended   and   required   accounts   receivable
disclosures. 

4. D

5. D
e
6. N

Task­Based Simulation 12
Inventory Audit Objectives
and Procedures Authoritative
Literature Help
The auditor determines that each of the following objectives will be part of the audit of 
Enright Corporation. For each audit objective, select a substantive procedure that would help 
to achieve the audit objectives. Each of the procedures may be used once, more than once, or 
not at all.
MODULE 4   RESPONDING TO RISK ASSESSMENT 135

Substantive procedure
A. Review   minutes   of   board   of   directors   meetings   and   contracts,   and   make   inquiries   of
management. 
B. Test inventory transactions between a preliminary physical inventory date and the balance
sheet date. 
C. Obtain confirmation of inventories pledged under loan agreement. 
D. Review   perpetual   inventory   records,   production   records,   and   purchasing   records   for
indication of current activity. 
E. Reconcile   physical   counts   to   perpetual   records   and   general   ledger   balances   and
investigate significant fluctuation. 
F. Examine sales after year­end and open purchase order commitments. 
G. Examine paid vendors’ invoices, consignment agreements, and contracts. 
H. Analytically review and compare  the relationship of inventory  balance  to recent
purchasing, production, and sales ac­tivity. 

1. I

2. D

3. D

4. D
e
5. D
Task­Based Simulation 13

Spreadsheet Authoritative
Completion Literature Help

Analytical procedures are evaluations of financial information made by a study of 
plausible relationships among financial and nonfinancial data. Understanding and evaluating 
such relationships are essential to the audit process.
The following spreadsheet with the financial statements were prepared by Holiday 
Manufacturing Co. for the year ended December 31, 20X1. Also presented are various 
financial statement ratios for Holiday as calculated from the prior year’s finan­cial statements. 
Sales represent net credit sales. The total assets and the receivables and inventory balances at 
December 31, 20X1, were the same as at December 31, 20X0.
A B C D E F G
1 Holiday Manufacturing Co.
2 Balance Sheet
3 December 31, 20X1
4
5 Cash $240,000 Accounts Payable $160,000
6 Receivables 400,000 Notes payable 100,000
7 Inventory 600,000 Other current liabilities 140,000
8 Total current assets $1,240,000 Total current liabilities 400,000
9
10 Plant and equipment—net 760,000 Long-term debt 350,000
11 Common stock 750,000
12 Retained earnings 500,000
13 Total assets $2,000,000 Total liabilities and capital $2,000,000
14
15
16 Income Statement
17 Year ended December 31, 20X1
18
19 Sales $3,000,000
20 Cost of goods sold
21 Materials 800,000
22 Labor 700,000
23 Overhead 300,000 1,800,000
24 Gross margin 1,200,000
25
26 Selling expenses 240,000
27 General and admin. exp. 300,000 540,000
28 Operating income 660,000
29 Less: interest expense 40,000
136 MODULE 4 RESPONDING TO RISK ASSESSMENT

A B C D E F G
30 Income before taxes 620,000
31 Less: federal income taxes 220,000
32 Net income $400,000
33
34
35
36 Ratios 12/31/X1 12/31/X0
37 Current ratio (1) 2.5
38 Quick ratio (2) 1.3
39 Accounts receivable turnover (3) 5.5
40 Inventory turnover (4) 2.5
41 Total asset turnover (5) 1.2
42 Gross margin % (6) 35%
43 Net operating margin % (7) 25%
44 Times interest earned (8) 10.3
45 Total debt to equity % (9) 50%

Insert spreadsheet formulas into the worksheet to allow the direction calculation of 
each ratio (1 through 9). Use cell location rather than amounts.
MODULE 4   RESPONDING TO RISK ASSESSMENT 137

MULTIPLE­CHOICE ANSWERS

1. b __ __ 31. c __ __ 61. a __ __ 91. b __ __ 121. c __ __ 151. d __ __


2. b __ __ 32. d __ __ 62. c __ __ 92. a __ __ 122. a __ __ 152. c __ __
3. c __ __ 33. c __ __ 63. b __ __ 93. d __ __ 123. c __ __ 153. d __ __
4. c __ __ 34. b __ __ 64. c __ __ 94. c __ __ 124. b __ __ 154. c __ __
5. d __ __ 35. a __ __ 65. a __ __ 95. a __ __ 125. d __ __ 155. d __ __
6. d __ __ 36. a __ __ 66. d __ __ 96. b __ __ 126. a __ __ 156. d __ __
7. a __ __ 37. a __ __ 67. d __ __ 97. a __ __ 127. a __ __ 157. c __ __
8. a __ __ 38. c __ __ 68. a __ __ 98. c __ __ 128. a __ __ 158. d __ __
9. d __ __ 39. c __ __ 69. b __ __ 99. c __ __ 129. d __ __ 159. a __ __
10. d __ __ 40. d __ __ 70. d __ __ 100. a __ __ 130. c __ __ 160. b __ __
11. d __ __ 41. b __ __ 71. a __ __ 101. d __ __ 131. c __ __ 161. c __ __
12. d __ __ 42. a __ __ 72. d __ __ 102. c __ __ 132. a __ __ 162. b __ __
13. c __ __ 43. d __ __ 73. c __ __ 103. c __ __ 133. a __ __ 163. c __ __
14. a __ __ 44. c __ __ 74. a __ __ 104. b __ __ 134. d __ __ 164. c __ __
15. b __ __ 45. b __ __ 75. a __ __ 105. a __ __ 135. b __ __ 165. c __ __
16. c __ __ 46. b __ __ 76. d __ __ 106. c __ __ 136. c __ __ 166. b __ __
17. a __ __ 47. c __ __ 77. c __ __ 107. b __ __ 137. c __ __ 167. d __ __
18. d __ __ 48. b __ __ 78. b __ __ 108. a __ __ 138. d __ __ 168. a __ __
19. b __ __ 49. d __ __ 79. a __ __ 109. d __ __ 139. b __ __ 169. d __ __
20. c __ __ 50. b __ __ 80. b __ __ 110. a __ __ 140. c __ __
21. c __ __ 51. b __ __ 81. a __ __ 111. d __ __ 141. b __ __
22. c __ __ 52. b __ __ 82. b __ __ 112. d __ __ 142. b __ __
23. a __ __ 53. c __ __ 83. d __ __ 113. d __ __ 143. a __ __
24. b __ __ 54. d __ __ 84. a __ __ 114. b __ __ 144. a __ __
25. c __ __ 55. b __ __ 85. c __ __ 115. c __ __ 145. b __ __
26. b __ __ 56. c __ __ 86. c __ __ 116. c __ __ 146. a __ __
27. c __ __ 57. b __ __ 87. c __ __ 117. a __ __ 147. b __ __
28. b __ __ 58. c __ __ 88. c __ __ 118. c __ __ 148. c __ __
29. c __ __ 59. b __ __ 89. a __ __ 119. b __ __ 149. c __ __ 1st:  __/169 = __%
30. c __ __ 60. a __ __ 90. d __ __ 120. d __ __ 150. d __ __ 2nd: __/169 = __%

MULTIPLE­CHOICE ANSWER EXPLANATIONS
transactions. Answer (a), (c), 
A.1.  Sufficient Appropriate Audit  and (d) are all incorrect because 
Evidence accuracy, cutoff and occurrence 
are class of transaction 
1. (b) The requirement is to identify the 
assertions. 
statement thatbest describes the meaning of
generally accepted auditing standards.  3. (c) The requirement is to identify the type 
Answer (b) is correct because generally  of evidence that is generally most reliable. 
accepted auditing standards deal with  Answer (c) is correct because audit 
measures of the quality of the performance  evidence obtained from knowledgeable 
of audit procedures. Answer (a) is incorrect independent sources outside the client 
because procedures relate to acts to be  company is more reliable than audit 
performed, not di­rectly to the standards.  evidence obtained from nonindependent 
Answer (c) is incorrect because gen­erally  sources (e.g., company sources who may be
accepted auditing standards have been  biased). Answer (a) is incorrect because 
issued by predecessor groups, as well as by audit evidence obtained from direct sources
the Auditing Standards Board. Answer (d)  is more reliable. Answer (b) is 
is incorrect because there may or may not 
be universal compliance with the 
standards.
2. (b) The requirement is to 
identify the reply that is not an 
assertion for classes of 
transactions. The assertionsfor 
classes of transactions are 
occurrence, completeness, 
accuracy, cutoff and 
classification. Answer (b) is 
correct since consistency is not 
an assertion for classes of 
incorrect because it is not clear as to  representation letter all represent 
whether copies or facsimile copies are  internally generated documents, 
more reliable. Answer (d) is incorrect  generally considered less 
because it is unclear whether audit  persuasive than externally 
evidence provided by original documents  generated documents. See SAS 
is more reliable than that provided through 106 for information on the 
a system of effective controls—indeed,  persua­siveness of audit evidence.
they may be one and the same.
5. (d) The requirement is to identify a correct 
4. (c) The requirement is to identify  pre­sumption about the reliability of audit 
the most persua­sive type of  evidence. Answer (d) is correct because 
evidence. Answer (c) is correct  SAS 106 indicates that effective internal 
because a bank statement  control provides more assurance about the 
represents evidence prepared  reliability of audit evidence than ineffective
outside of the entity and is  control. Answer (a) is incorrect because 
considered an audit evidence  information obtained directly is considered 
source which provides the auditor more reliable than that obtained indirectly. 
with a high level of assurance.  Answer (b) is incorrect because audit 
Answers (a), (b), and (d) are  evidence is normally persuasive rather than 
incorrect because prenumbered  convincing. Answer (c) is incorrect because
client purchase order forms,  reliability of audit evidence relates to the 
client work sheets and a  appropriateness of audit evidence. 
138 MODULE 4   RESPONDING TO RISK ASSESSMENT
are sub­jective and susceptible to bias. 
Answer (b) is incorrect be­cause all things 
6. (d) The requirement is to  being equal, an auditor would expect as­
determine the correct statement  sumptions that are consistent with prior 
with respect to the  periods. Answer (c) is incorrect because 
appropriateness of audit evi­ assumptions that are insensitive to 
dence. To be appropriate,  variations in underlying data have little 
evidence must be both reliable  predictive ability. Answer (d) is incorrect 
and relevant. Answer (a) is  because, often, one would expect 
incorrect because while exter­ assumptions similar to industry guidelines. 
nally generated evidence is 
generally considered to provide  A.2.  Types of Evidence
greater assurance of reliability,  9. (d) The requirement is to identify 
there are important excep­tions,  the reply that is not a basic 
(e.g., the confirmation is  procedure used in an audit. 
erroneously returned with no  Answer (d) is correct because the 
exception when one actually  term “test of direct evidence” is 
exists). Answer (b) is incorrect  not used in the professional 
because while evidence so  standards. Answer (a) is incorrect 
gathered is typically considered to because risk assessment 
provide greater assurance  procedures are used while 
concerning reliability, no simi­lar  obtaining an under­standing of the
generalization can be made about  client. Answer (b) is incorrect 
its relevance. An­swer (c) is  because sub­stantive procedures 
incorrect because oral  test account balances. Answer (c) 
representations from man­ is incorrect because tests of 
agement, when corroborated by  controls test the operating effec­
other forms of evidence, are  tiveness of controls. 
considered valid evidence. 
10. (d) The requirement is to identify 
7. (a) The requirement is to identify the procedure an auditor would 
the least persua­sive type of  perform first in evaluating 
evidence. Answer (a) is correct  management’s accounting estimates
because evi­dence secured solely for reasonableness. Answer (d) is 
from within the entity, here  cor­rect because in evaluating 
prenum­bered purchase order 
reasonableness, the auditor should 
forms, is considered less 
first obtain an understanding of how
persuasive than evidence 
management developed the 
obtained from independent 
estimate. Answers (a), (b), and (c) 
sources. An­swer (b) is incorrect 
because a bank statement (even  are all incorrect be­cause 
though received from the client)  developing independent 
is externally created and  expectations, considering 
therefore more persuasive than  appropriateness of key factors or 
audit evidence secured solely  assumptions, and testing 
from within the entity. Answer  calculations occur after obtaining 
(c) is incorrect because evidence  the understanding. See AU 342 for 
obtained directly by the auditor  information on auditing accounting 
through observation is con­ estimates. 
sidered relatively persuasive. 
Answer (d) is incorrect be­cause 
correspondence from the client’s 
attorney about liti­gation is 
obtained directly from 
independent sources and is 
therefore more persuasive than 
audit evidence secured from 
within the entity. 
8. (a) The requirement is to identify an area of
concern to auditors when evaluating the 
reasonableness of an entity’s accounting 
estimates. Answer (a) is correct because 
AU 342 states that in evaluating the 
reasonableness of estimates auditors 
normally concentrate on assumptions that 
aggregated nature of the financial
11. (d) The requirement is to identify a factor  statements makes the use of 
that an auditor would concentrate upon  poten­tially unrecorded items 
when evaluating the reason­ableness of an  unlikely as a method of 
accounting estimate. Answer (d) is correct  identifying unrecorded items. 
because AU 342 states that the auditor  Answer (b) is incorrect because 
normally concen­trates on key factors and  testing potentially unrecorded 
assumptions that are deviations from  items to the financial statements 
historical patterns, as well as those that are  ad­dresses the completeness 
significant to the accounting estimate,  assertion. Answer (d) is incorrect 
sensitive to variations, and sub­jective and  because testing from the 
susceptible to misstatement and bias.  supporting evidence to the ac­
Answer (a) is incorrect because deviations  counting records addresses the 
from historical patterns, not consistent  completeness assertion. See the 
patterns, are concentrated upon. Answer (b) outline of SAS 106 for 
is incorrect because factors and  information on the financial 
assumptions that are similar to industry  statement assertions. 
guidelines are often reasonable. Answer (c) 
is in­correct because subjective factors and  14. (a) The requirement is to 
assumptions that are susceptible to bias are  determine which balance 
concentrated upon, not objective ones that  remaining in a “suspense” 
are not susceptible to bias. See AU 342 for  account would be of most con­
cern to an auditor. Answer (a) is 
information on the manner in which 
correct because suspense debits 
auditors consider accounting esti­mates. 
that management believes will 
12. (d) The requirement is to  benefit future opera­tions must be
identify one of an audi­tor’s  audited carefully to determine 
objectives when evaluating an  whether they have value and 
entity’s accounting es­timates.  should be classified as an asset. 
Answer (d) is correct because  Answer (b) is incorrect because 
when evaluating accounting  when an auditor has already 
estimates an auditor’s objectives  determined that a suspense debit 
are to obtain sufficient  has value it becomes a relatively 
appropriate audit evidence that  straightforward issue of the 
(1) all material accounting  item’s proper classification. 
estimates have been developed,  Answer (c) is incorrect because 
(2) those ac­counting estimates  the conservative approach taken 
are reasonable, and (3) those  in audits is likely to cause the 
accounting estimates are in  auditor to have some­what more 
conformity with GAAP.  concern about suspense purported
to be assets [i.e., answer (a)], 
13. (c) The requirement is to  than for those classified as 
determine the proper test for the  current liabili­ties. Answer (d) is 
existence assertion of an asset.  incorrect because when the 
Answer (c) is correct because  auditor determines that the 
testing from accounting records  suspense credits represent 
to the supporting evidence  customer de­posits, establishing a
discloses whether recorded  proper accounting will ordinarily 
transactions occurred and  be relatively simple. 
whether the asset exists. Answer 
(a) is incorrect be­cause testing  B.1.a. Analytical Procedures
for the completeness assertion  15. (b) The requirement is to identify the 
addresses whether there are  procedure thatwould not be considered an 
unrecorded items. Also, the 
analytical procedure. Analyti­
MODULE 4   RESPONDING TO RISK ASSESSMENT 139
analytical procedures is 
identification of unusual 
cal procedures consist of evaluations of  transactions and events, and 
financial informa­tion made by a study of  amounts, ratios and trends that 
plausible relationships among both  might indicate misstatements. 
financial and nonfinancial data. Answer (b) Answer (a) is incorrect be­cause 
is correct be­cause projecting an error rate  the limited number of journal 
from a statistical sample to an actual  entries traced to the general 
population is not a comparison of a  ledger in a sample is unlikely to 
plausible relation­ship. Answers (a), (c),  include the errone­ous journal 
and (d) are all incorrect because they all  entries. Answer (b) is incorrect 
include a study of plausible relationships. because while an evaluation of 
16. (c) The requirement is to identify the type  the effectiveness of internal 
of ana­lytical procedure an auditor would  control might help detect such 
most likely use in devel­oping relationships misstatements, it is somewhat 
among balance sheet accounts when  doubtful due to the fact that it is 
reviewing the financial statements of a  likely that few journal entries are 
nonpublic entity. Answer (c) is correct  in­volved. Answer (c) is incorrect
because balance sheet accounts may be  because there is no indica­tion 
analyzed through a number of ratios (e.g.,  that the fraud involves 
current ratio). Answer (a) is incorrect  differences between controlling 
because trend analysis is often more  accounts and subsidiary records. 
appropriate for income statement analysis.  19. (b) The requirement is to 
Answer (b) is incorrect because regression  identify the account that would 
analysis, while used in practice, is not used  yield the highest level of 
as frequently as is ratio analysis. Answer  evidence through the per­
(d) is incorrect because risk analysis in and  formance of analytical 
of itself is not a type of analytical  procedures. As higher levels of 
procedure. See AR 100 for information on  as­surance are desired from 
reviews of nonissuer (nonpublic) entities.  analytical procedures, more pre­
17. (a) The requirement is to identify  dictable relationships are 
how an auditor may achieve audit required to develop the auditor’s
expectation. Relationships 
objectives related to particular 
assertions. Answer (a) is correct  involving income statement ac­
counts tend to be more 
because an auditor may perform 
ana­lytical procedures to achieve  predictable than relationships in­
volving only balance sheet 
an audit objective related to a 
particular assertion. Answer (b) is accounts, and relationships in­
volving transactions not subject 
incorrect because a sys­tem of 
quality control provides the CPA  to management discretion are 
generally more predictable. 
firm with reason­able assurance 
of conforming with professional  Answer (b) is correct be­cause 
interest expense relates to the 
standards. Answer (c) is incorrect
because while working papers  income statement, and 
pro­vide support for the audit 
report and aid in the conduct and 
supervision of the audit, they do 
not in and of themselves achieve 
audit objectives (see SAS 103 for
information on audit working 
papers). Answer (d) is incorrect 
because in­creasing the level of 
detection risk does not in and of 
itself achieve audit objectives. 
18. (d) The requirement is to identify
the procedure most likely to 
detect a fraud involving 
misstatement of in­come 
statements due to the recording 
of journal entries with unusual 
combinations of debits and 
credits to expense and revenue 
accounts. Answer (d) is correct 
because an objec­tive of 
are often less predictable. 
because interest expense is subject to  For example, management might incur 
only limited manage­ment discretion,  maintenance expense rather than replace 
given the existence of the related debt.  plant and equipment, or they may delay 
Answers (a) and (c) are incorrect because  advertising expenditures. Answer (d) is 
accounts receiv­able and accounts  incorrect because prior year data is 
payable are balance sheet accounts. An­ sometimes not a reliable predictor of sub­
swer (d) is incorrect because travel and  sequent year’s data. 
entertainment ex­pense is normally 
subject to management discretion. See  22. (c) The requirement is to identify a 
AU 329 for more information on the use  basic premise underlying the 
of analytical pro­cedures. application of analytical 
procedures. An­swer (c) is correct 
20. (c) The requirement is to determine what is  because, as indicated in AU 329, a 
in­cluded when analytical procedures are  basic premise underlying the 
used in the overall review stage of an audit. application of analytical procedures
Answer (c) is correct because the overall  is that plausible relationships 
review stage includes reading the financial  among data may reasonably be 
state­ments and notes and considering the  expected to exist and continue in 
adequacy of evidence gathered in response  the absence of known con­ditions to
to unusual or unexpected balances. Answer  the contrary. Answer (a) is incorrect
(a) is incorrect because analytical  because the study of financial ratios
procedures are not particularly aimed at  is an approach to identifying un­
gathering evidence on account bal­ances 
usual fluctuations, not an acceptable
that have not changed. Answer (b) is 
alternative to investi­gating them. 
incorrect be­cause analytical procedures do 
Answer (b) is incorrect because 
not directly test control procedures. Answer
analytical procedures may be either 
(d) is incorrect because tests of transac­
statistical or nonstatistical. An­swer
tions to corroborate management’s 
financial statement asser­tions are  (d) is incorrect because analytical 
performed when considering internal  procedures may be used as 
control and for substantive tests of  substantive tests, and may result in 
transactions. See AU 329 for infor­mation  modification of the scope of tests of
on analytical procedures.  balances and transactions. 

21. (c) The requirement is to determine 
the most pre­dictable relationship  23. (a) The requirement is to identify
for purposes of analytical  the stages of an audit for which 
procedures applied as substantive  the use of analytical procedures 
tests. Answer (c) is correct because  is required. AU 329 requires the 
AU 329 indicates that relationships  use of analytical procedures at 
involving income state­ment  both the planning and overall 
accounts tend to be more  review stages of the audit, but not
predictable than relationships 
as a substantive test. 
involving only balance sheet  24. (b) The requirement is to 
accounts. Answer (a) is incor­rect  determine the reply that would 
because, as indicated, relationships  facilitate an auditor’s analytical 
involving income statements are  procedures. An­swer (b) is 
considered more predictable.  correct because use of a standard 
Answer (b) is incorrect because  cost system (a form of budgeting)
relationships involving transactions  produces variance reports that 
sub­ject to management discretion  will allow 
140 MODULE 4   RESPONDING TO RISK ASSESSMENT
warranty ex­pense with actual 
warranty expense. 
the auditor to compare the financial 
information with the standard cost system  27. (c) The requirement is to identify the best 
data to identify unusual fluctuations. See  individual to perform analytical procedures 
AU 329 for the approach used. Answer (a)  in the overall review stage of an audit. At 
is incorrect because segregating obsolete  this stage of an audit the objective of 
inventory before the inventory count is  analyti­cal procedures is to assist the 
related to the auditor’s physical inventory  auditor in assessing the con­clusions 
observa­tion and will not necessarily affect reached and in the evaluation of the overall 
analytical procedures. Answer (c) is  finan­cial statement presentation. Answer 
incorrect because correcting a material  (c) is correct because an experienced 
weak­ness in internal control before the  individual with business and industry 
beginning of the audit generally will have  knowl­edge is likely to be able to fulfill this
minimal, if any, effect on the historical  function. Answer (a) is incorrect because a 
information used for analytical procedures. staff accountant who has performed the 
Answer (d) is incorrect because data from  substantive auditing procedures may not be 
independent sources outside the entity is  able to objec­tively perform the analytical 
more likely to be reliable than purely  procedure and may not have the necessary 
internal sources. experience to perform the function. Answer
25. (c) The requirement is to identify  (b) is incorrect because the managing 
partner of the office may not be close 
the most likely effect on an audit of
enough to the audit to perform the function 
having performed analytical 
effec­tively. Answer (d) is incorrect 
procedures in the overall review 
because the individual in charge of quality 
stage of an audit which suggest that control and the peer review program should
several accounts have unexpected  be as independent as possible of the audits 
relationships. Answer (c) is correct  he or she is considering. See AU 329 for 
because when unexpected  guidance on analytical pro­cedures. 
relationships exist, ad­ditional tests 
of details are generally required to  B.1.b.  Tests of Details of Transactions 
determine whether misstatements  and Balances
exist. Answer (a) is incorrect be­
cause irregularities (fraud) may or  28. (b) The requirement is to identify the 
may not exist. An­  best exampleof a substantive test. Answer
(b) is correct because confir­
swer (b) is incorrect because internal 
control activities may or may not be 
operating effectively. Answer (d) is 
incorrect because ordinarily the situation 
need not be communicated to the audit 
committee. 
26. (b) The requirement is to identify 
the comparison an auditor most 
likely would make in evaluating 
an entity’s costs and expenses. 
Answer (b) is correct because 
payroll expense is an income 
statement expense and because it 
may be expected to have a 
relationship with that of the prior 
year. Answer (a) is incorrect 
because the accounts receivable 
ac­count is not a cost or expense. 
Answer (c) is incorrect be­cause 
comparing budgeted sales with 
actual sales of the cur­rent year is 
more likely to be performed than 
comparing budgeted sales with 
those of prior years. Answer (d) is
in­correct because comparing the 
budgeted current year’s war­ranty 
expense with the current year’s 
contingent liabilities is less direct 
than that in answer (b), and 
because one would be more likely
to compare current year budgeted 
mation of balances of accounts receivable  B.2.  Preparing Substantive Test Audit 
will provide a test of the ending account  Programs
balance and is therefore a detailed test of a 
32. (d) The requirement is to 
balance, a type of substantive test. Answers 
determine the most likely 
(a) and (c) are incorrect because examining  approach for auditing the 
approval of cash disburse­ments and  statement of cash flows. An­swer 
comparison of signatures on checks to a list  (d) is correct because the 
of authorized check signers are tests of  statement of cash flows in­cludes 
controls. Answer (d) is incorrect because  accounts considered during the 
flowcharting a client’s cash receipts system is audit of the balance sheet and 
a method used to document the auditor’s  income statements and, 
under­standing of internal control. accordingly, the most fre­quent 
29. (c) The requirement is to identify the  approach is to reconcile amounts. 
Answers (a), (b), and (c) are all 
objective of tests of details of transactions 
incorrect because they suggest 
performed as substantive tests. Answer (c) 
approaches not typically followed
is correct because SAS 110 states that the 
when auditing the statement of 
objec­tive of tests of details of transactions 
cash flows. 
performed as substan­tive tests is to detect 
material misstatements in the financial  33. (c) The requirement is to determine 
statements. Answer (a) is incorrect because  the direction of audit testing to 
while perform­ing tests of details of  determine whether transactions 
transactions as substantive tests com­plies  have been recorded. Answer (c) is 
with generally accepted auditing standards,  correct because to determine 
this is not their objective. Answers (b) and  whether transactions have been 
(d) are incorrect because neither attaining  recorded the auditor will test from 
assurance about the reliability of the ac­
the original source documents to 
counting system nor the evaluation of the 
operating effec­tiveness of management’s  the recorded entries.Answers (a), 
policies and procedures are the objective of  (b), and (d) are incorrect because 
tests of details of transactions performed as  when testing from the general 
substantive tests.  ledger, the adjusted trial balance, or 
from 
30. (c)   The   requirement   is   to   find
the   statement   which   describes
substantive audit tests.  Answer
(c)   is   correct   be­cause
substantive tests are defined as
tests of transactions, direct tests
of   financial   balances,   or
analytical procedures. 
Answer (a) is incorrect because substantive
tests may not be eliminated due to the 
limitations of internal control. An­swer (b) 
is incorrect since substantive tests 
primarily di­rectly test ending financial 
statement balances, not subse­quent 
events. Answer (d) is incorrect because 
substantive 
tests decrease with increased reliance on 
internal control.
31. (c) The requirement is to determine the 
account forwhich the auditor is most likely to 
perform extensive tests for possible 
understatement. An analysis of past audits, in 
which existing financial statement errors were
not discov­ered prior to the issuance of the 
financial statements, reveals that the great 
majority of the errors resulted in overstated 
profits. Therefore, the risk to a CPA is that the
client is overstating profits. Answer (c) is 
correct because it is the only item whose 
understatement results in overstated profits. 
Answers (a), (b), and (d) are incorrect because
understate­ment of these items would result in
understated profits.
MODULE 4   RESPONDING TO RISK ASSESSMENT 141
37. (a) The requirement is to identify 
a primary purpose of an auditor’s 
general journal entries the auditor is  working papers. SAS 103 states 
dealing with the entries that have been  that working papers serve mainly 
recorded, not whether all transactions have to provide the principal support 
been recorded. for the auditor’s report and to aid 
B.3.  Documentation the auditor in the conduct and 
supervision of the audit. 
34. (b) The requirement is to identify 
38. (c) The requirement is to identify the least 
the correct statement concerning 
likely item to be included in the permanent 
the deletion of audit 
file of an auditor’s working papers. Answer
documentation. Answer (b) is 
(c) is correct because permanent files 
correct because after the audit file
include information affecting a number of 
has been completed (ordinarily 60
years’ audits, and the working trial balance 
days or less after the issuance of 
relates most directly to the current and, to a 
the audit report), no portions of 
limited extent, the subsequent year’s audit. 
audit documentation should be 
Answers (a), (b), and (d) are all incorrect 
deleted. Answer (a) is incorrect 
because bond in­denture agreements, lease 
because after completion of the 
agreements, and a flowchart of internal 
audit file no documentation 
control affect more years’ audits than does 
should be deleted or discarded. 
a specific year’s working trial balance. 
Answer (c) is incorrect because 
professional skepticism is not the  39. (c) The requirement is to 
basis for determining deletions.  determine a difference between 
Answer (d) is incorrect because  an auditor’s working trial balance
prior to the file completion date  and financial statements without 
most superseded documentation  footnotes. Answer (c) is correct 
may be deleted (an exception is  because a working trial balance 
that information that reflects a  includes columns for 
disparate point of view should be  reclassification and adjustments. 
retained).  Answers (a), (b), and (d) are all 
incorrect because while they 
35. (a) The requirement is to 
suggest information that might be
determine how long audit 
in­cluded on a working trial 
documentation must be retained. 
balance, they will not be included
Answer (a) is correct because 
in the form of additional 
SAS 103 requires that they be 
columns. 
maintained a minimum of five 
years. Answers (b), (c) and (d)  40. (d) SAS 103 states that the degree
all present other, incorrect time  of documentation for a particular
periods.  audit  area   should   be  affected   by
(1) the risk 
36. (a) The requirement is to identify 
the most likely pair of accounts to
be analyzed on the same working 
paper. Answer (a) is correct 
because an auditor will often 
consider interest income with 
notes receivable because the 
interest is earned on those notes 
and therefore closely related. An­
swer (b) is incorrect because 
interest receivable relates to an 
asset account (notes receivable) 
while accrued interest pay­able 
relates to a liability account 
(notes payable) and ac­cordingly 
one would expect separate 
working papers. An­swer (c) is 
incorrect because notes payable 
and receivable are entirely 
separate accounts. Answer (d) is 
incorrect be­cause interest 
income relates to interest­bearing 
securities while interest expense 
relates to debt accounts. 
incorrect because an interbank 
of   material   misstatement,   (2)   extent   of transfer schedule summa­rizes 
judgment,   (3)   nature   of   the   auditing transfers between banks among 
procedures,   (4)   significance   of   the accounts. Answer (b) is incorrect 
evidence  obtained, (5)  nature  and  extent because the term “carryforward 
of  the   exceptions   identified,   and   (6)   the schedule” is not frequently used. 
need to document a conclusion that is not Answer (c) is incorrect because 
obvi­ous   from   the   documentation   of   the supporting schedules present the 
work. details supporting the information
on a lead schedule. For example, 
41. (b) The requirement is to identify
a detailed bank reconciliation for 
the required docu­mentation in an
a cash account might serve as a 
audit.   Answer   (b)   is   correct
supporting schedule for an 
because 
account on the cash lead schedule.
SAS 108 requires a written audit program 
setting forth in detail the procedure  44. (c) The requirement is to identify the 
necessary to accomplish the engage­ment’s documenta­tion that is required for an audit 
objectives. Answer (a) is incorrect because  in accordance with gener­ally accepted 
while flowcharts and narratives are  auditing standards. Answer (c) is correct 
acceptable methods of docu­menting an  be­cause SAS 103 requires that the working
auditor’s understanding of internal control,  papers show that the accounting records 
they are not required. Answer (c) is  agree or reconcile with the financial 
incorrect because a planning memorandum  statements. Answer (a) is incorrect because 
is not required. Answer (d) is incorrect be­ neither a flow­chart nor an internal control 
cause completion of an internal control  questionnaire is required. An­swer (b) is 
questionnaire is not required.  incorrect because the Professional 
42. (a) The requirement is to identify  Standards do not require the use of an 
a factor that would most likely  engagement letter. Answer (d) is incorrect 
affect the auditor’s judgment  because when control risk is assessed at the
about the quantity, type, and  maxi­mum level, the auditor’s 
content of the working papers. 
understanding of internal control needs to 
be documented, but the basis for the 
An­swer (a) is correct because the
conclusion that it is at the maximum level 
Professional Standards state 
need not be documented. 
that the assessed level of control risk will 
affect the quantity, type and content of  45. (b) The requirement is to identify
working papers. SAS 103 provides a listing the point at which no deletions of
of this and other factors. For example, the  audit documentation are allowed.
quantity, type and content of working  Answer 
papers will be affected by whether tests of  (b) is correct because the professional 
controls have been performed.  standards indicate that audit documentation 
43. (d) The requirement is to identify  may not be deleted after the documen­tation 
the type of audit working paper  completion date. Answer (a) is incorrect 
that reflects the major components because documentation may be deleted 
of an amount reported in the  between the client’s year­end date and the 
financial statements. Answer (d)  documentation completion date. Answer (c) is
is correct because lead schedules  incorrect because the last date of significant 
aggregate the major compo­nents  fieldwork is prior to the documentation 
to be reported in the financial  completion date. Answer (d) is incorrect 
statements. Answer (a) is  because the report release date is up to 60 
days 
142 MODULE 4   RESPONDING TO RISK ASSESSMENT
relationships that the bank has with the 
client. Answer (a) is incorrect because bank
prior to the documentation completion  employees who com­plete the form realize 
date; 45 days for issuer (public)  that they may verify confidential in­
company audits. formation with the auditor. Answer (b) is 
incorrect because while it is correct that the
46. (b) The requirement is to determine the type
employee who completes the con­firmation 
of audi­tor that should be able to understand form will not generally inspect the accuracy
audit documentation of the nature, timing,  of the bank reconciliation, this does not 
extent, and results of audit procedures  limit the confirmation’s usefulness. Answer
performed. Answer (b) is correct because  (c) is incorrect because the employee who 
PCAOB Standard 3 requires that audit  completes the form does not need to have 
documentation be understandable to an  access to the client’s cutoff bank statement 
experienced auditor having no prior  to complete the confirmation. 
connection with the engagement. Answer 
(a) is incorrect because the require­ment is  50. (b) The requirement is to 
not limited to audit team members. Answer  determine the type of evi­dence 
(c) is incorrect because the requirement is  that is likely to result in an 
more limited than to any certified public  auditor limiting substan­tive 
accountant. Answer (d) is incorrect both  audit tests of sales transactions 
because there is no certification of a peer  when control risk is as­sessed as 
review specialist and because this is not a  low for the occurrence assertion 
requirement.  for sales. Answer (b) is correct 
because an auditor may analyze 
47. (c) The requirement is to identify 
the completeness of sales using 
the period for which audit  cash receipts and accounts 
documentation should be retained receivable (for example, an 
for issuer (public) company  auditor may add year­end re­ 
audits. Answer (c) is correct 
ceivables to cash receipts and subtract 
because PCAOB Standard 3 
beginning receivables to obtain an estimate 
requires that audit documentation of sales). Answer (a) is incorrect be­cause 
be retained for the longer of  the opening and closing inventory balances 
seven years or the period required will only provide indirect evidence on sales
by law. Answer (a) is incorrect  through calculation of the cost of goods 
both because of the seven­year  sold. Answer (c) is incorrect because while 
requirement, and because it is the  shipping as of year­end will help assure an 
longer of seven years or the  accurate cutoff of 
period required by law. Answer 
(b) is incorrect be­cause while 
seven years is the general 
requirement, a longer period may 
be required by law. Answer (d) is 
incorrect be­cause audit 
documentation need not be 
retained indefinitely. 
C.1.  Evidence—Cash
48. (b) The requirement is to identify
the information that an auditor 
usually confirms on one form. 
Answer (b) is correct because the 
standard form to confirm account 
bal­ance information with 
financial institution requests 
informa­tion on both the cash in 
bank and collateral for loans. An­
swers (a), (c), and (d) are all 
incorrect because they suggest 
pairs of information that are not 
usually confirmed on one form. 
49. (d) The requirement is to determine a 
reason that the usefulness of the standard 
bank confirmation request may be limited. 
Answer (d) is correct because the bank 
employee who completes the form often 
will not have access to all the financial 
represent deposits in transit at 
sales, the receiving activity has no necessary  December 31, 2001. Deposits in 
relationship to the sales figure. Answer (d) is  transit are those that have been 
incorrect because while the cutoff of sales  sent to the bank prior to year­end,
will provide evidence on the completeness of  but have not been received by the
sales, the purchases portion of the reply is not bank as of year­end. Answer (b) 
appropriate. is correct because both check 
#101 and check #303 have been 
51. (b) The requirement is to determine the two disbursed per books as of year­
checks that might indicate kiting, a form of  end, but have not yet been 
fraud that overstates cash by simultaneously received by the bank as of De­
including it in two or more bank accounts.  cember 31. Checks #202 and 
Answer (b) is correct because checks #202  #404 have been received by the 
and #404 include cash in two accounts at  bank as of year­end and 
year­end. The cash represented by check  accordingly are not in transit. 
#202 is included in both State Bank and  Answer (a) is incorrect because 
County Bank cash as of December 31. This  check #202 has been re­ceived 
is because its receipt is recorded prior to  before year­end. Answer (c) is 
year­end, but its disbursement is recorded  incorrect because both checks 
after year­end. (For the cash receipts  #202 and #404 have been 
journal to remain in balance prior to year­ received before year­end. Answer
end, some account must have been credited  (d) is incorrect because check 
on December 30 to offset the debit to cash.) #404 has been received before 
Check #404 also represents a situation in  year­end. 
which the cash is included in two accounts 
as of year­end; the check may represent a  53. (c) The requirement is to identify why 
situation in which a shortage in the account  auditors trace bank transfers for the last 
is concealed through deposit of the check  part of the audit period and the first part of 
that is not re­corded on the books at year­ the subsequent period. Answer (c) is 
end. Check #101 does not re­sult in a  correct because auditors use a bank transfer
misstatement of cash since the books  schedule to analyze transfers so as to detect
recorded both the debit and credit portions  kiting that overstates cash balances. 
of the entry before year­end, while both  Answers (a) and (b) are incorrect because 
banks recorded them after year­end. Check  the process of analyzing transfers is not an 
#303 represents a situation in which funds  efficient way to determine whether the cash
are disbursed on the Federal account as of  receipts journal was held open or when the 
year­end, but not received into the  year’s final checks were mailed. Auditors 
American account (per bank or per books)  use a bank cutoff statement rather than a 
until after year­end; check #303  bank transfer schedule to help detect these 
accordingly understates cash and the nature  situations. Answer (d) is incorrect because 
of the debit for the entry on December 31 is the process of analyzing transfers is 
unknown. An­swer (a) is incorrect because  unlikely to identify any unusual payments 
neither check #101 nor #303 overstates  or receipts from related parties. 
cash. Answer (c) is incorrect because check 
54. (d) The requirement is to determine
#101 does not overstate cash. Answer (d) is 
the source of evidence which does 
incorrect because check #303 understates 
cash.  not contain information on the 
balance per bank in a bank 
52. (b) The requirement is to  reconciliation. Answer (d) is correct
determine the checks which 
MODULE 4   RESPONDING TO RISK ASSESSMENT 143
not request, nor expect, the 
financial institution to conduct a 
because the general ledger contains only  comprehensive, detailed, search 
the client’s cash balance, not the balance  of its records for other accounts. 
per bank. Answer (a) is incorrect because  Answer (a) is incorrect because a
the beginning balance on a cutoff statement standard confirmation request 
repre­sents the year­end bank balance.  will not detect kiting, a 
Answer (b) is incorrect because the primary manipulation causing an amount 
purpose of a year­end bank statement is to  of cash to be included 
present information on the balance per  simultaneously in the balance of 
bank. Answer (c) is incorrect because the  two or more bank accounts. 
first question on a standard bank  Answer (c) is incorrect because 
confirmation form requests information on  bank statements available from 
the year­end balance per bank. the client allow the CPA to 
55. (b) The requirement is to identify the cash  prepare a proof of cash. Answer 
transfer which will result in a misstatement  (d) is incorrect because the 
of year­end cash. An­swer (b) is correct  standard form does not request 
because the receipt is recorded on the books infor­mation about contingent 
prior to year­end, while the disbursement is  liabilities and secured 
recorded subsequent to year­end. Therefore, transactions. 
the cash on the books is overstated.  58. (c) The requirement is to determine the 
Answers (a), (c), and (d) are incorrect  assertion (or assertions) being tested by a 
because they do not reveal a cutoff error.  test of a control in which an auditor 
Answer (a) is incorrect because both the  observes the mailing of monthly statements
disbursement and receipt are recorded on  to a cli­ent’s customers and reviews 
the books prior to year­end; note that one  evidence of follow­up on errors reported by
would expect to see an outstanding check  the customers. Answer (c) is correct 
on the disbursing bank reconcilia­tion as of  because observing the mailing of monthly 
year­end. Answer (c) is incorrect because  statements and follow­up of errors will 
both the disbursement and receipt are  provide evidence to the auditor as to 
recorded on the books prior to year­end;  whether 
one would expect the disbursing bank  the receivables exist at a given date; the tests 
reconcilia­tion to show an outstanding  do not directly address the presentation and 
check and the receiving bank to show a 
disclosure assertion since little evidence is 
deposit in transit as of year­end. Answer (d)
obtained about whether financial statement 
is in­correct because the entire transaction 
com­ponents are properly classified, 
is recorded after year­end. 
described, and disclosed.
56. (c) The requirement is to 
determine an approach for 
detecting the concealing of a cash
shortage by transporting funds 
from one location to another or 
by converting nego­tiable assets 
to cash. Answer (c) is correct 
because the tim­ing of the 
performance of auditing 
procedures involves the proper 
synchronizing of their application
and thus compre­hends the 
possible need for simultaneous 
examination of, for example, cash
on hand and in banks, securities 
owned, bank loans, and other 
related items. 
57. (b) The requirement is to identify
the primary pur­pose of sending a
standard confirmation request to 
financial institutions with which 
the client has done business 
during the year. Answer (b) is 
correct because CPAs generally 
provide the account information 
on the form and ask for balance 
corroboration. The form 
explicitly states that the CPAs do 
the year­end bank confirmations 
See SAS 106 for a discussion of the  will not include deposits in 
various financial statement assertions.  transit. 
Answer (a) is incorrect because the 
presentation and disclosure assertion is  C.2.  Evidence—Receivables
not addressed. Answer (b) is incorrect  61. (a) The requirement is to identify the 
because the presentation and disclosure 
correct state­ment concerning the use of 
assertion is not addressed and because the 
negative confirmation requests. Answer (a) 
existence or occurrence assertion is 
addressed. Answer (d) is incorrect because is correct because AU 330 states that 
the existence or occurrence assertion is  unreturned negative confirmation requests 
addressed. rarely provide significant evidence 
concerning financial statements assertions 
59. (b) The requirement is to determine the  other than certain aspects of the existence 
most likely audit step summarized by the  assertion. Answer (b) is incorrect because 
tick mark placed under “date disbursed per  positive, not negative, confirmation 
bank.” Answer (b) is correct because the  requests are normally used when a low 
checks were written in December but  level of detection risk is to be achieved. 
cleared in January and should therefore be  Answer (c) is incorrect because alter­native
listed as outstanding on the year­end  procedures are not generally performed on 
outstanding check list of the applicable  unreturned negative confirmation requests 
bank reconciliation. Answer (a) is incorrect since it is assumed that the respondent did 
because the tick marks are beside the date  not reply because of agreement with the 
per bank, and not per books. Answer (c) is  bal­ance on the confirmation request. 
incorrect because the December cash  Answer (d) is incorrect because 
disbursements journal, and not the January  respondents may not reply when 
cash disbursements journal, will include  misstatements are in their favor. 
these disbursements. Answer (d) is 
62. (c) The requirement is to identify 
incorrect because the year­end bank 
confirmations do not include information  the most likely action taken by an 
on out­standing checks.  auditor when no reply is received 
to positive confirmation requests. 
60. (a) The requirement is to  Answer (c) is correct be­cause 
determine the most likely audit  asking the client to contact 
step summarized by the tick mark customers to ask that con­firmation
placed under “date deposited per  requests be returned may increase 
bank.” Answer (a) is correct  response rates. Answer (a) is 
because deposits recorded on the  incorrect because the lack of a 
books as of 12/31 should be  reply to a con­firmation request 
included as deposits in transit on  does not necessarily indicate that 
the applicable bank  the ac­count needs to be written 
reconciliation. Answer (b) is  off. Answer (b) is incorrect be­
incorrect because the tick mark is  cause accounts receivable 
placed be­side the “bank”  confirmations deal more directly 
column, and not the books  with existence than with valuation 
column. An­swer (c) is incorrect  or completeness and be­cause 
because the December cash  alternative procedures may provide
receipts journal, and not the  the auditor with the desired 
January cash receipts journal 
assurance with respect to the 
should include the deposit. 
nonrespondents. 
Answer (d) is incorrect because 
144 MODULE 4   RESPONDING TO RISK ASSESSMENT
debtor is asked to respond only if s/he 
disagrees with the information on the 
Answer   (d)   is   incorrect   because   the confirmation; thus, no reply is assumed to 
assessed   level   of   inher­ent   risk   will   not indicate agree­ment. Answers (c) and (d) 
normally be modified due to confirmation are incorrect because while ad­ditional 
results. procedures may be required when 
collectibility isquestionable, alternative 
63. (b) The requirement is to 
procedures are those used in lieu of 
determine the best sam­pling unit confirmation. 
for confirmation of accounts 
receivable when many  66. (d) The requirement is to identify 
differences between the recorded the most likely alternate 
account balances and the  procedure when replies have not 
confirmation replies have  been received to either first or 
occurred in the past. An­swer (b) second accounts receivable 
is correct because the  confirmation re­quests. Answer 
misstatements may have oc­ (d) is correct because the 
curred because respondents are  inspection of shipping records 
not readily able to confirm  will provide evidence that the 
account balances. AU 330  merchandise was actually shipped
suggests that in such circum­ to the debtor. Answer (a) is 
stances certain respondents’  incorrect because, a review of the
accounting systems may facili­ cash receipts journal prior to 
tate the confirmation of single  year­end is unlikely to provide 
transactions (individual in­ evidence on account recorded as 
voices) rather than of entire  unpaid as of year­end. Also, the 
account balances.  procedure would only de­tect one
specific type of misstatement, 
64. (c) The requirement is to identify  that in which pay­ments were 
the assertion most directly  recorded in the cash receipts 
addressed by accounts receivable  journal, but not credited to the 
confirmations. SAS 106 presents  customers’ accounts. Answer (b) 
information on financial statement  is incorrect because the lack of a 
assertions. Answer (c) is correct  reply to the confirmation provides
because a confirmation addresses  no particular evidence that the 
whether the entity replying to the  scope of procedures related to 
confirmation believes that a debt  internal control should be 
exists. Answer (a) is incorrect  modified. Answer (c) is incorrect 
because while confirmations  because the lack of a reply need 
provide limited information on  not necessarily lead to a 
valua­tion, they do not directly  presumption that the account is 
misstated. See AU 330 for 
address whether the entity replying 
procedures typically performed 
will pay the debt (or whether the 
for year­end accounts re­ceivable 
account has been factored). Answer
confirmation requests for which 
(b) is incorrect because limited  no reply is re­ceived. 
classification infor­mation is 
received via confirmations. Answer
(d) is incor­rect because 
confirmations are generally sent to 
recorded receivables, and are of 
limited assistance in the determina­
tion of whether all accounts are 
recorded (completeness). 
65. (a) The requirement is to determine when 
alterna­tive procedures should be 
performed in order to substantiate the 
existence of accounts receivable. Answer 
(a) is correct because the auditor should 
employ alternative procedures for 
nonresponses to positive confirmations to 
satisfy himself/herself as to the existence of
accounts receivable. Those procedures may
include examination of evidence of 
subsequent cash receipts, cash remittance 
advices, sales and shipping documents, and 
other records. Answer (b) is incor­rect 
because with negative confirmations the 
because verification of the 
67. (d) The requirement is to identify the  sources and contents through 
circumstances in which use of the negative  telephone calls will address 
form of accounts receivable confirmation  whether the information on the 
most likely would be justified. Negative  fax (which may have been sent 
con­firmations are used when (1) the  from almost anywhere) is correct.
combined assessed level of inherent and  Answer (a) is incor­rect because 
control risk is low, (2) a large number of  an examination of the shipping 
small balances is involved, and (3) the  documents is less complete than 
auditor has no reason to believe that the  is verification of the entire 
recipients of the requests are unlikely to  balance. An­swer (c) is incorrect 
give them consideration. Positive  because such faxes need not be 
confirmations are used when those  treated as nonresponses. Answer 
conditions are not met as well as in other  (d) is incorrect because inspection
circum­stances in which it seems desirable  of the faxes is unlikely to reveal 
to request a positive re­sponse, such as  forgeries or alterations, even 
when accounts are in dispute. Answer (d) is when such circumstances have 
best because small balances are involved  occurred. 
and few accounts are in dispute. Answer (a)
70. (d) The requirement is to 
is incorrect because it refers to a substantial
number of accounts in dispute and sales are identify the circumstance in 
to a few major customers. Answer (b) is  which the negative form of 
incorrect because it refers to a substantial  confirmation request most likely
number of accounts in dispute. An­swer (c) would be used. Answer (d) is 
is incorrect because it refers to sales to a  correct because AU 
few major customers.  330 states that negative confirmations 
may be used when (1) the 
68. (a) The requirement is to identify  combined assessed level of 
a method to re­duce the risk  inherent and control risk is low 
associated with accepting e­mail  [answer (d)], (2) a large number 
responses to accounts receivable  of small balances is involved, and
confirmation requests. Answer  (3) the auditor has no reason to 
(a) is correct because a response  believe that the recipients of the 
by mail will confirm the e­mail  requests are unlikely to give them
response. Answer (b) is incorrect  con­sideration. Answer (a) is 
because while such sub­sequent  incorrect because when the ac­
cash receipts will ordinarily be  counts receivable are immaterial, 
examined, this represents an  a decision may be made to send 
alternative, complementary  no confirmations. Answer (b) is 
approach to con­firmation.  incorrect because an inadequate 
Answer (c) is incorrect because  rate is not an acceptable reason to 
send negative confirmations. 
the auditor need not consider e­
Answer (c) is incorrect because 
mail responses to be 
negative confirmations are only 
confirmations with ex­ception. 
of value when the auditor has no 
Answer (d) is incorrect because a 
reason to believe that the 
second request is more likely to  recipients of the requests are 
elicit either no response or  unlikely to give them 
another e­mail response.  consideration. 
69. (b) The requirement is to  71. (a)   The   requirement   is   to
determine the most likely  identify   the   circumstance   in
procedure to reduce the risks  which an auditor would use the
associated with accepting fax  blank   form   of   confirma­tions
responses to requests for  (one which includes no amount
confirmations of accounts receiv­ and asks the respon­ 
able. Answer (b) is correct 
MODULE 4   RESPONDING TO RISK ASSESSMENT 145
confirmation of ac­counts 
receivable. Answer (a) is correct 
dent to supply the amount due) rather than  because including a list of items 
positive confir­mations. Answer (a) is  or invoices that constitute the 
correct because if a recipient simply signs a  account balance makes it easier 
blank confirmation and returns it the  for the potential respondent to 
confirmation will have no amount on it and  reply. An­swer (b) is incorrect 
the auditor will know that additional  because customers with relatively
procedures are necessary. Answers (b) and  large balances may or may not be
(c) are incorrect because there is no  more likely to reply. An­swers (c)
necessary relationship between the use of  and (d) are incorrect because 
blank confirmations and subsequent cash  there is no research available 
receipt verification difficulty and analytical  indicating that requesting a fax or
procedures results. Answer (d) is incorrect  e­mail reply, or threatening a 
because when the combined assessed level  second request is likely to 
of inherent risk and control risk is low it is  improve response rate. 
unlikely to lead to the blank form of 
confirmation. In fact, when that risk is low,  C.3.  Evidence—Inventory
and when adequate other substantive tests 
of details, no confirmation may be  75. (a) The requirement is to determine the
necessary. See AU 330 for information on  financialstatement most directly related to 
the confirmation process. the procedure of making inquiries 
concerning possible obsolete or slow­
72. (d) The requirement is to identify the most  moving in­ventory. Answer (a) is correct 
likely information that would be included in a  because inquiries concerning possible 
client’s confirmation letter that is being used  obsolete or slow­moving inventory deal 
to confirm accounts receivable bal­ances  with whether the inventory is being carried
rather than individual invoices. Answer (d) is  at the proper value
correct since including details of the account 
is likely to make it easier for the customer to 
respond in a meaningful manner. Answer (a) 
is incorrect because no such auditor­prepared 
letter will be included and because only in the 
case of the negative form of confirmation does
a nonresponse lead to an inference that the 
account balance is correct. Answer (b) is 
incorrect because confirmation requests do not
ordinarily include a letter suggesting that a 
second request will be sent. Answer (c) is 
incorrect because the auditor does not enclose 
a letter requesting that the information be 
supplied. See AU 
330 for   information   on   the   confirmation
process. 
73. (c) The requirement is to identify a 
statement that an auditor would be most 
likely to add to the negative form of 
confirmation of accounts receivable to 
encourage a timely consideration by the 
recipient. Answer (c) is correct because 
providing such information might increase 
timely consid­eration in that the recipient 
may realize the importance of a reply when 
the information is incorrect. Answers (a) 
and (b) are incorrect because while a 
confirmation request may in­clude these 
statements, the statements are unlikely to 
en­courage timely consideration of the 
request. Answer (d) is incorrect because 
many accounts that are not overdue are 
sampled, and because even for those 
overdue such a state­ment is not ordinarily 
included with the confirmation re­quest. 
74. (a) The requirement is to identify 
the strategy most likely to 
improve the response rate for 
the auditor’s recorded count sheets will 
and this is most directly related to the  provide assurance that the listed items 
valuation assertion. The other assertions  actually exist. An­swer (a) is incorrect 
are less directly related. Answer (b) is  because tracing from inventory tags to 
incorrect because the rights assertion deals  items in the inventory listing schedule tests 
with whether assets are the rights of the  the complete­ness of the inventory listing 
entity and liabilities are the obli­gations of  sheet, not whether all of the items it lists 
the entity at a given date. Answer (c) is  are valid. Answer (b) is incorrect because it
incorrect because the existence assertion  does not directly test the client’s inventory 
deals with whether assets exist at a given  listing schedule. Answer (d) is incorrect 
date. Answer (d) is incorrect because the  because tracing items listed in re­ceiving 
presentation assertion deals with whether  reports and vendors’ invoices to the 
particular compo­nents of the financial  inventory listing schedule will provide 
statements are properly classified. See SAS assurance on the completeness of the 
106 for information on the financial  inventory listing sheet; it will also be a 
statement asser­tions. difficult procedure to accomplish due to the
76. (d) The requirement is to identify  fact that a number of these items will not be
the type of omit­ted journal entry  in inventory due to sales. See SAS 106 for 
that would result in inventory test  informa­tion on the testing of the various 
counts that are higher than the  financial statement asser­tions. 
recorded quantities in the client’s  78. (b) The requirement is to identify 
perpetual records. Answer (d) is  the factor that average inventory 
correct because a failure to record is divided into to calculate 
sales returns results in a situation  inventory turn­over. Answer (b) is
in which the item is returned by a  correct because the average 
customer and included in the  inventory is divided into the cost 
inventory count, but not recorded  of goods sold to calculate the 
in the perpetual records;  inventory turnover. 
accordingly the test counts are 
higher than the recorded  79. (a) The requirement is to identify the 
quantities. An­swer (a) is  auditing pro­cedure that most likely would 
incorrect because purchase  provide assurance about a manufacturing 
discounts do not af­fect quantities entity’s inventory valuation. Answer (a) is 
in inventory. Answers (b) and (c)  correct because testing the overhead 
are incor­rect because a failure to  computation will pro­vide evidence on 
record purchase returns or sales  whether inventory has been included in the 
result in a situation in which less  financial statements at the appropriate 
inventory will be counted (since  amount. Answer (b) is incorrect because 
the items are no longer physically obtaining confirmation of inventories 
in inventory) than is recorded on  pledged under loan agreements relates more
the perpetual records.  directly to the presentation assertion. 
Answers (c) and (d) are incorrect because 
77. (c) The requirement is to identify a  reviewing shipping and receiving cutoff 
procedure that will provide assurance that  procedures for inventories and tracing test 
all inventory items in a client’s inventory  counts to the inventory listing relate more 
listing schedule are valid. Answer (c) is  directly to the existence and completeness 
correct because tracing from the inventory  asser­tions. 
listing schedule to in­ventory tags and to 
146 MODULE 4   RESPONDING TO RISK ASSESSMENT
rights of the entity and this is not being 
tested when an auditor traces test counts to 
80. (b) The requirement is to identify  an inventory listing. An­swer (c) is 
an action that an auditor might take incorrect because existence deals with 
when the assessed level of control  whether the inventory existed at the date of 
risk is high for inventory. Answer  the count. To test exis­tence the auditor 
(b) is correct because a high level  would sample from the inventory listing 
of control risk will generally result  and compare quantities to the test counts. 
in a low acceptable level of  Answer (d) is incorrect because valuation 
detection risk, which may be  deals with whether the inventory is properly
achieved by changing the timing of included in the balance sheet at the 
substantive tests to year­end,  appropriate dollar amount and this is not 
changing the nature of substantive  being tested here. See SAS 
tests to more effective procedures,  106 for   more   information   on
and/or by changing the extent of  management’s   financial   state­
substantive tests. An­swer (a) is  ment assertions. 
incorrect because control risk has  83. (d) The requirement is to determine the 
been assessed and tests of controls,  assertion most directly related to an 
if any, will already have been com­ auditor’s analysis of inventory turnover 
pleted. Answer (c) is incorrect  rates. Answer (d) is correct because an 
because a year­end count of  analysis of inventory turnover rates will 
inventory is more appropriate when provide the auditor with evi­dence on slow­
control risk is high. Answer (d) is  moving, excess, defective, and obsolete 
incorrect because gross profit tests  items included in inventories. These items 
will gener­ally provide less  may be improperly valued. 
assurance than is required in 
circumstances such as this when  84. (a)   The   requirement   is   to
control risk is assessed at a high  determine which types of entries
level.  will   be   supported   when   the
auditor   examines   receiv­ing
81. (a) The requirement is to identify the  reports.   Answer   (a)   is   correct
financial state­ment assertion (other than  because receiving reports will be
presentation and disclosure) most directly  prepared   when   goods   are
related to an auditor’s conclusion that no  received   through   purchase   (as
excessive costs for idle plant were charged  recorded in the voucher register)
to inventory. Answer (a) is correct because  and when goods are 
the assertion deals with whether the inven­
tory has been included in the financial 
statements at the ap­propriate amount, and 
therefore that no excessive costs were 
charged to inventory. Answer (b) is 
incorrect because the completeness 
assertion deals with whether all inventory 
items that should be presented are so 
included. Answer (c) is incorrect because 
existence deals with whether the in­ventory
actually exits at the given date. Answer (d) 
is incor­rect because rights deal with 
whether the inventory is owned by the 
client. For more information on 
management’s fi­nancial statement 
assertions, see SAS 106. 
82. (b) The requirement is to identify the 
financial state­ment assertion most directly 
related to an auditor’s tracing of inventory 
test counts to the client’s inventory listing. 
Answer (b) is correct because the 
completeness assertion deals with whether 
all transactions are included. Tracing from 
the inventory items observed to the 
inventory listing will help determine 
whether all the transactions are included 
and the inventory listing is complete. 
Answer (a) is incor­rect because the rights 
assertion deals with whether assets are the 
for any subsequent mis­
received through sales returns (as  placement or misappropriation of
recorded in the sales re­turns journal).  the securities. Answer (a) is 
Answers (b), (c), and (d) are incorrect be­ incorrect because the client’s 
cause entries in sales journals result in  representative will not in general 
items being shipped, not received. Note,  help the CPA to detect fraudulent
however, that answers (b), (c), and (d) are  securities. An­swers (b) and (d) 
partially correct because the sales returns  are incorrect because while the 
journal, voucher register, and check  client’s representative will help 
register all result from transac­tions  the CPA to gain access to the 
related to the receipt of goods. secu­rities and may coordinate 
their return, these are not the 
85. (c) The requirement is to identify the 
auditor’s primary purpose. 
responsewhich does not represent one of 
the independent auditor’s objectives  87. (c) The requirement is to identify the best 
regarding the examination of inventory.  procedure other than inspection to establish
An­swer (c) is correct because verifying  the existence and owner­ship of a long­term
that all inventory owned by the client is on  investment in a publicly traded stock. 
hand at the time of the count is not an  Answer (c) is correct because confirmation 
objective. For example, purchased items in  of the number of shares owned that are held
transit at year­end, for which title has  by an independent custodian is effective at 
passed, should be included in inventory.  testing existence. Answer (a) is incorrect 
Similarly, inventory out on consignment  be­cause auditors do not in general 
should also be included in inventory.  correspond with the investee company and 
Answer (a) is incorrect be­cause proper  because that company may or may not have
presentation of inventory pertains to the  detailed information on the identity of 
pre­sentation and disclosure assertion and  shareholders at any point in time. Answer 
therefore would be subject to auditor  (b) is incorrect because while inspec­tion of
verification. Answers (b) and (d) are in­ the audited financial statements of the 
correct because proper accounting for  investee com­pany may provide limited 
damaged and obsolete items and proper  information on valuation of the investment,
inventory pricing pertain to the valuation  it does not directly address existence; note 
assertion and therefore would be subject to  that this procedure is of limited use here 
auditor verifica­tion. See SAS 106 for  since the stock is pub­licly traded and 
details on financial statement asser­tions. obtaining its value through stock price quo­
tations should not be difficult. Answer (d) 
C.4.  Evidence—Investment Securities
is incorrect because this procedure 
86. (c) The requirement is to  addresses the valuation of the securi­ties. In
determine why an auditor should  addition, under SFAS 115, investments are 
insist that a client representative  no longer carried at the lower of cost or 
be present when he or she  market. See SAS 106 for more information 
physically examines securities.  on management’s financial statement 
Answer (c) is correct because  assertions. 
requiring that a client  88. (c) The requirement is to determine
representative acknowledge the  the   best   proce­dure   when   an
receipt of the securities will 
auditor has been unable to inspect
eliminate any question con­
and count a 
cerning the CPA’s responsibility 
MODULE 4   RESPONDING TO RISK ASSESSMENT 147
accumulated depreciation expense. Answer 
(b) is correct because debits to accumulated
client’s investment securities (held in a  depreciation are properly recorded upon 
safe­deposit box) until after the balance  retirement of a plant asset. An­swer (a) is 
sheet date. Answer (c) is correct be­cause  incorrect because changing the useful lives 
banks maintain records on access to safe­ of plant assets does not affect accumulated 
deposit boxes. Thus, the confirmation of no depreciation. An­swer (c) is incorrect 
access during the pe­riod will provide the  because understatement of the prior year’s 
auditor with evidence that the securi­ties in  depreciation expense does not result in an 
the safe­deposit box at the time of the  adjustment to accumulated depreciation. 
count were those available at year­end.  Answer (d) is incorrect be­cause overhead 
Answers (a) and (b) are incor­rect because  allocations do not ordinarily affect accumu­
the bank will not generally be able to  lated depreciation. 
provide a list of securities added and 
removed from the box (typically, only  92. (a) The requirement is to identify the best 
records on access are maintained by the  procedure for testing unrecorded 
bank). There­fore, the bank will have no  retirements of equipment. Answer (a) is 
information on reconciling items between  correct because selecting items from the 
the subsidiary ledger and the securities on  accounting rec­ords and attempting to 
hand. Answer (d) is incorrect because it is  locate them will reveal unrecorded 
the responsibility of the auditor and the  retirements when the item cannot be 
client, not the bank, to count the securities  located. Answer (b) is incorrect because 
maintained in a safe­deposit box. depreciation entries will continue when 
retirements have not been recorded. Answer
89. (a) The requirement is to determine the 
(c) is incorrect 
most likelyuse of analytical procedures 
when testing long­term invest­ments. 
Answer (a) is correct because the 
predictable rela­tionship between long­term
investments and investment income creates
a situation in which analytical procedures 
may provide substantial audit assurance. 
Answer (b) is in­correct because the 
classification between current and non­
current portfolios may be expected to 
fluctuate in an unpre­dictable manner as 
investment goals and the environment 
change. Answers (c) and (d) are incorrect 
because the valuation of marketable equity 
securities at the lower of cost or market and
unrealized gains or losses do not result in a 
predictable relationship on which analytical
procedures may provide effective results.
C.5.  Evidence—Property, Plant, and 
Equipment
90. (d) The requirement is to identify the 
account whose analysis is least likely to 
reveal evidence relating to re­corded 
retirements of equipment. Answer (d) is 
correctbecause the purchase returns and 
allowances account deals with returns and 
allowances for purchases of merchandise, 
not equipment. Answer (a) is incorrect 
because analysis of accumulated 
depreciation will reveal the retirement 
through charges made to the accumulated 
depreciation account. Answer (b) is 
incorrect because companies will ordinarily
modify insurance coverage when assets are 
retired. An­swer (c) is incorrect because the
property, plant, and equip­ment account 
will reflect the retirement. 
91. (b) The requirement is to identify a likely 
explana­tion for a situation in which 
significant debits have been posted to the 
detailed knowledge as to the ade­
because the direction of the test is  quacy of the provision for 
incorrect since beginning with the item is  uncollectible accounts or the 
unlikely to reveal a situation in which an  amount of insurance which is 
unrecorded retirement has occurred.  desirable. Answer (b) is in­
Answer (d) is incorrect because scanning  correct because the plant manager
the general journal for recorded entries is  will have limited knowl­edge 
unlikely to reveal unrecorded retirements  concerning physical inventory 
of equipment. observation procedures and their 
appropriateness. 
93. (d) The requirement is to 
determine why an auditor  95. (a) The requirement is to identify 
analyzes repairs and maintenance the document least likely to 
accounts. Answer (d) is correct  provide evidence regarding 
because clients often erroneously mortgage acquisi­tion costs. 
charge expendi­tures for property Deeds generally consist of a legal 
and equipment acquisitions as  conveyance of rights to use real 
expenses rather than capitalize  property. Frequently the sales 
them as assets. An analysis of  price is not even specified and the
repairs and maintenance accounts related mortgage acquisition costs
will reveal such errors. An­swer  are much less likely to be stated in
(a) is incorrect because while  a deed. Answer (b) is incor­rect 
auditors will want to determine  because cancelled checks would 
that noncapitalizable expenses  provide verification of mortgage 
for repairs and maintenance have  acquisition costs. Answer (c) is 
been recorded in the proper  incorrect because the closing 
period, ana­lyzing only the  statement would provide a 
recorded entries is an incomplete  detailed listing of the costs of 
test since entries occurring after  acquiring the real property, 
year­end will also need to be  including possible mortgage 
exam­ined. Answer (b) is  acquisition costs. Answer (d) is 
incorrect because procedures  incorrect because examination of 
relating to the property and  interest expense would also relate 
equipment account will be  to the mortgage acquisition costs. 
performed to determine whether  96. (b) The requirement is to determine 
such entries have been recorded  the assertion(s) involved when an 
in the  auditor is inspecting new additions 
proper period. Answer (c) is incorrect  on a list of property, plant, and 
because analyzing the repairs and 
equipment. Answer (b) is correct 
maintenance accounts only considers 
because an auditor who inspects 
recorded entries and not whether all 
noncapitalizable expenditures for repairs  new additions relating to property 
and maintenance have been properly  and equipment balances addresses 
charged to ex­pense.  existence or occurrence, but not 
presentation and disclosure; 
94. (c) The requirement is to  presentation and disclosure relates 
determine the information an  more directly to proper 
auditor is most likely to seek  classification and note disclosures 
from the plant manager. The  rather than account balances. 
plant manager comes into day­to­
day contact with the machinery  C.6.  Evidence—Prepaid Assets
when producing a product; that 
97. (a) The requirement is to determine the 
contact is likely to provide 
information on its condition and  financialstatement assertion most directly 
usefulness. An­swers (a) and (d)  related to the procedure of reviewing or 
are incorrect because the plant  recomputing amortization of intangible 
manager will generally not have  assets.
148 MODULE 4   RESPONDING TO RISK ASSESSMENT
the appropriate population when 
using accounts payable 
Answer (a) is correct because the  confirmations directed towards 
amortization of intangible assets deals with  obtaining evidence on the 
whether the accounts are properly valued,  completeness as­sertion. Answer (a)
the valuation assertion. The other assertions is correct because to address 
are less directly related. Answer (b) is  complete­ness the auditor attempts 
incorrect because the existence or  to determine that all accounts pay­
occurrence assertion deals with whether  able are reflected, and a company 
assets or liabilities exist at a given date and  potentially may be liable to any of 
whether recorded transactions have 
its vendors. Answer (b) is incorrect 
occurred during a given period. Answer (c) 
because con­firming based on 
is incorrect be­cause the completeness 
recorded amounts addresses 
assertion deals with whether all 
transactions and accounts that should be  existence more directly than 
presented in the financial statements are so  completeness. Answer (c) is 
included. Answer (d) is incorrect because  incorrect because basing the sample
the rights and obligations assertion deals  on payees after year­end only deals 
with whether assets are the rights of the  with those payables that have been 
entity and liabilities are the obligations of  paid as of that point. Answer (d) is 
the entity at a given date. See SAS 106 for  incorrect because open invoices are 
information on the financial statement  a less complete population than are 
assertions. vendors. See AU 330 for 
information on the confirmation 
98. (c) The requirement is to determine the  process, and SAS 106 for 
most likelyreason for the absence of the  information on management’s 
original insurance policy on plant  financial statement assertions. 
equipment. Answer (c) is correct because 
the holder of the lien may also in certain  101. (d) The requirement is to 
circumstances maintain the original  determine why confirma­tion of 
insurance policy. Answer (a) is incorrect  accounts payable is unnecessary. 
because an insurance premium which is  Accounts payable are usually not 
due but not recorded is unlikely to account  confirmed because there is better 
for the lack of the original insurance  evidence available to the auditor, 
policy. Answer (b) is incorrect because  (i.e., examination of cash 
while coinsurance provisions are outlined  payments subsequent to the 
in the policy, they are unlikely to be a  balance sheet date). If the auditor 
reason that the policy is not available for  reviews 
inspection. An­swer (d) is incorrect 
because there is no obvious relationship 
between the understatement of insurance 
expense and the presence or absence of an 
insurance policy.
C.7.  Evidence—Payables (Current)
99. (c) The requirement is to identify the best 
audit pro­cedures for identifying 
unrecorded liabilities. Answer (c) is correct 
because unrecorded liabilities eventually 
become due and must be paid. Accordingly,
a review of cash disburse­ments after the 
balance sheet date is an effective procedure 
for detecting unrecorded payables. Answer 
(a) is incorrect because tracing a sample of 
accounts payable that have been recorded is
not likely to result in identification of 
unrecorded liabilities. Answer (b) is 
incorrect because purchase orders issued 
after year­end will not result in liabilities as 
of year­end. Answer (d) is incorrect 
because disbursement entries recorded 
before year­end generally relate to accounts
pay­able that have been paid before year­
end. 
100. (a) The requirement is to identify 
Answer (a) is incorrect because the existence 
all cash payments for a sufficient time  as­sertion deals with whether recorded 
after the balance sheet date for items  accounts payable are overstated, thereby 
pertaining to the period under audit and  understating income. Answer (b) is in­correct 
finds no such payments which were not  because payables often require no particularly
recorded as liabili­ties at year­end, the  trou­blesome presentations and disclosures. 
auditor is reasonably assured that ac­ Answer (d) is incor­rect because payables are 
counts payable were not understated.  most frequently simply valued at the cost of 
Answer (a) is a non­sense answer. Answer the related acquisition. See SAS 106 for more 
(b) is incorrect because AP balances could information on management’s financial 
be paid during year­end audit work after 
statement assertions. 
the balance sheet date. Answer (c) is 
incorrect because whether or not legal  C.8.  Evidence—Long­Term Debt
action has been taken against the client is 
irrelevant to the confirmation procedure. 104. (b) The requirement is to identify 
a likely reason for a recorded 
102. (c) The requirement is to identify  interest expense that seems 
the substantive test to be  excessive in relation to the 
performed to verify the existence  balance in the bonds payable 
and valuation of recorded  account. Answer (b) is correct 
accounts payable. Answer (c) is  because understated bonds 
correct because the vouching of  payable will result in a lower 
various payable accounts to  account balance than is proper 
purchase orders and receiving  and thereby create a situation in 
reports will provide evidence that  which the interest expense 
the debt was incurred and the  appears excessive. Answers (a) 
related goods received, thereby  and (d) are incorrect because an 
pro­viding evidence on the  understated discount or an 
existence of the debt and its  overstated premium on bonds 
amount, or valuation. Answer (a)  payable result in situations in 
is incorrect because determining  which the recorded interest 
whether prenumbered purchase  expense seems lower than 
orders are used and ac­counted  expected since the net bonds 
for relates more directly to the  payable are over­stated. Answer 
completeness with which  (c) is incorrect because 
purchases and accounts payable  understatements, not 
were recorded. An­swers (b) and  overstatements, of bonds payable 
(d) are incorrect because the  will result in what appears to be 
question ad­dresses the existence  an excessive rate of interest 
and valuation of recorded  expense. 
accounts payable, not unrecorded 
payables or payables with a zero  105. (a) The requirement is to determine the 
balance.  financial statement assertion most directly 
related to an auditor’s inspection of loan 
103. (c) The requirement is to determine  agreements under which an entity’s in­
management’s accounts payable assertion that ventories are pledged. Answer (a) is correct 
an auditor will primarily focus on. Experience because the presentation and disclosure 
has indicated that overstated income is more  assertion deals with whether par­ticular 
of a risk than is understated income. Answer  components of the financial statements—such
(c) is correct because the completeness  as loan agreement covenants—are properly 
assertion focuses upon whether payables have classified, described, and disclosed. The other
been omitted, thereby overstating income.  assertions are less directly related. 
MODULE 4   RESPONDING TO RISK ASSESSMENT 149
answer (c) in that it is desirable 
but does not address the actual 
Answer (b) is incorrect because the  bond sinking fund transactions 
valuation or allocation assertion deals with  and year­end balance. 
whether asset, liabilities, revenue, and 
expense components have been included in  C.9.  Evidence—Owners’ Equity
the financial statements at the appropriate 
108. (a) The requirement is to 
amounts. Answer (c) is incor­rect because 
the existence or occurrence assertion deals  determine how an auditor 
with whether assets or liabilities exist at a  ordinarily obtains evidence of 
given date and whether recorded  stockholders’ equity transac­
transactions have occurred during a given  tions. Answer (a) is correct 
period. Answer (d) is incorrect because the  because the board of directors 
completeness assertion deals with whether  will, in general, authorize 
all transactions and accounts that should be changes in stockholders’ equity. 
presented in the financial statements are so  Answer (b) is less complete in 
included. See SAS 106 for information on  that for small clients there may 
the financial statement asser­tions. be no transfer agent, and because
the transfer agent deals most 
106. (c) The requirement is to identify a  directly with transfers of 
procedure an auditor would perform in  outstanding stock. An­swer (c) is
auditing long­term bonds payable. Answer  incorrect because canceled stock 
(c) is correct because comparing interest  certificates are ordinarily 
expense with the bond payable amount will  available only for small clients. 
provide evidence as to reasonableness. Such Answer (d) is incorrect because 
a procedure may reveal either interest not  companies do not ordinarily 
expensed or debt not properly recorded.  have a “trea­sury stock 
Answer (a) is incorrect because analytical  certificate book.” 
procedures will not in general be performed
on bond premiums and discounts since  109. (d) The requirement is to identify
these accounts may easily be verified by  the most likely audit procedure, 
examining details of the entry recording the  in addition to analytical 
debt issuance and any subsequent amor­ procedures, when control risk for
tization. Answer (b) is incorrect because an  payroll is assessed as low. 
examination of the documentation of assets  Answer (d) is correct because 
purchased with bond proceeds is only  accrual of payroll at year­end is 
necessary when such a use of the funds is a  not an entry made frequently 
requirement of the debt issuance. Answer  throughout the year and 
(d) is incorrect because con­firmation of  accordingly re­cording of the 
bonds outstanding will often be with the  entry is often not controlled by 
trustee rather than with individual  the payroll 
bondholders. 
107. (b) The requirement is to 
determine how an auditor can best
verify a client’s bond sinking fund
transactions and year­end balance.
Answer (b) is correct because 
confirma­tion with the bond 
trustee represents externally 
generated evidence received 
directly by the auditor. Such 
evidence is considered very 
reliable. Answer (a) is incorrect 
because individual holders of 
retired bonds will have no 
information on actual bond 
sinking fund transactions or year­
end bal­ances. Answer (c) is 
incorrect because, while 
recomputing interest expense, 
interest payable, and amortization 
of bond discount or premiums are 
desirable procedures, they do not 
directly address bond sinking fund
transactions and year­end 
balances. Answer (d) is similar to 
112. (d) The requirement is to identify 
portion   of   the   internal   control   structure. the procedure that is most likely 
Answers   (a),   (b),   and   (c)   are   incorrect when an auditor is performing tests 
because   observing   the   distribution   of concern­ing the granting of stock 
paychecks, the footing and crossfooting of options. Answer (d) is correct be­
the   payroll   reg­ister,   and   inspection   of cause authorizing the issuance of 
payroll   tax   returns   are   recurring   op­ stock options is ordinarily a 
erations   that   will   have   been   considered decision made by the board of 
when assessing con­trol risk at a low level.
directors. Answer (a) is incorrect 
110. (a) The requirement is to identify  because the Secretary of State of 
the circumstance that most likely  the state of incor­poration will not 
would cause an auditor to suspect  have this information on stock 
an em­ployee payroll fraud  options. Answer (b) is incorrect 
scheme. Answer (a) is correct  because the existence of the option 
because significant unexplained  holders is not ordinarily a 
variances between standard and  significant question. Answer (c) is 
actual labor cost may lead an  incorrect because stock to be issued
auditor to suspect fraud. An­swer  relating to options may be either 
(b) is incorrect because one would from treasury stock or new 
expect payroll checks to be  issuances; ac­cordingly, sufficient 
distributed by the same employees treasury stock need not be 
each pay­day. Answer (c) is  available. 
incorrect because time cards are 
ordinar­ily approved by individual 113. (d) The requirement is to identify 
departmental supervisors. An­ the assertion to which determining 
swer (d) is incorrect because the  whether there are restrictions on 
maintenance of a separate payroll  retained earnings relates most 
bank account is considered a  directly. Answer (d) is correct be­
control, not an indica­tion of  cause such restrictions will result in
fraud.  disclosures and thus the 
presentation and disclosure 
111. (d) The requirement is to identify the 
assertion is most directly being 
procedure that an auditor most likely would
verified. Answer (a) is incorrect 
perform when auditing payroll. Answer (d) 
because the existence or occurrence
is correct because a comparison of payroll 
assertion addresses whether assets 
costs with entity standards or budgets will 
or liabilities of the entity exist at a 
generally be included in the audit program 
as a test of overall payroll reasonable­ness.  given date and whether recorded 
Answer (a) is incorrect because unclaimed  transactions have occurred during a 
wages will not be mailed unless an  given period. An­ 
employee so requests and this often will not swer (b) is incorrect because the 
be tested by an auditor. Answer (b) is  completeness assertion addresses whether 
incorrect because total employee deductions all transactions and accounts that should 
will be traced to journal entries. Answer (c)  be presented in the financial statements 
is incorrect because observing entity  are so included. Answer (c) is incorrect 
employees during a payroll distribution is  because the valuation or allocation 
generally only included in an audit program assertion addresses whether asset, 
when internal control is weak; accordingly,  liability, revenue, and expense 
it is more likely that a comparison of  components have been included in the 
financial statements at appropriate 
payroll costs with entity standards or 
amounts. See SAS 106 for a dis­cussion 
budgets [answer (d)] will be included. 
of financial statement assertions. 
150 MODULE 4   RESPONDING TO RISK ASSESSMENT
less substan­tive testing. Answer 
(d) is incorrect because the nature 
114. (b) The requirement is to identify  of accrued payroll expense being 
the information an auditor should  unpaid commissions need not 
confirm with a client’s transfer  necessarily result in more 
agent and registrar. Answer (b) is  substantive testing. 
correct because when a client em­
ploys a transfer agent and  117. (a) The requirement is to determine a 
registrar, there will be no stock  source an auditor uses to test the 
certificate book to examine, and  reasonableness of dividend income from 
accordingly, information on  investments in publicly held companies. 
shares issued and outstanding  Answer (a) is correct because dividend 
should be confirmed. An­swers  record books produced by invest­ment 
(a), (c), and (d) are incorrect  advisory services provide summaries of 
because the transfer agent and  dividends paid for various securities, and an
registrar often will not have  auditor is able to compare the 
information on divi­dend  reasonableness of a client’s recorded 
restrictions, guarantees of  dividend income from investments with this
preferred stock liquidation values, information. Answers (b) and (c) are 
and the number of shares subject  incorrect because auditors do not, in 
to agreements to repurchase.  general, determine the reasonableness of 
dividend income by examining stock “in­
115. (c) The requirement is to  dentures” or “stock ledgers.” Answer (d) is 
determine a likely step in the  incorrect be­cause while annual financial 
audit program for retained  statements of investee compa­nies may 
earnings. The legality of a  include such information, examining such 
dividend depends in part on  financial statements is not generally an 
whether it has been properly  efficient approach for testing the 
authorized (state laws differ on  reasonableness of dividend income. Also, 
specific requirements). Thus, the  the current year financial statements of the 
auditor must determine that  investees often are not avail­able when the 
proper authorization exists, as  auditor is performing the current audit. 
both cash and stock dividends 
affect retained earnings. Answer  C.10.  Revenue
(a) is incorrect since only a memo 118. (c) The requirement is to identify the 
entry is required for a stock split.  most likelyrisk involved with a bill and 
Answer (b) is incorrect because  hold transaction at year­end. Answer (c) is 
the write­down of an account  correct because a bill and hold transaction 
receivable will not, in general, be  results in the recording of a sale prior to 
recorded in retained earnings.  delivery of the goods—accordingly, sales 
Answer (d) is incorrect because  may be inappropriately recorded.
gains from the disposition of 
treasury shares are recorded in 
paid­in capital accounts. 
116. (c) The requirement is to 
determine when an auditor would 
be most likely to perform 
substantive tests of details on 
payroll transactions and balances. 
Answer (c) is correct because 
analytical procedures result in 
further investigation when 
unexpected differences occur. 
This investigation will generally 
involve substantive tests of details
of transactions and balances. AU 
329 provides detailed information
on analytical procedures. Answer 
(a) is incorrect because a 
substantial amount of accrued 
payroll expense as indicated by a 
cutoff test will not necessarily 
result in additional sub­stantive 
tests. Answer (b) is incorrect 
because a low as­sessed level of 
control risk is likely to result in 
equipment.
Answer (a) is incorrect because accrued
liabilities are not ordinarily affected by  C.12.  Client Representation Letters
bill and hold transactions. An­ 121. (c) The requirement is to identify
swers (b) and (d) are incorrect because an  the matter on which an auditor 
absolute purchase commitment and the  should obtain written 
assuming of risk and reward relating to the
management repre­sentations. 
product represent conditions which 
Answer (c) is correct because 
increase the likeli­hood that recording of a 
written represen­tations are 
sale for such a transaction is appro­priate.
ordinarily obtained on 
119. (b) The requirement is to identify the  noncompliance with as­pects of 
most likelylisted effect of “channel  contractual agreements that may 
stuffing.” Answer (b) is correct because  affect the financial statements. 
channel stuffing is a marketing practice that Answer (a) is incorrect because 
suppli­ers sometimes use to boost sales by  auditors do not ordinarily obtain 
inducing distributors to buy substantially  a cost­benefit justification from 
more inventory than they can promptly  manage­ment related to internal 
resell; accordingly, increased sales returns  control weaknesses. Answer (b) 
in the future are likely. Answers (a), (c) and is incorrect because written 
(d) are incorrect because ac­crued  representations are not ordinarily 
liabilities, cash, and marketable  obtained on such future plans. 
investments are less likely to be affected by Answer (d) is incorrect be­cause 
channel stuffing, which results in entries  management may or may not be 
increasing accounts receivable, cost of  responsible for em­ployee 
good sold, and sales, while decreasing  violations of laws, and because 
inventory. such a representation is not 
ordinarily obtained. See AU 333 
C.11.  Expenses for information on client 
representations. 
120. (d) The requirement is to identify the 
account inwhich a recorded entry is most  122. (a) The requirement is to determine
likely to relate to the prop­erty, plant, and  a matter to which materiality limits
equipment completeness assertion. The  do not apply in obtaining written 
completeness assertion addresses whether  management representations. 
all transactions have been recorded in an  Answer (a) is correct because 
account (here, property, plant, and  materiality considerations do not 
equipment). Answer (d) is correct because  apply to management’s 
the purchase of property, plant, and  acknowledgment of its 
equipment may inappropriately have been 
responsibility for fair presentation 
recorded in the repairs and maintenance 
of financial statements, the 
account rather than in property, plant, and 
availability of all financial records,
equipment; this is a frequent bookkeeping 
the completeness and availability 
error since the individual recording the 
entry may frequently see similar invoices  of all minutes and meet­ings of 
which do represent re­pairs and  stockholders, directors, and 
maintenance expense. Answers (a), (b),  committees of directors, and 
and (c) are all incorrect because the  communication from regulatory 
allowance for doubtful ac­counts,  agencies. Answers (b), (c), and (d) 
marketable securities, and sales has no  are all incorrect because 
apparent relationship to the completeness  materiality considera­tions relate to
of recording of property, plant, and  losses from purchase 
commitments, compen­ 
MODULE 4   RESPONDING TO RISK ASSESSMENT 151
meant to complement, but not 
replace, sub­stantive tests. 
sating balances, and obsolete inventory.  Answer (b) is incorrect because 
AU 333 discusses client representations. the comple­ 
mentary nature of such representations is not 
123. (c) The requirement is to 
considered sufficient, even when combined 
determine the proper date for a 
with reliance upon internal control. The 
client’s representation letter. AU 
333 states that the representation  inherent limitations of internal control do not 
letter should be dated as of the  permit the auditor to replace substantive tests 
date of the auditor’s report.  with complete reliance on internal control. 
Answer (c) is incorrect because the written 
124. (b) The requirement is to identify the  representations are considered complementary
matter that an auditor most likely would  evidence in support of various assertions. 
include in a management repre­sentation  Answer (d) is incorrect because such written 
letter. Auditors will generally request  representations are not con­sidered to be 
assurance as to the completeness and  replacements for reliance upon internal 
availability of minutes of stock­holders’  control.
and directors’ meetings. See AU 333 for 
written representations ordinarily obtained  128. (a) The requirement is to 
by the auditor.  determine who should sign a 
letter of representation. AU 333 
125. (d) The requirement is to  states that, normally, the chief 
determine the year(s) on which a  executive officer and the chief 
CPA must obtain written  financial officers should sign the 
representations from man­ letter of representation. 
agement, when comparative 
financial statements are being  129. (d) The requirement is to identify 
issued, but current management  the scope limita­tion which in all 
has only been employed for a  cases is sufficient to preclude an 
portion of one of those years. AU  unquali­fied opinion. Answer (d) 
333 states that if current  is correct because the professional
management was not present  standards state that management 
during all periods reported upon,  refusal to furnish written 
the auditor should nevertheless  representations constitutes a 
obtain written repre­sentations  limitation on the scope of the 
from current management on all  auditor’s examination sufficient 
such periods.  to preclude an unqualified 
126. (a) The requirement is to identify 
opinion. Answer (a) is incorrect 
because management’s refusal to 
the information ordinarily included
allow the auditor to review the 
among the written client 
predecessor’s work may not 
representations obtained by the  necessarily result in report 
auditor. Answer (a) is correct  modification. An­ 
because 
AU 333 includes information on 
compensating balances in the list of 
representations normally obtained. Answer
(b) is incorrect because management need 
not acknowledge a re­sponsibility for 
employee illegal actions. Answer (c) is in­
correct because the auditor, not the client, 
determines whether sufficient audit 
evidence has been made available. Answer
(d) is incorrect because, for purposes of a 
financial statement audit, management 
need not attempt to determine whether 
material weaknesses in internal control 
exist. 
127. (a) The requirement is to 
determine the correct statement 
with respect to the use of a 
management repre­sentation 
letter as audit evidence about the 
completeness assertion. Answer 
(a) is correct because such 
written repre­sentations are 
correct state­ment concerning an auditor’s 
swer (b) is incorrect because alternate  use of the work of a specialist. Answer (a) 
procedures may be available that will make is correct because the work of a specialist 
report modification unnecessary when the  who is related to the client may be 
auditor has been engaged after completion  acceptable under certain circum­stances. 
of the year­end physical count. Answer (c)  Answer (b) is incorrect because if the 
is incorrect because management may  auditor be­lieves that the findings of the 
choose not to correct a significant  specialist are unreasonable, it is generally 
deficiencies in internal control without a  appropriate to obtain the findings of 
resulting limitation on the scope of the  another specialist. Answer (c) is incorrect 
audit. because a material differ­ence between a 
specialist’s findings and those included in 
130. (c) The requirement is to identify a purpose
the financial statements may result in the 
of a management representation letter. 
need for an ex­planatory paragraph, a 
Answer (c) is correct because a 
qualified opinion, a disclaimer, or an 
management representation letter is meant 
adverse opinion. Answer (d) is incorrect 
to reduce the possibility of a 
because an auditor may use a specialist in 
misunderstanding concerning 
the determination of various physical 
management’s responsibility for the 
characteristics of assets. 
financial statements. Answer (a) is 
incorrect because reducing audit risk to an  133. (a) The requirement is to identify 
aggregate level of misstatement that could  a circumstance in which an 
be considered material is not a logically  auditor may refer to the findings 
sound statement. Answer (b) is incorrect  of a specialist in the auditor’s 
because the management representation  report. Answer (a) is correct 
letter does not modify an auditor’s  because the auditor may refer to 
responsibility to detect material mis­ the specialist when the 
statements. Answer (d) is incorrect because specialist’s findings result in 
management representation letters are not a inclusion of an explanatory 
substitute for other proce­dures.  paragraph to an audit report, in 
this case on going concern status. 
131. (c) The requirement is to identify
An­swers (b), (c), and (d) are all 
the most likely source of a 
incorrect because a specialist is 
statement suggesting that there 
only referred to in an audit report 
have been no communications 
when that specialist’s findings 
from regulatory agencies. 
identify a circumstance requiring 
Answer (c) is correct because 
modification of the audit report. 
information such as this is 
Auditors do not modify audit 
ordinarily in­cluded in a 
reports to simply inform the user 
management representation 
that a specialist was involved. 
letter. Answers (a), (b), and (d) 
are incorrect because such a  134. (d) The requirement is to identify
disclosure is not ordinarily  the statement that is correct about
included in a report on internal  the auditor’s use of the work of a 
control, a special report, or a  specialist. Answer (d) is correct 
letter for an underwriter. See AU because the auditor should obtain 
333 for guid­ance on  an understanding of the nature of 
representation letters.  the work performed by the 
specialist. Answer (a) is incorrect
C.13.  Using the Work of a Specialist
because ordinarily a spe­cialist 
132. (a) The requirement is to identify the  will have a basic understanding 
of the auditor’s cor­ 
152 MODULE 4   RESPONDING TO RISK ASSESSMENT
report. 

roborative use of the findings. Answer (b) is  137. (c) The requirement is to 


incorrect be­cause the auditor need not  determine which individ­ual is 
perform substantive procedures to verify the  not considered a specialist upon 
specialist’s assumptions and findings. Answer whose work an inde­pendent 
(c) is incorrect because the client may have an auditor may rely. The 
understanding of the nature of the work  professional standards relat­ing to
performed by the specialist. See using the work of a specialist do 
AU 336 (revised in 1994 by SAS 73) for  not apply to using the work of an 
information on the auditor’s use of the  internal auditor. Answers (a), (b),
work of a specialist. and (d), actuary, appraiser, and 
engineer, respectively, are all 
135. (b) The requirement is to  exam­ples of specialists per the 
identify the circumstance in  professional standards. Note here 
which an auditor may  that the question and its reply do 
appropriately refer to the  not imply that a CPA can­not use 
findings of a specialist. Answer  the work of an internal auditor. 
(b) is correct because an auditor  What is being sug­gested is that 
may refer to a specialist when  an internal auditor is not 
the report is being modified due  considered a special­ist under the 
to the specialist’s findings.  professional standards. 
Answers (a) and (c) are 
incorrect because a client’s  C.14.  Inquiry of a Client’s Lawyer
familiarity with a specialist or  138. (d) The requirement is to identify 
understanding of the auditor’s  which partiesshould reach an understanding
use of the findings of a spe­ on the limits of materiality for purposes of a
cialist does not result in  lawyer’s response to an auditor’s inquiry 
modification of the audit report.  concerning litigation, claims, and 
Answer (d) is incorrect because  assessments. Answer (d) is correct because 
an auditor does not divide  AU 337 states that a lawyer’s response to 
responsibility with a specialist.  an inquiry may be limited to material items,
136. (c) The requirement is to identify  provided the lawyer and the auditor have 
the statement that an auditor must reached an understanding on the limits of 
document when using the work  materiality for this purpose. Answer (a) is 
of a spe­cialist. Answer (c) is  incor­rect because it includes the client’s 
correct because the specialist’s  management. An­
under­standing of the auditor’s  swer (b) is incorrect because it includes
corroborative use of his or her  the   client’s   audit   committee   and   omits
findings must be documented.  the   auditor.   Answer   (c)   is   incorrect
See AU 336 for this and other  because   it   includes   the   client’s
documentation requirements.  management and omits the auditor.
Answer (a) is incorrect because 
no statement concerning an 
update of the special­ist’s report 
is required to be documented. 
Answer (b) is incorrect because a
division of responsibility relates 
to cir­cumstances in which other 
auditors, not specialists, are in­
volved. Answer (d) is incorrect 
because an auditor will not 
normally issue a disclaimer 
related to whether the special­
ist’s findings corroborate the 
representations in the financial 
statements. The specialist’s 
report is only referred to when 
there is a material difference 
between the specialist’s find­ings
and the representations in the 
financial statements. See AU 336
for information on the effect of a 
specialist’s work on an auditor’s 
the client’s attorney is the 
139. (b) The requirement is to identify  auditor’s primary means of 
the correct state­ment concerning  obtaining corroboration of the 
the refusal of a client’s attorney to  information furnished by 
provide information requested in  management concerning 
an inquiry letter. Answer (b) is  litigation, claims, and 
correct because AU 337 indicates  assessments. Answer (a) is 
that this is a limitation on the  incorrect because it will often be 
scope of the audit. Answer (a) is  impossible to determine the 
incorrect because the lack of  probable outcome of asserted 
information is unlikely to lead to  claims and pending or threatened 
an adverse opinion since no  litigation. Answer (c) is incorrect 
information has been provided  because no such opinions on 
indicating that the financial  historical experiences are 
statements are misstated. Answer 
generally available. Answer (d) is 
incorrect because the description 
(c) is incorrect because withdrawal
of litigation, claims, and 
is not generally necessary due to 
assessments is generally prepared 
the client’s attorney’s failure to 
by the client. 
provide information. An­ 
swer   (d)   is   incorrect   because   significant 142. (b) The requirement is to identify 
deficiencies   pertain   to   weaknesses   in the procedure that is not performed
internal control.  regarding litigation, claims, and 
140. (c) The requirement is to identify the audit  assess­ments. Answer (b) is correct
proce­dure that an auditor most likely  because the CPA does not confirm 
would perform concerning litigation,  directly with the client’s lawyer 
claims, and assessments. Answer (c) is  that all claims have been recorded 
correct because auditors must discuss with  in the financial statements. 
management its policies and procedures for Answers (a), (c), and (d) are all 
evaluating and accounting for litigation,  incorrect because they represent 
claims and assessments. See AU 337 for  information obtained from 
this and other re­quirements. Answer (a) is  management regarding litigation, 
incorrect because the client’s lawyer is not  claims, and assessments as 
ordinarily asked to make an assessment  summarized in AU 337. 
about whether the client has a going 
concern problem (see AU 341 for  143. (a) The requirement is to identify 
information on an auditor’s consideration  the appropriate limitation for an 
of a client’s ability to continue as a going  attorney’s response to a client’s 
concern). Answer (b) is incor­rect because  letter of audit inquiry. Answer (a) is
an auditor will not ordinarily examine legal correct because AU 337 states that 
documents in the client’s lawyer’s  an attorney may appropriately limit 
possession. Answer (d) is incorrect because his response to matters to which 
an auditor will not ordinarily confirm with  s/he has given substantive attention 
the client’s lawyer that all litigation,  in the form of legal consultation or 
claims, and assessment have been recorded. representation. Answers (b), (c), 
and (d) are incorrect because AU 
141. (b) The requirement is to identify  337 presents a variety of other 
the primary rea­son that an  requests in addition to information 
auditor should request a client to  on the likelihood of an unfavorable 
send a letter of inquiry to its 
outcome of the matters disclosed by
attorneys. Answer (b) is correct 
the entity, similar litigation, and 
because a letter of audit inquiry to
probable outcomes. 
MODULE 4   RESPONDING TO RISK ASSESSMENT 153
147. (b) The requirement is to identify the least 
likely approach for auditing the fair values 
1. (a) If a client’s lawyer resigned  of assets and liabilities. Answer (b) is 
shortly after the receipt of an  correct because it is doubtful that audit 
attorney’s letter which indicated  com­mittee members will have information 
no significant disagreements with on the valuation. Answers (a), (c), and (d) 
the client’s assessment of  are all incorrect because they repre­sent the 
contingent liabilities, the auditor  three approaches presented for auditing fair 
should inquire why the attorney  values (as well as other estimates). 
re­signed. The auditor’s concern 
is whether any undisclosed  C.16.  Related­Party Transactions
unasserted claims have arisen. Per
AU 337, a lawyer may be  148. (c) The requirement is to identify the 
required to resign if his advice  auditing pro­cedure that would most likely 
concerning reporting for  assist an auditor in identifying related­party 
litigation, claims, and  transactions. Answer (c) is correct because 
assessments is disregarded by the  reviewing confirmations of loans receivable
client. Accordingly, the  and payable for indications of guarantees 
resignation shortly after issuance  may reveal unusual transactions that 
of an attorney’s letter may  involve related parties. See AU 334 for 
indicate a problem. Answer (b) is  procedures related to identifying 
incorrect because the attorney  transactions with related parties. An­swer 
issued a letter indicating no  (a) is incorrect because inspecting the 
significant disagreement with the  correspondence with lawyers for evidence 
client’s assessment of contingent  of unreported contingent liabili­ties does 
liabilities. Answers (c) and (d) are not generally relate directly to related­party 
incorrect be­cause AU 337 only  trans­actions. Answer (b) is incorrect 
suggests that the auditor should  because nonrecurring trans­actions are more
consider the need for inquiries  indicative of related­party transactions. 
(i.e., AU 337 does not require a  Answer (d) is incorrect because analytical 
com­plete new exam of  procedures per­formed to identify possible 
contingent liabilities or an  financial difficulties do not relate directly to
adverse opin­ion).  related­party transactions. 
2. (b) The requirement is to identify  149. (c) The requirement is to determine an 
the lawyer’s letter comment that  auditor’s responsibility after having 
is most likely to cause the auditor  determined that a related­party transaction 
to request clarification. Answer  has occurred. Answer (c) is correct because 
(b) is correct because a statement  after 
that the action can be settled for 
less than the damages claimed is 
unclear as to the details of the 
attorney’s belief. An­ 
swers (a), (c), and (d) are all incorrect because
they represent responses that may be clearly 
interpreted by the auditor. See AU 337, 
interpretation 7, for information on assessing 
law­yer’s evaluations of the likely outcome of
litigation. 
C.15.  Fair Values
146. (a) The requirement is to 
determine whether valua­tion 
issues arise at initial recording, 
subsequent to initial recording, or 
at both times when considering 
fair value of an asset or liability. 
Answer (a) is correct because AU 
328 makes clear that valuation 
issues arise at both times. An­
swers (b), (c), and (d) are all 
incorrect because they suggest 
that there are no valuation issues 
at one or both of these time 
periods. 
Answer (b) is incorrect because 
identifying the existence of such an act, the  the auditor will not in gen­eral be 
auditor should obtain an understanding of the  able to determine whether a 
business purpose of the trans­action. See AU  particular transaction would have 
334 for this and other responsibilities. An­ occurred if the parties had not 
swer (a) is incorrect because the mere  been related. Answer (c) is 
existence of a related­party transaction may or incorrect because, if proper 
may not lead to audit report modification.  disclosures are made, the related­
Answer (b) is incorrect because the perfor­ party transactions are not required
mance of analytical procedures is not  to be recorded on terms 
required. Answer (d) is incorrect because  equivalent to arm’s­length 
except for routine transactions, it will  transactions. 
generally not be possible to determine 
152. (c) The requirement is to identify 
whether the transac­tion would have taken 
place, or whether it was consummated on 
the circumstance in which an 
terms equivalent to an arm’s­length 
auditor most likely would modify 
an unqualified opinion if the 
transaction.
entity’s financial statements 
150. (d) The requirement is to  include a footnote on related­
identify the correct state­ment  party transactions. Answer (c) is 
concerning related­party  correct because it generally will 
transactions. Answer (d) is  not be possible to determine 
correct because AU 334 requires  whether a par­ticular transaction 
that the auditor should place  was consummated on terms 
primary emphasis on the  equivalent to those with unrelated 
adequacy of disclosure. An­swer  parties. Therefore, the auditor 
(a) is incorrect because  may be required to express a 
ascertaining rights and obliga­ qualified or an adverse opinion 
tions is only part of the auditor’s  when such an unsubstantiated 
total responsibility and not the  disclosure is included. Answers 
primary emphasis. Answer (b) is  (a), (b), and (d) are all incorrect 
incorrect because while auditors  because they represent situa­tions 
attempt to determine the  that may be disclosed in related­
existence of related parties, this  party transaction dis­closures. 
is not the primary emphasis. 
Answer (c) is in­correct because  C.17.  Subsequent Events
verifying the valuation of 
related­party transactions will  153. (d) The requirement is to identify the 
often not be possible.  most likelyprocedure to be performed in 
obtaining evidence about sub­sequent 
151. (d) The requirement is to identify  events. Answer (d) is correct because 
the correct state­ment concerning  changes in long­term debt occurring after 
related­party transactions.  year­end may require note disclosure. 
Answer (d) is correct because AU Answers (a) and (b) are incorrect because 
334 requires that procedures  audi­tors will not generally test changes in
directed toward identifying  employee pay rates after year­end or 
related­party transactions should  recompute depreciation expense for plant 
be per­formed, even if the auditor assets sold. Answer (c) is incorrect 
has no reason to suspect their  because payroll checks issued near year­
existence. Answer (a) is incorrect  end may frequently be cashed after year­
because, in the absence of  end and their investigation will not in 
evidence to the contrary, related­ general be directly related to obtaining 
party transactions need not be  evidence about subsequent events. See 
assumed to be outside the  AU 560 for the responsibilities of auditors
ordinary course of business.  with respect to subsequent events.
154 MODULE 4   RESPONDING TO RISK ASSESSMENT
completion of the auditor’s fieldwork. 
Answer (c) is correct because the 
154. (c) The requirement is to identify the event  transaction described is a type 2 subsequent
occur­ring after the issuance of an auditor’s  event (since the acquisition provided 
report that would most likely cause the  evidence of a condition which came into 
auditor to make further inquiries about the  existence after year­end) and therefore the 
previously issued financial statements.  proper accounting approach would be note 
Answer (c) is correct because when an  disclosure rather than adjustment. Answer 
auditor becomes aware of information  (a) is incorrect because adjust­ments are 
which relates to the financial statements  only appropriate for type 1 subsequent 
previously reported upon, but which was  events (events which provide evidence that 
not known at the date of the report, he or  the condition was in existence at year­end). 
she should undertake to determine whether  Answer (b) is incorrect because the auditor 
the informa­tion is reliable and whether the  does not issue financial statements for the 
facts existed at the date of the audit report;  client. Answer (d) is incorrect because the 
in this circumstance it seems that the lease  opinion paragraph of the report need not be 
transactions existed as of the date of the  modified; if any report modification were 
audit report. An­swer (a) is incorrect  considered necessary, it would be an 
because the natural disaster occurred  explanatory paragraph emphasizing the 
subsequent to the issuance of the audit  matter. 
report. Answer (b) is incorrect because the 
contingency had been properly dis­closed.  158. (d) The requirement is to identify a 
Answer (d) is incorrect because the sale of  procedure that an auditor would perform to 
the sub­sidiary occurred subsequent to the  obtain evidence about the oc­currence of 
issuance of the audit re­port.  subsequent events. Answer (d) is correct 
be­cause an auditor will inquire of officers 
155. (d) The requirement is to  and other executives having responsibility 
determine proper account­ing and for financial and accounting matters 
auditing treatment of  whether any unusual adjustments have been
uncollectibility of an account  made during the period from the balance 
receivable resulting from a  sheet date to the date of inquiry. See AU 
customer’s bankruptcy due to a  560 for auditing procedures performed to 
natural disaster occurring after a  identify subsequent events. 
client’s balance sheet date. 
Answer (d) is correct because a 
customer’s major casualty loss 
after year­end will result in a 
financial statement note 
disclosure with no adjustment and
no audit report modifica­tion due 
to consistency. 
156. (d) The requirement is to 
determine an auditor’s 
responsibility when subsequent to
issuance of an audit re­port a 
client sells the shares of a major 
subsidiary. An­swer (d) is correct 
because no action need be taken 
since the event arose after the 
issuance of the auditor’s report. 
An­swers (a), (b), and (c) are all 
incorrect because they outline 
responsibilities which are not 
appropriate in this circum­stance. 
See AU 561 for a discussion of 
auditor responsibil­ity when 
subsequent to the issuance of the 
auditor’s report the auditor 
becomes aware of a fact that 
existed at the dateof the 
auditor’s report.
157. (c) The requirement is to determine the 
auditor’s responsibility with respect to a 
client acquisition of 25% of its outstanding 
capital stock after year­end and prior to the 
informed. Answer (c) is incorrect
159. (a) The requirement is to determine a  because the financial statements 
procedure thatan auditor should generally  are the responsibility of manage­
perform regarding subsequent events.  ment, and the auditor will not 
Answer (a) is correct because the  revise or distribute them. Answer
ProfessionalStandards state that the auditor (d) is incorrect because no 
generally should comparethe latest  revised report will be issued. 
available interim financial statements with  161. (c) The requirement is to identify the 
the financial statements being audited. See  circumstance in which an auditor who finds
AU 560 for this and other requirements. 
that he or she has omitted a substantive 
Answer (b) is incorrect because second  procedure at the time of an audit may 
accounts receivable confirmation requests 
decide not to apply that procedure. Answer 
will be sent well before the auditor’s  (c) is correct because when results of other 
review of subsequent events. An­
procedures tend to compensate for the pro­
swer (c) is incorrect because the  cedure it may be omitted. Answer (a) is 
communication of material weaknesses is not incorrect because, even when distribution 
a subsequent event procedure. See of the financial statement has been limited 
AU 325 for the required communication to management and the board of directors, 
of  significantdeficiencies.   Answer   (d) it may be necessary to perform the 
is   incorrect   because   auditorsgenerally procedure. Answer (b) is incorrect because 
only   receive   cutoff   statements   for   the the type of report issued does not affect the 
period immediately after year­end, not need to perform the procedure. Answer (d) 
for multiple months. is incorrect because delays by the client in 
providing data are not an acceptable reason 
C.18.  Omitted Procedures Discovered  not to perform that procedure. 
After the Report
Date 162. (b) The requirement is to 
determine professional 
160. (b) The requirement is to identify  responsibility when, subsequent 
an auditor’s re­sponsibility when a  to issuance of an audit re­port, an 
client refuses to adjust recently  auditor has determined that a 
issued financial statements for a  necessary audit proce­dure has 
subsequent event related to the  been omitted. Answer (b) is 
bankruptcy of the entity’s principal  correct because an auditor must 
customer. Answer (b) is correct  apply procedures that would 
because if the client refuses to make provide a satis­factory basis for 
disclosures, AU  the opinion issued. Answer (a) is 
561 requires the auditor to notify each incorrect because stockholders 
member of the board of directors  need not be informed at this point
of such refusal and that he or she  that the audit report should not be
will take steps to prevent future  relied upon. Answer (c) is 
reliance upon the audit report.  incorrect because the auditor’s 
Ordinarily the auditor will then  report will not be reissued unless 
notify the clients and regulatory  the financial statements are 
agen­cies that the report should  restated. Answer (d) is incorrect 
no longer be associated with the  because tests of controls will not 
financial statements, and when  compensate for the omitted 
possible, notify persons known to procedure. See AU 390 for 
be relying upon the financial  overall procedures relating to 
statements. An­swer (a) is  considering omitted procedures 
incorrect because it is less likely  after an audit report has been 
that all of the creditors will be  issued. 
MODULE 4   RESPONDING TO RISK ASSESSMENT 155
Answer (a) is incorrect because 
shipments lacking sales invoices 
163. (c) The requirement is to identify an  and shipping documents will be 
auditor’s firstresponsibility upon  very difficult to identify; also, 
discovering six months after completion of  this reply is more limited than 
an audit that engagement personnel failed  answer (c). Answer (b) is 
to confirm sev­eral of the client’s material  incorrect because excessive 
accounts receivable balances. Answer (c) is write­offs of ac­counts receivable
correct because the auditor must first  will not usually be detected when
assess the importance of the omitted  testing the sales cutoff. Answer 
procedures to the auditor’s ability to  (d) is incorrect because it is 
support the previously expressed opinion.  unlikely that lapping in the 
An­swers (a) and (b) are incorrect because  application of cash receipts will 
prior to attempting any such confirmation  be detected by sales cutoff 
or performing alternative procedures, an  testing. More frequently, proce­
assessment of whether the procedures are  dures such as confirmations, 
needed is to be performed. Answer (d) is  analytical procedures, and an 
incorrect because a consideration of  analysis of deposit tickets reveal 
whether anyone is relying on, or is likely to lapping. 
rely on, the unqualified opinion is made 
after assessing the importance of the  166. (b) The requirement is to identify the 
omitted procedure. assertion be­ing tested by cutoff tests 
designed to detect credit sales made before 
D. Completing the Audit the end of the year that have improperly 
been recorded in the subsequent year. 
164. (c) The requirement is to  Answer (b) is correct be­cause the 
determine the procedure least  completeness assertion deals with whether 
likely to be performed before the all transactions have been included in the 
balance sheet date.Answer (c) is  proper period. An­swer (a) is incorrect 
correct because the search for  because the presentation or disclosure 
unrecorded li­abilities relies  assertion deals with whether particular 
upon a review of documents  components of the financial statements are 
unrecorded at year­end, as well  properly classified, described, and 
as inspection of purchases and  disclosed. Answer (c) is incorrect because 
disburse­ments recorded after  the rights and obligations assertion deals 
year­end, to determine whether a with whether assets are the rights of the 
proper cutoff of transactions  entity and liabilities are the obligations of 
between periods has occurred.  the entity at a given date. Answer (d) is 
Answer (a) is incorrect because  incorrect because the existence or 
auditors are able to test in­ternal  occurrence assertion deals with whether 
control over cash prior to year­ assets or liabili­ties of the entity exist at a 
end. Answers (b) and (d) are  given date and whether recorded 
incorrect because in cases of 
good internal con­trol, 
receivables may be confirmed 
and inventory observed prior to 
year­end. 
165. (c) The requirement is to 
determine the most likely type of 
transaction that would be 
detected by an auditor’s review 
of a client’s sales cutoff. Answer 
(c) is correct be­cause the 
auditor’s review will include a 
study of sales re­corded late in 
December and early in January. 
This will be accomplished by 
reviewing the period when the 
revenue was earned by shipment 
of goods or performance of ser­
vices, as compared to the period 
in which the revenue was 
recorded. Accordingly, the 
review of sales recorded in 
January may reveal unrecorded 
sales for the preceding year. 
well after the conclusion of the au­dit.
transactions have occurred during a 
given period. In this question, the  F. Operational Auditing
existence assertion would be tested if  168. (a) The requirement is to identify the 
the auditor sampled from sales recorded correct state­ment with respect to the 
prior to year­end to determine whether 
primary orientation of operational auditing.
the sale occurred before or after year­
Answer (a) is correct because operational 
end.
audits deal primarily with evaluating the 
D.1.  Procedures Completed near the  efficiency and effective­ness with which 
End of the Audit operations function, often with the 
intention of making improvements to 
167. (d) The requirement is to identify the  accomplish the goals of man­agement. 
procedure anauditor would most likely  Answers (b) and (c) are incorrect because 
perform during the overall review stage of  finan­cial statement audits are oriented 
formulating an opinion on an entity’s  toward such determina­tions, not 
financial statements. Answer (d) is correct  operational audits. Answer (d) is incorrect 
because a consideration of results relating  because examinations of internal control 
to the assessment of the risk of material  are not performed on operational audits. 
mis­statement due to fraud may reveal that 
the audit has inade­quately addressed that  169. (d) The requirement is to identify a typical 
risk; in such a case additional proce­dures  objective of an operational audit. Answer 
would be required. Answer (a) is incorrect  (d) is correct because op­erational audits 
because such assurance from the entity’s  typically address efficiency and effective­
attorney is ordinarily ob­tained prior to the  ness. Answer (a) is incorrect because while 
overall review stage of an audit. An­swer  the adequacy of internal control design may
(b) is incorrect because the verification of  be addressed during an opera­tional audit, 
the accuracy of the proof of cash is  this is less complete than answer (d). An­ 
ordinarily performed prior to the overall  swer (b) is incorrect because operational 
review and because little verification of the audits may or may not be related to 
bank cutoff statement is usually necessary  compliance with generally accepted gov­
since it is ordinarily received directly by  ernmental auditing standards. Answer (c) 
the auditor from the bank. Answer (c) is  is incorrect be­cause financial statement 
incorrect because such provisions for the  audits, not operational audits, ad­dress 
safeguarding of assets may be corrected  whether results of operations are fairly 
presented. 
156 MODULE 4 RESPONDING TO RISK ASSESSMENT

SOLUTIONS TO SIMULATIONS

Task­Based Simulation 1
Audit Investments and
Accounts Receivable Authoritative
Literature Help

A
u
1. I

2. R

3. T

E
1. (E) The verification of transfers from the current to the noncurrent investment portfolio will
provide assurance that the investments are properly classified in the financial statements. 
2. (F) Positive confirmation replies as of the balance sheet date for investments held by
independent custodians will pro­vide assurance that the recorded investments are in
fact owned by the audit client. 
3. (D)   Because   trading   investments   should   be   valued   at   fair   market   value,
determining   whether   any   impairments   in   the   price   of   investments   have   been
recorded will provide assurance that investments are properly valued. 

A
u
4. A

5. T

6. A

7. A

E
4. (B)   Performance   of   sales   cutoff   tests   will   provide   assurance   that   sales
transactions and the related receivables are re­corded in the proper period. Thus,
sales cutoff tests will provide assurance that all amounts owed to the entity at the
balance sheet date are recorded in that period. 
5. (E) A review of loan agreements, paying special attention to accounts receivable
that have been factored, will provide assurance as to whether the entity has a legal
right to all accounts receivable at the balance sheet date. 
6. (C)   An   analysis   of   the   aged   trial   balance   for   significant   past   due   accounts   will
provide evidence with respect to accounts that may be uncollectible. Accordingly,
the procedure will address the net realizable value of accounts receivable. 
7. (F) Because material amounts due from officers and employees should be segregated from other
receivables, a review of the trial balance for amounts due from officers and employees will 
provide assurance that accounts receivable are properly de­scribed and presented in the financial
statements. 

R
e
A
u
L
i

1. W
h
2. E

MODULE 4  RESPONDING TO RISK ASSESSMENT 157

Task­Based 
Simulation 3
Authoritati
Illegal Acts and ve
Related­ Literature
Party  Help IA
Transactions RP
Statement

1. A note payable has an interest rate well below the market rate at the time at which the
loan was obtained. 
2. The company has a properly documented loan but the loan has no scheduled  repayment
terms. 
3. Unexplained payments have been made to government officials. 
4. The company exchanged certain real estate property for similar real estate property. 
5. Large cash receipts near year­end have been received based on cash sales for which
there is no documentation. 

Task­Based Simulation 4
Accounts Receivable
Confirmations Authoritative
Literature Help

S
1. t
T
h
2. A
 
3. S
e
4. C

5. C

6. A
b
7. A
u
8. A

9. T
h
10. A
c
T

A
Iu
n

L
i
S
t
1. W
i
2. W
h
3. A

4. A
u
158 MODULE 4   RESPONDING TO RISK ASSESSMENT

S
t
5. W
h
6. I

7. I
n
8. A

9. O

10. A
t
Task­Based Simulation 6
Research
Authoritative
Literature Help

(A) (B) (C) (D) (E) (F) (G) (H)


1. Which title of the Professional Standards addresses this issue and will be
helpful in responding to her?
2. Enter the exact section and paragraph with helpful information. 332 22

Task­Based Simulation 7
Bank
Reconciliation Authoritative
Literature Help

1. (D,   I)  The   balance   per   bank   may  be   traced   to  a   standard   form  used   to   confirm
account balance information with financial institutions and to the cutoff statement
(on which will appear the beginning balance). 
2. (A,  G,  H,  I,  J)    One  of  the  deposits   in  transit  does  not  appear  on the   cutoff  bank
statement (the 9/29/05 deposit for 
$4,500). Accordingly, that deposit should be traced to the cash receipts journal (procedure A), 
the reason for the delay should be investigated (procedure G), and supporting documents 
should be inspected (procedure H). Both deposits should be traced to and from the bank 
reconciliation and the cutoff statement (procedures I and J).
3. (B, G, H, I, J) One of the checks does not appear on the cutoff statement (check 
#988 dated 8/31/05 for $2,200). Ac­cordingly, that check should be traced to the 
cash disbursements journal (procedure B), the reason for the delay should be in­
vestigated (procedure G), and supporting documents should be inspected 
(procedure H). All checks should be traced to and from the bank reconciliation 
and cutoff statement (procedures I and J). 
4. (E)  The credit memo from the bank for the note collected should be investigated. 
5. (E, I) The credit for the check that  was  charged  by the bank for an incorrect
amount should be investigated on both the bank credit memo and on the cutoff
statement. 
6. (C)  The only source of the balance per books is the cash general ledger account as of
9/30/05. 

Task­Based Simulation 8
Audit Authoritative
Procedures Literature Help

1. (E) Kiting involves manipulations causing an amount of cash to be included simultaneously in 
the balance of two or more bank accounts. Kiting schemes are based on the float period—the 
time necessary for a check deposited in one bank to clear the bank on which it was drawn. To 
detect kiting, a bank transfer schedule is prepared to determine whether cash is improperly 
included in two accounts. 
2. (D) A comparison of the cleared checks to the year­end bank reconciliation will identify checks 
that were not mailed until after the first week of the subsequent year because most of those 
checks will not be returned with the cutoff statement and will appear to remain outstanding an 
abnormally long period of time. 
3. (H)  Among the terms confirmed for such a borrowing arrangement will be information
on liens. 
MODULE 4  RESPONDING TO RISK ASSESSMENT 159

4. (K, L) A reply to the second request, or information from the credit agency, may
confirm   the   existence   of   the   new   cus­tomer.   Also,   examination   of   shipping
documents will reveal where the goods were shipped, and ordinarily to which
party. 
5. (P) Observing the payroll check distribution on a surprise basis will assist in
detection since the auditor will examine details related to any paychecks not
picked up by employees. 
6. (Q) Vouching data in the payroll register to document authorized pay rates will
reveal situations in which an employee is earning income at a rate that differs from
the authorized rate. 
7. (A) A comparison of the details of the cash receipts journal to the details on the
daily deposit slips will reveal a circum­stance since the details will have been
posted to accounts during the last week of the year under audit. 
8. (U) When vouchers are processed for merchandise not ordered or received, there
will be no supporting purchase orders and receiving reports and this will alert the
auditor to the problem. 
9. (B) Scanning the debits to the fixed asset accounts and vouching selected amounts
will reveal repairs that have improp­erly been capitalized. 
10. (A, J) Lapping involves concealing a cash shortage by delaying the recording of 
journal entries for cash receipts. Since lapping includes differences between the 
details of postings to the cash receipts journal and corresponding deposit slips, com­
paring these records will reveal it. Also, confirmation requests may identify lapping 
when payments of receivables (as indicated by confirmation replies) appear to have 
taken too much time to be processed. 
11. (E) Increasing cash by drawing a check in this manner is a form of kiting (see
answer   1).   Preparation   of   a   bank   transfer   schedule   will   assist   the   auditor   in
identifying such transactions. 
12. (J, L) Confirmations will identify overstated accounts receivable when customers
disagree with the recorded balance due. Also, the related overstated sales will not
have shipping documents indicating that a shipment has occurred. 

Task­Based Simulation 9
Substantive Procedures for
Property, Plant and Equipment Authoritative
Literature Help

1. D

2. D
i
3. D
i
4. D
i
5. D

6. D

E
1. (D) The requirement is to identify the best substantive procedure to determine that DietWeb has
legal rights to the property and equipment acquired during the year. Answer (D) is correct 
because the deeds and title insurance certificates will provide evidence that the company owns 
the property and equipment. 
2. (G) The requirement is to identify the best substantive procedure to determine that
DietWeb recorded property and equip­ment actually exists. Answer (G) is correct
because physically examining the items will provide this evidence. 
3. (B) The requirement is to identify a substantive procedure to test whether 
DietWeb’s net property and equipment wasproperly valued at the balance sheet 
date. Answer (B) is correct because reviewing depreciation expense (and the 
related allowance for doubtful accounts) will indicate whether the net value is 
proper. 
4. (E) The requirement is to identify how an auditor may test whether DietWeb 
recorded all property and equipment assetsthat were purchased during the year. 
Answer (E) is correct because performance of a cutoff test will indicate whether 
additions made during the year were properly recorded. 
5. (G) The requirement is to identify a substantive procedure to test whether DietWeb
recorded all property retirements thatoccurred during the year. Answer (G) is 
correct because examining the major recorded property and equipment items may 
identify situations in which an item has been retired (often due to its replacement) 
and is no longer available for physical examination. 
160 MODULE 4   RESPONDING TO RISK ASSESSMENT

6. (H) The requirement is to identify a substantive procedure to test whether DietWeb capitalized acquisitions. Answer (H) 
iscorrect because an analysis of repairs and maintenance accounts will reveal a situation in which such an acquisition has 
inappropriately been recorded as an expense and not capitalized.

Task­Based Simulation 10
Risk Analysis
Authoritative
Literature Help

Risk identified

1. TWD may not have legal title to certain
prop­erty   and   equipment   recorded   as
acquired dur­ing the year. 
2. Recorded property and equipment acquisitions
may include nonexistent assets. 
3. Recorded net property and equipment are
for proper amounts. 
Explanation of solutions
Related financial statement assertion Audit procedures
(A) (B) (C) (D) (E) (F) (G) (H) (I)  (J) (K) (L)

1. (C, I) Legal titles relates most directly to the client having rights over the assets; an examination of deeds and title
insur­ance certificates will provide assurance that the client has a legal right to the property and equipment acquired
during the year. 
1. D

2. E

3. D

1. (C) Because ownership information is included on invoices, examining vendors’ invoices will provide evidence that
the company legally owns inventory raw material items. 
2. (F) Selecting a sample of items and agreeing to the physical count sheet will establish that those items have been included in the
count, and this will address completeness of inventories. 
3. (C)  Examining vendors’ invoices will provide evidence as to the cost of the inventory items. 

4. D

5. D

6. N

E
4. (E) When examining the January 20X9 sales journal the auditor may identify sales that should have been recorded 
inDecember of 20X8.
MODULE 4  RESPONDING TO RISK ASSESSMENT 161

5. (F) Auditors will generally inquire of the credit manager as to his or her beliefs concerning the collectability of 
cariousreceivables, and thereby obtain evidence on the net realizable value of accounts receivable. Often one would 
expect an answer such as “analyze aging of receivables.” Since that was not present here, (F) is the best reply. 
6. (G)  A disclosure checklist is used to determine that the disclosure requirements of generally accepted accounting
principleshave been met. 

Task­Based Simulation 12
Inventory Audit Objectives
and Procedures Authoritative
Literature Help

1. I

2. D

3. D

4. D
e
5. D

E
1. (A) The requirement is to identify a procedure for identifying inventory transactions involving related parties. The best 
procedure listed is review minutes of Board of Directors’ meeting and contracts, and to make inquiries of management; these 
are all procedures used to identify related­party transactions. 
2. (E) The requirements is to identify a procedure for determining that items counted are included in the count sheet. 
The best procedure is to reconcile physical counts to perpetual records and general ledger balances and investigate
significant fluctuations. This will allow the auditor to identify items not included. 
3. (F) The requirement is to determine that a proper cutoff of purchases has occurred at year end. The best procedure 
listedis to review sales after year­end and open purchase order commitments—this will help determine whether 
transactions recorded after year­end should have been recorded prior to year­end. Another procedure, not listed, is to 
perform the procedure on transactions recorded right before year­end. 
4. (C)  The requirement is to determine that the financial statements include proper disclosures relating to inventory.
AnswerC is correct because inventories pledged under loan agreement should be disclosed. 
5. (G) The requirement is to determine that recorded inventory is owned. Examining invoices is best because invoice
willpresent information on the purchase. 

Task­Based Simulation 13
Spreadsheet
Completion
Authoritative
Literature Help

A B C D E F G
1 Holiday Manufacturing Co.
2 Balance Sheet
3 December 31, 20X1
4
5 Cash $240,000 Accounts Payable $160,000
6 Receivables 400,000 Notes payable 100,000
7 Inventory 600,000 Other current liabilities 140,000
8 Total current assets $1,240,000 Total current liabilities 400,000
9
10 Plant and equipment—net 760,000 Long-term debt 350,000
11 Common stock 750,000
12 Retained earnings 500,000
13 Total assets $2,000,000 Total liabilities and capital $2,000,000
14
15
MODULE 
162 4 RESPONDING TO RISK ASSESSMENT

A B C D E F G
16 Income Statement
17 Year ended December 31, 20X1
18
19 Sales $3,000,000
20 Cost of goods sold
21 Materials 800,000
22 Labor 700,000
23 Overhead 300,000 1,800,000
24 Gross margin 1,200,000
25
26 Selling expenses 240,000
27 General and admin. exp. 300,000 540,000
28 Operating income 660,000
29 Less: interest expense 40,000
30 Income before taxes 620,000
31 Less: federal income taxes 220,000
32 Net income $400,000
33
34
35
36 Ratios 12/31/X1 12/31/X0
37 Current ratio (1) 2.5
38 Quick ratio (2) 1.3
39 Accounts receivable turnover (3) 5.5
40 Inventory turnover (4) 2.5
41 Total asset turnover (5) 1.2
42 Gross margin % (6) 35%
43 Net operating margin % (7) 25%
44 Times interest earned (8) 10.3
45 Total debt to equity % (9) 50%

Ratio Spreadsheet Formula Calculation


1. (H) Current assets $1,240,000
Current ratio = =C8/G8 = 3.1
Current liabilities $400,000
2. (E) Quick assets* $240,000 + $400,000 $1,240,000
Quick ratio = =(C5+C6)/G8 = = 3.1
Current liabilities $400,000 $400,000
* Cash + Accounts receivable. Also marketable 
securities would be included if the company owned 
any. 
Sales
3. (K) Accounts receivable  Accounts receivable
turnover =
Cost of goods sold
4. (G)
Inventory turnover = Inventory
Sales
5. (D)
Total asset turnover = Total assets
Gross margin
6. (T) Gross margin
Sales
percentage =
Operating income
7. (P) Net operating
Sales
margin % =
8. (N) Operating income
Times interest earned = Interest expense
$3,000,000
=E19/C6 = 7.5
$400,000
$1,800,000
=E23/C7 = 3.0
$600,000
$3,000,000
=E19/C13 = 1.5
$2,000,000
$1,200,000
=E24/E19 = 40%
$3,000,000
$660,000
=E28/E19 = 22%
$3,000,000
$660,000
=E28/E29 = 16.5
$40,000
9. (U) Total debt to equity Total debt*
percentage = Owners’ equity**

* Total current liabilities + Long­term debt. 
** Common stock + Retained earnings
=(G8+G10)/(G11+G12) $400,000 + $350,000 $750,000
= = 60%
$750,000 + $500,000 $1,250,000  
MODULE 5   REPORTING 163

REPORTING
that the client not duplicate the restricted­use 
MULTIPLE­CHOICE QUESTIONS (1­164)
report for any purposes. 
1. The objective of an accountant’s compilation of  d. An   auditor   is   not   responsible   for
the financial statements of a nonissuer  controlling   the   distribution   of   such
(nonpublic company) is to provide what type of  reports. 
assurance?  6. Which   of   the   following   statements   is   a   basic
a. Absolute assurance.  element of the auditor’s standard report? 
b. Limited assurance. 
c. No assurance. 
d. Reasonable assurance. 
2. The existence of audit risk is recognized by the
state­ment in the auditor’s standard report that
the auditor 
a. Obtains reasonable assurance about 
whether the fi­nancial statements are free
of material misstate­ment. 
b. Assesses the accounting principles used 
and also evaluates the overall financial 
statement presenta­tion. 
c. Realizes some matters, either 
individually or in the aggregate, are 
important while other matters are not 
important. 
d. Is responsible for expressing an opinion
on the fi­nancial statements, which are
the responsibility of management. 
3. When an accountant performs more than one level
of service (for example, a compilation and a 
review, or a com­pilation and an audit) 
concerning the financial statements of a nonissuer
(nonpublic) entity, the accountant generally 
should issue the report that is appropriate for 
a. The lowest level of service rendered. 
b. The highest level of service rendered. 
c. A compilation engagement. 
d. A review engagement. 
4. Which   of   the   following   is  least  likely   to   be   a
restricted use report? 
a. A   report   on   internal   control   significant
deficiencies noted in an audit. 
b. A   required   communication   with   the
audit commit­tee. 
c. A report on financial statements prepared
follow­ing a comprehensive basis of 
accounting other than generally accepted 
accounting principles. 
d. A report on compliance with aspects of
contractual agreements. 
5. Which of the following statements is correct 
concerning an auditor’s responsibility for 
controlling the distributionby the client of a 
restricted­use report?
a. An auditor must inform the client that a 
restricted­use report is not intended for 
distribution to non­specified parties. 
b. When an auditor is aware that a client 
has distrib­uted a restricted­use report to
inappropriate third parties, the auditor 
should immediately inform the client to 
cease and desist. 
c. An auditor controls distribution through insisting 
a. The disclosures provide reasonable 
assurance that the financial statements 
are free of material mis­statement. 
b. The auditor evaluated the overall internal control. 
c. An audit includes assessing significant
estimates made by management. 
d. The financial statements are consistent
with those of the prior period. 
7. In May 2009, an auditor reissues the auditor’s 
report on the 2007 financial statements at a 
continuing client’s re­quest. The 2007 financial 
statements are not restated and the auditor does 
not revise the wording of the report. The auditor 
should 
a. Dual date the reissued report. 
b. Use the release date of the reissued report. 
c. Use the original report date on the reissued report. 
d. Use the current period auditor’s report
date on the reissued report. 
8. Which paragraphs of an auditor’s standard report 
on financial statements should refer to generally 
accepted au­diting standards (GAAS) and 
generally accepted accounting principles 
(GAAP)? 
GAAS GAAP
a. Opening Scope
b. Scope Scope
c. Scope Opinion
d. Opening Opinion
9. An auditor expressed a qualified opinion on the 
prior year’s financial statements because of a lack
of adequate disclosure. These financial statements
are properly restated in the current year and 
presented in comparative form with the current 
year’s financial statements. The auditor’s up­
dated report on the prior year’s financial 
statements should 
a. Be accompanied by the auditor’s original
report   on   the   prior   year’s   financial
statements. 
b. Continue   to   express   a   qualified
opinion on the prior year’s financial
statements. 
c. Make no reference to the type of 
opinion ex­pressed on the prior year’s 
financial statements. 
d. Express an unqualified opinion on the
restated   fi­nancial   statements   of   the
prior year. 
10. An auditor’s responsibility to express an opinion
on the financial statements is 
a. Implicitly   represented   in   the   auditor’s
standard re­port. 
b. Explicitly   represented   in   the   opening
paragraph   of   the   auditor’s   standard
report. 
c. Explicitly   represented   in   the   scope   paragraph   of
the auditor’s standard report. 
d. Explicitly   represented   in   the   opinion
paragraph   of   the   auditor’s   standard
report. 
11. Which of the following phrases should be 
included in the opinion paragraph when an auditor
expresses a qualified opinion? 
When read in
conjunction with With the foregoing
Note X Explanation
a. Yes No
b. No Yes
164 MODULE 5   REPORTING
a. It applies 
equally to a 
c. Yes complete set 
d. No of financial 
statements 
12.  How does  an auditor make
and to each 
the   following   representa­tions individual 
when   issuing   the   standard financial 
auditor’s report on com­parative state­ment. 
financial statements? b. It   applies   only
Examination of Consistent to   a   complete
evidence on a application of set   of   financial
test basis accounting principles state­ments. 
a. Explicitly c. It   applies
b. Implicitly equally   to
c. Implicitly each   item   in
d. Explicitly each
13. The   fourth   standard   of financial
reporting   requires   the statement. 
auditor’s   report   to d. It   applies
contain   either   an equally to each
expression   of   opinion material   item
regarding   the   financial in   each   fi­
statements   taken   as   a nancial
whole or an assertion to statement. 
the effect that an opinion 15. Wilson, CPA, completed the 
cannot   be   expressed. fieldwork of the audit of Abco’s 
The   objec­tive   of   the December 31, 2009 financial 
fourth   standard   is   to statements on March 6, 2010. A 
prevent  subsequent event requiring 
a. An   auditor adjustment to the 2009 financial 
from statements occurred on April 10, 
expressing 2010, and came to Wilson’s 
different attention on April 24, 2010. If the 
opinions   on adjustment is made without 
each   of   the disclosure of the event, Wilson’s 
basic   financial report ordi­narily should be dated 
statements.  a. March 6, 2010. 
b. Restrictions on  b. April 10, 2010. 
the scope of the  c. April 24, 2010. 
audit, whether 
d. Using dual dating. 
im­posed by the
client or by the  16. An auditor issued an audit report 
inability to  that was dual dated for a subsequent
obtain evidence. event occurring after the completion
c. Misinterpretatio of field­work but before issuance of 
ns regarding the the auditor’s report. The audi­tor’s 
degree   of responsibility for events occurring 
respon­sibility subsequent to the completion of 
the   auditor   is fieldwork was 
assuming.  a. Extended   to
d. An   auditor subsequent
from reporting events
on   one   basic occurring
financial through   the
statement   and date   of
not the others. issuance of the
report. 
14. Which of the following  b. Extended   to
best describes the  include   all
reference to the  events
expression “taken as a  occurring since
whole” in the fourth  the   completion
generally accepted  of fieldwork. 
auditing standard of  c. Limited   to   the   specific
reporting?  event referenced. 
d. Limited   to
include   only 17. A financial statement 
events audit report issued for the 
occurring   up   to audit of an issuer (public) 
the   date   of   the company concludes that 
last   subsequent the financial statements 
event follow 
referenced.  a. Generally   accepted
accounting principles. 
b. Public
Company
Accounting
Oversight
Board
standards. 
c. Generally   accepted
auditing standards. 
d. International   accounting
standards. 
18. Which of the 
following is not 
correct concerning 
information 
included in an audit 
report of financial 
statements issued 
under the 
requirements of the 
Public Company 
Accounting 
Oversight Board? 
a. The report 
should include 
the title “Report 
of Independent 
Registered 
Public 
Accounting 
Firm.” 
b. The   report
should refer
to   the
standards
of   the
PCAOB. 
c. The report 
should include 
a paragraph 
referring to the 
auditor’s report 
on compliance 
with laws and 
regulations. 
d. The   report
should
contain   the
city   and
state   or
country   of
the   office
that   issued
the report. 
19. A principal auditor 
decides not to refer to the 
audit of another CPA who
audited a subsidiary of the
principal auditor’s client. 
After making inquiries  20. The   introductory
about the other CPA’s  paragraph of an auditor’s
professional reputation  report   con­tains   the
and independence, the  following sentences: 
princi­pal auditor most  We did not audit the financial
likely would  statements of EZ Inc., a 
a. Add an  wholly owned subsidiary, 
which statements reflect total 
explanatory  as­sets and revenues 
paragraph to  constituting 27% and 29%, 
the auditor’s  respectively, of the related 
re­port  consolidated totals. Those 
statements were audited by 
indicating that  other auditors whose report 
the subsidiary’s has been furnished to us, and 
financial  our opinion, insofar as it 
statements are  relates to the amounts 
not material to  included for EZ Inc., is based 
solely on the report of the 
the  other auditors.
consolidated fi­
nancial  These sentences
statements.  a. Indicate   a   division   of
b. Document in the responsibility. 
engagement b. Assume   responsibility   for
letter   that   the the other auditor. 
princi­pal c. Require   a   departure   from
auditor   assumes an unqualified opinion. 
no d. Are   an   improper   form   of
responsibility reporting. 
for   the   other
21. In which of the 
CPA’s work and
opinion.  following situations 
c. Obtain written  would an auditor 
ordinarily issue an 
permission from 
unqualified audit opinion
the other CPA to 
without an ex­planatory 
omit the  paragraph? 
reference in the 
a. The   auditor   wishes   to
principal  emphasize   that   the   entity
auditor’s report.  had   significant   related­
d. Contact   the party transactions. 
other   CPA b. The auditor 
and review the decides to make 
audit   pro­ reference to the 
grams   and report of another
working auditor as a 
papers basis, in part, for
pertaining   to the audi­tor’s 
the   sub­ opinion. 
sidiary. 
MODULE 5   REPORTING 165
opinion on the
subsidiary’s
c. The entity  financial
issues financial  statements. 
statements that  c. Is   unable   to
present financial review   the
position and  audit programs
results of  and   work­ing
operations, but  papers   of   the
omits the  other CPA. 
statement of  d. Is satisfied as to
cash flows.  the
independence
d. The auditor has substantial
and   professional
doubt about the entity’s 
reputation of the
ability to continue as a 
other CPA. 
going concern, but the cir­
cumstances are fully  24. When financial 
disclosed in the financial  statements of a company 
statements.  that follows GASB 
22. An   auditor   may   issue   the standards would be 
misleading due to 
standard audit report when the  unusual cir­cumstances 
a. Auditor   refers   to   the depart from those 
findings of a specialist.  standards, the auditor 
b. Financial statements are  should explain the 
derived and condensed  unusual circumstances in
from complete audited  a separate paragraph and 
financial statements that  express an opinion that is
are filed with a regulatory  a. Unqualified. 
agency.  b. Qualified. 
c. Financial c. Adverse. 
statements   are d. Qualified   or   adverse,
prepared   on
depending on materiality. 
the   cash   re­
ceipts   and 25. An auditor concludes that
disbursements there is substantial doubt 
basis   of about an entity’s ability 
accounting.  to continue as a going 
d. Principal concern for a reasonable 
auditor period of time. If the 
assumes entity’s financial state­
responsibilit ments adequately disclose
y   for   the its financial difficulties, 
work   of the audi­tor’s report is 
another required to include an 
auditor.  explanatory paragraph 
23. In the auditor’s report, the that specifically uses the 
principal auditor decides  phrase(s) 
not to make reference to  “Reasonable period of
another CPA who audited time, not to
a client’s subsidiary. The  exceed 1 year” “Going concern”
principal auditor could  a. Yes Yes
justify this decision if,  b. Yes No
among other  c. No Yes
requirements, the  d. No No
principal auditor  26. Mead, CPA, had substantial
a. Issues   an doubt about Tech Co.’sability to
unqualified continue   as   a   going   concern
opinion   on   the when   reporting   on   Tech’s
consolidated audited financial statements for
financial
the year ended
statements. 
June 30, 2009. That doubt has been 
b. Learns   that
removed in 2010. What is Mead’s 
the other CPA
reporting responsibility if Tech is 
issued   an
presenting its financial statements 
unqualified
for the year ended June 30, 2010, on
a comparative basis with those of 
2010? c. A different 
a. The   explanatory explanatory 
paragraph   included   in paragraph 
the   2010   auditor’s describing 
report   should  not  be Mead’s reasons 
repeated.  for the removal 
b. The explanatory  of doubt should 
paragraph included in the  be included. 
2010 auditor’s report  d. A different 
should be repeated in its  explanatory 
entirety.  paragraph 
describing 
Tech’s plans 
for financial 
recovery 
should be in­
cluded. 
27. When an auditor 
concludes there is 
substantial doubt about a 
continuing audit client’s 
ability to continue as a 
go­ing concern for a 
reasonable period of 
time, the auditor’s 
responsibility is to 
a. Issue a qualified
or adverse 
opinion, 
depending upon 
materiality, due 
to the possible 
effects on the 
financial 
statements. 
b. Consider   the
adequacy   of
disclosure about
the   cli­ent’s
possible
inability   to
continue   as   a
going con­cern. 
c. Report   to   the
client’s   audit
committee
that   man­
agement’s
accounting
estimates may
need   to   be
adjusted. 
d. Reissue the prior year’s 
auditor’s report and add 
an explanatory paragraph 
that specifically refers to 
“substantial doubt” and 
“going concern.” 
28. Green, CPA, concludes 
that there is substantial 
doubt about JKL Co.’s 
ability to continue as a 
going concern. If JKL’s 
financial statements 
adequately disclose its 
financial difficulties,  management’s
Green’s auditor’s report  records
should  retention
Include an policy. 
explanatory
paragraph Specifically
30. After considering an 
following the use the entity’s negative trends 
opinion words “going and fi­nancial 
paragraph concern” difficulties, an auditor 
a. Yes Yes has substantial doubt 
b. Yes Yes about the entity’s ability 
c. Yes No to continue as a going 
d. No Yes concern. The auditor’s 
considerations relating to
29. In   which   of   the   following management’s plans for 
circumstances would an audi­tor most dealing with the adverse 
likely   add   an   explanatory   paragraph effects of these 
to   the   standard   report   while  not conditions most likely 
affecting   the   auditor’s   unqualified would include 
opinion?  management’s plans to 
a. The   auditor   is a. Increase   current   dividend
asked to report on distributions. 
the balance sheet, b. Reduce   existing   lines   of
but  not  on   the credit. 
other   basic c. Increase ownership equity.
financial d. Purchase   assets   formerly
statements.  leased. 
b. There   is 31. Which of the following conditions 
substantial doubt or events most likely would cause 
about   the an auditor to have substantial doubt 
entity’s ability to about an entity’s ability to continue 
continue   as   a as a going concern? 
going concern.  a. Signi
c. Management’ fican
s estimates of t
the   effects   of relate
future   events d­
are party
unreasonable. trans
d. Certain actio
transactions ns
cannot  be are
tested   because perva
of ­sive.
166 MODULE 5   REPORTING
conditions and events that
may indicate substantial 
b. Usual   trade   credit   from doubt about an entity’s 
suppliers is denied.  ability to continue as a 
c. Arrearages   in   preferred going concern? 
stock dividends are paid.  a. Inspecting   title
d. Restrictions   on documents   to
the   disposal   of verify   whether
principal   assets any   assets   are
are present.  pledged   as
collateral. 
47. Cooper, CPA, believes there is  b. Confirming
substantial doubt about the ability  with   third
of Zero Corp. to continue as a going parties   the
concern for a reasonable period of  details   of  ar­
time. In evaluating Zero’s plans for  rangements
dealing with the adverse effects of  to   maintain
future conditions and events,  financial
Cooper most likely would consider,  support. 
as a mitigating factor, Zero’s plans  c. Reconciling the 
to  cash balance per 
a. Discuss with books with the 
lenders   the cutoff bank 
terms   of   all statement and the
debt   and bank 
loan confirmation. 
agreements.  d. Comparing the
b. Strengthen   controls   over entity’s 
cash disbursements.  depreciation 
c. Purchase and asset 
production capitalization 
facilities policies to 
currently other entities 
being   leased in the in­
from   a dustry. 
related
party.  50. Which of the following 
d. Postpone audit procedures would 
expenditures most likely assist an 
for   research auditor in identifying 
and   develop­ conditions and events 
ment projects.  that may indicate there 
could be substantial 
48. Which of the following conditions  doubt about an entity’s 
or events most likely would cause  ability to continue as a 
an auditor to have substantial doubt  going concern? 
about an entity’s ability to continue  a. Review
as a going concern?  compliance with
a. Cash   flows   from the   terms   of
operating   activities   are debt   agree­
negative.  ments. 
b. Research b. Confirmation   of
and accounts
developmen receivable   from
t   projects princi­pal
are   post­ customers. 
poned.  c. Reconciliation
c. Significant of   interest
related­party expense   with
transactions debt   out­
are perva­sive. standing. 
d. Stock   dividends   replace d. Confirmation   of   bank
annual cash dividends.  balances. 
49. Which of the following  51. Davis, CPA, believes 
auditing procedures most  there is substantial 
likely would assist an  doubt about the ability 
auditor in identifying  of Hill Co. to continue 
as a going concern for a
reasonable period of  c. Purcha
time. In evaluating  se
Hill’s plans for dealing  equipm
with the adverse effects ent and
of future conditions and produc
events, Davis most  tion
likely would consider,  faciliti
as a mitigating factor,  es   cur­
Hill’s plans to  rently
a. Accelerate being
research   and leased. 
development d. Negot
projects   re­lated iate
to   future reduct
products.  ions
b. Accumulate in
treasury   stock requir
at   prices ed
favorable   to divide
Hill’s   historic nds
price range.  being
paid
on
prefer
red
stock. 
37. The adverse effects of 
events causing an auditor
to believe there is 
substantial doubt about 
an entity’s ability to 
continue as a going 
concern would most 
likely be mitigated by 
evidence relating to the 
a. Abi
lity
to
exp
and
ope
rati
ons
into
ne
w
pro
duc
t
line
s in
the
fut
ure.
b. Feasib
ility of
plans
to
purcha
se
leased
equip
ment
at   less
than
market
value. 
c. Marke auditor’s report for 
tabilit An unjustified A material weakness
y   of accounting change in the internal control
assets a. Yes Yes
that b. Yes No
manag c. No Yes
ement d. No No
plans
to sell. 40. Under   which   of   the
d. Co following
m circumstances   would   a
mit disclaimer   of   opinion
ted not be appropriate? 
arr a. The   auditor   is
an unable   to
ge determine   the
me amounts   as­
nts sociated with an
to employee   fraud
co scheme. 
nv
b. Management   does  not
ert
pre provide   reasonable
fer justifica­tion for a change
red in accounting principles. 
sto c. The client 
ck refuses to 
to permit the 
lon auditor to 
g­ confirm certain 
ter accounts 
m receivable or 
de apply 
bt.  alternative 
procedures to 
38. For   which   of   the verify their 
following   events balances. 
would an auditor issue d. The   chief
a report that omits any executive
reference   to officer   is
consistency?  unwilling   to
a. A change in the sign   the
method   of management
accounting   for representation
invento­ries.  letter. 
b. A change from 
41. Digit Co. uses the FIFO 
an accounting 
method of costing for its 
principle that is
inter­national subsidiary’s 
not generally 
inventory and LIFO for its 
accepted to one
domestic inventory. Under 
that is 
generally ac­ these circumstances, the 
cepted.  auditor’s report on Digit’s 
c. A   change   in financial statements should
the   useful express an 
life   used   to a. Unqualified opinion. 
calculate   the b. Opinion qualified because
provision for of a lack of consistency. 
depreciation c. Opinion
expense.  qualified
d. Management’s   lack   of because   of
reasonable justification for a   departure
a   change   in   accounting from
principle.  GAAP. 
d. Adverse opinion. 
39. An auditor would express
an   unqualified   opinion 42. In the first audit of a new
with   an   explanatory client, an auditor was 
paragraph   added   to   the able to extend auditing 
procedures to gather  a. Not report on the client’s
sufficient evidence about income statement. 
consistency. Under these b. Not refer to consistency in
circumstances, the  the auditor’s report. 
auditor should 
MODULE 5   REPORTING 167
c. State   the
auditor’s explicit
c. State   that   the   consistency concurrence
standard does not apply.  with   or   op­
d. State   that   the position   to   the
accounting change. 
principles   have d. Refer   to   the
been   ap­plied financial
consistently.  statement   note
that   discusses
43. When management does not  the   change   in
provide reasonable justifi­cation  detail. 
that a change in accounting 
principle is preferable and it  45. An entity changed from 
presents comparative financial  the straight­line method 
statements, the auditor should  to the declining balance 
express a qualified opinion  method of depreciation 
a. Only   in for all newly ac­quired 
the   year assets. This change has no
of   the material effect on the 
accountin current year’s financial 
g statements, but is 
principle reasonably certain to have
change.  a substantial effect in 
b. Each   year   that later years. If the change 
the   financial is disclosed in the notes to
statements the financial statements, 
initially   re­ the auditor should issue a 
flecting   the report with a(n) 
change   are a. “Except   for”   qualified
presented.  opinion. 
c. Each   year b. Explanatory paragraph. 
until c. Unqualified opinion. 
management d. Consistency modification. 
changes   back
to   the 46. When reporting on 
accounting comparative financial 
principle statements, an auditor 
formerly used. ordinarily should change 
d. Only   if   the the previously issued 
opinion on the prior year’s
change   is   to
financial statements if the 
an
a. Prior year’s 
accounting
financial 
principle that statements are 
is  not restated to 
generally conform with 
accepted.  generally 
accepted 
44. When an entity changes 
accounting 
its method of accounting 
prin­ciples. 
for income taxes, which 
b. Auditor is a 
has a material effect on 
predecessor 
comparability, the 
auditor who has 
auditor should refer to 
been re­quested 
the change in an 
by a former 
explanatory paragraph 
client to reissue 
added to the auditor’s 
the previously 
report. This paragraph 
issued report. 
should identify the nature
c. Prior year’s 
of the change and 
opinion was 
a. Explain why the change is
justified   under   generally unqualified and 
accepted   accounting the opin­ion on 
principles.  the current 
b. Describe   the   cumulative year’s financial 
effect of the change on the statements is 
audited   financial modified due to 
statements.  a lack of 
consistency. 
d. Prior   year’s BI. Indicate   the   substantive   reasons
financial for Jewel’s qualification. 
statements   are a. I only. 
restated   fol­ b. I and II only. 
lowing   a c. II and III only. 
pooling   of d. I, II, and III. 
interests in the
current year.  48. Before reissuing the prior
year’s auditor’s report on 
47. Jewel, CPA, audited  the financial statements 
Infinite Co.’s prior year  of a former client, the 
financial statements.  predecessor auditor 
These statements are  should obtain a letter of 
presented with those of  representations from the 
the current year for 
comparative purposes  Former client’s management Successor auditor
without Jewel’s auditor’s  a. Yes Yes
b. Yes No
report, which expressed a 
c. No Yes
qualified opinion. In 
d. No No
drafting the current year’s 
auditor’s report, Crain,  49. When single­year 
CPA, the successor  financial statements are 
auditor, should  presented, an auditor 
I. Not name Jewel as the  ordinarily would express 
predecessor auditor. an unqualified opinion in 
II. Indicate the type of report issued  an unmodified report if 
by Jewel. the 
a. Audit
or   is
unable
to
obtain
audite
d
financ
ial
state­
ments
suppor
ting
the
entity’
s
invest
ment
in   a
for­
eign
affiliat
e. 
b. Entity
decline
s   to
present
a
stateme
nt   of
cash
flows
with  its
balance
sheet
and
related
stateme
nts   of
in­
come
and auditor
retaine should
d issue
earning one
s.  audit
c. Audito report
r  that   is
wishes  on
to  both
empha present
size an ed
accoun years. 
ting  b. The
matter  audito
affecti r
ng the  should
compa issue
rability two
of the  audit
financi report
al  s,   one
state­ on
ments  each
with  year. 
those  c. The
of the  auditor
prior  should
year.  issue
d. Prior one
year’ audit
s  report,
finan but
cial  only
state on   the
ment most
s  recent
were  year. 
audit d. The 
ed by audit
anoth or 
er  may 
CPA  issue 
whos either
e  one 
repor audit 
t,  report
whic on 
h  both 
expre prese
nted 
ssed 
years,
an 
or 
unqu
two 
alifie
audit 
d  report
opini s, one
on, is on 
not  each 
prese year. 
nted. 
51. The predecessor auditor, 
50. A client is presenting  who is satisfied after 
comparative (two­year)  properly communicating 
financial statements.  with the successor auditor, 
Which of the following is  has reissued a report 
correct concerning  because the audit client 
reporting responsibilities  desires comparative 
of a continuing auditor?  financial statements. The 
a. The predecessor auditor’s 
report should make  succ
a. Ref esso
ere r
nce audit
to or
the only
rep in
ort the
of opin
the ion
suc para
ces
grap
sor
h. 
aud
itor d. No
onl refer
y in ence
the to the
sco repor
pe t   or
par the
agr work
aph of the
.  suc­
b. Refer cesso
ence r
to the audit
work or. 
of the 52. Unaudited financial statements for 
succe the prior year pre­sented in 
ssor comparative form with audited 
audit financial statements for the current 
or   in year should be clearly marked to 
the indicate their status and 
scope
and I. The report
opini on   the
on prior
period
parag should   be
raphs reissued to
.  accompan
c. Refe y   the
renc current
e   to period
both report. 
the 19. The report on the 
wor current period should 
k include as a separate 
and paragraph a description
of the responsibility 
the
assumed for the prior 
repo
period’s financial 
rt   of
statements. 
the
168 MODULE 5   REPORTING
Board   (GASB),   the
auditor should issue a(n) 
a. I only.  a. Adverse opinion. 
b. II only.  b. Qualified  opinion  with  an
c. Both I and II.  explanatory paragraph. 
d. Either I or II.  c. Unqualified opinion. 
d. Unqualified
53. In an audit of a nonissuer opinion   with   an
company,   which additional
statement   is   correct explanatory
concerning   required paragraph. 
supplementary
information   by   a 55. If an auditor is asked to 
designated   accounting provide an opinion 
standards setter?  relating to information 
a. The  auditor   has accompanying the 
no financial statements in a
responsibility document, the opinion 
for   required will ordinarily be upon 
sup­plementary whether the information 
information   as is fairly stated in 
long   as   it   is a. Accordance
outside   the with   US
basic   financial generally
statements.  accepted
b. The auditor’s  auditing
only  standards. 
responsibility  b. Conformity   with   US
for required  generally   accepted
sup­plementary  accounting principles. 
information is  c. All   material
to determine  respects   in
that such  relation   to   the
information has basic   finan­cial
not been  statements
taken   as   a
omitted. 
whole. 
c. The auditor 
d. Accordance with
should apply 
attestation
certain limited 
standards
proce­dures to 
expressing   a
the required  conclusion about
supplementary  management’s
information, and  assertions. 
report 
deficiencies in,  56. An auditor concludes 
or omissions of,  that there is a material 
such in­ inconsis­tency in the 
formation.  other information in an 
d. The auditor  annual report to share­
should apply  holders containing 
tests of details of audited financial 
transac­tions and statements. If the 
balances to the  auditor concludes that 
required  the financial statements 
supplementary  do not re­quire revision,
information, and  but the client refuses to 
report any  revise or eliminate the 
material  material inconsistency, 
misstatements in  the auditor may 
such  a. Revise the auditor’s report
information.  to   include   a   separate   ex­
planatory   paragraph
54. If management declines to describing   the   material
present   supplementary incon­sistency. 
infor­mation   required   by b. Issue an “except
the   Governmental for” qualified 
Accounting   Standards
opinion after 
dis­cussing the  a. Perform   inquiry
matter with the  and   analytical
client’s board of procedures   to
direc­tors.  ascer­tain whether
c. Consider   the the   other
matter   closed information   is
since   the   other reasonable. 
informa­tion   is b. Add   an
not  in   the explanatory
audited   financial paragraph   to
statements.  the auditor’s re­
d. Disclaim   an port   without
opinion   on   the changing   the
financial opinion   on   the
statements   af­ter financial
explaining   the statements. 
material c. Perform   the   appropriate
inconsistency   in substantive auditing proce­
a   sepa­rate dures   to   corroborate   the
explanatory other information. 
paragraph.  d. Read   the   other
information   to
57. When audited financial  determine   that
statements are presented 
it   is   consistent
in a client’s document 
with   the
containing other 
information, the auditor  audited
should  financial
statements. 

58. An auditor includes a 
separate paragraph in an 
other­wise unmodified 
report to emphasize that 
the entity being reported 
on had significant 
transactions with related 
parties. The inclusion of 
this separate paragraph 
a. Is   considered
an   “except
for”
qualification
of the opinion.
b. Violates 
generally 
accepted 
auditing 
standards if this 
information is 
already 
disclosed in 
footnotes to the 
financial 
statements. 
c. Necessitates a 
revision of the 
opinion 
paragraph to 
include the 
phrase “with the
foregoing 
explana­tion.” 
d. Is   appropriate
and would not
negate   the
unquali­fied
opinion.  dis­cussed in
the
59. An auditor concludes that preceding
a client’s illegal act,  paragraph. 
which has a material  d. Does  not  present fairly in
effect on the financial 
all material respects. 
statements, has not been 
properly accounted for or  61. In which of the following
disclosed. Depending on  circumstances   would   an
the materiality of the  audi­tor be most likely to
effect on the financial  express   an   adverse
statements, the auditor  opinion? 
should express either a(n) a. The   chief
a. Adverse   opinion   or   a executive
disclaimer of opinion.  officer   refuses
b. Qualified   opinion   or   an the   auditor   ac­
adverse opinion.  cess   to   minutes
c. Disclaimer of of   board   of
opinion or an directors’
unqualified
meetings. 
opinion   with
b. Tests   of
a   separate
explanatory controls show
paragraph.  that   the
d. Unqualified entity’s
opinion   with internal
a   separate control   is   so
explanatory poor   that   it
paragraph   or cannot  be
a   qualified relied upon. 
opinion.  c. The financial 
60. Which of the following  statements are 
phrases would an auditor not in 
most likely include in  conformity 
the auditor’s report when with the FASB
expressing a qualified  Statements 
opinion because of  regarding the 
inadequate disclosure?  capitali­zation 
a. Subject to the  of leases. 
departure from  d. Information 
US generally  comes to the 
ac­cepted  auditor’s 
accounting  attention that 
raises 
principles, as 
substantial 
described 
doubt about the 
above.  entity’s ability 
b. With   the to continue as a 
foregoing going concern. 
explanation
of   these 62. When an auditor qualifies 
omitted an opinion because of 
disclosures.  inade­quate disclosure, the
c. Except   for auditor should describe 
the omission the nature of the omission 
of   the in a separate explanatory 
information paragraph and modify the 
MODULE 5   REPORTING 169
cause   the
auditor  to  have
Introductory Scope substantial
paragraph paragraph doubt about the
a. Yes No en­tity’s   ability
b. Yes Yes to continue as a
c. No Yes going concern. 
d. No No
65. In which of the following
63. If a publicly held  situations would an 
company issues financial  auditor ordinarily choose 
statements that purport to  between expressing a 
present its financial  qualified opinion or an 
position and results of  adverse opinion? 
operations but omits the  a. The auditor did
statement of cash flows, 
not  observe
the auditor ordinarily will 
the   entity’s
express a(n) 
physical
a. Disclaimer of opinion. 
inventory   and
b. Qualified opinion. 
is   unable   to
c. Review report. 
become
d. Unqualified satisfied   about
opinion   with its   balance   by
a   separate
other   auditing
explanatory
procedures. 
paragraph. 
b. Conditions   that
64. In which of the following  cause   the
situations would an  auditor   to   have
auditor ordinarily choose  substan­tial
between expressing an  doubt   about   the
“except for” quali­fied  entity’s   ability
opinion or an adverse  to continue as a
opinion?  going   concern
a. The   auditor are inadequately
did   not disclosed. 
observe   the c. There has been
entity’s a   change   in
physical accounting
inventory   and principles   that
is   unable   to has   a   material
become effect   on   the
satisfied   as   to comparability
its   balance   by of   the   entity’s
other   auditing financial
procedures.  statements. 
b. The financial  d. The auditor is 
statements fail  unable to apply
to disclose  necessary 
informa­tion  proce­dures 
that is required
concerning an 
by generally 
investor’s share
accepted ac­
counting  of an inves­
principles.  tee’s earnings 
c. The   auditor   is recognized on 
asked to report the equity 
only   on   the method. 
entity’s
66. In the first audit of a client, an 
balance   sheet
and  not on  the auditor was not able to gather 
other   basic sufficient evidence about the 
financial consistent application of accounting
statements.  principles between the current and 
the prior year, as well as the 
d. Events
amounts of assets or liabilities at the
disclosed in the
beginning of the current year. This 
financial
was due to the client’s record re­
statements
tention policies. If the amounts in 
question could materially affect  67. In   which   of   the   following
current operating results, the auditor circumstances   would   an   audi­tor
would  not express an unqualified opinion?
a. Be unable to a. There   has   been
express   an a   material
opinion   on change   between
the   current periods   in
year’s accounting
results   of principles. 
operations b. Quarterly
and   cash financial   data
flows.  required   by
b. Express a  the   SEC   has
qualified  been omitted. 
opinion on the  c. The   auditor
financial state­ wishes   to
ments because  emphasize   an
of a client­ unusually   im­
imposed scope  portant
limita­tion.  subsequent
c. Withdraw event. 
from   the d. The auditor is
engagement unable   to
and   refuse   to obtain audited
be   associated financial
with   the statements   of
financial a consolidated
statements. 
investee. 
d. Specifically
state   that   the 68. Due to a scope limitation, 
financial an auditor disclaimed an 
statements   are opinion on the financial 
not statements taken as a 
comparable   to whole, but the auditor’s 
the   prior   year report included a 
due   to   an statement that the current 
uncer­tainty.  asset portion of the 
entity’s balance sheet was 
fairly stated. The 
inclusion of this statement
is 
a. Not
appropriate
because   it
may   tend
to   over­
shadow the
auditor’s
disclaimer
of opinion. 
b. Not
appropriate
because   the
auditor   is
prohibited
from   reporting
on   only   one
basic   financial
state­ment. 
c. Appropriate 
provided the 
auditor’s scope
para­graph 
adequately 
describes the 
scope 
limitation.  sufficiency   of   evidential   matter
d. Appropriate  should describe the limita­tions in
provided the  an   explanatory   paragraph.   The
statement is in a  auditor   should   also   refer   to   the
separate  limitation in the
paragraph  Scope Opinion Notes to the
preceding the  paragraph paragraph financial statements
disclaimer of  a. Yes No Yes
opinion  b. No Yes No
paragraph.  c. Yes Yes No
d. Yes Yes Yes
69. Park, CPA, was engaged 
to audit the financial state­ 71. Harris, CPA, has been asked 
ments of Tech Co., a new  to audit and report on thebalance 
client, for the year ended  sheet of Fox Co. but not on the 
Decem­ber 31, 2009. Park statements of in­come, retained 
obtained sufficient audit  earnings, or cash flows. Harris 
evidence for all of Tech’s  will have access to all information
financial statement items  underlying the basic financial 
except Tech’s opening  statements. Under these 
inventory. Due to  circumstances, Harris may
inadequate financial  a. Not   accept   the
records, Park could not  engagement   because   it
verify Tech’s January 1,  would   con­stitute   a
2009 inventory balances.  violation   of   the
Park’s opinion on Tech’s  profession’s ethical stan­
2009 financial statements  dards. 
most likely will be  b. Not   accept   the
Balance sheet Income statement engagement   because   it
a. Disclaimer Disclaimer would be tantamount to
b. Unqualified Disclaimer rendering   a   piecemeal
c. Disclaimer Adverse opinion. 
d. Unqualified Adverse
c. Accept   the   engagement
70.  An   auditor   who   qualifies   an because   such   engagements
opinion   because   of   an   in­ merely   involve   limited
reporting objectives. 
170 MODULE 5   REPORTING
ventory   is
completed. 
d. Accept the  c. Requests   that
engagement but certain   material
should disclaim  accounts
an opinion  receivable  not
because of an  be confirmed.
inability to  d. Refuses to 
apply the proce­ acknowledge its 
dures  responsibility 
considered  for the fair 
necessary.  presentation of 
72. When disclaiming an  the financial 
opinion due to a client­ statements in 
imposed scope limitation, con­formity with
an auditor should indicate GAAP. 
in a separate paragraph  75. An   auditor   may  not  issue   a
why the audit did not 
qualified opinion when 
comply with generally 
ac­cepted auditing  a. An   accounting
standards. The auditor  principle   at
should also omit the  variance   with
GAAP is used. 
Scope Opinion b. The   auditor
paragraph paragraph lacks
a. No Yes independence
b. Yes Yes
with   respect   to
c. No No
the   audited
d. Yes No
entity. 
73. An auditor decides to issue a  c. A   scope
qualified opinion on an entity’s  limitation
financial statements because a  prevents   the
major inadequacy in its  auditor   from
computerized accounting records  com­pleting   an
prevents the auditor from applying  important   audit
necessary procedures. The opinion  procedure. 
paragraph of the auditor’s report  d. The   auditor’s
should state that the qualification  report   refers   to
pertains to  the   work   of   a
a. A   client­imposed   scope special­ist. 
limitation.  76. An auditor may express 
b. A   departure an opinion on an entity’s 
from   generally ac­counts receivable 
accepted balance even if the 
auditing   stan­ auditor has disclaimed an 
dards.  opinion on the financial 
c. The possible effects on the statements taken as a 
financial statements.  whole pro­vided the 
d. Inadequate   disclosure   of a. Report on the 
necessary information.  accounts 
74. A   scope   limitation receivable 
sufficient   to   preclude   an discloses the 
unqualified   opinion reason for the 
always   will   result   when disclaimer of 
management  opinion on the 
a. Prevents   the finan­cial 
auditor   from statements. 
reviewing   the b. Distribution of
working the   report   on
papers   of   the the   accounts
predecessor receiv­able   is
auditor.  restricted   to
b. Engages   the internal   use
auditor after the only. 
year­end c. Auditor   also
physical   in­ reports   on   the
current   asset
portion   of   the
entity’s   balance audited financial statements with 
sheet.  March’s auditor’s report to 1st 
d. Report on the  Federal Bank to obtain financing in 
accounts  Monday’s attempt to purchase 
receivable is  Wall. In these circumstances, 
presented  March’s auditor’s report would 
separately from  usually be addressed to
the disclaimer of a. Monday   Corp.,   the   client
opinion on the  that engaged March. 
fi­nancial  b. Wall   Corp.,   the   entity
statements.  audited by March. 
77. March,   CPA,   is   engaged c. 1st Federal Bank. 
by   Monday   Corp.,   a d. Both   Monday   Corp.   and
client,   to   audit   the 1st Federal Bank. 
financial   statements   of 78. When   an   auditor
Wall   Corp.,   a   company expresses an adverse
that is not March’s client. opinion,   the   opinion
Monday   expects   to paragraph   should
present Wall’s  include 
a. The   principal
effects   of   the
departure   from
gener­ally
accepted
accounting
principles. 
b. A   direct
reference   to   a
separate
paragraph
disclos­ing   the
basis   for   the
opinion. 
c. The
substantive
reasons   for
the financial
state­ments
being
misleading. 
d. A   description   of
the   uncertainty
or   scope
limitation   that
prevents   an
unqualified
opinion. 
79. An   auditor   should
disclose   the   substantive
reasons for expressing an
adverse   opinion   in   an
explanatory paragraph 
a. Preceding   the   scope
paragraph. 
b. Preceding   the   opinion
paragraph. 
c. Following   the   opinion
paragraph. 
d. Within   the   notes   to   the
financial statements. 
80. When   an   independent
CPA assists in preparing
the   fi­nancial   statements
of a publicly held entity,
but   has  not  au­dited   or
reviewed them, the CPA a. I only. 
should issue a disclaimer b. II only. 
of   opinion.   In   such c. Both I and II. 
situations,   the   CPA   has d. Either I or II. 
no  responsibil­ity   to
apply   any   procedures
beyond 
a. Documenting
that   internal
control   is  not
being   re­lied
on. 
b. Reading   the
financial
statements   for
obvious   mate­
rial
misstatements. 
c. Ascertaining
whether   the
financial
statements   are
in   conformity
with GAAP. 
d. Determining
whether
management
has   elected   to
omit
substantially
all   required
disclosures. 
81. When an independent 
CPA is associated with 
the fi­nancial statements
of a publicly held entity 
but has not au­dited or 
reviewed such 
statements, the 
appropriate form of 
report to be issued must 
include a(n) 
a. Regulation   S­X
exemption. 
b. Report   on   pro   forma
financial statements. 
c. Unaudited   association
report. 
d. Disclaimer of opinion. 
82. Green, CPA, is aware that Green’s 
name is to be in­cluded in the 
interim report of National 
Company, a publicly held entity. 
National’s quarterly financial 
statements are contained in the 
interim report. Green has not 
audited or reviewed these interim 
financial statements. Green should 
request that 
I.  Green’s name not be included in 
the communication.
AI. The financial statements
be marked as unaudited
with   a   notation   that  no
opinion is expressed on
them. 
MODULE 5   REPORTING 171
accordance with
standards 
83. The objective of a review  established by 
of interim financial  the Securities 
informa­tion of a public  and Exchange 
entity (issuer) is to  Commission. 
provide an accountant  d. Accountant
with a basis for reporting  obtained
whether  corroborating
a. Material evidence   to   de­
modifications termine   whether
should   be   made material
to   conform   with modifications are
generally needed   for   such
accepted information   to
accounting conform   with
principles.  GAAP. 
b. A reasonable basis exists 
for expressing an updated  85. A modification of the 
opinion regarding the  CPA’s report on a review
financial statements that  of the interim financial 
were previously audited.  statements of a publicly 
c. Condensed financial  held company would be 
statements or pro forma  necessitated by which of 
finan­cial information  the following? 
should be included in a  a. An uncertainty. 
registra­tion statement.  b. Lack of consistency. 
d. The   financial c. Reference   to   another
statements   are accountant. 
presented   fairly d. Inadequate disclosure. 
in   ac­cordance
with   generally 86. Which of the following 
accepted procedures ordinarily 
accounting prin­ should be applied when 
ciples.  an independent 
accountant conducts a 
84. An independent  re­view of interim 
accountant’s report is  financial information of 
based on a re­view of  a publicly held en­tity? 
interim financial  a. Verify   changes   in   key
information. If this  account balances. 
report is pre­sented in a  b. Read   the
registration statement, a minutes  of the
prospectus should in­ board   of
clude a statement  directors’
clarifying that the  meet­ings. 
a. Accountant’s c. Inspect the open purchase
review   report   is order file. 
not  a part of the d. Perform
reg­istration cut­off   tests
statement   within for   cash
the   meaning   of receipts and
the   Secu­rities dis­
Act of 1933.  bursements.
b. Accountant  87. Which of the following is
assumes no  least likely to be a proce­
responsibility  dure included in an 
to update the  accountant’s review of 
report for  interim financial 
events and  information of a public 
circumstances  entity? 
occurring after  a. Compare
the date of the  disaggregated
report.  revenue data by
c. Accountant’s  month   to   that
review was  of the previous
performed in 
interim period. 
b. Read   available a. Statement that 
minutes   of the interim 
meetings   of financial 
stockhold­ers.  information was 
c. Observe   counting   of examined in 
physical inventory.  accordance with 
d. Inquire   of standards of the 
managemen Public Company
t concerning Accounting 
significant Oversight Board.
journal
b. Statement   that
entries   and
the   interim
other
financial
adjustments
.  information   is
the
88. An accountant’s review responsibility
report   on   interim of   the   entity’s
financial information of shareholders. 
a public entity is most c. Description   of   the
likely to include a  procedures for a review. 
d. Statement that 
a review of 
interim 
financial infor­
mation is less 
in scope than a 
compilation 
con­ducted in 
accordance 
with AICPA 
standards. 
89. An auditor may report on 
condensed financial state­
ments that are derived 
from complete financial 
statements if the 
a. Condensed 
financial 
statements are 
distributed to 
stockholders 
along with the 
complete 
financial 
statements. 
b. Auditor
described   the
additional
procedures
per­formed   on
the   condensed
financial
statements. 
c. Auditor indicates whether 
the information in the 
condensed financial 
statements is fairly stated in
all material respects in 
relation to the complete 
finan­cial statements from 
which it has been derived. 
d. Condensed 
financial 
statements are 
presented in 
comparative 
form with the  accountancy   to
prior year’s  perform   the
condensed  engagement. 
financial 
statements.  92. The financial statements of KCP 
America, a US entity, are prepared 
90. An auditor is engaged to  for inclusion in the consolidated 
report on selected  financial statements of its non­US 
financial data that are  parent. These financial statements 
included in a client­ are prepared in conformity with the 
prepared document con­ accounting principles generally 
taining audited financial  accepted in the parent’s country and
statements. Under these  are for use only in that country. 
circum­stances, the report How may KCP America’s auditor 
on the selected data  re­port on these financial 
statements? 
should 
a. Be   limited   to I.  A US­style report (unmodified).
data   derived AI. A   US­style   report
from   the modified   to   report   on
audited   finan­ the   accounting
cial statements. principles   of   the
parent’s country. 
b. Be   distributed
III.   The report form of the parent’s
only   to   senior country. 
management
and   the   board
of directors. 
c. State   that   the
presentation   is   a
comprehensive
basis   of
accounting   other
than GAAP. 
d. Indicate   that   the   data   are
not fairly stated in all ma­
terial respects. 
91. Before reporting on the 
financial statements of a 
US entity that have been 
prepared in conformity 
with another country’s 
accounting principles, an 
auditor practicing in the 
US should 
a. Understand   the
accounting
principles
generally   ac­
cepted   in   the
other country. 
b. Be   certified   by
the   appropriate
auditing   or
accoun­tancy
board   of   the
other country. 
c. Notify
management
that   the   auditor
is   required   to
disclaim   an
opinion   on   the
financial
statements. 
d. Receive   a
waiver from the
auditor’s   state
board   of
172 MODULE 5 REPORTING

I II III c. Application of accounting principles to specified
a. Yes No No transactions.
b. No Yes No d. Limited use prospective financial statements such
c. Yes No Yes as a financial projection.
d. No Yes Yes 98.   Delta Life Insurance Co. prepares its financial state­
93.   Field is an employee of Gold Enterprises.  Hardy, ments on an accounting basis insurance companies use pur­
CPA, is asked to express an opinion on Field’s profit par­ suant to the rules of a state insurance commission.  If Wall,
ticipation in Gold’s net income.  Hardy may accept this en­ CPA, Delta’s auditor, discovers that the statements are not
gagement only if suitably titled, Wall should
a. Hardy also audits Gold’s complete financial state­ a. Disclose any reservations in an explanatory para­
ments. graph and qualify the opinion.
b. Gold’s financial statements are prepared in confor­ b. Apply to the state insurance commission for an ad­
mity with GAAP. visory opinion.
c. Hardy’s report is available for distribution to c. Issue a special statutory basis report that clearly
Gold’s other employees. disclaims any opinion.
d. Field owns controlling interest in Gold. d. Explain in the notes to the financial statements the
94.   When an auditor reports on financial statements pre­ terminology used.
pared on an entity’s income tax basis, the auditor’s report 99.   A CPA is permitted to accept a separate engagement
should (not in conjunction with an audit of financial statements) to
a. Disclaim an opinion on whether the statements audit an entity’s
were examined in accordance with generally ac­
Schedule of Schedule of
cepted auditing standards. accounts receivable royalties
b. Not express an opinion on whether the statements
a. Yes Yes
are presented in conformity with the comprehen­ b. Yes No
sive basis of accounting used. c. No Yes
c. Include an explanation of how the results of opera­ d. No No
tions differ from the cash receipts and disburse­
ments basis of accounting. 100.  Financial information is presented in a printed form
d. State that the basis of presentation is a comprehen­ that prescribes the wording of the independent auditor’s
sive basis of accounting other than GAAP. report.  The form is not acceptable to the auditor because the
form calls for statements that are inconsistent with the audi­
95.   Helpful Co., a nonprofit entity, prepared its financial tor’s responsibility.  Under these circumstances, the auditor
statements on an accounting basis prescribed by a regulatory most likely would
agency solely for filing with that agency.  Green audited the a. Withdraw from the engagement.
financial statements in accordance with generally accepted b. Reword the form or attach a separate report.
auditing standards and concluded that the financial state­ c. Express a qualified opinion with an explanation.
ments were fairly presented on the prescribed basis.  Green d. Limit distribution of the report to the party who de­
should issue a signed the form.
a. Qualified opinion. 
b. Standard three­paragraph report with reference to 101.  A registration statement filed with the SEC contains
footnote disclosure. the reports of two independent auditors on their audits of
c. Disclaimer of opinion. financial statements for different periods.  The predecessor
d. Special report. auditor who audited the prior period financial statements
generally should obtain a letter of representation from the
96.   An auditor’s special report on financial statements a. Successor independent auditor.
prepared in conformity with the cash basis of accounting b. Client’s audit committee.
should include a separate explanatory paragraph before the c. Principal underwriter.
opinion paragraph that d. Securities and Exchange Commission.
a. Justifies the reasons for departing from generally 
accepted accounting principles. 102.  Which of the following statements is correct concern­
b. States whether the financial statements are fairly ing letters for underwriters, commonly referred to as comfort
presented in conformity with another comprehen­ letters?
sive basis of accounting. a. Letters for underwriters are required by the Securi­
c. Refers to the note to the financial statements that ties Act of 1933 for the initial public sale of regis­
describes the basis of accounting. tered securities.
d. Explains how the results of operations differ from b. Letters for underwriters typically give negative as­
financial statements prepared in conformity with surance on unaudited interim financial information.
generally accepted accounting principles. c. Letters for underwriters usually are included in the
registration statement accompanying a prospectus.
97.   An auditor’s report would be designated a special re­ d. Letters for underwriters ordinarily update auditors’
port when it is issued in connection with opinions on the prior year’s financial statements.
a. Interim financial information of a publicly held
103.  Comfort letters ordinarily are signed by the client’s
company that is subject to a limited review.
b. Compliance with aspects of regulatory require­ a. Independent auditor.
ments related to audited financial statements. b. Underwriter of securities.
MODULE 5   REPORTING 173
supplementary
disclosures. 
c. Audit committee.  c. A   limited
d. Senior management.  opinion   on
pro   forma
104. Comfort letters ordinarily are financial
addressed to  state­ments. 
a. Creditor   financial d. A   disclaimer   on
institutions.  prospective   financial
b. The   client’s   audit statements. 
committee. 
c. The   Securities   and 106. When an independent 
Exchange Commission.  audit report is 
d. Underwriters of securities.  incorporated by reference 
in a SEC registration 
105. When an accountant issues to an  statement, a prospectus 
underwriter a comfort letter  that includes a statement 
containing comments on data that  about the independent 
have not been au­dited, the  accountant’s involvement
underwriter most likely will receive 
should refer to the 
a. Negative   assurance   on
independent accountant 
capsule information. 
as 
b. Positive   assurance   on
a. Auditor   of   the   financial d. No Yes No
reports. 
109. In connection with a 
b. Management’s   designate
proposal to obtain a new 
before the SEC. 
auditclient, a CPA in public 
c. Certified   preparer   of   the practice is asked to prepare a 
report.  report on the application of 
d. Expert   in   auditing   and accounting principles to a specific
accounting.  transaction. The CPA’s report 
107. Which   of   the   following should include a statement that
matters   is   covered   in   a a. The engagement was 
typical comfort letter?  performed in accordance
a. Negative assurance  with Statements on 
concerning whether the  Standards for 
entity’s internal control  Accounting and Review 
procedures operated as  Services. 
designed during the period b. Responsibility   for   the
being audited.  proper   accounting
b. An opinion  treatment   rests   with   the
regarding  preparers   of   the   financial
whether the  statements. 
entity complied  c. The evaluation of the 
with laws and  application of accounting
regulations  principles is hypothetical
under  and may not be used for 
Government Au­ opinion­shopping. 
diting Standards 
and the Single 
Audit Act of 
1984. 
c. Positive 
assurance 
concerning 
whether 
unaudited 
condensed 
financial 
information 
complied with 
generally 
accepted 
accounting 
principles. 
d. An opinion as to
whether the 
audited financial
statements 
comply in form 
with the 
accounting re­
quirements of 
the SEC. 
108. When unaudited 
financial statements are 
presented in comparative
form with audited 
financial statements in a 
document filed with the 
Securities and Exchange 
Commis­sion, such 
statements should be 
Withheld
Marked as until
“unaudited” audited
a. Yes No
b. Yes No
c. No Yes
entity’s   audit
d. The guidance is  committee,
provided for  the continuing
management’s  CPA,   and
use only and  man­agement.
may not be  c. Disclaim any opinion that
communicated  the   hypothetical   applica­
to the prior or  tion   of   accounting
continuing  principles   conforms   with
auditor.  gener­ally   accepted
accounting principles. 
110. In connection with a  d. Notify the entity
proposal to obtain a new  that the report is 
client, an accountant in  for the restricted
public practice is asked to  use of 
prepare a written report on management and
the application of  outside parties 
accounting principles to a  who are aware 
spe­cific transaction. The  of all relevant 
accountant’s report should facts. 
include a statement that 
a. Any difference 112. Which of the following statements 
in   the   facts, is not included in an accountant’s 
circumstances, report on the application of 
or   as­ accounting prin­ciples? 
sumptions a. The engagement was 
presented   may performed following stan­
change   the dards established by the 
report.  American Institute of Cer­
b. The  tified Public Accountants. 
engagement  b. The   report   is
was performed  based   on   a
in accordance  hypothetical
with Statements transaction not
on Standards  involving facts
for Consulting  or
Ser­vices.  circumstances
c. The   guidance   provided   is of   this   par­
for   management   use   only ticular entity. 
and   may  not  be c. The   report   is
communicated to the prior intended
or con­tinuing auditors.  solely   for   the
d. Nothing came  information
to the  and   use   of
accountant’s  specified
attention that  parties. 
caused the  d. Responsibility for
accountant to  the   proper
believe that the  accounting
account­ing  treatment   rests
principles  with the preparers
violated GAAP. of   the   financial
111. Blue, CPA, has been asked to  statements. 
render an opinion on the application 113. Which   of   the   following
of accounting principles to a  services   would   be   most
specific transaction by an entity that likely to be structured as
is audited by another CPA. Blue  an attest engagement? 
may ac­cept this engagement, but  a. Advocating   a   client’s
should  position in tax matter. 
a. Consult   with b. A   consulting
the   continuing engagement   to
CPA   to   obtain develop a new
infor­mation data­base
relevant   to   the system   for  the
transaction.  revenue cycle. 
b. Report   the c. An engagement 
engagement’s to issue a report 
findings to the addressing an 
en­tity’s  report ordinarily may refer 
compliance with to 
requirements of  a. Only the assertion. 
specified laws.  b. Only   the
d. The   compilation   of   a subject
client’s   forecast matter   to
information.  which   the
assertion
114. An   unqualified   attestation
relates. 
174 MODULE 5   REPORTING
suitable   only
for   a   limited
c. Either   the number of par­
assertion or the ties. 
subject   matter b. Subject   matter   available
to   which   the only to specified parties. 
assertion c. A written assertion has not
relates.  been obtained. 
d. Neither   the d. Criteria   developed   by   an
assertion industry association. 
nor   the
subject 120. Which of the following is least likely
matter   to to   be   included   in   an   agreed­upon
which   the procedures   attestation   engagement
assertion report? 
relates.  a. The   specified
party   takes
115. A practitioner is issuing a  responsibility for
standard unqualified  the   suf­ficiency
exami­nation report under of procedures. 
the attestation standards.  b. Use   of   the   report   is
The CPA’s conclusion  restricted. 
may be on  c. Limited   assurance   on   the
Management’s information presented. 
Subject matter written assertion d. A summary of procedures
a. Yes Yes performed. 
b. Yes No
c. No Yes 121. A   summary   of   findings
d. No No rather  than  assurance  is
most   likely   to   be
116. Conditions exist that result in a included in
material deviationfrom the criteria  a. Agreed­upon   procedures
against which the subject matter  report. 
was evalu­ated during an  b. Compilation report. 
examination. The CPA’s conclusion c. Examination report. 
may be on d. Review report. 
122. Which of the following is
Subject matter Written assertion
a. Yes not correct concerning 
b. Yes “specified parties” of an 
c. No agreed­upon procedures 
d. No report un­der either the 
auditing or attestation 
117. When   performing   an standards? 
attestation   engagement,
which of the following is
least likely to be present? 
a. Assertion. 
b. Practitioner independence.
c. Subject matter. 
d. Suitable criteria. 
118. Suitable   criteria   in   an
attestation   engagement
may be available 
Publicly In CPA’s report
a. Yes Yes
b. Yes No
c. No Yes
d. No No
119. Which   of   the
following   is  least
likely   to   result   in   a
restricted   use   attest
report? 
a. Criteria
prospective
a. They   must financial
agree   on statements   are
the also   ex­
procedures amined. 
to   be   per­ c. Responsibility
formed.  for   the
b. They must take adequacy of the
responsibility procedures
for   the performed   is
adequacy of the taken   by   the
procedures accountant. 
performed.  d. Negative
c. They   must   sign   an assurance   is
engagement letter.  expressed on the
d. After prospective
completion   of financial
the   engagement, statements   taken
another   party as a whole. 
may be added as
a specified user.  125. An   accountant’s
compilation report on
123. When an accountant  a   financial   forecast
examines projected  should   include   a
financial statements, the  statement that 
accountant’s report should a. The   forecast
include a separate  should   be
paragraph that  read   only   in
a. Describes   the conjunction
limitations   on with   the
the   usefulness audited
of   the historical
presentation.  financial
b. Provides   an statements. 
explanation   of b. The accountant expresses 
the   differences only limited assurance on 
between   an the forecasted statements 
examination   and and their assumptions. 
an audit.  c. There   will
c. States   that   the usually   be
accountant   is differences
responsible   for between   the
events   and fore­casted   and
circumstances actual results. 
up   to   one   year d. The
after the report’s hypothetical
date.  assumptions
d. Disclaims   an used   in   the
opinion   on forecast   are
whether   the reasonable   in
assumptions the
provide   a circumstances. 
reasonable
basis   for   the 126. Accepting   an
projection.  engagement to examine
an   entity’s   fi­nancial
124. An accountant may  projection   most   likely
accept an engagement to  would be appropriate if
apply agreed­upon  the   projection   were   to
procedures to prospective be distributed to 
financial statements  a. All   employees   who   work
provided that  for the entity. 
a. Use   of   the b. Potential
report   is stockholders
restricted to the who   request   a
specified   par­ prospectus   or   a
ties.  registration
b. The statement. 
c. A   bank   with
which   the
entity   is
negotiating for
a loan. 
d. All stockholders of record
as of the report date. 
127. A CPA in public practice is required
to comply with the provisions of the
Statements on Standards for 
Attestation Engagements (SSAE) 
when 
Testifying as an procedures to prospective
Compiling a client’s
expert witness in financial projection
financial statements 
128. An accounting and
  accountant’s that presents a
provided that 
auditing matters given
compilation report on hypothetical course ofa. The
stipulated facts
a   financial   forecast Action prospective
a. should Yes   include   a Yes financial
b. statement that the 
Yes No statements   are
c. No Yes also   ex­
a.
d. Compilation does not 
No No
include evaluation of the  amined. 
MODULE 5   REPORTING b. Responsibility175
support of the assumptions
underlying the forecast.  for   the
b. Hypothetical adequacy of the
assumptions procedures
used   in   the performed   is
forecast   are taken   by   the
reasonable.  accountant. 
c. Range of  c. Negative
assumptions  assurance   is
selected is one  expressed on the
in which one  prospective
end of the range  financial
is less likely to  statements   taken
occur than the  as a whole. 
other.  d. Distribution   of
d. Prospective  the   report   is
statements are  restricted to the
limited to  speci­fied
presenting, in  parties. 
the form of a  132. When an accountant 
forecast, 
examines a financial 
information that 
forecast that fails to 
is the ac­
disclose several 
countant’s 
significant assumptions 
representation. 
used to pre­pare the 
129. Which of the following is a  forecast, the accountant 
prospective financial state­ment for  should describe the as­
general use upon which an  sumptions in the 
accountant may appro­priately  accountant’s report and 
report?  issue a(n) 
a. Financial projection.  a. “Except   for”   qualified
b. Partial presentation.  opinion. 
c. Pro   forma   financial b. “Subject   to”   qualified
statement.  opinion. 
d. Financial forecast.  c. Unqualified
opinion with a
130. Given one or more  separate
hypothetical  explanatory
assumptions, a re­ paragraph. 
sponsible party may  d. Adverse opinion. 
prepare, to the best of its 
knowledge and belief, an  133. An accountant’s report
entity’s expected  on   a   review   of   pro
financial position, results  forma   fi­nancial
of operations, and  information   should
changes in financial  include a 
position. Such pro­ a. Statement   that
spective financial  the   entity’s
statements are known as  internal   control
a. Pro   forma   financial was   not   relied
statements.  on in the review.
b. Financial projections.  b. Disclaimer   of
opinion   on   the
c. Partial presentations. 
financial
d. Financial forecasts. 
statements from
131. An accountant may  which   the   pro
accept an engagement to  forma   financial
apply agreed­upon  information   is
derived. 
c. Caveat that it is 134. Which of the following is 
uncertain not an objective of a 
whether   the CPA’s examination of a 
transaction   or client’s management 
event   reflected discussion and analy­sis 
in the pro forma (MD&A) prepared 
financial   infor­ pursuant to Securities and 
mation   will Exchange Commission 
ever occur.  rules and regulations? 
d. Reference   to a. The historical 
the   financial amounts have 
statements been accurately
from which the de­rived, in all 
historical material 
financial respects, from 
information   is the entity’s fi­
derived. 
nancial 
statements. 
b. The 
presentation 
is in 
conformity 
with rules and
regulations 
adopted by 
the Securities 
and Ex­
change 
Commission. 
c. The   underlying
information,
determinations,
esti­mates   and
assumptions   of
the   entity
provide   a   rea­
sonable basis for
the   disclosures
contained herein.
d. The
presentation
includes   the
required
elements   of
MD&A. 
135. Which of the following
is   an   assertion
embodied   in
management’s
discussion and analysis
(MD&A)? 
a. Valuation. 
b. Reliability. 
c. Consistency   with   the
financial statements. 
d. Rights and obligations. 
136. Which  of  the   following
statements   is   correct
relating   to   an   auditor’s
review   engagements   on
an entity’s manage­ment
discussion   and   analysis
(MD&A)? 
a. A   review
consists Standards. 
principally   of b. Statements   on
applying Standards   for
analyti­cal Attestation
procedures and Engage­ments.
search   and c. Statements   on   Standards
verification for Trust Engagements 
proce­dures.  d. Statements   on   Auditing
b. The   review Standards. 
report   of   a
public   entity 139. The   WebTrust   seal   of
should   be   re­ assurance relates most directly
stricted   to   the to 
use   of a. Financial   statements
specified maintained on the Internet.
parties.  b. Health care facilities. 
c. No   consideration   of c. Risk   assurance
internal   control   is procedures. 
necessary.  d. Web sites. 
d. The   report
issued   will 140. A   CPA’s   examination
ordinarily report   relating   to   a
include   a WebTrust   engagement
sum­mary   of is   most   likely   to
findings,   but include 
no   negative a. An opinion on whether the
assurance.  site is “hackproof.” 
b. An   opinion
137. Which of the following is a term for  on   whether
an attest engage­ment in which a CPA the   site
assesses a client’s commercial  meets   the
Internet site for predefined criteria  Web­Trust
such as those over online privacy?  criteria. 
a. ElectroNet.  c. Negative   assurance   on
whether   the   site   is
b. EDIFACT.  electroni­cally secure. 
c. TechSafe.  d. No   opinion   or
d. WebTrust. other
138. Trust   Service assurance,   but
a   summary   of
engagements   are
findings
performed   under   the relating   to   the
provisions of  Web site. 
a. Statements   on   Assurance
176 MODULE 5   REPORTING
plied by Modern Co. 
regarding Modern’s 
141. An engagement in which  written assertion about 
a CPA considers security, its compliance with 
availability, processing  contractual requirements
integrity, online privacy,  to pay royalties. Mill’s 
and/or confidentiality  report on these agreed­
over any type of defined  upon procedures should 
electronic system is most  contain a(n) 
likely to considered  a. Disclaimer   of   opinion
which of the following  about the fair presentation
types of engagements?  of   Modern’s   financial
a. Internal   control   over statements. 
financial reporting.  b. List   of   the
b. SysTrust.  procedures
c. Web siteAssociate.  performed
d. WebTrust.  (or
reference
142. A client’s refusal to  thereto) and
provide a written  Mill’s
assertion in a Trust  findings. 
Services engagement is  c. Opinion   about
most likely to result in  the   effectiveness
which of the following  of   Modern’s
types of opinions?  inter­nal   control
a. Adverse.  activities
b. Disclaimer.  concerning
c. Qualified.  royalty
d. Unqualified   with payments. 
explanatory language. 
143. Which of the following 
types of engagements is 
not permitted under the 
professional standards 
for reporting on an 
entity’s compliance? 
a. Agreed­upon
procedures   on
compliance
with   the
specified
requirements
of a law. 
b. Agreed­upon
procedures   on
the   effectiveness
of   in­ternal
control   over
compliance   with
a law. 
c. Review   on
compliance   with
specified
requirements   of
a law. 
d. Examination   on
compliance   with
specified
require­ments   of
a law. 
144. Mill, CPA, was engaged 
by a group of royalty 
recipi­ents to apply 
agreed­upon procedures 
to financial data sup­
agencies. 
147. In auditing a not­for­profit c. Have   a   direct
entity that receives  and   material
govern­mental financial  effect   on   the
assistance, the auditor has  amounts   in   the
a responsibility to  organization’s
a. Issue a separate report that financial
describes the expected  statements. 
benefits and related costs  d. Demonstrate the existence
of the auditor’s suggested  of material weaknesses. 
changes to the entity’s 
149. A governmental audit may extend 
internal control. 
beyond an examina­tion leading to 
b. Assess whether
the expression of an opinion on the 
management 
fairness of financial presentation to 
has identified 
include 
laws and 
regulations that
have a direct 
and material ef­
fect on the 
entity’s 
financial 
statements. 
c. Notify   the
governmental
agency
providing the fi­
nancial
assistance   that
the audit is  not
designed   to
provide   any
assurance   of
detecting
misstatements
and fraud. 
d. Render an 
opinion 
concerning the 
entity’s contin­
ued eligibility 
for the 
governmental 
financial as­
sistance. 
148. Hill, CPA, is auditing the financial 
statements of Help­ing Hand, a not­
for­profit organization that receives 
finan­cial assistance from 
governmental agencies. To detect 
mis­statements in Helping Hand’s 
financial statements resulting from 
violations of laws and regulations, 
Hill should focus on violations that 
a. Could   result   in
criminal
prosecution
against   the   or­
ganization. 
b. Involve 
significant 
deficiencies to 
be communi­
cated to the 
organization’s 
trustees and the 
funding 
Program Economy &
results Compliance efficiency
a. Yes Yes No
b. Yes Yes Yes
c. No Yes Yes
d. Yes No Yes
2. Acknowledgment that the sufficiency of the proce­ 150.  When auditing an
entity’s financial statements in ac­
dures is solely Mill’s  cordance with Government 
responsibility. Auditing Standards (the 
“YellowBook”), an auditor is 
145. A CPA’s report on  required to report on
agreed­upon procedures 
related to an entity’s  I. Noteworthy   accomplishments
compliance with  of the program. 
specified requirements  II.       The   scope   of   the   auditor’s
should contain  testing of internal controls. 
a. A statement of limitations a. I only. 
on the use of the report.  b. II only. 
b. An opinion about whether c. Both I and II. 
management’s assertion is d. Neither I nor II. 
fairly stated. 
c. Negative 151. When auditing an entity’s 
assurance   that financial statements in ac­cordance 
control   risk with Government Auditing 
has   not   been Standards (the “YellowBook”), an 
assessed.  auditor is required to report on
d. An   acknowledgment   of
I.  Recommendations for actions to 
responsibility for the suffi­
improve operations.
ciency of the procedures. 
AI. The   scope   of   the
146. When reporting on an  auditor’s   tests   of
examination of a  compliance   with   laws
company’s compliance  and regulations. 
with requirements of  a. I only. 
specified laws, the practi­ b. II only. 
tioner has identified an  c. Both I and II. 
instance of material  d. Neither I nor II. 
noncompliance. 
Management has agreed 
to include this instance in 
its writ­ten assertion. The 
examination report should
include 
a. No modification from the
standard form. 
b. An   opinion
paragraph that
is  unqualified,
and   an
explanatory
paragraph. 
c. A   qualified   or   adverse
opinion. 
d. A disclaimer of opinion. 
MODULE 5   REPORTING 177
c. An   auditor
should   report
152. Which   of   the   following the   views   of
statements   is   a   standard responsible
appli­cable   to   financial officials
statement   audits   in concerning   the
accordance   with   Gov­ auditor’s
ernment   Auditing findings. 
Standards   (the   “Yellow d. Internal   control
Book”)?  activities
a. An   auditor designed   to
should   report detect   or   pre­
on the scope of vent   fraud
the   audi­tor’s should   be
testing   of reported   to   the
compliance inspector   gen­
with   laws   and eral. 
regula­tions. 
b. An auditor  154. In reporting under 
should assess  Government Auditing 
whether the  Standards, an auditor most
entity has re­ likely would be required 
portable  to report a falsifica­tion of
measures of  accounting records 
economy and  directly to a federal 
efficiency that  inspector general when the
are valid and  falsification is 
reliable.  a. Discovered after
c. An   auditor the auditor’s 
should   report report has been 
recommendatio made available 
ns   for   ac­tions to the federal 
to   correct inspector general
problems   and and to the 
improve public. 
operations.  b. Reported   by   the
d. An   auditor auditor   to   the
should audit   committee
determine   the as   a   significant
extent   to deficiency   in
which   the internal control. 
entity’s c. Voluntarily
programs disclosed   to
achieve   the the auditor by
desired results. low­level
personnel as a
153. Which   of   the   following result   of   the
statements   is   a   standard auditor’s
appli­cable   to   financial inquiries. 
statement   audits   in d. Communicate
accordance   with   Gov­ d   by   the
ernment   Auditing auditor  to  the
Standards   (the   “Yellow auditee   and
Book”)?  the   auditee
a. An   auditor fails   to   make
should   report a   required
on the scope of report   of   the
the   audi­tor’s matter. 
testing   of
internal 155. Although the scope of 
controls.  audits of recipients of 
b. All   instances   of federal financial 
abuse, waste, and assistance in accordance 
mismanagement with federal audit regula­
should   be tions varies, these audits 
reported   to   the generally have which of 
audit committee.  the fol­lowing elements 
in common? 
a. The   auditor   is b. To   specific
to   determine legislative   and
whether   the regulatory
federal   fi­ bodies   when
nancial reporting   under
assistance   has Government
been Auditing
administered   in Standards. 
accor­dance c. To a court­
with   applicable appointed 
laws   and creditors’ 
regulations.  committee when
b. The materiality levels are  the client is 
lower and are determined  operating under 
by the government entities Chapter 11 of 
that provided the fed­eral  the Fed­eral 
financial assistance to the  Bankruptcy 
recipient.  Code. 
c. The auditor should obtain  d. To shareholders 
written management rep­ with significant 
resentations that the  influence (more 
recipient’s internal  than 20% equity 
auditors will report their 
ownership) when
findings objectively 
significant defi­
without fear of political 
repercussion.  ciencies are 
d. The auditor is  deemed to be 
required to  material 
express both  weaknesses. 
positive and  157. Wolf is auditing an entity’s 
negative 
compliance with require­ments 
assurance that 
governing a major federal financial 
illegal acts that 
assistance pro­gram in accordance 
could have a 
with Government Auditing 
material effect 
Standards. Wolf detected 
on the recipient’s
noncompliance with requirements 
financial state­
that have a material effect on the 
ments are 
program. Wolf’s report on 
disclosed to the 
compliance should express 
inspector 
a. No   assurance   on   the
general. 
compliance tests. 
156. An   auditor   most   likely b. Reasonable   assurance   on
would be responsible for the compliance tests. 
com­municating c. A   qualified   or   adverse
significant deficiencies in opinion. 
the   design   of   internal d. An   adverse   or   disclaimer
control  of opinion. 
a. To the Securities
and   Exchange 158. Which of the following 
Commission is a specific 
when   the   client documentation 
is   a   publicly requirement that an 
held entity.  auditor should follow 
when auditing in 
accordance with 
Government Auditing 
Standards? 
a. The auditor 
should obtain 
written 
representations 
from 
management 
acknowledging 
responsibility for 
correcting 
instances of  the   auditor
fraud, abuse, and  does  not
waste.  possess
b. Before the report legal   skills
is   issued, and  cannot
evidence   of make   legal
supervisory judgments. 
review   of   the
audit.  160. In reporting under 
c. The   auditor Government Auditing 
should Standards, an auditor most
document   the likely would be required 
procedures   that to communicate man­
assure agement’s 
discovery of all misappropriation of assets 
illegal   acts   and directly to a federal 
contingent inspector general when the
liabilities fraudulent activities are 
resulting   from a. Concealed   by
noncompliance. management by
d. The auditor’s  circumventing
working papers spe­cific
should contain  internal
a ca­veat that  controls
all instances of  designed   to
material  safeguard   those
misstatements  assets. 
and fraud may  b. Reported to 
not be  the entity’s 
governing 
identified. 
body and the 
159. In performing a  governing 
financial statement audit body fails to 
in accor­dance with  make a 
Government Auditing  required report
Standards, an auditor is  to the federal 
required to report on the  inspector 
entity’s compliance with general. 
laws and regulations.  c. Accompanied   by
This report should  fraudulent
a. State   that financial
compliance reporting   that
with   laws   and results in material
regulations   is misstatements   of
the asset balances. 
responsibility d. Perpetrated   by
of   the   entity’s several   levels
management.  of management
b. Describe   the in   a   scheme
laws   and that is likely to
regulations continue   in
that the entity
future years. 
must   comply
with.  161. In auditing compliance 
c. Provide   an with requirements 
opinion   on governing major federal 
overall financial assistance 
compliance programs under the Single 
with   laws Audit Act, the auditor’s 
and
consideration of materiality
regulations. 
differs 
d. Indicate that
178 MODULE 5   REPORTING

from materiality under generally accepted auditing stan­
dards. Under the Single Audit Act, materiality is
a. Calculated in relation to the financial
statements taken as a whole. 
b. Determined separately for each major
federal fi­nancial assistance program. 
c. Decided   in   conjunction   with   the
auditor’s risk as­sessment. 
d. Ignored, because all account balances,
regardless of size, are fully tested. 
162. Kent is auditing an entity’s compliance with 
require­ments governing a major federal financial 
assistance pro­gram in accordance with the Single 
Audit Act. Kent de­tected noncompliance with 
requirements that have a material effect on that 
program. Kent’s report on compliance should 
express a(n) 
a. Unqualified   opinion   with   a   separate
explanatory paragraph. 
b. Qualified opinion or an adverse opinion. 
c. Adverse opinion or a disclaimer of opinion. 
d. Limited assurance on the items tested. 
163. When   performing   an   audit   of   a   city   that   is
subject   to   the   requirements   of   the   Uniform
Single   Audit   Act   of   1984,   an   auditor   should
adhere to 
a. Governmental   Accounting   Standards
Board Gen­eral Standards. 
b. Governmental Finance Officers 
Association Gov­ernmental 
Accounting, Auditing, and Financial 
Reporting Principles. 
c. General   Accounting   Office
Government Auditing Standards.
d. Securities   and   Exchange   Commission
RegulationS­X. 
164. A CPA has performed an examination of the 
general­purpose financial statements of Big City. 
The examination scope included the additional 
requirements of the Single Audit Act. When 
reporting on Big City’s internal account­ing and 
administrative controls used in administering a 
fed­eral financial assistance program, the CPA 
should 
a. Communicate   those   weaknesses   that
are material in relation to the general­
purpose financial state­ments. 
b. Express an opinion on the systems used 
to admin­ister major federal financial 
assistance programs and express 
negative assurance on the systems used 
to administer nonmajor federal financial
as­sistance programs. 
c. Communicate   those   weaknesses   that
are   material  in   relation   to  the  federal
financial assistance pro­gram. 
d. Express negative assurance on the 
systems used to administer major federal
financial assistance pro­grams and 
express no opinion on the systems used 
to administer nonmajor federal financial 
assistance programs. 
MODULE 5   REPORTING 179

SIMULATIONS

Task­Based Simulation 1
Accounting
Changes
Authoritative

Literature Help
The audit of Park Publishing Co., for the year ended September 30, 20X2,
is near completion. Your senior, Dave Moore, at Tyler & Tyler CPAs, has 
asked you to draft the audit report, considering the following:
• During fiscal year 20X2, Park changed its depreciation 
method. The engagement partner concurred with this 
change in accounting principle and its justification, and 
Moore wants it properly reflected in the auditors’ report; 
the change is dis­cussed in Note 7 to the financial 
statements. 
• The 20X2 financial statements are affected by an 
uncertainty concerning a lawsuit over patent infringement, 
the outcome of which cannot presently be estimated. Moore
has suggested the need for an explanatory paragraph in the 
auditors’ re­port related to this matter which is discussed in 
Note 4 to the financial statements. 
• The financial statements for the year ended September 30,
20X1,   are   to   be   presented   for   comparative   purposes.
Wilson & Wilson previously audited these statements and
expressed a standard unqualified opinion. 
2. Identify the paragraphs in the Professional Standards that provide
guidance   regarding   the   modification   of   the   audit   report   for   the
change in accounting principle. 
3. Identify the paragraph in Professional Standards that provides the
guidance   regarding   the   modification   of   the   audit   report   for   the
uncertainty. 
4. Identify the paragraphs in the Professional Standards that provide
the guidance regarding the effect of Wilson & Wilson’s opinion on
the 20X1 financial statements on Tyler & Tyler’s audit report on
the 20X2 financial statements. 

Task­Based Simulation 2
Report
Authoritative
Modifications
Literature Help

Assume that items 1 through 8 are situations that Jones, CPA, has 
encountered during his audit of Welles Incorporated.List A represents the 
types of opinions the auditor ordinarily would issue and List B represents the 
report modifications (if any) that would be necessary. For each situation, select
one response from List A and one from List B. Select as the best answers for 
each item the action the auditor would normally take. The types of opinions in 
List A and the report modifications in List B may be selected once, more than 
once, or not at all.
Assume
• The auditor is independent. 
• The auditor previously expressed an unqualified opinion on the prior
year’s financial statements. 
• Only   single­year   (not   comparative)   statements   are   presented   for   the
current year. 
• The conditions for an unqualified opinion exist unless contradicted by
the facts. 
• The conditions stated in the items to be answered are material, unless
otherwise indicated. 
• Each item to be answered is independent of the others. 
• No report modifications are to be made except in response to the factual
situation. 
• The auditor will not treat a situation as an “emphasis of a matter” in 
what remains an unqualified audit report unless it is one of those 
circumstances specifically illustrated in the Professional Standards as an
example of a matter an auditor may wish to emphasize. 
180 REPORTING
MODULE 5

List B
List A Report modifications
Types of opinions H. Describe   the
A. Either   an   “except   for” circumstances   in   an
qualified   opinion   or   an explanatory   para­graph
adverse opinion  without   modifying  the
B. Either   a   disclaimer   of three   standard   para­
opinion   or   an   “except graphs. 
for” qualified opinion  I. Describe   the
C. Either   an   adverse circumstances   in   an
opinion or a disclaimer explanatory   para­graph
of opinion  and  modify  the  opinion
D. An   “except   for”   qualified paragraph. 
opinion  J. Describe the 
E. An unqualified opinion  circumstances in an 
F. An adverse opinion  explanatory para­graph 
G. A disclaimer of opinion  and modify the scope and
opinion para­graphs. 
K. Describe the 
circumstances in an 
explanatory para­graph 
and modify the 
introductory, scope, and
opinion paragraphs.
L. Describe   the
circumstances   within   the
scope  para­graph   without
adding   an   explanatory
paragraph. 
M. Describe   the   circumstances
within   the  opinion  para­graph
without   adding   an   explanatory
paragraph. 
N. Describe the 
circumstances within the 
scope and opinion 
paragraphs without 
adding an 
explanatoryparagraph. 
O. Describe the circumstances
within the introduc­tory, 
scope, and opinion 
paragraphs without 
addingan explanatory 
paragraph. 
P. Issue   the   standard   auditor’s
report  without   modifi­
cation. 
Types of Report
opinions modifications
(A­G) (H­P)
1. Jones hired an actuary to assist in corroborating Welles’ complex 
pension calcula­tions concerning accrued pension liabilities that 
account for 35% of the client’s total liabilities. The actuary’s findings
are reasonably close to Welles’ calculations and support the financial 
statements. 
2. Welles holds a note receivable consisting of principal and accrued 
interest payable in 20X4. The note’s maker recently filed a 
voluntary bankruptcy petition, but Welles failed to reduce the 
recorded value of the note to its net realizable value, which is 
approximately 20% of the recorded amount. 
3. Jones was engaged to audit a client’s financial statements after the annual 
physical inventory count. The accounting records were not sufficiently 
reliable to enable him to become satisfied as to the year­end inventory 
balances. 
4. Jones found an immaterial adjustment relating to inventory. Welles
has   refused   to   adjust   the   financial   statements   to   reflect   this
immaterial item. 
5. Welles’ financial statements do not disclose certain long­term lease
obligations.   Jones   determined   that   the   omitted   disclosures   are
required by FASB. 
6. Jones decided not to take responsibility for the work of another CPA
who audited a wholly owned subsidiary of Welles. The total assets
and revenues of the subsidiary represent 27% and 28%, respectively,
of the related consolidated totals. 
7. Welles changed its method of accounting for the cost of 
inventories from FIFO to LIFO. Jones concurs with the change 
although it has a material effect on the comparability of the 
financial statements. 
8. Due to losses and adverse key financial ratios, Jones has substantial 
doubt about Welles’ ability to continue as a going concern for a 
reasonable period of time. The client has adequately disclosed its 
financial difficulties in a note to its financial statements, which do 
not include any adjustments that might result from the outcome of 
this uncertainty. Also, Jones has ruled out the use of a disclaimer of 
opinion. 
MODULE 5   REPORTING 181

Task­Based Simulation 3
Research
Authoritative
Literature Help
Contacting a Predecessor Auditor
You recently graduated from college and have worked with Tice & 
Co. CPAs for several months. A partner in Tice has indicated that the firm 
has a potential nonpublic new audit client and that he would like to 
research the matters that are ordinarily addressed by a successor auditor’s 
inquiry of the predecessor auditor prior to accepting a new engagement.
Selections
A. AU
B. PCAOB
C. AT
D. AR
E. ET
F. BL
G. CS
H. QC

(A) (B) (C) (D) (E)  (F) (G) (H)
1. Which title of the Professional Standards addresses this issue and will be
helpful in responding to the partner?
2. Enter the exact section and paragraph with the appropriate guidance.

Task­Based Simulation 4

Research
Authoritative
Literature Help
Standard Report Elements

The senior on your job has pointed out to you that the CPA firm
always signs the report both with its signature and its location (Yuma,
Arizona)   and   that   she   suggests   that   she   thinks   it   isn’t   necessary   to
provide the location.
1. Identify the title, section, and paragraph of the auditing standards
that   provide   the   basic   elements   that   must   be   included   in   the
auditor’s standard report. 

2. Is she correct or incorrect concerning inclusion of the location?
Yes No

Task­Based Simulation 5
Audit Report
Authoritative
Details Literature Help

On September 30, 20X2, White & Co. CPAs was engaged to audit the 
consolidated financial statements of National Motors Inc. for the year ended 
December 31, 20X2. The consolidated financial statements of National had not 
been audited the prior year. National’s inadequate inventory records precluded 
White from forming an opinion as to the proper application of generally 
accepted accounting principles to inventory balances on January 1, 20X2. 
Therefore, White decided not to express an opinion on the results of operations 
for the year ended December 31, 20X2. National decided not to present 
comparative finan­cial statements.
Rapid Parts Company, a consolidated subsidiary of National, was audited 
for the year ended December 31, 20X2, by Green & Co. CPAs. Green 
completed its audit procedures on February 28, 20X3, and submitted an 
unqualified opinion on Rapid’s financial statements on March 7, 20X3. Rapid’s
statements reflect total assets and revenues constituting $700,000 and 
$2,000,000, respectively, of the consolidated totals of National. White decided 
not to assume responsibility for the work of Green. Green’s report on Rapid 
does not accompany National’s consolidated statements.
The following lists potential effects on White’s audit report.
182 MODULE 5   REPORTING

Effect on the Audit Report
A. Disclaim on balance sheet, income statement and statement of cash flows. 
B. Disclaim only on results of operations. 
C. Qualified audit opinion on results of operations. 
D. Modification of report to set forth a division of responsibility for the audit. 
E. Issuance of two separate reports. 
F. Date the report March 28, 20X3. 
G. Date the report April 4, 20X3. 
H. Management of National Motors. 
I. National Motors’ Board of Directors. 
J. Managements of both National Motors and Rapid Parts. 
K. White & Co. only. 
L. White & Co. and Green & Co. 
For each circumstance described below select the most appropriate statement that reflects the effect or details of the 
audit report.

1. N
a
2. R
a
3. W
h
4

5
MODULE 5 REPORTING 183

MULTIPLE­CHOICE ANSWERS

1. c __ __ 35. a __ __ 69. b __ __ 103. a __ __ 137. d __ __


2. a __ __ 36. d __ __ 70. c __ __ 104. d __ __ 138. b __ __
3. b __ __ 37. c __ __ 71. c __ __ 105. a __ __ 139. d __ __
4. c __ __ 38. c __ __ 72. d __ __ 106. d __ __ 140. b __ __
5. d __ __ 39. d __ __ 73. c __ __ 107. d __ __ 141. b __ __
6. c __ __ 40. b __ __ 74. d __ __ 108. a __ __ 142. b __ __
7. c __ __ 41. a __ __ 75. b __ __ 109. b __ __ 143. c __ __
8. c __ __ 42. b __ __ 76. d __ __ 110. a __ __ 144. b __ __
9. d __ __ 43. b __ __ 77. a __ __ 111. a __ __ 145. a __ __
10. b __ __ 44. d __ __ 78. b __ __ 112. b __ __ 146. c __ __
11. d __ __ 45. c __ __ 79. b __ __ 113. c __ __ 147. b __ __
12. d __ __ 46. a __ __ 80. b __ __ 114. c __ __ 148. c __ __
13. c __ __ 47. d __ __ 81. d __ __ 115. a __ __ 149. b __ __
14. a __ __ 48. a __ __ 82 d __ __ 116. b __ __ 150. b __ __
15. a __ __ 49. d __ __ 83. a __ __ 117. a __ __ 151. b __ __
16. c __ __ 50. a __ __ 84. a __ __ 118. a __ __ 152. a __ __
17. a __ __ 51. d __ __ 85. d __ __ 119. d __ __ 153. a __ __
18. c __ __ 52. d __ __ 86. b __ __ 120. c __ __ 154. d __ __
19. d __ __ 53. c __ __ 87. c __ __ 121. a __ __ 155. a __ __
20. a __ __ 54. d __ __ 88. c __ __ 122. c __ __ 156. b __ __
21. b __ __ 55. c __ __ 89. c __ __ 123. a __ __ 157. c __ __
22. d __ __ 56. a __ __ 90. a __ __ 124. a __ __ 158. b __ __
23. d __ __ 57. d __ __ 91. a __ __ 125. c __ __ 159. a __ __
24. a __ __ 58. d __ __ 92. d __ __ 126. c __ __ 160. b __ __
25. c __ __ 59. b __ __ 93. a __ __ 127. c __ __ 161. b __ __
26. a __ __ 60. c __ __ 94. d __ __ 128. a __ __ 162. b __ __
27. b __ __ 61. c __ __ 95. d __ __ 129. d __ __ 163. c __ __
28. a __ __ 62. d __ __ 96. c __ __ 130. b __ __ 164. c __ __
29. b __ __ 63. b __ __ 97. b __ __ 131. d __ __
30. c __ __ 64. b __ __ 98. a __ __ 132. d __ __
31. b __ __ 65. b __ __ 99. a __ __ 133. d __ __
32. d __ __ 66. a __ __ 100. b __ __ 134. b __ __
33. a __ __ 67. d __ __ 101. a __ __ 135. c __ __ 1st: __/164 = __%
34. b __ __ 68. a __ __ 102. b __ __ 136. b __ __ 2nd: __/164 = __%

MULTIPLE­CHOICE ANSWER EXPLANATIONS
(d) is 
A.1.  Overall Issues
1. (c) The requirement is to identify the type of assurance (if 
any) provided in a financial statement compilation report. 
Answer (c) is correct because the objective of a 
compilation is to present in the form of financial statements
information that is the presentation of management 
(owners) without undertaking to express any assurance on 
the statements. Answer (a) is incorrect because accountants
never provide absolute assurance. Answer (b) is incorrect 
because reviews result in limited (negative) assurance. 
Answer (d) is incorrect because audits, not compilations, 
provided reasonable assurance.
2. (a) The requirement is to identify the statement in
the standard audit report that indicates the 
existence of audit risk. Answer (a) is correct 
because the existence of audit risk is recognized 
by the statement in the auditor’s standard report 
that the auditor obtained “reasonable assurance.” 
Answer (b) is incorrect because while the 
standard report does indicate that the CPA 
assesses the accounting princi­ples used and the 
overall financial statement presentation, this does
not indicate the existence of audit risk. Answer 
(c) is incorrect because while the standard report 
does indicate that the audit relates to whether the 
financial statements are free of material 
misstatement, it does not discuss materiality and 
the audit risk associated with materiality. Answer
incorrect because while the financial statements are the re­ level is reported on. Answer (c) is incorrect 
sponsibility of management and the CPA’s responsibility  because regardless of the other type of service 
is to express an opinion, the indication that the CPA  performed, the compilation level is always the 
expresses an opinion does not address audit risk and is less lowest level and therefore should not be the basis
precise than the statement that the auditor obtains  of the re­port. Answer (d) is incorrect because in 
reasonable assur­ance. circumstances in which an audit has been 
performed, an audit report, not a review report, is
3. (b) The requirement is to determine an 
appropriate. 
accountant’s reporting responsibility when more 
than one level of service concerning the financial  4. (c) The requirement is to identify the report that is least likely
statements of a nonissuer (nonpub­lic) entity has  to be a restricted­use report. Answer (c) is cor­rect because 
been performed. Answer (b) is correct be­cause  AU 623 indicates that reports on most compre­hensive basis 
AR 100 requires that the accountant report on the  financial statements are not restricted. An­swers (a) and (b) 
highest level of service rendered. Answer (a) is  are incorrect because reports on significant deficiencies and 
incorrect because the highest, and not the lowest,  reports to audit committees are restricted 
184 MODULE 5   REPORTING
standards and generally accepted ac­
counting principles. Answer (c) is 
under AU 325 and AU 380,  correct because the scope paragraph
respectively. Answer (d) is  indicates that generally accepted 
incorrect because AU 623  auditing stan­dards have been 
restricts such reports on compli­ followed, while the opinion 
ance. Also, see AU 532 for  paragraph indi­cates that the 
information on restricting the use  financial statements follow 
of an audit report. generally accepted accounting 
principles. 
5. (d) The requirement is to 
determine an  9. (d) The requirement is to determine 
auditor’sresponsibility for  auditor report­ing responsibility 
controlling the distribution by the  when prior period financial 
client of a restricted­use report.  statements which received a 
Answer (d) is correct because AU  qualified opinion due to a lack of 
532 states that an auditor is not  adequate disclosure have been 
responsible for controlling the  restated to eliminate the lack of 
distribution of such reports.  disclo­sure. Answer (d) is correct 
Answer (a) is incorrect be­cause  because AU 508 states that an 
while an auditor should consider  auditor should express an 
informing a client that restricted­ unqualified opinion on the restated 
use reports are not intended for  financial statements of the prior 
such distribu­tion, there is no such  year (with an explanatory paragraph
requirement. Answer (b) is  describing the circumstance). 
incorrect because the auditor need  Answer (a) is in­correct because the 
not inform the client to cease and  auditor’s original report is not 
desist. Answer (c) is incorrect  reissued. Answer (b) is incorrect 
because an auditor need not insist  because the qualified opinion is 
that the client not duplicate the  eliminated. Answer (c) is incorrect 
restricted­use report. because reference to the type of 
opinion expressed is included in the 
A.2. Financial Statement Audit 
reissued report’s explanatory 
Reports—Nonissuer paragraph. 
(Nonpublic) Companies
10. (b) The requirement is to 
6. (c) The requirement is to  identify the correct state­
identify the statement that ment concerning an 
is included in the  auditor’s responsibility to 
auditor’s standard report.  express an opinion on the 
Answer (c) is correct  financial statements. 
because the auditor’s  Answer (b) is correct 
standard report states that because the opening 
an audit includes  (introductory) paragraph 
assessing significant  of the audi­tor’s standard 
estimates made by man­ report states that the 
agement; see AU 508 for  auditor’s responsibility is 
this and other required  to express an opinion on 
elements included in a  the financial statements 
standard report.  based on the audit. 
7. (c) The requirement is to Answer (a) is incorrect 
because of the explicit 
determine the proper date
statement in the 
of a reissued audit report
introductory paragraph. 
on   financial   statements
Answers (c) and 
that have 
(d)   are   incorrect   because   the
not been restated. Answer (c) is 
introductory paragraph, not the 
correct because use of theoriginal 
date on the reissued audit report 
removes any impli­cation that 
records, transactions or events after
the date of the audit report have 
been examined or reviewed.
8. (c) The requirement is to identify 
which paragraph of an auditor’s 
standard audit report should refer to 
generally accepted auditing 
the financial statements 
scope or opinion paragraphs,  (e.g., if the count of the 
includes the statement on the  beginning inventory has 
auditor’s responsibility. not been observed, an 
auditor may disclaim an 
11. (d) The requirement is to  opinion on the income 
determine whether theterms “when  statement and yet express 
read in conjunction with Note X,”  an unquali­fied opinion on
and “with the foregoing  the balance sheet). Answer
explanation” should be included in  (b) is incorrect because the
the opinion paragraph of a qualified objective does not relate 
opinion. AU 508 states that an au­ directly to scope limi­
dit report with a qualified opinion  tations. Answer (d) is 
should not include either phrase in  incorrect because an 
the opinion paragraph. auditor may report on only
one statement. 
12. (d) The requirement is to
determine   the 14. (a) The requirement is to 
representa­tions   made determine the meaning 
explicitly   and   implicitly of the expression “taken 
when   issuing   the   stan­ as a whole” in the fourth 
dard   auditor’s  report  on generally accepted 
comparative   financial auditing standard of 
statements.  reporting. AU 508 states 
Answer (d) is correct because the  that “taken as a whole” 
standard audit report ex­plicitly  applies equally to a 
states that the examination of  complete set of fi­nancial
evidence is made on a test basis  statements and to an 
and implicitly assumes consistent  individual financial 
application of accounting  statement. 
principles. Answer (a) is incorrect 
because con­sistency of application 15. (a) The requirement is to 
of accounting principles is not indi­ determine the appropriate 
cated explicitly. Answer (b) is  date for an audit report 
incorrect because examina­tion of  when a subsequent event 
evidence on a test basis is referred  requiring adjustment of 
to explicitly. Answer (c) is  financial statements, 
incorrect because examination of  without disclosure of the 
evidence on a test basis is explicitly event, comes to the 
referred to and because consistent  auditor’s attention. AU 
application of accounting principles 530 states that the date of 
is not explicitly referred to.  completion of fieldwork 
should be used in such 
13. (c) The requirement is to 
circumstances. 
determine the objective of 
the fourth reporting  16. (c) The requirement is to determine 
standard which requires  an auditor’s responsibility for 
either an opin­ion  subsequent events when an audit 
regarding the financial  report has been dual dated for a 
statements taken as a  subsequent event. Answer (c) is cor­
whole or an assertion to  rect because, when dual dating is 
the effect that an opinion  used, auditor responsibility for 
cannot be expressed.  events subsequent to the completion
Answer (c) is correct  of fieldwork is lim­ited to the 
because the standard  specific event referred to in the 
states that the objective is  notes to the finan­cial statement. 
to prevent  Answers (a), (b), and (d) are all 
misinterpretation of the  incorrect because they establish 
degree of responsibility  more responsibility than required by
that the auditor is  the professional standards. Note, 
assuming when his/her  however, that if the auditor chooses 
name is associated with  to date the report as of the date of 
financial statements.  the subsequent event, his/her 
Answer (a) is incorrect  responsibility for other subsequent 
because differing opinions events extends to the date of the 
may be issued on each of  audit report. 
MODULE 5   REPORTING 185
(3) reviewing the work­ing papers 
of the other auditor, and (4) 
A.3. Financial Statement Audit  performing additional audit 
Reports—Issuer procedures. Answer (a) is incorrect 
(Public) Companies because no ex­planatory paragraph 
is added to the audit report. Answer 
17. (a) The requirement is to determine  (b) is incorrect because the principal
the accounting principles that an  auditor is assuming re­sponsibility 
issuer (public) company audit report for the other auditor’s work when a 
refers to. Answer (a) is correct  decision is made not to refer to the 
because the financial statements  other auditor’s report. Answer (c) is 
follow generally accepted  incorrect because written 
accounting principles. Answer (b) is permission is not required when the 
incorrect because, while the audit is  principal auditor is taking 
performed in accor­dance with  responsibility for the work of the 
PCAOB standards, the financial  other auditor.
statements do not follow those 
standards. Answer (c) is incorrect  20. (a) The requirement is to 
because the financial statements do  determine the meaning of 
not follow generally accepted  sentences added to an 
auditing standards. Answer (d) is  introductory paragraph of 
incorrect because the fi­nancial  an auditor’s report that 
statements ordinarily follow  states that another auditor 
generally accepted ac­counting  audited a portion of the 
principles, not International  entity. Answer (a) is 
Accounting Standards.  correct because AU 508 
provides that such a 
18. (c) The requirement is to identify  statement indicates a 
the incorrect state­ment concerning  division of responsibility. 
information included in an audit  An­swer (b) is incorrect 
report of financial statements issued because when the other 
under the requirements of the  auditor is re­ferred to the 
PCAOB. Answer (c) is correct since CPA, the CPA is not 
the report should refer to the  assuming responsibility 
auditor’s report on internal control,  for the other auditor. 
not on compliance with laws and  Answer (c) is incorrect 
regulations. Answer (a) is incorrect  because an unqualified 
because the report should include  opinion may be issued. 
the title “Report of Independent  Answer (d) is incorrect 
Registered Public Accounting  because the sentences are 
Firm.” Answer (b) is incor­rect  proper. 
because the report should refer to 
21. (b) The requirement is to determine 
the standards of the PCAOB. 
Answer (d) is incorrect because the  the situation in which an auditor 
report should contain the city and  would ordinarily issue an unqualified 
state or country of the office that  audit opinion without an explanatory 
issued the report.  paragraph. Answer (b) is correct 
because when an auditor makes 
B.1.a. Opinion Based, in  reference to the re­port of another 
Part, on Report of  auditor, each of the three standard 
Another Auditor paragraphs of the report are modified, 
but no additional paragraph is 
19. (d) The requirement is to 
determine a principal audi­tor’s 
responsibility, in addition to making
inquiries of the other auditor’s 
reputation and independence, after 
having decided not to refer to the 
audit of the other auditor. An­swer 
(d) is correct because when a 
decision is made not to make 
reference to the other audit—that is,
to take responsi­bility for that 
auditor’s work—the principal 
auditor should consider (1) visiting 
the other auditor, (2) reviewing the 
audit programs of the other auditor, 
another CPA who has audited a 
added to the report. Answer (a) is  client’s subsidiary. Answer (d) is 
incorrect because empha­sizing that correct because, regard­less of 
the entity had significant related­ whether the other auditor is referred
party transac­tions is normally  to, the principal auditor must be 
accomplished through the addition  satisfied as to the independence and
of an explanatory paragraph.  profes­sional reputation of the other
Answer (c) is incorrect because the  CPA. Answer (a) is incorrect 
omission of a statement of cash  because the principal auditor need 
flows when an entity issues  not issue an unqualified opinion on 
financial statements that present  the consolidated financial 
financial position and result of  statements. An­ 
operations results in a qualified  swer (b) is incorrect because it is 
audit opinion with an explanatory  not necessary that the other CPA 
paragraph. Answer (d) is incorrect  issue an unqualified opinion on the 
because substantial doubt about the  subsidiary’s finan­cial statements. 
entity’s ability to continue as a  Answer (c) is incorrect because the 
going concern normally results in  princi­pal auditor should consider 
either an unqualified opinion with  reviewing the audit programs and 
an explanatory paragraph or a  working papers of the other CPA 
disclaimer of opinion. when a decision is made to not 
22. (d) The requirement is to  make reference to that CPA. 
identify the situation in  B.1.b. Unusual Circumstances 
which an auditor may issue  Requiring a Departure
the standard audit report. 
from Promulgated 
An­swer (d) is correct 
GAAP
because a standard report 
may be issued in  24. (a) The requirement is to 
circumstances in which the  determine the type ofopinion to be 
principal auditor assumes  issued when financial statements of 
responsibility for the work  an entity that follows GASB 
of another auditor. Answer  standards include a justified 
(a) is incorrect because the  departure from GAAP. Answer (a) 
standard report does not  is correct because the auditor 
include refer­ence to a  should issue an unqualified opinion 
specialist. Thus, reference  and should include a separate 
to a specialist within a  explanatory paragraph explaining 
report by definition causes  the departure from GAAP. Answers
modification of the  (b), (c), and (d) are incorrect 
standard re­port. Answer  because when the auditor believes 
(b) is incorrect because the  that the departure is justified, 
auditor is required to issue  neither a qualified nor adverse 
a modified report on  opinion is appropriate.
condensed financial 
B.1.c. Substantial Doubt 
statements per AU 552. 
about Ability to 
Answer (c) is incorrect 
because audit reports on  Remain a Going 
financial statements  Concern
prepared on a  25. (c) The requirement is to 
comprehensive basis other 
determine whether theterm 
than GAAP are considered  “reasonable period of time, not to 
to be “special reports”  exceed one year” and/or “going 
which require departures  concern” is included in an 
from the standard form.  explanatory para­graph relating to 
23. (d) The requirement is to determine  going concern status. Answer (c) is 
a principal audi­tor’s reporting  correct because while the term 
responsibility when a decision has  “going concern” must be included, 
been made to not make reference to  the first term is not included in such
a report.
186 MODULE 5   REPORTING
reporting responsibility 
when there is substantial 
26. (a) The requirement is to  doubt about a client’s 
determine an auditor’s  ability to continue as a 
reporting responsibility  going concern. An­swer 
when reporting on  (a) is correct because the 
comparative fi­nancial  audit report must include 
statements in which the  an explanatory paragraph 
first year presented origi­ following the opinion 
nally received a going  paragraph, and must use 
concern modification on a  the terms “going concern”
matter that has now been  and “substantial doubt.” 
resolved, thus removing 
the auditor’s substan­tial  29. (b) The requirement is to identify 
doubt. Answer (a) is  the situation in which an 
correct because if  explanatory paragraph may be 
substantial doubt has been added to an un­qualified report. 
removed in the current  Answer (b) is correct because 
period, the explanatory  substantial doubt about the entity’s 
paragraph included in the  ability to continue as a going con­
auditor’s report on the  cern leads to either an unqualified 
financial statements of the report with an explanatory 
prior period should not be  paragraph or a disclaimer of 
repeated. An­swers (b),  opinion. Answer (a) is incor­rect 
(c), and (d) are all  because an auditor may issue an 
incorrect because they  opinion on a balance sheet without 
suggest the need for an  reporting on the other basic 
explanatory paragraph.  financial state­ments. Answer (c) is 
incorrect because unreasonable esti­
27. (b) The requirement is to  mates lead to either a qualified or an
determine the auditor’s  adverse opinion. An­swer (d) is 
responsibility when s/he  incorrect because inadequate 
concludes that there is  management record retention 
substantial doubt about an  policies are a scope limitation that 
entity’s ability to continue may result in a qualified opinion or 
as a going con­cern for a  a disclaimer. 
reasonable period of time. 
Answer (b) is correct  30. (c) The requirement is to identify 
because when the auditor  the management plan an auditor 
concludes there is  would most likely positively 
substantial doubt, s/he  consider when a question 
should consider the  concerning an entity’s ability to 
possible effects on the  continue as a going concern exists. 
finan­cial statements, and  Answer (c) is correct because 
the adequacy of the  increasing the ownership equity will
related disclosures.  bring in funds to possibly overcome 
Answer (a) is incorrect  the negative trends and financial 
because either an  difficulties. Answers (a), (b), and 
unqualified opin­ion with  (d) are all incorrect because 
an explanatory paragraph  increasing dividend distributions, 
or a disclaimer is gener­ reducing lines of credit, and 
ally appropriate, not a  purchasing assets will all use funds, 
qualified or adverse  they will not provide funds. See AU
opinion. An­swer (c) is  341 for guidance on an auditor’s 
incorrect because the  consideration of an entity’s ability 
substantial doubt of going  to continue as a going concern. 
concern status does not  31. (b) The requirement is to 
require adjusting  identify the condition or 
accounting esti­mates.  event most likely to cause 
Answer (d) is incorrect  an auditor to have 
because the prior year’s  substantial doubt about an 
audit report need not be  entity’s ability to continue 
reissued with an  as a going con­cern. 
explanatory paragraph.  Answer (b) is correct 
28. (a) The requirement is to  because denial of usual 
determine an auditor’s  trade from suppliers is 
ordinarily an indicator that
the company is in weak  while such related­party transactions
financial condition.  may be considered risky, there is 
Answer (a) is incorrect  less likely to be a question 
because  concerning going concern status 
than suggested by answer (a). 
Answer (c) is incorrect because the 
payment of such stock dividends 
does not indicate financial 
weakness. Answer (d) is incorrect 
because restrictions on the disposal 
of principal assets is a condition 
often present in various loan 
agreements.
32. (d) The requirement is to identify 
the most likely mitigating factor a 
CPA would consider when a 
client’s ability to continue as a 
going concern is in question. An­
swer (d) is correct because the 
ability to postpone expendi­tures for
research and development projects 
may mitigate the circumstance. See 
AU 341 for this and other 
mitigating factors. Answer (a) is 
incorrect because there is no 
guaran­tee that Zero’s discussions 
with its lenders will lead to a 
restructuring of the debt and loan 
agreements. Only existing or 
committee agreements to restructure
the debt would be considered a 
mitigating factor. Answer (b) is 
incorrect be­cause weak internal 
control over cash disbursements 
may or may not have caused the 
going concern problem. An­ 
swer   (c)   is   incorrect   because   an
entity   with   a   going   concern
problem is unlikely to be able to
purchase   such   production
facilities. 
33. (a) The requirement is to identify 
the condition or event that is most 
likely to cause an auditor to have 
substan­tial doubt about an entity’s 
ability to continue as a going 
concern. Answer (a) is correct 
because AU 341 includes negative 
cash flows as one of its examples of
such condi­tions and events. 
Answer (b) is incorrect because 
while the postponement of research 
and development projects may 
sometimes be due to extreme 
financial difficulties, often it is not. 
Answers (c) and (d) are incorrect 
because neither sig­nificant related­
party transactions nor stock 
dividends need not indicate 
substantial doubt about an entity’s 
ability to continue as a going 
concern. See AU 341 for 
information on an auditor’s 
consideration of an entity’s ability 
to continue as a going concern. 
status. Answer (c) is 
34. (b) The requirement is to  incorrect because rec­
identify the condition or  onciling the cash balances
event that might indicate  with the cutoff bank 
to an auditor substantial  statement is an acceptable 
doubt about an entity’s  audit procedure, but will 
ability to continue as a  not normally identify a 
going concern. Answer  going concern question. 
(b) is correct because  Answer (d) is incorrect 
confirmation with related  because comparing an 
and third parties of the  entity’s depreciation and 
details of arrangements to  asset capitalization 
provide or maintain  policies will not normally 
financial support is a  indicate a question of 
procedure that would  going con­cern status. 
assist an auditor in 
identifying a question  35. (a) The requirement is to 
concerning going con­ identify the audit proce­
cern status. See AU 341  dure most likely to assist 
for this and other such  an auditor in identifying 
conditions and events  conditions and events that 
indicating doubt about an  may indicate there could 
entity’s ability to con­ be substantial doubt about 
tinue as a going concern.  an entity’s ability to 
Answer (a) is incorrect  continue as a going 
because the pledging of  concern. Answer (a) is 
assets as collateral is a  correct because a review 
normal business  of compliance with terms 
transaction and it need not of debt and loan 
necessarily indicate a  agreements may reveal 
question of going concern conditions of 
MODULE 5   REPORTING 187
because it also requires cash 
resources which may not be 
noncompliance due to poor  available. An­swer (d) is incorrect 
financial condition. See the outline  because converting preferred stock 
of AU 341 for a list of procedures  to long­term debt will not generally 
that may identify such conditions  alleviate a question con­cerning an 
and events. Answers (b), (c), and  entity’s ability to continue as a 
(d) are all incorrect because, while  going concern. 
they might in some circumstances 
reveal a question concerning the  B.1.d. Inconsistency in 
company’s ability to con­tinue as a  Application of GAAP
going concern, they are not 
considered to be as effective as  38. (c) The requirement is to identify 
answer (a). the circumstances in which an 
auditor would issue a report that 
36. (d) The requirement is to  omits any ref­erence to consistency.
identify the factor which  Answer (c) is correct because, as 
a CPA would most likely  discussed in AU 508, a change in 
consider as mitigating  the useful life of assets is a change 
substantial doubt about  in estimate, and a change in 
the ability of an entity to  estimate does not result in a 
continue as a going  consistency modification. Answers 
concern. Answer (d) is  (a) and (b) are in­correct because 
correct because  they both represent a change in 
management’s abil­ity to  accounting principle, and a change 
negotiate reductions of  in accounting principle requires a 
required dividends will  consistency modification. Answer 
de­crease required cash  (d) is incorrect because 
outflows, and thereby  management’s lack of reasonable 
increase the like­lihood  justification for a change in 
that the entity will be able accounting principle is a departure 
to continue as a going  from generally ac­cepted accounting
concern. AU 341 provides principles, and the description of the
examples of information  de­parture will discuss the 
that might mitigate such  inconsistency. 
concern. Answers (a), (b),
and (c) are all incorrect  39. (d) The requirement is to 
because they involve  determine whether an un­
spending cash, rather than justified accounting 
reducing outflows of  change, a material 
cash.  weakness in internal 
control, or both, would 
37. (c) The requirement is to  cause an auditor to 
identify the circumstance express an un­qualified 
most likely to mitigate an opinion with an 
auditor’s substantial  explanatory paragraph. 
doubt about an entity’s  An­ 
ability to continue as a  swer (d) is correct because an 
going concern. An­swer  unjustified accounting change will 
(c) is correct because the  result in either a qualified or an 
marketable assets that  adverse opinion and a material 
man­agement intends to  weakness will ordinarily result in 
sell may potentially  no report modifi­cation (see AU 
provide the neces­sary  325 for information on the 
financial resources to  treatment of ma­terial weaknesses); 
mitigate the substantial  accordingly, an unqualified opinion 
doubt about the entity’s  with an explanatory paragraph 
ability to continue as a  added to the auditor’s report is not 
going concern. Answer  appropriate in either case. 
(a) is incorrect because  40. (b) The requirement is to 
the ability to expand 
identify the circumstance 
opera­ 
in which a disclaimer of 
tions into new product lines is a 
suspect circumstance, given the  opinion is not 
substantial doubt about the entity’s  appropriate. An­swer (b) 
ability to continue as a going  is correct because when 
concern. Answer (b) is incorrect  management does not 
pro­ 
vide reasonable justification of a 
change in accounting prin­ciples 
either a qualified or an adverse 
opinion is appropriate, not a 
disclaimer. Answers (a), (c), and 
(d) are all incorrect because they 
represent scope limitations that 
lead to either a qualified opinion or
a disclaimer of opinion.
41. (a) The requirement is to 
determine the effect on an
audit report of a client’s 
decision to use differing 
inventory costing methods
for various portions of its 
inventory. An­swer (a) is 
correct because a standard
unqualified opinion may 
ordinarily be issued (see 
AU 420 for a discussion 
of the consistency 
standard). Answer (b) is 
incorrect because there is 
no lack of consistency 
between accounting 
periods. An­swer (c) is 
incorrect because there is 
no departure from GAAP.
Answer (d) is incorrect 
because adverse opinions 
are only issued when a 
departure from GAAP 
exists that makes the 
financial statements 
misleading. 
42. (b) The requirement is to 
identify an auditor’s re­
porting responsibility when 
performing a first audit of a 
new client and when the 
auditor was able to extend 
auditing pro­cedures to 
gather sufficient evidence 
about consistency. Answer 
(b) is correct because, when
the auditor has obtained 
assurance as to the 
consistency of application 
of accounting principles 
between the current and 
preceding year, no men­tion
of consistency is included 
in the audit report. An­ 
swer (a) is incorrect because the 
auditor may report on the client’s 
income statement. Answer (c) is 
incorrect because the consistency 
standard does apply. Answer (d) is 
incorrect because the auditor does 
not refer to consistency when ac­
counting principles have been 
applied consistently. 
43. (b) The requirement is to determine 
auditor report­ing responsibility 
when management does not provide statement note that 
rea­sonable justification for a  discusses the change in 
change in accounting principle and  detail. Answer (a) is 
presents comparative financial  incorrect because while the 
statements. Answer (b) is correct  auditor must believe that 
because the auditor should continue the change is justified, it is 
to express his/her exception with  not necessary to explain it 
the financial statements for the year in the report. Answer (b) is 
of change as long as they are  incorrect because the 
presented and reported on. Answer  cumulative effect of the 
(a) is incorrect because the auditor 
change need not be 
must express his/her exception for 
described in the audit 
as long as the financial statements 
for the year of change are presented report. Answer (c) is 
and reported on. An­  incorrect because the 
swer (c) is incorrect because the  auditor need not make 
auditor need not qualify the report  explicit concurrence with 
until management changes back to  the change. 
the accounting principle formerly 
45. (c) The requirement is to 
used. Answer (d) is incorrect 
because the qualification is  determine the proper re­
necessary despite the fact that the  porting option for a change 
principle is generally accepted.  in accounting principles 
with an immaterial current 
44. (d) The requirement is to  year effect, but which is 
determine the information  expected to have a 
that must be presented  substantial effect in 
when a client has changed  subsequent years. Answer 
accounting principles.  (c) is cor­rect because the 
Answer (d) is correct  auditor need not recognize 
because in addition to  the change in the audit 
identifying the nature of the report and may issue a 
change, the auditor must  standard unqualified 
refer to the financial  opinion. 
188 MODULE 5   REPORTING
management, the 
successor auditor, or both,
B.1.e. Certain Circumstances  before reissuing the prior 
Affecting Comparative year’s audit record. 
Financial Statements Answer (a) is correct 
because the predecessor 
46. (a) The requirement is to identify  auditor should obtain a 
the circumstance in which an  representa­tion letter from
auditor reporting on comparative  both management (AU 
financial statements would  333) and the succes­sor 
ordinarily change the previously  auditor (AU 508). 
issued opinion on the prior year’s 
financial statements. Answer (a) is  49. (d) The requirement is to identify 
correct because when an auditor has the circumstance in which a 
previously expressed a qualified or  standard unqualified report may be 
an adverse opinion on financial  issued when single­year financial 
statements of a prior period and  statements are presented. Answer 
those financial statements have been (d) is correct because when the prior
re­stated, the auditor’s updated  year’s financial statements are not 
report is changed. Answer (b) is  being presented, the CPA need not 
incorrect because, ordinarily, the  refer to them or include the 
reissued report by a predecessor  predecessor auditor’s report. See 
auditor will be the same as that  AU 508 for information on 
originally is­sued. Answer (c) is  reissuance of a predecessor’s report 
incorrect because the prior year’s  when comparative financial 
opin­ion will remain unqualified if  statements are being issued. An­
the current year’s audit report is  swer (a) is incorrect because 
modified due to a lack of  inability to audit an investment in a 
consistency. Answer (d) is in­ foreign affiliate is a scope limitation
correct because restatement of prior  that is likely to result in either a 
year’s financial state­ments  qualified opinion or a disclaimer. 
following a pooling of interest will  An­ 
not lead to a change in the  swer (b) is incorrect because a 
previously issued opinion.  qualified opinion is appropri­ate 
when an entity declines to present a 
47. (d) The requirement is to  statement of cash flows with its 
determine the information balance sheet and related statements
to be included in an audit  of in­come and retained earnings. 
report on comparative  Answer (c) is incorrect because the 
financial statements when emphasis of an accounting matter 
a predecessor auditor’s  by an auditor results in inclusion of 
report is not being  an explanatory paragraph to the 
reissued. Answer (d) is  unqualified audit report. 
correct because the 
50. (a) The requirement is to 
introductory paragraph of
identify the correct form 
the successor’s report 
of an audit report on 
should indicate (1) that 
comparative financial 
the financial statements 
statements when a 
of the prior period were 
continuing auditor has 
audited by another 
audited the two years of 
auditor (whose name is 
financial statements 
not presented), (2) the 
being presented. Answer 
date of the predecessor’s 
(a) is correct because one 
report, (3) the type of 
audit report should be 
report issued by the 
issued that includes the 
predecessor, and (4) if the
years involved. Answer 
report was other than a 
(b) is incorrect because 
stan­dard report, the 
one report, not two 
substantive reasons 
reports, should be issued. 
therefor. 
Answer (c) is incorrect 
48. (a) The requirement is to  be­cause both years 
determine whether the  should be reported upon. 
predecessor auditor  Answer (d) is 
should obtain a 
representation letter from 
is incor­rect because the 
incorrect because auditors do not  auditor does have some 
have the option of issuing two  responsibility for the 
audit reports in this circumstance. supplementary 
information. Answer (b) 
51. (d) The requirement is to  is incorrect be­cause the 
determine the manner in  auditor must apply 
which a predecessor  limited procedures to 
auditor who has reissued  informa­tion presented 
a report for comparative  and report deficiencies in 
financial statements  the information in 
should refer to the  addition to determining 
succes­sor auditor. AU  whether it has been 
508 indicates that the  omitted. An­swer (d) is 
predecessor auditor  incorrect because tests of 
should not refer in the  details of transactions and
reissued report to the  balances need not be 
report or work of the  performed. 
successor auditor. 
54. (d) The requirement is to 
52. (d) The requirement is to 
determine the proper audit
determine the proper re­ report when management 
porting procedure for  declines to present 
comparative financial  supplementary 
statements for which the  information required by 
prior year is unaudited, and  the Governmental 
the current year is audited.  Accounting Stan­dards 
AU 504 states that when  Board. Answer (d) is 
unaudited financial state­ correct because omission 
ments are presented in  of required supplementary
comparative form with  information, which when 
audited finan­cial  presented is not 
statements, the report on  considered audited, leads 
the prior period may be  to an unqualified opinion 
reis­sued to accompany the  with an explanatory 
current period report. In  paragraph. Answers (a) 
addition, the report on the  and (b) are incorrect 
current period may include  because neither an adverse
a separate para­graph  opinion nor a qualified 
describing responsibility  opinion is appropriate 
assumed for the prior  since the supplementary 
period financial statements. information is not audited.
If these statements are filed  Answer (c) is incorrect 
with the SEC, the  because it is incom­plete 
statements should be  since an unqualified 
clearly marked as “un­ opinion with an additional
audited” but should not be  ex­planatory paragraph is 
referred to in the auditor’s  required. 
report.  B.1.h. Other Information 
B.1.g. Required Supplementary  with Audited 
Information Financial 
Statements
53. (c) The requirement is to 
identify an auditor’s re­ 55. (c) The requirement is to 
sponsibility for required  identify the correct state­ment that 
supplementary  may be included in an auditor’s 
information that is placed report when an auditor provides an 
outside the basic financial opinion on information 
statements. Answer (c) is  accompanying the basic financial 
correct because AU 558  statements. Answer (c) is correct 
requires that the auditor  because the report indicates whether
apply li­mited procedures the accompanying information is 
to the information and  fairly stated in all material respects 
report deficiencies in, or  in relation to the basic financial 
the omission of, the  statements taken as a whole. 
information. Answer (a)  Answer (a) is incor­rect because the
information is not presented in  in addition to that re­quired by 
accordance with generally accepted  generally accepted accounting 
auditing standards. Answer (b) is  principles. An­
incorrect because the information is 
MODULE 5   REPORTING 189
58. (d) The requirement is to 
identify the proper state­ment about 
swer (d) is incorrect because it  an audit report that includes a 
is not in accordance with  separate paragraph in an otherwise 
attestation standards. unmodified report that emphasizes 
that the entity being reported on had
56. (a) The requirement is to identify 
significant transactions with related 
the auditor’s re­porting 
parties. Answer (d) is correct 
responsibility for a material 
because AU 508 al­lows such 
inconsistency between the audited 
emphasis of a matter and states that 
financial statements and the other 
it does not negate the unqualified 
information in an annual report to 
opinion. Answer (a) is incorrect be­
shareholders containing audited 
cause the report is considered 
financial statements. Answer (a) is 
unqualified. Answer (b) is incorrect 
correct because AU 550 states that 
because such emphasis of a matter 
if a material inconsistency exists 
does not violate generally accepted 
and the client refuses to revise the 
auditing standards if this 
other information, the auditor 
information is disclosed in notes to 
should include an explanatory 
the financial statements. Answer (c)
paragraph that explains the 
is incorrect because the report 
inconsistency. The auditor may also
should not include the phrase “with 
withhold the use of the audit report 
or the auditor may withdraw from  the foregoing explanation.”
the engagement. Answer (b) is  B.1.j. Departures from GAAP
incorrect because the financial 
statements are not misstated.  59. (b) The requirement is to 
Answer (c) is incorrect because the  identify the appropriate 
auditor must review the other  types of audit reports when 
information to ensure that it is  an illegal act with a 
consistent with the fi­nancial  material effect on the 
statements. Answer (d) is incorrect  financial statements has not
because the fi­nancial statements  been properly ac­counted 
are not misstated and therefore a  for or disclosed. Answer 
dis­claimer of opinion is  (b) is correct because 
inappropriate.  omission of required 
57. (d) The requirement is to  disclosures, a departure 
from generally accepted 
determine an auditor’s 
responsibility when  accounting principles, leads
audited financial  to either a qualified or an 
statements are pre­sented  adverse opinion. Answer 
in a document containing  (a) is incorrect because a 
other information. An­ dis­claimer of opinion is 
swer (d) is correct because not appropriate when the 
the auditor is required to  auditor knows of such 
read the other information misstatements. Answer (c) 
to determine that it is  is incorrect be­cause 
consistent with the  neither a disclaimer of 
audited financial  opinion nor an unqualified 
statements. Answers (a)  opinion with a separate 
and (c) are incor­rect  explanatory paragraph is 
because no such inquiry,  appropri­ate. Answer (d) is 
analytical procedures, or  incorrect because an 
other substantive auditing  unqualified opinion with a 
procedures are required.  separate explanatory 
Answer (b) is incorrect  paragraph is not 
because, unless the  appropriate. 
information seems 
incorrect or inconsistent  60. (c) The requirement is to
with the audited financial  identify the phrase that an
statements, no ex­ auditor   would   include   in
planatory paragraph needs an   audit   report   with   a
to be added to the  qualified 
auditor’s report. 
B.1.i. Emphasis of a Matter
financial statements that 
opinion because of inadequate  pur­port to present its 
disclosure. AU 508 indicates that  financial position and 
the phrase “except for the omission results of operations but 
of the information discussed in the  omits the statement of 
opinion paragraph” is the proper  cash flows. Answer (b) is 
phrase. Answers (a), (b), and (d)  correct because failure to 
are all incorrect because they are  include a statement of 
phrases not allowed in reports with  cash flows will lead the 
qualified opinions. auditor to qualify the 
opinion. 
61. (c) The requirement is to 
identify the circumstance  64. (b) The requirement is to 
that would most likely  identify the circumstance 
result in an auditor  in which an auditor will 
expressing an adverse  choose between expressing
opinion. Answer (c) is  an “except for” qualified 
correct because  opinion and an adverse 
departures from GAAP,  opinion. An­swer (b) is 
such as inappropriately  correct because omissions 
reporting leases, result in  of required informa­tion, a 
either a qualified or an  departure from generally 
adverse opinion. Answer  accepted accounting princi­
(a) is incorrect because  ples, leads to either a 
client refusal to provide 
qualified or an adverse 
access to minutes is a 
opinion. Answer (a) is 
client imposed scope 
incorrect because a scope 
limitation that will 
limitation such as the 
normally result in a 
failure to observe a client’s
disclaimer of opinion. 
Answer (b) is incorrect  physical inventory leads to 
because weak internal  either a qualified opinion 
control will not in general or a disclaimer of opinion. 
result in an adverse  Answer (c) is incorrect 
opinion; if controls are so because an auditor may 
weak that an audit cannot  issue an unqualified 
effec­tively be completed, opinion on one statement. 
a disclaimer of opinion or Answer (d) is incorrect 
withdrawal may be  because substantial doubt 
appropriate. Answer (d)  about an entity’s ability to 
is incorrect because sub­ continue as a going 
stantial doubt about going concern leads to either an 
concern status results in  unqualified report with 
either an unqualified  explanatory language or a 
opinion with an  disclaimer of opinion. 
explanatory paragraph or 
a dis­claimer of opinion.  65. (b) The requirement is to identify 
the situation in which an auditor 
62. (d) The requirement is to  will ordinarily choose between 
identify the paragraphs of expressing a qualified opinion or an
an audit report that are  adverse opinion. Answer (b) is 
modified when an auditor correct because departures from 
qualifies an opinion  generally accepted ac­counting 
because of inadequate  principles result in either a qualified
disclosure. In addition to  opinion or an adverse opinion—
requiring the inclusion of such lack of disclosure is a 
a separate explanatory  departure from generally accepted 
paragraph, AU 508  accounting principles. Answer (a) is
indicates that only the  incorrect because the inability to 
opinion paragraph should observe the physical in­ventory and 
be modified.  inability to become satisfied about 
its balance represents a scope 
63. (b) The requirement is to  limitation that will result in either a 
determine the appropriate  quali­fied opinion or a disclaimer of
report modification that  opinion. Answer (c) is incor­rect 
results when the  because a change in accounting 
management of a publicly  principles leads to an unqualified 
held company issues  opinion with an explanatory 
paragraph added to the report.  inability to apply 
Answer (d) is incorrect because 
190 MODULE 5   REPORTING
Answer (c) is incorrect 
because an auditor’s 
necessary procedures represents a  emphasis of an unusually 
scope limitation that will result in  important subsequent 
either a qualified opinion or a  event results in a report 
disclaimer of opinion. See AU 508  with an un­qualified 
for information on audit reports. opinion with an 
explanatory paragraph. 
B.1.k.  Scope Limitations
68. (a) The requirement is to 
66. (a) The requirement is to identify 
determine the propriety of 
the type of opin­ion that should be 
including a statement that 
issued on the financial statements 
the current asset portion of
when an auditor has been unable to 
an entity’s balance sheet 
obtain sufficient evidence relating 
was fairly stated in an 
to the consistent application of 
audit report that disclaims 
accounting principles be­tween the 
an opinion on the overall 
current and prior year. Answer (a) is
financial statements. 
correct be­cause the scope 
Answer (a) is correct 
limitation will affect the year’s 
because expressions of 
beginning balances and thereby 
opinion as to certain 
affect the current year’s results of 
identified items in 
op­erations and cash flows. Answer 
financial statements 
(b) is incorrect because the year­end
(referred to as “piecemeal 
balance sheet will be unaffected by 
opinions”) should not be 
the scope limi­tation (any retained 
expressed when the 
earnings misstatement of the 
auditor has disclaimed an 
preceding year will be offset in the 
opinion or has expressed 
current year). Answer (c) is incor­
an ad­verse opinion. Such 
rect because the auditor need not 
opinions tend to 
withdraw in such circum­stances. 
overshadow or contra­dict 
Answer (d) is incorrect because this 
the disclaimer or adverse 
situation repre­sents a scope 
opinion. Answer (b) is 
limitation, and not an uncertainty. 
incor­rect because an 
67. (d) The requirement is to  auditor may report on one 
identify the circumstance  basic financial statement. 
in which an auditor would Answers (c) and (d) are 
not express an unqualified incorrect because pro­
opin­ion. Answer (d) is  viding such assurance is 
correct because an  not appropriate. 
inability to obtain the 
69. (b) The requirement is to identify 
audited financial 
the type of opin­ion that should be 
statements of a 
issued on the balance sheet and the 
consolidated investee 
in­come statement when an auditor 
represents a scope 
did not observe a client’s taking of 
limitation, and a 
the beginning physical inventory 
significant scope limita­
and was unable to become satisfied 
tion results in either a 
about its accuracy by using other 
qualified opinion or a 
auditing procedures. Answer (b) is 
disclaimer of opinion. 
correct because the scope limita­tion
Answer (a) is incorrect 
will not affect the year­end balance 
because a material change 
sheet account bal­ances. However, 
between periods in 
because evidence with respect to the
accounting principles will 
be­ginning inventory is lacking, 
result in an explanatory 
verification of cost of goods sold, an
paragraph being added to 
income statement element, is 
a report with an un­
impossible. Although year­end 
qualified opinion. Answer 
retained earnings will not be 
(b) is incorrect because 
affected, both the current and prior 
the omis­sion of the SEC 
years’ retained earnings statements 
required quarterly 
will be affected (by an offsetting 
financial data, which is 
amount) by the cost of goods sold 
considered “unaudited,” 
misstatement. If no other problems 
results in a report with an 
arise, the auditor will be 
unquali­fied opinion with 
an explanatory paragraph. 
Answer (a) is incorrect 
able to issue an unqualified opinion  because it suggests that 
on the balance sheet and a disclaimer  the scope paragraph is 
on the income statement (and on the  not omitted but that the 
retained earnings statement). Answer  opinion paragraph is 
(a) is incorrect because an  omitted. Answer (b) is 
unqualified opinion may be issued on incorrect because it states
the balance sheet. Answer (c) is  that the opinion 
incorrect because an unqualified  paragraph is omitted. 
opinion may be issued on the balance Answer (c) is incorrect 
sheet with a disclaimer on the in­ be­cause it states that the 
come statement. Answer (d) is  scope paragraph is not 
incorrect because a dis­claimer  omitted. 
should be issued on the income  73. (c) The requirement is to identify the 
statement. information included in the opinion 
70. (c) The requirement is to  paragraph of an auditor’s report that is
determine whether the  qualified due to a major inadequacy in
scope paragraph, opinion  the computerized accounting records. 
paragraph, and/or notes to Answer (c) is correct because the 
the fi­nancial statements  opin­ion paragraph indicates that the 
should refer to an audit  exception is due to the pos­sible 
scope limitation. Answer  effects on the financial statements. 
(c) is correct because the  Answer (a) is in­correct because the 
suggested report  opinion paragraph will not include a 
presented for a scope  reference to client­imposed scope 
limitation includes  limitations. Answer (b) is incorrect 
modification of both the  because no indication of a departure 
scope and opinion  from generally accepted auditing 
paragraphs. In addition, it standards is provided in the opinion 
is not appro­priate for the  para­graph and this situation is not a 
scope of the audit to be  departure from GAAS. An­swer (d) is
explained in a note to the  incorrect because there is no 
financial statements.  indication that there is inadequate 
disclosure of necessary information. 
71. (c) The requirement is to identify a 
CPA’s responsi­bility when asked  74. (d) The requirement is to
to report on only one financial  identify the circumstance
statement. Answer (c) is correct  in which a scope 
because the auditor may accept the  limitation is sufficient to 
engagement because the situation  preclude an un­qualified 
involves limited reporting  opinion. Answer (d) is 
objectives, not a limitation on the  correct because AU 333 
scope of audit procedures. Answers  states that management’s
(a), (b), and (d) are incorrect  refusal to furnish such a 
because the auditor is able to accept written representation 
such an engagement and because  constitutes a limitation 
the auditor is able to apply the  on the scope of an audit 
procedures considered necessary.  sufficient to preclude an 
unqualified opinion. An­
72. (d) The requirement is to
swers (a), (b), and (c) are
determine whether either
all incorrect because 
the scope paragraph, the 
while they represent 
opinion paragraph, or 
scope limitations, they 
both should be deleted 
may sometimes not 
when an auditor is 
result in a report that is 
disclaiming an opinion 
other than unqualified. 
due to a client­imposed 
scope limitation. Answer B.1.l. Lack of Independence
(d) is correct because the
scope paragraph is  75. (b) The requirement is to 
omitted in this situation  identify the situation inwhich an
and the opinion  auditor may not issue a 
paragraph is modified to  qualified opinion. An­
disclaim an opinion. 
MODULE 5   REPORTING 191
the opinion paragraph 
should include a direct 
swer (b) is correct because the  refer­ence to a separate 
auditor who lacks indepen­dence  paragraph disclosing the 
must disclaim an opinion, not  basis for the opinion. 
qualify an opinion. An­swer (a) is  Answer (a) is incorrect 
incorrect because a departure from  because the principal ef­
GAAP will result in either a  fects, if available, should 
qualified opinion or an adverse  be described in a separate
opinion. Answer (c) is incorrect  ex­planatory paragraph, 
because scope limitations result in  and not in the opinion 
either a qualified opinion or a  paragraph. An­swer (c) is
disclaimer of opinion. An­swer (d)  incorrect because while a
is incorrect because a specialist may separate explanatory 
be referred to when an auditor is  paragraph provides a 
issuing a qualified opinion, an  description of the 
adverse opinion, or a disclaimer of  substantive reasons for 
opinion. the adverse opinion, the 
B.2. Report Preparation opinion paragraph does 
not. Answer (d) is 
76. (d) The requirement is to identify  incorrect because neither 
the manner in which an auditor may an uncertainty nor a 
express an opinion on an entity’s  scope limitation leads to 
ac­counts receivable when that  an adverse opinion. 
auditor has disclaimed an opin­ion 
on the financial statements taken as  79. (b) The requirement is to 
a whole. Answer (d) is correct  determine the proper 
because such a report is considered  placement of an explanatory
a “specified elements, accounts, or  paragraph disclosing the 
items report,” and should include  sub­stantive reasons for 
the opinion on the accounts  expressing an adverse 
receivable separately from the dis­ opinion. AU 508 requires 
claimer of opinion on the financial  that such paragraphs 
statement. Answer (a) is incorrect  precede the opinion 
because reason for the disclaimer of paragraph. 
opinion need not be provided. 
Answer (b) is incorrect because  C.1.a.  Unaudited Statements
distribu­tion of such a report is not  80. (b) The requirement is to 
restricted to internal use only.  determine the CPA’s re­
Answer (c) is incorrect because the  sponsibility when s/he assists in 
auditor need not report on the  preparing financial state­ments of 
current asset portion of the entity’s  a publicly held entity, but has not 
balance sheet to issue such a report. 
audited or re­viewed them. Answer
77. (a) The requirement is to determine  (b) is correct because the CPA 
the proper ad­dressee of a report in  must, at a minimum, read the 
a circumstance in which one  financial statements for obvious 
company has hired a CPA to audit  material misstatements. Answer 
another company’s financial state­ (a) is incorrect because no 
ments. Answer (a) is correct  documentation with respect to 
because while audit reports are  internal control is necessary. 
ordinarily addressed to the company Answer (c) is incorrect because the
whose financial state­ments are  limited scope of proce­dures being 
being audited, when a CPA audits  performed does not allow the CPA
the financial statements of a  to ascertain
company that is not his or her client 
(as is the case here) the report is 
addressed to the company that hired
the CPA. 
78. (b) The requirement is to 
identify the information 
that should be included 
in the opinion paragraph 
of an audit report with an
adverse opinion. Answer 
(b) is correct be­cause 
ac­counting principles. Answer (b) 
whether the financial statements are is incorrect because no updated 
in conformity with gen­erally  opinion is being issued. Answer (c) 
accepted accounting principles.  is incorrect because condensed 
Answer (d) is incor­rect because  statements or pro forma financial 
omitting all required disclosures is  infor­mation are not being 
not expected for a publicly held  considered in this question. An­ 
entity in these circumstances. swer (d) is incorrect because the 
statements may or may not be 
81. (d) The requirement is to 
presented in conformity with 
identify the appropriateform of  generally accepted ac­counting 
report to issue when the CPA is  principles. 
associated with the financial 
statements of a publicly held entity  84. (a) The requirement is to 
but has not audited or reviewed  identify the correct state­
such statements. Answer (d) is  ment with respect to an 
correct because the standards  independent accountant’s 
require the CPA to disclaim an  review report on interim 
opin­ion on the financial statements  financial information 
when the accountant has not audited presented in a regis­tration
or reviewed such statements.  statement. Answer (a) is 
Answer (a) is incorrect because  correct because an 
Regulation S­X exemption is not a  accoun­tant’s review 
form of audit report. Answer (b) is  report is not a part of the 
incorrect because pro forma  registration statement 
informa­tion is not involved.  within the meaning of 
Answer (c) is incorrect because  Section 11 of the 
there is no such report as an  Securities Act of 1933. 
unaudited association report. Answer (b) is incorrect 
because under certain 
82. (d) The requirement is to  condi­tions an accountant 
identify a CPA’s responsi­bility  is required to update the 
when his/her name is to be  report. An­swers (c) and 
included in the interim report of a  (d) are incorrect because 
publicly held entity and the CPA  the prospectus in­cludes 
has not audited or reviewed the  neither a statement that 
interim financial statements.  the review was performed 
Answer (d) is correct because when in accordance with SEC 
an accountant is aware that his/her  standards, nor a statement 
name is to be included in a client­ that the accountant 
prepared written commu­nication  obtained corroborating 
of a public entity containing  evidence. 
financial statements that have not 
been audited or reviewed, he/she  85. (d) The requirement is to 
should re­quest (1) that his/her  determine the circum­
name not be included in the  stances which will lead to 
communi­cation or (2) that the  a modification of an 
statements be marked as unaudited  interim report. Departures 
and note that there is no opinion  from generally accepted 
expressed on them. accounting prin­ciples, 
which include adequate 
C.1.c. Reviewed Interim  disclosure, require 
(Quarterly) Statements modifica­tion of the 
accountant’s report. 
83. (a) The requirement is to identify 
Normally neither an 
the objective of a review of interim  uncer­tainty [answer (a)] 
financial information. Answer (a) is nor a lack of consistency 
cor­rect because AU 722 states that  [answer (b)] would cause 
the objective of a review of interim  a report modification. 
financial information is to provide a Reference to another 
basis for re­porting on whether  accountant [answer (c)] is 
material modification should be  not considered a 
made for such information to  modification. 
conform with generally accepted 
192 MODULE 5   REPORTING
review is less in scope 
than an audit, not than a 
86. (b) The requirement is to identify  compilation. 
the procedure that would ordinarily 
be applied when an accountant  C.1.d.  Condensed Financial 
conducts a review of the interim  Statements
financial information of a publicly 
89. (c) The requirement is to determine 
held entity. Answer (b) is correct 
the circum­stance under which an 
because the accountant will 
auditor may report on condensed 
ordinarily read the minutes of 
financial statements that are derived
meetings of stockholders, the board 
from complete audited financial 
of directors, and committees of the 
statements. Answer (c) is correct 
board of directors to identify actions
because a report may be issued 
that may affect the interim financial 
when the information in the 
in­formation. AU 722 describes the 
condensed finan­cial statements is 
nature of procedures for conducting 
fairly stated in all material respects 
a review of interim financial 
in rela­tion to the financial 
information. An­swers (a), (c), and 
statements. Answer (a) is incorrect 
(d) are all incorrect because they 
because the condensed financial 
represent verification procedures 
statements need not be dis­tributed 
typically beyond the scope of a re­
with the complete financial 
view of interim financial 
statements. Answer (b) is incorrect 
information. 
because the report need not indicate 
87. (c) The requirement is to  the nature of any additional 
identify the least likely  procedures. Answer (d) is incorrect 
procedure to be included  because prior year condensed 
in an accountant’s review  financial information is not 
of in­terim financial  necessary. See AU 552 for 
information of an issuer  information on condensed financial 
(public) entity. An­swer  state­ments. 
(c) is correct because a  90. (a) The requirement is to determine 
review consists  the appropriate response relating to 
principally of performing  selected financial data that are 
analytical procedures and  included in a client’s prepared 
making inquiries, not  document containing audited 
procedures such as  financial statements. Answer (a) is 
observation, inspection,  correct because the selected data 
and confirma­tion.  should be limited to data derived 
Answers (a), (b), and (d)  from the audited financial 
are all incorrect because  statements. Answer (b) is incorrect 
they include review  because distribution of the report 
procedures, as presented  need not be limited to senior 
in AU 722.  management and the board of 
directors. Answer (c) is incorrect 
88. (c) The requirement is to 
because the selected data need not 
identify the most likely 
follow a comprehensive basis of ac­
information included in a
counting other than GAAP. Answer 
review report. Answer 
(d) is incorrect because the report 
(c) is cor­rect because 
will ordinarily state that the selected
AU 722 requires that the 
data are fairly stated in all material 
report include a de­
respects in relation to the 
scription of procedures 
consolidated financial statements. 
performed. Answer (a) is
incorrect because the 
information was 
reviewed, not examined, 
in accordance with 
standards of the PCAOB.
Answer (b) is incorrect 
because the interim 
financial information is 
the responsibility of the 
entity’s management, not
the shareholders. Answer
(d) is incorrect because a 
because the financial statements 
C.1.e. Financial Statements  need not be prepared in conformity 
Prepared for Use in  with GAAP, as other bases of 
Other Countries accounting may be followed. 
Answer (c) is incorrect because the 
91. (a) The requirement is to  report need not be made available 
identify audit reporting  for distribution to other employees. 
requirements when  Answer (d) is incorrect because the 
reporting on financial  individual in the profit participation 
statements of a US entity  plan need not own a controlling 
prepared in accordance  interest in the company. 
with another country’s ac­
counting principles.  94. (d) The requirement is to
Answer (a) is correct  determine the information
because AU 534 states  that should be included in
that the auditor should  an   audit   report   on
understand the accounting  financial   state­ments
principles generally  prepared   on   the   income
accepted in the other  tax   basis   of   accounting.
country. An­swer (b) is  AU 
incorrect because the  623 presents the form of the 
auditor does not have to  report to be issued. 
obtain certification outside Answer (d) is correct 
of the United States.  because AU 623 requires 
Answer (c) is incorrect  that the report indicate 
because the auditor does  that the income tax basis 
of accounting is a 
not have to disclaim an 
comprehensive basis of 
opinion. Answer (d) is 
accounting other than 
incorrect because the 
GAAP. 
auditor does not have to 
receive a waiver from the  95. (d) The requirement is to 
auditor’s State Board of  identify the appropriate 
Accountancy.  type of audit report to be 
issued for a nonprofit 
92. (d) The requirement is to 
entity’s fi­nancial 
determine the appropriate 
statements prepared 
types of reports that may 
be issued when the  following an accounting 
financial state­ments of a  basis prescribed by a 
US subsidiary are  regulatory agency solely for
prepared following the  filing with that agency. 
princi­ples of a non­US  Answer (d) is correct 
parent company’s country because audit reports for 
for inclusion in that parent such financial statements 
company’s non­US  are considered special 
consolidated financial  reports. An­swer (a) is 
state­ments. AU 534  incorrect because an 
allows either a modified  unqualified report may be 
US style report or the  issued if there are no 
report form of the parent’s departures from the 
country. A US style  prescribed basis. The report
unmodi­fied report is not  would not be qualified 
appropriate.  because the financial 
statements were prepared 
C.2.a.  Special Reports using an accounting basis 
93. (a) The requirement is to identify a  pre­scribed by a regulatory 
requirement for a CPA to express an agency. Answer (b) is 
opinion on a profit participation  incorrect be­cause the 
plan relating to an entity’s net  report issued has five 
income. Answer (a) is correct  paragraphs. Answer (c) is 
because if a specified element is, or  incorrect because a 
is based upon, an en­tity’s net  disclaimer of opinion need 
income or stockholders’ equity, the  not be issued. 
CPA should have audited the 
96. (c) The requirement is to identify the 
complete financial statements in 
order to express an opinion on the  disclosure included in a separate 
element. Answer (b) is incorrect  explanatory paragraph of an auditor’s 
special report on financial statements  prepared in conformity 
MODULE 5   REPORTING 193
insurance commission is 
necessary. Answer (c) is 
with the cash basis of accounting.  incorrect because a 
Answer (c) is correct because the  disclaimer of opinion is 
explanatory paragraph refers to the  not appropriate when 
note to the financial statements that known misstatements 
describes the basis of accounting.  exist. Answer (d) is 
AU 623 presents complete details  incorrect because, as 
on such special reports. Answer (a) indicated, more than 
is incorrect because the report need describing the 
not justify the reasons for  terminology is necessary. 
following a basis other than 
generally ac­cepted accounting  99. (a) The requirement is to
principles. Answer (b) is incorrect  determine whether a CPA
be­cause the explanatory paragraph is   permitted   to   accept   an
contains no statement on fair  engagement to audit either
presentation, and because the  a   sched­ule   of   accounts
opinion paragraph states whether  receivable,   a   schedule   of
the presentation is in conformity  royalties, or both. 
with the basis de­scribed in the  Answer (a) is correct because 
appropriate financial statement  auditors may audit “specified 
note. An­swer (d) is incorrect  elements, accounts or items of a 
because no explanation of how the  financial statement,” in­cluding 
re­sults of operations differ from  either a schedule of accounts 
financial statements prepared in  receivable or a sched­ule of 
conformity with generally accepted royalties. Answer (b) is incorrect 
accounting principles is necessary. because an auditor may audit a 
schedule of royalties. Answer (c) is
97. (b) The requirement is to 
incorrect because an auditor may 
identify the example of a  audit a schedule of accounts 
“special report.” AU 623  receiv­able. Answer (d) is incorrect
defines reports on  because an auditor may audit both 
compliance with aspects  a schedule of accounts receivable 
of regulatory  and a schedule of royalties. 
requirements related to 
audited financial  100. (b) The requirement is to identify an
statements as special  auditor’s re­porting responsibility 
reports. [The other types  when a printed form prescribes the 
of special reports include  wording of the independent 
(1) other comprehensive  auditor’s report that will accom­
basis finan­cial  pany it, but that wording is not 
statements, (2) specified  acceptable to the auditor. AU 623 
elements, (3) financial  suggests that the auditor reword the 
presen­tations to comply  report (or attach a separate report) 
with contracts, and (4)  when involved with this type of 
financial informa­tion  “special report.” 
presented in prescribed 
forms.]  C.2.b.  Letters for Underwriters

98. (a) The requirement is to  101.  (a)   The   requirement   is   to


determine the type of re­ determine   a   predecessorauditor’s
port to issue when a client  responsibility   when   the   financial
who uses a comprehensive statements   he   or   she   audited   are
basis of accounting has  being   included   in   an   SEC
not appropriately titled its  registration   state­ment   filing.
financial state­ments.  Answer (a) is correct because AU
Answer (a) is correct  711   requires   that   the   predecessor
because any such  (1)   read   pertinent  portions   of  the
exceptions or reservation  docu­
should be described in an 
explanatory paragraph and
possibly a qualified (or 
adverse) opinion should 
be is­sued. Answer (b) is 
incorrect because no such 
application to the state 
will be included on the pro­
ment, and (2) obtain a letter of  spective financial statements.
representation from the suc­cessor
auditor. 106. (d) The requirement is to 
determine the appropriate
102. (b) The requirement is to identify  reference to an 
the statement that is correct  independent accountant 
concerning letters for underwriters.  in a prospectus (re­lating 
Answer (b) is correct because letters to an SEC registration 
for underwriters typically provide  statement) that includes a
negative assurance on unaudited  statement about his/her 
interim financial informa­tion.  involvement with an 
Answer (a) is incorrect because  independent audit report. 
letters for underwriters are not  AU 711 indicates that the
required by the Securities Act of  independent ac­countant 
1933. Answer (c) is incorrect  is an expert in auditing 
because letters for underwriters are  and accounting. 
not included in registrations 
statements. Answer (d) is incorrect  107. (d) The requirement is to identify 
because auditors’ opinions on the  the information included in a typical
prior year’s financial statement are  comfort letter. Answer (d) is correct
not updated.  because in a comfort letter auditors 
provide an opinion as to whether 
103. (a) The requirement is to the audited financial statements 
determine  who ordinarily comply in form with the accounting
signs   a   comfort   letter. requirements of the SEC. Answer 
Answer   (a)   is   correct (a) is incorrect because negative 
because   a   com­fort  letter assurance concerning whether the 
(also known as letter to an entity’s internal control procedures 
underwriter) is sent by the operated as designed during the 
independent auditor to the period is not provided. Answer (b) 
underwriter.  is incorrect because a comfort letter 
does not include an opinion on 
104. (d) The requirement is to  whether the entity complied with 
identify to whom comfort Government Auditing Standards 
letters are ordinarily  and the Single Audit Act. Answer 
addressed. Answer (d) is  (c) is incorrect because negative, 
correct be­cause comfort  not positive, assurance is provided 
letters, also referred to as  on unaudited condensed financial 
letters for under­writers,  information. 
are ordinarily addressed 
to underwriters.  108. (a) The requirement is to 
determine the proper treat­
105. (a) The requirement is to determine 
ment of unaudited 
the type of opinion or assurance  financial statements 
provided by an accountant who issues presented in com­parative 
a  form with audited 
comfort letter containing  financial statements in a 
comments on data that have not  docu­ment filed with the 
been audited. Answer (a) is correct Securities and Exchange 
because when proce­dures short of  Commission. Answer (a) 
an audit are applied to information  is correct because those 
such as capsule information, a  statements should be 
comfort letter will generally  marked “unaudited,” not 
provide negative assurance.  withheld until they are 
Answer (b) is incorrect because  audited, and not referred 
CPAs do not provide positive  to in the auditor’s report. 
assurance on supplementary 
disclo­sures. Answer (c) is  109. (b) The requirement is to 
incorrect because no “limited  identify the requirement 
opinion” is issued on pro forma or  relating to a CPA’s report 
other information. Answer (d) is  when reporting on the 
incorrect because no disclaimer  application 
194 MODULE 5   REPORTING
action. Answer (b) is incorrect 
because the engagement’s findings 
of accounting principles to a  need not be reported to all of the 
specific transaction. An­swer (b) is  groups listed—the entity’s audit 
correct because AU 625 requires  committee, the continuing CPA, and 
that the report include a statement  manage­ment. Answer (c) is incorrect 
that responsibility for the proper  because the accountant need not 
ac­counting treatment rests with the disclaim an opinion. Answer (d) is 
preparers of the financial incorrect because the report’s 
statements. Answer (a) is incorrect  distribution need not be restricted to 
because the report states that the  management and outside parties who 
engagement was performed in  are aware of all relevant facts. 
accordance with ap­plicable AICPA
standards, not Statements on  112. (b) Answer (b) is correct because 
Standards for Accounting and  AU 625 indicates that an accountant
Review Services. Answer (c) is  should not undertake such an 
incorrect as no such statement about engagement when the report would 
opinion­shopping is included. An­ be based on such a hypothetical 
swer (d) is incorrect because the  transaction. Answers (a), (c), and 
information may be com­municated (d) are all incorrect be­cause they 
to a prior or continuing auditor. include information included in an 
accountant’s report on the 
C.2.c. Application of Accounting
application of accounting principles.
Principles
C.3.a.  Attestation Engagements
110. (a) The requirement is to determine  —General
an auditor’s reporting responsibility
when asked by a prospective client  113. (c) The requirement is to select
to render an opinion on the  the service that ismost likely to be 
application of accounting princi­ structured as an attest engagement. 
ples to a specific transaction.  An­swer (c) is correct because 
Answer (a) is correct because AU  CPAs may provide assurance as to 
625 indicates that the report must  compliance with requirements of 
include a statement that any  specified laws through a variety of 
difference in the facts,  services, including agreed­upon 
circumstances, or assump­tions  procedures en­gagements and 
presented may change the report, as various compliance audits. Answers
well as various other disclosures.  (a)
Answer (b) is incorrect because the  and (b) are incorrect because 
report indicates that the engagement advocating a client’s tax posi­tion 
was performed in accordance with  and consulting on a new database 
AICPA standards, not Statements  system are examples of 
on Standards for Consulting  professional services not typically 
Services. Answer (c) is incorrect  structured as attest services. 
because the report need not indicate Answer (d) is incorrect because 
that the guidance is for management compilations are not a form of 
use only and may not be  attest engagement.
communicated to the prior or con­
tinuing auditors. Answer (d) is 
incorrect because the report does 
not include negative assurance 
(“nothing came to our attention”). 
See AU 625 for performance and 
reporting standards relating to 
reports on the application of 
accounting principles. 
111. (a) The requirement is to determine 
an auditor’s responsibility when 
asked to render an opinion on the 
appli­cation of accounting principles 
to a specific transaction by an entity 
that is audited by another CPA. 
Answer (a) is cor­rect because the 
accountant must consult with the 
continuing CPA to attempt to obtain 
information relevant to the trans­
because it states that the 
114. (c) Answer (c) is correct conclusion may not be 
because   AT   101 upon the subject matter 
indicatesthat   an and may be upon the 
unqualified   may written assertion. Answer 
ordinarily   refer   to   that (d) is incorrect because it 
assertion   or   to   the states that the conclusion 
subject   matter   to   which may not be upon the 
the   assertion   relates. subject matter. 
Answer 
117. (a) The requirement is to 
(a) is incorrect because it suggests 
reporting only on the assertion. 
determine the element that
Answer (b) is incorrect because it 
is least likely to be 
suggests reporting only on the subject present when a 
practitioner performs an 
matter. Answer (d) is incorrect 
attest engagement. 
because it suggests that reporting on 
Answer (a) is correct 
neither the assertion nor the subject 
because while an assertion
matter is appropriate. Note, however, 
is generally present, it is 
that AT 
not ordinarily required. 
101 also states that when a  Answers (b), (c), and (d) 
deviation from the  are all incorrect because 
criteria being reported  practition­er 
upon exits (e.g., a  independence, subject 
material weakness in 
matter, and suitable 
internal control” the 
criteria are all required. 
CPA should report 
directly upon the subject  118. (a) The requirement is to determine 
matter and not upon the  whether suitable criteria in an 
assertion.  attestation engagement may be 
115. (a) The requirement is to  available pub­licly, and/or in the 
identify the correct state­ CPA’s report. Answer (a) is correct 
ment. When a standard  be­cause suitable criteria may be 
unqualified examination  available publicly in the CPA’s 
report is being issued,  report, included with the subject 
that report may be upon  matter or in the as­sertion, well 
the subject matter or the  understood by users (e.g., the 
written assertion.  distance between A and B is twenty 
Answers (b), (c), and (d)  feet) or available only to specified 
are all incor­rect because  parties. Answers (b), (c), and (d) are
they suggest that the  all incorrect because they sug­gest 
report may not be upon  that suitable criteria may not be 
either the subject matter,  available publicly, in the CPA’s 
the written assertion, or  report, or both. 
both.  119. (d) The requirement is to 
116. (b) The requirement is to  identify the situation that 
determine whether a  is least likely to result in 
CPA’s conclusion may be a restricted use attest 
upon the subject matter,  report. An­swer (d) is 
the writ­ten assertion, or  correct because criteria 
both when conditions  developed by an industry 
exist that result in a  association may or may 
material deviation from  not result in a restricted 
the criteria against which  use attest report. Answers
the subject matter was  (a), (b), and (c) always 
evaluated during an  result in a restricted use 
examination. Answer (b)  report. 
is correct because in such  120. (c) The requirement is to 
circumstances the  identify the information 
conclusion should be  that is least likely to be 
directly upon the subject 
included in an agreed­
matter. Answer (a) is 
upon proce­dures 
incorrect because it 
attestation report. 
suggests that the 
Answer (c) is correct 
conclusion may be upon 
because an agreed­upon 
the written assertion. 
procedures report 
Answer (c) is incorrect 
provides a summary of 
pro­cedures performed  specified party does not 
and findings, not limited  take responsibility for the
assurance. An­swer (a) is sufficiency of 
incorrect because the  procedures. An­ 
MODULE 5   REPORTING 195
statements. An­ 
swer (b) is incorrect because the 
swer (b) is incorrect because the report includes no such statement 
report’s use is restricted. Answer attempting to distinguish between 
(d)   is   incorrect   because   a an examination and an audit. 
summary   of   procedures Answer (c) is incorrect because the 
performed is included. report includes no such disclosure 
121. (a) The requirement is to  and because the accountant is not 
identify the type of reportthat is  responsible for events and 
most likely to include a summary  circumstances up to one yearafter 
of findings rather than assurance.  the report’s date. Answer (d) is 
Answer (a) is correct because  incorrect because the report 
agreed­upon procedures reports  suggests that the assumptions do 
include a summary of findings. An­ provide a reasonable basis. 
swer (b) is incorrect because a  124. (a) The requirement is to 
compilation report does not provide identify the circumstancein 
a summary of findings. Answer (c)  which an accountant may 
is incorrect be­cause an  accept an engagement to 
examination report includes  apply agreed­upon 
positive assurance and not a  procedures to prospective 
summary of findings. Answer (d) is financial statements. 
incorrect because a review report  Answer (a) is correct 
includes limited (negative) 
because AT 301 states that 
assurance, not a summary of 
an accoun­tant may accept 
findings.
an engagement to apply 
C.3.b.  Agreed­Upon Procedures  agreed­upon proce­dures to 
Engagements prospective financial 
statements provided that (1)
122. (c) The requirement is to  the specified parties 
identify the statement thatis not  involved have participated 
correct concerning “specified  in establish­ing the nature 
parties” of an agreed­upon  and scope of the 
procedures report under either the  engagement and take 
auditing or attesta­tion standards.  respon­sibility for the 
Answer (c) is correct because while  adequacy of the procedures 
a practi­tioner should establish a  to be performed, 
clear understanding regarding the  (2) use of the report is to be 
terms of the engagement, preferably restricted to specified parties 
in an engagement letter, no such  involved, and (3) the prospective 
engagement letter is required.  financial statements in­clude a 
Answers (a) and (b) are incorrect  summary of significant 
because the specified parties must  assumptions. Answer (b) is 
agree on the procedures to be  incorrect because the prospective 
performed and take responsibility  financial statements need not be 
for their adequacy. Answer (d) is  examined. Answer (c) is incorrect 
incorrect because an additional  because responsi­bility for the 
party may be added as a specified  adequacy of the procedures is taken 
party after completion of the  by the spe­cified parties. Answer (d)
engagement. is incorrect because a summary of 
findings may be provided based on 
C.3.c. Financial Forecasts and  the agreed­upon proce­dures. 
Projections
123. (a) The requirement is to determine 
the information to be included in a 
separate paragraph included in an 
ac­countant’s report on the 
examination of projected financial 
statements. Answer (a) is correct 
because AT 301 requires that such a
report include a separate paragraph 
that describes the limitations on the 
usefulness of the presentation. See 
AT 301 for information that should 
be included in an exami­nation 
report of prospective financial 
financial projec­tion, or 
125. (c) The requirement is to both are engagements 
identify the statement  governed by the 
which should be  provisions of the 
included in an  Statement on Standards 
accountant’s  for Attestation Statements.
compilation report on  Answer (c) is correct 
financial forecasts.  because the attestation 
Answer (c) is correct  standards expli­citly 
because when the  exclude expert witness 
accountant is preparing a work, but include the 
standard compilation  compila­tion of a financial
report on prospective  projection; note that in 
financial statements, AT most areas compi­lations 
301 requires that the  are not included in 
accountant include a  attestation standard 
statement indicating that coverage, but in the area 
the prospective results  of prospective financial 
may not be achieved.  statement (forecasts as 
well as projections) 
126. (c) The requirement is to  coverage is included. 
identify the appropriate  Answer (a) is incorrect 
distribution of an entity’s  because it states that 
financial projection. A  expert witness work is in­
financial projection is  cluded. Answer (b) is 
sometimes prepared to  incorrect both because it 
present one or more  states that expert witness 
hypothetical courses of  work is included and that 
action for evaluation in  compiling a projec­tion is 
response to a question  not included. Answer (d) 
such as “What would  is incorrect because it 
happen if...?” It is based  states that compilations of 
on a responsible party’s  projections are not 
assumptions reflecting  included. 
conditions it ex­pects 
would exist and the  128. (a) The requirement is to 
course of action it expects identify the statement that
would be taken, given one should be included in a 
or more hypothetical  compilation report on a 
assumptions. Pro­jections  financial forecast. 
are “limited use” financial Answer (a) is correct 
statements meant for the  because the report should 
responsible party  state that the compilation 
(generally management)  does not include 
and third parties with  evaluation of the support 
whom the responsible  of the assumptions 
party is negotiating  underlying the forecast. 
directly. Answer (c) is  An­swer (b) is incorrect 
correct because a bank  because no such 
might be expected to  statement is included in a 
receive such a projection.  compilation report, and 
Answers (a), (b), and (d)  because hypothetical 
are all incorrect because  assump­tions pertain to 
projections are meant for  financial projections, not 
“limited use” and not to  financial forecasts. 
be broadly distributed to  Answer (c) is incorrect 
groups such as all em­ because the report makes 
ployees or potential or  no state­ment concerning 
current stockholders. AT  the range of assumptions.
301 pro­vides overall  Answer (d) is incorrect 
guidance on the area of  because the statement is 
financial forecasts and  not included in the report,
projections.  and because the 
prospective statements 
127. (c) The requirement is to  are management’s, not 
determine whether either  the accountant’s, 
testifying as an expert  representation. 
witness, compiling a 
129. (d) The requirement is to identify  for general use. An­swer (a) is 
the type of general use prospective  incorrect because financial 
financial statement on which the  projections are only appropriate for 
accountant may appropriately  the party responsible for preparing 
report. Answer (d) is correct  them or for third parties with whom 
because financial forecasts are  the responsible party is negoti­ating 
considered prospective financial  directly. Answers (b) and (c) are 
statements, and they are appropriate incorrect because 
196 MODULE 5   REPORTING
procedures is taken by the specified 
parties, not by the accountant. 
partial presentations and pro  Answer (c) is incorrect because 
forma financial statements are not  when the accountant reports on the 
considered prospective financial  results of applying agreed­upon 
statements. procedures he or she should not 
express any form of negative 
130. (b) The requirement is to 
assurance on the prospective 
identify the type of pro­ financial statements taken as a 
spective financial  whole. 
statement that includes 
one or more hy­pothetical  132. (d) The requirement is to determine 
(“what if?”) assumptions.  the appropriate type of audit report 
Answer (b) is correct  to be issued when an accountant 
because financial  exam­ines a financial forecast that 
projections include one or  fails to disclose several signifi­cant 
more hypotheti­cal  assumptions used to prepare the 
assumptions. Answer (a)  forecast. AT 301 states that an 
is incorrect because pro  adverse opinion is appropriate when
forma financial  significant as­sumptions are not 
presentations are designed disclosed. 
to demonstrate the ef­fect 
of a future or hypothetical C.3.d.  Pro Forma Financial 
transaction by showing  Information
how it might have 
affected the historical  133. (d) The requirement is to 
financial statements if it  determine the statementthat should 
had been consummated  be included in an accountant’s 
during the period covered  report on a review of pro forma 
by those statements.  financial information. Answer (d) is
Answer (c) is incorrect  correct because the report must 
because partial presen­ include a reference to the financial 
tations are presentations  statements from which the historical
that do not meet the  financial information is derived and 
minimum presentation  a statement as to whether such 
guidelines of AT 301.  financial state­ments were audited 
Answer (d) is incorrect  or reviewed.
because financial  C.3.e. Management Discussion 
forecasts present, to the 
and Analysis
best of the respon­sible 
party’s knowledge and  134. (b) The requirement is to 
belief, an entity’s  determine the reply that is
expected fi­nancial  not an objective of a 
position, results of  CPA’s examination of a 
operations, and changes in client’sMD&A. Answer 
finan­cial information.  (b) is correct because an 
131. (d) The requirement is to determine  examination of a client’s 
an accountant’s responsibility when  MD&A does not directly 
he or she accepts an engagement to  address overall 
ap­ply agreed­upon procedures to  conformity with such 
prospective financial state­ments.  rules and regulations. 
Answer (d) is correct because  Answers (a), (c), and (d) 
distribution of such a report is to be  are the three objectives of 
restricted to the specified parties  an MD&A examination 
involved. AT 301 also requires that  agree­ment. 
the specified parties participate in  135. (c) The requirement is to 
establishing and taking  identify an assertion em­
responsibility for the adequacy of  bodied in MD&A. Answer 
the procedures, and that the  (c) is correct because the 
prospective financial statements 
at­testation standards on 
include a summary of significant 
MD&A indicate that 
assumptions. Answer (a) is incorrect
consistency with 
because the prospective financial 
statements need not be examined. 
Answer (b) is incorrect because re­
sponsibility for the adequacy of the 
www.aicpa.org. Answers 
the financial statements is an  (a) and (c) are incorrect 
assertion—in addition, occur­rence,  because such standards do
completeness, and presentation and  not exist. Answer (d) is 
disclosure are embodied assertions.  incorrect because 
Answers (a), (b), and (d) are all  Statements on Auditing 
incor­rect because valuation,  Standards do not address 
reliability, and rights and obligations  Trust Ser­vices 
are not considered to be assertions  engagements. 
embodied in the MD&A.
139. (d) The requirement is to identify 
136. (b) Answer (b) is correct  what the Web­Trust seal of 
because the MD&A re­view of an  assurance relates most directly to. 
issuer (public) entity should be  Answer (d) is correct because the 
restricted to the use of specified  WebTrust seal is designed to 
parties. Answer (a) is incorrect  provide assurance on Web site 
because a review consists  security, availability, processing in­
principally of applying analytical  tegrity, online privacy and 
proce­dures, rather than also  confidentiality. Answers (a), (b), 
including search and verification  and (c) are all incorrect since 
pro­cedures. Answer (c) is incorrect WebTrust isn’t specially aimed at 
because a consideration of relevant  financial statements, health care 
portion of internal control is  facilities, or risk assurance 
necessary to identify types of  procedures. 
potential misstatements and to  140. (b) The requirement is to 
select the inquiries and analytical 
determine the type of 
procedures. Answer (d) is incorrect 
opinion or assurance most
because a review report ordinarily 
likely to be included in a 
provides negative assurance, not a 
CPA’s report relating to 
summary of findings.
WebTrust engagements. 
C.3.f. Trust Services Answer (b) is correct 
because the WebTrust 
137. (d) The requirement is to  examination report 
identify the proper term  provides an opinion on 
for an attest engagement  whether the site meets the
in which a CPA assesses a Trust Services cri­teria 
client’s commercial  for one or more of the 
Internet site for predefined Trust Services Principles. 
criteria such as those over  An­swer (a) is incorrect 
online privacy. Answer  because no opinion on 
(d) is correct because the  being “hack­proof” is 
AICPA’s Trust Services  issued. Answer (c) is 
Principles relate to this  incorrect because 
area, and WebTrust is the  negative assurance is not 
most likely product—see  provided. Answer (d) is 
www.aicpa.org. Answers  incorrect because an 
(a), (b), and (c) all  agreed­upon procedures 
represent names of  engagement, not an 
products not included in  examination engagement 
the professional standards. results in a summary of 
findings. 
138. (b) The requirement is to 
identify the standards un­ 141. (b) The requirement is to 
der which Trust Services  identify the type of en­
engagements are  gagement that considers 
performed. An­swer (b) is security, availability, 
correct because the  processing integrity, 
Statements on Standards  online privacy and/or 
for Attestation  confidentiality over any 
engagements address such type of defined electronic 
engagements. More  system. Answer (b) is 
information on Trust  correct because SysTrust 
Services engagements  engagements consider any
(WebTrust and SysTrust)  type of defined electronic 
is available on the  system. Answer (a) is 
AICPA’s Web site—  incorrect because an en­ 
MODULE 5   REPORTING 197
of the procedures. See AT 201 for 
guidance on agreed­upon 
gagement   to   consider   internal procedures en­gagements. 
control   over   financial   report­ing
does   not   directly   address   these 145. (a) The requirement is to 
attributes.   Answer   (c)   is   incorrect identify the statement that
because   there   is   no   such is included in a CPA’s 
engagement   as   a   Web   site report on agreed­upon 
Associate. Answer (d) is incorrect procedures on 
because   WebTrust   deals   more management’s assertion 
directly with company Web sites. about an entity’s 
compliance with 
142. (b) The requirement is to  specified requirements. 
identify the most likelyreport when  Answer (a) is correct 
a client refuses to provide a written  because such an agreed­
assertion in a Trust Services  upon procedures report 
engagement. Answer (b) is correct  includes a statement of 
because this represents a scope  limitations on the use of 
limitation, and client imposed scope the report because it is 
limitations are most likely to result  intended solely for the 
in a disclaimer of opin­ion. Answer  use of specified parties. 
(a) is incorrect because an adverse  See AT 601 for in­
opinion is appropriate when a CPA  formation that should be 
believes that the information is so  included in such an 
misstated as to be misleading.  agreed­upon procedures 
Answer (c) is incorrect be­cause  report. Answer (b) is 
client imposed scope limitations  incorrect because no 
generally result in disclaimers, not  “opinion” is included. 
qualified opinions. Answer (d) is  Answer (c) is incorrect 
incorrect because an unqualified  because a summary of 
opinion is most likely not  findings, not negative 
appropriate in such a circumstance. assurance is provided. 
D.1.a.  Compliance Attestation  Answer (d) is incorrect 
because the CPA makes 
Engagements—Agreed­
no repre­sentation 
Upon Procedures  regarding the sufficiency 
Engagements of procedures. 
143. (c) The requirement is to D.1.b.  Compliance Attestation 
identify the type of asso­ Engagements—
ciation  not  permitted Examination 
under   the   compliance Engagements
attestation stan­dards. AT
601   does   not   allow   the 146. (c) The requirement is to 
CPA to perform a review identify the correct state­ment 
over compliance.  concerning an examination report 
when management has properly 
144. (b) The requirement is to identify 
disclosed an instance of material 
the information provided in an  noncompli­ance. AT 601 states 
agreed­upon procedures report on  that the opinion should be 
compliance with contractual  qualified or adverse. Note that AT
requirements to pay royalties.  601 requires the CPA’s report to 
Answer (b) is correct because  relate directly to the subject 
agreed­upon procedures reports  matter when the opinion is 
include a list of the procedures  modified.
performed (or reference thereto) and
findings. Answer (a) is incorrect 
because no such disclaimer of 
opinion is provided in an agreed­
upon procedures report. Answer (c) 
is incorrect because no opinion is 
included in an agreed­upon 
procedures report. Answer (d) is 
incorrect be­cause an agreed­upon 
procedures report includes a 
statement disclaiming an opinion on
the sufficiency of procedures, not an
acknowledgement of the sufficiency
“Yellow Book” suggests 
D.2.a.  Compliance Auditing of  that in addition to 
Federal Financial Assis­ financial statements, such 
tance Programs— an audit may include 
consideration of 
GAAS Audits
(1)   program   results,   (2)
147. (b) The requirement is to determine  compliance   with   laws   and
an auditor’s responsibility when  regula­tions,   and   (3)   economy
auditing a not­for­profit entity that re­ and efficiency. 
ceives governmental financial  150. (b) The requirement is to identify 
assistance. Answer (b) is correct  whether an audi­tor performing an 
because AU 801 requires that the  audit in accordance with 
auditor assess whether management  Government Auditing Standards 
has identified laws and regulations  (the “Yellow Book”) is required to 
that have a direct and material effect  report on noteworthy 
on the entity’s financial statements;  accomplishments of the program, 
AU 801 also presents procedures to  the scope of the auditor’s testing of 
be followed in assessing such laws  internal controls, or both. An­swer 
and regulations. Answer (a) is incor­ (b) is correct because the “yellow 
rect because such a separate report  book” requires re­porting only upon
describing expected benefits and costs the scope of the auditor’s testing of 
does not need to be issued. Answer  inter­nal controls. Answers (a), (c), 
and (d) all include an incorrect 
(c) is incorrect because the CPA will 
combination of reporting replies. 
not notify the governmental agency 
that the audit is not designed to  151. (b) The requirement is to 
provide assurance. Answer (d) is  identify whether an audi­tor
incorrect because the CPA does not  performing an audit in 
express an opinion on the entity’s  accordance with 
continued eligibility for governmen­ Government Auditing 
tal financial assistance. AU 801  Standards (the “Yellow 
presents requirements re­lating to  Book”) is required to report 
compliance auditing for governmental on recommendations for 
entities and recipients of  actions to improve 
governmental financial assistance.  operations, the scope of 
tests of compliance with 
148. (c) The requirement is to  laws and regulations, or 
determine the focus of an  both. Answer (b) is correct 
auditor’s attention in  because the “yellow book” 
detecting misstatements  re­quires reporting upon the
resulting from violations of  scope of the auditor’s tests 
laws and regulations when  of compliance with laws 
auditing a not­for­profit  and regulations. Answers 
organization that receives  (a), (c), and 
financial assistance from  (d)   all   include   an   incorrect
governmental agencies.  combination of reporting replies. 
Answer (c) is correct 
because the focus of such  D.2.b.  Compliance Auditing of 
procedures should be on  Federal Financial Assis­
violations that have a direct tance Programs—
and material effect on the  GAGAS Audits
amounts in the organiza­
tion’s financial statements  152. (a) The requirement is to 
(AU 801). Answers (a), (b), identify the correct state­ment with 
and (d) all represent a focus that respect to a financial statement audit 
is   not   as   accurate   as   that conducted in accordance with 
provided in answer (c).  Government Auditing Standards (the 
“Yel­low Book”). Answer (a) is 
149. (b) The requirement is to  correct because the auditor issuesa 
determine the proper  report on compliance with laws and 
scope of a governmental  internal control, and a report on the 
audit. The General  financial information. Answer (b) is 
Accounting Office’s  incorrect
198 MODULE 5   REPORTING
general. 
Answers (a), 
because a financial statement audit  (b), and 
does not address econ­omy and  (c) all provide inaccurate 
efficiency in the manner suggested.  descriptions of auditor 
Answer (c) is incorrect because  reporting responsibility. 
recommendations for actions to  See Government 
correct problems and improve  Auditing Standards (the 
operations are not ordinarily in­ “Yellow Book”) for 
cluded. Answer (d) is incorrect  information on reporting 
because a financial state­ment audit under Govern­ment 
does not address whether programs  Auditing Standards. 
are achieving the desired results.
155. (a) The requirement is to identify a 
153. (a) The requirement is to identify  common aspect of various types of 
the correct state­ment with respect  audits of recipients of federal 
to a financial statement audit  financial assistance in accordance 
conducted in accordance with  with federal audit regulations. An­
Government Auditing Standards  swer (a) is correct because audits of 
(the “Yel­low Book”). Answer (a) is  recipients of federal financial 
correct because the auditor issuesa  assistance include reports on (1) the 
report on compliance with laws and  financial statements, and (2) a 
internal control, and a report on the  separate or combined report on 
financial information. Answer (b) is internal control and on compliance 
incorrect because not all instances  with laws and regulations. An­swer 
of abuse, waste and mismanage­ (b) is incorrect because materiality 
ment are so reported. Answer (c) is  levels are not ordi­narily lower or 
incorrect because the views of 
always determined by the 
officials are not reported. Answer 
governmental en­tity. Answer (c) is 
(d) is incorrect because internal 
incorrect because the auditor need 
control activities designed to detect 
not obtain such written management
or pre­vent fraud are not reported to
the inspector general.  representations. An­ 
swer (d) is incorrect because 
154. (d) The  requirements for reporting ille­gal 
requirement is  acts may vary depending upon the 
to identify the  type of audit being performed. AU 
circumstance in 801 provides requirements related 
which an  to audit­ing entities that have 
auditor is  received governmental financial 
required to  assis­tance. In addition, guidance is
report a  provided by Government Auditing 
falsification of  Standards (GAS), also referred to 
accounting  as the “YellowBook,” published by 
records directly  the Comptroller General of the 
to a federal  UnitedStates. 
inspector 
general. Answer 156. (b) The requirement is to identify to
(d) is correct  whom an audi­tor most likely would
because under  be responsible for communicating 
Government  significant deficiencies in the design
Auditing  of internal control. Answer (b) is 
Standards a  correct because in audits under 
falsification of  Government Auditing Standards, 
accounting  significant deficiencies in the design
records must  of internal control are 
ordi­narily be  communicated to legislative and 
communicated  regula­tory bodies (AU 801). 
by the auditor  Answer (a) is incorrect because the 
to the auditee  Securities and Exchange 
and, if the  Commission does not ordinarily 
auditee fails to  receive information on such 
make  deficiencies. Answer (c) is incorrect
appropriate  because while a court­appointed 
disclosure, by  creditors’ com­mittee might in some
the auditor to a  circumstances receive information 
federal  on such deficiencies, this practice is 
inspector  not as frequent as is done under 
Government Auditing Standards. 
Answer (d) is incor­  rect because shareholders do not 
normally receive reports on 
significant deficiencies or material 
weaknesses (see AU 325).
157. (c) The requirement is to determine 
the opinion which an auditor should
express in a report on compliance 
when s/he has detected material 
instances of noncompliance within 
the program. AU 801 defines these 
instances of ma­terial 
noncompliance as failures to follow 
requirements, or violations of 
regulations or grants which cause 
the auditor to conclude that the total
of the misstatements resulting from 
these failures or violations is 
material to the financial state­ments.
Therefore, answer (c) is correct 
because the auditor should issue a 
qualified or an adverse opinion. 
Answer (a) is incorrect because the 
auditor is required under 
Governmen­tal Auditing Standards 
to provide reasonable assurance on 
the entity’s compliance with the 
applicable laws and regula­tions. 
Answer (b) is incorrect because the 
auditor must dis­close the instances 
of noncompliance. Answer (d) is 
incor­rect because the auditor 
should not disclaim an opinion as a 
result of noncompliance. 
158. (b) The requirement is to
determine a documenta­
tion requirement that an 
auditor should follow 
when audit­ing in 
accordance with (also 
referred to as the 
“YellowBook”). Answer 
(b) is correct because 
Government Audit­ing 
Standards require 
documentation of 
supervisory review 
before the report is 
issued. 
159. (a) The requirement is to 
identify the statement that
should be included in an 
auditor’s report on an 
entity’s compliance with 
laws and regulations 
when performing an audit
in accordance with 
Government Auditing 
Standards. Answer (a) is 
correct because such 
compliance reports re­
quire a statement that 
management is 
responsible for com­
pliance with laws, 
regulations, contracts,  Answer (d) is incorrect 
and grants. See AU 801  be­cause the expected 
for this requirement and  duration of the scheme is 
others.  not what leads to reporting
to a federal inspector 
160. (b) The requirement is to  general. 
determine when an 
auditor reporting under  D.2.c. Compliance Auditing of 
would most likely be  Federal Financial Assis­
required to communi­cate  tance Programs—Single
management’s  Audit Act
misappropriation of assets
directly to a federal  161. (b) The requirement is to 
inspector general. Answer identify the auditor’s 
(b) is correct because  proper measure of 
Government Auditing  materiality for major 
Standards requires that  federal financial 
when a gov­erning body  assistance programs 
fails to make a required  under the Single Audit 
report on such acts the  Act. AU 801 requires 
auditors should  that it be determined 
communicate the matter  separately for each major
to the external body  pro­gram. 
specified in the law or 
regulation. Answer (a) is  162. (b) The requirement is to 
incorrect because such  identify the appropriate 
concealment will not  compliance report under 
necessarily lead to com­ the Single Audit Act 
munication to a federal  when a CPA has detected
inspector general. Answer noncompliance with 
(c) is incorrect because  requirements that have a 
material misstatement  material effect on that 
does not necessarily lead  program. AU 801 states 
to such communication.  that under 
MODULE 5   REPORTING 199

such 
circumstan
ces the 
auditor 
should 
express a 
qualified 
or adverse 
opinion.
163. (c

T
he
re
q
ui
re
m
en

is 
to
id
en
tif

th

so
ur
ce
of
au
­
th
or
it
at
iv

g
ui
da
nc

fo

pe
rf
or
m
in

au
di
ts 
of

ci
ty
th
at
is 
su
bj
ec

to
th

re
q
ui
re
m
en
ts 
of
th

U
ni
fo
r
m
Si
n
gl

A
u
di

A
ct
of
1
9
8
4.
A
ns
w
er
(c

is 
co
rr
ec

be
ca
us

w
hi
le
th

A
I
C
P
A
’s
ge
ne
ra
ll

ac
ce
pt
ed
au
di
ti
n

st
an
da
rd

m
us

be
fo
ll
o
w
ed
to
th

ex
te
nt
th
ey
ar

pe
rti
ne
nt

th

G
en
er
al
A
cc
o
u
nt
in

O
ffi
ce
G
o
ve
rn
m
en

A
u
di
ti
n

St
an
da
rd

m
us

al
so
be
ad
he
re

to

T
he
ot
he

re
pl
ie

al

re
la
te
to
st
an
da
rd

n
ot
di
re
ct
ly
re
la
te

to
th

U
ni
fo
r
m
Si
n
gl

A
u
di

A
ct

164. (c

T
h

re
q
ui
re
m
e
nt
is
to
id
e
nt
if

th

c
or
re
ct
st
at

m
e
nt
w
hi
c

w
o
ul

c
o
m
m
u
ni
c
at

w
e
a
k
n
es
se

in
in
te
rn
al
c
o
nt
ro

u
se

in
a
d
m
in
is
te
ri
n


fe
d
er
al
fi
n
a
n
ci
al
as
si
st
a
n
c

pr
o
gr
a
m
w
h
e


C
P
A
h
as
e
x
a
m
in
e

th

g
e
n
er
al
p
ur
p
o
se
fi
n
a
n
ci
al
st
at
e
m
e
nt

of

m
u
ni
ci
p
al
it
y.
T
h

A
I
C
P
A
A

c
o
u
nt
in

a
n

A
u
di

G
ui
d
e,
A
u
d
it

o

S
t
a
te
a
n
d
L
o
c
a

G
o
v
­
e
r
n
m
e
n
t
a

U
n
it
s,
re
q
ui
re

th

c
o
m
m
u
ni
c
at
io

of
w
e
a
k
n
es
se
st
h
at
ar

m
at
er
ia

in
re
la
ti
o

to
th

fe
d
er
al
fi
n
a
n
ci
al
as
si
st
a
n
c

pr
o
gr
a
m

200 MODULE 5   REPORTING

SOLUTIONS TO SIMULATIONS

Task­Based Simulation 1
Accounting
Authoritative
Changes
Literature Help

1. The appropriate guidance regarding reporting on consistency is found in
AU 508.16­.18 
2. The appropriate guidance regarding emphasis of a matter is found in AU
508.19 
3. The   appropriate   guidance   regarding   reporting   on   comparative   financial
statements is found in AU 508.65­.66 

Task­Based Simulation 2
Authoritative
Report
Literature Help
Modifications

1. (E, P) When an auditor hires a specialist to assist in corroborating a
client   estimate   (here   complex   pension   calculations),   and   that
specialist’s findings are reasonably close to those of the client, no
report modification is required or permitted. Since the specialist’s
findings support the financial statements in this situation, a standard
unqualified audit report is appropriate. 
When   major   unresolved   differences   between   the   findings   of
management   and   the   specialist   exist,   report   modification   is
appropriate. 
2. (A, I) When the client’s financial statements materially depart from generally 
accepted accounting principles, either a qualified opinion or an adverse opinion
is appropriate, depending on the magnitude of the misstatement. The value of 
the client’s note receivable has been impaired and therefore the client should 
write the note receivable down to its net realizable value. The auditor will have 
to determine whether to issue a qualified opinion or an adverse opinion on the 
basis of the materiality of the misstatement. Factors the auditor will consider 
include the significance of the account, the pervasiveness of the misstatement 
and the misstatement’s effect on the financial statement taken as whole. The 
audit report, for either opinion, will include an explanatory paragraph to 
describe the substantive reasons for the modification, and the opinion 
paragraph will be modified. 
3. (B, J) A situation where the auditor is unable to obtain sufficient 
competent evidential matter is referred to as a scope limitation. A 
scope limitation may require the auditor to either qualify his or her 
opinion or to disclaim an opinion altogether. Since the auditor was 
unable to observe the inventory count or to obtain evidence through 
alternative procedures, the auditor will have to decide whether to 
issue a qualified opinion or a disclaimer of opinion. The decision will 
be based on the auditor’s judgment as to the nature and magnitude of 
the potential effects of the matters in question and by their 
significance to the financial statements. A qualified opinion will 
describe the circumstances in an explanatory paragraph and will 
modify the scope and opinion paragraphs. A disclaimer of opinion 
will omit the scope paragraph and will include modification of the 
opinion paragraph. 
4. (E, P)  An auditor need not modify a report for an immaterial item that
the client declines to reflect. 
5. (A, I) Since the client’s financial statements omitted required 
disclosures on certain long­term lease obligations, they are not 
prepared in accordance with generally accepted accounting 
principles. As a result, the auditor should express either a quali­fied 
opinion or an adverse opinion. The decision to express either a 
qualified or adverse opinion is based on the significance of the lack 
of disclosure, the pervasiveness of the misstatement, and the overall 
effect the lack of disclosure has on the financial statements. The audit
report, for either opinion, will include an explanatory paragraph to 
describe the substantive reasons for the modification, and the opinion
paragraph will be modified. 
6. (E, O) When a principal auditor decides not to take responsibility for
the work of another auditor, the principal auditor should make 
reference to the work of the other auditor in the audit report. The 
audit report should clearly indicate the division of responsibility 
between the two auditors in the introductory, scope, and opinion 
paragraphs. Reference to the other auditor in the audit report does 
not prevent the principal auditor from issuing an unqualified opinion.
The reference to the other auditor is designed to emphasize the 
divided responsibilities between the two auditors. 
7. (E, H) When an auditor agrees with a change in accounting 
principles, a lack of consistency results in an unqualified opinion
with an explanatory paragraph following the opinion paragraph. 
There is no modification of the three standard paragraphs. 
8. (E, H) The auditor has substantial doubt about the client’s ability to
remain a going concern for a reasonable period of time. The audit 
report should emphasize this concern to the financial statement 
users. As a result, the auditor’s report will in­clude an unqualified 
opinion with an explanatory paragraph following the opinion 
paragraph. 
MODULE 5   REPORTING 201

Task­Based Simulation 3
Research
Authoritative
Literature Help
(A) (B) (C) (D) (E) (F) (G) (H)
1. Which section of the Professional Standards addresses this issue and will
be helpful in responding to the partner?
2. Enter the exact section and paragraph with the appropriate guidance. 315 09

Task­Based Simulation 4
Research
Authoritative
Literature Help

1. AU 508 08
2. She   is   correct,   as   including  the   location   is   not required   (although   it  is
acceptable). 

Task­Based Simulation 5
Li
Audit Report
te
Details
Authoritative ra
ture

Help

(A) (B) (C) (D) (E) (F) (G) 
1. National’s   inadequate   inventory   records (I)  (J) (K) (L)
precluded White from forming an opinion
as to the proper application of generally
accepted   accounting   principles   to
inventory balances on January 1, 20X2. 

2. Rapid Parts Company, a consolidated subsidiary of National, was audited
for the year ended December 31, 20X2, by Green & Co. CPAs. 
3. White has completed its audit procedures on March 28, 20X3, and
planned to submit its auditor’s report to National on April 4, 20X3. 
4. The most appropriate addressee of the audit report. 
5. National’s audit report should be signed by 
Explanation of solutions
1. (B)   The lack of evidence about the beginning inventory affects only the
results of operations (the income statement). 
2. (D)   The   involvement   of   the   other   audit   firm   results   in   a   division   of
responsibility because White does not want to take responsibility for the
other auditor’s work. 
3. (F)   The report should be dated as of the date that all substantive audit
evidence has been obtained. 
4. (I)   Ordinarily   the   report   should   be   addressed   to   the   board   of
directors,   audit   committee,   or   the   company   itself,   not
management. 
5. (K) Only White & Co. will sign the audit report. 
202 MODULE 6   ACCOUNTING AND REVIEW SERVICES

ACCOUNTING AND REVIEW SERVICES
reviewing the financial 
MULTIPLE­CHOICE QUESTIONS (1­49)
statements? 
1. Statements on Standards for  Compilation Review
Accounting and Review Services  a. No No
establish standards and procedures b. Yes No
for which of the following  c. No Yes
engagements?  d. Yes Yes
a. Assisting   in   adjusting
5. Which of the following statements
the   books   of   account
is correct concerning both an 
for a partnership. 
engagement to compile and an 
b. Reviewing   interim
engagement to review a 
financial   data
nonissuer’s financial statements? 
required   to   be   filed
with the SEC.  a. The   accountant
does   not
c. Processing   financial
contemplate
data   for   clients   of
obtaining   an
other   ac­counting
understanding
firms. 
of   internal
d. Compiling   an control. 
individual’s   personal
b. The
financial   state­ment   to
accountant
be   used   to   obtain   a
must   be
mortgage.  independent
2. The authoritative body designated  in   fact   and
to promulgate stan­dards  appearance. 
concerning an accountant’s  c. The
association with unaudited  accountant
financial statements of an entity that is not expresses   no
required to file financial statements with  assurance   on
an agency regulating the issuance of the  the   fi­nancial
entity’s securities is the statements. 
a. Financial   Accounting   Standards d. The
Board.  accountant
b. General Accounting Office.  should   obtain
c. Accounting   and   Review   Services a   written
Committee.  manage­ment
d. Auditing Standards Board.  representation
letter. 
3. Which of the following 
accounting services may an  6. An accountant is required to comply with 
accountant perform without  the provisions of Statements on Standards 
for Accounting and Review Services when 
being required to issue a com­
pilation or review report under  I. Reproducing   client­
the Statements on Standards for  prepared   financial
Accounting and Review  statements,   without
Services?  modification,   as   an
accommodation   to   a
I.  Preparing a working trial balance. client. 
AI. Preparing   standard   monthly   journal
entries. 
a. I only. 
b. II only. 
c. Both I and II. 
d. Neither I nor II. 
4. May an accountant accept an 
engagement to compile or review 
the financial statements of a not­
for­profit entity if the accountant 
is unfamiliar with the specialized 
industry accounting principles, but
plans to obtain the required level 
of knowledge before compiling or
AI. Preparing   standard   monthly   journal by   the   client
entries for depreciation and expiration of that   change
prepaid expenses.  client­
a. I only.  prepared
b. II only.  financial
c. Both I and II.  statements. 
d. Neither I nor II.  d. Generated, 
through the use
7. Kell engaged March, CPA, to submit to Kell of computer 
a written personal financial plan containing  software, 
unaudited personal finan­cial statements.  financial 
March anticipates omitting certain disclo­ statements 
sures required by GAAP because the  prepared in 
engagement’s sole purpose is to assist Kell  accordance 
in developing a personal financial plan. For  with a 
March to be exempt from complying with  comprehensive
the requirements of Statements on Standards basis of 
for Accounting and Review Services. Kell is accounting 
required to agree that the  other than 
a. Financial GAAP. 
statements   will
9. Davis, CPA, accepted an 
not  be
engagement to audit the fi­nancial
presented   in statements of Tech Resources, a 
com­parative nonissuer. Before the completion 
form   with of the audit, Tech requested Davis
those   of   the to change the engagement to a 
prior period.  compilation of financial 
b. Omitted statements. Before Davis agrees to
disclosures change the engagement, Davis is 
required   by required to consider the 
GAAP   are
Additional audit Reason given
not effort necessary to for
material.  complete the audit Tech’s request
c. Financial   statements   will  not  be a. No No
disclosed   to   a   non­CPA   financial b. Yes Yes
planner.  c. Yes No
d. Financial d. No Yes
statements 10. An accountant may compile a 
will  not  be nonissuer’s financialstatements that omit all 
used   to of the disclosures required by GAAP only if
obtain the omission is
credit. 
I.  Clearly indicated in the accountant’s 
8. Statements   on   Standards   for report.
Accounting   and   Review AI. Not   undertaken   with   the
Services (SSARS) apply  when intention   of   misleading   the
an accountant has  financial statement users. 
a. Typed   client­prepared   financial a. I only. 
statements,   without   modification, b. II only. 
as an accommodation to the client.  c. Both I and II. 
b. Provided   a d. Either I or II. 
client   with   a
financial
statement
format   that
does  not
include   dollar
amounts,   to   be
used   by   the
client   in
preparing
financial
statements. 
c. Proposed
correcting
journal   entries
to be recorded
MODULE 6   ACCOUNTING AND REVIEW SERVICES 203
cor­roborate 
management’s
11. When engaged to compile the  assertions that 
financial statements of a  are em­bodied 
nonissuer (nonpublic) entity, an  in the financial
accountant is required to possess  statement 
a level of knowledge of the  components. 
entity’s accounting principles and
practices. This requirement most  13. One of the conditions required for 
likely will include obtaining a  an accountant to submit a written 
general understanding of the  personal financial plan containing 
a. Stated qualifications of the entity’s unaudited financial statements to a
accounting per­sonnel.  client without complying with the 
b. Design of the entity’s requirements of Statements on 
internal   controls Standards of Accounting and 
placed in operation.  Review Services, is that the 
c. Risk   factors   relating   to a. Client   agrees   that   the
misstatements   arising financial statements will
from illegal acts.  not  be   used   to   obtain
d. Internal   control credit. 
awareness   of   the b. Accountant compiled 
entity’s   senior or reviewed the client’s
management.  financial statements for
the immediate prior 
12. Which of the following  year. 
procedures is ordinarily per­ c. Engagement   letter
formed by an accountant in a  acknowledges   that   the
compilation engagement of a  financial statements will
nonissuer (nonpublic) entity?  contain   departures   from
a. Reading the  generally   accepted
financial  accounting principles. 
statements to  d. Accountant expresses 
consider  limited assurance that 
whether they  the financial 
are free of  statements are free of 
obvious  any material 
mistakes in  misstatements. 
the 
application of 14. While performing a compilation of financial
accounting  statements, information indicating that the 
principles.  entity whose information is being compiled 
b. Obtaining may lack the ability to continue as a going 
written concern has come to the accountant’s 
representations attention. The client agrees that such a 
from   manage­ situation does exist, but refuses to add 
ment disclosures relating to it. What effect is this 
indicating   that most likely to have on the accountant’s 
the   compiled review report? 
financial state­ a. No   effect,   a
ments will  not standard
be   used   to unqualified
obtain credit.  report   is
c. Making  appropriate. 
inquiries of  b. The report should 
management  indicate a departure from
concerning  gener­ally accepted 
ac­tions taken  accounting principles, 
at meetings of with modifi­cation of the
the  report’s third paragraph 
stockholders  and addition of an 
and the board  explanatory paragraph. 
of directors.  c. An adverse opinion 
d. Applying  should be issued, with 
analytical  mod­ification of the 
procedures  opinion paragraph and 
designed to  addition of an 
explanatory paragraph. 
d. A qualified opinion 
should be issued, with  15. When   compiled   financial
mod­ification of the  statements are accompanied by an
opinion paragraph and  accountant’s   report,   that   report
addition of an  should state that 
explanatory paragraph.  a. A   compilation
includes   assessing   the
accounting   principles
used   and   significant
management   esti­
mates,   as   well   as
evaluating   the   overall
financial   statement
presentation. 
b. The accountant 
compiled the financial 
statements in 
accordance with 
Statements on 
Standards for 
Accounting and 
Review Services. 
c. A compilation is 
substantially less in 
scope than an audit in 
accordance with GAAS, 
the objective of which is 
the expression of an 
opinion. 
d. The accountant is not 
aware of any material 
modi­fications that 
should be made to the 
financial statements to 
conform with GAAP. 
16. Miller, CPA, is engaged to 
compile the financial state­ments 
of Web Co., a nonissuer 
(nonpublic) entity, in conformity 
with the income tax basis of 
accounting. If Web’s financial 
statements do not disclose the 
basis of accounting used, Miller 
should 
a. Disclose   the   basis   of
accounting   in   the
account­ant’s
compilation report. 
b. Clearly label each page
“Distribution   Restricted
—   Material
Modifications
Required.” 
c. Issue   a   special   report
describing   the   effect
of   the   incomplete
presentation. 
d. Withdraw   from   the
engagement   and
provide  no  further
services to Web. 
17. When an accountant is engaged
to compile a nonissuer’s 
financial statements that omit 
substantially all disclosures 
required by GAAP, the 
accountant should indicate in  in the subsequent year, the 
the compilation report that the  unaudited financial statements 
financial statements are  should be clearly marked to 
a. Not  designed  for  those indicate their status and 
who   are   uninformed I. The   report   on   the   unaudited
about   the   omitted financial   statements   should   be
disclosures.  reissued. 
b. Prepared   in   conformity   with   a AI. The report on the audited financial 
comprehensive basis of accounting statements should include a 
other than GAAP.  separate paragraph describing the 
c. Not   compiled   in responsibility assumed for the 
accordance   with unaudited financial statements. 
Statements   on a. I only. 
Standards   for b. II only. 
Accounting   and c. Both I and II. 
Review Services.  d. Either I or II. 
d. Special­purpose
financial   statements 19. Clark, CPA, compiled and properly 
that   are  not reported on thefinancial statements of 
comparable to those of Green Co., a nonissuer, for the year ended 
prior periods.  March 31, 2008. These financial 
statements omitted substantially all 
18. When unaudited financial  disclosures required by generally accepted
statements of a nonissuer are  accounting principles (GAAP). Green 
presented in comparative form  asked Clark to compile the statements for 
with audited financial statements  the year ended March 31, 2009,
204 MODULE 6   ACCOUNTING AND REVIEW SERVICES
of financial statements, 
infor­mation that is the 
and to include all GAAP disclosures for the  representation of 
2009 statements only, but otherwise present management. 
both years’ financial statements in 
comparative form. What is Clark’s  21. How does an accountant make the following
responsibility concerning the proposed  representa­tions   when   issuing   the   standard
engagement? report for the compilation of a nonissuer’s
a. Clark may not report on  financial statements? 
the comparative  The financial The accountant
financial statements  statements have has compiled the
because the 2008   not  been audited financial statements
statements are not  a. Implicitly Implicitly
b. Explicitly Explicitly
comparable to the 2009 
c. Implicitly Explicitly
statements that include  d. Explicitly Implicitly
the GAAP disclosures. 
b. Clark may report on  22. An   accountant’s   compilation
the comparative  report should be dated as of the
financial statements  date of 
provided the 2009  a. Completion of fieldwork. 
statements do not  b. Completion of the compilation. 
contain any obvious  c. Transmittal  of   the   compilation
material  report.
misstatements.  d. The   latest   subsequent
c. Clark may report on the event   referred   to   in   the
comparative financial  notes   to   the   financial
statements provided an  statements. 
explanatory paragraph 
is added to Clark’s  23. An accountant has compiled the 
report on the  financial statements of a 
comparative finan­cial  nonissuer in accordance with 
statements.  Statements on Standards for 
d. Clark may report on the  Accounting and Review Services 
comparative financial  (SSARS). Does SSARS require 
statements provided  that the compilation report be 
Clark updates the report  printed on the accountant’s 
on the 2008 statements  letterhead and that the report be 
that do not include the  manually signed by the 
accountant? 
GAAP disclosures. 
Printed on the Manually signed
20. Which of the following  accountant’s letterhead by the accountant
statements should not be in­ a. Yes Yes
cluded in an accountant’s  b. Yes No
standard report based on the com­ c. No Yes
pilation of an entity’s financial  d. No No
statements? 
a. A statement that the 
compilation was 
performed in accordance
with standards 
established by the 
American Institute of 
CPAs. 
b. A   statement   that   the
accountant   has  not
audited or reviewed the
financial statements. 
c. A statement that the 
accountant does not 
express an opinion but 
expresses only limited 
assurance on the 
financial statements. 
d. A statement that a 
compilation is limited to 
pre­senting, in the form 
c. Not independent and, therefore, may
24. Which of the following is correct issue  a review  report,   but may  not
relating to compiled financial  issue an auditor’s opinion. 
statements when third­party  d. Independent because the
reliance upon those statements is financial interest is im­
anticipated?  material and, therefore, 
a. A   compilation   report   must   be may issue a review 
issued.  report. 
b. Omission   of   note   disclosures   is
unacceptable.  28. Moore, CPA, has been asked to issue a 
review report on the balance sheet of Dover 
c. A   written   engagement   letter   is
Co., a nonissuer. Moore will not be 
required. 
reporting on Dover’s statements of income, 
d. Each   page   of   the
retained earnings, and cash flows. Moore 
financial   statements
may issue the review report provided the 
should have a restriction
such   as   “Restricted   for a. Balance   sheet   is
presented   in   a
Management’s   Use
prescribed   form   of   an
Only.” 
industry   trade
25. Which communication option(s)  association. 
may be used when an accountant  b. Scope of the inquiry and
submits compiled financial  analytical   procedures
statements to be used only by  has not been restricted. 
management?  c. Balance sheet is not to
be used to obtain credit
Compilation report Written engagement letter
or   distributed   to
a. Yes Yes
b. Yes No
creditors. 
c. No Yes d. Specialized   accounting
d. No No principles  and  practices
of Dover’s industry are
26. A   compilation   report   is  not disclosed. 
required   when   compiled
financial statements are expected 29. Baker, CPA, was engaged to 
to be used by  review the financial state­ments of
Hall Co., a nonissuer. During the 
a. Management only. 
engagement Baker uncovered a 
b. Management and third parties. 
complex scheme involving client 
c. Third parties only. 
illegal acts that materially affect 
d. A compilation report is Hall’s financial statements. If 
required   whenever
Baker believes that modification 
finan­cial   statements
of the standard review report is 
are compiled. 
not adequate to indicate the 
27. If requested to perform a review deficiencies in the fi­nancial 
engagement for a nonissuer in  statements, Baker should 
which an accountant has an  a. Disclaim an opinion. 
immaterial direct financial  b. Issue an adverse opinion. 
interest, the accountant is  c. Withdraw from the engagement. 
a. Not   independent   and,   therefore, d. Issue a qualified opinion. 
may  not  be   associ­ated   with   the
financial statements.  30. Which of the  following is  not
b. Not   independent   and,   therefore, generally   considered   a
procedure   followed   by   an
may not issue a re­view report. 
accountant in obtaining a 
MODULE 6   ACCOUNTING AND REVIEW SERVICES 205
d. No Yes

reasonable basis for the expression of  34. Which of the following inquiry or 
limited assurance for a review of financial  analytical proceduresordinarily is 
statements? performed in an engagement to review a 
a. Apply analytical procedures.  nonissuer’s financial statements?
b. Assess fraud risk. 
c. Make inquiries of management. 
d. Obtain   written   representations
from management. 
31. Which of the following procedures would an
accountant least likely perform during an 
engagement to review thefinancial 
statements of a nonissuer? 
a. Observing   the
safeguards   over   access
to and use of assets and
records. 
b. Comparing   the
financial   statements
with   antici­pated
results in budgets and
forecasts. 
c. Inquiring   of
management   about
actions   taken   at   the
board   of   directors’
meetings. 
d. Studying   the
relationships   of
financial   statement
elements   expected   to
conform   to   predictable
pat­terns. 
32. Which   of   the   following
procedures   should   an   ac­
countant perform during an
engagement   to   review   the
financial   statements   of   a
nonissuer? 
a. Communicating
significant   deficiencies
discov­ered   during   the
assessment   of   control
risk. 
b. Obtaining  a  client representation
letter   from   mem­bers   of
management. 
c. Sending   bank
confirmation   letters   to
the   entity’s   financial
institutions. 
d. Examining   cash
disbursements   in   the
subsequent   period   for
unrecorded liabilities. 
33. An accountant should perform
analytical procedures during an
engagement to 
Compile a Review a
nonissuer’s nonissuer’s
financial statements financial statements
a. No No
b. Yes Yes
c. Yes No
engagement   can   pro­ceed
a. Sales returns and allowances.  only   if   distribution   of   the
b. Credit sales.  accountant’s   report   is
c. Sales of consigned goods.  restricted to internal use. 
d. Cash sales.  3 Determine the effects of the
departures from GAAP and
1. When performing an  issue a special report on the
engagement to review a  financial statements. 
nonissuer’s financial statements,  4 Issue a modified review 
an accountant most likely would  report provided the entity 
1 Confirm   a agrees that the financial 
sample   of statements will not be used 
significant to obtain credit. 
accounts
receivable 3. When providing limited 
balances.  assurance that the financial 
2 Ask   about   actions statements of a nonissuer 
taken   at   board   of (nonpublic entity) require no 
directors’ meetings.  material modifications to be in 
3 Obtain   an   understanding   of accordance with generally 
internal control.  accepted accounting principles, 
4 Limit   the   distribution   of   the the accountant should 
accountant’s report.  1 Assess   the   risk   that   a
2. An accountant has been engaged  material   misstatement
to review a nonis­suer’s financial  could occur in a financial
statement assertion. 
statements that contain several 
2 Confirm with the entity’s lawyer
departures from GAAP. If the 
that   material   loss   contingencies
financial statements are not  are disclosed. 
revised and modification of the  3 Understand   the
standard review report is not  accounting   principles
adequate to indicate the  of   the   in­dustry   in
deficiencies, the accountant should which   the   entity
1 Withdraw   from   the operates. 
engagement   and   provide 4 Develop   audit   programs
no  further   services to determine whether the
concerning   these   financial entity’s   financial
statements   are   fairly
state­ments.  presented. 
2 Inform management that the
a. Analytical procedures designed to test the ac­ 
counting records by obtaining corroborating audit 40.  Smith, CPA, has been asked to issue
a review report on
evidence. purchases transactions.
b. Inquiries   concerning
the entity’s procedures 36. Which of the following would the
for   recording   and accountant most likely investigate
summarizing during the review of financial 
transactions.  statements of a nonissuer if 
c. Analytical procedures  accounts receivable did not 
designed to test manage­ conform to a predictable pattern 
ment’s assertions  during the year? 
regarding continued 
existence. 
d. Inquiries of the entity’s attorney
concerning   con­tingent
liabilities. 
35. Which   of   the   following
procedures would most likely be
included in a review engagement
of a nonissuer? 
a. Preparing   a   bank   transfer
schedule. 
b. Inquiring   about   related­party
transactions. 
c. Assessing internal control. 
d. Performing   cutoff
tests   on   sales   and
the   balance   sheet   of   Cone   Company,   a c. The   scope   of   Smith’s
nonissuer,   and   not   on   the   other   related inquiry   and   analytical
financial statements. Smith may do so only pro­cedures   is  not
if restricted. 
a. Smith compiles and  d. Cone   is   a   new   client
reports on the related  and   Smith   accepts   the
state­ments of  en­gagement   after   the
income, retained  end   of   Cone’s   fiscal
earnings, and cash  year. 
flows. 
b. Smith is not aware of  41. In reviewing the financial 
any material  statements of a nonissuer, an 
modifications needed  accountant is required to modify 
for the balance sheet to  the standard report for which of 
conform with GAAP.  the following matters? 
206 MODULE 6   ACCOUNTING AND REVIEW SERVICES
45. Gole, CPA, is engaged to review 
Inability to assess Discovery of significant the 20X8 financial statements of 
the risk of material deficiencies in the design North Co., a nonissuer. 
misstatement of the entity’s Previously, Gole audited North’s 
due to fraud internal control 20X7 financial statements and 
a. Yes Yes expressed an unqualified opinion. 
b. Yes No Gole decides to include a separate 
c. No Yes paragraph in the 20X8 review 
d. No No report because North plans to 
present comparative financial 
42. Each page of a nonissuer’s  statements for 20X8 and 20X7. 
financial statements reviewed  This separate paragraph should 
by an accountant should  indicate that 
include the following  a. The 20X8 review report
reference:  is intended solely for 
a. See   Accompanying   Accountant’s the information of 
Footnotes.  management and the 
b. Reviewed,   No   Material board of di­rectors. 
Modifications Required.  b. The   20X7   auditor’s
c. See Accountant’s Review Report.  report may  no  longer be
d. Reviewed,   No   Accountant’s relied on. 
Assurance Expressed.  c. No auditing procedures
43. Financial   statements   of   a were   performed   after
nonissuer   that   have   been the   date   of   the   20X7
reviewed   by   an   accountant auditor’s report. 
should be accompanied by a d. There   are   justifiable
report stating that a review  reasons for changing the
a. Provides   only   limited level of service from an
assurance   that   the audit to a review. 
financial statements are 46. An accountant’s standard report 
fairly presented.  on a review of the fi­nancial 
b. Includes examining, on statements of a nonissuer should 
a   test   basis, state that the accountant 
information that is the a. Does  not  express   an
representation   of opinion or any form of
management.  lim­ited   assurance   on
c. Consists   principally   of the   financial
inquiries   of   company statements. 
per­sonnel   and
analytical   procedures
applied   to   finan­cial
data. 
d. Does  not  contemplate   obtaining
corroborating evi­dential matter or
applying  certain  other  procedures
ordinarily   performed   during   an
audit. 
44. An accountant who had begun an 
audit of the financial statements 
of a nonissuer was asked to 
change the engage­ment to a 
review because of a restriction on 
the scope of the audit. If there is 
reasonable justification for the 
change, the accountant’s review 
report should include reference to 
the 
Scope limitation Original
that caused the engagement
changed engagement that was agreed to
a. Yes No
b. No Yes
c. No No
d. Yes Yes
d. A review is greater in scope than a
b. Is not aware of any  compilation, the objective of 
material  which is to present financial state­
modifications that  ments that are free of material 
should be made to  misstatements. 
the financial  48. During a review of the financial 
statements for them  statements of a nonis­suer, an 
to conform with  accountant becomes aware of a 
GAAP.  lack of adequate disclosure that 
c. Obtained   reasonable is material to the financial 
assurance   about statements. If management 
whether   the   financial refuses to correct the financial 
statements   are   free   of statement pre­sentations, the 
material   misstate­ accountant should 
ment.  a. Issue an adverse opinion. 
d. Examined evidence, on  b. Issue   an   “except   for”   qualified
a test basis, supporting  opinion. 
the amounts and 
c. Disclose this departure from 
disclosures in the 
generally accepted ac­counting 
financial state­ments. 
principles in a separate paragraph 
47. Financial statements of a  of the report. 
nonissuer that have been re­ d. Express only limited
viewed by an accountant should  assurance   on   the
be accompanied by a report  financial   statement
stating that  presentations. 
a. The scope of the inquiry 49. An   accountant   who   reviews   the
and analytical  financial   statements   of   a
procedures performed  nonissuer   should   issue   a   report
by the accountant has  stating that a review 
not been re­stricted.  a. Is substantially less in scope than
b. All information  an audit. 
included in the 
b. Provides   negative
financial state­ments is assurance   that   internal
the representation of  control is functioning as
the management of the designed. 
entity.  c. Provides   only   limited
c. A review includes  assurance   that   the
examining, on a test  financial statements are
basis, evi­dence  fairly presented. 
supporting the amounts d. Is substantially more in scope than
and disclosures in the  a compilation. 
financial statements. 
MODULE 6   ACCOUNTING AND REVIEW SERVICES 207

SIMULATIONS

Task­Based Simulation 1
Research
Authoritative
Literature Help

Auditors who audit public nonpublic companies must be familiar with professional 
standards developed by a variety of sources. For each of the types of services below indicate 
the proper source of professional requirements. Each source may be used once, more than 
once, or not at all.

S
o
A.
Acc
B.
Aud
C.
PC

1. A
2. A

3. A
4. A

5. A

6. A

7. A
8. A
9. A
Task­Based Simulation 2
Research
Authoritative
Literature Help
Review Reports
The president of Enright Corporation, a nonpublic client, asked you to perform a review of 
the financial statements for the current year only. You have now completed your inquiry and 
other review procedures and find that you can issue a standard review report.
Selections
A. AU
B. PCAOB
C. AT
D. AR
E. ET
F. BL
G. CS
H. QC

(A) (B) (C) (D) (E)  (F) (G) (H)
1. Which title of the Professional Standards presents a standard review report
on one year?
2. Enter the exact section and paragraph with the needed information.
208 MODULE 6   ACCOUNTING AND REVIEW SERVICES

MULTIPLE­CHOICE ANSWERS

1. d __ __ 10. c __ __ 19. a __ __ 28. b __ __ 37. b __ __ 46. b __ __


2. c __ __ 11. a __ __ 20. c __ __ 29. c __ __ 38. a __ __ 47. b __ __
3. c __ __ 12. a __ __ 21. b __ __ 30. b __ __ 39. c __ __ 48. c __ __
4. d __ __ 13. a __ __ 22. b __ __ 31. a __ __ 40. c __ __ 49. a __ __
5. a __ __ 14. b __ __ 23. d __ __ 32. b __ __ 41. d __ __
6. d __ __ 15. b __ __ 24. a __ __ 33. d __ __ 42. c __ __
7. d __ __ 16. a __ __ 25. a __ __ 34. b __ __ 43. c __ __
8. d __ __ 17. a __ __ 26. a __ __ 35. b __ __ 44. c __ __ 1st:  __/49 = __%
9. b __ __ 18. d __ __ 27. b __ __ 36. b __ __ 45. c __ __ 2nd: __/49 = __%

MULTIPLE­CHOICE ANSWER EXPLANATIONS
procedures. An­swer (d) is correct 
Compilation and Review—General because the Statements apply 
1. (d) The requirement is to identify  when a CPA either compiles or 
the engagement for which  reviews the financial statements of
Statements on Standards for  a nonissuer. Answer (a) is 
Accounting and Re­view Services  incorrect because the Statements 
establish standards and  do not apply when the CPA is 
assisting in adjusting the books of  5. (a) The requirement is to determine the 
account for a partnership or other  correctstatement concerning both an 
organization.  engagement to compile and an engagement 
Answer (b) is incorrect because the  to review a nonissuer’s financial 
Statements only apply to nonissuer  statements. Answer (a) is correct because 
(nonpublic) entities. Answer (c) is incorrect  neither a compilation nor a review 
because the Statements do not apply when  contemplates obtaining an understanding of
processing the financial data for clients of  internal control. Answer (b) is incorrect 
other accounting firms.  because when performing a compilation the
accountant need not be independent; 
2. (c) The requirement is to identify the 
independence is required for reviews. 
authoritative body designated to promulgate 
Answer (c) is incorrect because a review 
standards concerning an ac­countant’s 
provides limited assurance. An­swer (d) is 
association with unaudited financial 
incorrect because the accountant is not 
statements of an entity that is not required to required to obtain a written management 
file financial statements with an agency  representation letter for a compilation; a 
regulating the issuance of the entity’s  management representation letter is 
securities. Answer (c) is correct because the  required for a review.
Accounting and Review Services 
Committee is so designated. Answer (a) is  Compilation—General
incor­rect because the Financial Accounting 
Standards Board is the authoritative body  6. (d) The requirement is to 
designated to promulgate financial  determine whether repro­ducing 
accounting standards. Answer (b) is  client­prepared financial 
incorrect because Gov­ernment  statements without modifi­cation 
Accountability Office is not one of the  and preparing standard monthly 
bodies des­ignated by the AICPA to  journal entries are included in the
promulgate technical standards. Answer (d)  provisions of Statements on 
is incorrect because the Auditing Standards  Standards for Accounting and 
Board is the authoritative body designated to Review Services. Answer (d) is 
promulgate statements on auditing  correct because AR 100 allows 
standards. ET Appendix A presents the  these services as long as the ac­
bodies designated to promulgate technical  countant’s name is not associated
standards.  with the financial state­ments. 

3. (c) The requirement is to  7. (d) The requirement is to identify the 
determine which of the two  correct state­ment about unaudited personal 
listed accounting services an  financial statements included in a personal 
accountant may perform  financial plan. Answer (d) is correct because
without being required to issue a  AT 600 requires that the financial 
compilation or reviewreport under the  statements be used solely to assist the client 
Statements on Standards for Accounting and and the client’s advisor and not be used to 
Review Services. Answer (c) is correct  obtain credit. Answer (a) is incorrect 
because the Statements on Standards for  because financial statements may be 
Accounting and Review Ser­vices do not  presented in comparative form. An­swer (b) 
apply to preparing a working trial balance or is incorrect because omitted disclosures may
to preparing standard monthly journal  be material. Answer (c) is incorrect because 
entries. See SSARS for these and additional  such financial statements may be disclosed 
circumstances in which the standards do not  to a non­CPA financial planner. 
apply. Accordingly, no compilation or  8. (d) The requirement is to identify 
review report needs to be issued when these  the circumstance in which 
services are provided. Statements on Standards for 
4.  (d)   The   requirement   is   to   determine Accounting and Re­view Services 
whether   an   ac­countant   can   accept   a apply. The Standards apply when 
compilation   or   a   review   engagement   when a CPA submits unaudited financial
statements of a nonissuer (non­
s/he is unfamiliar with a prospective client’s
public) entity to his or her client 
special­ized   industry   accounting   principles,
or others. Accordingly, answer (d)
but s/he plans to obtain the required level of
is correct. Answers (a), (b), and 
knowledge prior to the engagement.
(c) are all incor­rect because they 
Answer (d) is correct because an accountant are all included as services that do
may accept either a compilation or a review  not constitute a submission of 
engagement in such circum­stances. financial statements to a client. 
9. (b) The requirement is to 
determine whether either or both 
of (1) the additional audit effort 
necessary to com­plete the audit  engagement be changed to a 
and (2) the reason for the change  compi­lation. Answer (b) is 
in the en­gagement should be  correct because SSARS states 
considered by a CPA whose  that additional necessary audit 
client has requested that an audit  effort and the reason for the 
MODULE 6   ACCOUNTING AND REVIEW SERVICES 209
when the plan 
(1) is to be used to assist the client and the 
change—as well as the additional  client’s advisors in developing financial 
cost to complete the audit—be  goals and objectives, (2) will not be used 
considered. to obtain credit, and (3) when nothing 
comes to the accountant’s attention that 
10. (c) The requirement is to determine  would lead him/her to believe that the 
when a CPAmay compile and be associated  statements will be used for credit or for 
with financial statements that omit  any other purposes. Answer (b) is incorrect
disclosures required by GAAP. Answer (c)  because any work per­formed on the prior 
is correct because the CPA may compile  year statements is not applicable to the 
such financial statements provided that the  statements of the current year. Answer (c) 
omission of substantially all disclosures (1)  is incorrect be­cause the engagement letter
is clearly indicated in the audit report and  need not acknowledge depar­tures from 
(2) is not, to the CPA’s knowledge,  GAAP. Answer (d) is incorrect because no
undertaken with the intention of mis­leading as­surance is provided in such reports. 
those who might reasonably be expected to 
use the financial statements. 14. (b) The requirement is to identify 
an accountant’s reporting 
Compilation Procedures responsibility when performing a 
compilation and he or she 
11. (a) The requirement is to determine an 
determines that a going concern 
accountant’s responsibility relating to 
uncertainty has not been properly 
knowledge of the client’s account­ing 
principles and practices when performing a  disclosed by a client. Answer (b) is 
compilation. Answer (a) is correct because  cor­rect because this is a departure 
to compile financial state­ments the  from GAAP and results in 
accountant should possess a general  modification of the report’s third 
understanding of the nature of the entity’s  paragraph and addition of an 
business transactions, the form of its  explanatory paragraph. Answer (a) 
accounting records, the stated qualifications  is incorrect because the information
of its ac­counting personnel, the accounting  on the client’s ability to continue as
basis on which the finan­cial statements are  a going concern should be included
to be presented, and the form and content of  in the financial statements; if it is 
the financial statements. Answer (b) is  not, a departure from GAAP exists.
incorrect because the accountant need not  Answer (c) is incor­ 
have a general understanding of the entity’s 
controls. Answer (c) is incorrect because no 
such consideration of risk factors is 
envisioned in a compilation. Answer (d) is 
incorrect because no such consideration of 
internal control awareness of senior 
management is made. 
12. (a) The requirement is to identify 
the procedure an accountant 
ordinarily performs in a 
compilation engagement of a 
nonissuer. Answer (a) is correct 
because the accountant is required,
at a minimum, to read the financial
statements to consider whether 
they are free from obvious 
material errors. 
Compilation Reporting
13. (a) The requirement is to identify a 
condition re­quired for an 
accountant to submit a written 
personal finan­cial plan containing 
unaudited financial statements to a 
cli­ent without complying with the 
compilation and review 
requirements presented in SSARS. 
Answer (a) is correct because 
SSARS allow such an exception 
accountant must clearly indicate 
rect because adverse opinions are not issued  in the com­pilation report that 
based on com­pilations. Answer (d) is  substantially all disclosures 
incorrect because qualified opinions are not  required by GAAP have been 
issued based on compilations. omitted. Answer (b) is incorrect 
because the financial statements 
15. (b) The requirement is to identify  are not compiled on a comprehen­
the statement that should be  sive basis other than GAAP. 
included in a compilation report.  Answer (c) is incorrect be­cause a
Answer (b) is correct because  compilation may be performed on
compilation reports indicate that  financial state­ments lacking such
the ac­countant compiled the  disclosures. Answer (d) is 
financial statements in accordance incorrect because these financial 
with Statements on Standards for  statements are not considered 
Accounting and Review Services.  “special­purpose financial 
16. (a) The requirement is to  statements.” 
determine how a CPA should  18. (d) The requirement is to 
indicate that a client’s compiled  determine the proper re­porting 
financial statements were  procedure for comparative 
prepared in conformity with the  financial statements for which the 
income tax basis of accounting  prior year is unaudited, and the 
when the financial statements  current year is audited. AU 504 
provide no such disclosure.  states that when unaudited 
Answer (a) is correct because if  financial state­ments are presented
the basis of accounting is not  in comparative form with audited 
disclosed in the financial  finan­cial statements, the report on
statements, the accountant should  the prior period may be reis­sued 
disclose the basis in the  to accompany the current period 
compilation re­port. Answer (b) is report. In addition, the report on 
incorrect because each page of the the current period may also 
fi­nancial statements should only  include a separate paragraph 
include the reference “See  describing responsibility assumed 
Accountant’s Compilation  for the prior period financial 
Report.” Answer (c) is incorrect  statements. If these statements are 
because the auditor is not required filed with the SEC, the statements 
to issue a special report. See AU  should be clearly marked as 
623 for information on special  “unaudited” but should not be 
reporting. An­swer (d) is incorrect referred to in the auditor’s report. 
because the auditor does not have 
to withdraw from the  19. (a) The requirement is to determine a CPA’s
engagement.  re­sponsibility when the first year of 
compiled comparative financial statements 
17. (a) The requirement is to  omit substantially all disclosures re­quired 
determine how the com­pilation  by generally accepted accounting principles,
report should be modified to  while the second year’s statements include 
indicate that the en­tity’s financial such disclosures. An­swer (a) is correct 
statements do not include all  because the CPA may not report on the 
disclosures re­quired by GAAP.  comparative financial statements because of
Answer (a) is correct because AR  a lack of com­parability. Answers (b), (c), 
100 states that while the  and (d) are all incorrect because they allow 
accountant may compile such  such reporting to occur under certain 
financial statements, the  circum­stances. 
210 MODULE 6   ACCOUNTING AND REVIEW SERVICES
appropriate communication 
option(s) when compiled financial 
20. (c) The requirement is to identify statements are only going to be used
the  statement that should  not  be by management. Answer (a) is 
included   in   a   CPA’s   financial correct because when an ac­
statement   com­pilation   report. countant submits to a client 
Answer   (c)   is   correct   because   a compiled financial statements that 
compilation   report   provides   no are not expected to be used by a 
assurance   on   the   financial third party, either a compilation 
statements.  report or a written engagement letter
(or both) may be used. Answer (b), 
21. (b) The requirement is to  (c), and (d) are all incorrect be­
determine the represen­tations  cause they suggest that either a 
made explicitly and implicitly  compilation report, a written 
when issuing a stan­dard  engagement letter, or both are 
compilation report on a 
unacceptable. 
nonissuer’s financial state­ments. 
Answer (b) is correct because the  26. (a)   The   requirement   is   to   identify   the
report explicitly states that the  circumstance in which a compilation report
financial statements have not been  is   not   required.   Answer   (a)   is   correct
audited and that the accountant has because when financial statements are only
compiled them.  for man­agement, no compilation report  is
required. Answers (b) and 
22. (b) The requirement is to determine the 
(c) are incorrect because when third 
appropriate date for an auditor’s compilation
parties are expected to use compiled 
report. SSARS require that the date of 
financial statements, a compilation report 
completion of the compilation should be 
is required. Answer (d) is incorrect 
used.  because a compilation report is not always
23. (d) The requirement is to  required. 
determine whether an ac­ Review—General
countant’s compilation report 
must be printed on the ac­ 27. (b) The requirement is to determine the 
countant’s letterhead, manually  effect of animmaterial direct financial 
signed by the accountant, or both.  interest on accountant indepen­dence. 
Answer (d) is correct because the  Answer (b) is correct because even 
professional stan­dards require  immaterial direct financial interests impair 
neither that the report be printed  the independence that is required for the 
on the ac­countant’s letterhead,  performance of reviews and other 
nor that it be manually signed by  attestation ser­vices. Answer (a) is incorrect
the accountant. Answers (a), (b),  because a CPA who lacks independence 
and (c) are all incorrect be­cause  may compile those financial statements 
they include an inappropriate  when this lack of independence is disclosed
“yes” to one or both issues. See  in the compilation
SSARS for information on 
reporting on a com­pilation of 
financial statements. 
24. (a) The requirement is to identify 
the correct state­ment concerning 
compiled financial statements that 
are to be made available to third 
parties. Answer (a) is correct be­
cause a compilation report must be
issued when third­party reliance 
upon compiled financial 
statements is anticipated. Answer 
(b) is incorrect because note 
disclosures may be omitted. 
Answer (c) is incorrect because 
while advisable, use of an 
engagement letter is not required 
in such circum­stances. Answer 
(d) is incorrect because no such 
restriction is necessary. 
25. (a) The requirement is to identify 
the correct state­ment concerning 
procedures. Answer (c) is 
report. Answer (c) is incorrect because a  incorrect because a review in­
review report may not be issued. Answer  cludes inquiries of management. 
(d) is incorrect because the CPA is not  Answer (d) is incorrect because 
independent. auditors obtain written 
representation from man­agement 
28. (b) The requirement is to identify the  when performing a review. 
circumstance in which a CPA may issue a 
review report on a single finan­cial  31. (a) The requirement is to identify 
statement. Answer (b) is correct because an  the procedure least likely to be 
accountant may issue a review report on a  performed in a review of the 
financial statement, such as a balance sheet,  financialstatements of a 
and not report on the other related financial  nonissuer. Answer (a) is correct 
statements if the scope of his or her inquiry  because a review of a nonissuer 
and analytical procedures has not been  financial statements does not 
restricted. Answer (a) is incorrect because  specifically address observing the 
the balance sheet need not be presented in  safeguards over access to and use 
pre­scribed form. Answer (c) is incorrect  of assets and records. Answers 
because the balance sheet may be used to  (b), (c), and (d) are all incorrect 
obtain credit or to distribute to credi­tors.  because they are included in the 
Answer (d) is incorrect because specialized  procedures suggested for a review
accounting principles and practices in an  by SSARS. 
industry may or may not need to be 
disclosed depending upon the  32. (b) The requirement is to identify 
circumstances.  the procedures that an accountant 
would perform during an 
29. (c) The requirement is to identify a CPA’s  engagement to review the 
responsi­bility when he or she believes that  financial statements of a 
modification of the stan­dard review report  nonissuer. Answer (b) is correct 
is not adequate to indicate deficiencies in  because AR 100 requires that the 
financial statements affected by illegal acts.  CPA obtain a representation letter.
Answer (c) is correct because whenever a  Answers (a), (c), and (d) are 
CPA believes that modification of the  incorrect because they are not 
standard report is not adequate to indicate  included in SSARS which present 
the defi­ciencies in the financial statements,  a list of procedures performed 
he or she should with­draw from the review  during a review. 
engagement and provide no further services 
with respect to those financial statements.  33. (d) The requirement is to 
determine whether ana­lytical 
Review Procedures procedures need to be performed 
on a compilation and/or a review 
30. (b) The requirement is to identify  engagement. Answer (d) is correct
the procedure not followed by an  because a compilation does not 
accountant in obtaining a  require performance of analytical 
reasonable basis for the  procedures, while a review does. 
expression of limited assurance 
for a review of fi­nancial  34. (b) The requirement is to 
statements. Answer (b) is correct  determine the type of in­quiry or 
because reviews ordinarily do not  analytical procedures ordinarily 
include an assessment of the risk  performed in an engagement to 
of fraud. Answer (a) is incorrect  review a nonissuer’s financial 
because reviews include analytical statements. Answer (b) is correct
because an accountant will make
MODULE 6   ACCOUNTING AND REVIEW SERVICES 211
is incorrect because reviews 
consist primarily of inquiry and 
inquiries concerning the entity’s procedures  analytical procedures and do not 
for recording, classifying, and summarizing  generally include confir­mation of
transactions, and accumulating information for accounts receivable. Answer (c) is
disclosure in the financial statements. Answer  incorrect be­cause a review of a 
(a) is incorrect because the analytical  nonpublic entity does not 
procedures and other procedures involved in a  normally include obtaining an 
review do not in general obtain corroborating  understanding of internal control 
audit evidence as do the procedures of an  or as­sessing control risk. Answer
audit. Answer (c) is incorrect because the  (d) is incorrect because distri­
procedures for reviews are not specially  bution of a review report need not
designed to test management’s assertion  be limited. See SSARS for 
regarding continued existence. Answer (d) is  specific procedures included in 
incorrect because inquiries of the entity’s  reviews. 
attorney are not normally required when a  Review Reporting
review is being performed.
38. (a) The requirement is to determine a 
35. (b) The requirement is to 
CPA’sresponsibilities when performing a 
determine the most likely  review of a nonissuer’s financial statements 
procedures to be included in a  that contain uncorrected departures from 
review engagement of a nonissuer. GAAP and the CPA believes that the review
Answer (b) is correct because a  report is not adequate to indicate the 
review consists primarily of  deficiencies. Answer (a) is correct because 
inquiries and analytical  SSARS state that in such circumstances the 
procedures. Answer (a) is incorrect CPA should withdraw from the engagement
because a bank transfer schedule is and provide no further services with respect 
generally not prepared for a review to those financial statements. Answers (b) 
engagement. Answer (c) is  and (d) are incorrect because restricting 
incorrect because a review does  distri­bution is not adequate or appropriate 
not include assessing the control  in such a circumstance. Answer (c) is 
structure. Answer (d) is incorrect  incorrect because the standards on special 
because cutoff tests on sales and  reports do not apply in this circumstance.
purchases are not normally 
performed on a review. Note that 
the procedures included in answers
(a), (c), and (d) are typically 
performed in an audit. 
36. (b) The requirement is to determine the type 
of transaction the accountant is most likely 
to investigate dur­ing a review when the 
year’s accounts receivable did not conform 
to a predictable pattern. Answer (b) is 
correct be­cause accounts receivable are 
generated from credit sales and an 
accountant would therefore investigate them.
An­swer (a) is incorrect because sales 
returns and allowances would be less likely 
to cause large shifts in accounts receiv­able 
than credit sales. Answer (c) is incorrect 
because it is less complete than answer (b) 
since sales of consigned goods represent 
only one possible type of sale that might 
impact accounts receivable. Answer (d) is 
incorrect because cash sales do not affect 
accounts receivable. 
37. (b) The requirement is to identify 
the most likely procedure to be 
included in a review of a 
nonissuer’s financial statements. 
Answer (b) is correct because 
SSARS state that reviews 
ordinarily include inquires 
concerning actions taken at board 
of directors’ meetings. Answer (a)
deficiencies, or both. Answer (d) is 
39. (c) The requirement is to  correct because neither of these 
determine the listed re­quirement  circumstances requires modification
when an accountant is providing  of a review report. Answers (a), (b), 
limited assur­ance that the  and (c) are all incorrect because 
financial statements of a nonissuer  they suggest that one or the other of 
require no material modifications  these circum­stances results in 
to be in accordance with generally  modification of a review report. A 
accepted accounting principles.  depar­ture from GAAP is the 
Accountants perform reviews to  primary cause of a review report 
provide such limited assurance.  modification. SSARS provide 
Answer (c) is correct because  guidance on review reports. 
obtaining an understanding of the 
ac­counting principles in the  42. (c) The requirement is to identify 
industry is required for reviews.  the reference that should be 
See SSARS for this and other  included in each page of a 
requirements. Answer (a) is  nonissuer’s reviewed financial 
incorrect because reviews do not  statements. SSARS require that 
require the accountant to assess the each page of the financial 
risk of material misstatement.  statements include a reference 
Answer (b) is in­correct because  such as “See Accountant’s 
reviews generally do not include  Review Report.” 
any com­munication with the 
43. (c) The requirement is to identify 
entity’s lawyer. Answer (d) is 
incorrect because an “audit”  the statement that is included in an
program is not required since a  accountant’s review report on the 
review is being performed.  financial statements of a 
nonissuer. Answer (c) is correct 
40. (c) The requirement is to identify the  because a review report includes a
circumstances in which a CPA may issue a  statement that a review consists 
review report on the balance sheet of a  principally of inquires of company
nonissuer, and not report on the related  personnel and analytical 
financial statements. Answer (c) is correct  procedures applied to financial 
because an accountant may issue a review  data. See SSARS for infor­mation 
report on one financial statement and not on  that should be included in a 
the other related statements if the scope of  review report. 
the inquiry and analytical procedures has not
44. (c) The requirement is to determine 
been restricted. 
a CPA’s report­ing responsibility 
Answer (a) is incorrect because the CPA 
when an audit engagement for a 
need not compile or report on the related 
statements of income, retained earnings, and nonissuer has been changed to a 
cash flows when reviewing only the balance  review engagement because of what 
sheet. Answer (b) is incorrect because, when the CPA believes to be a reasonable 
material modi­fications are needed, a CPA  restriction on the scope of the audit. 
may still report on the balance sheet, but  Answer (c) is correct because in 
must indicate the modifications in the  such circumstances the CPA should 
review re­port. Answer (d) is incorrect  neither include reference to the 
because the client need not be new.  original engagement nor to the 
scope limitation. 
41. (d) The requirement is to identify 
whether a review report is modified  45. (c) The requirement is to identify
due to either inability to assess the  the correct state­ment relating to
risk of material misstatement due to  a   CPA’s   report   on   comparative
fraud, a discovery of internal control statements 
212 MODULE 6   ACCOUNTING AND REVIEW SERVICES

when the current year has been reviewed and the previous 
year has been audited. Answer (c) is correct because when a 
separate paragraph is being added to the CPA’s review re­
port the CPA should clearly indicate the difference in the 
levels of assurance for the two years. In this situation, 
SSARS require the auditor to indicate that the previous 
year’s financial statements were audited, the date of the re­
port, the type of opinion expressed and if the opinion was 
other than unqualified, the substantive reasons for that 
opinion, and that no auditing procedures were performed 
after the date of the previous report. Answer (a) is incorrect 
because the review report is not solely intended for man­
agement or the board of directors. Answer (b) is incorrect 
because the prior year’s audit report may still be appropriate.
Answer (d) is incorrect because this statement does not need
to be included within the review report.
46. (b) The requirement is to identify the statement 
in­cluded in the standard report issued by an 
accountant after reviewing the financial 
statements of a nonissuer. An­swer (b) is correct 
because the report states that the accoun­tant is 
not aware of any material modifications that 
should 
be made to the financial statements in order for them to be in
conformity with generally accepted accounting principles. 
SSARS present the required disclosures for a review report. 
47. (b) The requirement is to identify the information 
presented in a review report of financial statements
of a nonissuer. Answer (b) is correct because the 
report indicates that all information included in the 
financial statements is the representation of the 
management of the entity. 
48. (c) The requirement is to determine an 
accountant’sreporting responsibility when 
associated with a nonissuer’s reviewed statements 
which contain a material departure from generally 
accepted accounting principles. Answer (c) is 
correct because SSARS require the inclusion of a 
separate paragraph describing the departure. 
Answers (a) and (b) are incorrect because an 
adverse opinion or an “except for” qualified 
opinion may only be issued when an audit has been
performed. Answer (d) is incorrect because 
expressing limited assurance (as is normally 
provided in reviews) on the financial statements is 
not adequate to disclose the departure. 
49. (a) The requirement is to identify the reply which
is   correct   concerning   the   content   of   a   review
report. An­ 
swer (a) is correct because a review report indicates that a 
review is substantially less in scope than an audit. An­swer (b) 
is incorrect because a review report provides no information on
internal control. Answer (c) is incorrect because while a review
report states that the accountant is not aware of any material 
modifications that should be made to the financial statements, it
does not provide limited assur­ance that the financial 
statements are fairly presented. An­ 
swer (d) is incorrect because while a review report does state
that a review is substantially less in scope than an audit, it 
does not refer to a compilation. 
MODULE 6   ACCOUNTING AND REVIEW SERVICES 213

S
O
T

R
e
A
u
L
i

1. A
2. A
3. A
4. A

5. A

6. A

7. A
8. A

9. A
T

Research
Authoritative
Literature

1. W
h
2. E
214 MODULE 7   AUDIT SAMPLING

AUDIT SAMPLING
justifies such an 
MULTIPLE­CHOICE QUESTIONS (1­
58)
assessment. 
b. Contains 
1. An advantage of using  misstatements that 
statistical over  could be material 
nonstatistical sampling  to the financial 
methods in tests of  statements when 
controls is that the  aggregated with 
statistical methods  mis­statements in 
a. Can   more other account 
easily   convert balances or 
the sample into transac­tions 
a   dual­purpose classes. 
test   useful   for c. Contains 
substantive proportionately 
testing.  fewer monetary 
b. Eliminate   the errors or 
need   to   use deviations from 
judgment   in prescribed 
determining controls than exist
appropriate in the balance or 
sample sizes.  class as a whole. 
c. Afford   greater d. Does  not  support
assurance   than   a the   tolerable   error
nonstatistical sam­ for some or all of
ple of equal size.  management’s
d. Provide   an assertions. 
objective basis for
quantitatively 4. The   risk   of   incorrect
evalu­ating sample acceptance   and   the
risk.  likelihood   of   assessing
control risk too low relate
2. An   advantage   of   statistical to the 
sampling   over   nonstatistical a. Allowable   risk   of   tolerable
sampling   is   that   statistical misstatement. 
sampling helps an auditor to  b. Preliminary   estimates   of
a. Eliminate   the   risk   of materiality levels. 
nonsampling errors.  c. Efficiency of the audit. 
b. Reduce the level d. Effectiveness of the audit. 
of audit risk and
materiality   to   a 5. Which   of   the   following
relatively   low best   illustrates   the
amount.  concept of sampling risk?
c. Measure   the a. A randomly 
sufficiency   of chosen sample 
the   evidential may not be 
matter representa­tive of 
obtained.  the population as a
d. Minimize   the   failure   to whole on the 
detect errors and fraud.  character­istic of 
interest. 
3. The   likelihood   of b. An   auditor   may
assessing   control   risk   too select   audit
high   is   the   risk   that   the procedures that are
sample   selected   to   test not  appropriate   to
controls 
achieve   the
a. Does not support 
specific objective. 
the auditor’s 
c. An   auditor   may   fail   to
planned assessed 
recognize errors in the docu­
level of control  ments   examined   for   the
risk when the true chosen sample. 
operating effec­
d. The   documents
tiveness of the 
related   to   the
control structure 
chosen   sample Items 7 and 8 are based on the 
may  not  be following information:
available   for
The diagram below depicts the 
inspection.
auditor’s estimated de­viation rate 
6. In   assessing   sampling compared with the tolerable rate, 
risk, the risk of incorrect and also de­picts the true population
rejec­tion and the risk of deviation rate compared with the 
assessing control risk too tolerable rate.
high relate to the  True State of
a. Efficiency of the audit.  Population
b. Effectiveness of the audit.  Deviation rate Deviation rate is
c. Selection of the sample.  exceeds tolerable less than tolerable
d. Audit quality controls.  rate rate
Auditor’s
estimate
based on
sample results
Deviation rate I. II.
exceeds
tolerable
rate

Deviation rate
is less than
tolerable III. IV.
rate

7. In which of the situations 
would the auditor have 
prop­erly concluded that 
control risk is at or below 
the planned assessed level? 
a. I. 
b. II. 
c. III. 
d. IV. 
8. As a result of tests of 
controls, the auditor 
assesses con­trol risk too 
high and thereby increases 
substantive testing. This is 
illustrated by situation 
a. I. 
b. II. 
c. III. 
d. IV. 
9. While performing a test of 
details during an audit, an 
auditor determined that the 
sample results supported 
the conclusion that the 
recorded account balance 
was materially misstated. It 
was, in fact, not materially 
misstated. This situation 
illustrates the risk of 
a. Assessing   control   risk   too
high. 
b. Assessing   control   risk   too
low. 
c. Incorrect rejection. 
d. Incorrect acceptance. 
10. The   size   of   a   sample
designed for dual purpose
testing should be 
a. The larger of the  samples that 
samples that  would oth­erwise 
would otherwise  have been 
have been  designed for the 
designed for the  two separate 
two separate  purposes. 
purposes.  d. More than the larger of the 
b. The smaller of the  samples that would oth­
samples that  erwise have been designated
would otherwise  for the two separate 
have been  purposes, but less than the 
combined total of the 
designed for the 
samples that would 
two separate 
otherwise have been 
purposes.  designed for the two 
c. The combined  separate purposes. 
total of the 
MODULE 7   AUDIT SAMPLING 215
c. Item   in   the
accounting
11. The   expected   population population
deviation   rate   of   client should   have   an
billing   errors   is   3%.   The opportunity to be
auditor   has   established   a selected. 
tolerable rate of 5%.  In  the d. Item   must   be
review of client invoices the systematically
auditor should use  selected   using
a. Stratified sampling.  replace­ment. 
b. Variable sampling.  15. Which   of   the   following
c. Discovery sampling.  statistical   selection
d. Attribute sampling.  techniques   is  least
12. Which of the following  desirable   for   use   by   an
sampling methods would  auditor? 
be used to estimate a  a. Systematic selection. 
numerical measurement of  b. Stratified selection. 
a population, such as a  c. Block selection. 
dollar value?  d. Sequential selection. 
a. Attribute sampling. 
16. Which   of   the   following
b. Stop­or­go sampling. 
statistical   sampling   plans
c. Variables sampling. 
does  not  use   a   fixed
d. Random­number sampling. 
sample   size   for   tests   of
13. For which of the following controls?
audit tests would an auditor a. Dollar­unit sampling. 
most   likely   use   attribute b. Sequential sampling. 
sampling?  c. PPS sampling. 
a. Making   an   independent d. Variables sampling. 
estimate of the amount of a
LIFO inventory.  17. If   certain   forms   are   not
b. Examining consecutively numbered 
invoices   in a. Selection   of   a
support   of   the random   sample
valuation   of probably   is   not
fixed   asset pos­sible. 
additions.  b. Systematic sampling may be
c. Selecting appropriate. 
accounts c. Stratified   sampling   should
receivable   for be used. 
confirmation   of d. Random   number   tables
account cannot be used. 
balances. 
d. Inspecting 18. When performing a test of
employee a control with respect to 
time cards for control over cash receipts,
proper   ap­ an auditor may use a 
proval   by systematic sampling 
supervisors.  technique with a start at 
any randomly selected 
14. An   underlying   feature item. The biggest 
of   random­based disadvantage of this type 
selection of items is that of sampling is that the 
each  items in the population 
a. Stratum   of a. Must   be
the systematically
accounting replaced   in   the
population   be population   after
given   equal sampling. 
representation
in the sample.
b. Item   in   the
accounting
population   be
randomly   or­
dered. 
d. More   than   the
b. May deviation   rate   in
systematically the   auditor’s
occur   more sam­ple. 
than once in the
sample.  21. Which of the following
c. Must   be   recorded factors   is(are)
in   a   systematic considered   in
pattern   before   the determining   the   sample
sample   can   be size   for   a   test   of
drawn.  controls? 
d. May   occur   in   a Expected Tolerable
systematic deviation rate deviation rate
pattern,   thus a. Yes Yes
destroying   the b. No No
sample c. No Yes
randomness.  d. Yes No

19. What   is   the   primary   objective   of 22. Which of the following statements is


using   stratification   as   a   sampling correct   concerning   statistical
method in auditing?  sampling in tests of controls? 
a. To increase the  a. Deviations   from
confidence level  control   procedures
at which a deci­ at   a   given   rate
sion will be  usually   result   in
reached from the  misstatements   at   a
results of the  higher rate. 
sample selected.  b. As   the
b. To   determine   the population size
occurrence   rate doubles,   the
for   a   given   char­ sample   size
acteristic   in   the should   also
population   being double. 
studied.  c. The   qualitative
c. To   decrease   the   effect   of aspects   of
variance   in   the   total   popu­ deviations   are
lation.  not  con­sidered
d. To   determine   the by the auditor. 
precision range of d. There   is   an
the   sample   se­ inverse
lected.  relationship
between the sam­
20. As a result of tests of  ple   size   and   the
controls, an auditor  tolerable rate. 
assessed con­trol risk too 
low and decreased  23. In determining the sample 
substantive testing. This  size for a test of controls, 
assessment occurred  an auditor should consider 
because the true deviation  the likely rate of 
rate in the population was  deviations, the allowable 
a. Less   than   the   risk risk of assessing control 
of   assessing risk too low, and the 
control   risk   too a. Tolerable deviation rate. 
low,   based   on   the b. Risk   of   incorrect
auditor’s sample.  acceptance. 
b. Less than the deviation rate in c. Nature   and   cause   of
the auditor’s sample.  deviations. 
c. More   than   the d. Population size. 
risk   of
assessing 24. Which   of   the   following
control   risk combinations results in a
too low, based de­crease in sample size
on   the in   a   sample   for
auditor’s attributes? 
sample. 
Risk of
assessing Expected
control risk Tolerable population
too low rate deviation rate
a. Increase Decrease Increase
b. Decrease Increase Decrease
216 MODULE 7 AUDIT SAMPLING

c. Increase Increase Decrease d.   Unrelated to the expected rate of errors in the re­


d. Increase Increase Increase lated accounting records.
deviations from control 
25. An auditor is testing internal control  procedures in­crease the 
procedures that are evidenced on an  risk of material 
entity’s vouchers by matching  misstatements, such 
random numbers with voucher  deviations do not 
numbers. If a random number  necessarily result in errors. 
matches the number of a voided  This explains why 
voucher, that voucher ordi­narily 
a. A recorded 
should be replaced by another 
disbursement 
voucher in the random sample if the 
that does not 
voucher 
a. Constitutes a deviation.  show evi­dence 
of required 
b. Has been properly voided. 
approval may 
c. Cannot be located. 
nevertheless be a
d. Represents   an   immaterial
transaction that 
dollar amount. 
is properly 
26. An auditor plans to examine authorized and 
a sample of twenty pur­ re­corded. 
chase orders for proper  b. Deviations would 
approvals as prescribed by  result in errors in 
the cli­ent’s control  the accounting 
procedures. One of the  records only if the 
purchase orders in the  deviations and the 
chosen sample of twenty  errors oc­curred 
cannot be found, and the  on different 
auditor is unable to use  transactions. 
alternative procedures to  c. Deviations from 
test whether that pur­chase  pertinent control 
order was properly  procedures at a 
approved. The auditor  given rate 
should  ordinarily would 
a. Choose   another be expected to 
purchase   order   to result in errors at a
replace   the   miss­ higher rate. 
ing purchase order d. A   recorded
in the sample.  disbursement
b. Consider this test  that   is   properly
of control invalid  autho­rized  may
and proceed with  nevertheless   be
substantive tests  a   transaction
since internal  that   con­tains   a
control cannot be  material error. 
relied upon.  28. The   objective   of   the
c. Treat   the   missing tolerable   rate   in   sampling
purchase   order   as for   tests   of   controls   of
a deviation for the internal control is to 
purpose   of a. Determine   the
evaluating   the probability of the
sample.  auditor’s   conclu­
d. Select   a sion   based   upon
completely   new reliance factors. 
set   of   twenty b. Determine
purchase or­ders.  that   financial
27. When assessing the  statements
tolerable rate, the auditor  taken   as   a
should consider that, while  whole are not
materially   in
error.  30. If the auditor is concerned 
c. Estimate   the   reliability   of that a population may con­
substantive tests.  tain exceptions, the 
d. Estimate   the determination of a sample 
range   of size sufficient to include at 
procedural least one such exception is a 
deviations   in   the characteristic of 
population. 
a. Discovery sampling. 
29. The   tolerable   rate   of b. Variables sampling. 
deviations   for   a   test   of   a c. Random sampling. 
control is generally  d. Dollar­unit sampling. 
a. Lower than the
expected   rate 31. In determining the number 
of errors in the of documents to select for a
re­lated test to obtain assurance that
accounting all sales have been properly
records.  authorized, an auditor 
b. Higher than the should consider the 
expected rate of tolerable rate of deviation 
errors in the re­ from the control activity. 
lated The auditor should also 
accounting consider the 
records.  I. Likely rate of deviations. 
c. Identical   to   the AI. Allowable risk of assessing control
expected rate of risk too high. 
errors in related a. I only. 
accounting b. II only. 
records.  c. Both I and II. 
d. Either I or II. 
  An auditor should consider the tolerable
rate of devia­tion when determining the 
number of check requests to se­lect for 
a test to obtain assurance that all check 
requests have been properly authorized. 
The auditor should also consider 
The allowable
The average dollar value risk of assessing
of the check requests control risk too low
a. Yes Yes
b. Yes No
c. No Yes
d. No No
33. Which of the following statements is
correct   concerning   statistical
sampling in tests of controls? 
a. As   the
population   size
increases,   the
sample   size
should   increase
proportionately.
b. Deviations from 
specific internal 
control proce­
dures at a given 
rate ordinarily 
result in 
misstate­ments 
at a lower rate. 
c. There   is   an
inverse
relationship
between   the
ex­pected
population
deviation   rate expected 
and the sample population de­
size.  viation rate. 
d. In   determining b. Accept the 
tolerable   rate,  an sample results as 
auditor   considers support for the 
detection   risk planned assessed
and   the   sample level of control 
size.  risk because the 
sample deviation
34. What is an auditor’s  rate plus the 
evaluation of a statistical  allowance for 
sample for attributes when sam­pling risk 
a test of fifty documents  exceeds the 
results in three deviations  tolerable rate. 
if tolerable rate is 7%, the  c. Accept the sample
expected population  results as support 
deviation rate is 5%, and  for the planned 
the allowance for  assessed level of 
sampling risk is 2%? 
control risk 
a. Modify the 
because the 
planned assessed 
tolerable rate less 
level of control 
the allowance for 
risk because the 
tolerable rate plus sampling risk 
the allowance for  equals the 
sampling risk  expected 
exceeds the  population 
deviation rate. 
MODULE 7   AUDIT SAMPLING 217
b. 4 1/2% 
c. 3 1/2% 
d. Modify   the   planned d. 1% 
assessed   level   of   control
risk   because   the   sample 37. Which of the following statements is
deviation   rate   plus   the correct   concerning   statistical
allow­ance   for   sampling sampling in tests of controls? 
risk exceeds the tolerable a. The population 
rate.  size has little or 
no effect on deter­
Items 35 and 36 are based on the  mining sample 
following: size except for 
An auditor desired to test  very small popula­
credit approval on 10,000 sales  tions. 
invoices processed during the  b. The expected 
year. The auditor de­signed a  population 
statistical sample that would  deviation rate has 
provide 1% risk of assessing  little or no effect 
control risk too low (99%  on determining 
confidence) that not more than 7% sample size except
of the sales invoices lacked  forvery small 
approval. The auditor estimated  populations. 
from previous experience that  c. As   the
about population size
2 1/2% of the sales invoices lacked  doubles,   the
approval. A sample of 200 invoices  sample   size
was examined and 7 of them were  also   should
lacking ap­proval. The auditor then  double. 
determined the achieved upper  d. For a given 
precision limit to be 8%. tolerable rate, a 
larger sample 
35. In the evaluation of this  size should be 
sample, the auditor  selected as the 
decided to increase the  expected 
level of the preliminary  population de­
assessment of control risk  viation rate 
because the  decreases. 
a. Tolerable   rate
(7%)   was   less 38. When   an   auditor   has
than   the   achieved chosen a random sample
up­per   precision and   is   using
limit (8%).  nonstatistical   attributes
b. Expected sampling, that auditor 
deviation   rate a. Need  not consider
(7%) was more the   risk   of
than   the assessing   control
percentage   of risk too low. 
errors   in   the b. Has   committed   a
sample   (3 nonsampling error. 
1/2%).  c. Will   have   to
c. Achieved use   discovery
upper precision sampling   to
limit (8%) was evaluate   the
more   than   the results. 
percentage   of d. Should   compare
errors   in   the the   deviation   rate
sample  of   the   sample   to
(3 1/2%).  the   tolerable
d. Expected deviation rate. 
deviation   rate   (2 39. How would increases in 
1/2%)   was   less
tolerable misstatement and 
than   the   tolerable
assessed level of control 
rate (7%). 
risk affect the sample size 
36. The allowance for sampling risk in a sub­stantive test of 
was  details? 
a. 5 1/2%  Increase in tolerable Increase in assessed
misstatement level of control risk
a. Increase sample size Increase sample size
c. Decrease sample size Increase sample size
b. Increase sample size Decrease sample size
d. Decrease sample size Decrease sample size
40. Which of the following 
courses of action would an 
auditor most likely follow 
in planning a sample of 
cash dis­bursements if the 
auditor is aware of several 
unusually large cash 
disbursements? 
a. Set  the   tolerable
rate of deviation
at   a   lower   level
than   originally
planned. 
b. Stratify   the   cash
disbursements
population so that
the   unusually
large
disbursements   are
selected. 
c. Increase   the
sample   size   to
reduce   the   effect
of   the   unusually
large
disbursements. 
d. Continue   to   draw
new  samples  until
all   the   unusu­ally
large
disbursements
appear   in   the
sample. 

41. Which   of   the   following


sample   planning   factors
would influence the sample
size for a substantive test of
details   for   a   specific
account? 
Expected Measure of
amount of tolerable
misstatements misstatement
a. No No
b. Yes Yes
c. No Yes
d. Yes No
42. When planning a sample 
for a substantive test of 
de­tails, an auditor should
consider tolerable 
misstatement for the 
sample. This 
consideration should 
a. Be   related   to   the   auditor’s
business risk. 
b. Not   be   adjusted   for
qualitative factors. 
c. Be   related   to
preliminary
judgments   about
materi­ality
levels. 
d. Not   be   changed   during   the the population. 
audit process.  c. The acceptable level of risk.
d. The   estimated
43. A number of factors  rate   of
influences the sample size  misstatements   in
for a substantive test of  the popula­tion. 
details of an account 
balance. All other factors  45. An auditor established a $60,000 
being equal, which of the  tolerable misstatement for an asset 
following would lead to a  with an account balance of 
larger sample size?  $1,000,000. The auditor selected a 
a. Greater reliance on internal sample of every twentieth item from 
control.  the population that represented the 
b. Greater   reliance   on asset account balance and discovered 
analytical procedures.  overstatements of $3,700 and 
c. Smaller expected frequency understatements of $200. Under these
of errors.  circumstances, the auditor most likely
d. Smaller   measure   of would conclude that 
tolerable misstatement.  a. There is an 
unacceptably high 
44. In estimation sampling for risk that the actual 
variables,   which   of   the misstatements in 
fol­lowing must be known the population 
in   order   to   estimate   the exceed the toler­
appropriate   sample   size able misstatement 
required   to   meet   the because the total 
auditor’s needs in a given
projected mis­
situation? 
statement is more 
a. The   qualitative   aspects   of
than the tolerable 
errors. 
misstatement. 
b. The   total   dollar   amount   of
218 distribution. 
MODULE 7 b. Overstated   units   have
a lower probability of
sam­ple selection than
b. There is an  units   that   are
unacceptably high  understated. 
risk that the toler­ c. The auditor controls the
able misstatement  risk of incorrect accep­
exceeds the sum of tance by specifying that
actual over­ risk level for the 
statements and  sampling plan. 
understatements.  d. The sampling interval is
c. The asset account  calculated by dividing 
is fairly stated  the number of physical 
because the total  units in the population 
projected  by the sample size. 
misstatement is  47. Hill has decided to use 
less than the  probability­proportional­to­size 
tolerable 
(PPS) sampling, sometimes called 
misstatement. 
dollar­unit sampling, in the audit 
d. The asset account is
of a client’s accounts receivable 
fairly stated because
balances. Hill plans to use the 
the   toler­able
following PPS sampling table: 
misstatement
exceeds   the   net   of TABLE
projected   ac­tual Reliability Factors for Overstatements
Number
overstatements   and of over­ Risk of incorrect acceptance
understatements.  statements 1% 5% 10% 15% 20%
0 4.61 3.00 2.31 1.90 1.61
46. Which of the  following statements  is 1 6.64 4.75 3.89 3.38 3.00
correct   concerning   probability­ 2 8.41 6.30 5.33 4.72 4.28
proportional­to­size   (PPS)   sampling, 3 10.05 7.76 6.69 6.02 5.52
4 11.61 9.16 8.00 7.27 6.73
also known as dollar unit sampling? 
a. The   sampling
distribution should
approximate   the
normal
AUDIT SAMPLING 51. The use of the ratio estimation
sampling   technique   is   most
effective when 
Variability in the dollar a.The   calculated   audit
amounts of inventory items amounts   are
a. Yes approximately
b. Yes proportional   to   the
c. No client’s   book
d. No
amounts. 
50. In statistical sampling  b. A   relatively   small
methods used in substantive  number   of
test­ing, an auditor most  differences   exist   in
likely would stratify a  the population. 
population into meaningful  c.Estimating   populations
groups if  whose   records
a.Probability­ consist of quantities,
proportional­to­ but not book values. 
size   (PPS) d. Large   overstatement
sampling is used.  differences   and   large
b. The under­statement
population differences   exist   in
has   highly the population. 
variable
recorded 52. In   the   application   of   statistical
amounts.  techniques   to   the   estima­tion   of
c.The   auditor’s dollar   amounts,   a   preliminary
estimated sample   is   usually   taken   primarily
tolerable for   the   purpose   of   estimating   the
misstatement   is population 
extremely small.  a. Variability. 
d. The standard deviation b. Mode. 
of recorded amounts is c. Range. 
rela­tively small.  d. Median. 
Additional information ple size, the auditor 
Tolerable misstatements
usually determines the 
(net of effect of expected misstatements)
Risk of incorrect acceptance
Number of misstatements
Recorded amount of accounts receivable
Number of accounts
What sample size should Hill use?
a. 120 
b. 108 
c. 60 
d. 30 
48. In a probability­proportional­to­size 
sample with a sam­pling interval of 
$5,000, an auditor discovered that a 
se­lected account receivable with a 
recorded amount of $10,000 had an 
audit amount of $8,000. If this were 
the only error discovered by the 
auditor, the projected error of this 
sample would be 
a. $1,000 
b. $2,000 
c. $4,000 
d. $5,000 
49. An auditor is determining 
the sample size for an in­
ventory observation using 
mean­per­unit estimation, 
which is a variables 
sampling plan. To 
calculate the required sam­
53. Using statistical sampling to assist and   corresponds
in verifying the year­end accounts  to the sum of all
payable balance, an auditor has  the   individual
accumulated the following data: book values. 
b. A   book   value   for   each
Number of Book sample item is unknown. 
accounts balance c. There   are   some
Population 4,100 $5,000,000 observed
Sample 200 $  250,000 differences
Using   the   ratio   estimation between   au­dited
technique, the auditor’s estimate of values   and   book
year­end   accounts   payable   balance values. 
would be d. The   audited
a. $6,150,000  values   are
b. $6,000,000  nearly
c. $5,125,000  proportional   to
d. $5,050,000  the book values.

54. Use of the ratio estimation 55. An auditor is performing 


sampling   technique   to substantive tests of pricing 
esti­mated dollar amounts and extensions of perpetual 
is inappropriate when  inventory balances consisting
a. The   total   book of a large number of items. 
value   is   known Past experience indicates 
numer­ 
MODULE 7   AUDIT SAMPLING 219

ous pricing 
and 
extension 
errors. 
Which of 
the 
following 
statistical 
sampling 
approaches
is most 
appropriate
?
a. Unstratified mean­per­unit. 
b. Probability­proportional­to­size. 
c. Stop or go. 
d. Ratio estimation. 
56. The 
maj
or 
reas
on 
that 
the 
diffe
renc

and 
ratio
esti
ma­
tion 
met
hods
wou
ld 
be 
expe
cted
to 
prod
uce 
audi

effic
ienc
y is 
that 
the 
a.
Num
b
e
r
 
o
f
 
m
e
m
b
e
r
s
 
o
f
 
t
h
e
 
p
o
p
u
l
a
t
i
o
n
s
 
o
f
 
d
i
f
f
e
r
­
e
n
c
e
s
 
o
r
 
r
a
t
i
o
s
 
i
s
 
s
m
a
l
l
e
r
 
t
h
a
n
 
t
h
e
 
n
u
m
b
e
r
 
o
f
 
m
e
m
­
b
e
r
s
 
o
f
 
t
h
e
 
p
o
p
u
l
a
t
i
o
n
 
o
f
 
b
o
o
k
 
v
a
l
u
e
s
.
 
b. Beta risk may be completely ignored. 
c.
Calc
u
l
a
t
i
o
n
s
 
r
e
q
u
i
r
e
d
 
i
n
 
u
s
i
n
g
 
d
i
f
f
e
r
e
n
c
e
 
o
r
 
r
a
t
i
o
 
e
s
t
i
m
a
t
i
o
n
 
a
r
e
 
l
e
s
s
 
a
r
d
u
o
u
s
 
a
n
d
 
f
e
w
e
r
 
t
h
a
n
 
t
h
o
s
e
 
r
e
q
u
i
r
e
d
 
w
h
e
n
 
u
s
i
n
g
 
d
i
r
e
c
t
 
e
s
t
i
m
a
t
i
o
n
.
 
d.
Vari
a
b
i
l
i
t
y
 
o
f
 
t
h
e
 
p
o
p
u
l
a
t
i
o
n
s
 
o
f
 
d
i
f
f
e
r
e
n
c
e
s
 
o
r
 
r
a
­
t
i
o
s
 
i
s
 
l
e
s
s
 
t
h
a
n
 
t
h
a
t
o
f
 
t
h
e
 
p
o
p
u
l
a
t
i
o
n
s
 
o
f
 
b
o
o
k
 
v
a
l
­
u
e
s
 
o
r
 
a
u
d
i
t
e
d
 
v
a
l
u
e
s
.
57. Whic
h   of
the
follo
wing
state
ment
s   is
corre
ct
conc
ernin
g   the
audit
or’s
use
of
statis
tical
samp
ling? 
a.
An 
a
u
d
i
t
o
r
 
n
e
e
d
s
 
t
o
 
e
s
t
i
m
a
t
e
 
t
h
e
 
d
o
l
l
a
r
 
a
m
o
u
n
t
 
o
f
 
t
h
e
 
s
t
a
n
d
a
r
d
 
d
e
v
i
a
t
i
o
n
 
o
f
 
t
h
e
 
p
o
p
u
l
a
t
i
o
n
 
t
o
 
u
s
e
 
c
l
a
s
s
i
c
a
l
 
v
a
r
i
a
b
l
e
s
 
s
a
m
p
l
i
n
g
.
 
b.
An 
a
s
s
u
m
p
t
i
o
n
 
o
f
 
P
P
S
 
s
a
m
p
l
i
n
g
 
i
s
 
t
h
a
t
 
t
h
e
 
u
n
d
e
r
­
l
y
i
n
g
 
a
c
c
o
u
n
t
i
n
g
 
p
o
p
u
l
a
t
i
o
n
 
i
s
 
n
o
r
m
a
l
l
y
 
d
i
s
t
r
i
b
­
u
t
e
d
.
 
c.

c
l
a
s
s
i
c
a
l
v
a
r
i
a
b
l
e
s
s
a
m
p
l
e
 
n
e
e
d
s
t
o
 
b
e
 
d
e
s
i
g
n
e
d
 
w
i
t
h
 
s
p
e
c
i
a
l
c
o
n
s
i
d
e
r
a
t
i
o
n
s
t
o
 
i
n
c
l
u
d
e
 
n
e
g
a
t
i
v
e
 
b
a
l
­
a
n
c
e
s
i
n
 
t
h
e
 
s
a
m
p
l
e
.
d.
The 
s
e
l
e
c
t
i
o
n
 
o
f
 
z
e
r
o
 
b
a
l
a
n
c
e
s
 
u
s
u
a
l
l
y
 
d
o
e
s
 
n
o
t
 
r
e
­
q
u
i
r
e
 
s
p
e
c
i
a
l
 
s
a
m
p
l
e
 
d
e
s
i
g
n
 
c
o
n
s
i
d
e
r
a
t
i
o
n
s
 
w
h
e
n
 
u
s
i
n
g
 
P
P
S
 
s
a
m
p
l
i
n
g
.
 
58. Whi
ch of
the 
follo
wing
most
likel

woul
d be 
an 
adva

tage 
in 
usin

class
ical 
varia
bles 
sam
pling
rathe

than 
prob
abilit

prop
ortio
nal­
to­
size 
(PPS

sam
pling

a.
An
e
s
t
i
m
a
t
e
 
o
f
 
t
h
e
 
s
t
a
n
d
a
r
d
 
d
e
v
i
a
t
i
o
n
 
o
f
 
t
h
e
 
p
o
p
u
­
l
a
t
i
o
n

s
 
r
e
c
o
r
d
e
d
 
a
m
o
u
n
t
s
 
i
s
 
n
o
t
 
r
e
q
u
i
r
e
d
.
 
b.
The
a
u
d
i
t
o
r
 
r
a
r
e
l
y
 
n
e
e
d
s
 
t
h
e
 
a
s
s
i
s
t
a
n
c
e
 
o
f
 
a
 
c
o
m
­
p
u
t
e
r
 
p
r
o
g
r
a
m
 
t
o
 
d
e
s
i
g
n
 
a
n
 
e
f
f
i
c
i
e
n
t
 
s
a
m
p
l
e
.
 
c.
Incl
u
s
i
o
n
 
o
f
 
z
e
r
o
 
a
n
d
 
n
e
g
a
t
i
v
e
 
b
a
l
a
n
c
e
s
 
g
e
n
e
r
a
l
l
y
 
d
o
e
s
 
n
o
t
 
r
e
q
u
i
r
e
 
s
p
e
c
i
a
l
 
d
e
s
i
g
n
 
c
o
n
s
i
d
e
r
a
t
i
o
n
s
.
 
d.
Any
a
m
o
u
n
t
t
h
a
t
i
s
 
i
n
d
i
v
i
d
u
a
l
l
y
 
s
i
g
n
i
f
i
c
a
n
t
i
s
 
a
u
t
o
­
m
a
t
i
c
a
l
l
y
 
i
d
e
n
t
i
f
i
e
d
 
a
n
d
 
s
e
l
e
c
t
e
d
.
220 MODULE 7   AUDIT SAMPLING

SIMULATIONS

Task­Based Simulation 1
Analyzing PPS
Authoritative
Results
Literature Help

The following is a computer printout generated by audit software using probability­
proportional­to­size (PPS) sampling:
Winz Corporation
Receivable Sampling Evaluation Results
December 31, 20X2
Population book value = $2,400,000;  Tolerable misstatement = $280,000
Projected Misstatement
Tainting Projected
Book value Audited value Misstatement percentage Sampling interval misstatement
$1,000 $ 0 $1,000 100% $80,000 $   1,000
750 600 150 20% $80,000 16,000
85,000 60,000 25,000 NA NA 25,000
$42,000
Basic Precision = 3.0 * $80,000 $240,000
Incremental Allowance
Reliability factor Increment (Increment –1) Projected misstatement Incremental allowance
3.00
4.75 1.75 .75 $16,000 $12,000
6.30 1.55 .55 1,000 __550
$12,550
The software uses factors from the following PPS sampling table:
TABLE
Reliability Factors for Overstatements
Number of Risk of incorrect acceptance
overstatements 1% 5% 10% 15% 20%
0 4.61 3.00 2.31 1.90 1.61
1 6.64 4.75 3.89 3.38 3.00
2 8.41 6.30 5.33 4.72 4.28
3 10.05 7.76 6.69 6.02 5.52
4 11.61 9.16 8.00 7.27 6.73

Answer the following questions relating to the above worksheet:
Answers
Questions
A. 30 items 
1. What was the planned sample size? 
B. 60 items 
2. What is the total misstatement in the sample?  C. 76 items 
D. 90 items 
3. What is the most likely total misstatement in the E. 0 
population?  F. $  12,550 
4. Calculate the upper limit on misstatement.  G. $  26,150 
H. $  42,000 
5. Calculate the allowance for sampling risk. 
I. $  54,550 
6. Would one “accept” or “reject” the population as being J. $  80,000 
materially correct?  K. $252,550 
7. What is the risk of incorrect acceptance?  L. $280,000 
M. $294,550 
N. $354,550 
O. Accept 
P. Reject 
Q. 5% 
R. 20% 
S. 100% 
MODULE 7   AUDIT SAMPLING 221

Task­Based Simulation 2
PPS Sampling
Concepts
Authoritative
Literature Help

Reply as to whether you believe the following statements are correct or 
incorrect concerning PPS sampling:

1. S

2. P

3. I

4. P

5. I
f
6. O

7. A

8. P

Tolerable misstatement
Sample size
Expected of misstatement
Recorded amount of accounts receivable
9. The sampling interval is $500. 
10. Increasing the expected misstatement to $10,000 will increase the sample size.

Task­Based Simulation 3
Research
Authoritative
Literature Help

Sampling vs. nonsampling risk
As a new assistant with Webber & Co. CPAs you have been asked to perform 
research on the nature of sampling risk versus nonsampling risk.
Selections
A. AU 
B. PCAOB 
C. AT 
D. AR 
E. ET 
F. BL 
G. CS 
H. QC 

(
A
)
(
B
)
(
C
)
(
D
)
(
E
)
(
F
)
(
G
)
(
H
)
1. Which title of the Professional Standards addresses this issue?

2. Enter   the   exact


section   and
paragraph(s)   which
distinguish   between
sampling   and
nonsampling risk. 
222 MODULE 7 AUDIT SAMPLING

MULTIPLE­CHOICE ANSWERS

1. d __ __ 13. d __ __ 25. b __ __ 37. a __ __ 49. a __ __


2. c __ __ 14. c __ __ 26. c __ __ 38. d __ __ 50. b __ __
3. a __ __ 15. c __ __ 27. a __ __ 39. c __ __ 51. a __ __
4. d __ __ 16. b __ __ 28. d __ __ 40. b __ __ 52. a __ __
5. a __ __ 17. b __ __ 29. b __ __ 41. b __ __ 53. b __ __
6. a __ __ 18. d __ __ 30. a __ __ 42. c __ __ 54. b __ __
7. d __ __ 19. c __ __ 31. a __ __ 43. d __ __ 55. d __ __
8. b __ __ 20. d __ __ 32. c __ __ 44. c __ __ 56. d __ __
9. c __ __ 21. a __ __ 33. b __ __ 45. a __ __ 57. a __ __
10. a __ __ 22. d __ __ 34. d __ __ 46. c __ __ 58. c __ __
11. d __ __ 23. a __ __ 35. a __ __ 47. d __ __ 1st: __/58 = __%
12. c __ __ 24. c __ __ 36. b __ __ 48. b __ __ 2nd: __/58 = __%

MULTIPLE­CHOICE ANSWER 
EXPLANATIONS
require the use of judgment, al­
A.2. General Approaches  though that judgment is quantified 
to Audit Sampling when statistical sampling is used. 
— Nonstatistical  Answer (c) is incorrect because 
and Statistical either statistical or nonstatistical 
sampling may provide equal 
1. (d) The requirement is to  assurance to the auditor. 
identify an advantage of 
statistical sampling over  2. (c) The requirement is to identify an 
nonstatistical sampling.  advantage of statistical sampling over 
An­swer (d) is correct  nonstatistical sampling. Answer (c) is 
because statistical  correct because statistical sampling 
sampling helps the auditor  helps the auditor to: 
to: (1) design an efficient  (1) design an efficient sample, (2) 
sample, (2) measures the  measure the sufficiency of the 
sufficiency of the  evidential matter obtained, and (3) 
evidential matter obtained, evaluate the sample results (AICPA 
and  Audit Sampling Guide). Answer (a) is 
(3) evaluate the sample results  incor­rect because the risk of 
(AICPA Audit SamplingGuide).  nonsampling errors is not directly 
Answer (a) is incorrect because dual­ affected by whether statistical or 
purpose tests,which both test a  nonstatistical sampling is used. 
control and serve as substantive test,  Answer (b) is incorrect because either
may be performed with either a  statistical or nonstatistical sampling 
statistical or a nonstatistical sam­ple.  can be used to reduce the level of 
Answer (b) is incorrect because both  audit risk to a low level; the 
statistical and nonstatistical sampling materiality level should not be 
affected by the type of sampling used. sampling applied to substantive 
Answer (d) is incor­rect because  testing and not to tests of controls 
either statistical or nonstatistical  and because the meaning of 
sampling may be used to minimize  “support the tolerable error” is 
the failure to detect errors and fraud.  uncertain.
A.3.  Uncertainty and Audit  4. (d) The requirement is to determine 
Sampling the nature of the risk of incorrect 
acceptance and the risk of assessing 
3. (a) The requirement is to determine  control risk too low. Answer (d) is 
the meaning ofthe likelihood of  correct because the risk of incor­rect 
assessing control risk too high in a test  acceptance and the risk of assessing 
of controls. Answer (a) is correct  control risk too low relate to the 
because the risk of assessing control  effectiveness of an audit in detecting 
risk too high is the risk that the sample  an existing material misstatement or 
does not support the auditor’s planned  deviation. Answer (a) is incorrect 
assessed level of control risk when the  because the term “allowable risk of 
true operating effectiveness of the  tolerable misstatement” is not used in 
control structure justifies such an  the professional standards. Answer 
assessment. Answer (b) is incorrect be­ (b) is in­correct because preliminary 
cause the risk of assessing control risk  estimates of materiality levels relate 
too high relates to the deviation rate  most directly to the risk of incorrect 
from a control procedure in a  acceptance, and only indirectly to the 
population, not to monetary 
risk of assessing control risk too low. 
Answer (c) is incorrect because the 
misstatements. Answer (c) is incorrect 
risk of incorrect rejec­tion and the 
because the risk of assessing control 
risk of assessing control risk too high 
risk too high does not directly relate to 
relate to the efficiency of the audit. 
monetary misstatements. Answer (d) is 
incorrect because tolerable error  5. (a) The requirement is to determine 
(misstatement) relates to variables which answer represents the concept 
of sampling risk. Sampling risk arises
from the possibility that an auditor’s 
conclusions based upon a sample 
would differ from the conclusions 
which would be drawn from 
examining the entire population (i.e., 
the risk that the sample examined is 
not representative of the population). 
Answers (b), (c), and (d) are all 
incorrect because they relate to errors 
which could occur even if 100% of 
the population were examined, that is,
nonsampling risk. 
6. (a) The requirement is to 
determine what is related to 
the risk of incorrect 
rejection and the risk of 
assessing control risk too 
high. Answer (a) is correct 
because AU 350 states that 
the risk of incorrect 
rejection and the risk of as­
sessing control risk too high
relate to the efficiency of 
the audit. These two errors 
generally result in an 
auditor per­forming 
unnecessary additional 
procedures. Answer (b) is 
incorrect because the risk of
incorrect acceptance and the
risk of assessing control 
risk too low relate to the 
effective­ness of an audit.  which an auditor has 
Answer (c) is incorrect  properly concluded that 
because the risks do not  control risk is at or below 
relate directly to the actual  the planned assessed level. 
selection of the sample.  Answer (d) is cor­rect 
Answer (d) is incorrect  because to support the 
because the audit quality  planned level, the deviation 
controls do not directly  rate must be less than the 
mention either of these  tolerable rate and the auditor
risks.  must conclude that the 
7. (d) The requirement is to  deviation rate is less than the
determine the situation in  tolerable 
MODULE 7   AUDIT SAMPLING 223
is not misstated. Answers 
(a) and (b) are incorrect 
rate. Answer (a) is incorrect because  because the risk of 
it represents a situation in which the  assessing control risk too 
auditor appropriately decides that the  high and the risk of 
deviation rate exceeds the tolerable  assessing control risk too 
rate. Answer (b) is incorrect be­cause  low relate to tests of 
it represents a situation in which an  controls, and not to 
auditor errone­ously concludes that  substantive tests of details. 
the deviation rate exceeds the  Answer (d) is incorrect 
tolerable rate when it actually does  because the risk of 
not. Answer (c) is incorrect be­cause  incorrect acceptance is the 
the auditor erroneously concludes that risk that the sample support
the deviation rate is less than the  the conclusion that the 
tolerable rate when it actually exceeds account is not misstated 
it. when in fact it is misstated.
8. (b) The requirement is to  A.4.  Types of Audit Tests in Which
determine the situation in 
Sampling May Be
which the auditor assesses 
control risk too high and  Used
thereby increases  10. (a) The requirement is to 
substantive testing. Answer  identify the correct state­ment with 
(b) is correct because to  respect to the size of a sample 
assess control risk too high,  required for dual purpose testing. 
an auditor must estimate  Answer (a) is correct because the 
that the deviation rate  auditor should select the larger of 
exceeds the tolerable rate  the required sample sizes.
when it actually is less than 
the tolerable rate. Answer  A.5.  Types of Statistical Sampling 
(a) is incorrect be­cause it  Plans
represents a situation in 
which the auditor appropri­ 11. (d) The requirement is to identify the 
ately decides that the  type of sam­pling involved in a 
deviation rate exceeds the  review of client invoices in which an 
tolerable rate. Answer (c) is  expected population deviation rate 
incorrect because the  and an established toler­able rate are 
auditor errone­ously  provided. Answer (d) is correct 
concludes that the deviation  because attri­bute sampling is used to 
rate is less than the toler­ reach a conclusion about a popula­
able rate when it actually  tion in terms of a rate of occurrence 
exceeds the tolerable rate.  (Audit Sampling Guide). Answer (a) 
An­swer (d) is incorrect  is incorrect because stratified 
because to properly rely on  sampling is gener­ally used to reach a 
internal control, the  dollar­based conclusion in variables 
deviation rate must be less  sampling approaches. Answer (b) is 
than the tolerable rate and  incorrect because, as indicated, 
the auditor must conclude  variables sampling deals with a dollar
that the deviation rate is less amount conclusion, not deviation 
than the tolerable rate.  rates. Answer (c) is incorrect be­cause
discovery sampling is only used in 
9. (c) The requirement is to  cases in which the auditor expects 
determine the type of risk  deviation rates to be extremely low 
demonstrated when an  (ap­proaching zero). 
auditor concludes that an 
account is misstated when  12. (c) The requirement is to 
in fact it is not. Answer (c)  identify the sampling 
is correct be­cause the risk  method that would be used 
of incorrect rejection is the  to estimate a numerical 
risk that the sam­ple  mea­surement such as a 
supports the conclusion  dollar value of a 
that the recorded account  population. Answer (c) 
bal­ance is materially 
misstated when the account
systematic. Ad­ditionally, 
is correct because sampling for  random sampling may be 
variables addresses such numerical  performed without re­
measurements. Answer (a) is  placement. 
incorrect because attributes sampling 
deals with deviation rates. Answer (b) 15. (c) The requirement is to 
is incorrect because stop­or­go  determine the least desir­
sampling (also referred to as  able statistical selection 
sequential sampling) is a form of  technique. Answer (c), 
attributes sampling. An­swer (d) is  block se­lection, is correct 
incorrect because random­number  because, ideally, a sample 
sampling is simply a sample selection should be se­lected from 
technique that may be used with  the entire set of data to 
either an attributes or a variables  which the resulting 
form of sampling; accord­ingly, a  conclusions are to be 
numerical measurement such as a  applied. When block 
dollar value is not necessary for  sampling is used, the 
random­number sampling. selection of blocks often 
precludes items from being 
13. (d) The requirement is to  so selected. In most cases, 
determine the audit testfor which an  systematic [answer (a)], 
auditor would most likely use  stratified [answer (b)], and 
attribute sam­pling. Answer (d) is  sequential [answer (d)] 
correct because attribute sampling is  selection techniques all 
used to reach a conclusion about a  provide a better 
population in terms of a rate of  representation of the entire 
occurrence. Here the rate of  population than does block 
occurrence will be the rate of  selection. 
(un)approved time cards. Answers 
(a), (b), and (c) are all incorrect  16. (b) The requirement is to identify the 
because they all relate more directly  type of sam­pling plan that does not use
to vari­ables sampling which is  a fixed sample size for tests of controls. 
generally used to reach conclusion  Answer (b) is correct because 
about a population in terms of a  sequential sampling results in the 
dollar amount. See the AICPA Audit  selection of a sample in several steps, 
and Accounting Guide, Audit  with each step conditional on the result 
Sampling and AU 350 for more  of the previous steps. Therefore, sample
information on audit sampling. size will vary depending upon the 
General Sampling Questions number of stages that prove necessary. 
Answers (a), (c), and (d) are all 
14. (c) The requirement is to  incorrect because dollar­unit sampling, 
determine the correct  PPS sampling, and variables sampling 
statement with respect to  all use a fixed sample size. 
random sampling. Answer 
(c) is correct because every  17. (b) The requirement is to 
item in the accounting  identify the correct state­
population should have an  ment concerning a statistical 
opportunity to be selected.  sampling application where 
Answer (a) is incorrect  the population consists of 
because with stratified  forms which are not 
random sampling, each  consecutively numbered. 
stratum need not be given  Answer (b) is correct because
equal representation.  systematic sam­pling is a 
Answer (b) is incorrect  procedure where a random 
because while sample units  start is obtained and then 
should be randomly  every nth item is selected. 
selected, there is no  For example, a sample of 
requirement that the  forty from a population of a 
accounting popula­tion be  thousand would require 
randomly ordered. Answer  select­ing every 25th item 
(d) is incorrect because  after obtaining a random start
random sampling, by its  between 
very nature, is not 
224 MODULE 7   AUDIT SAMPLING
subpopulations to reduce 
the effect of dispersion in 
items 1 through 25. Answer (a) is  the population. 
incorrect because selec­tion of a 
random sample is possible even  B.1. Tests of Controls—Sampling 
though the popula­tion is not  Risk
consecutively numbered. Answer (c) 
is incorrect because there is no  20. (d) The requirement is to 
special reason for using stratified  identify the circumstance 
sam­pling. Stratified sampling breaks that would cause an 
down the population into  auditor to assess control 
subpopulations and applies different  risk too low and decrease 
selection methods to each  substantive testing 
subpopulation. This selection method inappropriately. An­swer 
is used when the population consists  (d) is correct because when
of different types of items (e.g., large  the true deviation rate in 
balances and small balances).  the population exceeds that
Answer (d) is incorrect be­cause  in the sample, the auditor 
random number tables can be used  may assess control risk too
even though the forms are not  low. The AICPA Audit 
consecutively numbered. If random  SamplingGuide discusses 
numbers are selected for which there  tests of controls and 
are no forms, they are ignored. This  attributes sampling inits 
is the same as if there were 86,000  second chapter. Answers 
items in a consecu­tively numbered  (a) and (c) are incorrect be­
population and random numbers  cause the true deviation rate and the 
selected between 86,000 and 99,999  risk of assessing control risk too low 
are ignored. do not have such a relationship, either
positive or negative. Answer (b) is 
18. (d) Answer (d) is correct because 
incorrect because a deviation rate in 
systematic items occurrence in a  the population that is less than the 
population may destroy a sample’s  deviation rate in the auditor’s sample 
random­ness. Answer (a) is incorrect  may lead the auditor to assess control 
because items need not be replaced in risk too high. 
the population, and therefore is not a 
disadvan­tage of systematic sampling. 21. (a) The requirement is to determine 
Answer (b) is incorrect be­cause an  whether the expected deviation rate, 
individual item will not occur more  the tolerable deviation rate, or both 
than once in a sample when  affect the sample size for a test of 
systematic sampling is being used  controls. Answer (a) is correct 
(because the auditor selects every nth  because attribute sampling formulas 
item). Answer (c) is incorrect be­ and tables used in auditing generally 
cause systematic sampling refers to  require the auditor to specify an ex­
the type of sampling selection plan  pected deviation rate, a tolerable 
used and not the manner in which  deviation rate and the risk of 
items in the population are recorded.  assessing control risk too low. See 
Also, as indicated in (d) above, a  AICPA Audit Sam­pling Guide and 
systematic pattern in the population is AU 350 for more information on audit
a hindrance to sys­tematic sampling.  sam­pling. 
19. (c) Stratified sampling is a 
technique of breaking the 
population down into 
subpopulations and 
applying dif­ferent sample 
selection methods to the 
subpopulations. Stratified 
sampling is used to 
minimize the variance 
within the overall 
population [answer (c)]. 
Recall that as variance 
increases, so does the 
required sample size 
(because of the extreme 
values). Thus, stratification
allows the selection of 
25. (b) The requirement is to 
22. (d) The requirement is to  identify the correct state­
identify the correct state­ ment with respect to 
ment about sampling for  treatment of a voided 
attributes. Answer (d) is  voucher that has been 
correct because the sample  selected in a sample. 
size increases as the  Answer (b) is correct 
tolerable rate de­creases, an because the AICPA Audit 
inverse relationship.  Sampling Guide states that 
Answer (a) is incorrect  the auditor should obtain 
because many deviations do reasonable assurance that 
not necessarily result in a  the voucher has been prop­
mis­statement. Answer (b)  erly voided, and should then
is incorrect because a  replace it with another 
doubling of the population  voucher. Answer (a) is 
size will result in less than a incorrect because the 
doubling of the re­quired  voided voucher is not 
sample size. Answer (c) is  normally considered to be a 
incorrect because auditors  deviation. Answer (c) is 
must consider the  incor­rect because the 
qualitative aspects of  auditor must obtain 
deviations.  reasonable assurance that 
the misplaced voucher has 
23. (a) The requirement is to identify the  been voided. Answer (d) is 
information needed in addition to the  incorrect because the level 
likely rate of deviations and the  of materiality normally does
allowable risk of assessing control  not directly affect the 
risk too low to determine the sample  decision. 
size for a test of controls in an 
attributes sam­pling plan. Answer (a)  26. (c) The requirement is to 
is correct because the tolerable de­ determine the proper 
viation rate is also needed. Answer  method of handling a 
(b) is incorrect because the risk of  sample item which cannot 
incorrect acceptance relates to  be located for evaluation 
variables sampling applied to  purposes. Answer (c) is 
substantive testing, not attributes  correct because an auditor 
sampling ap­plied to tests of controls. would ordinarily consider 
Answer (c) is incorrect because the  the selected item to be a 
auditor will examine the nature and  deviation. Answers (a) and 
cause of deviations after the sample  (d) are incorrect since a 
has been selected. Answer (d) is  possible cause for the 
incorrect be­cause auditors often do  missing purchase order 
not consider population size when  could be a breakdown in 
performing attributes sampling  one of the controls of the 
applied to tests of controls.  system. Thus, in selecting a 
new sample item(s) the 
24. (c) The requirement is to  auditor may be ignoring a 
determine when a sample  portion of the population 
size would be decreased  which is in error and may 
when sampling for  be artificially skewing the 
attributes. An­swer (c) is  results of the tests 
correct because the sample performed on the sample. 
size will decrease when  Answer (b) is incorrect 
the risk of assessing  because there is no reason 
control risk too low is  to believe that the entire test
increased, the tolerable  is invalid and cannot be 
rate is increased, and the  relied upon. 
expected population 
deviation rate is decreased  27. (a) The requirement is to 
(Audit and Accounting  determine why deviations 
Guide Audit Sampling).  from control procedures do
Answers (a), (b), and (d)  not necessarily result in 
are all incorrectbecause  errors. Answer (a) is 
they include combinations  correct because it provides 
of changes that would not  an example of a situation 
necessarily decrease  in which a deviation from a
sample size.  control procedure ex­ists 
(lack of documentation of  authorized and proper. 
transaction approval), al­ Thus, such a 
though the entry was 
MODULE 7   AUDIT SAMPLING 225
the type of sam­pling which is most 
directly related to finding at least one 
deviation does not necessarily result  exception. Discovery sample sizes 
in an error in the finan­cial  and related discovery sampling tables 
statements. Answer (b) is incorrect  are constructed to measure the 
because a deviation from control  probability of at least one error 
procedure and an error may occur in  occurring in a sample if the error rate 
the same transaction. Answer (c) is  in the population exceeds the 
incorrect since the fact that all  tolerable rate. Answer (b) is incorrect 
deviations do not lead to errors will  because variables sampling need not 
result in a lower error rate. Answer  include at least one exception (mean 
(d) is incorrect because while it  per unit sampling, for example, needs 
represents a correct statement, it does no errors). Answer (c) is incorrect 
not follow from the point of the  since random sampling only deals 
question which is based on the idea  with the technique used to select 
that deviations do not directly result  items to be included in the sample. 
in errors. Answer (d) is incorrect be­cause 
28. (d) The requirement is to  dollar­unit sampling results are not 
determine the objective of  directly related to finding at least one 
the tolerable rate in  exception. 
sampling. Tolerable rate is  31. (a) The requirement is to determine 
calculated to determine the  whether an auditor should consider 
range of procedural  the likely rate of deviation, the al­
deviations in the popula­ lowable risk of assessing control risk 
tion. Answer (a) is incorrect too high, or both, when performing a 
because probabilities relate  test of a control. Answer (a) is correct
more directly to reliability.  because an auditor will consider the 
Answer (b) is incorrect  likely rate of deviations, but will not 
because errors on financial  ordinarily consider the allowable risk 
statements in materiality  of assess­ing control risk too high 
terms relate to variables  when following the approach out­
sampling. Answer (c) is  lined in the AICPA Audit Sampling 
incorrect because the  Guide. 
tolerable rate does not relate
directly to substantive tests.  32. (c) The requirement is to determine 
whether an auditor would consider the
29. (b) The requirement is to determine  average dollar value of check 
the correct rela­tionship between the  requests, the allowable risk of 
tolerable rate of deviations and the  assessing control risk too low, or 
expected rate of deviations for a test  both, when testing whether check 
of a control. The toler­able rate of  requests have been properly 
deviations is the maximum rate of  authorized. Answer (c) is correct 
deviations from a prescribed control  because a test of authorization such as
procedure that an auditor would be  this is an attributes test which requires
willing to accept and, unless the 
expected error rate is lower, reliance 
on internal control is not justified. 
Answer (a) is incorrect because if the 
tolerable rate of deviations is less 
than the expected rate, the auditor 
would not plan to rely on internal 
control and would therefore omit tests
of controls. Answer (c) is incorrect 
because testing of controls is inap­
propriate if the expected rate of errors
equals the tolerable rate of deviations 
(mathematically, the precision of zero
makes the sample size equal to 
population size). Answer (d) is 
incorrect because, as indicated above,
to perform tests of controls one must 
assume that the tolerable rate of 
devia­tions is more than the expected 
error rate. 
30. (a) The requirement is to determine 
assessed level of control risk 
the auditor to determine an  may increase. Here the 
allowable risk of assessing con­trol  deviation rate of 6% (3 
risk too low, but does not deal  deviations/50 documents) 
directly with dollar val­ues.  plus the allowance for 
Answers (a), (b) and (d) are all  sampling risk of 2% equals 
incorrect because they include  8% and exceeds the tolerable 
incorrect combinations of the  rate of 7%. Answer (a) is 
replies. See AU 350 and the AICPA incorrect because the 
Audit Sampling Guide for  tolerable rate plus the 
information on sampling. allowance for sampling risk 
B.2. Statistical (Attributes)  will always exceed the 
Sampling for Tests of  expected population deviation
Controls rate when tests of controls are
being performed. Answer (b) 
33. (b) The requirement is to identify the  is incorrect because when the 
correct state­ment concerning statistical sam­ple deviation rate plus 
sampling in tests of controls. Answer  the allowance for sampling 
(b) is correct because while deviations  risk ex­ceeds the tolerable 
from perti­nent control procedures  rate the sample results do not 
increase the risk of material mis­ support the planned assessed 
statements, any specific deviation need  level of control risk. Answer 
not necessarily result in a misstatement. (c) is incor­rect because the 
For example, a recorded disbursement  tolerable rate less the 
that does not show evidence of required allowance for sam­pling risk 
approval might nevertheless be a  should be compared with the 
transaction that is properly authorized  actual deviation rate. 
and recorded (AICPA Audit Sampling 
35. (a) The requirement is to determine 
Guide). Answer (a) is incorrect because
increases in population size result in 
the circum­stance in which an auditor
would decide to increase the level of 
small increases in sample size. Answer 
the preliminary assessment of control 
(c) is incorrect because a direct 
risk. Answer (a) is correct because the
relationship, not an inverse relationship,
assessment of control risk will 
exists be­tween the expected population
increase when the achieved upper 
deviation rate and the sample size—that
precision limit (here 8%) exceeds the 
is, increases in the expected population  tolerable rate (here 7%). Answer (b) 
deviation rate result in an increase in  is incorrect because the expected 
the required sample size. An­swer (d) is deviation rate was 2 1/2%, not 7%. 
incorrect because when determining the Also, if the expected deviation rate is 
tolerable rate, the auditor does not yet  higher than the percentage of error in 
have the required sample size.  the sample, the preliminary 
34. (d) The requirement is to  assessment does not need to be 
increased. Answer (c) is incorrect 
determine the proper 
because the achieved upper precision 
evaluation of a statistical 
limit will always exceed the 
sample for attributes when a 
percentage of errors in the sample. 
test of  Answer (d) is incorrect because, in 
50 documents results in 3  circumstances in which the auditor 
deviations, given a tolerable  decides to sample the population, the 
rate of 7%, an expected  expected deviation rate will always be
population deviation rate of  less than the tolerable rate. 
5%, and an allowance for 
sampling risk of 2%. Answer  36. (b)  The  requirement is  to
(d) is correct because when  determine   the   allowance
the deviation rate plus the  for   sampling   risk   of   the
allowance for sam­pling risk  presented   sample.   When
exceeds the tolerable rate, the consider­ 
226 MODULE 7   AUDIT SAMPLING
correct because while an 
increase in tolerable 
ing the allowance for sampling risk,  misstatement de­creases 
one may consider both the planned  sample size for a 
allowance for sampling risk or the  substantive test of details, 
adjusted al­lowance based on the  an in­crease in the 
sample results. Answer (b) is correct  assessed level of control 
because both the planned and  risk increases the sample 
adjusted allowance for sam­pling risk size for a substantive test 
are 4 1/2% (7% – 2.5% for planning  of details because a lower 
purposes, and 8% – 3.5% [7/200] as  level of detection risk is 
adjusted). required. 
37. (a) The requirement is to identify  40. (b) The requirement is to 
the correct state­ment concerning  determine the proper course 
statistical sampling for tests of  of action when an auditor is 
controls. Answer (a) is correct  planning a sample of cash 
because population size has little or  disbursements and he or she
no effect on sample size. Answer (b)  is aware of several unusu­
is incorrect because the population  ally large cash 
deviation rate has a significant effect  disbursements. Given the 
on sample size. Answer (c) is  description of the several 
incorrect because sample size  disbursements as “unusually
increases to a much lesser extent than large,” an auditor will 
doubling as the population size  generally test them. Answer 
doubles. Answer (d) is incorrect  (b) is therefore correct 
because for a given toler­able rate, a  because stratifying the 
smaller, and not a larger, sample size  population will allow the 
should be selected as the expected  auditor to ensure inclusion 
population deviation rate decreases. of the disbursements. The 
sampling procedure 
B.3.  Nonstatistical Sampling for 
(selecting less than all 
Tests of Controls items) will then be applied 
38. (d) The requirement is to identify  only to the smaller 
the proper state­ment concerning a  disbursements. Answer (a) 
random sample when nonstatistical  is incorrect because the 
at­tributes sampling is being used.  existence of the large 
Answer (d) is correct be­cause the  disbursements will have no 
deviation rate of the sample should be necessary relationship to the
compared to the tolerable deviation  tolerable rate of deviation 
rate regardless of whether statistical  when attributes sampling is 
or nonstatistical sampling is being  being used. Answer (c) is 
used. Answer (a) is in­correct because incorrect because while 
the risk of assessing control risk too  increasing the sample size 
low should be considered, although it  might be appropriate in 
may be done judgmentally. Answer  some variables sampling 
(b) is incorrect because nonsampling  applications (we are not told
error relates to “human” type errors  in this problem whether 
such as not identifying a deviation,  attributes or variables 
and not specifically to the use of  sampling is being followed),
nonstatistical sampling. An­swer (c)  the fact that the 
is incorrect because discovery  disbursements are described
sampling will not be used to evaluate  as “unusually large” leads 
the results. one to include them. 
Answer (d) is incorrect 
C.1.  Tests of Details—Sampling  because an auditor will not 
Risk draw numerous sam­ples to 
assure inclusion of the large 
39. (c) The requirement is to  disbursements. 
determine whether either 
or both of an increase in 
tolerable misstatement and
an in­crease in the 
assessed level of control 
risk increase sample size 
in a substantive test of 
details. Answer (c) is 
given risk of incorrect 
41. (b) The requirement is to  acceptance increases as the 
determine whether either  auditor’s assessment of 
or both of the expected  tolerable misstatement for 
amount of misstatement  the balance or class 
and the measure of  decreases. Answer (a) is 
tolerable misstatement  incorrect because a greater 
influence sample size for a  reliance on internal control 
substantive test of details.  will lead to a smaller 
Answer (b) is correct  sample size in a substantive 
because both the expected  test of details. Answer (b) is
amount of misstatement  incorrect because greater 
and the tolerable  reliance upon analytical 
misstatement affect sample procedures will result in a 
size (AICPA Audit  need for less reli­ance on 
SamplingGuide). Increases  substantive tests of details 
in the expected amount of  and therefore will result in a
misstatementsincrease  smaller sample. Answer (c) 
sample size, while  is incorrect because a 
increases in the tolerable  smaller expected frequency 
mis­statement decrease  of errors will generally 
sample size.  include properly 
functioning internal control 
42. (c) The requirement is to  and will therefore re­sult in 
determine the correct  a smaller sample for 
statement concerning the  substantive tests of details. 
auditor’s consideration of 
tolerable misstatement.  44. (c) The requirement is to 
Answer (c) is correct because identify the factor which 
the considera­tion of  must be known in order to 
tolerable misstatement is  estimate the appropriate 
related to preliminary  sample size when using 
judgments in a manner such  variables sampling. Answer 
that when the auditor’s  (c) is correct because the 
prelimi­nary judgments about auditor must set an 
tolerable misstatement levels  acceptable level of risk for 
for ac­counts or transaction  both variables sampling and 
types are combined for the  attribute sampling. Answer 
entire audit plan, the  (a) is incorrect because while 
preliminary judgments about  the auditor will consider the 
materiality levels for the  qualitative aspects of errors 
financial statements are not  when evaluating the sample, 
exceeded. Answer (a) is  they need not be considered 
incorrect because the  in determining an appropriate
auditor’s judgment of  sample size. Answer (b) is 
business risk related to a  incorrect because a primary 
client is not directly related to ob­jective of variables 
tolerable mis­statement.  sampling is to estimate the 
Answer (b) is incorrect  audited dollar amount of the 
because tolerable mis­ population. Also, in some 
statement may be adjusted for forms of variables sampling, 
qualitative factors. An­  knowledge of book values is 
swer (d) is incorrect because  not necessary (e.g., mean­
tolerable misstatement may be  per­unit). Answer (d) is 
changed during the audit process,  incorrect because a rate of 
especially as misstate­ments are  error in the population relates
identified and the auditor considers  to attribute sampling. 
the nature of the misstatements. 
45. (a) The requirement is to identify an 
43. (d) The requirement is to  auditor’s most likely response to a 
determine the factor that  circumstance in which there is a toler­
would lead to larger sample  able misstatement of $60,000, and the 
size in a substantive test of  auditor has discovered misstatement of 
de­tails. Answer (d) is  a net overstatement of $3,500 ($3,700 – 
correct because the sample  $200) when 1/20 of the account has 
size required to achieve the  been included in the sample. Because 
auditor’s objective at a  auditors must project the misstatements 
to the entire population, one would  expect a misstatement of 
MODULE 7   AUDIT SAMPLING 227
mis­statement already 
adjusted for expected 
approximately $70,000 (20 times the  misstatements (as is the 
misstatement of $3,500). Since this  situation in this problem), 
exceeds the tolerable misstatement,  one divides that total by the
there is little question that the risk of  number of expected 
material misstatement is too high and  misstatements column for 
that the misstatement in the  the appropri­ate risk of 
population exceeds the tolerable  incorrect acceptance (the 
misstatement, therefore answer (a) is  fact that some over­
correct. An­swer (b) is incorrect  statements are expected is 
because it seems that the sum of  not used). Here that 
actual overstatement and  computation is 
understatement is likely to exceed the  $24,000/1.61 = $14,906.83.
tolerable misstatement. Also, answer  Sample size is computed by
(b) makes little sense since there  dividing the recorded 
probably is no such thing as “an  amount by the sampling 
unacceptably high risk” that the  interval, here 
tolerable misstatement exceeds the  $240,000/14,906.83, for a 
sum of actual overstatements and  sample size of 16. An 
understatements; in such a cir­
alternate approach is to use 
cumstance the auditor simply accepts 
the reliability factor for the 
the population as be­ing materially 
expected number of 
correct. Answers (c) and (d) are 
overstatements. In that case
incorrect because the total projected 
misstatement must be calculated as  the computations become 
indicated above. See AU 350 and the  $24,000/3.00 = $8,000. 
Audit SamplingGuide for information  Sample size is 
on sampling. $240,000/$8,000 = 30. In 
either case answer (d) is the
C.2.  Probability­Proportional­to­ clos­est and therefore the 
Size (PPS) Sampling correct reply. 

46. (c) The requirement is to  48. (b) The requirement is to 


identify the correct state­ determine the projected 
ment regarding probability­ error (misstatement) of a 
proportional­to­size (PPS)  probability­proportional­
sam­pling. Answer (c) is  to­size (PPS) sample with
correct because when using  a sampling interval of 
PPS sam­pling, the auditor  $5,000, when an auditor 
controls the risk of incorrect has discovered that an 
acceptance by specifying a  account with a recorded 
risk level when planning the amount of $10,000 had 
sample. An­  an audit amount of 
swer (a) is incorrect because PPS  $8,000. An­swer (b) is 
sampling does not assume a normal  correct since, when an 
distribution. Answer (b) is incorrect  account’s recorded 
because the book value of the unit  amount exceeds the 
determines how probable it is that it  sampling interval, the 
will be included in the sample, not  projected error equals the
whether it is over or un­derstated.  actual misstatement, here 
Answer (d) is incorrect because the  $2,000 ($10,000 – 
sampling interval is calculated by  $8,000). 
dividing the book value of the 
population by the sample size.  C.3.  Classical Variables Sampling

47. (d) The requirement is to  49. (a) The requirement is to 


determine the sample size  determine whether anauditor uses the
that should be used in a  variability in dollar values, the risk of
probability­proportional­to­ incor­rect acceptance, or both, in a 
size (PPS) sample. One  mean­per­unit estimation vari­ables 
approach is to first calculate sampling plan. Answer (a) is correct 
a sampling interval and use  because both
it to determine an 
appropriate sample size. 
Using this approach, when 
provided with the tolerable 
swer (c) is incorrect because 
factors are included in a sampling  while the ratio estimation 
plan. Answers (b), (c) and (d) are all  sam­pling technique may be 
incorrect because they suggest that  used with quantities, it offers 
one or both factors are not considered. no particular advantage over 
See AU 350 and the AICPA Audit  other methods. Answer (b) is 
Sampling Guide for information on  incorrect because the 
sampling. absolute size of differences 
50. (b) The requirement is to  does not make the ratio 
identify the circumstance in  estimation method most 
which an auditor most  effective. 
likely would stratify a  52. (a) The requirement is to 
population into meaningful  determine the purpose of 
groups. Answer (b) is  taking a preliminary sample 
correct because stratified 
when one uses statistical tech­
sampling is used to 
niques. It is necessary to 
minimize the effect on 
obtain an estimate of a 
sample size of variation 
popula­tion’s standard 
within the overall 
population and results in the deviation (variability) when 
largest savings for  calculating the required 
populations with high  sample size and when using 
variability. Answer (a) is  sampling techniques. 
incorrect because while PPS Answers (b), (c), and (d) are 
sampling may in essence  incorrect because, in most 
stratify a population, the  sta­tistical techniques used by
items selected, other than  auditors, the mode (most 
those larger than the  frequent balance), the median
sampling interval, are not in (middle balance) and the 
“meaning­ful groups.”  range (differ­ence between 
Answer (c) is incorrect  the highest and lowest values)
because the tolerable  are not used. 
misstatement in and of itself
will not lead to  53. (b) The requirement is to 
stratification. Answer (d) is  determine the estimated 
incorrect because  audited accounts payable 
stratification is used pri­ balance using the ratio 
marily when there is a  estimation technique. 
relatively large standard  Answer (b) is correct 
deviation, not a relatively  because the ratio estima­
small standard deviation.  tion technique estimates 
the audited value by 
51. (a) The requirement is to  multiplying the audited 
determine when ratio esti­ value/book value of the 
mation sampling is most  sample times the popu­
effective. The ratio  lation book value. In this 
estimation sampling  case, ($300,000/$250,000) 
technique uses the ratio  × $5,000,000 = 
between the audited to book  $6,000,000. 
amounts as a measure of 
54. (b) The ratio estimation sampling 
standard deviation in its sam­
technique is based on comparing the 
ple size computation. Answer
ratio of the book value to the audited 
(a) is correct because when 
value of the sampled items. Answer 
audit differences are 
(b) is correct because this method 
approximately proportional 
cannot be used when there is no book 
to account size the standard 
value to make the comparison. The 
deviation of the ratio is small circumstances described in an­swers 
and this re­sults in a  (a) and (c) are necessary for ratio 
relatively small required  point and interval estimation. Answer 
sample size. Answer (b) is  (d) describes the circumstances in 
incorrect because a relatively which the use of ratio estimation will 
large number of differences  be efficient in terms of required 
between book and audited  sample size. 
values must exist to calculate
a reliable standard deviation  55. (d)   The   requirement   is   to
under the ratio method. An­ determine   the   most   appro­
priate sampling approach for and 
substantive   tests   of   pricing
228 MODULE 7   AUDIT SAMPLING
57. (a) The requirement is to 
identify the correct state­
extensions of perpetual inventory  ment concerning the 
balances consisting of a large  auditor’s use of statistical 
number of items for which past  sampling. Answer (a) is 
experience indicates numerous  correct because an estimate 
expected pricing and extension  of the variation of the 
errors. An­swer (d) is correct  population, the standard 
because ratio estimation is  deviation, is needed to use 
appropriate clas­sical variables 
when testing a population for which a sampling (AICPA Audit 
large number of errors of this nature  and Accounting Guide, 
is expected (Audit and Accounting  Audit Sampling). Answer 
Guide, Audit Sampling). Answer (a)  (b) is incorrect because PPS
is incorrect because the un­stratified  sampling does not make an 
mean­per­unit method will typically  assumption that the under­
provide a larger sample size than the  lying population is 
ratio estimation method to achieve the normally distributed. 
same level of sampling risk. Thus, the Answers (c) 
ratio estimation method would be  and   (d)   are   incorrect   because
more appropriate. Answer (b) is  classical variables sample se­lected
incorrect because probability­ accounts   and   therefore   need   not
proportional­to­size sam­pling is most include   special   con­siderations   to
efficient for testing populations with  those with a negative balance. 
relatively low expected error rates. 
Answer (c) is incorrect because “stop  C.4. Comparison of PPS 
or go” or “sequential” sampling is  Sampling to 
most frequently used in attribute  Classical Variables
sampling. Sampling
56. (d) Difference and ratio estimation  58. (c) The requirement is to identify 
methods are statistical sampling  an advantage inusing classical 
methods. They measure the difference variables sampling rather than 
between audit and book values or the  probability­proportional­to­size 
ratio of audit to book values. As these sampling. Answer (c) is correct be­
differences should not be great, the  cause the inclusion of zero and 
popu­lation of these differences will  negative balances generally does not 
have little variance. In sta­tistical  require special design considerations 
sampling, the less variation in a  when using classical sampling, while 
population, the smaller the required  it does when using probability­
sample to provide an estimate of the  proportional­to­size sampling 
population. In other words, difference (AICPA Audit SamplingGuide). 
and ratio estimation methods are  Answer (a) is incorrect because 
more efficient because the differences classical variablessampling does 
between audit and book values are  require an estimate of the standard 
expected to vary less than the ac­tual  deviation of the population’s 
items in the population. Answer (a) is recorded amounts. Answer (b) is 
incorrect because the number of  incor­rect because the computational 
members in the population for  process involved with classi­
differences or ratio methods would be
the same as the number of items in 
the population for a direct estimation 
method. In difference sampling, many
items would be zero because audit 
and book are the same, and in ratio 
sampling, many of the members 
would be one for the same reason. 
Answer (b) is incorrect because beta 
risk can never be ignored, as beta risk
is the risk of accepting an incorrect 
(unacceptable) population. Answer 
(c) is incorrect because the 
calculations required in difference 
and ratio sampling are similar to 
those used in direct estimation 
sampling. 
(d) is incorrect because probability­
cal variables sampling may make use of proportional­to­size sampling, not 
a computer program desirable. Answer  classical sampling, auto­matically 
stratifies individually significant items.
MODULE 7   AUDIT SAMPLING 229

SOLUTIONS TO SIMULATIONS

Task­Based Simulation 1
Analyzing PPS
Results
Authoritative
Literature Help

1. (A)   Thirty   items.   Calculated   by   dividing   the   population   book   value


($2,400,000)   by   the   sampling   interval   size   ($80,000).   Accordingly
$2,400,000/80,000 = 30 items. 
2. (G)   $26,150.   Calculated by summing the misstatements ($1,000 + $150 +
$25,000 = $26,150). 
3. (H)     $42,000.     Projected   misstatement   represents   the   most   likely   total
misstatement in the population ($42,000). 
4. (M) $294,550. The upper limit on misstatement is the sum of projected
misstatement, basic precision, and the incremental allowance ($42,000 +
$240,000 + $12,550 = $294,550). 
5. (K) $252,550. The allowance for sampling risk is the sum of basic
precision   and   the   incremental   allowance   ($240,000   +   $12,550   =
$252,550). 
6. (Reject)  Reject because the upper limit on misstatement ($294,550) exceeds
the tolerable misstatement ($280,000). 
7. (5%)  Because basic precision uses a 3.0 factor, the test is being performed at
5%. 

Task­Based Simulation 2
PPS Sampling
Authoritative
Concepts
Literature Help

Correct Incorrect
1. Size   of   a   PPS   sample   is   not   based   on   the   estimated   variation   of   audited
amounts. 
2. PPS sampling results in a stratified sample. 
3. Individually significant items are automatically identified. 
4. PPS   sampling   results   in   a   smaller   sample   size   when   numerous   small
misstatement are expected. 
5. If no misstatements are expected, PPS sampling will usually result in a smaller
sample size than classical variables sampling methods. 
6. One does not need a book value for individual items to evaluate a PPS sample.
7. A PPS sample eliminates the need to project results to the overall population. 
8. PPS sampling is “preferred” by the professional standards. 
9. The sampling interval is $500.*
10. Increasing the expected misstatement to $10,000 will increase the sample size.
*  It is the recorded amount divided by sample size: $300,000 / 100= $3,000 

Task­Based Simulation 3
Research
Authoritative
Literature Help
1. W

2. E
n
230 MODULE 8   AUDITING WITH TECHNOLOGY

AUDITING WITH TECHNOLOGY
replaced by online
MULTIPLE­CHOICE QUESTIONS (1­32)
real­time
1. An advantage of using systems  processing   in   all
flowcharts to document  but   legacy
information about internal control systems. 
instead of using internal control  c. It   is   more   likely   to
questionnaires is that systems  result   in   an   easy­to­
follow audit trail than
flowcharts 
is   online   transaction
a. Identify   internal processing. 
control   weaknesses d. It   is   used   only   in   nondatabase
more promi­nently. 
applications. 
b. Provide   a   visual   depiction   of
clients’ activities.  5. An auditor would be most likely to assess 
c. Indicate   whether control risk at the maximum level in an 
control   procedures electronic environment with auto­mated 
are   operating system­generated information when 
effectively.  a. Sales   orders   are
d. Reduce   the   need   to   observe initiated   using
clients’   employees   per­forming predetermined,
routine tasks.  automated
decision rules. 
2. A   flowchart   is   most
b. Payables are based on
frequently used by an auditor many transactions and
in connection with the  large   in   dollar
a. Preparation   of amount. 
generalized
c. Fixed   asset
computer   audit transactions   are
pro­grams.  few in number, but
b. Review   of   the   client’s   internal large   in   dollar
control.  amount. 
c. Use   of   statistical   sampling   in d. Accounts
performing an audit.  receivable   records
d. Performance   of are based on many
analytical transactions   and
procedures   of are   large   in   dollar
account balances.  amount. 
3. Matthews Corp. has changed from a  6. In   a   highly   automated
system of re­cording time worked on  information   processing
clock cards to a computerized pay­roll  system tests of control 
system in which employees record time in a. Must   be   performed   in   all
and out with magnetic cards. The  circumstances. 
computer system automatically updates all b. May   be   required   in   some
payroll records. Because of this change  circumstances. 
a. A   generalized c. Are never required. 
computer   audit
d. Are required in first year audits. 
program   must   be
used. 
b. Part of the audit trail is altered. 
c. The   potential   for
payroll­related
fraud   is   dimin­
ished. 
d. Transactions   must   be   processed
in batches. 
4. Which   of   the   following   is
correct   concerning   batch   pro­
cessing of transactions? 
a. Transactions   are
processed in the order
they   occur,   regardless
of type. 
b. It has largely been
7. Which of the following is least  a. May   enable
likely to be considered by an  unauthorized
auditor considering  changes to data files
engagement of an information  if  not  properly
technology (IT) specialist on an controlled.
audit?  b. Are   very   versatile
a. Complexity   of   client’s   systems programs   that   can
and IT controls.  be   used   on
b. Requirements   to   assess   going hardware   of   many
concern status.  manufacturers. 
c. Client’s   use   of   emerging c. May   be   significant
technologies.  components   of   a
d. Extent   of   entity’s client’s   appli­cation
participation   in programs. 
electronic   com­ d. Are   written
merce.  specifically   to
enable   auditors   to
8. Which of the following  ex­tract   and   sort
strategies would a CPA most  data. 
likely consider in auditing an 
entity that processes most of its  11. An auditor would most likely 
financial data only in electronic  be concerned with which of the 
form, such as a paperless  following controls in a 
system?  distributed data processing 
a. Continuous   monitoring system? 
and   analysis   of a. Hardware controls. 
transaction   processing b. Systems   documentation
with an embedded audit controls. 
module.  c. Access controls. 
b. Increased   reliance   on   internal d. Disaster recovery controls. 
control activities that emphasize
12. Which of the following types 
the segregation of duties. 
c. Verification   of of evidence would an auditor 
most likely examine to 
encrypted   digital
determine whether internal 
certificates   used   to
control is operating as 
monitor   the designed? 
authorization   of
a. Gross   margin   information
transactions.  regarding the client’s in­dustry. 
d. Extensive   testing   of b. Confirmations   of
firewall   boundaries receivables
that   re­strict   the verifying   account
recording   of   outside balances. 
network traffic.  c. Client   records
documenting the use
9. Which of the following is not a
of   computer
major   reason   for   main­taining programs. 
an   audit   trail   for   a   computer d. Anticipated   results
system?  documented in budgets
a. Deterrent to fraud.  or fore­casts. 
b. Monitoring purposes. 
c. Analytical procedures.  13. An auditor anticipates 
d. Query answering.  assessing control risk at a low 
level in a computerized 
10. Computer systems are typically environment. Under these 
supported by a variety of utility circum­stances, on which of 
software   packages   that   are the following activities would 
important to an auditor because the auditor initially focus? 
they 
MODULE 8   AUDITING WITH TECHNOLOGY 231
output. 
b. A system that affects
a. Programmed control activities.  a number of essential
b. Application control activities.  master   files   and
c. Output control activities.  produces   a   limited
d. General control activities.  output. 
c. A system that 
14. After the preliminary phase of  updates a few 
the review of a client’s  essential master files 
computer controls, an auditor  and produces no 
may decide not to perform  printed output other 
tests of controls related to the  than final balances. 
controls within the computer  d. A system that performs
portion of the client’s internal  relatively   complicated
control. Which of the  pro­cessing   and
following would not be a valid produces   very   little
reason for choosing to omit  detailed output. 
such tests? 
a. The   controls 17. An auditor who wishes to 
duplicate   operative capture an entity’s data as 
controls   existing transactions are processed and 
elsewhere   in   the continuously test the entity’s 
structure.  computerized information 
b. There   appear   to   be system most likely would use 
major   weaknesses which of the following 
that  would  preclude techniques? 
reliance   on   the a. Snapshot application. 
stated procedure.  b. Embedded audit module. 
c. The   time   and   dollar   costs   of c. Integrated data check. 
testing   exceed   the   time   and d. Test data generator. 
dollar   savings   in   substantive
testing   if   the   tests   of   controls 18. Which of the following 
show   the   controls   to   be computer­assisted auditing 
operative.  techniques processes client 
d. The controls appear adequate.  input data on a controlled pro­
gram under the auditor’s 
15. Auditing by testing the input and control to test controls in the 
output   of   a   computer   system com­puter system? 
instead of the computer program a. Test data. 
itself will  b. Review of program logic. 
a. Not detect program errors which c. Integrated test facility. 
do  not  show   up   in   the   output d. Parallel simulation. 
sampled. 
19. To   obtain   evidence   that
b. Detect   all   program
errors,   regardless   of online access controls are
the   nature   of   the properly   functioning,   an
output.  auditor most likely would 
c. Provide   the   auditor a. Create checkpoints at
with the same type of periodic   intervals
evidence   as   tests   of after   live   data
application controls.  processing to test for
unauthorized   use   of
d. Not   provide   the
the system. 
auditor   with
confidence in the re­
sults   of   the   auditing
procedures. 
16. Which of the following client 
information technology (IT) 
systems generally can be 
audited without examining or 
directly testing the IT computer
programs of the system? 
a. A   system   that
performs   relatively
uncomplicated
processes   and
produces   detailed
units of measure. 
b. Examine the  d. Illogical   entries   in
transaction log to  fields whose logic is
discover whether any  tested   by
transactions were lost  programmed
or entered twice due to consistency checks. 
a system malfunction. 
c. Enter   invalid 23. Which of the following 
identification   numbers computer­assisted auditing 
or   passwords   to techniques allows fictitious 
ascertain   whether   the and real transactions to be pro­
cessed together without client 
system rejects them. 
operating personnel being 
d. Vouch   a   random aware of the testing process? 
sample   of   processed
a. Integrated test facility. 
transactions   to   assure
proper authorization.  b. Input controls matrix. 
c. Parallel simulation. 
20. An   auditor   most   likely   would d. Data entry monitor. 
introduce   test   data   into   a
computerized   payroll   system   to 24. Which   of   the   following
test controls related to the  methods of testing application
controls utilizes a generalized
a. Existence   of
audit   software   package   pre­
unclaimed   payroll
pared by the auditors? 
checks   held   by   su­
pervisors.  a. Parallel simulation. 
b. Early cashing of payroll checks b. Integrated   testing   facility
by employees.  approach. 
c. Test data approach. 
c. Discovery   of   invalid   employee
I.D. numbers.  d. Exception report tests. 
d. Proper approval of overtime by 25. In creating lead schedules for an
supervisors.  audit engagement, a CPA often 
21. When an auditor tests a computerized  uses automated workpaper 
software. What client 
accounting sys­tem, which of the following 
information is needed to begin 
is true of the test data approach? 
this process? 
a. Several transactions of each type a. Interim financial 
must be tested.  information such as 
b. Test   data   are third quarter sales, net
processed   by   the income, and 
client’s   computer inventory and 
programs under the receivables balances. 
auditor’s control. 
b. Specialized journal 
c. Test data must consist information such as the
of   all   possible   valid invoice and purchase 
and   in­valid order numbers of the 
conditions.  last few sales and 
d. The   program   tested purchases of the year. 
is different from the c. General   ledger
program   used information   such   as
throughout   the   year account   num­bers,
by the client.  prior   year   account
22. Which of the following is not  balances,   and   current
among the errors that an  year   unadjusted
auditor might include in the  information. 
test data when auditing a cli­ d. Adjusting   entry
ent’s computer system?  information   such   as
a. Numeric   characters   in deferrals   and
alphanumeric fields.  accruals,   and
b. Authorized code.  reclassification
journal entries. 
c. Differences   in   description   of
232 MODULE 8   AUDITING WITH TECHNOLOGY
a. Compiler programs. 
b. Supervisory programs. 
26. Using microcomputers in auditing  c. Utility programs. 
may affect the methods used to  d. User programs. 
review the work of staff assistants 
because  30. Smith Corporation has 
a. The   audit   fieldwork numerous customers. A cus­
standards for supervision tomer file is kept on disk 
may differ.  storage. Each customer file 
con­tains name, address, 
b. Documenting   the
credit limit, and account 
supervisory review may
balance. The auditor wishes 
require   assistance   of
consulting   services to test this file to determine 
personnel.  whether credit limits are 
being exceeded. The best 
c. Supervisory personnel 
procedure for the audi­tor to 
may not have an 
follow would be to 
under­standing of the 
a. Develop test data that would 
capabilities and  cause some account balances 
limitations of mi­ to exceed the credit limit and 
crocomputers.  determine if the system 
d. Working   paper properly detects such 
documentation   may situations. 
not   contain   readily b. Develop   a
observable   details   of program   to
calculations.  compare   credit
27. An   auditor   would  least  likely   use limits   with
computer software to  account   balances
a. Access client data files.  and   print   out   the
details   of   any
b. Prepare spreadsheets. 
account   with   a
c. Assess computer control risk.  balance   exceeding
d. Construct parallel simulations.  its credit limit. 
28. A   primary   advantage   of   using c. Request   a   printout
generalized   audit   soft­ware of   all   account
packages to audit the financial balances   so   they
statements of a client that uses a can   be   manually
computer   system   is   that   the checked against the
auditor may  credit limits. 
a. Access information  d. Request a printout 
stored on computer files  of a sample of 
while having a limited  account balances so 
understanding of the  they can be 
client’s hardware and  individually checked
software features.  against the credit 
b. Consider   increasing  the limits. 
use   of  substantive   tests 31. An   auditor   most   likely
of transactions  in place would test for the presence
of   analytical of   unauthorized   computer
procedures.  program   changes   by
c. Substantiate   the running a 
accuracy   of   data a. Program with test data. 
through   self­ b. Check   digit   verification
checking   digits   and program. 
hash totals. 
c. Source   code   comparison
d. Reduce   the   level   of
program. 
required   tests   of
controls to a relatively d. Program   that   computes
small amount.  control totals. 
32. An   entity   has   the   following
29. Auditors often make use of 
computer programs that perform  invoices in a batch: 
routine processing functions such
as sorting and merging. These 
programs are made available by 
electronic data processing 
companies and others and are 
specifically referred to as 
Invoice # Product Quantity Unit price
201 F10 150 $  5.00
202 G15 200 $10.00
203 H20 250 $25.00
204 K35 300 $30.00

Which of the following numbers represents 
the record count?
a. 1 
b. 4 
c. 810 
d. 900 
MODULE 8   AUDITING WITH TECHNOLOGY 233

SIMULATION

Task­Based Simulation 1
Computer Audit
Techniques and Terms Authoritative
Literature Help

Computer processing has become the primary means used to process financial 
accounting information in most businesses. Consistent with this situation, CPAs must 
have knowledge of audit techniques using computers and of computer terminology.

Select the type of audit technique being described in items 1 through 5.  Computer 
audit techniques may be used once,
more than once, or not at all.
Computer audit technique
A. Auditing “around” the computer E. Processing output control
B. I/O audit approach F. Test data
C. Integrated test facility G. Write extract routine
D. Parallel simulation

D
e
1. A

2. D
u
3. F
i
4. M
a
5. U
For items 6 through 10 select the type of computer control that is described in the 
definition that is presented. Each con­trol may be used once, more than once, or not at all.
Control
H. Backup and recovery M.  Hash total
I. Boundary protection N. Missing data check
J. Check digit O. Personal identification codes
K. Control digit P. Visitor entry logs
L. File protection ring

Description (H) (I)  (J) (K) (L) (M) (N) (O) (P)
6. A control that will detect blanks existing in input data when they should not.
7. A control to ensure that jobs run simultaneously in a multiprogramming
environment cannot change the allocated memory of another job.
8. A digit added to an identification number to detect certain types of data
transmission or transposition errors.
9. A terminal control to limit access to programs or files to authorized users.
10.  A total of one field for all the records of a batch where the total is
meaningless for financial purposes.
234 MODULE 8   AUDITING WITH TECHNOLOGY

MULTIPLE­CHOICE ANSWERS

1. b __ __ 7. b __ __ 13. d __ __ 19. c __ __ 25. c __ __ 31. c __ __


2. b __ __ 8. a __ __ 14. d __ __ 20. c __ __ 26. d __ __ 32. b __ __
3. b __ __ 9. c __ __ 15. a __ __ 21. b __ __ 27. c __ __
4. c __ __ 10. a __ __ 16. a __ __ 22. a __ __ 28. a __ __
5. c __ __ 11. c __ __ 17. b __ __ 23. a __ __ 29. c __ __ 1st: __/32 = __%
6. b __ __ 12. c __ __ 18. d __ __ 24. a __ __ 30. b __ __ 2nd: __/32 = __%

MULTIPLE­CHOICE ANSWER EXPLANATIONS
because while a flowchart 
A. Auditor’s   Consideration   of describes a system, the 
Internal   Control   When   a flowchart alone does not 
Computer Is Present  indicate whether that system is 
operating effectively. Answer 
1. (b) The requirement is to  (d) is incorrect because auditors
identify an advantage of using  still need to determine whether 
systems flowcharts to document the system has been placed in 
information about internal  operation and therefore the need
control instead of using internal  to observe employees 
control question­naires. Answer performing routine tasks re­
(b) is correct because  mains. 
flowcharts provide a visual 
depiction of clients’ activities  2. (b) The requirement is to 
which make it possible for  determine when a flow­chart is 
auditors to quickly understand  most frequently used by an 
the design of the system.  auditor. Answer (b) is correct 
Answer (a) is incorrect because  because flowcharts are 
while the flow of operations is  suggested as being appropri­ate 
visually depicted, internal  for documenting the auditor’s 
control weaknesses are not as  consideration of internal control.
obvious. Answer (c) is incorrect Answer (a) is incorrect because 
auditors do not frequently write  rect because batch processing may be 
their own generalized computer  used for database ap­plications.
audit pro­grams, the most likely 
time a flowchart would be used  5. (c) The requirement is to 
with respect to such software.  determine when an auditor 
Answers (c) and (d) are  would be most likely to assess 
incorrect because statistical  control risk at the maximum 
sampling and analytical  level in an electronic 
procedures do not in general  environment with automated 
require the use of flowcharts.  system­generated information. 
Answer (c) is correct because 
3. (b) The requirement is to  the few transactions involved in 
identify the correct state­ment  fixed assets make it most likely 
with respect to a computerized,  to be one in which a substantive 
automatically updating payroll  approach of restricting de­
system in which magnetic cards  tection risk is most likely to be 
are used instead of a manual  effective and efficient. An­swer 
payroll system with clock cards.  (a) is incorrect because an 
Answer (b) is cor­rect because  auditor might be expected to 
the automatic updating of  perform tests of controls to 
payroll records alters the audit  assess control risk below the 
trail which, in the past, included  maximum when automated 
steps pertaining to manual  decision rules are involved for 
updating. Answer (a) is  an account (sales) which 
incorrect because although an  ordinarily has many 
auditor may choose to use a  transactions. Answers (b) and 
generalized computer audit  (d) are incorrect because the 
program, it is not required.  numerous transactions in 
Answer (c) is incorrect because  payables and receivables make it
no information is presented that  likely that control risk will be 
would necessarily indicate a  assessed below the maximum. 
change in the likelihood of 
fraud. Answer (d) is incorrect  6. (b) The requirement is to 
because given automatic  identify the most accurate 
updating, a large portion of the  statement with respect to tests of
transactions are not processed in controls of a highly auto­mated 
batches.  information processing system. 
Answer (b) is correct because 
4. (c) The requirement is to identify the  SAS 110 states that in some 
correct state­ment concerning the batch  such circumstances substantive 
processing of transactions. Batch  tests alone will not be sufficient 
processing involves processing  to restrict detection risk to an 
transactions through the system in groups  acceptable level. Answer (a) is 
of like transactions (batches). Answer (c)  incorrect because tests of 
is correct because the similar nature of  controls need not be performed 
transactions involved with batch  in all such circumstances. 
processing ordinarily makes it relatively  Answer (c) is incorrect because 
easy to follow the transactions throughout  such tests are sometimes 
the system. Answer (a) is incorrect  required. Answer (d) is incorrect
because transactions are processed by  because tests of controls are not 
type, not in the order they occur  in such circumstances in all first 
regardless of type. Answer (b) is incor­ year audits. 
rect because many batch applications still 
exist and might be expected to exist well  7. (b) The requirement is to 
into the future. Answer (d) is incor­  identify the least likely 
circumstance in which an 
auditor would consider engage­
ments of an IT specialist on an 
audit. Answer (b) is correct 
because the requirement to 
assess going concern status re­
mains the same on all audits, 
and thus does not directly af­
fect engagement of an IT 
specialist. Answers (a), (c), and
(d) are all incorrect because complexity, 
the use of emerging technologies, and 
participation in electronic commerce are 
all factors which SAS 110 suggests make 
it more likely that an IT specialist will be  exists as to how many embedded audit 
engaged.  modules CPAs have actually used in prac­
tice. Answer (b) is incorrect because there 
8. (a) The requirement is to identify a strategy 
may well be a decrease in reliance on 
that a CPA most likely would consider in  internal control activities that empha­size 
auditing an entity that processes most of its  this segregation of duties since so many 
financial data only in electronic form.  controls are in the hardware and software of 
Answer (a) is correct because continuous  the application. Answer (c) is incorrect 
monitoring and analysis of transaction  because digital certificates deal with 
processing with an embedded audit module  electronic commerce between companies, a 
might provide an effective way of auditing  topic not directly ad­dressed by this 
these processes—although some question  question, and because such certificates pro­ 
MODULE 8   AUDITING WITH TECHNOLOGY 235
while requiring concern, are nor­mally 
considered less critical than proper access 
vide limited evidence on authorization.  controls for this situation. 
Answer (d) is incor­rect because while 
firewalls do control network traffic, this  12. (c) The requirement is to 
is not the most significant factor in the  identify the type of evi­dence an
audit of electronic form financial data. auditor would examine to 
determine whether inter­nal 
9. (c) The requirement is to identify the reply control is operating as designed.
that is not a major reason for maintaining  Answer (c) is correct because 
an audit trail for a com­puter system.  the inspection of documents and
Answer (c) is correct because analytical  records such as those related to 
pro­cedures use the outputs of the system,  computer programs represents 
and therefore the audit trail is of limited  an approach for obtaining an 
importance. Answer (a) is incorrect be­ understanding of internal 
cause an audit trail may deter fraud since  control. An­swer (a) is incorrect
because examining gross 
the perpetrator may realize that his or her 
margin infor­mation is more 
act may be detected. Answer (b) is 
likely to be performed during 
incorrect because an audit trail will help  the perfor­mance of analytical 
management to monitor the computer  procedures. Answer (b) is 
system. Answer (d) is incorrect be­cause  incorrect because confirming of 
an audit trail will make it much easier to  receivables is a substantive test. 
answer queries.  An­swer (d) is incorrect because
anticipated results documented 
10. (a) The requirement is to  in budgets or forecasts are much
identify a reason that util­ity  more frequently used in the 
software packages are  performance of analytical 
important to an auditor. An­ procedures. 
swer (a) is correct because  13. (d) The requirement is to 
client use of such packages re­ determine the procedures on 
quires that the auditor include  which the auditor would 
tests to determine that no  initially focus when anticipating
unplanned interventions using  assessing control risk at a low 
utility routines have taken place level. Answer (d) is correct 
during processing (Audit and  because auditors usually begin 
Accounting Guide,  by considering general con­trol 
Computer Assisted Audit Techniques).  procedures. Since the 
Answer (b) isincorrect because a client’s  effectiveness of specific 
use of such programs implies that they  applica­ 
are useful on his/her computer hardware, 
and therefore any flexibility is not of 
immediate relevance to the auditor. 
Answer (c) is incorrect because the 
primary purpose of utility programs is to 
support the computer user’s applications 
(Computer Assisted Audit Techniques). 
Answer (d) is incorrect because utility 
software programs have a variety of uses 
in addition to enabling auditors to extract
and sort data (Computer Assisted Audit 
Techniques). 
11. (c) The requirement is to identify the 
types of con­trols with which an auditor 
would be most likely to be con­cerned in a
distributed data processing system. A 
distributed data processing system is one 
in which there is a network of remote 
computer sites, each having a computer 
connected to the main computer system, 
thus allowing access to the com­puters by 
various levels of users. Accordingly, 
answer (c) is correct because numerous 
individuals may access the sys­tem, 
thereby making such controls of extreme 
importance. Answers (a), (b), and (d), 
be different than the evidence 
tion controls is often dependent on the  obtained by testing the program 
existence of effective general controls  itself. Answer (d) is incorrect 
over all computer activities, this is usually be­cause such auditing of inputs
an efficient approach. Answers (a), (b),  and outputs may well satisfy the
and (c) are all incor­rect because they  auditor. 
represent controls that are usually tested 
subsequent to the general controls. 16. (a) The requirement is to identify the type
of com­puter system that can be audited 
14. (d) The requirement is to  without examining or di­rectly testing 
determine an inappropri­ate  computer programs (i.e., auditing around 
reason for omitting tests of  the system). Auditing around the system 
controls related to  is possible if the sys­tem performs 
computercontrol procedures.  uncomplicated processes and produces 
Answer (d) is correct because  de­tailed output (i.e., is a fancy 
the fact that the controls appear bookkeeping machine). An­swers (b), (c), 
adequate is not sufficient  and (d) all describe more complicated 
justifica­tion for reliance; tests  computer systems that produce only 
of controls must be performed  limited output. In these more complicated 
before the auditor can actually  systems, the data and related controls are 
rely upon a control procedure to within the system, and thus the auditor 
reduce control risk. Answer (a)  must examine the system itself. Auditors 
is incorrect because when  must identify and evaluate the ac­
controls duplicate other controls counting controls in all computer systems.
the auditor who wishes to rely  Further, complex computer systems 
upon internal control need not  require auditor specialized expertise to 
test both sets. An­  perform the necessary procedures. 
swer (b) is incorrect because if weak  17. (b) The requirement is to 
controls are not to be relied upon, the  determine an audit tech­nique to 
auditor need not test their effectiveness.  determine whether an entity’s 
Answer (c) is incorrect because tests of  transactions are pro­cessed and to 
controls may be omitted if their cost  continuously test the 
exceeds the savings from reduced sub­ computerized informa­tion 
stantive testing resulting from reliance  system. Answer (b) is correct 
upon the controls.  because an embedded audit 
module is inserted within the 
15. (a) The requirement is to 
client’s information sys­tem to 
determine the correct statement 
with respect to testing inputs  continuously test the processing of
and outputs of a computer  transactions. An­swer (a) is 
system instead of testing the  incorrect because a snapshot 
actual computer pro­gram itself. application analyzes the 
Answer (a) is correct because  information system at one point in
portions of the program which  time. Answer (c) is incorrect 
have errors not reflected on the  because an integrated data check 
output will be missed. Thus, if a simply tests data at one point in 
“loop” in a program is not used  time. Answer (d) is incorrect 
in one application, it is not  because a test data generator 
tested. Answer (b) is incorrect  provides a sample of possible 
because the lack of an  circumstances in which data might
understanding of the entire  be improperly processed. 
program precludes the detection
18. (d) The requirement is to identify
of all errors. Answer (c) is 
incorrect because while auditing the computer­assisted audit 
inputs and outputs can provide  technique that processes client 
valuable evi­dence, it will often  input data on a 
236 MODULE 8   AUDITING WITH TECHNOLOGY
must then determine whether supervisors 
are following those criteria; it is less 
controlled program under the auditor’s  likely that this will all be included in a 
control to test con­trols in a computer  computer program than a test for invalid 
system Answer (d) is correct because  em­ployee ID numbers. 
parallel simulation processes actual client 
data through an auditor’s generalized  21. (b) The requirement is to 
audit software program. Answer (a) is  identify the correct state­ment 
incorrect because test data is a set of  regarding the test data approach.
dummy transactions developed by the  Answer (b) is correct because 
auditor and processed by the client’s pro­ the test data approach consists 
grams to determine whether the controls  of processing a set of dummy 
which the auditor intends to test are  transactions on the client’s 
operating effectively. Answer (b) is incor­ computer system. The test data 
rect because a review of program logic is  approach is used to test the 
an approach in which an auditor reviews  operating effectiveness of 
the steps by which the client’s program  controls the auditor intends to 
processes data. Answer (c) is incorrect  rely upon to assess control risk 
because an integrated test facility  at a level lower than the 
introduces dummy transactions into a  maximum. Answer (a) is in­
client’s system in the midst of live  correct because only one 
transactions. transaction of each type is 
gener­ally tested. Answer (c) is 
B. Computerized Audit Tools
incorrect because it is not 
19. (c) The requirement is to determine the  possible 
best way to obtain evidence that on­line  to include all possible valid and invalid
access controls are properly functioning.  conditions. An­swer (d) is incorrect 
Answer (c) is correct because entering  because the program that should be 
invalid identification numbers or  tested is the client’s program which is 
passwords will provide the auditor with  used throughout the year.
evidence on whether controls are 
22. (a) The requirement is to determine 
operating as designed. Answer (a) is 
incorrect because directly testing access  which reply isnot among the errors 
con­trols is more direct than testing data  which are generally detected by test
through checkpoints at intervals. Answer 
(b) is incorrect because a transaction log 
will not in general, by itself, identify 
whether transactions were lost or entered 
twice. Answer (d) is incorrect because 
vouching proper authorization is only one 
measure of whether controls are properly 
functioning. 
20. (c) The requirement is to identify the 
situation in which it is most likely that an 
auditor would introduce test data into a 
computerized payroll system. Test data is 
a set of dummy transactions developed by 
the auditor and processed by the client’s 
computer programs. These dummy 
transac­tions are used to determine 
whether the controls which the auditor 
tests are operating effectively. Answer (c) 
is correct because test data with invalid 
employee I.D. numbers could be 
processed to test whether the program 
detects them. An­swer (a) is incorrect 
because the unclaimed payroll checks are 
held by the supervisors and no testing of 
the computer program is involved. 
Answer (b) is incorrect because no 
computer processing is generally involved
when payroll checks are “cashed early” by
employees. Answer (d) is in­correct 
because to test whether the approval of 
overtime is proper, one must determine 
what criteria are used for the decision and 
because the parallel simulation 
data. An auditor uses test data to  method processes the client’s 
determine whether pur­ported controls  data using the CPA’s software. 
are actually functioning. Answer (a) is  An­swers (b) and (c) are 
cor­rect because one would not use test  incorrect because the client’s 
data to test numeric characters in  hardware and software are 
alphanumeric fields; numeric characters  tested using test data designed 
are accepted in alphanumeric fields and  by the CPA. Answer (d) is 
thus do not represent error conditions.  incorrect because although a 
Answer (b) is incorrect because  CPA may test a client’s 
authoriza­tion codes may be tested by  exception reports in various 
inputting inappropriate codes. Answer (c) manners, generalized software is
is incorrect because differing descriptions unlikely to be used. Exception 
of units of measure may be inputted to  reports are gener­ally tested via 
test whether they are accepted. Answer  CPA­prepared test data 
(d) is incorrect because illogical combi­ containing all the possible error 
nations may be inputted to test whether  conditions. The test data are 
they are detected by the system. then run on the client’s 
23. (a) The requirement is to  hardware and software to 
identify the computer­assisted  ascertain whether the ex­ception
auditing technique which allows  reports are “picking up” the 
fictitious and real transactions to CPA’s test data. 
be processed together without  25. (c) The requirement is to 
client operat­ing personnel being identify the information that 
aware of the testing process. An­ must be available to begin 
swer (a) is correct because the  creating automated lead 
integrated test facility ap­proach  schedules. A lead schedule is 
introduces dummy transactions  used to summarize like ac­
into a system in the midst of live counts (e.g., if a client has five 
transactions. Accordingly, client  cash accounts those accounts 
operating personnel may not be  may be summarized on a lead 
aware of the testing process. An­ schedule). Answer (c) is cor­
swer (b) is incorrect because an  rect because lead schedules 
input control matrix would  include information such as 
simply indicate various controls  account numbers, prior year 
in the form of a matrix.  account balances, and current 
Answer (c) is incorrect because the  year unadjusted information. 
parallel simulation tech­nique requires the Answer (a) is incorrect be­cause
processing of actual client data through an interim information is not 
auditor’s software program. In this case,  necessary. Answer (b) is 
the client would be aware of the testing  incorrect because invoice and 
process since the auditor would need to  purchase order numbers are not 
request copies of data run on the actual  summarized on lead schedules. 
system so that the data could then be run  Answer (d) is incorrect because 
on the auditor’s software program.  adjusting entry information is 
Additionally, only valid transactions  identified subsequent to the 
would be tested under parallel simulation.  creation of lead schedules. 
Answer (d) is incorrect because the client 
would generally be aware of an auditor  26. (d) The requirement is to identify 
using a data entry monitor (screen) to  an effect on audit work review 
input transactions.  methods of using microcomputers
in auditing. Answer (d) is correct 
24. (a) The requirement is to  because microcomputers typically
determine the auditing technique produce a number of the 
which utilizes generalized audit  “working papers” in computer 
software. An­swer (a) is correct  disk 
MODULE 8   AUDITING WITH TECHNOLOGY 237
such as sorting and merging. Answer (c) 
is correct because a utility program is a 
form and because many computations, etc.  standard routine for perform­ing 
will be per­formed directly by the  commonly required processing such as 
computer with few details of the cal­ sorting, merging, editing, and 
culations conveniently available. Answer  mathematical routines. Answer (a) is 
(a) is incorrect because the fieldwork  incorrect because compiler programs 
standards remain the same regardless of  translate programming lan­guages such as 
whether or not computers are being  COBOL or FORTRAN to machine 
utilized. Answer (b) is incorrect because  language. Answer (b) is incorrect because 
one would not normally expect con­sulting  supervisory programs or “operating 
services personnel to help with  systems” consist of a series of programs 
documentation. An­swer (c) is incorrect  that per­form functions such as scheduling
because supervisory personnel must have  and supervising the appli­cation programs,
an understanding of the capability and  allocating storage, controlling peripheral 
limitations of microcomputers before they  devices, and handling errors and restarts. 
are utilized on audits. Answer (d) is incorrect because user or 
“application programs” perform specific 
27. (c) The requirement is to  data processing tasks such as general 
determine an auditor’s least  ledger, ac­counts payable, accounts 
likely use of computer software. receivable, and payroll. Applica­tion 
Answer (c) is correctbecause an programs make use of utility routines. 
auditor will judgmentally assess
30. (b) The requirement is to 
control risk related to both the 
computer and manual systems  determine the best ap­proach 
after hav­ing performed the  for determining whether credit 
various tests of controls.  limits are being ex­ceeded 
Answer (a) is incorrect because  when accounts receivable 
computer software will be used  information is stored ondisk. 
to access client data files.  Answer (b) is correct because 
Answers (b) and (d) are  a program to compare actual 
incorrect because software is  account balances with the 
used to prepare spreadsheets  predetermined credit limit and 
and to perform par­allel  thereby prepare a report on 
simulations.  whether any actual credit 
limits are being exceeded will 
28. (a) The requirement is to  accomplish the stated objec­
identify a primary advan­tage  tive. Answer (a) is incorrect 
of using generalized audit  because while test data will 
software packages to audit the  indicate whether the client’s program 
financial statements of a client  allows credit limits to be exceeded, it 
that uses a computer sys­tem.  will not indicate whether credit limits 
Answer (a) is correct because 
areactually being exceeded. Answer (c) 
generalized audit soft­ware 
is incorrect because amanual check of all 
allows an auditor to perform 
audits tests on a client’s  account balances will be very time con­
computer files. Answer (b) is  suming. Answer (d) is incorrect because 
incorrect because generalized  a sample will pro­
audit software packages may 
assist the auditor with either 
substantive tests of transactions
or analytical procedures. 
Answer (c) is incorrect because
while generalized audit 
software might be used to 
perform such operations, this is 
not their primary advantage. 
Answer (d) is incorrect be­
cause generalized audit 
software packages have no 
direct relationship to the 
performance of tests of 
controls. 
29. (c) The requirement is to determine the 
type of com­puter programs which 
auditors use to assist them in func­tions 
detecting unauthorized changes than will 
vide less complete information than  source code comparison. Answer (b) is 
the audit of the entire population  incorrect because check digits are 
which is indicated in answer (b). primarily used as an input control to 
determine that input data is proper. 
31. (c) The requirement is to identify how an  Answer (d) is incorrect because prop­erly 
auditor would test for the presence of  computing control totals is only one 
unauthorized computer pro­gram changes. possible unautho­rized change that might 
Answer (c) is correct because comparing  be made to a program. 
source code of the program with a correct 
version of the program will disclose  32. (b) The requirement is to identify the 
unauthorized computer program changes.  number that represents the record count. 
Answer (a) is incorrect because test data  Answer (b) is correct because the record 
is gener­ally used to test specific controls  count represents the number of records in 
and it will generally be less effective for  a file, in this case 4. 
238 MODULE 8   AUDITING WITH TECHNOLOGY

SOLUTION TO SIMULATION

Task­Based Simulation 1
Computer Audit Authoritative
Techniques and Terms Literature Help

D
e
1. A

2. D
u
3. F
i
4. M
a
5. U

E
1. (A) Auditing “around” the computer involves examining inputs into and 
outputs from the computer while ignoring pro­cessing, as contrasted to 
auditing “through” the computer which in some manner directly utilizes the 
computer’s processing ability. 
2. (F)   Test   data   is   a   set   of   dummy   transactions   developed   by   the   auditor   and
processed by the client’s computer programs to determine whether the controls
that the auditor intends to rely upon are functioning as expected. 
3. (C) An integrated test facility introduces dummy transactions into a system in
the midst of live transactions and is often built into the system during the
original design. 
4. (C) An integrated test facility approach may incorporate a simulated division or subsidiary
into the accounting system with the sole purpose of running test data through it. 
5. (D)   Parallel   simulation   involves   processing   actual   client   data   through   an
auditor’s software program to determine whether the output equals that obtained
when the client processed the data. 

D
e
6. A

7. A
 
8. A
 
9. A

10. A
 
E
6. (N) A missing data check tests whether blanks exist in input data where they
should not (e.g., an employee’s division number). When the data is missing, an
error message is output. 
7. (I) Boundary protection is necessary because most large computers have more
than   one   job   running   simultaneously   (a   multiprogramming   environment).   To
ensure   that   these   simultaneous   jobs   cannot   destroy   or   change   the   allocated
memory   of   another   job,   the   systems   software   contains   boundary   protection
controls. 
8. (J) A check digit is an extra digit added to an identification number to detect certain types 
of data transmission or trans­position errors. It is used to verify that the number was entered
into the computer system correctly; one approach is using a check digit that is calculated as 
a mathematical combination of the other digits. 
9. (O) Personal identification codes require individuals to in some manner identify
themselves to determine that only au­thorized users access programs or files. 
10. (M) A hash total is the total of one field for all the records of a batch where the 
total is a meaningless total for financial purposes, such as a mathematical sum of 
employee social security numbers to determine that all employees have been 
processed. 
FINANCIAL ACCOUNTING 
AND REPORTING
The Financial Accounting and Reporting Exam is scheduled for four hours. Based on information released by the 
AICPA, candidates should expect three multiple­choice testlets of approximately thirty questions each, and two 
simulations.
The Uniform CPA Examination Content Specifications appear in Volume 1, Outlines and Study Guides.

Module 9/Basic Theory and Financial Reporting (TREP)
A.  Basic Concepts
83 Multiple­Choice 241 254
3 Task­Based Simulations 249 263
B.  Error Correction
15 Multiple­Choice 268 274
6 Task­Based Simulations 270 276
C.  Accounting Changes
29 Multiple­Choice 280 288
7 Task­Based Simulations 283 290
D.  Financial Statements
114 Multiple­Choice 295 315
13 Task­Based Simulations 306 324

Module 10/Inventory (INVY)
68 Multiple­Choice 333 346
11 Task­Based Simulations 340 354

Module 11/Fixed Assets (FA)
102 Multiple­Choice 361 377
11 Task­Based Simulations 372 386

Module 12/Monetary Current Assets and Current Liabilities (CACL)
116 Multiple­Choice 392 309
9 Task­Based Simulations 405 421

Module 13/Present Value (PV)
A.  Fundamentals
25 Multiple­Choice 426 432
5 Task­Based Simulations 429 435
B.  Bonds
43 Multiple­Choice 437 446
7 Task­Based Simulations 442 451
C.  Debt Restructure
8 Multiple­Choice 456 458
2 Task­Based Simulation 457 459
D.  Pensions
43 Multiple­Choice 460 467
3 Task­Based Simulation 465 471
E.  Leases
60 Multiple­Choice 473 484
9 Task­Based Simulations 480 490
239
240 CH 3  FINANCIAL ACCOUNTING AND REPORTING

Module 14/Deferred Taxes (DETX)
44 Multiple­Choice 494 503
8 Task­Based Simulations 499 509

Module 15/Stockholders’ Equity (STK)
94 Multiple­Choice 513 529
10 Task­Based Simulations 523 540

Module 16/Investments (IVES)
64 Multiple­Choice 546 559
11 Task­Based Simulations 553 567

Module 17/Statement of Cash Flows (SCF)
39 Multiple­Choice 573 582
6 Task­Based Simulations 577 588

Module 18/Business Combinations and Consolidations (BCC)
79 Multiple­Choice 592 605
8 Task­Based Simulations 601 613

Module 19/Derivative Instruments and Hedging Activities (DIHA)
51 Multiple­Choice 617 626
7 Task­Based Simulations 622 630

Module 20/Miscellaneous (MISC)
A.  Personal Financial Statements
17 Multiple­Choice 634 636
B.  Interim Reporting
19 Multiple­Choice 638 640
C.  Segment Reporting
12 Multiple­Choice 642 646
5 Task­Based Simulations 644 647
D.  Partnership Accounting
23 Multiple­Choice 649 655
6 Task­Based Simulations 652 658
E.  Foreign Currency Translation
13 Multiple­Choice 661 666
3 Task­Based Simulations 663 668

Module 21/Governmental (State and Local) Accounting
124 Multiple­Choice 670 682

Module 22/Not­for­Profit Accounting (NFP)
83 Multiple­Choice 695 704

Financial Accounting and Reporting Sample Examination 1233

Financial Accounting and Reporting AICPA Sample Testlets 1306

Financial Accounting and Reporting 2011 Released AICPA Questions 1344
MODULE 9  BASIC TH. & FIN. REP.: A. BASIC CONCEPTS 241

BASIC CONCEPTS
Accounting Con­cepts intended to establish? 
MULTIPLE­CHOICE QUESTIONS (1­83)
a. Generally   accepted   accounting
1. What   are   the   Statements   of   Financial principles   in   finan­cial   reporting   by
business enterprises.  comprehensive income for an enterprise in an 
b. The   meaning   of   “Present   fairly   in industry not having specialized accounting principles?
accordance   with   generally   accepted
accounting principles.” 
c. The objectives and concepts for use in
developing   standards   of   financial
accounting and reporting. 
d. The hierarchy of sources of generally
accepted ac­counting principles. 
2. According to the FASB conceptual framework, 
the objectives of financial reporting for business
enterprises are based on 
a. Generally accepted accounting principles. 
b. Reporting for regulators. 
c. The need for conservatism. 
d. The needs of the users of the information. 
3. According to the FASB conceptual framework, the 
relevance of providing information in financial statements 
is subject to the constraint of 
a. Comparability. 
b. Cost­benefit. 
c. Reliability. 
d. Faithful representation. 
4. The   enhancing   qualitative   characteristics   of
financial reporting are 
a. Relevance,   reliability,   and   faithful
representation. 
b. Cost­benefit and materiality. 
c. Comparability,   verifiability,
timeliness,   and
understandability. 
d. Completeness,   neutrality,   and   freedom   from
error. 
5. According   to   Statements   of   Financial
Accounting   Con­cepts,   neutrality   is   an
ingredient of 
Faithful
representation Relevance
a. Yes Yes
b. Yes No
c. No Yes
d. No No
6. According to the FASB conceptual framework,
which of the following is an enhancing quality
that   relates   to   both   relevance   and   faithful
representation? 
a. Comparability. 
b. Confirmatory value. 
c. Predictive value. 
d. Freedom from error. 
7. According to the FASB conceptual framework, the 
process of reporting an item in the financial statements of 
an entity is 
a. Allocation. 
b. Matching. 
c. Realization. 
d. Recognition. 
8. Under FASB Statement of Financial Accounting
Con­cepts   5,   which   of   the   following   items
would   cause   earnings   to   differ   from
a. Unrealized   loss  on  investments c. Cash was collected on accounts receivable. 
classified   as   available­for­sale d. Product   unit  costs   were   assigned   to
securities.  cost   of   goods   sold   when   the   units
b. Unrealized   loss   on   investments were sold. 
classified as trad­ing securities. 
c. Loss on exchange of similar assets.  12. What is the underlying concept that supports
d. Loss on exchange of dissimilar assets.  the   im­mediate   recognition   of   a   contingent
loss? 
9. Under   FASB   Statement   of   Financial a. Substance over form. 
Accounting   Con­cepts   5,   comprehensive b. Consistency. 
income   excludes   changes   in   equity   resulting c. Matching. 
from which of the following?  d. Conservatism. 
a. Loss from discontinued operations. 
b. Prior period error correction.  13. What is the underlying concept governing the 
c. Dividends paid to stockholders.  generally accepted accounting principles 
d. Unrealized loss on  securities pertaining to recording gain contingencies? 
classified   as   available­for­ a. Conservatism. 
sale.  b. Relevance. 
c. Consistency. 
10. The fundamental qualitative characteristic of d. Faithful representation. 
faithful representation has the components of 
a. Predictive value and confirmatory value.  14. FASB’s conceptual framework explains both 
b. Comparability,   consistency,   and financial and physical capital maintenance 
confirmatory value.  concepts. Which capital maintenance concept is 
c. Understandability,   predictive   value,   and applied to currently reported net income, and 
reliability.  which is applied to comprehensive income? 
d. Completeness,   neutrality,   and   freedom   from Currently
error.  reported net income Comprehensive income
a. Financial capital Physical capital
11. According to the FASB conceptual framework,  b. Physical capital Physical capital
which of the following statements conforms to  c. Financial capital Financial capital
the realization con­cept?  d. Physical capital Financial capital
a. Equipment depreciation was assigned to
a   produc­tion   department   and   then   to 15. According to the FASB conceptual framework, an 
product unit costs.  en­tity’s revenue may result from
b. Depreciated   equipment   was   sold   in a. A decrease in an asset from primary operations. 
exchange for a note receivable.  b. An   increase   in   an   asset   from   incidental
transac­tions. 
242 MODULE 9  BASIC TH. & FIN. REP.: A. BASIC CONCEPTS
with probabilities of 20%, 50%, 
and 30%, respectively. The rate of 
c. An increase in a liability interest on default risk­free 
from   incidental   transac­ investments is 5%. The pres­ent 
tions.  value factors are 
d. A   decrease   in   a   liability   from PV of 1, at 5%, for 1 year is .95238
primary operations.  PV of 1, at 5%, for 2 years is .90703
PV of 1, at 5%, for 3 years is .86384
16. According to the FASB conceptual
framework, which of the following What is the expected present value of 
is an essential characteristic of an Lydia Nickels’ cash flow (in whole 
asset?  dollars)?
a. The   claims   to   an a. $181,406 
asset’s   benefits   are b. $180,628 
legally en­forceable.  c. $  90,703 
b. An asset is tangible.  d. $  89,925 
c. An asset is obtained at a cost. 
22. Which   of   the   following
d. An asset provides future benefits. 
statements   regarding   interest
17. According to the FASB conceptual methods   of   allocations   is  not
framework, which of the following true? 
attributes would not be used to  a. The term “interest methods 
measure inventory?  of allocation” refers both to 
a. Historical cost.  the convention for periodic 
b. Replacement cost.  reporting and to 
c. Net realizable value. 
d. Present value of future cash flows. 
18. According to SFAC 7, Using Cash 
Flow Informationand Present Value
in Accounting Measurements, the 
mostrelevant measurement of an 
entity’s liabilities at initial rec­
ognition and fresh­start 
measurements should always reflect
a. The   expectations   of   the   entity’s
management. 
b. Historical cost. 
c. The credit standing of the entity. 
d. The   single   most­likely
minimum   or   maximum
pos­sible amount. 
19. Which of the following is not covered by 
SFAC 7, Us­ing Cash Flow Information and 
Present Value in Accounting Measurements?
a. Measurements at initial recognition. 
b. Interest method of amortization. 
c. Expected cash flow approach. 
d. Determining   when
fresh­start
measurements   are
appropriate. 
20. In   calculating   present   value   in   a
situation  with  a  range   of  possible
outcomes all discounted using the
same   interest   rate,   the   expected
present value would be 
a. The most­likely outcome. 
b. The maximum outcome. 
c. The minimum outcome. 
d. The   sum   of   probability­weighted
present values. 
21. A cash flow of $200,000 may be 
received by Lydia Nickels, Inc. in 
one year, two years, or three years, 
delivery, and the remaining 25% 
the several approaches to dealing  due on second delivery. What 
with changes in estimated future  amount of revenue should Acme 
cash flows. recognize from this sale during 
b. Interest   methods   of 2010? 
allocation   are   reporting a. $  75,000 
con­ventions   that   use b. $150,000 
present   value   techniques c. $225,000 
in the absence of a fresh­ d. $300,000 
start   measurement   to
compute   changes   in   the 27. Amar Farms produced 300,000 pounds of 
carrying   amount   of   an cotton during the 2010 season. Amar sells all 
asset   or   li­ability   from of its cotton to Brye Co., which has agreed to 
one period to the next.  purchase Amar’s entire production at the 
c. Interest   methods   of prevailing market price. Recent legislation 
allocation are grounded in assures that the market price will not fall 
the notion of current cost.  below $.70 per pound during the next two 
d. Holding gains and losses years. Amar’s costs of selling and distributing
are   generally   excluded the cotton are immaterial and can be 
from allocation systems.  reasonably estimated. Amar reports its 
inventory at expected exit value. During 2010,
23. Which of the following is not an Amar sold and delivered to Brye 200,000 
objective of using present value pounds at the market price of $.70. Amar sold 
in accounting measurements?  the remaining 100,000 pounds during 2011 at 
a. To capture the value of an the market price of $.72. What amount of 
asset   or   a   liability   in   the revenue should Amar recognize in 2010? 
context   of   a   particular a. $140,000 
entity.  b. $144,000 
b. To estimate fair value. 
c. To capture the economic
difference   between   sets
of future cash flows. 
d. To   capture   the   elements
that   taken   together   would
comprise a market price if
one existed. 
24. On December 31, 2010, Brooks Co. 
decided to end op­erations and 
dispose of its assets within three 
months. At December 31, 2010, the 
net realizable value of the equip­
ment was below historical cost. 
What is the appropriate 
measurement basis for equipment 
included in Brooks’ De­cember 31, 
2010 balance sheet? 
a. Historical cost. 
b. Current reproduction cost. 
c. Net realizable value. 
d. Current replacement cost. 
25. Which of the following accounting
literature   is   not   in­cluded   in   the
FASB   Accounting   Standards
Codification? 
a. AICPA Statements of Position. 
b. FASB Statements. 
c. Accounting Research Bulletins. 
d. Statements of Auditing Standards. 
26. On October 1, 2010, Acme Fuel Co.
sold 100,000 gal­lons of heating oil 
to Karn Co. at $3 per gallon. Fifty 
thou­sand gallons were delivered on
December 15, 2010, and the 
remaining 50,000 gallons were 
delivered on January 15, 2011. 
Payment terms were: 50% due on 
October 1, 2010, 25% due on first 
MODULE 9  BASIC TH. & FIN. REP.: A. BASIC CONCEPTS 243
December 31, 2010, revealed the follow­
ing:
c. $210,000 
• An opening balance  of $1,500 for
d. $216,000  Thrift’s   compre­hensive
28. Lin Co., a distributor of machinery,  insurance policy. Thrift had paid
bought a machine from the  an annual premium of $3,000 on
manufacturer in November 2010 for  July 1, 2009. 
$10,000. On December 30, 2010,  • A   $3,200   annual   insurance
Lin sold this machine to Zee  premium   payment  made   July   1,
Hardware for $15,000, under the  2010. 
following terms: 2% discount if paid • A $2,000 advance rental payment for a
within thirty days, 1% discount if  warehouse Thrift leased for one year
paid after thirty days but within sixty beginning January 1, 2011. 
days, or payable in full within ninety In its December 31, 2010 balance sheet, 
days if not paid within the discount  what amount should Thrift report as prepaid 
periods. However, Zee had the right  expenses?
to return this machine to Lin if Zee  a. $5,200 
was unable to resell the machine  b. $3,600 
before expiration of the ninety­day 
c. $2,000 
payment period, in which case Zee’s
d. $1,600 
obligation to Lin would be can­
celed. In Lin’s net sales for the year  32. Roro, Inc. paid $7,200 to  renew
ended December 31, 2010, how  its   only   insurance   pol­icy   for
much should be included for the sale three years on March 1, 2010, the
of this ma­chine to Zee?  effective date of 
a. $0 
b. $14,700 
c. $14,850 
d. $15,000 
29. Under a royalty agreement with 
another company, Wand Co. will 
pay royalties for the assignment of
a patent for three years. The 
royalties paid should be reported 
as expense 
a. In the period paid. 
b. In the period incurred. 
c. At   the   date   the   royalty   agreement
began. 
d. At   the   date   the   royalty   agreement
expired. 
30. Clark Co.’s advertising expense 
account had a balance of $146,000 
at December 31, 2010, before any 
necessary year­end adjustment 
relating to the following: 
• Included in the $146,000 is the 
$15,000 cost of print­ing catalogs 
for a sales promotional campaign in 
January 2011. 
• Radio advertisements broadcast 
during December 2010 were billed 
to Clark on January 2, 2011. Clark 
paid the $9,000 invoice on January 
11, 2011. 
What amount should Clark report as advertising
expense in its income statement for the year 
ended December 31, 2010?
a. $122,000 
b. $131,000 
c. $140,000 
d. $155,000 
31. An analysis of Thrift Corp.’s 
unadjusted prepaid ex­pense account at 
merchandise, that expire one year 
the policy. At March 31, 2010, Roro’s  after their issuance. Regal has the 
unadjusted trial bal­ance showed a balance of  following information pertaining to 
$300 for prepaid insurance and $7,200 for  its gift certificates sales and 
insurance expense. What amounts should be  redemptions: 
reported for prepaid insurance and insurance  Unredeemed at 12/31/09 $  75,000
expense in Roro’s financial statements for the  2010 sales 250,000
three months ended March 31, 2010? 2010 redemptions of prior year sales 25,000
2010 redemptions of current year sales 175,000
Prepaid insurance Insurance expense
a. $7,000 $300 Regal’s experience indicates that 10% of 
b. $7,000 $500 gift certificates sold will not be redeemed. 
c. $7,200 $300 In its December 31, 2010 bal­ance sheet, 
d. $7,300 $200 what amount should Regal report as 
unearned revenue?
33. Aneen’s Video Mart sells one­ and two­ a. $125,000 
year mail ordersubscriptions for its video­of­ b. $112,500 
the­month business. Subscrip­tions are 
c. $100,000 
collected in advance and credited to sales. An 
analysis of the recorded sales activity revealed  d. $  50,000 
the follow­ing: 74. Wren Corp.’s trademark was licensed
2009 to Mont Co. for royalties of 15% of 
Sales $420,000 $500,000
Less cancellations 20,000 30,000 sales of the trademarked items. 
Net sales $400,000 $470,000 Royal­ties are payable semiannually 
Subscriptions expirations: on March 15 for sales in July through
2009 $120,000 December of the prior year, and on 
2010 155,000 $130,000 September 15 for sales in January 
2011 125,000 200,000 through June of the same year. Wren 
2012 140,000
$400,000 $470,000 received the following royalties from 
Mont: 
In Aneen’s December 31, 2010 balance  March 15 September 15
sheet, the balance for unearned subscription  2009 $10,000 $15,000
revenue should be 2010 12,000 17,000
a. $495,000 
b. $470,000  Mont   estimated   that   sales   of   the
c. $465,000  trademarked items would total $60,000 for
July   through   December   2010.   In   Wren’s
d. $340,000 
2010 income statement, the royalty revenue
34. Regal Department Store sells gift  should be
certificates, redeem­able for store  a. $26,000 
b. $29,000 
244 MODULE 9  BASIC TH. & FIN. REP.: A. BASIC CONCEPTS
39. Cooke Company acquires patent 
c. $38,000  rights from other en­terprises and 
d. $41,000  pays advance royalties in some 
cases, and in others, royalties are 
36. In 2009, Super Comics Corp. sold a  paid within ninety days after year­
comic strip to Fan­tasy, Inc. and  end. The following data are 
will receive royalties of 20% of  included in Cooke’s December 31 
future reve­nues associated with the  balance sheets: 
comic strip. At December 31, 2010,  2009 2010
Super reported royalties receivable  Prepaid royalties $55,000 $45,000
of $75,000 from Fantasy. During  Royalties payable 80,000 75,000
2011, Super received royalty 
During  2010 Cooke remitted royalties  of
payments of $200,000. Fantasy 
$300,000. In its income statement for the
reported revenues of $1,500,000 in 
year   ended   December   31,   2010,   Cooke
2011 from the comic strip. In its 
should report royalty expense of
2011 income statement, what 
a. $295,000 
amount should Super report as 
royalty revenue?  b. $305,000 
a. $125,000  c. $310,000 
b. $175,000  d. $330,000 
c. $200,000 
d. $300,000 
37. Rill Co. owns a 20% royalty interest
in an oil well. Rill receives royalty 
payments on January 31 for the oil 
sold between the previous June 1 
and November 30, and on 
July 31 for oil sold between December 1 and
May   31.   Pro­duction   reports   show   the
following oil sales: 
June 1, 2009 ­ November 30, 2009
December 1, 2009 ­ December 31, 2009
December 1, 2009 ­ May 31, 2010
June 1, 2010 ­ November 30, 2010
December 1, 2010 ­ December 31, 2010
What amount should Rill report as royalty 
revenue for 2010?
a. $140,000 
b. $144,000 
c. $149,000 
d. $159,000 
38. Decker Company assigns some of its
patents to other enterprises under a 
variety of licensing agreements. In 
some instances advance royalties are
received when the agree­ments are 
signed, and in others, royalties are 
remitted within sixty days after each 
license year­end. The following data
are included in Decker’s December 
31 balance sheet: 
2009 2010
Royalties receivable $90,000 $85,000
Unearned royalties 60,000 40,000

During 2010 Decker received royalty 
remittances of $200,000. In its income 
statement for the year ended De­cember 
31, 2010, Decker should report royalty 
income of
a. $195,000 
b. $215,000 
c. $220,000 
d. $225,000 
ferred revenue account be affected 
40. The premium on a three­year insurance policy by each of the following 
expiring on December 31, 2012, was paid in  transactions? 
total on January 1, 2010. The original  Redemption of certificates Lapse of certificates
payment was initially debited to a prepaid  a. No effect Decrease
asset account. The appropriate journal entry  b. Decrease Decrease
has been recorded on December 31, 2010. The c. Decrease No effect
balance in the prepaid asset ac­count on  d. No effect No effect
December 31, 2010, should be 
a. Zero.  43. Jersey, Inc. is a retailer of home 
b. The same as it would have been if  appliances and offers a service 
the original pay­ment had been  contract on each appliance sold. 
debited initially to an expense ac­ Jersey sells appli­ances on 
count.  installment contracts, but all service 
c. The same as the original payment.  contracts must be paid in full at the 
time of sale. Collections received 
d. Higher than if the original
for service contracts should be 
payment   had   been   deb­
ited initially to an expense recorded as an increase in a 
account.  a. Deferred revenue account. 
b. Sales contracts receivable valuation
41. On January 1, 2010, Sip Co. signed  account. 
a five­year contract enabling it to  c. Stockholders’ valuation account. 
use a patented manufacturing  d. Service revenue account. 
process begin­ning in 2010. A 
royalty is payable for each product  44. Ward, a consultant, keeps her 
pro­duced, subject to a minimum  accounting records on a cash basis. 
annual fee. Any royalties in excess  During 2010, Ward collected 
of the minimum will be paid  $200,000 in fees from clients. At 
annually. On the con­tract date, Sip  December 31, 2009, Ward had 
prepaid a sum equal to two years’  accounts receivable of $40,000. At 
minimum annual fees. In 2010, only  December 31, 2010, Ward had 
minimum fees were incurred. The  accounts receivable of $60,000, and 
royalty prepayment should be  unearned fees of $5,000. On an 
reported in Sip’s Decem­ber 31,  accrual basis, what was Ward’s 
2010 financial statements as  service reve­nue for 2010? 
a. An expense only.  a. $175,000 
b. A current asset and an expense.  b. $180,000 
c. A current asset and noncurrent asset. c. $215,000 
d. A noncurrent asset.  d. $225,000 

42. A retail store received cash and  45. Zeta   Co.   reported   sales   revenue


issued gift certificates that are  of   $4,600,000   in   its   income
redeemable in merchandise. The  statement   for   the   year   ended
gift certificates lapse one year after  December   31,   2010.   Additional
they are issued. How would the de­ information is as follows: 
MODULE 9  BASIC TH. & FIN. REP.: A. BASIC CONCEPTS 245
49. Class Corp. maintains its accounting
records on the cash basis but restates
12/31/09 its financial statements to the 
Accounts receivable $1,000,000 accrual method of accounting. Class 
Allowance for uncollectible accounts (60,000)
had $60,000 in cash­basis pretax 
Zeta   wrote   off   uncollectible   accounts income for 2010. The following 
totaling   $20,000   dur­ing   2010.   Under   the information pertains to Class’s 
cash basis of accounting, Zeta would have operations for the years ended 
reported 2010 sales of December 31, 2010 and 2009: 
a. $4,900,000  2010 2009
b. $4,350,000  Accounts receivable $40,000 $20,000
Accounts payable 15,000 30,000
c. $4,300,000 
d. $4,280,000 
46. Marr Corp. reported rental revenue
of   $2,210,000   in   its   cash   basis
federal   income   tax   return   for   the
year   ended   No­vember   30,   2010.
Additional   information   is   as
follows: 
Rents receivable—November 30, 2010
Rents receivable—November 30, 2009
Uncollectible rents written off during the fiscal year

Under the accrual basis, Marr should report 
rental revenue of
a. $1,920,000 
b. $1,980,000 
c. $2,440,000 
d. $2,500,000 
47. The following information pertains to 
Eagle Co.’s 2010sales:
Cash sales
Gross $  80,000
Returns and allowances 4,000
Credit sales
Gross 120,000
Discounts 6,000
On January 1, 2010 customers owed Eagle 
$40,000. On December 31, 2010, customers 
owed Eagle $30,000. Eagle uses the direct 
writeoff method for bad debts. No bad debts 
were recorded in 2010. Under the cash basis 
of accounting, what amount of net revenue 
should Eagle report for 2010?
a. $  76,000 
b. $170,000 
c. $190,000 
d. $200,000 
48. The   following   balances   were
reported   by   Mall   Co.   at
December 31, 2010 and 2009: 
12/31/10 12/31/09
Inventory $260,000 $290,000
Accounts payable 75,000 50,000
Mall paid suppliers $490,000 during the year 
ended Decem­ber 31, 2010. What amount 
should Mall report for cost of goods sold in 
2010?
a. $545,000 
b. $495,000 
c. $485,000 
d. $435,000 
d. No Yes
Under the accrual method, what amount of 
52. White Co. wants to convert its 2010 
income before taxes should Class report in 
its December 31, 2010 income statement? financial state­ments from the accrual basis 
a. $25,000  of accounting to the cash basis. Both supplies
inventory and office salaries payable in­
b. $55,000 
creased between January 1, 2010, and 
c. $65,000 
December 31, 2010. To obtain 2010 cash 
d. $95,000 
basis net income, how should these increases 
50. On February 1, 2011, Tory began a service  be added to or deducted from accrual­basis 
proprietor­ship with an initial cash investment net income?
of $2,000. The pro­prietorship provided  Supplies inventory Office salaries payable
$5,000 of services in February and re­ceived  a. Deducted Deducted
full payment in March. The proprietorship  b. Deducted Added
incurred expenses of $3,000 in February,  c. Added Deducted
which were paid in April. During March, Tory d. Added Added
drew $1,000 against the capital ac­count. In 
the proprietorship’s financial statements for  53. Before 2010, Droit Co. used the cash 
the two months ended March 31, 2011,  basis of account­ing. As of December 31, 
prepared under the cash basis method of  2010, Droit changed to the accrual basis. 
accounting, what amount should be reported  Droit cannot determine the beginning balance
as capital?  of sup­plies inventory. What is the effect of 
a. $1,000  Droit’s inability to determine beginning 
b. $3,000  supplies inventory on its 2010 accrual­basis 
c. $6,000  net income and December 31, 2010 accrual­
d. $7,000  basis own­ers’ equity?
2010 net income 12/31/10 owners’ equity
51. Compared to the  accrual basis of a. No effect No effect
accounting,   the   cash   basis   of b. No effect Overstated
accounting understates income by c. Overstated No effect
the   net   decrease   during   the d. Overstated Overstated
accounting period of 
54. Gant Co., which began operations on 
Accounts receivable Accrued expenses
January 1, 2010,appropriately uses the 
a. Yes Yes
b. Yes No installment method of accounting. The 
c. No No following information pertains to Gant’s 
operations for the year 2010:
246 MODULE 9  BASIC TH. & FIN. REP.: A. BASIC CONCEPTS
Gross profit percentages 30% 40%
What amount of installment accounts 
Installment sales $500,000
receivable should Dolce report in its 
Regular sales
Cost of installment sales December 31, 2011 balance sheet?
Cost of regular sales a. $1,225,000 
General and administrative expenses b. $1,300,000 
Collections on installment sales c. $1,700,000 
In its December 31, 2010 balance sheet, what  d. $1,775,000 
amount should Gant report as deferred gross 
profit?
a. $250,000 
b. $200,000 
c. $160,000 
d. $  75,000 
55. Since there is no reasonable 
basis for estimating the degree 
of collectibility, Astor Co. uses 
the installment method of 
revenue recognition for the 
following sales: 
2010 2009
Sales $900,000 $600,000
Collections from:
2009 sales 100,000 200,000
2010 sales 300,000
Accounts written off:
2009 sales 150,000 50,000
2010 sales 50,000
Gross profit percentage 40%
What amount should Astor report as deferred
gross profit in its December 31, 2010 balance
sheet for the 2009 and 2010 sales?
a. $150,000 
b. $160,000 
c. $225,000 
d. $250,000 
56. Luge Co., which began operations
on January  2, 2010,  appropriately
uses   the   installment   sales   method
of   account­ing.   The   following
information is available for 2011: 
Installment accounts receivable, December 31, 2011
Deferred gross profit, December 31, 2011 (before
recognition of realized gross profit for 2011)
Gross profit on sales
For the year ended December 31, 2011, cash 
collections and realized gross profit on sales 
should be
Cash collections Realized gross profit
a. $400,000 $320,000
b. $400,000 $240,000
c. $600,000 $320,000
d. $600,000 $240,000
57. Dolce Co., which began operations on 
January 1, 2010,appropriately uses the 
installment method of accounting to record 
revenues. The following information is 
available for the years ended December 31, 
2010 and 2011:
2010
Sales $1,000,000
Gross profit realized on sales
made in:
2010 150,000
2011 ­­
cost   of   sales
58. On December 31, 2010, Mill Co.  have   been
sold construction equipment to  received. 
Drew, Inc. for $1,800,000. The  62. Income recognized using the 
equipment had a carrying amount of 
installment method of accounting 
$1,200,000. Drew paid $300,000 
cash on December 31, 2010, and  generally equals cash collected 
signed a $1,500,000 note bear­ing  multiplied by the 
interest at 10%, payable in five  a. Net operating profit percentage. 
annual installments of $300,000.  b. Net   operating
Mill appropriately accounts for the  profit
sale under the installment method.  percentage
On December 31, 2011, Drew paid  adjusted   for
$300,000 principal and $150,000  ex­pected
interest. For the year ended  uncollectible
December 31, 2011, what total  accounts. 
amount of revenue should Mill  c. Gross profit percentage. 
recognize from the construction  d. Gross   profit
equipment sale and financing?  percentage
a. $250,000  adjusted   for
b. $150,000  expected   un­
c. $120,000  collectible
d. $100,000  accounts. 

59. On January 2, 2010, Blake Co. sold a used  63. It   is   proper   to   recognize


machine to Cooper, Inc. for $900,000,  revenue   prior   to   the   sale   of
resulting in a gain of $270,000. On that date,  merchandise when 
Cooper paid $150,000 cash and signed a  I. The   revenue   will   be   reported   as   an
$750,000 note bearing interest at 10%. The  installment sale. 
note was pay­able in three annual installments  AI. The   revenue   will   be   reported
of $250,000 beginning January 2, 2011. Blake  under the cost recovery method. 
appropriately accounted for the sale under the  a. I only. 
installment method. Cooper made a timely  b. II only. 
pay­ment of the first installment on January 2, 
c. Both I and II. 
2011, of $325,000, which included accrued 
interest of $75,000. What amount of deferred  d. Neither I nor II. 
gross profit should Blake report at De­cember 
31, 2011? 
a. $150,000 
b. $172,500 
c. $180,000 
d. $225,000 
60. For   financial   statement
purposes,   the   installment
method of accounting may be
used if the 
a. Collection
period   extends
over more than
twelve months.
b. Installments   are   due   in   different
years. 
c. Ultimate   amount   collectible   is
indeterminate. 
d. Percentage­of­completion   method   is
inappropriate. 
61. According to the installment method
of accounting, gross profit on an 
installment sale is recognized in 
income 
a. On the date of sale. 
b. On the date the final cash collection
is received. 
c. In proportion to the cash collection. 
d. After   cash
collections
equal   to   the
MODULE 9  BASIC TH. & FIN. REP.: A. BASIC CONCEPTS 247
agreement, the nonrefundable down payment 
represents a fair measure of the services 
64. The following information pertains to already performed by Rice; however, 
a sale of realestate by Ryan Co. to Sud  substantial future services are required of 
Co. on December 31, 2010: Rice. Collectibility of the note is reasonably 
certain. In Rice’s December 31, 2010 balance 
Carrying amount $2,000,000
Sales price: sheet, unearned franchise fees from Graf’s 
Cash $  300,000 franchise should be reported as 
Purchase money mortgage 2,700,000 a. $132,000 
b. $100,000 
The mortgage is payable in nine annual 
c. $  90,000 
installments of $300,000 beginning December
31, 2011, plus interest of 10%. The December  d. $  72,000 
31, 2011, installment was paid as scheduled,  68. Each of Potter Pie Co.’s twenty­one 
together with interest of $270,000. Ryan uses  new franchisees contracted to pay 
the cost recovery method to account for the  an initial franchise fee of $30,000. 
sale. What amount of income should Ryan  By December 31, 2010, each 
recognize in 2011 from the real es­tate sale  franchisee had paid a nonrefund­
and its financing? able $10,000 fee and signed a note 
a. $570,000  to pay $10,000 principal plus the 
b. $370,000  market rate of interest on December 
c. $270,000  31, 2011, and December 31, 2012. 
d. $0  Experience indicates that one 
franchi­see will default on the 
65. Wren Co. sells equipment on  additional payments. Services for 
installment contracts. Which of the  the initial fee will be performed in 
following statements best justifies  2011. What amount of net 
Wren’s use of the cost recovery 
method of revenue recognition to 
ac­count for these installment 
sales? 
a. The sales contract 
provides that title to the 
equip­ment only passes 
to the purchaser when all 
pay­ments have been 
made. 
b. No   cash   payments   are
due   until   one   year   from
the date of sale. 
c. Sales   are   subject   to   a   high   rate   of
return. 
d. There   is   no   reasonable   basis   for
estimating collecti­bility. 
66. According   to   the   cost   recovery
method of accounting, gross profit
on   an   installment   sale   is
recognized in income 
a. After   cash   collections
equal   to   the   cost   of
sales   have   been
received. 
b. In proportion to the cash collections. 
c. On the date the final cash collection
is received. 
d. On the date of sale. 
67. On December 31, 2010, Rice, Inc. authorized 
Graf to operate as a franchisee for an initial 
franchise fee of $150,000. Of this amount, 
$60,000 was received upon signing the 
agreement and the balance, represented by a 
note, is due in three annual payments of 
$30,000 each be­ginning December 31, 2011. 
The present value on Decem­ber 31, 2010, of 
the three annual payments appropriately 
discounted is $72,000. According to the 
lle
unearned franchise fees would Potter report at cti
December 31, 2010? on
a. $400,000  ha
b. $600,000  s
c. $610,000  oc
d. $630,000  cu
rre
69. In which of the following examples  d
of real estate trans­actions would  for
the seller not transfer the usual risks all
and re­wards of ownership?  of
I. The buyer can compel the seller to th
repurchase the prop­erty.  e
AI. The   seller   guarantees   the   return   of   the se
buyer’s investment.  pa
BI. The  seller  is  required  to rat
support operations of the e
buyer   and   will   be un
reimbursed on a cost plus its.
5% basis.  d. The
a. I.  sep
arat
b. II. 
e
c. III.  unit
d. I and II.  s
70. Esker Inc. specializes in real estate  mu
st
transactions other than retail land 
be
sales. On January 1, 2011, Esker 
deli
consum­mated a sale of property to 
ver
Kame Ltd. The amount of profit on 
ed
the sale is determinable and Esker is wit
not obligated to perform any  hin
additional activities to earn the  90
profit. Kame’s initial and  day
continuing investments were  s of
adequate to demon­strate a  the
commitment to pay for the property. end
However, Esker’s receivable may  of
be subject to future subordination.  the
Esker should account for the sale  acc
using the  oun
a. Deposit method.  ting
b. Reduced recovery method.  peri
c. Cost recovery method.  od. 
d. Full accrual method. 
72. The   milestone   method   of
71. Which of the following is one of  accounting   may   be   used   to
the conditions that must exist for a recognize revenue for 
company to recognize revenue on  a. Multiple­deliverable   products   or
separate units under a multiple­ services. 
deliverables arrangement?  b. Research   and   development
a. The arrangements. 
deliver c. Long­term construction contracts. 
ed d. Franchise arrangements. 
item
has 73. The milestone method of revenue 
value recognition provides that if a 
on   a substantive milestone is achieved, 
stand­ what amount of revenue is 
alone recognized? 
ba­sis a. Rev
and enue
can   be is
sold reco
separat gniz
ely.  ed
b. The delivered item is not returnable.  up to
c. Co the
amo upon
unt the
of percen
cash tage
colle deliver
cted. ed   to
b. A date. 
provisi c. Contingent revenue is recognized in
ons its entirety. 
rata d. A percentage of total revenue based
share on the separate units delivered. 
of
revenu 74. Which   of   the   following
e organizations   is   responsible   for
based setting   International   Financial
Reporting Standards? 
248 MODULE 9  BASIC TH. & FIN. REP.: A. BASIC CONCEPTS
rewards   of   ownership   of
goods are transferred. 
a. Financial   Accounting   Standards 2 Payment has been received. 
Board.  3 The entity does not retain either 
b. International   Accounting   Standards a continuing managerial 
Committee.  involvement or control over the 
c. Financial Accounting Committee.  goods. 
d. International   Accounting   Standards 4 The costs incurred can be measured
Board.  reliably. 
• According to the IASB Framework  • Upon  first­time   adoption   of   IFRS,   an  entity
for the Preparationand  may elect to use fair value as deemed cost for 
Presentation of Financial  1 Biological   assets   related   to
Statements, the  agricultural   activity  for   which
fundamentalqualitative  there is no active market. 
characteristic of relevance includes  2 Intangible   assets   for   which
1 Predictive value and feedback value.  there is no active mar­ket. 
2 Verifiability,   neutrality,   and 3 Any   individual   item   of
representational faith­fulness.  property,   plant,   and   equip­
3 Predictive   value   and   confirmatory ment. 
value. 
4 Financial liabilities that are not held
4 Comparability and timeliness. 
for trading. 
• According to the IASB Framework,
• Under IFRS, which of the 
the financial state­ment element that
following is the first step within 
is defined as increases in economic 
the hierarchy of guidance to 
bene­fits during the accounting 
which management re­fers, and 
period in the form of inflows or 
whose applicability it considers, 
enhancements of assets or decreases
when selecting accounting 
of liabilities that result in increases 
policies? 
in equity, other than those relating 
to contribu­tions from equity 
participants, is 
1 Revenue. 
2 Income. 
3 Profits. 
4 Gains. 
• According to the IASB Framework, the two 
criteria required for incorporating items into 
the income statement or statement of financial 
position are that 
1 It   meets   the   definition   of
relevance   and   faithful
representation. 
2 It meets the definition of an
element   and   can   be
measured reliably. 
3 It   satisfies   the   criteria   of   capital
maintenance. 
4 It meets the requirements of
comparability   and
consistency. 
• If   the   outcome   of   rendering
services   cannot   be   esti­mated
®
reliably, IFRS   requires the use
of   which   revenue   recognition
method? 
1 Percentage­of­completion method. 
2 Completed contract method. 
3 Cost recovery method. 
4 Installment method. 
• Which   of   the   following   is   not
one   of   the   criteria   for   revenue
recognition   for   sales   of   goods
under IFRS? 
1 The   significant   risks   and
a. January 1, year 1. 
a. Consider the most recent  b. January 1, year 2. 
pronouncements of other  c. July 1, year 2. 
standard­setting bodies to  d. December 31, year 2. 
the extent they do not 
83. How should a first­time adopter of
conflict with the IFRS or the 
IFRS   recognize   the   adjustments
IASB Framework. 
required   to   present   its   opening
b. Apply   a   standard   from IFRS   statement   of   financial
IFRS   if   it   specifically   re­ position? 
lates   to   the   transaction, a. All of the adjustments
other event, or condition.  should   be   recognized
c. Consider the applicability  in profit or loss. 
of the definitions, rec­ b. Adjustments that are 
ognition criteria, and  capital in nature should be
measurement concepts in  recognized in retained 
the IASB Framework.  earnings and adjustments 
d. Apply   the   requirements   in   IFRS that are revenue in nature 
dealing   with   simi­lar   and   related should be recognized in 
issues.  profit or loss. 
82. On July 1, year 2, a company  c. Current adjustments should be 
recognized in profit or loss and 
decided to adopt IFRS. The 
noncurrent adjustments should be 
company’s first IFRS reporting 
rec­ognized in retained earnings. 
period is as of and for the year 
ended December 31, year 2. The  d. All   of   the   adjustments
company will present one year of  should   be   recognized
comparative information. What is  di­rectly   in   retained
the company’s date of transition to  earnings   or,   if
IFRS?  appropriate,   in   another
category of equity. 
MODULE 9  BASIC TH. & FIN. REP.: A. BASIC CONCEPTS 249

SIMULATIONS

Task­Based Simulation 1
Account
Classifications Authoritative
Literature Help

Suppose Winston incorporated and had the following accounts. Indicate how each of the 
following is classified on the financial statements. Below is a list of classifications.
Balance sheet classification Income statement classifications
A. Current asset H. Revenue
B. Noncurrent asset I. Expense
C. Current liability J. Contra revenue
D. Noncurrent liability 
E. Owner’s equity 
F. Contra asset 
G. Contra equity 

Balance sheet classification Income statement classification
(A) (B) (C) (D) (E) (F) (G) (H)  (I)  (J)
1. Bonds payable, due in 2018
2. Treasury stock
3. Accounts payable
4. Sales discounts
5. Notes payable, due in nine months
6. Inventory
7. Accounts receivable
8. Common stock
9. Cost of goods sold
10. Allowance for uncollectible accounts

Task­Based Simulation 2
Accrual Basis Authoritative
Worksheet Literature Help

The following information pertains to Baron Flowers, a calendar­year sole proprietorship, 
which maintained its books on the cash basis during the year.
Baron Flowers
TRIAL BALANCE
December 31, 2010
Dr. Cr.
Cash $ 25,600
Accounts receivable, 12/31/09 16,200
Inventory, 12/31/09 62,000
Furniture & fixtures 118,200
Land improvements 45,000
Accumulated depreciation, 12/31/09 $  32,400
Accounts payable, 12/31/09 17,000
Baron, Drawings
Baron, Capital, 12/31/09 124,600
Sales 653,000
Purchases 305,100
Salaries 174,000
Payroll taxes 12,400
Insurance 8,700
Rent 34,200
Utilities 12,600
Living expenses 13,000 ______
$827,000 $827,000
250 MODULE 9  BASIC TH. & FIN. REP.: A. BASIC CONCEPTS

Baron has developed plans to expand into the wholesale flower market and is in the process of negotiating a bank loan to
finance the expansion. The bank is requesting 2010 financial statements prepared on the accrual basis of accounting from 
Baron. During the course of a review engagement, Muir, Baron’s accountant, obtained the following additional information.
1. Amounts due from customers totaled $32,000 at December 31, 2010. 
2. An analysis of the above receivables revealed that an allowance for uncollectible accounts of $3,800 should be pro­ 
vided. 
3. Unpaid invoices for flower purchases totaled $30,500 and $17,000, at December 31, 2010, and December 31, 2009, re­
spectively. 
4. The inventory totaled $72,800 based on a physical count of the goods at December 31, 2010. The inventory was
priced at cost, which approximates market value. 
5. On May 1, 2010, Baron paid $8,700 to renew its comprehensive insurance coverage for one year. The premium on the
previous policy, which expired on April 30, 2010, was $7,800. 
6. On January 2, 2010, Baron entered into a twenty­five­year operating lease for the vacant lot adjacent to Baron’s retail 
store for use as a parking lot. As agreed in the lease, Baron paved and fenced in the lot at a cost of $45,000. The 
improvements were completed on April 1, 2010, and have an estimated useful life of fifteen years. No provision for 
depreciation or amortiza­tion has been recorded. Depreciation on furniture and fixtures was $12,000 for 2010. 
7. Accrued expenses at December 31, 2009 and 2010, were as follows: 
2009 2010
Utilities $ 900 $1,500
Payroll taxes 1,100 1,600
$2,000 $3,100
8. Baron is being sued for $400,000. The coverage under the comprehensive insurance policy is limited to $250,000. Baron’s 
attorney believes that an unfavorable outcome is probable and that a reasonable estimate of the settlement is $300,000. 
9. The salaries account includes $4,000 per month paid to the proprietor. Baron also receives $250 per week for living
expenses. 
Using the worksheet below, prepare the adjustments necessary to convert the trial balance of Baron Flowers to the accrual 
basis of accounting for the year ended December 31, 2010. Formal journal entries are not required to support your adjustments.
However, use the numbers given with the additional information to cross­reference the postings in the adjustment columns on 
the worksheet.
Baron Flowers
WORKSHEET TO CONVERT TRIAL BALANCE TO ACCRUAL BASIS
December 31, 2010
Cash basis Adjustments Accrual basis*
*
Account title Dr. Cr. Dr. Cr. Dr. * Cr. 
Cash 25,600
Accounts receivable 16,200
Inventory 62,000
Furniture & fixtures 118,200
Land improvements 45,000
Accumulated depreciation
& amortization 32,400
Accounts payable 17,000
Baron, Drawings
Baron, Capital 124,600

Sales 653,000
Purchases 305,100
Salaries 174,000
Payroll taxes 12,400
Insurance 8,700
Rent 34,200
Utilities 12,600
Living expenses 13,000

827,000 827,000
*Completion of these columns is not required.
MODULE 9  BASIC TH. & FIN. REP.: A. BASIC CONCEPTS 251

Task­Based Simulation 3
Concepts
Authoritative
Literature Help

This question consists of ten items that represent descriptions or definitions of the various elements of the FASB’s
State­ments of Financial Accounting Concepts. Select thebestanswer for each item from the terms listed in A – L. A term
may beused once, more than once, or not at all.
Terms
A. Recognition G. Gains
B. Comprehensive Income H. Net Income
C. Faithful representation I. Earnings
D. Revenues J. Realization
E. Predictive Value K. Replacement Cost
F. Comparability L. Current Market Value

C
o
1. C

2. I
n
3. T
h
4. E
n
5. T
h
6. All changes in net assets of an entity during a period except those resulting from investments by owners and distributions to
owners. 
7. Inflows or other enhancements of assets of an entity or settlements of its liabilities from delivering or producing goods, 
rendering services, or other activities that constitute the entity’s ongoing operations. 
8. The amount of cash, or its equivalent, that could be obtained by selling an asset in orderly liquidation. 
9. The quality of information that helps users to increase the likeli­hood of correctly forecasting the outcome of past or 
present events. 
10. A performance measure concerned primarily with cash­to­cash cycles. 

Task­Based Simulation 4

Trial Balance Authoritative
Worksheet Literature Help

The accounts listed below appeared in the December 31 trial balance of Jane Alexander Theater.
Equipment 192,000
Accumulated depreciation equipment 60,000
Notes payable 90,000
Admissions revenue 380,000
Advertising expense 13,680
Salaries expense 57,600
Interest expense 1,400
Additional information follows:
1. The equipment has an estimated life of 16 years and a salvage value of $40,000 at the end of that time. (Use straight­line
method.) 
252 MODULE 9  BASIC TH. & FIN. REP.: A. BASIC CONCEPTS

2. The note payable is a 90­day note given to the bank October 20 and bearing interest at 10%. (Use 360 days for de­
nominator.) 
3. In December, two thousand (2,000) coupon admission books were sold at $25 each. They could be used for admission
any time after January 1. 
4. Advertising expense paid in advance and included in Advertising Expense, $1,100. 
5. Salaries accrued but unpaid, $4,700. 
Using the attached Excel file, complete the adjustments, adjusted trial balance, and the income statement columns of the
worksheet. For the # column, insert the number of adjustments. Add accounts as needed.
Jane Alexander Theater
December 31, 200X
a) Trial balance # Adjustments Adjusted trial balance Income statement
Debit Credit Debit Credit Debit Credit Debit Credit
Equipment 192,000
Accumulated depreciation—
equipment 60,000
Notes payable 90,000
Admissions revenue 380,000
Advertising expense 13,680
Salaries expense 57,600
Interest expense 1,400
Depreciation expense
Interest payable
Unearned admissions revenue
Revenue
Prepaid advertising
Salaries payable

Task­Based Simulation 5
Concepts
Authoritative
Literature Help

SFAC 5 provides guidance on measuring assets and liabilities. The following measurement methods are available to
measure assets and liabilities, as shown in the list below. Identify the appropriate valuation method for each item.

1. L

2. A

3. E

4. W

5. S

6. A

7. B

8. T
MODULE 9  BASIC TH. & FIN. REP.: A. BASIC CONCEPTS 253

Task­Based Simulation 6
Adjusting Authoritative
Entries Literature Help

Indicate whether each of the following adjusting entries is an accrual or a deferral type entry.
Accrual Deferral
1. Depreciation expense xx
Accumulated deprecation xx
2. Interest receivable xx
Interest revenue xx
3. Rent expense xx
Prepaid rent xx
4. Unearned revenue xx
Rent revenue xx
5. Wage expense xx
Wages payable xx

Task­Based Simulation 7
Revenue and Expense Authoritative
Recognition Literature Help

Emco has the following transactions in 2010. Indicate the amount of revenue or expense recognized in 2010, 2011, and 
2012 for each of these items. You may show expense recognitions in parentheses (xx).
Amounts to be recognized 2010 
2011 2012
1. Emco sells $5,000 of goods to a customer, FOB shipping point on 12/30/10. 
2. Emco sells three pieces of equipment on a contract over a three­year period.
The sales price of each piece of equipment is $10,000. Delivery of each 
piece of equipment is on February 10 of each year. In 2010, the customer 
paid a $20,000 down payment, and paid $5,000 per year in 2011 and 2012. 
Collectibility is reasonably assured. 
3. In 1/1/10, Emco pays $9,000 for a membership to Wholesalers Association
for a two­year membership in the trade association. 
4. On 6/1/10, Emco signs a contract for $20,000 for goods to be sold on 
account. Payment is to be made in two installments of $10,000 each on 
12/1/11 and 12/1/12. The goods are delivered on 10/1/10. Collection is 
reasonably assured, and the goods may not be returned. 
5. Emco sells goods to a customer on July 1, 2010, for $50,000. If the customer 
does not sell the goods to retail customers by December 31, 2011, the goods 
can be returned to Emco. The customer sells the goods to retail customers on 
October 1, 2011. 
254 MODULE 9 BASIC TH. & FIN. REP.: A. BASIC CONCEPTS

*
MULTIPLE­CHOICE ANSWERS
1. c __ __ 19. d __ __ 37. c __ __ 55. d __ __ 73. c __ __
2. d __ __ 20. d __ __ 38. b __ __ 56. d __ __ 74. d __ __
3. b __ __ 21. b __ __ 39. b __ __ 57. c __ __ 75. c __ __
4. c __ __ 22. c __ __ 40. b __ __ 58. a __ __ 76. b __ __
5. b __ __ 23. a __ __ 41. b __ __ 59. a __ __ 77. b __ __
6. a __ __ 24. c __ __ 42. b __ __ 60. c __ __ 78. c __ __
7. d __ __ 25. d __ __ 43. a __ __ 61. c __ __ 79. b __ __
8. a __ __ 26. b __ __ 44. c __ __ 62. c __ __ 80. c __ __
9. c __ __ 27. c __ __ 45. d __ __ 63. d __ __ 81. b __ __
10. d __ __ 28. a __ __ 46. d __ __ 64. d __ __ 82. a __ __
11. b __ __ 29. b __ __ 47. d __ __ 65. d __ __ 83. d __ __
12. d __ __ 30. c __ __ 48. a __ __ 66. a __ __
13. a __ __ 31. b __ __ 49. d __ __ 67. d __ __
14. c __ __ 32. b __ __ 50. c __ __ 68. c __ __
15. d __ __ 33. c __ __ 51. d __ __ 69. d __ __
16. d __ __ 34. d __ __ 52. b __ __ 70. c __ __
17. d __ __ 35. a __ __ 53. c __ __ 71. a __ __ 1st: __/83 = __%
18. c __ __ 36. d __ __ 54. b __ __ 72. b __ __ 2nd: __/83 = __%

MULTIPLE­CHOICE ANSWER EXPLANATIONS
components of faithful representation include information to
A.1.  Basic Accounting Theory be verifiable and free from error. Neutrality is not an ingre­
1. (c) The Statements of Financial Accounting Con­ dient of relevance because relevance requires information to
cepts (SFAC) were issued to establish a  have predictive value and confirmatory value, or both.
framework from which financial accounting and  6. (a) Per SFAC 8, comparability is an enhancing 
reporting standards could be developed. The  qual­ity of financial reporting which relates to both
SFAC provide the theory behind accounting and  relevance and faithful representation. 
reporting and provide guidance when no GAAP  Confirmatory value and predictive value only 
exists. The SFAC are not included as GAAP.  relate to relevance. Freedom from error only re­
2. (d) Per SFAC 8, the objectives of financial  lates to faithful representation. 
reporting focus on providing present and potential  7. (d) Per SFAC 5, recognition is the process of for­
investors and credi­tors with information useful in  mally recording or incorporating an item into the 
making investment decisions. Financial statement  financial statements as an asset, liability, revenue, 
users do not have the authority to pre­scribe the 
expense, or the like. According to SFAC 6, allocation
data they desire. Therefore, they must rely on ex­
is the process of assigning or distributing an amount 
ternal financial reporting to satisfy their 
according to a plan or formula, matching is the 
information needs, and the objectives must be 
simultaneous recognition of revenues with expenses 
based on the needs of those users. 
that are related directly or jointly to the same trans­
3. (b) The FASB conceptual framework has identified the cost­ actions or events, and realization is the process of 
benefit constraint to the relevance of providing fi­nancial  converting noncash resources and rights into money. 
reports. Information is not disclosed if the costs of 
8. (a) Per SFAC 5, earnings and comprehensive income have the 
disclosure outweigh the benefits of providing the informa­
tion. Comparability is an enhancing qualitative characteris­ same broad components—revenues, expenses, gains, and losses
tic. Reliability is no longer part of the conceptual framework —but are not the same because certain classes of gains and 
according to SFAC 8. Faithful representation is a fundamen­ losses are excluded from earnings. Changes in market values of 
tal qualitative characteristic.  investments in marketable equity securities classified as 
available­for­sale securities are included in com­prehensive 
4. (c) The enhancing qualitative characteristics of fi­nancial  income, but are excluded from earnings until realized. Answers 
reporting are comparability (including consistency),  (b), (c), and (d) are incorrect because they would be included in 
verifiability, timeliness, and understandability. Answer (a) is both earnings and comprehensive in­come. Note that unrealized 
incorrect because relevance and faithful representation are  gains and losses on marketable equity securities classified as 
fundamental qualitative characteristics of financial informa­ trading securities are included in earnings. This treatment is in 
tion. Reliability is no longer listed as a fundamental quality.  accordance with SFAS 115. 
Answer (b) is incorrect because cost­benefit is a constraint, 
and materiality is a threshold for reporting useful informa­ 9. (c) Per SFAC 6, comprehensive income includes 
tion. Answer (d) is incorrect because completeness, neutral­ all changes in equity during a period except those 
ity, and freedom from error are characteristics of faithful  resulting from investments by owners and 
representation, a fundamental qualitative characteristic.  distributions to owners. Divi­dends paid to 
stockholders is a change in equity resulting from a 
5. (b) SFAC 8 defines neutrality as the quality of  distribution to owners, so it is excluded from com­
infor­mation which requires freedom from bias  prehensive income. Answers (a), (b), and (d) are 
toward a prede­termined result. Unbiased  all in­cluded in comprehensive income because 
information would always be more faithfully  they are changes 
represented than biased information. Other 

*
 Explanation of how to use this performance record appears on page 2.
MODULE 9  BASIC TH. & FIN. REP. :  A. BASIC CONCEPTS 255
because understandability is an enhanc­ing 
characteristic, predictive value is a component of 
in equity, but are not investments by, or distributions  relev­ance, and reliability is no longer a 
to, owners. characteristic in the concept statements. 
10. (d) The fundamental qualitative characteristic of  11. (b) According to SFAC 6, realization is the process
faithful representation has the components of  of converting noncash resources and rights into 
completeness, neutrality, and freedom from error.  money through the sale of assets for cash or claims
Answer (a) is incorrect because predictive value  to cash. When equipment is sold for a note 
and confirmatory value are the components of  receivable, money is realized since a note qualifies 
relevance. Answer (b) is incorrect because  as a claim to cash. Answers (a) and (d) relate to 
comparability and consistency are enhancing  cost allocation. Answer (c) is incorrect because 
characteristics, and confirmatory value is a  accounts receivable represents a claim to cash. 
component of relevance. An­swer (c) is incorrect,  Realization occurs at the time of sale rather than 
when cash is collected. 
12. (d) A loss contingency is accrued by a charge to expense if it of an entity’s major or primary operations. Two essential 
is reasonably estimable and it is probable thata liability  characteristics of revenues are that revenues (1) arise from a 
has been incurred as of the balance sheet date.The cause of company’s primary earnings activities and (2) are recurring 
the liability must have occurred on or beforethe date of  or continuing in nature. Therefore, answer (d) is correct 
the financial statements if the liability is to beaccrued,  because it meets the above criteria. Answers (b) and (c) are 
because the intention is to recognize the loss in the same  incorrect because they result from incidental transactions. 
period that the underlying cause took place. Therefore, some Answer (a) is incorrect because a decrease of an asset is not 
authorities would say that matching is the underlying  a revenue.
concept supporting the immediate recognition of a contin­ 16. (d) Per SFAC 6, the common quality shared by 
gent loss. A larger number, however, would mention con­ all assets is “service potential” or “future 
servatism as the more pervasive underlying concept sup­ economic benefit.” Per SFAC 6, assets 
porting the immediate recognition of these losses.  commonly have other distinguishing features, 
13. (a) Conservatism means that when in doubt, ac­ such as being legally enforceable, tangible or ac­
countants should choose the procedure that will be  quired at a cost. These features, however, are not
least likely to overstate assets and income. This  essential characteristics of assets. 
concept underlies the GAAP pertaining to  17. (d) Per SFAC 5, five different attributes are used to measure 
recording gain contingencies. In an effort to not  assets and liabilities in present practice: historical cost, 
overstate assets or income, gain contingencies are  current (replacement) cost, current market value, net 
not recorded until they are no longer  realizable value, and present value of future cash flows. 
contingencies, and are disclosed only when  Three of these (historical cost, replacement cost, and net 
probabilities are high that a gain contingency will  realizable value) are used in measuring inventory at lower of
become reality.  cost or market. Present value of future cash flows is not used
14. (c) Per SFAC 6, the major difference between  to measure inventory. 
finan­cial and physical capital maintenance is  Cash Flow Information and Present Value (SFAC 7)
related to the effects of price changes on assets 
held and liabilities owed during a period. The  18. (c) The most relevant measure of a liability 
financial capital concept is applied in current  always reflects the credit standing of the entity 
GAAP. Under this concept, the effects of the price  obligated to pay, according to SFAC 7. Those 
changes described above are considered “holding  who hold the entity’s obliga­tions as assets 
gains and losses,” and are included in computing  incorporate the entity’s credit standing in de­
return on capital. Compre­hensive income, which  termining the prices they are willing to pay. 
is described in SFAC 5, is “the change in equity of 
a business enterprise during a period from  19. (d) SFAC 7 provides a framework for using future 
transactions and other events and circumstances  cash flows as the basis for accounting measurements 
from nonowner sources.” It is also a measure of  at ini­tial recognition or fresh­start measurements and 
return on finan­cial capital. The concept of  for the inter­ 
physical capital maintenanceseeks to measure the  est method of amortization. FASB limited SFAC 7 
effects of price changes that are not currently  tomeasurement issues (how to measure) and chose not to 
captured under GAAP (e.g., replacement costs of  address recognition questions (when to measure). SFAC 
nonmonetary assets). Under this concept, holding  7introduces the expected cash flow approach, which differs 
gains and losses are considered “capital  from the traditional approach by focusing on explicit as­
maintenance adjustments” which would be  sumptions about the range of possible estimated cash flows 
included directly in equity and excluded from  and their respective probabilities.
return on capital. 
20. (d) The expected cash flow approach uses all ex­
15. (d) Per SFAC 6, revenues are inflows of assets or pectations about possible cash flows in developing
settlements of liabilities, or both, during a period  a mea­surement, rather than just the single most­
as a result  likely cash flow. By incorporating a range of 
possible outcomes (with their respective timing 
differences), the expected cash flow ap­proach 
accommodates the use of present value techniques
when the timing of cash flows is uncertain. Thus, 
the ex­pected cash flow is likely to provide a 
better estimate of fair value than the minimum, 
most­likely, or maximum taken alone. According 
to SFAC 7, expected present value refers to the 
sum of probability­weighted present values in a 
range of estimated cash flows, all discounted 
using the same inter­est rate convention. 
21. (b)   The   computation   of   expected   present   value
using a single interest rate is as follows: 
PV of $200,000 in one year at 5% $190,476
Probability 20% $38,095
PV of $200,000 in two years at 5% $181,406
Probability 50% 90,703
PV of $200,000 in three years at 5% $172,768
Probability 30% 51,830
$180,628
256 MODULE 9  BASIC TH. & FIN. REP.: A. BASIC CONCEPTS
Interpretations, AICPA Statements 
of Position, AICPA Audit and 
According to SFAC 7, expected present  Accounting Guides, and Practice 
value refers to the sum of probability­ Bulletins. The FASB Accounting 
weighted present values in a range of  Stan­dards Codification does not 
estimated cash flows, all discounted using  include the AICPA Statements of 
the same interest rate convention. Auditing Standards. The auditing 
standards are included in the 
22. (c) Like depreciation and 
Professional Standards issued by the 
amortization conventions, interest  AICPA. 
methods are grounded in notions of 
historical cost, not current cost.  A.2.  Income Determination
23. (a) According to SFAC 7, the  26. (b) Generally, sales revenue is recognized at 
objective of using present value in an the date of delivery, because that generally is 
accounting measurement is to  the time at which a sale has occurred. At that 
capture, to the extent possible, the  point the two criteria for revenue rec­ognition 
economic difference between sets of  were met; the revenue is (1) realized or 
future cash flows. The objective of  realizable and (2) it is earned (SFAC 6). 
present value, when used in  Therefore, the amount of sales revenue 
accounting measurements at initial  recognized in 2010 is $150,000 (50,000 × $3 =
recognition and fresh­start  $150,000). 
measurements, is to estimate fair 
value. Stated differ­ently, present  27. (c) Income generally accrues only at the time 
value should attempt to capture the  of sale, and gains may not be anticipated by 
elements that taken together would  reflecting assets at their current sales prices. 
comprise a market price, if one  Exception to this general rule isgranted, 
existed, that is fair value. Value­in­ however, for agricultural products that are 
use and entity­specific measurements homo­genous and have an immediate 
attempt to capture the value of an  marketability at quoted prices such as the 
asset or liability in the context of a  cotton in this problem (ASC. 905­330­30­1). 
particular entity. An entity­specific  When these inventories are stated at sales 
measurement substitutes the entity’s  prices, they should be reduced by expenditures
assumptions for those that  to be incurred in disposal. Amar Farms should,
marketplace participants would  therefore, recognize revenue on the entire 
make. 
24. (c) The Codification provides 
guidance on the determination of 
gain or loss on disposal of a 
component of a business. According 
to this guidance, such determination 
should be based on estimates of the 
net realizable value of the 
component. Since Brooks Co. plans 
to discontinue its entire operations, 
the appropriate measurement basis 
for its equipment is net realizable 
value. Historical cost and current 
reproduction and replacement costs 
are not appropriate measurement 
bases for assets once an entity has 
decided to discontinue its operations 
because these amounts do not reflect 
the entity’s probable future benefit, 
which is a characteristic of assets per
SFAC 6. 
25. (d) The FASB Accounting Standards
Codification includes all previous 
level A–D GAAP. The Codification 
includes the authoritative literature 
of the Financial Account­ing 
Standards Board, the Emerging 
Issues Task Force Ab­stracts, 
Accounting Principles Board 
Opinions, Accounting Research 
Bulletins, Accounting 
even though they were not billed to 
300,000 pound crop in 2010 at the guaranteed Clark or paid until 2011. The $9,000
(and prevail­ing) market price of $.70 per  must be accrued as an expense and a
pound. This amounts to $210,000 (300,000  liability at 12/31/10. Therefore, 2010
pounds × $.70 per pound = $210,000). Note  advertising expense should total 
that the additional $.02 per pound for the  $140,000 ($146,000 – $15,000 + 
cotton sold in 2011 would be recognized in  $9,000). 
2011, since its selling price exceeded the 
current (2010) market price. 31. (b)       The   opening   balance   in   prepaid
expenses 
28. (a) Revenue from the sale of a product ($1,500) results from a one­year insurance 
may berecognized at the time of sale only  premium paid on 7/1/09. Since this policy 
if all of the following conditions are met: would have expired by 6/30/10, no part of the 
1. The   seller’s   price   is   fixed   or   readily $1,500 is included in 12/31/10 prepaid 
determinable.  expenses. The insurance premium paid on 
2. The   buyer   has   paid   the   seller   or   is 7/1/10 ($3,200) would be partially expired 
obligated   to   pay   the   seller,   the (6/12) by 12/31/10. The remainder (6/12 × 
obligation   not   being   contingent   on $3,200 = $1,600) would be a prepaid expense 
resale of the product.  at year­end. The entire advance rental payment
3. The buyer’s obligation to the seller  ($2,000) is a prepaid ex­pense at 12/31/10 
remains un­changed in the event of  because it applies to 2011. Therefore, total 
damage or destruction of the  12/31/10 prepaid expenses are $3,600.
product. 
4. The   buyer   is   independent   from   the
seller. 
5. The   seller   does   not   have   any
significant   obligations   regarding
resale of the product by the buyer. 
6. The amount of future returns can be
reasonably esti­mated. 
Because the buyer, Zee, has the right to 
return the machine to the seller, Lin, 
condition (2) above has not been met. 
Therefore, the recognition of sales revenue 
and cost of sales is not allowable for this 
transaction.
A.3.  Accruals and Deferrals
29. (b) Under accrual accounting, events
that change an entity’s financial 
position are recorded in the period in 
which the events occur. This means 
revenues are recognized when earned
rather than when cash is received, 
and expenses are recognized when 
incurred rather than when cash is 
paid. Therefore, when the royalties 
are paid, Wand should debit an asset 
account (prepaid royalties) rather 
than an expense ac­count. The 
royalties paid should be reported as 
expense in the period incurred (by 
debiting royalty expense and credit­
ing prepaid royalties). 
30. (c) The balance in the advertising 
expense account on 12/31/10 before 
adjustment is $146,000. Since the 
sales promotional campaign is to be 
conducted in January, any associated
costs are an expense of 2011. Thus, 
the $15,000 cost of printing catalogs 
should be removed from the adver­
tising expense account and recorded 
as a prepaid expense as of 12/31/10. 
In addition, advertising expense 
must be in­creased by the $9,000 
cost of December’s radio advertise­
ments, which are an expense of 2010
Prepaid insurance ($3,200 × 6/12) $1,600 revenue because this amount 
Prepaid rent 2,000 pertains to revenues earned for July 
Total prepaid expenses $3,600
32. (b) Apparently Roro records policy  through December of 2009 and 
MODULE 9  BASIC TH. & FIN. REP. :  A. BASIC CONCEPTS
payments as charges to insurance  would have been accrued as revenue 257
expense and records prepaid  on 12/31/09. On 9/15/10, Wren 
insurance at the end of the quarter  received $17,000 in royalties for the
through an adjusting entry. The  first half of 2010. Royalties for the 
unadjusted trial balance amounts at  second half of 2010 will not be 
3/31/10 must represent the final two  received until 3/15/11. However, the
months of the old policy ($300 of  royalty payment to be received for 
prepaid in­surance) and the cost of  the second six months (15% × 
the new policy ($7,200 of insurance  $60,000 = $9,000) has been earned 
expense). An adjusting entry must  and should be accrued at 12/31/10. 
be prepared to reflect the correct  Therefore, 2010 royalty revenue is 
3/31/10 balances. Since the new  $26,000 ($17,000 + $9,000). 
policy has been in force one month 
(3/1 through 3/31), thirty­five  36. (d) The agreement states that Super is to 
months re­main unexpired.  receive royalties of 20% of revenues 
Therefore, the balance in prepaid  associated with the comic strip. Since 
insurance should be $7,000 ($7,200  Fantasy’s 2011 revenues from the strip were 
× 35/36). Insurance expense should  $1,500,000, Super’s royalty revenue is 
include the cost of the last two  $300,000 ($1,500,000 × 20%). The other 
months of the old policy and the first information in the problem about the 
month of the new policy [$300 +  receivable and cash payments is not needed to 
($7,200 × 1/36) = $500]. Roro’s  compute revenues. Super’s 2011 summary 
adjusting entry would transfer  entries would be 
$6,700 from insurance expense to  Cash 200,000
prepaid insurance to result in the  Royalties rec. 75,000
Royalty revenue 125,000 ($200,000 – $75,000)
correct balances.  Royalties rec. 175,000
Royalty revenue 175,000 ($300,000 – $125,000)
33. (c) At 12/31/10, the liability account
unearned sub­scription revenue  37. (c) Royalty revenues should be 
should have a balance which reflects recognized whenearned, regardless of 
all unexpired subscriptions. Of the  when the cash is collected. Royalty
2009 sales, $125,000 ex­pires 
during 2011 and would still be a 
liability at 12/31/10. Of the 2010 
sales, $340,000 ($200,000 + 
$140,000) expires during 2011 and 
2012, and therefore is a liability at 
12/31/10. Therefore, the total 
liability is $465,000 ($125,000 + 
$340,000). This amount would have
to be re­moved from the sales 
account and recorded as a liability in
a 12/31/10 adjusting entry. 
34. (d) Regal’s unredeemed gift 
certificates at 12/31/09 are $75,000. 
During 2010, these certificates are 
either re­deemed ($25,000) or expire
by 12/31/10 ($75,000 – $25,000 = 
$50,000). Therefore, none of the 
$75,000 affects the 12/31/10 
unearned revenue amount. During 
2010, additional certificates totaling 
$250,000 were sold. Of this amount, 
$225,000 is expected to be redeemed
in the future [$250,000 
– (10% × $250,000)]. Since $175,000 of 
2010 certificates were redeemed in 2010, 
12/31/10 unearned revenue is $50,000 
($225,000 – $175,000). 
35. (a) The requirement is to calculate 
Wren’s royalty revenue for 2010. 
The 3/15/10 royalty receipt 
($12,000) would not affect 2010 
($40,000) is subtracted since this amount was 
revenue earned from 12/1/09 to 5/31/10 is  collected, but not earned as revenue, by 
$80,000 ($400,000 × 20%). Of this amount,  12/31/10.
$10,000 ($50,000 × 20%) was earned in  39. (b) The requirement is to determine the 
December of 2009, so the portion earned in 
amount ofroyalty expense to be recognized 
the first five months of 2010 is $70,000 
in 2010. Cash paid for royalties totaled 
($80,000 – $10,000). Royalty revenue earned 
from 6/1/10 to 11/30/10 is $65,000 ($325,000  $300,000 in 2010. However, this amount 
× 20%). The amount earned from 12/1/10 to  must be adjusted for changes in the related 
12/31/10, which would be accrued at 12/31, is  accounts, as fol­lows:
$14,000 ($70,000 × 20%). Therefore, 2010  2010 cash paid $300,000
royalty revenue is $149,000. Royalties payable 12/31/09 (80,000)
Royalties payable 12/31/10 75,000
1/1/10 ­ 5/31/10 $70,000 Prepaid royalties 12/31/09 55,000
6/1/10 ­ 11/30/10 65,000 Prepaid royalties 12/31/10 (45,000)
12/1/10 ­ 12/31/10 14,000 $305,000
$149,000
The beginning payable balance ($80,000) is 
38. (b) The requirement is to calculate the  subtracted be­cause that portion of the cash 
amount ofroyalty income to be recognized in  paid was recognized as ex­pense during the 
2010. Cash collected for royalties totaled  previous year. The ending payable balance 
$200,000 in 2010. However, this amount  ($75,000) is added because that amount has 
must be adjusted for changes in the related  been accrued as 2010 expense, even though it 
accounts, as fol­lows: has not yet been paid. The beginning balance 
2010 cash received of prepaid royalties ($55,000) is added 
$200,000
Royalties receivable 12/31/09 because that amount is assumed to have 
(90,000)
Royalties receivable 12/31/10 expired during the year. Finally, the ending 
Unearned royalties 12/31/09 balance of prepaid royalties ($45,000) is 
Unearned royalties 12/31/10 (40,000
subtracted since this amount was paid, but not
Royalty income $215,000
incurred as an expense, by 12/31/10.
The beginning receivable balance ($90,000) is  40. (b) When the insurance policy was 
subtracted because that portion of the cash 
initially pur­chased, the entire balance was 
collected was recognized as revenue last year. 
debited to a prepaid asset ac­count (i.e., 
The ending receivable balance ($85,000) is 
prepaid insurance). The adjusting entry at 
added because that amount is 2010 revenue, 
even though it has not yet been collected. The  De­cember 31, 2010, to recognize the 
beginning balance of un­earned royalties  expiration of one year of the policy would be
($60,000) is added because that amount is  Insurance expense (1/3 of original pymt.)
assumed to be earned during the year. Finally,  Prepaid insurance (1/3 of original 
the ending balance of unearned royalties  pymt.)
258 MODULE 9  BASIC TH. & FIN. REP.: A. BASIC CONCEPTS

Service Revenue Acct . Receivable


After the adjusting entry, the prepaid asset 
1/1/ 10 40,000
account would contain 2/3 of the original  220,000 200,000 Cash
payment. If the original payment had instead  5,000 220,000 received
been debited to an expense account (i.e., 
insur­ance expense), then the adjusting entry  215,000 12/ 31/ 10 60,000
at December 31, 2010 would be
Prepaid insurance (2/3 of original pymt.) Unearned Revenue
Insurance expense (2/3 of original  0 1/1/ 10
pymt.)
5,000
This alternate approach would also result in 
1/3 of the origi­nal payment being expensed in 5,000 12/ 31/ 10
2010 and 2/3 of the original payment being 
carried forward as a prepaid asset. Thus,  45.  (d) To determine cash basis revenue, the
answer (b) is correct. Answer (a) is incorrect 
solutionsapproach   is   to   prepare   a   T­account
because the premium paid was for a three­year
policy, 2/3 of which had not yet expired and  for accounts receivable.
would therefore be carried forward in the  Accounts Receivable
prepaid asset account. Answer (c) is incorrect  12/31/09 1,000,000
because 1/3 of the original payment was  Sales 4,600,000 20,000 Write­offs
? Collections
already expensed. Answer (d) is incorrect 
12/31/10 1,300,000
because the amount would be the same as it 
would have been if the original payment had  The missing amount for cash collections is 
been debited initially to an expense account  $4,280,000. An­other approach is to use the 
(as explained for answer (b) above). following formula:
Sales + Decrease (– increase) in AR – Write­offs = Collections
41. (b) Current assets are identified as  $4,600,000 – $300,000 – $20,000   = $4,280,000
resources that are reasonably 
expected to be realized in cash or  The increase in receivables ($300,000) means
sold or con­sumed during the  that cash col­lected during the period was less
than sales during the period and therefore is 
normal operating cycle of the 
deducted from sales revenue. The write­offs 
business.These resources include  ($20,000) represent recognized sales that will 
prepaid expenses such as royalties.  never be col­lected in cash and therefore must
Since the balance remaining in Sip  also be deducted to com­pute collections.
Co.’s royalty prepayment (the 
payment relating to 2011 royalties)  46. (d) To determine rental revenue, the 
will be consumed within the next  solutions ap­proach is to prepare a T­account 
year, it should be reported as a  for rents receivable and rent revenues.
current asset. Additionally, the  Rent Receivable Rent Revenues
payment relating to 2010 should be  a a
11/30/09 800,000 800,000 800,000 2,210,000
re­ported as an expense.  b c b
1,090,000 30,000 1,090,000
42. (b) At the time the gift certificates
were   issued,   the   following   entry Bal 1,060,000 1,060,000 + 30,000 Bal 2,500,000
was   made,   reflecting   the   store’s a
 To remove 2009 revenue from the $2,210,000 of cash 
future   obli­gation   to   honor   the collected and relieve rent receivable of collections 
from 2009 receivables.
certificates:  b
  To recognize unrecorded rent earned including write­offs.
c
Cash xx   To recognize write­offs.  Debit would be to the 
Deferred revenue xx allowance account.

Upon redemption of the certificates, the 
obligation recorded in the deferred revenue 
account becomes satisfied and the revenue is 
earned. Similarly, as the certificates expire, 
the store is no longer under any obligation to 
honor the certifi­cates and the deferred 
revenue should be taken into income. In both 
instances, the deferred revenue account must 
be re­duced (debited) to reflect the earning of
revenue. This is done through the following 
entry:
Deferred revenue xx
Revenue xx
43. (a) The revenues from service contracts  47. (d) Under the cash basis method of 
should berecognized on a pro rata basis over the  accounting, reve­nue is recognized as it is 
term of the contract. This treatment allocates the  collected. Cash sales after returns and 
contract revenues to the pe­riod(s) in which they  allowances totaled $76,000 ($80,000 – 
are earned. Since the sale of a service contract  $4,000). Net credit sales for 2010 were 
does not culminate in the completion of the  $114,000 ($120,000 credit sales – $6,000 
earnings process (i.e., does not represent the  discounts). As made evident by the following 
seller’s performance of the contract), payments  T­account, cash collections from credit sales 
received for such a contract should be recorded  must equal $124,000 ($40,000 + $114,000 – 
initially in a deferred revenue account. $30,000), since $10,000 in excess of current 
credit sales was received and reduced 
A.4.  Cash to Accrual Accounts Receivable by this amount.
44. (c) The following formula is used to  Accounts Receivable
adjust servicerevenue from the cash basis to  Bal. 1/1 40,000
Net credit sales 114,000 ? Collections
the accrual basis:
Bal. 12/31 30,000
Beg. End.
Cash fees End. Beg.
collected
+
AR

AR
+   unearned   – unearned   = A summary journal entry would be
fees fees

$200,000  +  $60,000  –  $40,000  +  0  –  $5,000  
=  $215,000
As an alternative, T­accounts can be used.
Cash 124,000
March 31). Therefore, cash basis income is 
Sales discounts 6,000
$5,000. Drawings are $1,000. Therefore, 
Accounts receivable 130,000
Total cash basis revenue for 2010 is  ending capital is $6,000 ($2,000 + $0 + $5,000
MODULE 9  BASIC TH. & FIN. REP. :  A. BASIC CONCEPTS
$200,000 as shown be­low. – $1,000). 259
Cash sales ($80,000 – $4,000)
51. (d) The requirement of this question is to
Collections of credit sales [($40,000 + $120,000 –
$30,000) – $6,000] determineif a decrease in accounts receivable
Total cash received and/or a decrease in ac­crued expenses 
would result in cash­basis income being 
48. (a)    Cost of goods sold is computed as
lower than accrual­basis income.
follows: 
A decrease in the accounts receivable balance
Beg. inv. + Net purchases – End. inv. =  would gen­erally mean that cash was collected. 
CGS
In accordance with the cash basis of accounting 
Net purchases must be computed from the  when cash is received it is recorded as revenue 
information given using a T­account or a  (Dr. Cash, Cr. Revenue), whereas under the ac­
formula. crual basis the revenue would have been 
AP recorded when the
50,000 12/31/09
Payments 490,000 ? Purchases
75,000 12/31/10
The missing amount for purchases is 
$515,000. Another approach is to use the 
following formula:
Payments + Ending AP – Beginning AP = Purchases
$490,000 + $75,000 –$50,000= $515,000
This amount can be used to determine cost of 
goods sold
Beg. inventory $290,000
+ Net purchases 515,000
Cost of goods available 805,000
– End. inventory 260,000
Cost of goods sold $545,000

49.  (d) Cash­basis income of $60,000 must
be   adjustedfor   changes   in   accounts
receivable and accounts payable to compute
accrual income
Cash­basis income $ 60,000
12/31/09 AR (20,000)
12/31/10 AR
12/31/09 AP
12/31/10 AP (15,000
Accrual income $ 

The beginning AR ($20,000) is subtracted 
because although this amount was collected 
in 2010, it is properly accrued as 2009 
revenue. The ending AR is added because 
although not collected in 2010, it should be 
accrued as 2010 revenue. The beginning AP 
is added because although this amount was 
paid in 2010, it is properly accrued as 2009 
expense. The ending AP is subtracted 
because although not paid in 2010, it should 
be accrued as 2010 expense.
50. (c) The ending balance in Tory’s capital 
account oneither the accrual or cash basis is 
computed as follows:
Beginning
+ Investments   +  Income   –   Drawings  =
capital

Tory’s beginning capital is his initial cash 
investment of $2,000. No other investments 
were made. Under the cash basis method of 
accounting, income is the excess of cash 
revenues ($5,000) over cash expenses ($0, 
since the expenses were not paid until after 
unused supplies at each year­end 
receivable was recorded (Dr. AR, Cr.  would have been carried as an asset 
Revenue). Thus, a de­creased accounts  and, therefore, excluded from the 
receivable balance would result in increased  current year’s supplies expense. In 
revenue/income. Therefore, the answer for this 2010, the year Droit adopted the 
ac­count is No. accrual basis of accounting, Droit 
A decrease in the accrued expenses account would have inventoried unused 
would gener­ally mean cash was paid on some supplies at December 31 and 
expenses. Under the cash basis, when the cash is excluded those costs from 2010 net 
paid the expense is recorded (Dr. income. Since the cost of 2010’s 
Expense, Cr. Cash), whereas under the  beginning balance of supplies was 
accrual basis the expense would have been  expensed dur­ing 2009, even though 
recorded when the accrued ex­pense was  the supplies were not used until 
recorded (Dr. Expense, Cr. Accrued  2010, Droit’s inability to determine 
Expense). Thus, a decreased accrued  the beginning supplies inven­tory 
expenses account would result in increased  would result in an understatement of 
expenses/lower income for the cash basis.  supplies expense and overstatement 
Thus, the answer for this account is Yes. of 2010 net income. However, since 
Droit properly inventoried supplies 
52. (b) When a company operates on the at December 31, 2010, its 
accrual basis, supplies are  cumulative (inception­to­date) 
inventoried and expensed as they are supplies expense would be properly 
used. Un­der the cash method,  stated. Therefore, Droit’s inability to
however, supplies are expensed as  determine 2010’s beginning supplies
they are paid for. Therefore if White  expense would have no impact on 
Co. experiences an increase in  Droit’s December 31, 2010 retained 
supplies inventory during the year,  earnings. 
this increase must be deducted from 
accrual income to get to the cash  A.5.  Installment Sales
basis, be­cause the cost of the 
supplies would be expensed at the  54. (b) Under the installment method, gross 
time of purchase.  profit isdeferred at the time of sale and is 
Office salaries payable works the opposite recognized by applying the gross profit rate to
way. Under the accrual method, a liability  subsequent cash collections. At the time of 
would have been established resulting in  sale, gross profit of $250,000 is deferred 
additional expense over the amount of cash  ($500,000 in­stallment sales less $250,000 
paid to employees. Under the cash method, no  cost of installment sales). The gross profit rate
liability is accrued and the unpaid salaries are  is 50% ($250,000 ÷ $500,000). Since 2010 
not expensed. Therefore, the increase must be  collections on installment sales were 
added to the accrual basis net income.  $100,000, gross profit of $50,000 (50% × 
$100,000) is recognized in 2010. This would 
53. (c) Prior to 2010, Droit Co. used the  decrease the deferred gross profit account to a 
cash basis of accounting.  12/31/10 balance of $200,000 ($250,000 – 
Accordingly, Droit would have  $50,000). Note that regular sales, cost of 
expensed all purchases of supplies as regular sales, and general and ad­ministrative 
incurred. In contrast, under the ac­ expenses do not affect the deferred gross 
crual basis of accounting, the cost of  profit account.
260 MODULE 9  BASIC TH. & FIN. REP.: A. BASIC CONCEPTS
recognized is $250,000 ($100,000 
gross profit+ $150,000 interest 
55. (d) Under the installment sales method,  revenue). 
gross profitis deferred to future periods and 
recognized proportionately to collection of the 59. (a) The machine sale is accounted 
receivables. Therefore, at each year­end,  for using the in­stallment method, 
deferred gross profit can be computed by  where gross profit is deferred and 
multiplying the gross profit percentage by the  recog­nized in proportion to cash 
accounts receivable balance, as indicated  collected. Initially, the entire 
below. $270,000 gain is deferred. In 2010, 
$150,000 of the sales price was 
2010
collected, and the gross profit 
Sales $900,000
2009 Collections percentage is 30% ($270,000 ÷ 
2010 Collections (300,000) $900,000), so gross profit recognized
2009 Write­offs was $45,000 (30% × $150,000). In 
2010 Write­offs (50,000) 2011, $250,000 of the sales price 
12/31/10 AR $550,000 $100,000 was collected, so gross profit 
Gross profit % ×40%
12/31/10 Deferred GP $220,000 $ recognized was $75,000 (30% × 
$250,000). Therefore, at 12/31/11, 
Thus, total deferred gross profit at  deferred gross profit is $150,000 
12/31/10 is $250,000 ($220,000 +  ($270,000 – $45,000 – $75,000). As 
$30,000). a shortcut, you can compute the 
56. (d) Under the installment sales method,  12/31/11 note receivable bal­ance 
gross profitis deferred to future periods and is  ($750,000 – $250,000 = $500,000), 
recognized proportion­ately as cash is  and multiply by the 30% gross profit 
collected. To determine cash collections in this percentage (30% × $500,000 = 
case, first compute the 2011 installment sales  $150,000). 
by divid­ing deferred gross profit by the gross 
profit percentage ($560,000   40% = 
$1,400,000). Then, the 12/31/11 in­stallment 
accounts receivable is subtracted to determine 
cash collections ($1,400,000 – $800,000 = 
$600,000). Realized gross profit is then 
computed by multiplying cash collections by 
the gross profit percentage ($600,000 × 40% =
$240,000).
57. (c) When using the installment method, 
gross profitrealized is computed as indicated 
below.
Cash collections   × GP% = GP 
realized
This equation can be rearranged as follows:
GP realized ÷ GP% = Cash 
collections
Therefore, cash collected to date on 2010 
sales is $800,000 [($150,000 + $90,000) ÷ 
30%], and on 2011 sales is $500,000 
($200,000 ÷ 40%). Installment accounts 
receiv­able at 12/31/11 is computed by 
subtracting cash collections from the original 
sales amount.
Installment AR – 2010 ($1,000,000 – $800,000)
Installment AR – 2011 ($2,000,000 – $500,000)
Total 12/31/11 installment AR

58. (a) The equipment sale is accounted 
for using the installment method. 
The gross profit percentage on the 
sale is 33 1/3% ($600,000 profit ÷ 
$1,800,000 selling price). Since 
$300,000 of the sales price is 
collected in 2011, gross profit of 
$100,000 is recognized (33 1/3% × 
$300,000). The total revenue 
recognized after the sale, in 
Note that the interest collected ($75,000)  proportion to cash collected. Under 
does not affect the computation because it is  the cost recovery method (case II), 
not a collection of sales price. revenue (gross profit) is again 
recognized after the sale, when 
60. (c) The profit on a sale in the ordinary course 
cumulative receipts exceed the cost 
of business is considered to be realized at the  of the asset sold. Therefore, revenue 
time of sale unless it is uncertain whether the 
is not recognized prior to the sale of 
sale price will be collected. The Board 
mer­chandise in either case I or case 
concluded that use of the installment method 
II. 
of accounting is not acceptable unless this 
uncertainty exists. Answers (a), (b), and (d)  A.6.  Cost Recovery Method
are incorrect because they do not involve the 
element of uncertainty regarding the  64. (d) Under the cost recovery method 
collectibility of the sale price.  no profit of anytype is recognized 
until the cumulative receipts 
61. (c) According to the installment  (principaland interest) exceed the 
method of ac­counting, gross profit on cost of the asset sold. This means 
an installment sale is recognized in  that the entire gross profit 
income in proportion to the cash  ($3,000,000 – $2,000,000 = 
collection. The cash col­lected from a  $1,000,000) and the 2011 interest 
given year’s sales is multiplied by that  received ($270,000) will be deferred 
year’s gross profit percentage to  until cash collections exceed 
compute the amount of gross profit  $2,000,000. There­fore, no income is
to be recognized. Answer (a) describes the  recognized in 2011. 
point­of­sale recognition basis, while answer 
65. (d) The installment method is used 
(d) describes the cost recov­ery method.  when collection of the selling price 
Answer (b) does not describe any  is not reasonably assured. However, 
recognitionbasis currently used. when the uncertainty of collection is 
62. (c) The installment method of accounting is used  so great that even the use of the 
when there is a high degree of uncertainty  installment method is precluded, 
regarding the col­lectibility of the sale price. 
then the cost re­covery method may 
be used. Having no reasonable basis 
Under this method, sales reve­nues and the 
for estimating collectibility would 
related cost of goods sold are recognized in the 
provide a great enough un­certainty 
period of the sale. However, the gross profit is 
to use the cost recovery method. It is 
deferred to the periods in which cash is collected.
important to note that anytime the 
Income recognized in the period of collection is 
installment method is used, some 
generally computed by multiplying the cash  risk of 100% collection exists, but 
collected by the gross margin percentage. The in­ the risk must be extreme be­fore the 
stallment method is based upon deferral of the  cost recovery method is employed. 
gross profit, not the net operating profit. The 
installment method is gen­erally only applicable  66. (a) Installment methods of 
when the reporting company is unable to  recognizing revenue are appropriate 
estimate the amount of uncollectible accounts.  only when “collection of the sale 
price is not reasonably assured.” 
63. (d) Under the installment method  Under the cost recovery method, 
(case I) revenue (gross profit) is  gross 
MODULE 9  BASIC TH. & FIN. REP. :  A. BASIC CONCEPTS 261
at its own risk.

profit is deferred and recognized only when  70. (c) The problem states that the sale has 
the cumulative receipts exceed the cost of  been con­summated and that Kame’s initial 
the asset sold. and continuing invest­ments are adequate to 
demonstrate a commitment to pay for the 
A.7.  Franchise Agreements property. However, the fact that Esker’s 
receivable is subject to future subordination 
67. (d) Franchise fee revenue is recognized 
precludes recognition of the profit in full. 
when allmaterial services have been 
Instead, the cost recovery method must be 
substantially performed by the franchiser. 
used to account for the sale. The deposit 
Substantial performance means the franchiser 
method is to be used
has performed substantially all of required 
initial services and has no remaining  1. Until the sale is consummated, when
obligation to refund any cash received. The  all   activities   necessary   for   closing
$60,000 nonrefundable down payment applies  have been performed. 
to the initial services already performed by  2. If the buyer’s initial and continuing 
Rice. Therefore, the $60,000 may be  investments are not adequate to demonstrate a 
recognized as revenue in 2010. The three  commitment to pay for the property and the 
remaining $30,000 installments relate to  seller is not reasonably as­ 
substantial future services to be performed by 
Rice. The present value of these payments 
($72,000) is recorded as unearned franchise 
fees and recognized as revenue once 
substantial performance of the future services 
has occurred.
Cash 60,000
Notes receivable 90,000
Discount on notes receivable
Franchise revenue
Unearned franchise fees

68. (c) Initial franchise fees are not recognized
as reve­nue until the franchisor makes 
substantial performance of the required 
services, and collection is reasonably assured. 
Since Potter Pie has not yet performed the 
required services, the initial franchise fee (21 
× $30,000 = $630,000) is re­ported as 
unearned franchise fees at 12/31/10. The 
esti­mated uncollectible amount ($20,000) 
normally would be recorded as a debit to bad 
debt expense and a credit to al­lowance for 
uncollectible accounts. However, since 
norevenue has yet been recognized, it is 
inappropriate to record bad debt expense. 
Instead, unearned franchise fees is deb­ited, 
because an unearned revenue should not be 
recorded when, in effect, no related asset has 
been received. There­fore, the net unearned 
franchise fees is $610,000 ($630,000
– $20,000).
A.8.  Real Estate Transactions
69. (d) Items I and II do not transfer the risks 
and re­wards of ownership to the buyer since 
both scenarios entitle the buyer to a return of 
his/her initial investment. Thus, the risks of 
ownership still remain with the seller. The 
economic substance of such arrangements is 
that of financing, leasing, or profit­sharing 
transactions. Item III transfers the risks and 
rewards of ownership since the seller will be 
reimbursed for cost plus a 5% profit on the 
support provided. Therefore, the seller is not 
required to support operations of the property 
delivered item, the undelivered item must be 
sured of recovering the cost of the  substantially in the control of the vendor. 
property if the buyer defaults. Therefore answer (a) is correct.
The problem states that the sale has been  A.10. Research or Development 
consummated and that Kame’s initial and  Accounted for on the Milestone 
continuing investments are adequate. 
Method
Therefore, the deposit method will not be used
to account for the sale. The reduced profit  72. (b) The requirement is to identify the 
method is used only when the initial  subject matter of milestone 
investment is adequate to demonstrate a  accounting. Answer (b) is correct 
commitment to pay for the property but the 
because the milestone method of 
continuing investments are not. The 
accounting may be used to recognize 
continuing investments must also meet certain 
ad­ditional requirements for the reduced profit  revenue for research and development 
method to be used. Since Kame’s continuing  arrangements. Note that the milestone 
investments are adequate, the reduced profit  method is an option, but it is not 
method will not be used to account for the  required. 
sale. The full accrual method may be used 
73. (c) The requirement is to identify the amount 
only if profit on the sale is determinable, the 
of revenue recognized under the milestone 
earning process is virtually complete, and all 
of the following: method. Answer (c) is correct because 
contingent revenue may be recognized in its 
1. A sale is consummated.  entirety in the period the milestone is 
2. The   buyer’s   initial   and   continuing achieved. 
investments   are   adequate   to
demonstrate   a   commitment   to   pay A.15.  International Financial Reporting 
for the property.  Standards
3. The seller’s receivable is not subject (IFRS)
to future sub­ordination. 
4. The seller has transferred to the buyer  74. (d) The requirement is to identify 
the usual risks and rewards of  the body that sets international 
ownership in a transaction that is, in  accounting standards. Answer (d) is
substance, a sale and does not have a  correct because the International 
sub­stantial continuing involvement in  Accounting Standards Board 
the property.  (IASB) issues International 
Financial Reporting Standards. 
A.9.  Multiple­Deliverable Revenue 
Arrangements 75. (c) The requirement is to identify the 
qualitative characteristics of relevance. 
71. (a) The requirement is to identify the  Answer (c) is correct because the IASB 
condition thatmust be met to apply separate  Framework provides that relevance includes 
accounting. For a multiple­deliverables  the qualities of predictive value and 
arrangement, two conditions must be met for  confirmatory value. An­swer (a) is incorrect 
an item to be considered a separate unit of  because feedback value is not a char­acteristic 
accounting: (1) The delivered item has value  of relevance. Answer (b) is incorrect because 
on a stand­alone basis, and (2) if the  these qualities are the characteristics of 
arrangement includes a right of return for the  reliability in FASB Concept Statement 1, 
which is superseded by SFAC 8. 
262 MODULE 9  BASIC TH. & FIN. REP.: A. BASIC CONCEPTS

Answer (d) is incorrect because comparability and timeliness 
are enhancing characteristics in the IASB Framework.
76. (b) The requirement is to identify the element that 
is defined as increases in economic benefits in the 
form of inflows or enhancements of assets or 
decreases of liabilities that result in increases in 
equity other than those resulting from 
contributions from equity participants. Answer (b) 
is correct because the IASB Framework has five 
elements: asset, liability, equity, income, and 
expense. The definition given is that of income. 
Note that income includes both revenues and 
gains. 
77. (b) The requirement is to identify the criteria under
IFRS that must be met for an item to be included 
in financial statements. Answer (b) is correct 
because in order for an item to be recognized in 
the financial statements, IFRS requires that it meet 
the definition of an element and can be measured 
reliably. 
78. (c) The requirement is to identify the revenue rec­
ognition method that must be used if the outcome 
of render­ing services cannot be estimated 
reliably. Answer (c) is cor­rect because if the 
outcome of rendering services cannot be measured
reliably, IFRS requires use of the cost recovery 
method. Answer (a) is incorrect because the 
percentage­of­completion method is used when 
reliable estimates can be made. Answer (b) is 
incorrect because the completed con­tract method 
is not permissible under IFRS. Answer (d) is 
incorrect because the installment method is a 
revenue rec­ognition method used under US 
GAAP, not IFRS. 
79. (b) The requirement is to identify the item that is
not one of the criteria for revenue recognition for
sales of goods under IFRS. Answer (b) is correct
because it is not required that payment has been
received. 
80. (c) The requirement is to identify the assets for 
which the entity may use fair value as deemed cost
upon adoption of IFRS. Answer (c) is correct 
because the entity may use fair value as deemed 
cost for any individual item of property plant and 
equipment. 
81. (b) The requirement is to identify the first step 
within the hierarchy of guidance to which 
management re­fers when selecting accounting 
policies. Answer (b) is cor­rect because the 
highest level in the hierarchy is an IFRS standard 
applicable to the transaction. Answer (a), (c), and
(d) are incorrect because they all represent lower levels in
the hierarchy. 
82. (a) The requirement is to identify the transition 
date. Answer (a) is correct because the “date of 
transition to IFRS” is defined as the beginning of 
the earliest period for which an entity presents full 
comparative information under IFRS. 
83. (d)   The   requirement   is   to   identify   how
adjustments are reflected upon adoption of IFRS.
Answer   (d)   is   correct   because   upon   first­time
adoption   of   IFRS,   any   adjustments   required   to
present the opening balances of the statement of
financial position should be recognized directly in
retained   earnings   or,   if   appropriate,   in   another
category of equity. 
MODULE 9 BASIC TH. & FIN. REP. :  A. BASIC CONCEPTS

SOLUTIONS TO SIMULATIONS
Task­Based Simulation 1
Account
Classifications Authoritative
Literature Help

Balance sheet classification Income statement classification
(A) (B) (C) (D) (E) (F) (G) (H) (I) (J)
1.  Bonds payable, due in 2018
2. Treasury stock
3. Accounts payable
4. Sales discounts
5.  Notes payable, due in nine months
6. Inventory
7. Accounts receivable
8. Common stock
9.  Cost of goods sold
10.  Allowance for uncollectible accounts

Task­Based Simulation 2
Accrual Basis
Worksheet Authoritative
Literature Help
Baron Flowers
WORKSHEET TO CONVERT TRIAL BALANCE TO ACCRUAL BASIS
December 31, 2010
Cash basis Adjustments Accrual basis*
Account title Dr. Cr. Dr. Cr. Dr.* Cr.*
Cash 25,600 25,600
Accounts receivable 16,200 (1) 15,800 32,000
Inventory 62,000 (4) 10,800 72,800
Furniture & fixtures 118,200 118,200
Land improvements 45,000 45,000
Accumulated depreciation
& amortization 32,400 (6) 14,250 46,650
Accounts payable 17,000 (3) 13,500 30,500
Baron, Drawings (9) 61,000 61,000
Baron, Capital 124,600 (7) 2,000 (5) 2,600 125,200
Allowance for uncollectible
accounts (2) 3,800 3,800
Prepaid insurance (5) 2,900 2,900
Accrued expenses (7) 3,100 3,100
Estimated liability from lawsuit (8) 50,000 50,000
Sales 653,000 (1) 15,800 668,800
Purchases 305,100 (3) 13,500 318,600
Salaries 174,000 (9) 48,000 126,000
Payroll taxes 12,400 (7) 500 12,900
Insurance 8,700 (5) 300 8,400
Rent 34,200 34,200
Utilities 12,600 (7) 600 13,200
Living expenses 13,000 (9) 13,000
Income summary—inventory (4) 62,000 (4)  72,800 10,800
Uncollectible accounts (2) 3,800 3,800
Depreciation & amortization (6) 14,250 14,250
Estimated loss from lawsuit (8) 50,000 50,000
827,000 827,000 237,150 237,150 938,850 938,850
*
 Completion of these columns was not required.
264 MODULE 9  BASIC TH. & FIN. REP. :  A. BASIC CONCEPTS

Explanations of Adjustments [6]  To record 2010 depreciation and 
[1]  To convert 2010 sales to accrual basis. amortization expense.
Accounts receivable balances:
December 31, 2010
December 31, 2009
Increase in sales
Cost of leasehold improvement $45,000
Estimated life 15 years
Amortization ($45,000 × 1/15 × 9/12) 2,250
Depreciation expense on fixtures and equipment 12,000
$14,250
[2] To   record   provision   for   uncollectible [7]  To convert expenses to accrual basis.
accounts. 

[3] To   convert   2010   purchases   to   accrual


basis. 
Accounts payable balances:
December 31, 2010 $30,500
December 31, 2009
Increase in purchases $
Balances
December 31, Increase
2010 2009 in expenses
Utilities $1,500 $  900 $  600
Payroll taxes 1,600 1,100 500
$3,100 $2,000 $1,100
[4] To record increase in inventory from [8]  To record lawsuit liability at 12/31/10.
12/31/09 to 12/31/10. Attorney’s estimate of probable loss $300,000
Inventory balances: Amount covered by insurance 250,000
December 31, 2010 Baron’s estimated liability $ 50,000
December 31, 2009 [9]  To record Baron’s drawings for 2010.
Increase in inventory

[5]  To adjust prepaid insurance.
Prepaid balances:
December 31, 2010 ($8,700 × 4/12)
December 31, 2009 ($7,800 × 4/12)
Decrease in insurance expense
Salary ($4,000 × 12) $48,000
Living expenses 13,000
$61,000

C
o
A
u
L
i
C
o
1. C

2. I
n
3. T
h
4. E
n
5. T
h
6. A
l
7. I
n
8. T
h
9. T
h
10. A
 
MODULE 9  BASIC TH. & FIN. REP. :  A. BASIC CONCEPTS 265

Explanation of solutions
1. (E)   SFAC   5   states   that   “Relevance   is   a   primary   qualitative   characteristic.   To   be
relevant, information about an item must have feedback value or predictive value (or
both) for users and must be timely.” 
2. (G)   SFAC   6   states   that   “Gains   are   increases   in   equity   (net  assets)   from   peripheral   or
incidental transactions of an entity and from all other transactions and other events and
circumstances affecting the entity except those that result from revenues or investments
by owners.” 
3. (J) SFAC 5 states that “Revenues and gains are realized when products (goods or services),
merchandise, or other assets are exchanged for cash or claims to cash.” 
4. (F) SFAC 8 states that “Comparability, including consistency, is an enhancing quality that
interacts with relevance and faithful representation to contribute to the usefulness of
information.” 
5. (A) SFAC 5 states that “Recognition is the process of formally recording or incorporating 
an item into the financial statements of an entity as an asset, liability, revenue, expense, 
or the like.” SFAC 5 continues the recognition concept by stat­ing, “An item and 
information about it should meet four fundamental recognition criteria to be recognized 
and should be recog­nized when the criteria are met, subject to a cost­benefit constraint 
and a materiality threshold.” 
6. (B)   SFAC   5   states   that   “Comprehensive   income   is   a   broad   measure   of   the   effects   of
transactions and other events on an entity, comprising all recognized changes in equity
(net   assets)   of   the   entity   during   a   period   from   transactions   and   other   events   and
circumstances except those resulting from investments by owners and distributions to
owners.” 
7. (D) SFAC 6 defines revenues as “inflows or other enhancements of assets of an entity or 
settlements of its liabilities (or a combination of both) from delivering or producing 
goods, rendering services, or other activities that constitute the entity’s ongoing major or 
central operations.” 
8. (L) SFAC 5 defines current market value as “the amount of cash, or its equivalent, that
could be obtained by selling an asset in orderly liquidation.” 
9. (E)   Predictive   value   is   the   quality   of   information   that   helps   users   to   increase   the
likelihood of correctly forecasting the outcome of past or present events (SFAC 8). 
10. (I) SFAC 5 states that “Earnings is a measure of performance during a period that is 
concerned primarily with the extent to which asset inflows associated with cash­to­cash 
cycles substantially completed (or completed) during the period exceed (or are less than) 
asset outflows associated, directly or indirectly, with the same cycles.” 

Task­Based Simulation 4
Trial Balance
Worksheet Authoritative
Literature Help
Jane Alexander Theater
December 31, 200X
a) Trial balance # Adjustments Adjusted trial balance Income statement
Debit Credit Debit Credit Debit Credit Debit Credit
Equipment 192,000 192,000
Accumulated depreciation—
equipment 60,000 1 9,500 69,500
Notes payable 90,000 90,000
Admissions revenue 380,000 3 50,000 330,000 330,000
Advertising expense 13,680 4 1,100 12,580 12,580
Salaries expense 57,600 5 4,700 62,300 62,300
Interest expense 1,400 2 1,800 3,200 3,200
Depreciation expense 1 9,500 9,500 9,500
Interest payable 2 1,800 1,800
Unearned admissions revenue 3 50,000 50,000
Prepaid advertising 4 1,100 1,100
Salaries payable 5 4,700 4,700

266 MODULE 9 BASIC TH. & FIN. REP. :  A. BASIC CONCEPTS


Task­Based Simulation 5
Concepts
Authoritative
Literature Help

Account (A) (B) (C) (D) (E)
1. Long­term receivables
2. Available­for­sale securities
3. Equipment
4. Warranty obligations
5. Short­term payables
6.Accounts receivable
7. Bonds payable, due in ten years
8.Trading securities

Task­Based Simulation 6
Adjusting
Entries Authoritative
Literature Help

Accrual Deferral
1. Depreciation expense xx
Accumulated deprecation xx
2. Interest receivable xx
Interest revenue xx
3. Rent expense xx
Prepaid rent xx
4. Unearned revenue xx
Rent revenue xx
5. Wage expense xx
Wages payable xx

Task­Based Simulation 7
Revenue and Expense
Recognition Authoritative
Literature Help
Amounts to be recognized
2010 2011 2012
1. Emco sells $5,000 of goods to a customer, FOB shipping point on 12/30/10. 5,000 ­­ ­­
2. Emco sells three pieces of equipment on a contract over a three­year period.
The sales price of each piece of equipment is $10,000.  Delivery of each
piece of equipment is on February 10 of each year.  In 2010, the customer 10,000 10,000 10,000
paid a $20,000 down payment, and paid $5,000 per year in 2011 and 2012.
Collectibility is reasonably assured.
3. In   1/1/10,   Emco   pays   $9,000   for   a   membership   to
Wholesalers Association for a two­year membership in
the trade association. 
4. On 6/1/10, Emco signs a contract for $20,000 for 
goods to be sold on account. Payment is to be made 
in two installments of $10,000 each on 12/1/11 and 
12/1/12. The goods are delivered on 10/1/10. 
Collection is reasonably assured, and the goods may
not be returned. 
5. Emco sells goods to a customer on July 1, 2010, for $50,000. 
If the customer does not sell the goods to retail customers by 
December 31, 2011, the goods can be returned to Emco. The 
customer sells the goods to retail customers on October 1, 
2011. 
(4,500) (4,500) ­­

20,000 ­­ ­­

­­ 50,000 ­­
MODULE 9  BASIC TH. & FIN. REP. :  A. BASIC CONCEPTS 267

Explanation of solutions
1. When goods are sold FOB shipping point, title passes at the time the goods are shipped.
Therefore, Emco may recognize revenue on 12/30/10 at the time the goods are shipped.
2. Revenue is recognized when earned and realizable. This sale is not complete until each
piece of equipment is delivered. Therefore, Emco should recognize $10,000 each year
as the equipment is delivered, regardless of payment terms. 
3. Emco should match expenses to the year in which the benefits are received, which is over the
two­year period. 
4. Because Emco delivered the goods in 2010, and collectibility is reasonably assured,
Emco should recognize all revenue in 2010. 
5. Because there is a sale with a right of return, and the buyer may return the merchandise if
it is not sold to a retail customer, recognition should be delayed until the right of return
has expired. 
268 MODULE 9  BASIC TH. & FIN. REP.: B. ERROR CORRECTION

ERROR CORRECTION
30% income tax rate. What amount 
MULTIPLE­CHOICE QUESTIONS (1­15)
should Conn report as adjusted 
1.  Loeb Corp.  frequently  borrows  from  the begin­ning retained earnings in its 
bank in orderto maintain sufficient operating 2011 statement of retained 
cash.   The   following   loans   were   at   a   12% earnings? 
interest rate, with interest payable maturity. a. $420,000 
Loeb   repaid   each   loan   on   its   scheduled b. $428,000 
maturity date. c. $440,000 
Date of loan Amount Maturity date Term of loan
d. $442,000 
11/1/10 $ 5,000 10/31/11 1 year
2/1/11 15,000 7/31/11 6 months
5. Lore Co. changed from the cash 
5/1/11 8,000 1/31/12 9 months basis of accounting to the accrual 
basis of accounting during 2011. 
Loeb records interest expense when the loans  The cumula­tive effect of this 
are repaid. As a result, interest expense of  change should be reported in Lore’s 
$1,500 was recorded in 2011. If no correction  2011 financial statements as a 
is made, by what amount would 2011 interest 
expense be understated?
a. $540 
b. $620 
c. $640 
d. $720 
2. During 2011, Paul Company 
discovered that the ending 
inventories reported on its financial 
statements were incor­rect by the 
following amounts: 
2009 $60,000 understated
2010 75,000 overstated
Paul uses the periodic inventory system to 
ascertain year­end quantities that are converted 
to dollar amounts using the FIFO cost method. 
Prior to any adjustments for these errors and 
ignoring income taxes, Paul’s retained earnings
at Janu­ary 1, 2011, would be
a. Correct. 
b. $  15,000 overstated. 
c. $  75,000 overstated. 
d. $135,000 overstated. 
3. Tack,   Inc.   reported   a   retained   earnings
balance of 
$150,000 at December 31, 2010. In June 
2011, Tack dis­covered that merchandise 
costing $40,000 had not been in­cluded in 
inventory in its 2010 financial statements. 
Tack has a 30% tax rate. What amount 
should Tack report as adjusted beginning 
retained earnings in its statement of re­tained 
earnings at December 31, 2011?
a. $190,000 
b. $178,000 
c. $150,000 
d. $122,000 
4. Conn Co. reported a retained 
earnings balance of $400,000 at 
December 31, 2010. In August 
2011, Conn determined that 
insurance premiums of $60,000 for 
the three­year period beginning 
January 1, 2010, had been paid and 
fully expensed in 2010. Conn has a 
a. Prior   period   adjustment excluded   from   the   depreciation
resulting   from   the   correc­ computation when using the 
tion of an error. 
Straight­line method Production or use method
b. Prior   period   adjustment
a. Yes No
resulting   from   the   change b. Yes Yes
in accounting principle.  c. No No
c. Component   of   income   before d. No Yes
extraordinary item. 
d. Component   of   income   after 9. At the end of 2010, Ritzcar Co. failed to 
extraordinary item.  accrue salescommissions earned during 2010 
but paid in 2011. The er­ror was not repeated
6. Bren Co.’s beginning inventory at  in 2011. What was the effect of this error on 
January 1, 2011, was understated by  2010 ending working capital and on the 2011
$26,000, and its ending inventory  end­ing retained earnings balance?
was over­stated by $52,000. As a 
result, Bren’s cost of goods sold for  2010 ending 2011 ending
2011 was  working capital retained earnings
a. Understated by $26,000.  a. Overstated Overstated
b. Overstated by $26,000.  b. No effect Overstated
c. No effect No effect
c. Understated by $78,000. 
d. Overstated No effect
d. Overstated by $78,000. 
10. On December 31, 2011, special insurance 
7. On January 2, 2011, Air, Inc. agreed to pay its  costs wereincurred and unpaid, but were not 
former president $300,000 under a deferred  recorded. If these insur­ance costs were 
compensation arrange­ment. Air should have  related to a particular job order in work in 
recorded this expense in 2010 but did not do 
process that was not completed during the 
so. Air’s reported income tax expense would 
period, what is the effect of the omission on 
have been $70,000 lower in 2010 had it 
accrued liabilities and retained earnings in the
properly accrued this deferred compensation. 
December 31, 2011 balance sheet?
In its December 31, 2011 financial statements, 
Air should adjust the beginning balance of its  Accrued liabilities Retained earnings
retained earnings by a  a. No effect No effect
a. $230,000 credit.  b. No effect Overstated
b. $230,000 debit.  c. Understated No effect
c. $300,000 credit.  d. Understated Overstated
d. $370,000 debit.  11. Which of the following errors could 
8. Net   income   is  understated  if,  in   the result in an over­statement of both current 
first year, estimated salvage value is assets and stockholders’ equity?
MODULE 9  BASIC TH. & FIN. REP.: B. ERROR CORRECTION 269
balance in 2012 by
$10,000. 
a. An   understatement   of   accrued   sales b. Restate   the
expenses.  financial statements
b. Noncurrent   note   receivable with   corrected   bal­
principal is misclassi­fied as a ances   for   all
current asset.  periods presented. 
c. Annual   depreciation   on c. Adjust   the
manufacturing   machinery   is ending   balance
understated.  in   the   2012
d. Holiday   pay   expense   for retained
administrative   employees   is earnings
misclassified as manufacturing account. 
overhead.  d. Make   no   entry   because   the
error will self­correct. 
12. Galaxy Corporation had the following
financial state­ment information:  15. Jackson Company uses 
2011 2010 IFRS to report its financial 
Revenue $135,000 $100,000 results. During the current 
Expenses 98,000 65,000 year, the company 
Net income 37,000 35,000 discovered it had overstated
12/31/11 12/31/10 sales in the prior year. How
Total assets $157,000 $105,000 should the company handle 
Total liabilities 50,000 35,000 this issue? 
Total owners’ equity 107,000 70,000
a. Adjust   sales   for   the   current
Galaxy failed to record $12,000 of accrued wages  period. 
at the end of 2010. The wages were recorded and  b. Spread   the
paid in January 2011. Assuming that the correct  adjustment   over
accruals were made on December 31, 2011, what  the   current   and
are the corrected balances in the 2010 and 2011  future periods. 
restated financial statements? c. Present   the
Dec. 31, 2011 cumulative   effect
2010 Dec. 31, 2010 total owners’ of   the
net income total liabilities equity overstatement   as
a. $23,000 $23,000 $  95,000 an   item   in   the
b. $47,000 $47,000 $107,000 current   period
c. $23,000 $35,000 $  95,000 income statement. 
d. $23,000 $47,000 $107,000
13. Justin Corporation discovered an error 
in their 2011 financial statements after 
the statements were issued. This 
requires that 
a. The cumulative effect of the 
error is reported on the 2012 
income statement as a 
cumulative effect of change in
accounting principle. 
b. The   cumulative   effect   of   the   error   is
reported in the 2012 beginning balance of
each related account. 
c. The   financial   statements   are
restated to reflect the correction
of period­specific effects of the
error. 
d. An adjustment to beginning 
retained earnings in 2012 with
a footnote disclosure 
describing the er­ror. 
14. During 2012, Kelly Corporation 
discovered that ending inventory 
reported in its 2011 financial statements 
was un­derstated by $10,000. How 
should Kelly account for this 
understatement? 
a. Adjust the beginning inventory
presented for comparative purposes.
d. Restate   the   prior   year   financial   statements
270 MODULE 9  BASIC TH. & FIN. REP.: B. ERROR CORRECTION

SIMULATIONS

Task­Based Simulation 1
Corrected Financial
Statements
Authoritative
Literature Help

Situation
Cord Corp., a nonpublic enterprise, requires audited financial statements for credit purposes. After making normal adjust­
ing entries, but before closing the accounting records for the year ended December 31, 2011, Cord’s controller prepared the 
following financial statements for 2011:
Cord Corp.
STATEMENT OF FINANCIAL POSITION
December 31, 2011
Assets
Cash $1,225,000
Marketable equity securities 125,000
Accounts receivable 460,000
Allowance for doubtful accounts (55,000)
Inventories 530,000
Property and equipment 620,000
Accumulated depreciation (280,000)
Total assets $2,625,000
Liabilities and Stockholders’ Equity
Accounts payable and accrued liabilities $ 1,685,000
Income tax payable 110,000
Common stock, $20 par 300,000
Additional paid­in capital 75,000
Retained earnings 455,000
Total liabilities and stockholders’ equity $ 2,625,000
Cord Corp.
STATEMENT OF INCOME
For the Year Ended December 31, 2011
Net sales $1,700,000
Operating expenses:
Cost of sales 570,000
Selling and administrative 448,000
Depreciation 42,000
Total operating expenses 1,060,000
Income before income tax 640,000
Income tax expense 192,000
Net income $ 448,000

Cord’s tax rate for all income items was 30% for all affected years, and it made estimated tax payments when due. Cord
has been profitable in the past and expects results in the future to be similar to 2011. During the course of the audit, the 
following additional information (not considered when the above statements were prepared) was obtained:
1. The investment portfolio consists of short­term investments, classified as available­for­sale, for which total market
value equaled cost at December 31, 2010. On February 2, 2011, Cord sold one investment with a carrying value of
$100,000 for $130,000. The total of the sale proceeds was credited to the investment account. 
2. At December 31, 2011, the market value of the remaining securities in the portfolio was $142,000. 
3. The $530,000 inventory total, which was based on a physical count at December 31, 2011, was priced at cost. Subse­
quently, it was determined that the inventory cost was overstated by $66,000. At December 31, 2011, the inventory’s
market value approximated the adjusted cost. 
4. Pollution   control  devices   costing   $48,000,  which  is  high   in   relation   to  the  cost  of  the   original  equipment,   were
installed on December 29, 2010, and were charged to repairs in 2010. 
5. The original equipment referred to in Item 4, which had a remaining useful life of six years on December 29, 2010,
is being depreciated by the straight­line method for both financial and tax reporting. 
6. A lawsuit was filed against Cord in October 2011 claiming damages of $250,000. Cord’s legal counsel believes that
an unfavorable outcome is probable, and a reasonable estimate of the court’s award to the plaintiff is $60,000,
which will be paid in 2012 if the case is settled. 
7. Cord determined that its accumulated benefits obligation under the pension plan exceeded the fair value of plan
assets by $40,000 at December 31, 2011. Cord has unrecognized prior service cost of $50,000 at December 31, 2011.
Cord funds the total pension expense each year. 
Complete the following amounts in the corrected financial statements for Cord Corp.
MODULE 9  BASIC TH. & FIN. REP.: B. ERROR CORRECTION 271

Cord Corp.
STATEMENT OF FINANCIAL POSITION
December 31, 2011
Assets
Cash
Marketable equity securities
Accounts receivable
Allowance for doubtful accounts
Inventories
Deferred tax asset
Property and equipment
Accumulated depreciation
Total assets
Liabilities and Stockholders’ Equity
Accounts payable and accrued liabilities
Income tax payable
Estimated liability from lawsuit
Pension liability
Common stock, $20 par
Additional paid­in capital
Retained earnings
Other comprehensive income
Total liabilities and stockholders’ equity
Net sales
Operating expenses:
Cost of sales
Selling and administrative
Depreciation
Pension cost
Total operating expenses
Other income/loss:
Gain (loss) on marketable securities
Estimated loss from lawsuit
Income before income tax
Income tax expense
Net income

Task­Based Simulation 2
Effect of
Error
Authoritative
Literature Help

Situation
Klaus Corporation had $580,000 in inventory, which was based on a physical count at December 31, 2011. The inventory
was priced at cost. In February 2012, it was determined that the inventory cost was overstated by $50,000.
Indicate the effects of the inventory overstatement in the 2011 and 2012 financial statements by completing the 
following table. Mark each item overstate, understate, or OK.
2011 Effects 2012 Effects
(Overstate) (Understate) (OK) (Overstate) (Understate) (OK)
Inventory on balance sheet
Cost of goods sold
Net income
Retained earnings

Task­Based Simulation 3
Research
Authoritative
Literature Help
Assume that you are assigned to the audit of Young Corporation. Young has determined that it issued financial statements
in the prior year with a material error in the application of an accounting principle. Which section of the Professional Standards
addresses the issue of how to account for this situation?
272 MODULE 9  BASIC TH. & FIN. REP.: B. ERROR CORRECTION

Enter your response in the answer fields below.

Task­Based Simulation 4
Concepts
Authoritative
Literature Help

Situation

T
h
1. A
c
2. A

3. A
 
4. A

5. A
 

T
Analysis of Authoritative
Error Literature Help

Situation
The auditors of Cardiff Company have found the following errors in the company’s accounting 
records.
Indicate how the error will affect the current year’s financial statements by choosing an “O” in 
the column for overstate and “U” in the column for understate.

1. Cardiff   fails   to
record   a   sale   on
account   for
$8,000. 
2. The inventory was 
miscounted at year­end
and overstated by 
$5,000. 

3. Cardiff fails to record
depreciation   expense
of   $3,500   for   the
year. 
4. The   company
receives   a   utility   bill
for   $400   on
December   29,   but
fails to record the bill.
5. Available­for­sale 
securities were not marked 
to market. Later analy­sis 
reveals that the securities 
in­creased in value by 
$4,000 as of the end of the 
current year. Cardiff does 
not elect the fair value 
option for any of its 
available­for­sale se­
curities. 
Retained Other
Assets Liabilities earnings Net income comprehensive income
(O) (U) (O) (U) (O) (U) (O) (U) (O) (U)

6. Cardiff   fails   to
accrue   wages   of
$17,000 at the end
of the year. 

7.Trading securities were not
marked to market. Later 
analysis reveals that the 
securities declined in value
MODULE 9  BASIC TH. & FIN. REP.: B. ERROR CORRECTION 273
by $8,000 as of the end of 
the cur­rent year. 
Retained Other
Assets Liabilities earnings Net income comprehensive income
(O) (U) (O) (U) (O) (U) (O) (U) (O) (U)

Task­Based Simulation 6
Research
Authoritative
Literature Help

Assume that you are assigned to the audit of Hughes Corporation. Hughes is attempting to 
determine whether an error in the prior year’s financial statement is material. Which section of the 
Professional Standards provides guidance on this matter?
Enter your response in the answer fields below.
274 MODULE 9 BASIC TH. & FIN. REP.: B. ERROR CORRECTION

MULTIPLE­CHOICE ANSWERS

1. a __ __ 5. a __ __ 8. b __ __ 11. d __ __ 14. b __ __
2. c __ __ 6. c __ __ 9. d __ __ 12. d __ __ 15. d __ __
3. b __ __ 7. b __ __ 10. c __ __ 13. c __ __ 1st: __/15 = __%
4. b __ __ 2nd: __/15 = __%

MULTIPLE­CHOICE ANSWER EXPLANATIONS
[$40,000 – ($30% × $40,000)]. This
B. Error Correction
should increase beginning retained 
1. (a)    The correct amount of 2011 interest expense is earnings because the 
$2,040, as computed below. understatement of 12/31/10 
11/1/10 note
inventory would have resulted in an
Interest from 1/1/11 to 10/31/11 overstatement of cost of goods sold 
($5,000 × 12% × 10/12) and therefore an understatement of 
2/1/11 note retained earnings. Thus, the 
Interest from 2/1/11 to 7/31/11 adjustment 1/1/11 retained earnings
($15,000 × 12% × 6/12)
is $178,000 ($150,000 + $28,000). 
5/1/11 note
Interest from 5/1/11 to 12/31/11 Tack’s journal entry to record the 
($8,000 × 12% × 8/12) adjustment is 
Total 2011 interest Inventory 40,000
Since   interest   expense   of   $1,500   was Retained earnings 28,000
Taxes payable 12,000
recorded,   2011   inter­est   expense   was
understated by $540 ($2,040 – $1,500). 4. (b) A correction of an error is 
2. (c) The error in understating the  treated as a prior period 
adjustment and is reported in the 
2009 ending in­ventory would have
financial state­ments as an 
self­corrected by 1/1/11 (2009 
adjustment to the beginning 
income understated by $60,000; 
balance of re­tained earnings in 
2010 income overstated by 
the year the error is discovered. 
$60,000). The error in overstating 
The adjustment is reported net of 
the 2010 ending inventory would 
the related tax effect. In 2010, 
not have been corrected by 1/1/11. 
insurance expense of $60,000 was
This error overstates both 2010 
recorded. The correct 2010 
income and the 1/1/11 retained 
insurance expense was $20,000 
earnings balance by $75,000. 
($60,000 × 1/3). Therefore, 
3. (b) A correction of an error is  before taxes, 1/1/11 retained 
treated as a prior period adjustment, earnings is understated by 
recorded in the year the error is  $40,000. The net of tax effect is 
discov­ered, and is reported in the  $28,000 [$40,000 – (30% × 
financial statements as an ad­ $40,000)], so the adjusted 
justment to the beginning balance  beginning retained earnings is 
of retained earnings. The  $428,000 ($400,000 + $28,000). 
adjustment is reported net of the  5. (a) A change in accounting principle is a 
related tax effect. In this case the 
change from one generally accepted 
net­of­tax effect is $28,000 
principle to another gener­ally accepted  of existing facts, or a change from an 
principle. A correction of an error is  unacceptable prin­ciple to a generally 
thecorrection of a mathematical mistake, a  accepted one. Therefore, a switchfrom the 
mistake in the ap­plication of an accounting  cash basis (unacceptable) to the accrual basis 
principle, an oversight or misuse  (ac­ceptable) is a correction of an error 
reported as a prior pe­riod adjustment.
6.  (c) The requirement is to determine the
effect ofinventory errors on cost of goods
sold.   The   effect   of   the   errors   on   Bren’s
2011   cost   of   goods   sold   (CGS)   is   illus­
trated below.
BI
+ P – $26,000 CGS understated $26,000
GAFS
– EI (+ $52,000) CGS understated  52,000
CGS CGS understated $78,000

Beginning inventory is the starting point for 
the CGS com­putation, so BI errors have a 
direct effect on CGS. The understatement of 
BI ($26,000) causes an understatement of 
goods available for sale (GAFS) and thus of 
CGS. Ending inventory is subtracted in the 
CGS computation, so EI errors have an 
inverse effect on CGS. The overstate­ment of
EI ($52,000) means that too much was 
subtracted in the CGS computation, causing 
another understatement of CGS. Therefore, 
CGS is understated by a total of $78,000.
7. (b) The failure to record the $300,000 of 
deferred compensation expense in 2010 is 
considered an error. The profession requires that
the correction of an error be treated as a prior 
period adjustment. Thus, the requirement is to 
determine the retroactive adjustment that should 
be made to the beginning balance of the retained
earnings for 2011 (including any income tax 
effect). The net adjustment to beginning retained
earnings would be a debit for $230,000 
($300,000 less the income tax benefit of 
$70,000). 
8. (b) The depreciable base used to 
compute depre­ciation expense 
under both the straight­line and 
production methods is equal to the 
cost less estimated salvage value of 
the asset. Depreciation expense is 
overstated and net in­come is, 
therefore, understated when the 
estimated salvage value is excluded 
from the depreciation computation 
under both of these methods. 
9. (d) The entry Ritzcar should have
made to accrue sales commissions
earned but unpaid at the end of its
2010 fiscal year is 
Commission expense xxx
Commissions payable

xxx

Since Commissions payable is a current 
liability, the 2010 ending working capital is 
overstated due to Ritzcar’s fail­ure to record 
this entry. Since this error was not repeated at 
the end of Ritzcar’s 2011 fiscal year, the 
income impact of the 2010 error “self­
corrected” during 2011, when Ritz­car recorded
both the earned but unpaid 2010 commissions  the 2011
plus the 2011 earned commissions. Therefore, 
MODULE 9  BASIC TH. & FIN. REP. :  B. ERROR CORRECTION 275
statements must be restated with period­
specific effects. The restated financial 
ending retained earnings would not be  statements are
impacted by the error.
Galaxy Corporation
10. (c) A liability is accrued when an  RESTATED FINANCIAL 
obligation to pay or perform  STATEMENTS
services has been incurred. This is  2011 2010
the case even if the liability will not  Revenue $135,000 $100,000
be satisfied until a future date.  Expenses 86,000 77,000
Net income 49,000 23,000
Therefore, accrued liabilities will be
understated on the December 31,  12/31/11 12/31/10
Total assets $157,000 $105,000
2011 balance sheet because the  Total liabilities 50,000 47,000
special in­surance costs were not  Total owners’ equity 107,000 58,000
recorded. However, there will be no 
effect on the December 31, 2011  13. (c) The financial statements of all 
balance of retained earnings because periods shouldbe restated and corrections 
these costs relate to work in process, made to reflect any period­specific effects 
and work in process does not affect  of the error. Answer (a) is incorrect because
net income currently. Please note  this is an error, not a change in accounting 
that if the special insurance costs  principle. Answer (b) is incorrect because 
related to goods that were sold, cost  although the
of goods sold would have been 
understated that would have caused 
both net income and retained 
earnings to be overstated. 
11. (d) The classification of holiday pay
expense for administrative 
employees as manufacturing 
overhead would result in the 
capitalization of some or all of these
costs as a component of ending 
inventory, while these costs should 
be expensed as incurred. This error 
could overstate ending inventory, a 
current asset. The overstate­ment of 
ending inventory also understates 
the cost of goods sold (Beginning 
inventories + Net purchases – 
Ending inventories = Cost of goods 
sold), and overstates net in­come 
and stockholders’ equity. The 
understatement of accrued sales 
expenses would not affect current 
assets. The misclassification of the 
noncurrent note receivable princi­
pal as a current asset would have no 
impact on stockhold­ers’ equity. The
understatement of depreciation on 
manu­facturing machinery would 
understate the overhead addedto 
inventories, a current asset. 
12. (d) The entry for the 12/31/10 wage
accrual should have been 
Wages expense 12,000
Wages payable 12,000

Failure to accrue wage expense results in an 
understate­ment of wage expense and an 
understatement of wages payable by $12,000 
in 2010. As a result, net income and retained 
earnings are overstated by $12,000 in 2010. 
The wages were expensed and paid in 2011. 
Therefore, wage expense for 2011 is 
overstated, and 2011 net income is 
understated by $12,000. This is a 
counterbalancing error, and ending retained 
earnings in 2011 would be correct. Al­though 
this is a self­correcting error, the financial 
retained earnings is not sufficient disclosure 
asset accounts may be adjusted to reflect  for error correction. Answer (d) is incorrect 
correction of an error, income statement  because even though it is a self­correcting 
effects must also be disclosed. Answer (d)  error, financial statements must be restated 
is incorrect because the financial statements with period­specific effects of the error. 
must be restated for all periods presented.
15. (d) The requirement is to identify how
14. (b) The financial statements must be restated  anoverstatement of sales in prior year 
for all periods presented with period­specific  financial statement should be treated 
effects disclosed. Answer (a) is incorrect  under IFRS. Answer (d) is correct be­
because although beginning inventory may be cause the overstatement is an error 
adjusted, prior years’ financial statements  which must be ac­counted for by 
must also be restated. Answer (c) is incorrect  restating the prior year financial 
because correcting the balance of ending  statements. 
276 MODULE 9  BASIC TH. & FIN. REP.: B. ERROR CORRECTION

SOLUTIONS TO SIMULATIONS

Task­Based Simulation 1
Corrected Financial
Statements
Authoritative
Literature Help

The best way to calculate the new financial statement amounts is to make the required adjusting entries to correct 
the account balances as shown below.
Cord Corp.
ADJUSTING JOURNAL ENTRIES
December 31, 2011
(Explanations not required)
Dr. Cr.
(1)
Marketable equity securities $  30,000
Realized gain on sale of marketable equity securities $  30,000 [a]
(2)
Unrealized loss on marketable equity securities (other comprehensive income) 13,000
Adjustment to reduce marketable equity securities to market value 13,000 [b]
(3)
Cost of sales 66,000
Inventories 66,000
(4)
Property and equipment 48,000
Income tax payable 14,400 [c]
Retained earnings 33,600 [c]
(5)
Depreciation 8,000
Accumulated depreciation 8,000 [d]
(6)
Estimated loss from lawsuit 60,000
Estimated liability from lawsuit 60,000
(7)
Deferred pension cost 40,000
Additional pension liability 40,000
(8)
Deferred tax asset 21,900 [i]
Income tax payable 13,200 [ii]
Income tax expense 35,100 [iii]

Supporting Computations for Number 8
Corrected total income tax expense for 2011
Income before income tax, as reported $640,000
Add adjustment increasing income
Realized gain on sale of securities 30,000 [1]
670,000
Deduct adjustments decreasing income
Increase cost of sales for inventory overstatement $66,000
Depreciation on pollution control devices 8,000 74,000
Adjusted taxable income before income tax $596,000
Corrected current portion income tax expense and income taxes payable ($596,000 × 30%) $178,800
Deferred income tax benefits—based on following temporary differences:
Lawsuit expected to be settled in 2012 $60,000 [6]
Unrealized loss on short­term marketable equity securities 13,000 [2]
Total future deductible amounts $73,000
[i]    Deferred tax benefit of future deductible amounts ($73,000 × 30%) 21,900
Corrected total income tax expense $156,900
[ii]   Income tax expense as reported (all current, no deferred) $192,000
Corrected income tax payable and income tax expense 178,800
$  13,200
[iii]  Income tax expense, as reported $192,000
Corrected total income tax expense 156,900
$  35,100
MODULE 9  BASIC TH. & FIN. REP. :  B. ERROR CORRECTION 277

Explanations of Amounts
[a]   Gain on sale of marketable equity securities
Selling price $130,000
Cost 100,000
Gain $ 30,000
[b]   Adjustment to reduce marketable equity securities to market value
Marketable equity securities, at cost
Balance, 2/2/11 as reported $125,000
Adjustment for recording error 30,000
Adjusted balance, 12/31/11 155,000
Market valuation, 12/31/11 142,000
Adjustment required, 12/31/11 $ 13,000
[c]   Prior year adjustment for pollution control devices
Cost of installation, 12/29/10 $ 48,000
Deduct income tax effect ($48,000 × 30%) 14,400
Credit adjustment to retained earnings, 1/1/11 $ 33,600
[d]   Depreciation for 2011 on pollution control devices
Cost of the installation on 12/29/10 $ 48,000
Depreciation for 2011 ($48,000 ÷ 6 years) $ 8,000

Cord Corp.
STATEMENT OF FINANCIAL POSITION
December 31, 2011
Assets
Cash $1,225,000
Marketable equity securities 142,000
Accounts receivable 460,000
Allowance for doubtful accounts (55,000)
Inventories 464,000
Deferred tax asset 21,900
Deferred pension cost 40,000
Property and equipment 668,000
Accumulated depreciation (288,000)
Total assets $2,677,900
Liabilities and Stockholders’ Equity
Accounts payable and accrued liabilities $1,685,000
Income tax payable 111,200
Estimated liability from lawsuit 60,000
Pension liability 40,000
Common stock, $20 par 300,000
Additional paid­in capital 75,000
Retained earnings 419,700
Other comprehensive income (13,000)
Total liabilities and stockholders’ equity $2,677,900

Calculation of corrected retained earnings:
Previous retained earnings (incorrect) 455,000
Less:  Net income for 2011 (incorrect) (448,000)
Beginning of year retained earnings (correct) 7,000
Net income (corrected) 379,100
Prior period adjustment for accounting error in expense
pollution control devices in 2010 33,600
Corrected retained earnings 419,700

Cord Corp.
STATEMENT OF INCOME
For the Year Ended December 31, 2011
Net sales $1,700,000
Operating expenses:
Cost of sales (636,000)
Selling and administrative (448,000)
Depreciation (50,000)
Total operating expenses (1,134,000)
Other income/loss:
Gain (loss) on marketable securities 30,000
Estimated loss from lawsuit (60,000)
Income before income tax 536,000
Income tax expense (156,900)
Net income 379,100
278 MODULE 9  BASIC TH. & FIN. REP.: B. ERROR CORRECTION

Task­Based Simulation 2
Effect of
Error Authoritative
Literature Help

2011 Effects 2012 Effects
(Overstate) (Understate) (OK) (Overstate) (Understate) (OK)
Inventory on balance sheet
Cost of goods sold
Net income
Retained earnings

Task­Based Simulation 3
Research
Authoritative
Literature Help

ASC 250 10 45 23

Task­Based Simulation 4
Concepts
Authoritative
Literature Help
(T) (F)

1. Accounting errors are reported as adjustments to the end of the year retained earnings in the current year’s financial
statements. 
2. A self­correcting error need not be reported as an error if it has corrected itself in the current year. 
3. A change in depreciation method is treated as an error when adjusting the financial statements for the current year. 
4. An accounting error is treated as a prior period adjustment. 
5. A change in the useful lives of assets in calculating depreciation is treated as an accounting error and 
accumulated depreciation is adjusted to reflect this error.
Explanation of solutions
1. False. Accounting errors require an adjustment to the beginning of the year retained earnings in the current year
financial statements. 
2. False. A self­correcting error would require an adjustment to correct comparative year financial statements. Adjustments to
comparative years should be made to reflect retroactive application of the prior period adjustment to the accounts affected. 
3. False.  A change in depreciation method is treated as accumulative effect of change in accounting principle. 
4. True.  Correction of errors involve restatements of prior periods. 
5. False. A change in the useful lives of an asset is treated on a prospective basis, and depreciation for the current
year is calculated based on book value divided by the estimated number of years remaining. 

Task­Based Simulation 5
Analysis
of Error Authoritative
Literature Help

Retained Other
Assets Liabilities earnings Net income comprehensive income
(O) (U) (O) (U) (O) (U) (O) (U) (O) (U)
1.   Cardiff fails to record a sale on
account for $8,000.
MODULE 9  BASIC TH. & FIN. REP. :  B. ERROR CORRECTION 279

2. The inventory was miscounted 
at year­end and overstated by 
$5,000. 
3. Cardiff   fails   to   record
depreciation   expense   of
$3,500 for the year. 
4. The company receives a utility
bill for $400 on December 29,
but fails to record the bill. 
5. Available­for­sale securities were 
not marked to market. Later analy­
sis reveals that the securities in­
creased in value by $4,000 as of the 
end of the current year. Cardiff does
not elect the fair value option for 
any of its available­for­sale 
securities. 
Retained Other
Assets Liabilities earnings Net income comprehensive income
(O) (U) (O) (U) (O) (U) (O) (U) (O) (U)

6. Cardiff fails to accrue wages of
$17,000 at the end of the year. 
7. Trading securities were not marked to
market. Later analysis reveals that the
securities declined in value by $8,000
as of the end of the cur­rent year. 
Explanation of solutions
1. Cardiff should have debited Accounts Receivable and credited Sales. Therefore, accounts receivable is understated,
and retained earnings is understated. 
2. Cardiff has overstated ending inventory, which understates cost of goods sold. Understating cost of goods sold
results   in   income   being   overstated   for   the   period.   Net  income   flows   into   retained  earnings,   and   thus,  retained
earnings is also overstated in the current period. 
3. Cardiff should have debited Depreciation Expense and credited Accumulated Depreciation. By not making this entry,
de­preciation expense is too low, overstating net income, and overstating retained earnings. Accumulated 
depreciation is also too low, which overstates assets. 
4. The entry would be to debit Utility Expense and credit Accounts Payable. Failing to make this entry would understate
ex­penses, which results in overstated net income, overstated retained earnings, and understated liabilities. 
5. The unrealized gain on available­for­sale securities is not recognized in net income, but is recognized as an 
unrealized gain in other comprehensive income. Failing to mark to market when an available­for­sale security has
increased in value would result in understating assets and understating other comprehensive income for the 
period. 
6. The journal entry to accrue wages is to debit Wage Expense and credit Wages Payable. Failure to accrue wages 
results in expenses being understated, which in turn, overstates net income and overstates retained earnings for the 
period. Failure to accrue wages also results in liabilities being understated. 
7. The unrealized gains and losses from trading securities should be included in net income for the period. Failure to mark to 
market and recognize the loss would result in assets being overstated, net income being overstated, and retained earnings being 
overstated for the period. 

Task­Based Simulation 6
Research
Authoritative
Literature Help

ASC 250 10 45 27
280 MODULE 9 BASIC TH. & FIN. REP.: C. ACCOUNTING CHANGES

ACCOUNTING CHANGES
a. Cash basis of accounting
MULTIPLE­CHOICE QUESTIONS (1­29)
for   vacation   pay   to   the
1. On January 1, 2009, Bray Company  accrual basis. 
purchased for$240,000 a machine with a useful b. Straight­line method of 
life of ten years and no salvage value. The  depreciation for previously 
machine was depreciated by the double­ recorded assets to the 
declining balance method and the carrying  double­declining balance 
amount of the machine was $153,600 on  method. 
December 31, 2010. Bray changed to the  c. Presentation of statements of
straight­line method on January 1, 2011. Bray  individual companies to their
can justify the change. What should be the  inclusion   in   consolidated
depreciation expense on this machine for the  statements. 
year ended December 31, 2011? d. Completed­contract 
a. $15,360  method of accounting 
b. $19,200  for long­term 
construction­type 
c. $24,000 
contracts to the 
d. $30,720 
percentage­of­
Items 2 and 3 are based on the following: completion method. 

On January 1, 2009, Warren Co. purchased a  6. When  a company  changes from  the straight­line


$600,000 machine, with a five­year useful life  method   of   depreciation   for   previously   recorded
and no salvage value. The machine was  assets   to   the   double­declining   balance   method,
depreciated by an accelerated method for book  which of the following should be 
and tax purposes. The machine’s carrying  used?
amount was $240,000 on December 31, 2010.  Cumulative effects of change Retrospective
On January 1, 2011, Warren changed to the  in accounting principle application
straight­line method for financial reporting  a. No No
purposes. Warren can justify the change.  b. No Yes
Warren’s income tax rate is 30%. c. Yes Yes
d. Yes No
2. In its 2011 income statement, what 
amount shouldWarren report as the  Items 7 and 8 are based on the following:
cumulative effect of this change? During 2011, Orca Corp. decided to change 
a. $120,000  from the FIFO method of inventory valuation 
b. $  84,000  to the weighted­average method. Inventory 
c. $  36,000  balances under each method were as follows:
d. $0  FIFO Weighted­average
January 1, 2011 $71,000 $77,000
December 31, 2011 79,000 83,000
Orca’s income tax rate is 30%.
2009 and 3,500 during 2010. 
3. On   January   1,   2011,   what   amount
Holly’s   income   tax   rate   is   30%.   Holly
should   Warren   report   as   deferred should report the accounting change in its 2011
income   tax   liability   as   a   result   of   the financial statements as a(n) 
change?  a. Cumulative effect of a
a. $120,000  change   in   accounting
b. $  72,000  principle   of   $2,000   in
c. $  36,000  its income statement. 
d. $0  b. Entry   for   current   year
depreciation expense on the
4. On January 2, 2011, to better reflect 
income   statement   and
the variable use of its only machine, 
treated on a prospective ba­
Holly, Inc. elected to change its 
sis. 
method of depreciation from the 
c. Cumulative effect of a
straight­line method to the units of 
change   in   accounting
production method. The original cost 
of the machine on January 2, 2009,  principle   of   $1,400   in
was $50,000, and its estimated life  its income statement. 
was ten years. Holly estimates that the d. Adjustment   to
machine’s total life is 50,000 machine beginning   retained
hours.  earnings of $1,400. 
Machine   hours   usage   was   8,500   during 5. The effects of a change in accounting
principle   should   be   recorded   on   a 7. In   its   2011   financial   statements,
prospective basis when the change is what amount should Orca report as
from the  the gain or loss on the cumulative
effect of this accounting change? 
a. $2,800 
b. $4,000 
c. $4,200 
d. $0 
8. In   accordance   with   the   Codification,
Orca  should   report  the  effect  of  this
accounting change as a(n) 
a. Prior period adjustment. 
b. Component   of   income   from
continuing operations. 
c. Retrospective application to
previous   year’s   finan­cial
statements. 
d. Component   of   income   after
extraordinary items. 
9. On January 1, 2011, Roem Corp. 
changed its inventory method to FIFO
from LIFO for both financial and 
income tax reporting purposes. The 
change resulted in a $500,000 increase
in the January 1, 2011 inventory, 
which is the only change that could be
calculated from the accounting 
records. Assume that the income tax 
rate for all years is 30%. Retro­
spective application would result in 
a. An   increase   in   ending
inventory in the 2010 bal­
ance sheet. 
b. A decrease in ending inventory in the
2011 balance sheet. 
c. A decrease in net income in 2010. 
d. A   gain   from   cumulative
effect  of  change   on   the   in­
come statement in 2011. 
MODULE 9  BASIC TH. & FIN. REP. : C. ACCOUNTING CHANGES 281
$108,000 in 2011. 
c. A   cumulative   effect
10.  Which   of   the   following   would   receive adjustment   of   $108,000   on
treatment   as   acumulative   effect   on   an the 2011 income statement. 
accounting change on the income statement? d. An   increase   in
LIFO to FIFO to ending   retained
weighted­average weighted­average earnings   of
a. Yes Yes $180,000 in 2010. 
b. Yes No
c. No No
14. The effect of a change in 
d. No Yes accounting principle that is 
inseparable from the effect of a 
11. On August 31, 2011, Harvey Co.  change in accounting estimate 
decided to change from the FIFO  should be reported 
periodic inventory system to the  a. By restating the financial
weighted­average periodic inventory  statements   of   all   prior
system. Harvey is on a calendar year  periods presented. 
basis. The cumulative effect of the  b. As a correction of an error. 
change is deter­mined  c. As   a   component  of   income
a. As of January 1, 2011.  from continuing opera­tions,
b. As of August 31, 2011.  in the period of change and
c. During   the   eight   months future periods if the change
ending August 31, 2011, by affects both. 
a   weighted­average   of   the d. As a separate disclosure 
purchases.  after income from 
d. As of the earliest period presented if continuing operations, in 
practicable.  the period of change and 
future periods if the change 
12. In 2011, Brighton Co. changed from the  affects both. 
individual item approach to the aggregate 
approach in applying the lower of FIFO cost or 
market to inventories. The change should be 
reported in Brighton’s financial statements as a 
a. Change   in   estimate   on   a   prospective
basis. 
b. Cumulative effect of 
change in accounting 
principle on the current 
year income statement. 
c. Retrospective   application   to   the
earliest   period   pre­sented   if
practicable. 
d. Prior   period   adjustment   with   a
separate disclosure. 
13. On January 1, 2011, Poe Construction, Inc. 
changed to the percentage­of­completion 
method of income recognition for financial 
statement reporting but not for income tax re­
porting. Poe can justify this change in 
accounting principle. As of December 31, 2010,
Poe compiled data showing that income under 
the completed­contract method aggregated 
$700,000. If the percentage­of­completion 
method had been used, the accumulated income
through December 31, 2010, would have been 
$880,000. Assuming an income tax rate of 40% 
for all years, the cumulative effect of this 
accounting change should be reported by Poe as
a. An   increase   in
construction­in­
progress   for
$180,000 in the 2010
balance sheet. 
b. A   decrease   in   the
beginning   balance   of
retained   earnings   for
period presented. 
15. Which of the following is considered b. Net income of the current year. 
a   direct   effect   of   a   change   in c. Retained earnings of the current year. 
accounting principle?  d. The carrying value of the 
a. Deferred taxes.  assets and liabilities at the 
b. Profit sharing.  beginning of the earliest 
c. Royalty payments.  period to which it can be 
d. None of the above.  applied. 

16. Indirect   effects   from   a   change   in 19. On January 1, 2007, Taft Co. purchased a


accounting   principle   should   be patent for 
reported  $714,000. The patent is being amortized over 
a. Retrospectively  to  the   earliest period its remaining legal life of fifteen years expiring
presented.  on January 1, 2019. Dur­ing 2010, Taft 
b. As a cumulative change in determined that the economic benefits of the 
accounting principle  in the patent would not last longer than ten years 
current period.  from the date of acquisition. What amount 
c. In the period in which the should be reported in the balance sheet for the 
accounting   change   oc­ patent, net of accumulated amortization, at De­
curs.  cember 31, 2010?
d. As a prior period adjustment.  a. $428,400 
b. $489,600 
17. If it is impracticable to determine the  c. $504,000 
cumulative effect of an accounting  d. $523,600 
change to any of the prior periods, 
the accounting change should be  20. On January 1, 2007, Flax Co. purchased a 
accounted for  machine for $528,000 and depreciated it by the 
a. As a prior period adjustment.  straight­line method us­ing an estimated useful 
b. On a prospective basis.  life of eight years with no salvage value. On 
c. As   a   cumulative   effect January 1, 2010, Flax determined that the ma­
change on the income state­ chine had a useful life of six years from the date
ment.  of acquisi­tion and will have a salvage value of 
d. As   an   adjustment   to $48,000. An account­ing change was made in 
retained   earnings   in   the 2010 to reflect these additional data. The 
first period presented.  accumulated depreciation for this machine 
should have a balance at December 31, 2010, of
18. If the cumulative effect of applying  a. $292,000 
an accounting change can be  b. $308,000 
determined but the period­specific  c. $320,000 
effects on all periods cannot be  d. $352,000 
determined, the cumulative effect of 
the change should be applied to  21. How should the effect of a change
a. The end balance of retained in   accounting   esti­mate   be
earnings   of   the   earliest accounted for? 
282 MODULE 9 BASIC TH. & FIN. REP.: C. ACCOUNTING CHANGES
requiring   2009   financial
statements to be restated. 
a. By   restating   amounts
reported in financial state­ 25. A company has included in its consolidated 
ments of prior periods.  financial statements this year a subsidiary 
b. By   reporting   pro   forma   amounts   for acquired several years ago that was 
prior periods.  appropriately excluded from consolidation last 
c. As a prior period adjustment year. This should be reported as 
to   beginning   retained a. An   accounting   change   that
earnings.  should   be   reported   pro­
d. In   the   period   of   change spectively. 
and   future   periods  if   the b. An   accounting   change
change affects both.  that should be reported
retrospectively. 
22. During 2010, Krey Co. increased the  c. A correction of an error. 
estimated quantity of copper  d. Neither   an   accounting
recoverable from its mine. Krey uses  change nor a correction of
the units of production depletion  an error. 
method. As a result of the change, 
which of the following should be  26. Which of the following statements is
reported in Krey’s 2010 financial  correct regarding accounting 
statements?  changes that result in financial 
statements that are, in effect, the 
Cumulative effect of Pro forma effects of
a change in retroactive application statements of a different reporting 
accounting principle of new depletion base entity? 
a. Yes Yes a. Cumulative­effect 
b. Yes No adjustments should be 
c. No No reported as separate items 
d. No Yes on the financial statements 
per­taining to the year of 
23. Oak Co. offers a three­year warranty change. 
on its products. Oak previously 
estimated warranty costs to be 2% of
sales. Due to a technological 
advance in production at the begin­
ning of 2010, Oak now believes 1% 
of sales to be a better estimate of 
warranty costs. Warranty costs of 
$80,000 and $96,000 were reported 
in 2008 and 2009, respectively. 
Sales for 2010 were $5,000,000. 
What amount should be presented in
Oak’s 2010 financial statements as 
warranty expense? 
a. $  50,000 
b. $  88,000 
c. $100,000 
d. $138,000 
24. For 2009, Pac Co. estimated its two­
year equipment warranty costs based 
on $100 per unit sold in 2009. Experi­
ence during 2010 indicated that the 
estimate should have been based on 
$110 per unit. The effect of this $10 
differ­ence from the estimate is 
reported 
a. In   2010   income   from   continuing
operations. 
b. As an accounting change,
net   of   tax,   below   2010
income   from   continuing
operations. 
c. As   an   accounting   change
requiring   2009   financial
statements to be restated. 
d. As   a   correction   of   an   error
the financial 
b. No   restatements   or statements. 
adjustments   are   required   if b. Permitted if the entity encounters new 
the   changes   involve transactions, events, or conditions that 
consolidated   methods   of are substantively different from existing 
account­ing for subsidiaries. or previous transactions. 
c. No   restatements   or c. Required for material transactions, if 
adjustments   are   required   if the entity had previously accounted 
the changes involve the cost for similar, though immaterial, 
or   equity   methods   of   ac­ transactions under an unacceptable 
counting for investments.  accounting method. 
d. The   financial   statements d. Required if an alternate accounting 
of   all   prior   periods   pre­ policy gives rise to a material change 
sented   are   adjusted in assets, liabilities, or the current year
retrospectively.  net income. 
27. IFRS   requires   changes   in 29. Under   IFRS,   a   voluntary   change   in
accounting principles to be reported accounting   method   may   only   be
a. On a prospective basis.  made by a company if 
b. On a retrospective basis.  a. A new standard mandates the change
c. By restating the financial statements.  in method. 
d. By   a   cumulative b. Management prefers the new method. 
adjustment   on   the c. The   new   method
income statement.  provides   reliable   and
more   relevant
28. Under   IFRS,   changes   in   accounting information. 
policies are  d. There   is   no
a. Permitted if the change  prohibition   of   the
will result in a more  method   in   the
reliable and more  standards. 
relevant presentation of
MODULE 9  BASIC TH. & FIN. REP. : C. ACCOUNTING CHANGES 283

SIMULATIONS
Task­Based Simulation 1
Accounting
Changes
Authoritative
Literature Help

Situation
On January 2, 2011, Quo, Inc. hired Reed to be its controller. During the year, Reed, working 
closely with Quo’s president and outside accountants, made changes in accounting policies, corrected 
several errors dating from 2010 and before, and insti­tuted new accounting policies.
Quo’s 2011 financial statements will be presented in comparative form with its 
2010 financial statements. Items 1 through 10 represent Quo’s transactions.
List A represents possible classifications of these transactions as: a change in accounting 
principle, a change in accounting estimate, a correction of an error in previously presented financial 
statements, or neither an accounting change nor an account­ing error.
List B represents the general accounting treatment for these transactions.  These treatments are
• Retrospective application approach—Apply the new accounting principle to all prior periods 
presented showing the cumulative effect of the change in the carrying value of assets and 
liabilities at the beginning of the first period presented, and adjust financial statements 
presented to reflect period­specific effects of the change. 
• Retroactive   restatement   approach—Restate   the   2010   financial   statements   and   adjust   2010
beginning retained earnings if the error or change affects a period prior to 2010 financial
statements. 
• Prospective approach—Report 2011 and future financial statements on the new basis, but do
not restate 2010 financial statements. 
For each item, select one from List A and one from List B. List B (Select one 
approach)
List A (Select one treatment) X. Retrospective   application
A. Change in accounting principle.  approach. 
B. Change in accounting estimate.  Y. Retroactive   restatement
C. Correction  of an  error in  previously presented  financial approach. 
statements.  Z. Prospective approach. 
D. Neither an accounting change nor an accounting error. 

Items to be answered cash   method   to   the   accrual


method   of   recog­nizing
income   from   these   service
contracts. 
7. During 2011, Quo 
1. Quo manufactures heavy equipment to customer  determined that an 
specifications on a contract basis. On the basis that it is insurance premium paid and
preferable, account­ing for these long­term contracts  entirely expensed in 2010 
was switched from the completed­contract method to  was for the period January 
the percentage­of­completion method.  1, 2010, through January 1, 
2. As a result of a production breakthrough, Quo  2012. 
determined that manufacturing equipment  8. Quo changed its method of 
previously depreciated over fifteen years should be depreciating office equipment 
depreciated over twenty years.  from an accelerated method to 
3. The equipment that Quo manufactures is sold with a  the straight­line method to more 
five­year warranty. Because of a production  closely reflect costs in later 
breakthrough, Quo reduced its computation of warranty years. 
costs from 3% of sales to 1% of sales. 
4. Quo changed from LIFO to FIFO to account for
its finished goods inventory. 
5. Quo changed from FIFO to average cost to account for 
its rawmaterials and work in process inventories.
6. Quo   sells   extended   service   contracts   on   its
products.   Because   related   services   are   performed
over several years, in 2011 Quo changed from the
List A List B
Treatment Approach
(A) (B)  (C) (D) (X) (Y) (Z)

284 MODULE 9 BASIC TH. & FIN. REP.: C. ACCOUNTING CHANGES

List A List B
Treatment Approach
(A) (B)  (C) (D) (X) (Y) (Z)
9. Quo instituted a pension plan for all employees in 2011 and
adopted the accounting standards related to pensions.  Quo had
not previously had a pension plan.
10. During 2011, Quo increased its investment in Worth, Inc. from
a 10% interest, purchased in 2010, to 30%, and acquired a seat
on Worth’s board of directors.  As a result of its increased in­
vestment, Quo changed its method of accounting for invest­
ment in subsidiary from the cost adjusted for fair value method
to the equity method. Quo did not 
elect to use the fair value method to 
report its 30% investment in Worth.

Task­Based Simulation 2
Calculations
Authoritative
Literature Help

Kent has $500,000 in equipment and machinery that was acquired on January 2, 2009. Kent 
has been using the double­declining balance method to depreciate the equipment over an 
estimated 10­year economic life with no salvage. On January 1, 2011, Kent decides to change to 
the straight­line method with no salvage. Kent has a 40% tax rate.
Calculate the following amounts:
1. Accumulated depreciation as of 12/31/10 
2. Depreciation expense for 2011 
3. Accumulated depreciation as of 12/31/11 
4. Indicate the amount of the accounting change shown net of tax if appropriate 
Task­Based Simulation 3
Classification/ Authoritative
Disclosure Literature Help

Clark made the following changes in its accounting policies.
• Clark changed its depreciation method for its production machinery from the double­declining
balance method to the straight­line method effective January 1, 2011. 
• Clark appropriately changed the salvage values used in computing depreciation for its office
equipment. 
• Clark appropriately changed the specific subsidiaries constituting the group of companies for
which consolidated fi­nancial statements are presented. 
For each of the items that Clark changed, identify whether Clark should show the following by 
choosing Yes or No in the appropriate columns.
MODULE 9  BASIC TH. & FIN. REP. : C. ACCOUNTING CHANGES 285

Change item
1. Clark changed its 
depreciation method for its
production machinery 
from the double­declining 
balance method to the 
straight­line method 
effective January 1, 2011. 
2. Clark appropriately 
changed the sal­vage 
values used in computing 
de­preciation for its office 
equipment. 
3. Clark appropriately changed 
the spe­cific subsidiaries 
constituting the group of 
companies for which con­
solidated financial statements 
are presented. 
Retrospective
Cumulative effect of Pro forma effects of application to financial
change in principle in retroactive application statements of all prior
net income of the for all prior periods periods presented
period of change presented currently currently
Yes No Yes No Yes No

Task­Based Simulation 4
Research
Authoritative
Literature Help

Assume that you are assigned to the audit of Clark Corporation. Clark is considering changing the 
depreciation method for its production machinery from the double­declining balance method to the 
straight­line method. Which section of the Professional Standards provides guidance on the nature of 
this type of change?
Enter your response in the answer fields below.

Task­Based Simulation 5
Accounting Authoritative
Treatment Literature Help

Situation

Falk Co. began operations in January, 2010. On January 2, 2011, Falk Co. hired a new 
controller. During the year, the controller, working closely with Falk’s president and outside 
accountants, made changes in existing accounting policies, instituted new accounting policies, and
corrected several errors dating from prior to 2011.
Falk’s financial statements for the year ended December 31, 2011, will not be presented in 
comparative form with its 2010 financial statements.
List   A   represents   possible   classifications   of   these   transactions   as   a   change   in   accounting
principle, a change in accounting estimate, correction of an error in previously presented financial
statements, or neither an accounting change nor an error cor­rection.
List B represents the general accounting treatment required for these transactions.  These 
treatments are
• Retrospective application approach—Apply the new accounting principle to all prior periods 
presented showing the cumulative effect of the change in the carrying value of assets and 
liabilities at the beginning of the first period presented, and adjust financial statements presented
to reflect period­specific effects of the change. 
• Retroactive restatement approach—Adjust 2011 beginning retained earnings if the error or
change affects a period prior to 2011. 
• Prospective approach—Report 2011 and future financial statements on the new basis, but
do not adjust beginning re­tained earnings or include the cumulative effect of the change
in the 2011 income statements. 
presented   financial
List A—Type of change (Select one)
statements. 
A. Change in accounting principle.  D. Neither an accounting change nor an error
B. Change in accounting estimate.  correction. 
C. Correction of an error in previously
List B—General accounting treatment (Select one) Y. Retroactive restatement approach. 
X. Retroactive application approach.  Z. Prospective approach. 
286 MODULE 9 BASIC TH. & FIN. REP.: C. ACCOUNTING CHANGES

For Items 1 and 2, select a classification for each transaction from List A and the general accounting treatment required to
report the change from List B.

1. Falk manufactures customized equipment to customer speci­
fications on a contract basis. Falk changed its method of 
accounting for these long­term contracts from the completed­
contract method to the percentage­of­completion method because 
Falk is now able to make reasonable esti­mates of future 
construction costs. 
2. Based on improved collection procedures, Falk changed the 
percentage of credit sales used to determine the allowance for 
uncollectible accounts from 2% to 1%. 
List A List B
Type of change Accounting treatment
(A) (B)  (C)  (D) (X) (Y) (Z)

Task­Based Simulation 6
MODULE 9  BASIC TH. & FIN. REP. : C. ACCOUNTING CHANGES 287

Task­Based Simulation 7
Research
Authoritative
Literature Help

Assume that you are assigned to the audit of Wilson Construction Company. Wilson is considering changing its method 
of accounting from the completed­contract method to the percentage­of­completion method. Which section of the 
Professional Standards addresses the issue of how to account for the indirect effects of the change?
Enter your response in the answer fields below.
288 MODULE 9 BASIC TH. & FIN. REP.: C. ACCOUNTING CHANGES

MULTIPLE­CHOICE ANSWERS

1. b __ __ 8. c __ __ 15. a __ __ 22. c __ __ 29.  c __   __


2. d __ __ 9. a __ __ 16. c __ __ 23. a __ __
3. d __ __ 10. c __ __ 17. b __ __ 24. a __ __
4. b __ __ 11. d __ __ 18. d __ __ 25. b __ __
5. b __ __ 12. c __ __ 19. b __ __ 26. d __ __
6. a __ __ 13. a __ __ 20. a __ __ 27. b __ __ 1st: __/29 = __%
7. d __ __ 14. c __ __ 21. d __ __ 28. a __ __ 2nd: __/29 = __%

MULTIPLE­CHOICE ANSWER EXPLANATIONS
5. (b) The requirement is to determine which account­ing 
C.1.  Changes in Accounting Principles (SFAS 154)
change should be reported on a prospective basis. A change 
1. (b) A change in depreciation method is a change  in depreciation method is a change in principle that is not 
in method that is not distinguishable from a  distinguishable from a change in estimate, and is ac­counted
change in estimate, and is accounted for as a  for as a change in estimate. The change is reported on a 
change in estimate. The change is reported on a  prospective basis in the current year and future years. A 
prospective basis in the current year and future  change from the cash basis to the accrual basis of 
years. Book value as of 1/1/11 = $153,600/8 years accounting is a change from non­GAAP to GAAP 
remain­ing = $19,200 depreciation expense in  accounted for as the correction of an error. A change in 
2011.  reporting entity requires retrospective application to the 
earliest year presented if practicable. A change in the 
2. (d) Zero. A change in depreciation method is a  method of accounting for long­term contracts requires 
change in method that is not distinguishable  retrospective application to the earli­est year presented if 
from a change in estimate, and is accounted for  practicable. 
as a change in estimate. The change is reported 
on a prospective basis in the current year and  6. (a) The requirement is to determine whether a 
future years. A change in depreciation method is  change in depreciation method from straight­line to 
no longer given cumulative effect treatment on  double­declining balance should be reported as a 
the income statement.  cumulative effect of a change in accounting principle
and receive retrospective application. A change in 
3. (d) Zero. A change in depreciation method is a  depreciation method is a change in method that is not
change in method that is not distinguishable from  distinguishable from a change in estimate, and is 
a change in estimate, and is accounted for as a  accounted for as a change in estimate. The change is 
change in estimate. The change is reported on a  reported on a prospective basis in the current year 
prospective basis in the current year and future  and future 
years. Because a change in depreciation method is 
no longer given cumulative effect treatment on the
income statement, there are no deferred income 
tax liability effects. 

4. (b) A change in depreciation method is a change 
in method that is not distinguishable from a 
change in estimate, and is accounted for as a 
change in estimate. The change is reported on a 
prospective basis in the current year and future 
years. 
years. Therefore, it does not receive cumulative  if practicable. 
effect treatment or retrospective treatment.
12. (c) A change in inventory method no longer receives 
7. (d) A change in inventory method no longer receives  cumulative effect treatment on the income statement. The 
cumulative effect treatment on the income statement. In­ accounting change is given retrospective application to the 
stead, the accounting change is given retrospective applica­ earliest period presented, if practicable. 
tion to the earliest period presented, if practicable. There­
fore, the answer is zero.  13. (a) The requirement is to indicate how a change in 
accounting principle from the completed­contract 
8. (c) A change in inventory method no longer receives  method to the percentage­of­completion method 
cumulative effect treatment on the income statement. In­ should be reported. A change in the method of 
stead, the accounting change is given retrospective applica­ accounting for long­term contracts requires 
tion to the earliest period presented, if practicable.  retrospective application to the earliest year practi­
cable. This results in a $180,000 increase in 
9. (a) Retrospective application requires applying  income for 2010. The assets would be adjusted for 
the new principle to the earliest period presented  the earliest period affected, and construction­in­
if practicable. Because 2011 beginning inventory  progress would increase by $180,000. Net income 
is the previous year’s ending inventory, the new  for 2010 would increase by $108,000 ($180,000 
principle can be applied to the 2010 financial  less 40% tax effects), and the deferred tax liability 
statements. This would result in an increase in  account would increase in 2010 by $72,000. 
ending inventory in the balance sheet for 2010, a 
decrease in cost of goods sold in 2010, and an  14. (c) The effect of a change in accounting principle 
increase in the begin­ning inventory, which would which is inseparable from the effect of a change in
result in a higher cost of goods sold and a lower  account­ing estimate should be accounted for as a 
net income for 2011.  change in ac­counting estimate. Changes in 
estimate should be accounted for in the period of 
10. (c) A change in inventory method is given retro­ change and also in any affected future periods as a
spective application to the earliest period  component of income from continuing opera­
presented, if prac­ticable.  tions. Financial statements are only restated for 
11. (d) Retrospective application requires the change changes due to an error. Errors include 
to be calculated for the earliest period presented mathematical mistakes, mistakes 
MODULE 9  BASIC TH. & FIN. REP. : C. ACCOUNTING CHANGES 289
12/31/09 book value ($528,000 – $198,000 = 
$330,000) is depreciated down to the $48,000 
in applying accounting principles, oversights or misuse of  salvage value over a remaining useful life of 
available facts, and changes from unacceptable accounting  three years (six years total – three years already 
principles to GAAP. The situation described in this question recorded). Depreciation expense for 2010 is 
does not meet the description of an error. $94,000 [($330,000 – $48,000) ÷ 3], increasing 
accumulated depreciation to $292,000 ($198,000
15. (a) Deferred taxes is a direct effect from the 
+ $94,000). 
change in accounting principle, and its effects 
should be recorded in the earliest period  21. (d) Changes in accounting estimate are to be ac­
presented, if practicable. Profit sharing and  counted for in the period of change and in future
royalty payments are indirect effects and should  periods if 
be re­ported in the period of the change.  the change affects both (i.e., prospectively). Pro forma 
16. (c) Indirect effects of a change in accounting  amounts are presented for changes in accounting principle 
prin­ciple should be reported in the period in  accounted for using the current (cumulative effect) ap­
which the account­ing change occurs. Direct 
effects are reported retrospec­tively to the 
earliest period presented, if practicable. Answers 
(b) and (d) are incorrect since accounting 
changes are no longer treated as cumulative 
effect changes or prior period adjustments. 
17. (b) If it is impracticable to determine the cumulative effect 
of an accounting change to any of the prior periods, the 
accounting change should be accounted for on a pro­
spective basis. Answer (a) is incorrect because accounting 
changes are no longer treated as prior period adjustments. 
Answer (c) is incorrect because accounting changes are no 
longer treated as cumulative effect changes on the income 
statement. Answer (d) is incorrect because the change would
not be to retained earnings in the first period presented. 
18. (d) If the cumulative effect of applying an 
account­ing change can be determined, but the 
period­specific effects on all periods cannot be 
determined, the cumulative effect of the change 
should be applied to the carrying value of the 
assets and liabilities at the beginning of the 
earliest period to which it can be determined. 
C.2.  Change in Accounting Estimates
19. (b) This situation is a change in accounting 
estimate and should be accounted for currently 
and prospectively. From 1/1/07 to 12/31/09, 
patent amortization was recorded using a fifteen­
year life. Yearly amortization was $47,600 
($714,000 ÷ 15), accumulated amortization at 
12/31/09 was $142,800 ($47,600 × 3), and the 
book value of the patent at 12/31/09 was 
$571,200 ($714,000 – $142,800). Beginning in 
2010, this book value must be amortized over its 
remain­ing useful life of 7 years (10 years – 3 
years). Therefore, 2010 amortization is $81,600 
($571,200 ÷ 7) and the 12/31/10 book value is 
$489,600 ($571,200 – $81,600). 
20. (a) From 1/1/07 to 12/31/09, depreciation was 
re­corded using an eight­year life. Yearly 
depreciation was $66,000 ($528,000 ÷ 8), and 
accumulated depreciation at 12/31/09 was 
$198,000 (3 × $66,000). In 2010, the esti­mated 
useful life was changed to six years total with a 
sal­vage value of $48,000. Therefore, the 
term “restatement” re­fers only to correction of 
proach.   Answers   (a)   and   (c)   are   incorrect   because   they errors in previously issued financial statements. 
apply   to  retroactive­type   changes   in   principle   and   error
26. (d) An accounting change that is a change in 
correction, respectively.
report­ing entity is given retrospective application 
22. (c) The effect of a change in accounting estimate  to the earliest period presented, if practicable. The 
should be accounted for in (a) the period of  term “restatement” re­fers only to correction of 
change if the change affects that period only, or  errors in previously issued financial statements. 
(b) the period of change and future periods if the 
C.5.  International Financial Reporting Standards
change affects both. The Codifica­tion further 
states that a change in an estimate should not be  (IFRS)
accounted for by restating amounts reported in  27. (b) The requirement is to identify the item that 
financial statements of prior periods or by 
de­scribes how changes in accounting principles 
reporting pro forma amounts for prior periods. 
are reported under IFRS. Answer (b) is correct 
23. (a) A change in estimated warranty costs due to  because IFRS requires changes in accounting 
technological advances in production qualifies as a principles to be reported by giving retrospective 
change in accounting estimate. Changes in  application to the earliest period presented. 
estimate are treated pro­spectively; there is no  Answer (a) is incorrect because a change in 
retroactive restatement, and the new estimate is  accounting esti­mate is accounted for on a 
used in current and future years. Therefore, in  prospective basis in the current and future 
2010, Oak should use the new estimate of 1% and  periods. Answer (c) is incorrect because restate­
report warranty expense of $50,000 ($5,000,000 ×  ment is required for errors in the financial 
1%).  statements. An­swer (d) is incorrect because 
cumulative adjustments on the income statement 
24. (a) A change in equipment warranty costs based on  are not permitted. 
additional information obtained through experience 
qualifies as a change in accounting estimate. Changes in  28. (a) The requirement is to select the item that describes when
estimate should be accounted for in the period of change as  changes in accounting policies are permitted or required. 
a compo­nent of income from continuing operations and in  Answer (a) is correct because changes are permitted if it 
future pe­riods if necessary. No restatement is required for a will result in a more reliable and more relevant presentation 
change in estimate.  of the financial statements. 

C.3.  Change in Reporting Entity (ASC Topic 250) 29. (c) The requirement is to identify the 


circumstances that may justify a voluntary change
25. (b) An accounting change that is a change in  in accounting method. Answer (c) is correct 
report­ing entity is given retrospective application  because the new method must provide reliable 
to the earliest period presented, if practicable. The  and more relevant information. 
290 MODULE 9 BASIC TH. & FIN. REP.: C. ACCOUNTING CHANGES

SOLUTIONS TO SIMULATIONS

Task­Based Simulation 1
Accountin
Authoritative
g Literature
Changes
Help List A List B
Treatmen Approac
t h
(A) (B)  (C)  (X)  (Y) 
(D) (Z)

1. Quo manufactures heavy equipment to customer specifications on a contract basis. On the 
basis that it is preferable, accounting for these long­term contracts was switched from the 
completed­contract method to the percentage­of­completion method. 
2. As a result of a production breakthrough, Quo determined that manu­facturing equipment
previously depreciated over fifteen years should be depreciated over twenty years. 
3. The  equipment  that Quo manufactures  is  sold  with  a five­year  war­ranty.  Because  of a
production breakthrough, Quo reduced its com­putation of warranty costs from 3% of sales
to 1% of sales. 
4. Quo changed from LIFO to FIFO to account for its finished goods inventory. 
5. Quo changed from FIFO to average cost to account for its raw mate­rials and work in process 
inventories.
6. Quo sells extended service contracts on its products. Because related services are 
performed over several years, in 2011 Quo changed from the cash method to the 
accrual method of recognizing income from these service contracts. 
7. During 2011, Quo determined that an insurance premium paid and entirely 
expensed in 2010 was for the period January 1, 2010, through January 1, 2012. 
8. Quo changed its method of depreciating office equipment from an accelerated 
method to the straight­line method to more closely reflect costs in later years. 
9. Quo instituted a pension plan for all employees in 2011 and adopted the accounting standards 
related to pensions. Quo had not previously had a pension plan. 
10. During 2011, Quo increased its investment in Worth, Inc. from a 10% interest, purchased in
2010, to 30%, and acquired a seat on Worth’s board of directors. As a result of its increased
investment, Quo changed its method of accounting for investment in subsidiary from the 
cost adjusted for fair value method to the equity method. 
Explanation of solutions
1. (A,  X)  This   situation  represents   a  change  in   the   method  of  accounting   for  long­term
construction­type   contracts   and   re­quires   retrospective   application   for   all   periods
presented. 
2. (B, Z) This situation is not a change in accounting principle but is a change in an estimate
and should be handled prospec­tively. 
3. (B, Z) This situation is a change in an estimate. The change in the percentage of warranty costs is not a
change in account­ing principle. Changes in estimate are handled prospectively. 
4. (A,  X)  This  situation is  a  change  from  one  acceptable  accounting  principle  to
another and requires retrospective application. 
5. (A, X) This situation is a change from one acceptable accounting method to another
and requires the retrospective application approach. 
6. (C, Y) The use of the cash method is considered an error since that method is generally not
acceptable. The switch to an acceptable method (accrual) is considered the correction of an
error and requires the retroactive restatement approach. 
7. (C, Y) The write­off of the insurance premium was an error. Since comparative financial
statements are prepared, past years must be corrected using the retroactive restatement
approach even though this error is self­correcting after two years. 
MODULE 9  BASIC TH. & FIN. REP. : C. ACCOUNTING CHANGES 291
8. (B, Z) This situation is a change in method which is considered a change in estimate effected
by   a   change   in   principle.   Per   SFAS   154,   a   change   in   depreciation   method   requires
prospective application. 
9. (D, Z) Since the company did not previously account for pensions this situation is simply
the adoption of an appropriate accounting principle for pensions. No change in principle
occurs. 
10. (D, Y) Quo’s increase in ownership interest (10% to 30%) and its acquisition of a seat on 
Worth’s board of directors has given Quo the ability to exercise significant influence in the 
operating decisions of Worth. If Quo does not elect to use the fair value option to report its 
30% investment in Worth, Quo will use the equity method to account for its investment in 
Worth. The cost adjusted for fair value method is no longer applicable due to the change in 
Quo’s investment in Worth. However, the change from the cost adjusted for fair value 
method to the equity method is not a change in accounting principle. A change in accounting 
principle occurs when an accounting principle different from the one used previously for 
reporting purposes is adopted. For a change to occur, a choice between two or more 
accounting principles must exist. In this case, the change in Quo’s situation allows the equity 
method to account for the investment. Quo’s current economic situation is clearly different 
and this necessitates the adoption of a different accounting principle. Consequently, this is 
not an accounting change. However, the investment account should be retroactively restated 
to reflect balances as if the equity method had always been used. 

Task­Based Simulation 2
Calculations
Authoritative
Literature Help

1. Accumulated depreciation as of 12/31/10 $180,000
2. Depreciation expense for 2011 $ 40,000
3. Accumulated depreciation as of 12/31/11 $220,000
4. Indicate the amount of the accounting change shown net of tax if appropriate $ 0
Explanation of solutions
1.  Before change, Kent used double­declining balance and 10­year life
2009 1/10 × 2 × $500,000 book value = $100,000 depreciation expense
2010 1/10 × 2 × $400,000 new book value = $  80,000 depreciation expense
Accumulated depreciation 12/31/09 $180,000
2.  To calculate depreciation expense for 2011:
Historical cost $500,000
– Accum. depreciation (180,000)
Book value $320,000
($320,000 – $0 salvage) ÷ 8 years remaining life = $40,000 depreciation expense
3.  To calculate accumulated depreciation as of 12/31/11:
2009 $100,000
2010 80,000
2011 40,000
Accum. depr. 12/31/11 $220,000
4. Accounting change net of tax – zero. Accounting standards treat changes in depreciation 
method as a change in estimateeffected by a change in method. Therefore, a change in 
depreciation is accounted for on a prospective basis.
292 MODULE 9 BASIC TH. & FIN. REP.: C. ACCOUNTING CHANGES

Task­Based Simulation 3
Classification/ Authoritative
Disclosure Literature Help

Change item
1. Clark changed its depreciation 
method for its production 
machinery from the double­
declining balance method to the
straight­line method effective 
January 1, 2011. 
2. Clark appropriately changed 
the salvage values used in 
computing depreciation for 
its office equipment. 
3. Clark appropriately changed the 
specific subsidiaries constituting 
the group of companies for which
consolidated fi­nancial statements
are presented. 
Explanation of solutions
Retrospective
Cumulative effect of Pro forma effects of application to financial
change in principle retroactive application statements of all prior
in net income of the for all prior periods periods presented
period of change presented currently currently
Yes No Yes No Yes No

1. (N, N, N) Clark’s change in  depreciation method  is a  change  in accounting estimate


effected by a change in accounting principle. This change would be accounted for on a
prospective basis in the current year and future periods. 
2. (N, N, N) Clark’s change in salvage values is a change in accounting estimate. Clark would
not   report   a   cumulative   effect,   nor   pro   forma   effects,   nor   would   prior   period   financial
statements be restated. 
3. (N, N, Y) Clark’s change in the specific subsidiaries constituting the group of companies for which 
consolidated financial statements are presented is a change in reporting entity. The change in reporting 
entity requires the change to be retrospectively applied to the financial statements of all prior periods 
presented to show financial information for the new reporting entity. 

Task­Based Simulation 4
Research
Authoritative
Literature Help

ASC 250 10 45 18

Task­Based Simulation 5
Accounting
Treatment
Authoritative
Literature Help

1. Falk manufactures customized equipment to 
customer specifica­tions on a contract basis. Falk 
changed its method of accounting for these long­
term contracts from the completed­contract method 
to the percentage­of­completion method because 
Falk is now able to make reasonable estimates of 
future construction costs. 
2. Based on improved collection procedures, Falk 
changed the per­centage of credit sales used to 
determine the allowance for un­collectible 
accounts from 2% to 1%. 
List A List B
Type of Change Accounting Treatment
(A) (B)  (C)  (D) (X) (Y) (Z)

MODULE 9  BASIC TH. & FIN. REP. : C. ACCOUNTING CHANGES 293

Explanation of solutions
1. (A,   X)   Changing   from   using   the   completed­contract   method   to   the   percentage­of­
completion   method   is   a   change   in   ac­counting   principle.   Therefore,   retrospective
application to all periods presented is required. 
2. (B, Z) The percentage of net credit sales used in determining the amount to be added to the 
allowance for uncollectible ac­counts is an estimate made by management. Changing the 
percentage is a change in accounting estimate, which is treated pro­spectively. 

Task­Based Simulation 6
Accounting
Change Authoritative
Literature Help

Posey Corporation
INCOME STATEMENT
Adjusted Adjusted
for the year ending 12­31­11 for the year ending 12­31­10
Sales $130,000 $100,000
Cost of goods sold 87,000 68,000
Gross profit 43,000 32,000
Operating expenses 20,000 17,000
Earnings before taxes 23,000 15,000
Income tax expense (40%) 9,200 6,000
Net income 13,800 9,000

December 31, 2011 December 31, 2010
Inventory $14,000 $7,000
Income tax liability 10,000 6,000

Explanation of solutions
To calculate cost of goods sold using FIFO method, you must first calculate purchases using
the average cost method. Using T­accounts for analysis, the amounts are as shown below.
Inventory
(Weighted­average method)
1­1­10 ­0­
70,000 CGS
Purchases 75,000
12­31­10 5,000
90,000 CGS
Purchases 94,000

12­31­11 9,000

Inventory
(FIFO method)
1­1­10 ­0­
68,000 CGS
Purchases 75,000
12­31­10 7,000
Purchases 94,000 87,000 CGS

12­31­11 14,000

Income tax liability for December 31, 2011, is calculated as
2011 earnings before taxes 23,000 × 40% = $9,200
2010 adjustment to earnings before taxes 2,000 × 40% +  800
Total income tax liability at 12­31­11 $10,000
294 MODULE 9 BASIC TH. & FIN. REP.: C. ACCOUNTING CHANGES

Task­Based Simulation 7
Research
Authoritative
Literature Help

ASC 250 10 45 8
MODULE 9 BASIC TH. & FIN. REP.: D. FINANCIAL STATEMENTS 295

FINANCIAL STATEMENTS
MULTIPLE­CHOICE QUESTIONS (1­114) 3. What amount should Vane report as income after in­
come taxes from continuing operations?
1. In Baer Food Co.’s 2010 single­step income  a. $126,000
statement,the section titled “Revenues”  b. $129,500
consisted of the following: c. $140,000
Net sales revenue d. $147,000
Results from discontinued operations:
Loss from discontinued component Z in­
cluding loss on disposal of $1,200 $ 16,400
Less tax benefit 4,000
Interest revenue
Gain on sale of equipment
Extraordinary gain
Total revenues
In the revenues section of the 2010 income 
statement, Baer Food should have reported 
total revenues of
a. $216,300 
b. $215,400 
c. $203,700 
d. $201,900 
Items 2 and 3 are based on the following:
Vane Co.’s trial balance of income statement 
accounts for the year ended December 31, 
2011, included the following:
Debit
Sales
Cost of sales $240,000
Administrative expenses 70,000
Loss on sale of equipment 10,000
Sales commissions 50,000
Interest revenue
Freight out 15,000
Loss on early retirement of long­term debt 20,000
Uncollectible accounts expense 15,000
Totals $420,000
Other information
Finished goods inventory:
January 1, 2011 $400,000
December 31, 2011 360,000
Vane’s income tax rate is 30%. In Vane’s 
2011 multiple­step income statement,
2. What amount should Vane report as the cost of goods
manufactured?
a. $200,000
b. $215,000
c. $280,000
d. $295,000
One­half of the rented premises is occupied
7. In Yew Co.’s 2010 annual report, Yew 
by the sales department.
Brock’s total selling expenses for 2010 are described itssocial awareness expenditures 
a. $480,000  during the year as follows:
b. $400,000  The Company contributed $250,000 in cash to 
c. $370,000  youth and educa­tional programs. The Company 
also gave $140,000 to health and human service 
d. $360,000 
organizations, of which $80,000 was contributed 
by employees through payroll deductions. In 
5. The following costs were incurred by 
addition, consistent with the Company’s 
Griff Co., amanufacturer, during 2010: commitment to the environment, the Com­pany 
Accounting and legal fees $  25,000 spent $100,000 to redesign product packaging.
Freight­in 175,000 What amount of the above should be 
Freight­out 160,000
Officers salaries 150,000 included in Yew’s income statement as 
Insurance 85,000 charitable contributions expense?
Sales representatives salaries 215,000 a. $310,000 
What amount of these costs should be  b. $390,000 
reported as general and administrative  c. $410,000 
expenses for 2010? d. $490,000 
a. $260,000 
8. During 2010 both Raim Co. and Cane 
b. $550,000 
Co. suffered losses due to the flooding
c. $635,000  of the Mississippi River. Raim is 
d. $810,000  located two miles from the river and 
6. Which of the following should be included  sustains flood losses every two to 
in generaland administrative expenses? three years. Cane, which has been 
located fifty miles from the river for 
Interest Advertising the past twenty years, has never 
a. Yes Yes before had flood losses. How should 
b. Yes No the flood losses be reported in each 
c. No Yes company’s 2010 income statement? 
d. No No
Raim Cane
4. Brock Corp. reports operating expenses in  As an extraordinary 
a. As a component 
two catego­ of income from  item
ries:  (1) selling, and (2) general and  continuing 
administrative.  The operations 
adjusted trial balance at December 31, 2010,  b. As a component  As a component of 
included the of income from  income from 
following expense and loss accounts: continuing  continuing opera­
operations  tions
Accounting and legal fees $120,000
c. As   an As a component of 
Advertising 150,000
Freight­out 80,000 extraordinary income from 
Interest 70,000 item  continuing opera­
Loss on sale of long­term investment 30,000 tions
Officers’ salaries 225,000 As an extraordinary 
Rent for office space 220,000 d. As   an item
Sales salaries and commissions 140,000 extraordinary
item 
296 MODULE 9 BASIC TH. & FIN. REP.: D. FINANCIAL STATEMENTS
12. Ocean Corp.’s comprehensive 
insurance policy allows its assets to 
9. Witt Co. incurred the following infrequent  be replaced at current value. The 
losses during2010: policy has a $50,000 deductible 
• $175,000   from   a   major   strike   by clause. One of Ocean’s waterfront 
employees. 
warehouses was destroyed in a 
winter storm. Such storms occur 
• $150,000 from an earthquake (unusual). 
approximately every four years. 
• $125,000   from   the   abandonment   of
equipment used in the business.  Ocean incurred $20,000 of costs in 
dismantling the warehouse and plans 
In Witt’s 2010 income statement, the total  to replace it. The following data 
amount of infre­quent losses not considered  relate to the warehouse: 
extraordinary should be Current carrying amount $  300,000
a. $275,000  Replacement cost 1,100,000
b. $300,000  What amount of gain should Ocean report as 
c. $325,000  a separate component of income before 
d. $450,000  extraordinary items?
10. Kent Co. incurred the following infrequent a. $1,030,000 
losses dur­ing 2010: b. $  780,000 
c. $  730,000 
• A $300,000 loss was incurred on disposal d. $0 
of one of four dissimilar factories. 
• A major currency devaluation caused a
$120,000   ex­change   loss   on   an
amount   remitted   by   a   foreign   cus­
tomer. 
• Inventory valued at $190,000 was made
worthless by a competitor’s unexpected
product innovation. 
In its 2010 income statement, what amount 
should Kent report as losses that are not 
considered extraordinary?
a. $610,000 
b. $490,000 
c. $420,000 
d. $310,000 
11. Midway   Co.   had   the   following
transactions during 2010: 
• $1,200,000
pretax   loss   on
foreign   currency
exchange due to a
major unexpected
devaluation   by
the   foreign
government. 
• $500,000   pretax
loss   from
discontinued
operations   of   a
division. 
• $800,000 pretax loss on equipment 
damaged by a hurri­cane. This was the first 
hurricane ever to strike in Mid­way’s area. 
Midway also received $1,000,000 from its 
insurance company to replace a building, 
with a carrying value of $300,000, that had 
been destroyed by the hurri­cane. 
What amount should Midway report in its 
2010 income statement as extraordinary loss 
before income taxes?
a. $  100,000 
b. $1,300,000 
c. $1,800,000 
d. $2,500,000 
17. During   2010,   Peg   Construction   Co.
13. Purl Corporation’s income statement for the recognized substan­tial gains from 
year endedDecember 31, 2010, shows the  • An   increase   in   value   of   a   foreign
following: customer’s   remittance   caused   by   a
Income before income tax and extraordinary item major foreign currency revaluation. 
Gain on life insurance coverage—included in the • A   court­ordered   increase   in   a   completed
above $900,000 income amount long­term con­struction contract’s price
Extraordinary item—loss due to earthquake damage due to design changes. 
Purl’s tax rate for 2010 is 40%. How much  Should these gains be included in continuing 
should be re­ported as the provision for income  operations or reported as an extraordinary 
tax in Purl’s 2010 income statement? item in Peg’s 2010 income statement?
a. $200,000 
b. $240,000 
c. $320,000 
d. $360,000 
14. Thorpe Co.’s income statement for 
the year ended De­cember 31, 2011, 
reported net income of $74,100. The 
auditor raised questions about the 
following amounts that had been 
included in net income: 
Unrealized loss on decline in market value of
noncurrent investments in stock classified as
available­for­sale (net of tax) $(5,400)
Gain on early retirement of bonds payable (net of
$11,000 tax effect)
Adjustment to profits of prior years for errors in
depreciation (net of $3,750 tax effect)
Loss from fire (net of $7,000 tax effect) (14,000)
The loss from the fire was an infrequent but not 
unusual occurrence in Thorpe’s line of business. 
Thorpe’s Decem­ber 31, 2011 income statement 
should report net income of
a. $65,000 
b. $66,100 
c. $81,600 
d. $87,000 
15. On   January   1,   2010,   Brecon   Co.
installed   cabinets   to   display   its
merchandise   in   customers’   stores.
Brecon   ex­pects   to   use   these
cabinets   for   five   years.   Brecon’s
2010   multi­step   income   statement
should include 
a. One­fifth of the cabinet costs in cost of
goods sold. 
b. One­fifth   of   the   cabinet
costs   in   selling,   general,
and   administrative
expenses. 
c. All of the cabinet costs in cost of good
sold. 
d. All   of   the   cabinet   costs   in
selling,   general,   and   ad­
ministrative expenses. 
16. A material loss should be presented 
separately as a component of income 
from continuing operations when it is 
a. An extraordinary item. 
b. A   discontinued   component   of   the
business. 
c. Unusual   in   nature   and   infrequent   in
occurrence. 
d. Not unusual in nature but infrequent in
occurrence. 
Gain from major Gain from increase
currency revaluation in contract’s price
a. Continuing operations Continuing operations
b. Extraordinary item Continuing operations
c. Extraordinary item Extraordinary item
d. Continuing operations Extraordinary item
MODULE 9 BASIC TH. & FIN. REP.: D. FINANCIAL STATEMENTS 297
increase in production costs caused 
when a major supplier’s workers went
18. An extraordinary item should be reported  on strike. Which of these losses 
separately onthe income statement as a  should be reported as an 
component of income extraordinary item? 
Before discontinued operations Antitrust action Production costs
Net of income taxes of a component of a business a. No No
a. Yes Yes b. No Yes
b. Yes No c. Yes No
c. No No d. Yes Yes
d. No Yes
22. In open market transactions, Gold Corp. 
19. In 2010, hail damaged several of Toncan Co.’s  simultaneouslysold its long­term investment in 
vans. Hailstorms had frequently inflicted similar  Iron Corp. bonds and pur­chased its own 
damage to Ton­can’s vans. Over the years,  outstanding bonds. The broker remitted the net 
Toncan had saved money by not buying hail  cash from the two transactions. Gold’s gain on 
insurance and either paying for repairs, or selling the pur­chase of its own bonds exceeded its loss
damaged vans and then replacing them. In 2010,  on the sale of the Iron bonds. Gold should 
the dam­aged vans were sold for less than their  report the
carrying amount. How should the hail damage  a. Net effect of the two transactions as
cost be reported in Toncan’s 2010 financial  an extraordi­nary gain. 
statements?  b. Net effect of the two transactions in
a. The   actual   2010   hail income before extraordinary items. 
damage   loss   as   an c. Effect of its own bond transaction 
extraordi­nary   loss,   net   of gain in income before extraordinary 
income taxes.  items, and report the Iron bond 
b. The actual 2010 hail damage transaction as a loss in income 
loss   in   continuing   op­ before ex­traordinary items. 
erations,   with  no  separate d. Effect of its own bond transaction as
disclosure.  an extraordi­nary gain, and report the
c. The   expected   average   hail Iron bond transaction loss in income
damage   loss   in   continu­ing before extraordinary items. 
operations,   with  no  separate
disclosure. 
d. The   expected   average   hail
damage   loss   in   continu­ing
operations,   with   separate
disclosure. 
20. A transaction that is unusual in nature
and   infrequent   in   occurrence   should
be reported separately as a component
of income 
a. After   cumulative   effect   of
accounting   changes   and
before   discontinued
operations. 
b. After   cumulative   effect   of
accounting changes and after
discontinued operations. 
c. Before   cumulative   effect
of   accounting   changes
and   before   discontinued
operations. 
d. Before   cumulative   effect
of   accounting   changes
and   after   discontinued
operations. 
21. In 2010, Teller Co. incurred losses 
arising from its guilty plea in its first 
antitrust action, and from a substantial
reported   as   an
23. Under   ASC   220,  Comprehensive   Income, extraordinary loss. 
corrections of errors are reported in  d. $500,000 should be reported
a. Other comprehensive income.  as   revenues   from   op­
b. Other income/(expense).  erations   of   a   discontinued
c. Retained earnings.  component. 
d. Stockholders’ equity.  26. Which of the following criteria is not
24. Service   Corp.   incurred   costs required for a component’s results to 
associated with relocating employees be classified as discontinued opera­
in   a   restructuring   of   its   operations. tions? 
How should the company account for a. Management   must   have
these costs?  entered   into   a   sales   agree­
a. Measured   at   fair   value ment. 
and   recognized   over   the b. The   component   is   available   for
next two years.  immediate sale. 
b. Measured   at   fair   value   and c. The operations and cash 
recognized   when   the   li­ flows of the component will 
ability is incurred.  be eliminated from the 
c. Recognized when the costs are paid.  operations of the entity as a 
d. Measured   at   fair   value   and result of the disposal. 
treated   as   a   prior   period d. The entity will not have any 
adjustment.  significant continuing 
involvement in the 
25. On January 1, 2010, Deer Corp. met  operations of the component 
the criteria for discontinuance of a  after disposal. 
business component. For the period 
January 1 through October 15, 2010,  27. On November 1, 2010, management 
the component had revenues of  of Herron Corpo­ration committed to 
$500,000 and expenses of $800,000.  a plan to dispose of Timms Company, 
The assets of the component were sold  a major subsidiary. The disposal 
on October 15, 2010, at a loss for  meets the requirements for 
which no tax benefit is available. In its  classification as discontinued 
income state­ment for the year ended  operations. The carrying value of 
December 31, 2010, how should Deer  Timms Company was $8,000,000 and 
report the component’s operations  management estimated the fair value 
from January 1 to October 15, 2010?  less costs to sell to be $6,500,000. For 
a. $500,000 and $800,000  2010, Timms Company had a loss of 
should be included with  $2,000,000. How much should Herron
revenues and expenses,  Corporation present as loss from dis­
respectively, as part of  continued operations before the effect 
continuing operations.  of taxes in its income statement for 
2010? 
b. $300,000 should be reported
as   part   of   the   loss   on a. $0 
operations and disposal of a b. $1,500,000 
component.  c. $2,000,000 
c. $300,000   should   be d. $3,500,000 
298 MODULE 9 BASIC TH. & FIN. REP.: D. FINANCIAL STATEMENTS
d. None of the above. 
33. What is  the purpose of reporting
28. On December 1, 2010, Greer Co. committed to a
comprehensive in­come? 
plan to dispose of its Hart business component’s 
assets. The dis­posal meets the requirements to  a. To   report   changes   in
be classified as discontinued operations. On that  equity   due   to
transactions   with
date, Greer estimated that the loss from the 
owners. 
disposition of the assets would be $700,000 and 
Hart’s 2010 operating losses were $200,000.  b. To   report   a   measure   of
Disregarding income taxes, what net gain (loss)  overall   enterprise   perfor­
should be reported for discontin­ued operations  mance. 
in Greer’s 2010 income statement?  c. To   replace   net   income   with   a   better
a. $0  measure. 
b. $(200,000)  d. To combine income from 
continuing operations with 
c. $(700,000) 
income from discontinued 
d. $(900,000) 
operations and ex­
29. A   component   of   Ace,   Inc.   was traordinary items. 
discontinued   during   2011.   Ace’s
34. During 2010, the “other revenues and 
loss on disposal should 
a. Exclude   the   associated   employee gains” section of Totman Company’s 
Statement of Earnings and 
relocation costs. 
Comprehen­sive Income contains 
b. Exclude   operating   losses   for   the
$5,000 in interest revenue, $15,000 
period. 
equity in Harpo Co. earnings, and 
c. Include   associated   employee $25,000 gain on sale of available­for­
termination costs.  sale securities. Assuming the sale of 
d. Exclude   associated   lease   cancellation the secu­rities increased the current 
costs.  portion of income tax expense by 
30. When a component of a business has  $10,000, determine the amount of 
been discontinued during the year, this Totman’s reclassification adjustment 
component’s operating losses of the  to other comprehensive income. 
current period should be included in 
the 
a. Income   statement   as
part   of   revenues   and
ex­penses. 
b. Income statement as part of
the   loss   on   disposal   of   the
discontinued component. 
c. Income statement as part of
the   income   (loss)   from
continuing operations. 
d. Retained   earnings   statement
as   a   direct   decrease   in
retained earnings. 
31. When a component of a business has
been discontinued during the year, the
loss on disposal should 
a. Include operating losses of the current
period. 
b. Exclude   operating   losses   during   the
period. 
c. Be an extraordinary item. 
d. Be an operating item. 
32. On January 1, 2011, Shine Co. agreed 
to sell a business component on 
March 1, 2011. The gain on the 
disposal should be 
a. Presented as an extraordinary gain. 
b. Presented as an adjustment to retained
earnings. 
c. Netted   with   the   loss   from
operations of the compo­nent
as   a   part   of   discontinued
operations. 
as available for sale. The company 
a. $  5,000  does not elect to use the fair value 
b. $  2,500  option for reporting its available­for­
c. $35,000  sale securi­ties. The fair value of the 
d. $15,000  securities increases to $2,500 on 
December 31, 2009, and to $2,750 on 
35. Which of the following is not an  December 31, 2010. On December 31,
acceptable option of reporting other  2010, the company sells the securities.
comprehensive income and its  Assume no dividends are paid and 
components?  that the company has a tax rate of 
a. In   a   separate   statement   of 30%. What is the amount of the 
comprehensive income.  reclassification adjustment for other 
b. In a statement of earnings comprehensive income on Decem­ 
and   comprehensive   in­ ber 31, 2010? 
come.  a. $ 7,500 
c. In the footnotes.  b. $ (7,500) 
d. In   a   statement   of   changes   in c. $ 5,250 
stockholders’ equity.  d. $ (5,250) 

36. Accumulated   other   comprehensive 39. A company buys ten shares of 


income   should   be   reported   on   the securities at $1,000 each on January 
balance sheet as a component of  15, 2010. The securities are classified 
Retained earnings Additional paid­in capital as available­for­sale. The fair value of 
a. No Yes the securities increases to $1,250 per 
b. Yes Yes share as of December 31, 2010. The 
company does not elect to use the fair 
c. Yes No value option for reporting available­
d. No No for­sale securities. Assume no 
dividends are paid and that the 
37. Which   of   the   following   changes company has a 30% tax rate. What is 
during a period is not a component of the amount of the holding gain arising 
other comprehensive income?  during the period that is classified in 
a. Unrealized gains or losses  other comprehensive income for the 
as a result of a debt se­ period ending December 31, 2010? 
curity being transferred  a. 0 
from held­to­maturity to  b. $1,750 
available­for­sale.  c. $2,500 
b. Stock dividends issued to shareholders. d. $7,500 
c. Foreign   currency   translation
adjustments.  40. Searles does not elect the fair value option for 
d. Minimum   pension   liability recording financial assets and liabilities. What 
adjustments.  amount of comprehen­sive income should 
Searles Corporation report on its state­ment of 
38. A company buys ten shares of  income and comprehensive income given the 
securities at $2,000 each on December fol­lowing net of tax figures that represent 
31, 2008. The securities are classified  changes during a period? 
MODULE 9 BASIC TH. & FIN. REP.: D. FINANCIAL STATEMENTS 299
amounts
must   be
Pension liability adjustment recognized in OCI $ (3,000) presented
Unrealized gain on available­for­sale securities for   other
Reclassification adjustment, for securities gain
included in net income
comprehe
Stock warrants outstanding nsive
Net income income
but   not
a. $86,500 
for
b. $89,000  comprehe
c. $89,500  nsive
d. $90,500  income. 
41. If ($2,450) net of tax is the  c. Separate   EPS   amounts   must   be
reclassification adjustment included  presented   for   com­prehensive   income
in other comprehensive income in  but   not   for   other   comprehensive
the year the se­curities are sold, what income. 
is the gain (loss) that is included in  d. Separat
income from continuing operations  e   EPS
before income taxes? Assume a 30%  amounts
tax rate.  are   not
a. $(2,450)  required
b. $(3,500)  to   be
pre­
c. $ 2,450 
sented
d. $ 3,500 
for
42. Which of the following changes during either
a period is  not  a component of other other
comprehensive income?  compre
hensive
a. Pensio
income
n
or
liability
adjust compre
ment hensive
for income.
funded 44. Which   of   the   following   options   for
status displaying   compre­hensive   income
of plan. is(are) preferred by FASB? 
b. Treasury stock, at cost. 
c. Foreign   currency   translation I. A continuation from net
income at the bottom of
adjustment. 
the income statement. 
d. Reclassifi II. A separate statement that begins with 
cation net income. III. In the statement of 
adjustme changes in stockholders’ equity. 
nt,   for
securities a. I. 
gain   in­ b. II. 
cluded   in c. II and III. 
net d. I and II. 
income. 
45. Assume a company does not elect the 
43. Which of the following is true?  fair value option for reporting financial 
a. Separate  assets and liabilities. Which of the 
EPS  following is not classified as other 
amounts  comprehensive income? 
must be  a. An
presented adjust
for both  ment
other  to
compreh pensio
ensive  n
income  liabilit
and  y   to
compreh record
ensive  the
income.  funde
b. Separate d
EPS status
of   the
plan.  46. When a full set of general­purpose financial
b. Subsequent decreases of the fair value  statementsare presented, comprehensive 
of available­for­sale securities that  income and its components should
have been previously writ­ten down as  a. Appear as a part of discontinued 
impaired.  operations, ex­traordinary items, 
c. Decrease and cumulative effect of a change 
s   in   the in accounting principle. 
fair b. Be reported net of related income tax
value   of effect, in to­tal and individually. 
held­to­ c. Appear in a supplemental schedule in
maturity the notes to the financial statements. 
se­ d. Be displayed in a financial statement
curities.  that has the same prominence as other
d. None of the above.  financial statements. 
Items 47 through 49 are based on the 
following:
The following trial balance of Mint Corp. at 
December 31, 2010, has been adjusted except for
income tax expense.
Dr. Cr.
Cash $ 600,000
Accounts receivable, net 3,500,000
Cost in excess of billings on long­
term contracts 1,600,000
Billings in excess of costs on long­
term contracts $ 700,000
Prepaid taxes 450,000
Property, plant, and equipment, net 1,480,000
Note payable—noncurrent 1,620,000
Common stock 750,000
Additional paid­in capital 2,000,000
Retained earnings—unappropriated 900,000
Retained earnings—restricted for
note payable 160,000
Earnings from long­term contracts 6,680,000
Costs and expenses 5,180,000
$12,810,000 $12,810,000
Other financial data for the year ended 
December 31, 2010, are
• Mint uses the percentage­of­
completion method to account for 
long­term construction contracts for
fi­nancial statement and income tax
purposes. All re­ceivables on these 
contracts are considered to be 
collectible within twelve months. 
• During 2010, estimated tax payments of 
$450,000 were charged to prepaid taxes. 
Mint has not recorded income tax 
expense. There were no temporary or 
permanent differences, and Mint’s tax 
rate is 30%. 
In Mint’s December 31, 2010 balance sheet, 
what amount should be reported as
47. Total retained earnings? 
a. $1,950,000 
b. $2,110,000 
c. $2,400,000 
d. $2,560,000 
48. Total noncurrent liabilities? 
a. $1,620,000 
b. $1,780,000 
c. $2,320,000 
d. $2,480,000 
49. Total current assets? 
a. $5,000,000  50. Mirr,   Inc.   was   incorporated   on
b. $5,450,000  January 1, 2010, with proceeds from
c. $5,700,000  the issuance of $750,000 in stock and
d. $6,150,000  bor­ 
300 MODULE 9 BASIC TH. & FIN. REP.: D. FINANCIAL STATEMENTS

55. Dean Co. acquired 100% of Morey Corp. 
rowed funds of $110,000. During the first year  prior to 2010.During 2010, the individual 
of opera­tions, revenues from sales and  companies included in their financial statements
consulting amounted to $82,000, and operating  the following:
costs and expenses totaled $64,000. On  Dean Morey
December 15, Mirr declared a $3,000 cash  Officers’ salaries $  75,000 $50,000
dividend, payable to stockholders on January  Officers’ expenses 20,000 10,000
15, 2011. No additional activities affected  Loans to officers 125,000 50,000
owners’ equity in 2010. Mirr’s liabilities  Intercompany sales 150,000 ­­
increased to $120,000 by December 31, 2010.  What amount should be reported as related­
On Mirr’s December 31, 2010 balance sheet,  party disclosures in the notes to Dean’s 2010 
total assets should be reported at consolidated financial state­ments?
a. $885,000  a. $150,000 
b. $882,000  b. $155,000 
c. $878,000  c. $175,000 
d. $875,000  d. $330,000 
51. The following changes in Vel Corp.’s account
balances occurred during 2010: 
Increase
Assets $89,000
Liabilities 27,000
Capital stock 60,000
Additional paid­in capital 6,000
Except for a $13,000 dividend payment and the year’s earn­ings, there were no changes in retained 
earnings for 2010. What was Vel’s net income for 2010?
a. $  4,000 
b. $  9,000 
c. $13,000 
d. $17,000 
52. When preparing a draft of its 2010 balance sheet, Mont, Inc. reported net assets totaling 
$875,000. Included in the asset section of the balance sheet were the following: 
Treasury stock of Mont, Inc. at cost, which approxi­
mates market value on December 31 $24,000
Idle machinery 11,200
Cash surrender value of life insurance on corporate
executives 13,700
Allowance for decline in market value of noncurrent
equity investments 8,400
At what amount should Mont’s net assets be reported in the December 31, 2010 balance sheet?
a. $851,000 
b. $850,100 
c. $842,600 
d. $834,500 
53. In analyzing a company’s financial statements, which financial statement would a potential 
investor primarily use to assess the company’s liquidity and financial flexibility? 
a. Balance sheet. 
b. Income statement. 
c. Statement of retained earnings. 
d. Statement of cash flows. 
54. During 2010, Jones Company engaged in the following transactions: 
Salary expense to key employees who are also
principal owners $100,000
Sales to affiliated enterprises 250,000
Which of the two transactions would be disclosed as related­party transactions in Jones’ 2010 financial 
statements?
a. Neither transaction. 
b. The $100,000 transaction only. 
c. The $250,000 transaction only. 
d. Both transactions. 
56. Which type of material related­party transaction re­quires disclosure? 
a. Only those not reported in the body of the financial statements. 
b. Only those that receive accounting recognition. 
c. Those that contain possible illegal acts. 
d. All those other than compensation arrangements, expense allowances, and 
other similar items in the ordinary course of business. 
57. Financial   statements   shall   include   disclosures   of   ma­terial   transactions   between   related
parties except 
a. Nonmonetary exchanges by affiliates. 
b. Sales of inventory by a subsidiary to its parent. 
c. Expense   allowance   for   executives   which   exceed   normal   business
practice. 
d. A company’s agreement to act as surety for a loan to its chief executive
officer. 
58. Dex Co. has entered into a joint venture with an affiliate to secure access to additional inventory. Under 
the joint venture agreement, Dex will purchase the output of the venture at prices negotiated on an 
arm’s­length basis. 
Which of the following is(are) required to be disclosed about the related­party transaction? 
I. The amount due to the affiliate at the balance sheet date. II. The dollar amount of the
purchases during the year. 
a. I only. 
b. II only. 
c. Both I and II. 
d. Neither I nor II. 
59. What is the purpose of information presented in notes to the financial statements? 
a. To   provide   disclosures   required   by   generally   ac­cepted   accounting
principles. 
b. To correct improper presentation in the financial statements. 
c. To   provide   recognition   of   amounts  not  included   in   the   totals   of   the
financial statements. 
d. To present management’s responses to auditor comments. 
60. Which of the following information should be included in Melay, Inc.’s 2010 summary of 
significant accounting policies? 
a. Property, plant, and equipment is recorded at cost with depreciation 
computed principally by the straight­line method. 
b. During 2010, the Delay component was sold. 
MODULE 9 BASIC TH. & FIN. REP.: D. FINANCIAL STATEMENTS 301

c. Business   component   2010   sales   are   Alay   $1M,   Be­  66.    Which   of   the   following   describes   a
principal market for
lay $2M, and Celay $3M. issued, Colter lost $250,000 of 
d. Future   common   share inventory due to a fire. The inventory 
dividends are expected to was a total loss and was uninsured. 
approximate   60%   of How should Colter present this 
earnings.  information in its December 31, 2010 
financial statements? 
61. Which   of   the   following   information a. Colter   should   disclose   the
should be disclosed in the summary of loss   in   a   footnote   to   its
significant accounting policies?  2010 financial statements. 
a. Refinancing   of   debt b. Colter   should   report   an
subsequent   to   the extraordinary   loss   in   its
balance sheet date.  2010 income statement. 
b. Guarantees of indebtedness of others.  c. Colter   should
c. Criteria   for   determining report an allowance
which   investments   are for lost inventory in
treated   as   cash its   2010   balance
equivalents.  sheet. 
d. Adequacy   of   pension   plan d. Colter should not report the loss. 
assets   relative   to   vested
benefits.  64. Which   of   the   following   is   a   true
statement regarding disclosures for
62. Swift Corp. prepares its financial statements for  subsequent events? 
its fiscal year ending December 31, 2010. Swift  a. Recognize a loss for all 
estimates that its product warranty liability is  recognized and unrecognized
$28,000 at December 31, 2010. On February 12, subsequent events in the 
2011, before the financial statements were  current year financial 
issued, Swift received information about a  statements. 
product defect that will require a recall of all  b. Recognize a gain or 
units sold in 2010. It is expected the product  loss for any recognized 
recall will cost an additional $40,000 in  subsequent event in the
warranty repairs. What should Swift present in  current year financial 
its December 31, 2010 financial statements?  statements. 
a. A   footnote   disclosure   explaining   the c. Recognize a loss for a recognized 
product recall.  subsequent event in the financial 
b. A footnote disclosure listing statements in the year when the 
the   estimated   amount   of subsequent event occurs. 
$40,000 in warranty repairs d. Recognize   a   loss   for   a   recognized
and   an   explanation   of   the subsequent   event   in   the   current   year
recall.  financial statements. 
c. An   estimated   warranty   liability   of
$68,000.  65. The fair value of an asset should be based
d. No disclosure is necessary.  upon 
a. The replacement cost of an asset. 
63. Colter Corp. has a fiscal year­end of  b. The   price   that   would   be
December 31, 2010. On that date,  received   to   sell   the   asset   at
Colter reported total assets of  the measurement date. 
$600,000. On March 1, 2011, before  c. The   original   cost   of   the
the 2010 financial statements were  asset   plus   an   adjustment
for obsolescence.  establishing fair value of an asset?
d. The price that would be paid to acquire a. The   market   that   has   the
the asset.  greatest   volume   and   level
of activity for the asset. 
b. Any   broker   or   dealer
market that buys or sells the
asset. 
c. The   most   observable
market   in   which   the   price
of the asset is minimized. 
d. The   market   in   which   the   amount
received would be maximized. 
67. Which of the following is true for
valuing an asset to fair value? 
a. The price of the asset
should be adjusted for
transaction costs. 
b. The fair value of the asset
should   be   adjusted   for
costs to sell. 
c. The fair value price is based
upon   an   entry   price   to
purchase the asset. 
d. The price should be adjusted 
for transportation costs to 
transport the asset to its 
principal market. 
68. Which   of   the   following   would   meet
the   qualifications   as   market
participants in determining fair value? 
a. A   liquidation
market   in   which
sellers   are
compelled to sell. 
b. A subsidiary of the 
reporting unit interested in 
purchasing assets similar 
to those being valued. 
c. An independent entity that is
knowledgeable   about   the
asset. 
d. A   broker   or   dealer   that
wishes   to   establish   new
market for the asset. 
69. Which   of   the   following   is   an
assumption   used   in   fair   value
measurements? 
a. The asset must be in­use. 
b. The   asset   must   be   considered   in­
exchange. 
c. The   most   conservative   estimate   must
be used. 
d. The asset is in its highest and best use. 
70. The   fair   value   of   an   asset   at   initial
recognition is 
a. The price paid to acquire the asset. 
b. The price paid to acquire the
asset less transaction costs. 
c. The   price   paid  to  transfer  or  sell  the
asset. 
d. The book value of the asset acquired. 
71. Which   of   the   following   is   not   a
valuation   technique   used   in   fair
value estimates? 
a. Income approach. 
b. Residual value approach. 
c. Market approach. 
d. Cost approach. 
72. Valuation techniques for fair 
value that include the Black­
Scholes­Merton formula, a 
binomial model, or discounted 
cash flows are examples of which
valuation technique? 
a. Income approach. 
b. Market approach. 
c. Cost approach. 
d. Exit value approach. 
73. The   market   approach   valuation   technique   for
measuring   fair   value   requires   which   of   the
following? 
a. Present value of future cash flows. 
b. Prices and other relevant 
information of transactions 
from identical or 
comparable assets. 
302 MODULE 9 BASIC TH. & FIN. REP.: D. FINANCIAL STATEMENTS

c.   The price to replace the service capacity of the a. $ 0
asset. b. $ 20,000
d.   The weighted­average of the present value of c.
$600,000
future cash flows. d.
$ 620,000
information to the
74. A   change   in   valuation   techniques financial   state­
used to measure fair value should be ments. 
reported as  d. Management’s report to shareholders. 
a. A
change 78. Lewis Company was formed on 
in January 1, 2009. Se­lected balances 
accoun from the historical cost balance sheet 
ting at De­cember 31, 2010, were as 
princip follows: 
le with Land (purchased in 2009) $120,000
retrosp Investment in nonconvertible bonds (purchased in
ective 2009, and expected to be held to maturity) 60,000
restate Long­term debt 80,000
ment. 
The average Consumer Price Index was 100 
b. An   error
for 2009, and 110 for 2010. In a 
correction supplementary constant dollar balance sheet 
with (adjusted for changing prices) at December 
restatement 31, 2010, these selected account balances 
of   the should be shown at
financial
Land Investment Long­term debt
statements
a. $120,000 $60,000 $88,000
of   previous b. $120,000 $66,000 $88,000
periods.  c. $132,000 $60,000 $80,000
c. A   change   in d. $132,000 $66,000 $80,000
accounting
estimate 79. The following items were among those that 
reported   on   a appeared onRubi Co.’s books at the end of 
prospective 2010:
basis.  Merchandise inventory $600,000
d. An   extraordinary Loans to employees 20,000
item on the current What amount should Rubi classify as 
year’s   income monetary assets in preparing constant dollar
statement.  financial statements?
75. When   measuring   fair   value,   which
level   has   the   highest   priority   for
valuation inputs? 
a. Level 1. 
b. Level 2. 
c. Level 3. 
d. Level 4. 
76. Which   of   the   following   are
observable inputs used for fair value
measurements? 
I. Bank prime rate. 
II.  Default rates on loans.
BI. Financial forecasts. 
a. I only. 
b. I and II only. 
c. I and III only. 
d. I, II and III. 
77. A company that wishes to disclose 
information about the effect of 
changing prices should report this 
information in 
a. The body of the financial statements. 
b. The notes to the financial statements. 
c. Supplementary
80. In its financial statements, Hila Co.  d. A purchasing power loss, if
discloses supple­mental information on the  the   item   is   a   monetary
effects of changing prices. Hila computed the  liability. 
increase in current cost of inventory as fol­
83. The following information pertains to
lows:
each  unit  of  mer­chandise  purchased
Increase in current cost (nominal dollars) for resale by Vend Co.: 
Increase in current cost (constant dollars)
March 1, 2010
What amount should Hila disclose as the  Purchase price $  8
inflation compo­nent of the increase in  Selling price $12
current cost of inventories? Price level index 110
a. $  3,000  December 31, 2010
b. $12,000  Replacement cost $10
c. $15,000  Selling price $15
Price level index 121
d. $27,000 
Under current cost accounting, what is the 
81. When computing purchasing power  amount of Vend’s holding gain on each unit of 
gain or loss on net monetary items,  this merchandise?
which of the following accounts is  a. $0 
classi­fied as nonmonetary?  b. $0.80 
a. Advances   to   unconsolidated c. $1.20 
subsidiaries.  d. $2.00 
b. Allowance for uncollectible accounts. 
c. Unamortized   premium   on   bonds 84. Kerr Company purchased a machine 
payable.  for $115,000 on January 1, 2010, the 
d. Accumulated   depreciation   of company’s first day of operations. At
equipment.  the end of the year, the current cost 
of the machine was $125,000. The 
82. During a period of inflation in which a machine has no salvage value, a five­
liability   account   balance   remains year life, and is depreciated by the 
constant, which of the following occurs? straight­line method. For the year 
a. A purchasing power gain, if the item is ended December 31, 2010, the 
a nonmone­tary liability.  amount of the current cost 
b. A purchasing power gain, if depreciation expense which would 
the   item   is   a   monetary appear in supple­mentary current cost
liability.  financial statements is 
c. A   purchasing   power   loss,   if a. $14,000 
the   item   is   a   nonmone­tary b. $23,000 
liability.  c. $24,000 
d. $25,000 
MODULE 9 BASIC TH. & FIN. REP.: D. FINANCIAL STATEMENTS 303
purchas
ed less 
85. At December 31, 2010, Jannis Corp. owned ending 
two assetsas follows: inventor
Equipment Inventory y at 
Current cost $100,000 $80,000 current 
Recoverable amount $  95,000 $90,000 cost. 
Jannis voluntarily disclosed supplementary  88. Which of the following are examples 
information about current cost at December 
of concentrations that create 
31, 2010. In such a disclo­sure, at what 
vulnerabilities and therefore would 
amount would Jannis report total assets?
require dis­closure of risks and 
a. $175,000 
uncertainties? 
b. $180,000 
c. $185,000  I. Market   in   which   an   entity   conducts   its
d. $190,000  operations. 
AI. Available sources of supply of materials used
86. Could current cost financial statements  in operations of an entity. 
report holdinggains for goods sold during the  BI. Volume of business transacted with a certain
period and holding gains on inventory at the  contributor. 
end of the period? a. I and II. 
Goods sold Inventory b. II and III. 
a. Yes Yes c. I and III. 
b. Yes No d. I, II, and III. 
c. No Yes
d. No No 89. Which of the following is required
to be disclosed re­garding the risks
87. Manhof Co. prepares supplementary  and uncertainties that exist? 
reports on income from continuing  a. Factors   causing   an   estimate   to   be
operations on a current cost basis.  sensitive. 
How should Manhof compute cost of  b. The 
goods sold on a current cost basis?  potential 
a. Number impact of 
of   units estimates 
sold times
about 
average
values of 
current
assets and
cost   of
units liabilities 
during the when it is 
year.  reasonabl
b. Number y possible
of   units that the 
sold estimate 
times will 
current change in 
cost   of the near 
units   at future. 
year­end.  c. The
c. Number potential
of   units impact
sold of
times estimate
current s   about
cost   of values of
units   at
assets
the
beginning and
of   the liabilitie
year.  s when it
d. Beginni is
ng  remotely
inventor possible
y at  that   the
current  estimate
cost  will
plus  change
cost of  in   the
goods  near
future. 
d. A I. Basis of accounting used by an entity to
descri file its income tax return. 
ption AI. Cash   receipts   and   disbursements   basis   of
of   the accounting. 
operati a. I only. 
ons b. II only. 
both c. Both I and II. 
within d. Neither I nor II. 
and
outsid 91. Income tax basis financial statements 
e   of differ from those prepared under 
the GAAP in that income tax basis 
home financial statements 
countr a. Do  not
y.  include
90. Which of the following accounting  nontaxa
bases may be used to prepare financial  ble
statements in conformity with a  revenue
compre­hensive basis of accounting  s   and
other than generally accepted  nonde­
accounting principles?  ductible
expense
s   in
determi
ning
income.
b.Include
detailed
informati
on   about
current
and   de­
ferred
income
tax
liabilities.
c. Contain
no
disclosure
s   about
capital
and
operating
lease
transactio
ns. 
d. Recognize   certain   revenues   and
expenses   in   differ­ent   reporting
periods. 
92. In   financial   statements   prepared   on
the income tax ba­sis, how should the
nondeductible   portion   of   expenses
such   as   meals   and   entertainment   be
reported? 
a. Included
in   the
expense
category
in   the
determina
­tion   of
income. 
b. Include
d   in   a
separate
category
in   the mod
determin ifica
a­tion  of tion
income.  s
c. Excluded   from   the   determination   of mus
income   but   in­cluded   in   the t   be
determination of retained earnings.  the
d. Excluded   from   the   financial sam
statements.  e   as
thos
93. If a company uses the modified cash  e
basis of account­ing, the  requ
modifications from the pure cash  ired
basis should have substantial support.  by
In this context substantial support re­ tax
quires  law.
a. T c. The
h modificati
e ons   must
fi be   the
n same   as
a GAAP
n and   not
c illogical. 
i d. No modifications are allowed. 
a
l 94. Which of the following is false? 
st a. Prospecti
a ve
t financial
e informati
m on may be
e prepared
n for
ts general  or
h limited
a users. 
v b. The   responsible   party   is   the   only
e limited user. 
o c. The
n financial
l projectio
y n   may
m contain
i assumpti
n ons   not
o necessari
r ly
m expected
o to occur. 
d d. The
if financ
i ial
c projec
a tion
ti may
o be
n expres
s sed   as
fr a
o range
m of
  dollar
G s. 
A
95. Prospective   financial   information   is
A
P defined as 
.  a. Any
b. The financial
informati
on   about
the   past,
present,
or future. 
b. Any
financial
informati
on   about
the
present or
fu­ture. 
c. Any
financial
informati
on   about
the future
related   to
the   day­
to­day
operation
s. 
d. Any   financial   information   about   the
future. 
96. To   achieve   a   reasonably   objective
basis,   financial   fore­casts   and
projections should be prepared 
I. In accordance with GAAP. 
AI. Using   information   that   is   in
accordance with the plans of
the entity. 
III.  With due professional care.
304 MODULE 9 BASIC TH. & FIN. REP.: D. FINANCIAL STATEMENTS

a. I and III. 103.  Which financial statements should be presented for a
b. II and III. trust?
c. I, II, and III. I. Statement of assets and liabilities.
d. I and II. II. Statement of operations.
97.  Which of the following disclosures should prospective III. Statement of cash flows.
financial statements include? IV. Statement of changes in net assets.
Summary of significant Summary of a. I only.
accounting policies significant assumptions b.   I and II only.
a. Yes Yes c.   I, II, and IV.
b. Yes No d.   I, III and IV.
c. No Yes
d. No No 104.  What is the basis of accounting and at what amount
are assets measured on the financial statements of a trust?
98.  Which of the following is the SEC form used by issuer
Basis of accounting Measurement
companies to file as an annual report with the SEC?
a. Cash Cost
a. Form 10­Q.
b. Cash Fair value
b. Form 8­K. c. Accrual Cost
c. Form 10­K. d. Accrual Fair value
d. Form S­1. 
preceding fiscal 
99. Which of the following best describes years. 
the content of the SEC Form 10­Q?  c. The end of the preceding fiscal year. 
a. Quarterly   audited   financial d. The   end   of   the
information   and   other preceding   fiscal
information   about   the year and the end of
company.  the prior two fiscal
b. Annual audited financial years. 
information   and   non­
financial   information 102. A company is an accelerated filer that
about the company.  is required to file Form 10­K with the
c. Disclosure   of   material United States Securities and 
events that affect the com­ Exchange Commission (SEC). What 
pany.  is the maximum number of days after
d. Quarterly   reviewed   financial the company’s fiscal year­end that 
information   and   other the company has to file Form 10­K 
information   about   the with the SEC? 
company.  a. 60 days. 
b. 75 days. 
100. Which   of   the   following   is   the   SEC
c. 90 days. 
form used by issuer companies to file
d. 120 days. 
as a quarterly report with the SEC? 
a. Form 10­Q. 
b. Form 8­K. 
c. Form 10­K. 
d. Form S­1. 
101. A company is required to file quarterly financial 
statements with the United States Securities and 
Exchange Commission on Form 10­Q. The 
company operates in an industry that is not 
subject to seasonal fluctuations that could have a
significant impact on its financial condition. In 
addition to the most recent quarter­end, for 
which of the following periods is the company 
required to present balance sheets on Form 10­
Q? 
a. The   end   of   the
corresponding
fiscal   quarter   of
the   preceding
fiscal year. 
b. The end of the 
preceding fiscal 
year and the end of 
the corresponding 
fiscal quarter of the
105. Galaxy Corporation prepares its financial  c. Operating
statements in accordance with IFRS. Galaxy  expenses,
intends to refinance a $10,000 note payable due  nonoperating
on February 20, 2011. The com­pany expects  expenses,   and
the note to be refinanced for a period of five  extraordinary
years. Under what circumstances can Galaxy  items. 
report the note payable as a noncurrent liability  d. Gross   profit,   operating   profits,   and
on its December 31, 2010 statement of financial  net profits. 
position? 
107. Which of the following may not be
a. If Galaxy has the intent and
ability   to   refinance   be­fore disclosed   on   the   income   statement
December 31, 2010.  for   a   company   that   prepares   its
financial   statements   in   accordance
b. If Galaxy has  executed  an
with IFRS? 
agreement   to   refinance
before December 31, 2010.  a. Gain or loss. 
c. If   Galaxy   has   executed   an b. Tax expense. 
agreement   to   refinance c. Gain or loss from extraordinary items.
prior to the issuance of the d. Gain   or   loss   from   discontinued
financial   statements   in operations. 
March 2011.  108. Glenda   Corporation   prepares   its   financial
d. If Galaxy has the intent and  statements   in   accordance   with   IFRS.   Glenda
ability to refinance be­fore  must   report   finance   costs   on   the   statement   of
the issuance of the financial  cash flows 
statements in March 2011.  a. In operating activities. 
106. Largo   Corporation   prepares   its b. Either   in
financial   statements   in   accordance operating
with   IFRS.   Which   of   the   following activities   or
items   is   re­quired   disclosure   on   the financing   activi­
income statement?  ties. 
a. Revenues,   cost   of c. In financing activities. 
goods   sold,   and d. In   investing   activities   or   financing
advertising   ex­ activities. 
pense.  109. Larimer   Corporation   prepares   its
b. Finance   costs,   tax   expense,   and
financial statements in accordance with
income. 
IFRS. Larimer acquired equipment by 
MODULE 9 BASIC TH. & FIN. REP.: D. FINANCIAL STATEMENTS 305

issuing 5,000 shares of its 
common stock. How should this 
transaction be reported on the 
statement of cash flows?
a. As an outflow
of   cash   from
investing
activities   and
inflow of cash
from financing
activities. 
b. As   an   inflow   of
cash   from
financing
activities   and   an
outflow   of   cash
from   operating
activities. 
c. At   the   bottom
of the statement
of cash flows as
a   significant
noncash
transaction. 
d. In the notes to
the   financial
statements   as
a   sig­nificant
noncash
transaction. 
110. For IFRS purposes, cash 
advances and loans from 
bank overdrafts should 
be reported on the 
statement of cash flows 
as 
a. Operating activities. 
b. Investing activities. 
c. Financing activities. 
d. Other significant noncash activities. 
111. Which   of   the
following   are
acceptable   methods
for   reporting
comprehensive
income under IFRS? 
I. One comprehensive income statement. 
AI. Two   statements:   an
income   statement
and   a   com­
prehensive   income
statement. 
BI. In the statement of owner’s equity. 
a. I only. 
b. I and II only. 
c. I, II, and III. 
d. I and III only. 
112. Which   of   the
following   is   true
about   financial
statement
requirements
under IFRS? 
a. Prior   year
comparative
financial
statements
are required.
b. Income statements for three years are required. 
c. Balance sheets for three years are required. 
d. There   are
no specific
requireme
nts
regarding
comparati
ve
financial
statements

113. Under   IFRS,
operating expenses
on   the   income
statement   may   be
classified by 
Nature Function
a. Yes Yes
b. Yes No
c. No Yes
d. No No
114. Under IFRS, the 
statement of cash flows may 
bepresented on the
Direct Basis Indirect Basis
a. Yes Yes
b. Yes No
c. No Yes
d. No No
306 MODULE 9 BASIC TH. & FIN. REP.: D. FINANCIAL STATEMENTS

SIMULATIONS

Task­Based Simulation 1
Concepts
Authoritative
Literature Help

Situation
Lim Corporation is preparing its financial statements and has given the project to its new entry­level accountant, Sam.
Indicate whether each of the following statements made by Sam is True or False.

1. T
h
2. T
h
3. E

4. A
 
5. A
 
6. O

7. S
e
8. P
r
Task­Based Simulation 2
Financial Statement
Classification Authoritative
Literature Help

The outline presented below represents the various classifications suggested by the chief accountant for the balance 
sheet.
Assets Liabilities Owner’s equity Other
A. Current G. Current J. Preferred stock N. Items excluded from the
B. Investments H. Long­term K. Common stock balance sheet
C. Plant and equipment I. Other liabilities L. Paid­in capital excess of par X. Contra valuation account
D. Intangibles M. Retained earnings
E. Deferred charges 
F. Other assets 
Items 1 through 18 represent accounts of the Craven Corporation. Determine how each account would be classified 
fromthe list above. If the account is a contra or valuation account, mark “X” before the letter. For example: “Allowance 
for Doubtful Accounts” would be “X–A.” An answer may be selected once, more than once, or not at all.
Items to be answered
Assets Liabilities Owner’s equity Other
(A­F) (G, H, I) (J, K, L, M) (N or X)
1. Dividend payable (on Craven’s preferred stock). 
2. Plant construction in progress by the company. 
3. Factory building (retired from use and held for sale). 
4. Land (held for possible future building site). 
5. Merchandise   inventory   (held   by   Craven
Corporation on consignment). 
6. Stock dividend distributable (in common stock to
common stockholders and to be issued at par). 
7. Office supplies inventory. 
MODULE 9 BASIC TH. & FIN. REP.: D. FINANCIAL STATEMENTS 307

Assets Liabilities Owner’s equity Other


(A­F) (G, H, I) (J, K, L, M) (N or X)
8. Sinking fund cash (First National Bank, Trustee). 
9. Installment   sales   accounts   receivable   (average
collection period eighteen months). All sales are
installment sales. 
10. Temporary decline in inventory value. 
11. Advances to officers (indefinite repayment date). 
12. Estimated warranty costs. The warranty costs are
for a one­year warranty on parts and labor. 
13. Inventory of small tools used in the business. 
14. Treasury stock under par value method. 
15. Common stock subscribed (Craven Corporation’s stock). 
16. Convertible bonds. 
17. Securities held as collateral. 
18. Bank overdraft (only account with bank). 

Task­Based Simulation 3
Balance Sheet
Authoritative
Literature Help

Situation
You have been asked to assist the chief accountant of the Stephen King Corporation in the preparation of a 
balance sheet. Presented below is the balance sheet of Stephen King Corporation for the current year, 2011.
Stephen King Corporation
BALANCE SHEET
December 31, 2011

Current assets $ 435,000
Investments 640,000
Property, plant, and equipment 1,720,000
Intangible assets 305,000
$3,100,000
Current liabilities $ 330,000
Long­term liabilities 1,000,000
Stockholders’ equity 1,770,000
$3,100,000

Consider the following information:
1. The current assets section includes: cash $100,000, accounts receivable $170,000 less $10,000 for allowance 
for doubtful accounts, inventories $180,000, and unearned revenue $5,000. The cash balance is composed of 
$114,000, less a bank overdraft of $14,000. Inventories are stated on the lower of FIFO cost or market. 
2. The investments section includes: the cash surrender value of a life insurance contract $40,000; investment in 
com­mon stock, short­term (trading) $80,000 and long­term (available­for­sale) $270,000; and bond sinking 
fund $250,000. The cost and fair value of investments in common stock are the same. 
3. Property,   plant,   and   equipment   includes:   buildings   $1,040,000   less   accumulated   depreciation   $360,000;
equipment $450,000 less accumulated depreciation $180,000; land $500,000; and land held for future use
$270,000. 
4. Intangible assets include:  a franchise $165,000; goodwill $100,000; and discount on bonds payable $40,000. 
5. Current liabilities include: accounts payable $90,000; notes payable—short term $80,000 and long­term
$120,000; and taxes payable $40,000. 
6. Long­term liabilities are compose solely of 10% bonds payable due in 2020. 
7. Stockholders’ equity has: preferred stock, no par value, authorized 200,000 shares, issued 70,000 shares for 
$450,000; and common stock, $1.00 par value, authorized 400,000 shares, issued 100,000 shares at an average 
price of $10. In addition, the corporation has retained earnings of $320,000. 
8. The company’s management does not elect to use the fair value option for any of its financial assets or liabilities.
Complete the corrected balance sheet.  To make it a more realistic exam experience use Excel.
308 MODULE 9 BASIC TH. & FIN. REP.: D. FINANCIAL STATEMENTS

A B C D E
1 Stephen King Corporation
2 Balance Sheet
3 December 31, 2011
4
5 Current assets:
6 Cash
7 Trading securities
8 Accounts receivable (net of $xxx allowance for doubtful accounts)
9 Inventories (lower of FIFO cost or market)
10
11 Total current assets
12
13 Investments:
14 Available­for­sale securities
15 Bond sinking fund
16 Land held for future use
17 Cash surrender value of life insurance contract
18
19 Total investments
20
21 Property, plant, and equipment:
22 Land
23 Buildings (net of accumulated depreciation of $xxx)
24 Equipment (net of accumulated depreciation of $xxx)
25
26 Total property, plant, and equipment
27
28 Intangible assets:
29 Franchise
30 Goodwill
31
32 Total intangible assets
33 Total assets
34
35 Current liabilities:
36 Accounts payable
37 Notes payable
38 Taxes payable
39 Bank overdraft
40 Unearned revenue
41
42 Total current liabilities
43
44 Long­term liabilities:
45 Notes payable
46 Bonds payable, 10% due in 2020 (less discount of $xxx)
47
48 Total long­term liabilities
49 Total liabilities
50
51 Stockholders’ equity:
52 Paid­in capital
53 Preferred stock, no par, authorized xxx shares, issued xxx shares
54 Common stock, $1.00 par value, authorized xxx shares, issued xxx shares
55 Paid­in capital in excess of par value common
56
57 Total paid­in capital
58
59 Retained earnings
60
61 Total stockholders’ equity
62 Total liabilities and stockholders’ equity

MODULE 9 BASIC TH. & FIN. REP.: D. FINANCIAL STATEMENTS 309

Task­Based Simulation 4
Research
Authoritative
Literature Help

Assume that you are assigned to the audit of Russell Corporation. The CFO of Russell is trying to determine
how to classify items within comprehensive income. Which section of the Professional Standards addresses the
issue of how to classify items in comprehensive income?
Enter your response in the answer fields below.

Task­Based Simulation 5
Journal
Entries
Authoritative
Literature Help

Situation
Hillside had the following selected account balances as of December 31, 2010.
Accounts receivable $250,000
Notes receivable 75,000
Prepaid rent 168,000
Supplies 60,000
Inventory 420,000
Equipment (historical cost) 640,000
Accounts payable 176,000
Salaries payable 15,000
Accumulated depreciation 174,000
The following information was received from Hillside’s accountant.  Adjusting entries have not yet been made.
1. It is estimated that $16,000 of accounts will not be collectible. A provision for uncollectible accounts has
never been made by Hillside. 
2. Supplies remaining at the end of the year were $37,000. 
3. Equipment is depreciated over 20 years with a $60,000 salvage value. 
4. Accrued salaries at 12/31/10 were $37,500. 
5. The note receivable was signed by the customer on November 1, 2010. It is a 6­month note with an interest
rate of 12%, with the principle and interest paid at maturity. 
6. Rent was paid on August 1, 2010, for 24 months and recorded in a prepaid rent account. 
7. Hillside does not elect to use the fair value option for any of its financial assets or liabilities. 
Prepare the adjusting journal entries necessary for each item.  If no entry is necessary, write “no entry.”

Task­Based Simulation 6
Calculations
Authoritative
Literature Help

Situation
Hillside had the following selected account balances as of December 31, 2010.
Accounts receivable $250,000
Notes receivable 75,000
Prepaid rent 168,000
Supplies 60,000
Inventory 420,000
Equipment (historical cost) 640,000
Accounts payable 176,000
Salaries payable 15,000
Accumulated depreciation 174,000
The following information was received from Hillside’s accountant.  Adjusting entries have not yet been made.
1. It is estimated that $16,000 of accounts will not be collectible. A provision for uncollectible accounts has
never been made by Hillside. 
310 MODULE 9 BASIC TH. & FIN. REP.: D. FINANCIAL STATEMENTS

2. Supplies remaining at the end of the year were $37,000. 
3. Equipment is depreciated over 20 years with a $60,000 salvage value. 
4. Accrued salaries at 12/31/10 were $37,500. 
5. The note receivable was signed by the customer on November 1, 2010. It is a 6­month note with an interest rate of
12%, with the principle and interest paid at maturity. 
6. Rent was paid on August 1, 2010, for 24 months and recorded in a prepaid rent account. 
7. Hillside does not elect to use the fair value option for any of its financial assets or liabilities. 
Determine the adjustments necessary for December 31, and indicate the adjusted balances of the selected accounts at
December 31, 2010.
Accounts receivable (net)
Notes receivable
Prepaid rent
Supplies
Inventory
Equipment
Accounts payable
Salaries payable
Accumulated depreciation

Task­Based Simulation 7
Classification/ Authoritative
Disclosures Literature Help

Griffin Co. is in the process of preparing its financial statements for the year ended December 31, 2011.
Items 1 through 6 represent various transactions that occurred during 2011. The following two responses are required 
foreach item:
Compute the amount of gain, loss, or adjustment to be reported in Griffin’s 2011 financial statements. Disregard 
income taxes. On the CPA exam, a list of numeric answer choices would be provided to select from.
Select from the list below the financial statement category in which the gain, loss, or adjustment should be 
presented. A category may be used once, more than once, or not at all.
Financial Statement Categories
A. Income from continuing operations. 
B. Extraordinary item. 
C. Cumulative effect of change in accounting principle. 
D. Prior period adjustment to beginning retained earnings. 
E. Separate component of other comprehensive income. 
C
o
1. On June 30, 2011, after paying the 
semiannual interest due and recording  I
am­ortization of bond discount, Griffin  n
redeemed its fifteen­year, 8% 
$1,000,000 par bonds at 102. The bonds,
which had a carrying amount of 
$940,000 on January 1, 2011, had 
originally been issued to yield 10%. 
Griffin uses the effective interest 
method of amortization, and had paid 
interest and recorded amortization on 
June 30. Compute the amount of gain or
loss on redemption of the bonds and 
select the proper financial statement 
category. 
2. As of January 1, 2011, Griffin 
decided to change the method of 
computing depreciation on its sole
piece of equipment from the sum­
of­the­years’ digits method to the 
straight­line method. The 
equipment, acquired in January 
2008 for $520,000, had an 
estimated life of five years and a 
salvage value of $20,000. 
(A) (B) (C) (D) (E)
MODULE 9 BASIC TH. & FIN. REP.: D. FINANCIAL STATEMENTS 311

A
m
4. In November 2011, Griffin 
purchased two marketable 
securities, I and II, which it 
bought and held principally to 
sell in the near term by February  _
28, 2012. Relevant data is as  _
follows: 
Cost
I $125,000
II 235,000
Compute the amount of holding gain
or loss at December 31, 2011, and 
select the proper financial statement 
category, assuming Griffin classifies _
the securities as trading securities. _
5. During 2011, Griffin received 
$1,000,000 from its insurance 
company to cover losses suffered
during a hurricane. This was the 
first hurricane ever to strike in  _
Griffin’s area. The hurricane  _
destroyed a warehouse with a 
carrying amount of $470,000, 
containing equipment with a 
carrying amount of $250,000, 
and inventory with a carrying 
amount of $535,000 and a fair 
value of $600,000. Compute the 
amount of gain or loss from the 
financial statement category. 
6. At December 31, 2011, Griffin 
prepared the following worksheet 
summarizing the translation of its 
wholly owned foreign subsidiary’s 
financial statements into dollars. Griffin
purchased the foreign subsidiary for 
$324,000 on January 2, 2011. On that 
date, the carrying amounts of the 
subsidiary’s assets and liabilities 
equaled their fair values. 

currency amounts
Net assets January 2, 2011
(date of purchase) 720,000
Net income, 2011 250,000
Net assets at December 31, 2011 970,000
Net assets at December 31, 2011 970,000
Compute the amount of the foreign currency translation adjustment and 
select the proper financial statements category.

Task­Based Simulation 8
Research
Authoritative
Literature Help

Assume that you are assigned to the audit of Jane Corporation. Jane has committed itself to a formal plan for sale of a 
business component that meets the requirements for presentation as discontinued operations. Which section of the 
Professional Standards addresses the issue of how to account for the costs that will be incurred to relocate employees of the 
discontinued component?
Enter your response in the answer fields below.

Task­Based Simulation 9
Classifications
Authoritative
Literature Help

The illustrations below represent accounting transactions that affect the recognition of income for an accounting 
period. Their classification is the subject of this objective format matching question.
For each of the ten illustrations below, select the best classification from those listed A–I below. A classification may
be used once, more than once, or not at all.
Classification
A. Change in reporting entity F. Discontinued Operations—Gain or loss from dis­
B. Correction of an error continued operations
C. Change in accounting principle G. Not an accounting change
D. Change in estimate H. Part of net income before extraordinary items
E. Extraordinary item I.   Discontinued Operations—Gain or loss on disposal
312 MODULE 9 BASIC TH. & FIN. REP.: D. FINANCIAL STATEMENTS

(A) (B) (C) (D) (E) (F) (G) (H)


(I)
1. Newly acquired assets are depreciated using the sum-of-the-
years’ digits method, previously recorded assets are depreciated
using the straight-line method.
2. Accounting for acquisition of a 100% owned subsidiary.
3. Reported as a restatement of all periods presented.
4. Write-down of inventory due to obsolescence.
5. Gains or losses on the disposal of the net assets of a
component are included in this calculation.
6. Changing from the gross profit method for
determining year-end inventory balances to dollar
value LIFO.
7. Accounting for existing construction contracts is
changed from completed contract to percentage-of-
completion.
8. The effects of a change in estimate and a change in
principle are inseparable for the same event.
9. The excess of cash paid over the carrying value to extinguish bonds.
10. Income or loss of the component for the period of disposal included in this
calculation.

Task­Based Simulation 10
Multistep Income
Statement Authoritative
Literature Help

Situation
Presented below is information related to American Horse Company for 2010.
Retained earnings balance, January 1, 2010 $   980,000
Sales for the year 25,000,000
Cost of goods sold 17,000,000
Interest revenue 70,000
Selling and administrative expenses 4,700,000
Write­off of goodwill (not tax deductible) 820,000
Income taxes for 2010 905,000
Gain on the sale of investments (normal recurring) 110,000
Loss due to flood damage—extraordinary item (net of tax) 390,000
Loss on the disposition of the wholesale division 615,000
Loss on operations of the wholesale division 200,000
Income tax benefit from discontinued wholesale division 285,000
Dividends declared on common stock 250,000
Dividends declared on preferred stock 70,000

American Horse Company decided to discontinue its entire wholesale operations and to retain its
manufacturing operations. On September 15, American Horse sold the wholesale operations to Rogers
Company. During 2010, there were 300,000 shares of common stock outstanding all year.
Prepare a multistep income statement.
American Horse Company
INCOME STATEMENT
For the Year Ended December 31, 2010

Sales $25,000,000

Net income
MODULE 9 BASIC TH. & FIN. REP.: D. FINANCIAL STATEMENTS 313

Earnings per share
Net income

Task­Based Simulation 11
Research
Authoritative
Literature Help

Assume that you are assigned to the audit of Clark Corporation. Clark has incurred a 
significant loss that may meet the definition of an extraordinary item. Which section of the 
Professional Standards provides guidance on the definition of an extraordinary item?
Enter your response in the answer fields below.

Task­Based Simulation 12
Financial Statement Authoritative
Classification Literature Help

Select from the list of financial statement categories below the category in which the item should be
presented. A financial statement category may be selected once, more than once, or not at all. Assume
management does not elect to use the fair value option for any financial assets or liabilities.
Financial Statement Categories
A. Income from continuing operations, with no separate disclosure. 
B. Income from continuing operations, with separate disclosure (either on the face of statement or
in the notes). 
C. Other comprehensive income for the period. 
D. Extraordinary items. 
E. Separate component of stockholders’ equity. 
F. None of the above categories include this item. 

I
I
t
1. A
n
2. T
h
3. I
n
4. A
 
5. A
 
6. A
7. T
 
h
8. T
h
314 MODULE 9 BASIC TH. & FIN. REP.: D. FINANCIAL STATEMENTS

Task­Based Simulation 13
Calculate Net Income and Authoritative
Earnings Per Share Literature Help

Situation

Rap Corp. has 100,000 shares of common stock outstanding. In 2010, the company reports income 
from continuing opera­tions before taxes of $1,210,000.
Additional transactions not considered in the $1,210,000 are as follows:
1. In 2010, Rap Corp. sold equipment for $40,000. The machine had originally cost $80,000
and had accumulated de­preciation of $36,000. The gain or loss is considered ordinary. 
2. The company discontinued operations of one of its subsidiaries during the current year at a
loss   of   $190,000   before   taxes.   Assume   that   this   transaction   meets   the   criteria   for
discontinued operations. The loss of operations of the dis­continued subsidiary was $90,000
before taxes; the loss from disposal of the subsidiary was $100,000 before taxes. 
3. In  2010,  the company  reviewed its  accounts  receivable  and  determined that $26,000 of
accounts receivable that had been carried for years appeared unlikely to be collected. 
4. An   internal   audit   discovered   that   amortization   of   intangible   assets   was   understated   by
$35,000 (net of tax) in a prior period. The amount was charged against retained earnings. 
5. The   company   sold   its   only   investment   in   common   stock   during   the   year   at   a   gain   of
$145,000. The gain is taxed at a total effective rate of 40%. Assume that the transaction
meets the requirements of an extraordinary item. 
Complete the table below for the calculation of net income and earnings per share. Assume the 
income tax rate is 38% for income from continuing operations.
Rap Corp.
INCOME STATEMENT
For the Year Ended December 31, 2010
Income from continuing operations before income tax
Income tax
Income from continuing operations
Discontinued operations:

Income before extraordinary item
Extraordinary item:

Net income
Per share of common stock:
Income from continuing operations
Discontinued operations, net of tax

Income before extraordinary item
Extraordinary item, net of tax
Net income
MODULE 9 BASIC TH. & FIN. REP.: D. FINANCIAL STATEMENTS 315

MULTIPLE­CHOICE ANSWERS

1. d __ __ 21. c __ __ 41. d __ __ 61. c __ __ 81. d __ __ 101. c __ __


2. a __ __ 22. c __ __ 42. b __ __ 62. c __ __ 82. b __ __ 102. b __ __
3. a __ __ 23. c __ __ 43. d __ __ 63. a __ __ 83. d __ __ 103. c __ __
4. a __ __ 24. b __ __ 44. d __ __ 64. d __ __ 84. c __ __ 104. d __ __
5. a __ __ 25. b __ __ 45. c __ __ 65. b __ __ 85. a __ __ 105. b __ __
6. d __ __ 26. a __ __ 46. d __ __ 66. a __ __ 86. a __ __ 106. b __ __
7. a __ __ 27. d __ __ 47. b __ __ 67. d __ __ 87. a __ __ 107. c __ __
8. a __ __ 28. d __ __ 48. a __ __ 68. c __ __ 88. d __ __ 108. b __ __
9. b __ __ 29. c __ __ 49. c __ __ 69. d __ __ 89. b __ __ 109. d __ __
10. a __ __ 30. b __ __ 50. a __ __ 70. a __ __ 90. c __ __ 110. a __ __
11. a __ __ 31. a __ __ 51. b __ __ 71. b __ __ 91. d __ __ 111. b __ __
12. c __ __ 32. c __ __ 52. a __ __ 72. a __ __ 92. a __ __ 112. a __ __
13. c __ __ 33. b __ __ 53. a __ __ 73. b __ __ 93. c __ __ 113. a __ __
14. d __ __ 34. d __ __ 54. c __ __ 74. c __ __ 94. b __ __ 114. a __ __
15. b __ __ 35. c __ __ 55. c __ __ 75. a __ __ 95. d __ __
16. d __ __ 36. d __ __ 56. d __ __ 76. b __ __ 96. c __ __
17. a __ __ 37. b __ __ 57. b __ __ 77. c __ __ 97. a __ __
18. b __ __ 38. d __ __ 58. c __ __ 78. c __ __ 98. c __ __
19. b __ __ 39. b __ __ 59. a __ __ 79. b __ __ 99. d __ __ 1st: __/114 = __%
20. d __ __ 40. a __ __ 60. a __ __ 80. a __ __ 100. a __ __ 2nd: __/114 = __%

MULTIPLE­CHOICE ANSWER EXPLANATIONS
$54,000) must also be deducted ($180,000 – 
D.1.  Income and Retained Earnings  $54,000 = $126,000).
Statements
1. (d) Baer Food’s 2010 revenues should 
include net sales revenue ($187,000), 
interest revenue ($10,200), and gain on
sale of equipment ($4,700), for a total 
of $201,900. Discontinued operations 
(loss of $12,400) and the extraordi­
nary gain ($1,500) are both special 
items that should be re­ported as 
separate components of income, after 
incomefrom continuing operations. 
Therefore, these items should not be 
included in the revenues section of the 
income state­ment (which is placed 
before income from continuing op­
erations). 
2. (a) To directly compute cost of goods
manufactured (CGM), the formula is 
Beginning work in process
+ Direct materials used 
+ Direct labor 
+ Factory overhead 
– Ending work 
in processCost
of goods 
manufactured

However, none of these elements are given in 
this problem, so CGM must be computed 
indirectly, using the cost of sales formula
Beginning finished goods $400,000
+ Cost of goods manufactured +  CGM
– Ending finished goods –360,000
Cost of sales $240,000

Solving for the missing amount, CGM is 
$200,000.
3. (a) All of the revenues, gains, expenses, and 
lossesgiven in this problem are components of 
income from con­tinuing operations. Income 
before income taxes is $180,000, as computed 
below.
Revenues ($575,000 + $25,000) $600,000
Expenses and losses ($240,000 + $70,000 + $10,000
+ $50,000 + $15,000 + $15,000 + $20,000)
Income before income taxes $
To compute income from continuing operations 
(after taxes), income taxes ($180,000 × 30% = 
4. (a) The requirement is to compute the amount of Griff should include the following costs in 
expenses to be included in selling expenses for  G&A expense: 
2010. Ad­vertising ($150,000) and sales salaries  Accounting and legal fees $ 25,000
and commissions ($140,000) are clearly selling  Officers’ salaries 150,000
expenses, as is the rent for the office space  Insurance 85,000
occupied by the sales department ($220,000 ×  G&A expense $260,000
1/2 = $110,000). Additionally, freight­out  Freight­in ($175,000) is an inventoriable cost 
($80,000) is a selling expense because shipping  which should be reflected in cost of goods sold 
the goods from the point of sale to the customer  and ending inventory. Freight­out, the cost of 
is the final effort in the selling pro­cess. The  delivering goods to customers ($160,000), is 
total selling expense is, therefore, $480,000  included in selling expenses. Sales represen­
($150,000 + $140,000 + $110,000 + $80,000).  tatives salaries ($215,000) is also a selling 
The re­maining expenses given are general and  expense.
administrative ex­penses, except for interest and 
6. (d) Interest expense is generally considered 
the loss on sale of long­term investment, which 
are nonoperating items (other expenses and  to be anonoperating item and is therefore 
losses).  included in other ex­penses and losses. 
Operating expenses are usually dividedinto two
5. (a) Operating expenses are usually divided into  categories, selling expenses and general and 
two categories, selling expenses and general  ad­ministrative expenses. Since advertising 
and administra­tive (G&A) expenses. Selling  expense is di­rectly related to the sale of the 
expenses are related to thesale of a company’s  company’s products, it is included in selling 
products, while G&A expenses are re­lated to  expenses. Therefore, neither of the ex­penses 
the company’s general operations. Therefore,  given are general and administrative expenses.
316 MODULE 9 BASIC TH. & FIN. REP.: D. FINANCIAL STATEMENTS
inventory loss are not extraordinary 
because they are not unusual in nature.
7. (a) Charitable contributions expense 
should include all expenses incurred in 11. (a) Extraordinary items are material 
2010 by Yew Co. which involve  gains or losses which are both 
charitable contributions to other  unusual in nature and infrequent in 
entities. The total charita­ble  occur­rence. Foreign currency losses 
contributions expense is $310,000,  ($1,200,000) and losses due to 
consisting of the $250,000 donated to  discontinued operations ($500,000) 
youth and educational programs and  are not considered to be unusual in 
the $60,000 ($140,000 – $80,000)  nature and thus are not extraordinary. 
donated to health and human service  Items that may qualify as 
organizations. The other $80,000 was  extraordinary include some 
given to these organizations by the  casualties, expropriations, 
employees, with the company merely  prohibitions under a new law, and 
acting as an agent collecting that  extin­guishment of debt. Midway’s 
amount through payroll deductions  casualty loss appears to be 
and forwarding it on to the  extraordinary because the hurricane 
organizations. The expenditure for  was the first ever to strike in 
redesigning product packaging  Midway’s area. The net pretax loss 
($100,000) would be properly  was $100,000 [$800,000 equipment 
classified as research and development loss – $700,000 building gain 
expense.  ($1,000,000 – $300,000)]. 
12. (c) A gain (loss) must be recognized 
8. (a) Extraordinary items are material gains or 
when a non­monetary asset is 
losses which are both unusual in nature and 
involuntarily converted into 
infrequent in occur­rence. For Raim Co., which 
monetary assets even if the company
sustains flood losses every two to three years,  reinvests the monetary assets in 
the 2010 flood loss is not infrequent, and should  replacement nonmonetary assets. 
be recognized as a component of income from  The gain or loss is the difference 
con­tinuing operations. For Cane Co., the 2010  between the insurance proceeds 
flood loss is both unusual and infrequent, so it  ($1,100,000 re­ 
should be recognized as an extraordinary item. 
9. (b) Extraordinary items are material 
items which are both unusual in 
nature and infrequent in occurrence. 
Theeffect of a strike ($175,000) and a 
gain or loss from sale or 
abandonment of equipment ($125,000) is not 
considered extraordinary. The loss from the 
earthquake would be con­sidered unusual and 
infrequent. Therefore the $150,000 loss is 
extraordinary, and the total amount of 
infrequent losses not considered extraordinary 
is $300,000 ($175,000 +$125,000).
10. (a) Extraordinary items are material 
items which are both unusual in 
nature and infrequent in occurrence. 
Dis­posals of plant assets, foreign 
currency losses, and inventory losses 
are not considered to be unusual in 
nature, and thus are not extraordinary. 
Items that may qualify as extraordi­
nary items include some casualties, 
expropriations, and pro­hibitions under
a new law. In Kent Co.’s 2010 income 
statement, losses not considered 
extraordinary amount to $610,000. The
factory disposal ($300,000) is 
classified as discontinued operations 
because the operations carried on there
are dissimilar from operations carried 
on at the other factories. The $120,000 
foreign currency loss and $190,000 
should not be reported in the income 
placement cost – $50,000 deductible =  statement at all. An unreal­ized loss on
$1,050,000) and the carrying amount of the  noncurrent investments in stock 
assets destroyed or used up as a re­sult of the  ($5,400) is reported in other 
casualty loss. The warehouse (carrying amount  comprehensive income, not in net 
of $300,000) was destroyed, and cash of  income. A correction of an error 
$20,000 was used to cover removal costs.  ($7,500) is treated as a prior period 
Therefore, the gain is $730,000 [$1,050,000 –  adjustment. It is reported in the 
($300,000 + $20,000)]. It is not an extraordi­ financial statements as an adjustment 
nary item because storms similar to the one that  to the beginning balance of retained 
destroyed the warehouse occur frequently (every earnings, rather than in the income 
four years). statement. Since both of these items 
were subtracted in the computation of 
13. (c) In this situation, the provision for income reported net income, they must be 
tax(income tax expense) will be the amount of  added back to compute the correct net 
the tax liability to the government determined  income of $87,000 ($74,100 + $5,400 
without including the ex­traordinary loss. This  + $7,500). 
amount is determined by applying the 40% tax 
rate to pretax accounting income before  15. (b) In 2010, Brecon Co. would report 
extraordi­nary items adjusted for any permanent one fifth of the cabinet costs as 
differences. Ac­counting income before taxes is  depreciation expense in selling, 
$900,000, but that amount includes a gain on  general, and administrative expenses. 
life insurance coverage ($100,000). A gain on  Four fifths of the cabinet cost would 
life insurance coverage is a permanent  remain capitalized as fixed assets at 
difference because it is included in accounting  the end of 2010. The cabinets are 
income but will never be included in taxable  considered fixed assets and not a part 
income. Therefore, the amount of accounting  of cost of goods sold. 
income which will be subject to taxes is 
16. (d) A material gain or loss that is 
$800,000 ($900,000 – $100,000), and the 
provision for in­come taxes is $320,000  unusual in nature or infrequent in 
($800,000 × 40%). The tax savings from the  occurrence, but not both, should be 
extraordinary loss ($300,000 × 40% =  pre­sented as a separate component of
$120,000) will not affect the provision for  income or loss from con­tinuing 
income taxes because the extraordinary item  operations. Both discontinued 
must be reported together with its tax effect  operations and ex­ 
(extraordinary loss of $180,000; net of tax). traordinary items are reported separately after 
income from continuing operations.
14. (d) Net income as reported ($74,100) 
properly in­cluded the gain on early  17. (a) For an item to qualify as an 
retirement of bonds payable ($22,000)  extraordinary item itmust be both unusual in 
and the loss from fire ($14,000). The  nature and infrequent in occur­rence. The above
fact that the gain and loss were  criteria must take into account the envi­ronment 
reported net of taxes in the income  in which the entity operates. An entity with 
statement was incorrect, but does not  sales in different countries will experience 
cause the net income amount to be in  foreign currency re­valuations on a regular 
error. However, the other two items  basis, so a currency revaluation would not be 
infrequent in occurrence and it is not an ex­
MODULE 9  BASIC TH. & FIN. REP.:  D. FINANCIAL STATEMENTS 317
component, so it is part of 
discontinued operations, not con­
traordinary item. Gains from increases in  tinuing operations. It is combined with
contract prices are neither infrequent nor  the loss from dis­posal on the income 
unusual, so they cannot qualify as extraordinary statement. Discontinued operations is
items. a category distinct from 
18. (b)   Extraordinary   items   are   reported extraordinary items. 
net   of   incometaxes  as   a   separate 26. (a) Management is not required to 
component   of   income  after have entered into a sales agreement. It 
discontinuedoperations.  is sufficient if management is com­
19. (b) Extraordinary items are events and mitted to a disposal plan that is 
transactions that are distinguished by  reasonable. The other items are all 
both their unusual nature and the  required for presentation as 
infrequency of their occurrence.  discontinued operations. 
Losses from hail damage are both  27. (d) In discontinued operations, 
common and frequent for Toncan.  presentation of the income or loss 
Estimates of the losses would not be  from operations of the component and
presented in the financial statements.  the gain or loss on disposal is 
No amount is recorded until a loss  required. Since the company met the 
actually occurs.  requirements for “held for sale” status
in 2010, the sub­sidiary should be 
D.2.  Unusual or Infrequent Items written down to its fair value less cost 
20. (d) A transaction that is unusual in nature and to sell. This would result in a loss of 
infre­quent   in   occurrence   is   considered   an $1,500,000 ($8,000,000 carrying 
extraordinary   item.   An   extraordinary   item   is
amount – $6,500,000 fair value). 
Therefore, the loss from discontinued 
reported after discontinued operations.
operations would be $3,500,000 
21. (c) Extraordinary items are events and ($2,000,000 loss from operations + 
transactions that are distinguished by  $1,500,000 loss on planned disposal). 
both their unusual nature and the 
infrequency of their occurrence. 
Teller Co.’s loss arising from its first 
antitrust action meets both of these 
criteria (particularly since they 
pleaded guilty) and should therefore 
be reported as an extraordinary item. 
The strike against Teller’s major 
supplier, however, should not be 
reported as an extraordinary item 
because it is usual in nature but may 
be expected to recur as a consequence
of customary and continuing business 
activities. 
22. (c) Extraordinary items are material items which
are both unusual in nature and infrequent in 
occurrence.Therefore, the loss on sale of a bond 
investment is not ex­traordinary. Neither item is 
treated as extraordinary. 
23. (c) Corrections of errors shall continue
to be re­ported net of tax in retained 
earnings as an adjustment of the 
beginning balance. 
24. (b) Costs of exit activities (including
restructuring charges) should be 
measured and recognized at fair 
value when they are incurred. 
D.3.  Discontinued Operations
25. (b) The operating loss of $300,000 
($500,000 reve­nues less $800,000 
expenses) relates to a discontinued 
displayed. 
28. (d) The loss from discontinued 
34. (d) Once unrealized items recorded 
operations would equal the loss from
operations plus the estimated loss  and reported in the current or prior 
from disposal of the component.  period are recognized as realized and 
reported in net income, it is necessary 
29. (c) Costs of termination benefits, lease  to reverse them out of other 
termination, and consolidating facilities  comprehensive income. The 
or relocating employees related to a  reclassification adjust­ment is to avoid
disposal activity that involves  double counting. The reclassification 
discontinued operations should be  ad­justment in this situation is $15,000
included in the results of discontinued  ($25,000 gain on AFS securities, net 
operations.  of $10,000 tax). 

30. (b) The requirement is to determine  35. (c) The FASB allows three 


how a discon­tinued component’s  presentation formats [answer 
operating losses for the current  choices (a), (b), and (d)] with a 
period should be classified in the  preference for the first two 
financial statements. The “in­come  alternatives. 
(loss) from operations” is combined  36. (d) The accumulated balance of other
with the loss on disposal.  comprehen­sive income should be 
31. (a) Gains or losses from the  reported as a component of equity, 
operation of a discon­tinued business separate from retained earnings and 
component realized for the period are additional paid­in capi­tal. 
com­bined with the loss of disposal  37. (b) Comprehensive income is defined 
to determine the loss from  as the change in equity of a business 
discontinued operations.  during a period from transactions of 
32. (c) Discontinued operations should  nonowner sources. Stockholders are 
include the gain or loss on disposal  owners of the corpora­tion or entity, 
plus the results of operations during  therefore, transactions between the 
the period.  entity and shareholder are not a 
component of comprehensive income.
D.4.  Comprehensive Income
38. (d)   Assuming   the   fair   value   option   is   not
33. (b) The purpose of reporting  elected,   the   calculation   of   holding   gains
comprehensive income is to report a  recognized   in  other  comprehen­sive   income  is
measure of overall enterprise  as follows: 
performance by displaying all changes Before tax Income tax Net of tax
in equity of an enterprise that result  Year ended 12/31/09 $5,000 $1,500 $3,500
from recognized transactions and other Year ended 12/31/10 2,500 750 1,750
economic events of the period other  Total gain $7,500 $2,250 $5,250
than transactions with owners in their  The reclassification adjustment should be 
ca­pacity as owners. An enterprise  shown net of tax, so ($5,250) is the adjustment 
should continue to display an amount  amount. $5,250 had been pre­viously added to 
for net income with equal prominence  other comprehensive income when the gains 
to the comprehensive income amount  occurred. The $5,250 needs to be taken out in 
order to
318 MODULE 9 BASIC TH. & FIN. REP.: D. FINANCIAL STATEMENTS
as part of other comprehensive 
income. An­swers (a) and (b) are 
avoid   counting   the   gains   twice.   The incorrect because an adjustment to 
securities   have   been   sold   so   the   gains   are pension liability to record the funded 
now realized and will be part of net income. status of the plan, and subsequent 
decreases of the fair value of 
39. (b)  The   gain   for   the   period   is   10
available­for­sale securities that have
shares   times   theincrease   in   fair been previously written down as im­
value, which is $250. This gain of paired are included in other 
$2,500 must be shown net of tax, so comprehensive income. An­swer (d) 
the holding gain is $1,750.  is incorrect because decreases in the 
40. (a)    Comprehensive income is computed fair value of held­to­maturity 
as follows:  securities are not part of other 
comprehensive income. 
Net income
Other comprehensive income net of tax: 46. (d) Comprehensive income (net income plus 
Unrealized gain on securities 15,000
Less:  Reclassification adjustment (2,500)    12,500
other comprehensive income) should be 
Pension liability adjustment (3,000 displayed in a financial statement that has the 
Other comprehensive income same prominence as other financial statements. 
Comprehensive income Answer (a) is incorrect because the FASB pre­
Notice that stock warrants outstanding are  fers that comprehensive income be displayed 
not included as part of comprehensive  either at the bottom of the income statement, 
income. continuing from net income to arrive at a 
comprehensive income figure (equals net in­
41. (d) If $2,450 (net of tax) is being  come plus other comprehensive income), or in a 
deducted from other comprehensive  separate statement of comprehensive income. 
income as a reclassification adjust­ Answer (b) is incorrect 
ment, $2,450 must be the amount of 
unrealized gains (net of tax) that have 
been recognized in other 
comprehensive in­come. The realized 
gains will then be recognized in 
income from continuing operations 
before tax. The gains before tax 
effects are $3,500 ($2,450 ÷ 70%). 
42. (b) Comprehensive income disclosures include 
the changes during a period of the following 
components of other comprehensive income: 
unrealized gains and losses on available­for­sale 
investments and foreign currency items, 
including any reclassification adjustments, and 
the pension liability adjustment required to 
recognize the funded status of the plan. Treasury
stock is deducted from stockholders’ equity and 
not a component of other comprehensive 
income. 
43. (d)   Separate   EPS   calculations   are
not   required   for   other
comprehensive   income   or
comprehensive income. 
44. (d) The FASB prefers that 
comprehensive income be shown at 
the bottom of the income statement as 
a con­tinuation of net income or in a 
separate statement beginning with net 
income. Presenting comprehensive 
income in the statement of changes in 
stockholders’ equity is not a method 
that is preferred by the FASB. 
45. (c) If the fair value option is not 
elected, held­to­maturity securities 
are reported at amortized cost. Any 
de­creases or increases in fair value 
are reported neither in net income nor
of billings on long­term contracts 
because components of other comprehensive  ($1,600,000) and prepaid taxes 
income may be displayed net of related tax  ($450,000). However, income tax 
effects or before related tax ef­fects with one  expense has not yet been recorded. 
amount shown for the aggregate income tax  Earnings ($6,680,000) less costs and ex­
effect. Answer (c) is incorrect because  penses ($5,180,000) result in pretax 
comprehensive in­come can be displayed in the  income of $1,500,000. Since the tax rate 
two methods preferred by the FASB (mentioned is 30%, tax expense is $450,000 (30% × 
above) or in the statement of changes in  $1,500,000). Therefore, an adjustment is
stockholders’ equity.
necessary to debit income tax expense 
D.5. Balance Sheets and credit prepaid taxes for 
$450,000.Total current assets, after this 
47. (b) Total retained earnings includes both  adjustment, are $5,700,000. 
unappro­priated retained earnings and restricted 
Cash $ 600,000
retained earnings. Therefore, before closing 
Accounts receivable 3,500,000
entries, total retained earnings is $1,060,000  Cost in excess of billings 1,600,000
($900,000 + $160,000). Before computing 2010 $5,700,000
net income, tax expense must be recorded. 
Earnings ($6,680,000) less costs and expenses  50. (a) Mirr began operations on 1/1/10 with
($5,180,000) result in pretax income of  the fol­lowing balance sheet elements:
$1,500,000. Since the tax rate is 30%, tax  Assets = Liabilities + Owners’ equity
expense is $450,000 (30% × $1,500,000).  $860,000 = $110,000 + $750,000
Therefore, an adjustment is necessary to debit  During 2010, liabilities increased to $120,000, and
income tax expense and credit prepaid taxes  owners’ equity increased to $765,000 [$750,000 
for $450,000. After the adjustment, net income  beginning balance + $18,000 net income ($82,000 
is $1,050,000 ($6,680,000 – $5,180,000 –  revenues – $64,000 expenses)
$450,000). After closing entries, total retained  – $3,000 dividends declared]. Therefore, 
earnings is $2,110,000 ($1,060,000 +  12/31/10 assets must be $885,000.
$1,050,000).
Assets = Liabilities + Owners’ equity
48. (a) The only liabilities included in the  Assets = $120,000 + $765,000
Assets = $885,000
trial balance are billings in excess of 
costs on long­term contracts  51. (b) The requirement is to determine the net 
($700,000) and note payable­ incomefor 2010 by analyzing changes in the 
noncurrent ($1,620,000). Only the note balance sheet. Recall the accounting equation: 
is noncurrent. Billings in excess of  Assets – Liabilities = Stockhold­ers’ equity. By 
costs on long­term contracts is similar  inserting the changes given into this for­mula, we 
to unearned revenue and is always  find an increase of $62,000 exists in the entire 
reported as a current liability.  stockholders’ equity section ($89,000 – $27,000 = 
49. (c) Current assets listed in the trial  $62,000). Stockholders’ equity is composed of 
balance are cash ($600,000), accounts  capital stock, additional paid­in capital and 
receivable ($3,500,000), cost in excess  retained earnings. Because increases in
MODULE 9  BASIC TH. & FIN. REP.:  D. FINANCIAL STATEMENTS 319
business, however, are specifically 
excluded from this disclosure 
the other two balances are given that total  requirement. Therefore, only the 
$66,000 ($60,000 + $6,000), the retained  $250,000 sale to affiliated enterprises 
earnings balance must have de­creased by  must be disclosed. 
$4,000 ($66,000 – $62,000). When dividends 
are paid, this reduces retained earnings, while  55. (c) Disclosure of material transactions 
Vel Corp. net income increases the balance of  between related parties is required 
retained earnings. For the equation to balance,  except for (1) compensation 
the changes in retained earnings ac­count must  agreements, expense allowances, and 
reduce total stockholders’ equity by $4,000.  other similar items in the ordinary 
Therefore, if $13,000 in dividends are paid,  course of business, and (2) 
which reduce retained earnings, and total  transactions which are eliminated in 
retained earnings are to be re­duced by $4,000,  the preparation of consolidated or 
then net income, which increases retained  com­bined financial statements. The 
earnings, must be $9,000. This creates the  officers’ salaries and offi­cers’ 
$4,000 difference needed to make the equation  expenses fall into category (1), while 
balance ($89,000 – $27,000 = $66,000 –  the intercompany sales fall into 
$4,000). The following shows the analysis of the category (2). Therefore, only the loans
retained earnings account: to of­ficers ($125,000 + $50,000 = 
Retained earnings (beg.) $ xxx $175,000) are reported as related­party
Net income (plug) 9,000 disclosures. 
Dividends ( 13,000)
Retained earnings (decrease) ($ 4,000) 56. (d) Disclosure of material transactions 
between related parties is required 
52. (a) Idle machinery ($11,200) and  except for (1) compensation 
cash surrender value of life insurance agreements, expense allowances, and 
($13,700) are both assets. The al­ other similar items in the ordinary 
lowance for decline in market value  course of business and (2) transactions 
of noncurrent market­able equity 
elimi­ 
securities ($8,400) is a contra asset 
that is prop­erly included in the asset 
section of the balance sheet (as a 
deduction). The only item listed 
which should not be in­cluded in the 
asset section of the balance sheet is 
the trea­sury stock ($24,000). 
Although the treasury stock account 
has a debit balance, it is not an asset; 
instead, it is reported as a contra 
equity account. Therefore, the 
$24,000 must be excluded from the 
asset section, reducing the net asset 
amount to $851,000 ($875,000 – 
$24,000). 
53. (a) Although the statement of cash 
flows provides information about 
liquidity, solvency, and financial 
flexibil­ity, a potential investor would 
primarily use the balance sheet to 
assess liquidity and financial 
flexibility. The bal­ance sheet helps 
users analyze the company’s ability to
use current assets to pay current 
liabilities (liquidity) and the 
company’s ability to alter the amounts
and timing of future cash flows to 
adapt to unexpected needs or to take 
advan­tage of opportunities 
(flexibility). 
D.6.a.  Disclosures
54. (c) Financial statements must include 
disclosures of material transactions 
between related parties. Compensation
arrangements in the ordinary course of
ments should be properly presented in 
nated in the preparation of consolidated or  accordance with GAAP; thus no 
combined finan­cial statements. improper presentation should exist. An­
swer (c) is incorrect because the totals 
57. (b) Disclosure of any material  of the financial state­ments should 
related­partytransactions is required  include all items; thus, no items would 
except be ex­cluded. Answer (d) is incorrect 
1. Compensation agreements, expense because management’s responses to the 
allowances,   and   similar   items   in auditor’s comments would be contained
the ordinary course of busi­ness.  in the management letter, which is a 
2. Transactions   that   are   eliminated   in separate report typically presented to 
the   preparation   of   consolidated   or the audit committee or the board of 
combined financial statements.  directors. 
Since sales of inventory between subsidiary and 
D.6.b.  Accounting Policies
parent are eliminated in preparing consolidated 
financial statements, such sales need not be  60. (a) The requirement is to determine 
disclosed as a related­party transac­tion.  which informa­tion should be 
Nonmonetary exchanges by affiliates are not  included in the summary of 
specifi­cally exempted from disclosure, and  significant ac­counting policies. 
therefore must be dis­closed as related­party  Disclosure of accounting policies 
transactions. Compensation arrangements,  should identify and describe the 
expense allowances, and similar items in the  accounting principles followed by the 
ordinary course of business need not be disclosed  reporting entity and methods of 
as related­party transactions. However, in this  applying those prin­ciples. Answer (a)
case the allowances are in excess of normal  is correct because the method of 
business practice and therefore must be disclosed.  record­ing and depreciating assets is 
Surety and guaranty agreements between related  an example of such a required 
parties are not specifically exempted from  disclosure. Answers (b) and (c) are 
disclosure and therefore must be disclosed as  incorrect because both represent detail
related­party transactions. presented elsewhere in the financial 
state­ments. Answer (d) is incorrect 
58. (c)       Disclosures of material transactions because it is an estimate of earnings 
shall include  rather than an accounting policy. 
(1) nature of relationship(s), (2) description of 
61. (c) Disclosure of accounting policies 
transaction(s), including those assigned zero or 
nominal amounts, (3) dollar amounts of  should iden­tify and describe the 
transactions for each income statement period  accounting principles followed by 
and effect of any change in method of  the reporting entity and methods of 
establishing terms, and (4) amounts due to/from applying those principles. The 
related parties, including terms and manner of  criteria for determining which 
settlement.  investments are treated as cash 
equivalents is an example of how the
59. (a) The users of the information are the  entity applies accounting principles. 
focus of financial reporting. GAAP  These disclosures should not dupli­
requires disclosures in the notes to  cate details presented elsewhere as 
facilitate the users’ understanding of the part of the financial statements. 
financial state­ments. Answer (b) is  Answers (a), (b), and (d) are not 
incorrect because the financial state­ disclosures of 
320 MODULE 9 BASIC TH. & FIN. REP.: D. FINANCIAL STATEMENTS
because they do not meet the 
definition of principal market. Answer
accounting policies, and also would be  (d) is incorrect because it describes the
presented elsewhere in the financial statements. most advantageous mar­ket. 
62. (c) This is a recognized subsequent  67. (d) If location is an attribute of the asset, the 
event in thefinancial statement  price inthe principal market should be adjusted 
because Swift records an estimate for  for costs to transport the asset to its principal or 
warranty liability. Prior to issuing the  most advantageous market. An­swers (a) and (b)
financial statements, Swift must make  are incorrect because prices are not ad­justed for
an adjustment for warranty liability  transaction costs such as selling costs. Answer 
for the product recall. Therefore,  (c) is incorrect because fair values are not based 
warranty liability should be $68,000  upon entry prices to the market. 
on the 2010 balance sheet. 
68. (c) Market participants are buyers and 
63. (a) This is an unrecognized  sellers in theprincipal market or most 
subsequent event thatdid not exist as  advantageous market for the asset or 
of the balance sheet date of December liability. Market participants should be
31, 2010. Therefore, there should be  independent, knowledgeable, able to 
no adjustment of the 2010 financial  transact for the asset, and willing to 
statements. However, since the  transact (i.e., motivated but not 
inventory loss is material in amount  compelled to do so). An­swer (a) is 
and the loss is uninsured, to prevent  incorrect because market participants 
the financial statements from being  should not be compelled to sell. 
misleading, Colter should disclose the Answer (b) is incorrect because it is 
loss in a footnote to the 2010 financial not an independent, unrelated party. 
statements.  Answer (d) is incorrect 
64. (d) A loss for a recognized subsequent event 
shouldbe reported in the current year financial 
statements. There­fore, answer (d) is correct. 
Answer (a) is incorrect because a loss would not
be recognized in the current year for an 
unrecognized subsequent event. However, a 
footnote dis­closure would be required for the 
unrecognized subsequent event if the event is 
material and the financial statements would be 
misleading if the event were not disclosed. An­
swer (b) is incorrect because gains are not 
recognized until realized. Answer (c) is 
incorrect because recognized subse­quent events
result in a loss recognized in the current year 
and a corresponding adjustment to record the 
liability. 
D.6.d. Fair Value Measurements
65. (b) The definition of fair value is the 
price thatwould be received to sell an 
asset in an orderly transaction between
market participants at the 
measurement date. An­swer (a) is 
incorrect because it is an entry price, 
not an exit price. Answer (c) is 
incorrect because assets that require to 
be revalued to fair value should be 
based on an exit value, not an adjusted
historic cost. Answer (d) is incorrect 
be­cause the price paid to acquire the 
asset is an entry price, not an exit 
price. 
66. (a) The principal market is the market 
in which thereporting entity would sell
the asset or transfer the liability with 
the greatest volume and level of 
activity for the asset or liability. 
Answers (b) and (c) are incorrect 
bilities. Answer (a) is incorrect 
because the dealer is establishing a new market  because it refers to the in­come 
and is not in the principal market that exists for  approach. Answer (c) is incorrect 
the asset. because it refers to the cost approach. 
Answer (d) is incorrect because it is 
69. (d) A fair value measurement assumes the  not a technique for any of the 
highestand best use of the asset that is  permissible valuation techniques. 
physically possible, legally permissible, and 
financially feasible. Answers (a) and (b) are  74. (c) A change in valuation technique 
incorrect because to determine the valuation  used to mea­sure fair value should be 
premise, the asset can be classified as either in­ reported as a change in accounting 
use or in­exchange. An­swer (c) is incorrect  estimate. The change is reported on a 
because for valuation purposes, the highest and  prospective basis; however, the 
best use would maximize the value of the asset.  disclosures for change in accounting 
estimate are not required for a change 
70. (a) The fair value at initial recognition is the  in valuation technique. An­swers (a), 
trans­action price that represents the price paid  (b), and (d) are incorrect. 
to acquire the asset (an entry price). Answer (b) 
is incorrect because transaction costs are not  75. (a) The fair value hierarchy prioritizes the 
included. Answer (c) is incorrect because it is an inputs tovaluation techniques into three levels: 
exit price. Answer (d) is incorrect because it is  Level 1, Level 2, and Level 3. The highest 
not a fair value measure.  priority is given to Level 1 inputs, which are 
quoted prices in active markets for identical 
71. (b) There are three approaches to  assets and liabilities. Answers (b) and (c) are 
valuation tech­niques for fair value:  incorrect, because these levels are lower priority
the market approach, the income ap­ than Level 1. Answer (d) is incorrect because 
proach, and the cost approach. There  there is no Level 4 input. 
is no technique called the residual 
value approach.  76. (b) The bank prime rate and the default rates 
areboth observable inputs. A financial forecast 
72. (a) The income approach uses valuation  is developed by an entity and is an unobservable
techniquesto convert future amounts to a single  input or Level 3 input. Therefore, answers (a), 
present value amount. Therefore, Black­ (c), and (d) are incorrect. 
Scholes­Merton, binomial models, or dis­
counted cash flow models are examples of the  D.6.f. Constant Dollar Accounting
income ap­proach. Answers (b) and (c) are 
77. (c) The Codification encourages, but does not 
incorrect because these approaches do not use 
re­quire, business enterprises to disclose 
present value techniques. Answer (d) is 
incorrect because there is no exit value  supplementary in­formation on the effects of 
approach.  changing prices. The statementpresents 
requirements to be followed by enterprises that 
73. (b) The market approach valuation  voluntarily elect to disclose this information. 
technique usesprices and other  Answers (a) and (b) are incorrect because the 
relevant information generated by  information is not reported in the body of the 
market transactions involving  financial statements or in the notes to the financial
identical or comparable assets or lia­ statements. Answer (d) is incorrect because man­
MODULE 9  BASIC TH. & FIN. REP.:  D. FINANCIAL STATEMENTS 321
constant purchas­ing power the opening and 
closing balances of, and transac­tions in, 
agement’s report to shareholders identifies  monetary assets and liabilities. During a period 
management’s responsibilities, including  of rising prices, monetary liabilities give rise to 
responsibilities for the internal control  purchasing power gains because they will be 
system, and would not include information  settled with cash which can be used to purchase 
about changing prices. relatively fewer goods or services at a future 
78. (c) The requirement is to determine the amounts time. 
to be reported for three balance sheet accounts  D.6.g.  Current Cost Accounting
in a supple­mentary constant dollar balance 
sheet. In a constant dollar balance sheet,  83. (d) Current cost accounting is a method of 
nonmonetary items are restated to the cur­rent  valuingand reporting assets, liabilities, 
price level, while monetary items are not  revenues, and expenses at their current cost at 
restated be­cause they are already stated in  the balance sheet date or at the date of their use 
or sale. A holding gain is recorded as an 
current dollars. The invest­ment in bonds and 
increase in an item’s value. At December 31, 
the long­term debt are monetary items since 
2010, Vend Co. is holding merchandise which 
their amounts are fixed by contract in terms of 
is currently valued at $10 per unit (replacement 
number of dollars. Therefore, these items are 
cost), while the original recorded value of the 
not restated and are reported at $60,000 and 
merchandise was $8 per unit (purchase price). 
$80,000, respectively. The land, however, is a 
There­fore, the holding gain is $2 per unit.
nonmonetary item and its cost ($120,000) must 
be restated to current dollars by using the 
TO/FROM ratio (110/100), resulting in an 
adjusted amount of $132,000 ($120,000 × 
110/100). 
79. (b) The account “loans to employees” 
is a monetary asset account since its 
payment amount is fixed at some 
point in the future. Conversely, 
merchandise inventory is considered a
nonmonetary asset account since its 
value will change based on relative 
price levels in the future. The total 
value of monetary assets is the 
balance of the loans to employees 
account, or $20,000. 
80. (a) The increase in current cost (nominal 
dollars) of $15,000 is the total increase in 
current cost, including any increase caused by 
inflation. The effect of changes in the general 
price level is not separated from the effect of 
changes in specific value. The increase in 
current cost (con­stant dollars) of $12,000 is the
increase in current cost after eliminating any 
increase caused by inflation. Therefore, the 
inflation component of the increase in current 
cost of in­ventories is $3,000 ($15,000 – 
$12,000). 
81. (d) The requirement is to determine 
which item is classified as 
nonmonetary when computing the 
purchasing power gain or loss on net 
monetary items. A monetary item is 
one that is fixed or determinable 
without reference to fu­ture prices. 
Accumulated depreciation is not a 
monetary item. Advances to 
unconsolidated subsidiaries, 
allowance for doubtful accounts, and 
unamortized premium on bonds 
payable are all monetary items. 
82. (b) A purchasing power gain or loss is the net 
gain or loss determined by restating in units of 
volume of business trans­acted with a 
84. (c) The requirement is to calculate  certain contributor are all examples of
the amount of current cost  concen­trations that create 
depreciation expense which would  vulnerabilities that are required to be 
appear in supplementary current cost  disclosed. 
financial statements. Deprecia­tion is
to be measured based on the average 89. (b) The potential impact of estimates about 
current cost of the asset during the  values of assets and liabilities when it is 
period of use. The average current  reasonably possible that the estimate will 
cost of this machine during 2010 is  change in the near future is a required dis­
$120,000 [($115,000 + $125,000)    closure regarding significant risks and 
2]. Therefore, 2010 depreciation  uncertainties. Factors causing an estimate to be 
expense is $24,000 ($120,000   5­ sensitive is not a required disclo­sure, only 
year useful life).  recommended. The potential impact of estimates
about values of assets and liabilities when it is 
85. (a) Current cost for inventories and  remotely possible that the estimate will change 
equipment is measured at the lower of  in the near future is not a required disclosure. It 
current cost or recoverable amount. For  is only a required disclosure if it is reasonably 
equipment, recoverable amount  possible that the estimates will change in the 
($95,000) is lower than current cost; for  near future. Only a description of operations 
inventory, current cost ($80,000) is  outside the home country is a required 
lower than recoverable amount.  disclosure. 
Therefore, the total amount to be 
reported for these assets is $175,000  D.8.  Other Comprehensive Bases of 
($95,000 + $80,000).  Accounting

86. (a) Increases or decreases in the  90. (c) Other than generally accepted 


current cost of inventory result from  accounting princi­ples, the only other 
the difference between the measures  bases which may be used to prepare 
of assets at their entry dates  financial statements in conformity 
(beginning of year or purchase date)  with a comprehensive basis of 
and measures of assets at their exit  accounting are the cash basis and a 
dates (end of year or date of  basis of ac­counting used to file an 
use/sale). Based on this definition,  income tax return.
holding gains would be reported both
when inventory is sold during the  91. (d) When financial statements are 
year (realized gains), and when  prepared using an income tax basis, 
inventory is held at the end of the  two accounting methods can be used: 
year (unrealized gains).  (1) modified cash basis—hybrid 
method of IRS and (2) accrual basis—
87. (a) The current cost of goods sold is  IRS. The modified cash basis reflects 
computed by multiplying the average  the use of accrual basis for 
cost of units produced or purchased  inventories, cost of goods sold, sales, 
during the year times the number of  and depreciation, if these are 
units sold.  significant. The accrual basis uses 
D.6.h. Risks and Uncertainites accruals and deferrals with several 
exceptions (e.g., prepaid income, 
88. (d) The market in which an entity  warranty expense). When financial 
conducts its op­erations, the available statements are prepared on an income 
sources of supply of materials used in tax basis, the financial statements 
operations of an entity, and the  should not simply repeat items and 
amounts reported in the 
322 MODULE 9 BASIC TH. & FIN. REP.: D. FINANCIAL STATEMENTS
include infor­mation on the purpose
of the statements, assumptions, and
tax return. Thus, items such as nontaxable  significant accounting policies. 
municipal inter­est and the nondeductible 
portion of travel and entertainment expense  D.11. SEC Reporting Requirements
should be fully reflected in the income statement 98. (c) The requirement is to identify the form used 
on the basis used for tax purposes, with footnote tofile as an annual report with the SEC. Answer 
disclosure of the differences between the 
(c) is correct because Form 10­K is the title of 
amounts reported in the income statements and 
the annual report required to be filed annually by
tax return.
issuer companies. Answer (a) is incorrect 
92. (a) In financial statements prepared on because this is the title of the quarterly financial 
the income tax basis, the  report. Answer (b) is incorrect because Form 8­
nondeductible portion of meals and  K is the in­formation report that may be filed 
entertain­ment expense should be  any time during the year. Answer (d) is incorrect
included with the deductible portion as because a Form S­1 is an initial reg­istration 
a total amount of travel and  form for securities under the Securities Act of 
entertainment expense. Ad­ditionally,  1933. 
the nondeductible portion should be 
99. (d) The requirement is to identify the 
footnoted. Answer (a) is the best 
content of theSEC Form 10­Q. 
answer among the alternatives given, 
Answer (d) is correct because the 
although it does not indicate that a 
Form 10­Q presents reviewed 
footnote is required. 
quarterly information and other in­
93. (c) The requirement is to identify the  formation about the company. 
statement that best describes  Answer (a) is incorrect be­cause the 
appropriate modifications in cash  information is reviewed, not audited. 
basis finan­cial statements. Answer  Answer (b) is incorrect because this 
(c) is correct because the modifica­ describes the Form 10­K. An­swer (c)
tions should be the same as GAAP  is incorrect because this describes the 
and not illogical. An­swer (a) is  Form 8­K. 
incorrect because modified cash basis 
financial statements may have 
substantial modifications from GAAP.
Answer (b) is incorrect because the 
modifications are not required to be 
the same as those required by tax law. 
An­swer (d) is incorrect because 
modifications are allowed. 
D.9. Prospective Financial Information
94. (b) The responsible party is not the 
only limited user. Third parties with 
whom the responsible party is ne­
gotiating directly are also limited 
users. Answer (a) is incor­rect because
prospective financial information may 
be pre­pared for general or limited use.
Answer (c) is incorrect because it is 
true that assumptions not necessarily 
expected to occur may be included in 
the financial projection. An­swer (d) is
incorrect because the financial 
projection may be expressed as a range
of dollars. 
95. (d)   Prospective   financial
information   is   defined   as   any
financial   information   about   the
future. 
96. (c) Financial forecasts and projections must be 
pre­pared in accordance with GAAP, with the 
plans of the entity, and with due professional 
care in order to achieve a reason­ably objective 
basis. 
97. (a) Prospective financial statements
Standards
100. (a) The requirement is to identify the  (IFRS)
form used tofile as a quarterly report 
with the SEC. Answer (a) is correct  105. (b) The requirement is to identify the 
because Form 10­Q is the title of the  circumstancesin which Galaxy may 
quarterly report required to be filed  present the note as a noncurrent liabil­
by issuer companies.  ity. Answer (b) is correct because 
IFRS requires that Galaxy must have 
101. (c) The requirement is to identify the  executed an agreement to refinance at 
balance sheetrequired in the Form 10­Q. the balance sheet date in order to 
Answer (c) is correct because the SEC  classify the debt as a noncurrent liabil­
requires that a Form 10­Q contain an  ity. Inasmuch as no agreement existed 
interim bal­ance sheet as of the end of  at the balance sheet date, the note 
the most recent fiscal quarter and a  payable must be classified as a current 
balance sheet as of the end of the  liabil­ity. Therefore, answers (a), (c), 
preceding fiscal year. An interim  and (d) are incorrect. 
balance sheet for the fiscal quarter of 
106. (b) The requirement is to identify the 
the pre­ceding year does not need to be 
provided unless it is neces­sary for  item requiredto be disclosed on the 
understanding the impact of seasonal  income statement. The correct answer is 
fluctuations.  (b) because the income statement may be
prepared by presenting expenses either 
102. (b) The requirement is to identify the  by nature or by function. The minimum 
deadline forfiling a Form 10­K for an  required disclosures on the income 
accelerated filer. Answer (b) is correct  statement in­clude income, finance costs,
because the maximum number of days  share of profits and losses using the 
for an accel­erated filer to file a 10­K  equity method, tax expense, discontinued
with the SEC is 75 days after the  operations, profit or loss, noncontrolling 
company’s fiscal year­end. However, a interests in profits and losses, and the net
large accelerated filer with $700  profit (loss) attributable to equity holders
million of public float has a deadline  of the parent. Therefore, answers (a), (c),
of 60 days, and nonaccelerated filers  and (d) are incorrect. 
have a deadline of 90 days. 
107. (c) The requirement is to identify the 
D.12. Financial Statements of Trust item that maynot be disclosed on the 
income statement under IFRS. An­
103. (c) The requirement is to identify the financial 
swer (c) is correct because gain or 
state­ments of a trust. A statement of assets and  loss from extraordinary items is not 
liabilities, a state­ment of operations, and a  allowed on an income statement 
statement of changes in net assets are generally  prepared using IFRS. Answers (a), 
presented for a trust.  (b), and (d) are items that are 
104. (d) The requirement is to identify the  disclosed on the income statement. 
accountingand measurement bases for 108. (b) The requirement is to identify 
a trust. The financial statements of a 
where the financecosts are presented 
trust should be presented on the 
in the statement of cash flows. An­
accrual basis and the assets are 
swer (b) is correct because under 
generally measured at their fair 
IFRS finance costs (interest expense) 
values. 
may be reported in either the 
D.13. International Financial Reporting  operating or financing section of the 
statement of cash flows. However, 
MODULE 9  BASIC TH. & FIN. REP.:  D. FINANCIAL STATEMENTS 323

once it is disclosed in a particular 
section, it must be re­ported on a 
consistent basis. Therefore, answers
(a), (c), and (d), are incorrect.
109. (d) The requirement is to 
identify how the transac­
tion should be reported on
the statement of cash 
flows. An­swer (d) is 
correct because this 
transaction did not 
involve an exchange of 
cash; therefore, it is not 
included on the statement 
of cash flows. IFRS 
requires that significant 
non­cash transactions be 
reported in the notes to 
the financial statements. 
(Note that for US GAAP, 
if there are only a few 
significant noncash 
transactions, they may be 
reported at the bottom of 
the statement of cash 
flows, or they may be re­
ported in a separate 
schedule in the notes to 
the financial statements.) 
110. (a) The requirement is to 
identify how cash ad­
vances and loans from 
bank overdrafts should be 
reported on the statement 
of cash flows. Answer (a) 
is correct because IFRS 
requires cash advances 
and loans from bank 
overdrafts to be classified 
as operating activities. 
111. (b) The requirement is 
to identify the 
acceptablemethods for 
presenting other 
comprehensive income. 
An­swer (b) is correct 
because IFRS provides 
that comprehen­sive 
income may be 
presented in either one 
statement or in two 
statements. (US GAAP 
allows the presentation 
in all three ways.) 
112. (a) The requirement is to 
identify the true 
statementabout IFRS 
requirements for financial
statements. An­swer (a) is
correct because IFRS 
requires the presentation 
of prior year financial 
statements for 
comparative purposes. 
113. (a) The requirement is 
to identify the manner 
inwhich operating 
expenses may be 
classified on the income
statement under IFRS. 
Answer (a) is correct 
because they may be 
classified by nature or 
function. 
114. (a) The requirement is to 
identify how the 
statementof cash flows 
may be presented. Answer
(a) is correct because the 
statement may be 
presented on the direct or 
the indirect basis. 
324 MODULE 9 BASIC TH. & FIN. REP.: D. FINANCIAL STATEMENTS

S
O
T

Concepts
Authoritative
Literature Help

1. The gain or loss from discontinued operations is placed in a separate category under other
income or loss.
2. The gain or loss from infrequent or unusual items is given extraordinary treatment and
disclosed on the income statement after discontinued operations.
3. Exit and disposal activities are classified as discontinued operations.
4. A component of a company can be classified as discontinued in the first period that it meets
the criteria as being held for sale.
5. A correction of an error is included in the cumulative effect of change in accounting
principle on the income statement.
6. Other comprehensive income may be presented at the bottom of the income statement.
7. Separate earnings per share amounts must be presented for both other comprehensive
income and comprehensive income.
8. Prospective financial information includes information on the purpose of the statements,
assumptions, and significant accounting policies.

E
1. (F)   Discontinued   operations   are   placed   in   a   separate   category   after   income   from
continuing operations and before extraordinary items. 
2. (F)  An item must be both infrequent and unusual to be treated as an extraordinary item. 
3. (F) Exit and disposal activities that are not in connection with a component of the entity that qualifies for 
discontinued operations or extraordinary treatment should be reported as unusual or infrequent 
income/loss in income from continuing operations before income taxes. 
4. (T)  A component must be classified as discontinued when it meets the criteria. 
5. (F)  A correction of an error requires restatement of the financial statements. 
6. (T)  Flexibility is allowed for presentation. 
7. (F) Separate EPS amounts are only required for gains/losses from continuing operations,
discontinued   operations,   extraordinary   items,   and   cumulative   effect   of   accounting
changes. 
8. (T)  The notes to prospective financial statements must include those disclosures. 

Task­Based Simulation 2
Financial Statement
Classification Authoritative
Literature

Help

1. Dividend   payable   (on   Craven’s   preferred


stock). 
2. Plant   construction   in   progress   by   the
company. 
3. Factory building (retired from use and held for
sale). 
4. Land (held for possible future building site). 
5. Merchandise   inventory   (held   by
Craven Corporation on consignment). 
6. Stock dividend distributable (in common stock
to common stockholders and to be issued at
par). 
7. Office supplies inventory. 
Assets Liabilities Owner’s equity Other
(A­F) (G, H, I) (J, K, L, M) (N or X)
G
C
F
B
N

A
MODULE 9  BASIC TH. & FIN. REP.:  D. FINANCIAL STATEMENTS 325

8. Sinking   fund   cash   (First   National   Bank,


Trustee). 

9. Installment   sales   accounts


receivable (average collection
period   eighteen   months).   All
sales are installment sales. 
10. Temporary decline in inventory value. 
11. Advances   to   officers   (indefinite   repayment
date). 
12. Estimated warranty costs. The  warranty costs
are for a one­year warranty on parts and labor. 
13. Inventory of small tools used in the business. 
14. Treasury stock under par value method. 
15. Common   stock   subscribed   (Craven
Corporation’s stock). 
16. Convertible bonds. 
17. Securities held as collateral. 
18. Bank overdraft (only account with bank). 
Explanation of solutions
Assets Liabilities Owner’s equity Other
(A­F) (G, H, I) (J, K, L, M) (N or X)
B
A

N
F
G

C
X­K
K
H
N
G
1. (G) When a corporation declares a cash or property dividend, the amount to be paid becomes 
a liability of the corpora­tion. The dividends payable amount would be classified as a current 
liability on the balance sheet. It is important to note that dividends payable represent the 
amount distributed to stockholders as a return on their investment. Dividends are not 
expenses. 
2. (C) When an item of property, plant, or equipment is being constructed by the company that 
intends to use it, all of the relevant costs related to the construction should be included in the 
asset. The asset would be classified as plant and equipment. 
3. (F) A factory building that has been retired from use and held for sale should be classified as 
an other asset rather than as property, plant and equipment. The property, plant and 
equipment account should include only those tangible assets that are being used in operations. 
4. (B) Land that is held for speculative or investment purposes should be classified as an 
investment rather than as property, plant, and equipment. The property, plant, and equipment 
account should represent only those assets being used in current op­erations. 
5. (N) Consigned inventory is excluded from the balance sheet because it is not owned by 
Craven. Consigned inventory represents an arrangement whereby the owner of the goods 
transfers physical possession to an agent (Craven). The agent (con­signee) will attempt to sell
the goods on the owner’s behalf. The inventory remains an asset of its owner. 
6. (K) A stock dividend distributable represents a dividend to be distributed to shareholders in 
the form of additional shares of the corporation’s stock. Each shareholder will receive a 
proportional share of additional stock. The declaration of a stock dividend does not result in a
liability, as it does not result in any of the corporation’s assets being paid. Thus, a stock 
dividend distributable would be classified as an addition in the stockholders’ equity section in
the common stock account. 
7. (A) Current assets represent cash or other assets that are expected to be used within the
operating cycle. Office supplies inventory represents an asset that could be expected to be
used within the operating cycle. 
8. (B) Generally, a fund is a group of assets set aside for a future nonoperating purpose. A sinking fund
contains assets to be used in the future to retire bonds. These assets are generally invested while waiting
to be used and are noncurrent. 
9. (A) An installment sales accounts receivable results when the corporation makes a sale to a 
customer and does not receive payment in full on the date of sale. Since this company 
normally sells on the installment basis, its operating cycle becomes eighteen months. Its 
receivables are current assets because their average life falls within the operating cycle. 
10. (N)  Temporary decline in inventory value is a loss and appears on the income statement. 
11. (F) Advances to officers are considered nontrade receivables. These types of receivables
should be reported separately on the balance sheet as other assets. 
12. (G) A warranty is a guarantee made by the seller to the purchaser against defects in the 
product’s quality. Estimated war­ranty expense for a given period can be estimated as a 
percentage of sales. The percentage is based on past warranty experi­ence. The estimated 
warranty cost represents the warranty expenditures in the future for past sales and is a current 
liability. 
13. (C) Property, plant, and equipment consists of items used in the normal operations of a
business. The inventory of small tools represents tools used in the business and are not for
sale. 
14. (X­K) Treasury stock is a corporation’s own stock which has been issued and reacquired
by  the   corporation.  Treasury   stock   may  be   accounted   for  under  the   cost  or   par   value
method. When the par value method is used, treasury stock would be recorded on the
balance sheet as a reduction or contra to the common stock account. 
326 MODULE 9 BASIC TH. & FIN. REP.: D. FINANCIAL STATEMENTS

15. (K) Common stock subscribed represents stock subscriptions that have not been fully paid. The stock is not considered to be 
issued until the full price is paid. This account should be classified as an increase in the common stock and it is offset in 
stockholders’ equity by a contra account for the balance of the subscription due. 
16. (H) A convertible bond is a bond which may be converted to another form of the corporation’s securities during a
speci­fied time frame. A bond is typically classified as a long­term liability on the balance sheet. 
17. (N) Corporations do not own securities they hold as collateral. The corporation will retain the securities only in the 
event of a default on the receivable the securities are collateralizing. Thus, the securities are excluded from the 
balance sheet of Craven and they appear on the balance sheets of their owners. 
18. (G) A bank overdraft occurs when a check is written for a greater amount than the balance in the bank account. A
bank overdraft should be classified as a current liability. It should not be offset against other cash account balances
unless the cash account is in the same bank. 

Task­Based Simulation 3
Balance Sheet
Authoritative
Literature Help

A B C D E
1 Stephen King Corporation
2 Balance Sheet
3 December 31, 2011
4
5 Current assets:
6 Cash $ 114,000
7 Trading securities 80,000
8 Accounts receivable (net of $10,000 allowance for doubtful accounts) 160,000
9 Inventories (lower of FIFO cost or market) 180,000
10
11 Total current assets 534,000
12
13 Investments:
14 Available­for­sale securities 270,000
15 Bond sinking fund 250,000
16 Land held for future use 270,000
17 Cash surrender value of life insurance contract 40,000
18
19 Total investments 830,000
20
21 Property, plant, and equipment:
22 Land 500,000
23 Buildings (net of accumulated depreciation of $360,000) 680,000
24 Equipment (net of accumulated depreciation of $180,000) 270,000
25
26 Total property, plant, and equipment 1,450,000
27
28 Intangible assets:
29 Franchise 165,000
30 Goodwill 100,000
31
32 Total intangible assets 265,000
33 Total assets $3,079,000
34
35 Current liabilities:
36 Accounts payable $90,000
37 Notes payable 80,000
38 Taxes payable 40,000
39 Bank overdraft 14,000
40 Unearned revenue 5,000
41
42 Total current liabilities 229,000
43
44 Long­term liabilities:
MODULE 9  BASIC TH. & FIN. REP.:  D. FINANCIAL STATEMENTS 327
A B C D E
45 Notes payable 120,000
46 Bonds payable, 10% due in 2020 (less discount of $40,000) 960,000
47
48 Total long­term liabilities 1,080,000
49 Total liabilities $1,309,000
50
51 Stockholders’ equity:
52 Paid­in capital
53 Preferred stock, no par, authorized 200,000 shares, issued 70,000 shares 450,000
54 Common stock, $1.00 par value, authorized 40,000 shares, issued 100,000 shares 100,000
55 Paid­in capital in excess of par value common 900,000
56
57 Total paid­in capital $1,450,000
58
59 Retained earnings 320,000
60
61 Total stockholders’ equity 1,770,000
62 Total liabilities and stockholders’ equity $3,079,000

Task­Based Simulation 4
Research
Authoritative
Literature Help

ASC 220 10 45 13

Task­Based Simulation 5
Journal Entries
Authoritative
Literature Help

1. Bad debt expense 16,000
Allowance for uncollectible account 16,000

2. Supplies expense 23,000
Supplies 23,000
[remaining = 23,000 (60,000 – 37,000)]

3. Depreciation expense 29,000
Accumulated depreciation 29,000
(640,000 – 60,000) ÷ 20 years = 29,000 per year

4. Salary expense 37,500
Salaries payable 37,500

5. Interest receivable 1,500
Interest income 1,500
75,000 × 12% × 2/12 = 1,500

6. Rent expense 35,000
Prepaid rent 35,000
168,000 ÷ 24 mo. = 7,000 per month × 5 months = 35,000
328 MODULE 9 BASIC TH. & FIN. REP.: D. FINANCIAL STATEMENTS

Task­Based Simulation 6
Calculations
Authoritative
Literature Help

Accounts receivable (net) 234,000 (1)


Notes receivable 75,000 (2)
Prepaid rent 133,000 (3)
Supplies 37,000 (4)
Inventory 420,000 (5)
Equipment (historical cost) 640,000 (6)
Accounts payable 176,000 (7)
Salaries payable 52,500 (8)
Accumulated depreciation 203,000 (9)
Explanation of solutions
1. $250,000 – $16,000 allowance for uncollectible accounts = $234,000 
2. No change.  Interest receivable is recorded in a separate account. 
3. $168,000 – $35,000 = $133,000 
4. Given in the problem. 
5. No adjustments were made to inventory. 
6. Equipment is at historical cost. 
7. No change. 
8. $15,000 + $37,500 = $52,500 

9. $174,000 + $29,000 = $203,000 

Task­Based Simulation 7
Classification/
Disclosures
Authoritative
Literature Help

1. ($73,000, A) The net carrying amount of the bonds redeemed is determined by adjusting the carrying amount of the bondsat 
the beginning of the year for any partial year amortization of corresponding premiums or discounts and for any unamortized 
issue costs. In this example, Griffin would determine its discount amortization by computing the difference between its bond 
interest expense and its bond interest paid as of June 30, 2011.
Bond interest expense Bond interest paid
Carrying amount of bonds Effective Face value of Stated Amortization
× ×
at beginning of period interest rate – bonds interest rate = amount

$940,000 × 10% × 1/2 – $1,000,000 × 8% 1/2 = $7,000

This amount is added to the carrying amount of the bonds at the beginning of the year to determine the net carrying amount: 
$940,000 + $7,000 = $947,000. This amount is subtracted from the reacquisition price to determine the loss on redemption: 
$1,020,000 ($1,000,000 × 1.02) – $947,000 = $73,000. Gains or losses from early redemption of bonds are treated as gains or 
losses in determining net income. Early extinguishment of debt is not generally considered an extraordinary item.
2. ($50,000, A) The accounting change is accounted for on a prospective basis. Therefore, it is necessary to calculate 
thebook value of the asset as of the beginning of the year of the change.
SYD
2008 $500,000 × 5/15  = $166,667
2009 $500,000 × 4/15 = $133,333
2010 $500,000 × 3/15 = $100,000
$400,000
On January 1, 2011, the book value is calculated as follows:
Historical cost $520,000
– Accumulated depreciation (400,000)
Book value 1/1/11 $120,000
MODULE 9  BASIC TH. & FIN. REP.:  D. FINANCIAL STATEMENTS 329

Straight­line depreciation for 2011 and future years is calculated as
($120,000 BV – $20,000 salvage value) ÷ 2 years remaining life = $50,000 per year
The depreciation expense would be shown as a part of the calculation of income from continuing operations.
3. ([$125,000], A) Since an accrual was made for Griffin’s contingent liability in 2010, one must first determine the 
amount of the liability recorded. If a range of possible losses exists and no amount within the range is more likely 
than any other amount, an accrual should be made for the minimum amount of the loss, in this case $250,000. As 
Griffin accrued a $250,000 liability and loss in 2010, the 2011 loss would be $125,000 ($250,000 – $375,000). Such
a loss would be classified under in­come from continuing operations as it is incurred in conjunction with ordinary 
operations (amount paid to employee). Such a loss would not be extraordinary as it does not meet the criteria of 
being both unusual in nature and infrequent in occurrence. 
4. ([$10,000], A) Trading securities are securities that are bought and held principally for the purpose of selling them in 
the near term. As Griffin purchased these securities to sell in the near term, any unrealized holding gain or loss would
appear as income from continuing operations on the income statement. The holding gain or loss from marketable 
equity securities held in the trading portfolio is computed as of December 31, 2011, by comparing the cost of the 
securities at the purchase date with the fair market value at the end of the year. 
Cost FV 12/31/11 Gain (loss)
I $125,000 $145,000 $ 20,000
II 235,000 205,000 (30,000)
$(10,000)
5. ([$255,000], B) The amount of gain or loss from the hurricane would be computed by adding the carrying amounts of 
theassets destroyed and comparing this amount with the amount of insurance recovery. For Griffin, an extraordinary loss 
would result as follows:
Insurance recovery $ 1,000,000
Warehouse 470,000
Equipment 250,000
Inventory 535,000
Total carrying value $ 1,255,000
Extraordinary loss $ (255,000)

As this hurricane is unusual in that it is not related to the normal activities of the entity and infrequent in occurrence in that it is 
not expected to occur again in the foreseeable future (this was the first hurricane ever in Griffin’s area), the $255,000 loss 
would be classified as an extraordinary loss.
6. ($41,000, E) To compute the translation adjustment for 2011, both net asset figures in dollars must be compared. 
The$429,000 represents the translation at the date of purchase plus the translation of net income at a weighted­average 
exchange rate. The $388,000 represents the net assets at the translated rate at the balance sheet date. The $41,000 difference is 
the trans­lation adjustment necessary to make the balance sheet balance. This translation adjustment should not be included in 
net income, but shall be reported as a separate component of other comprehensive income.

Task­Based Simulation 8
Research
Authoritative
Literature Help

ASC 420 10 25 14

C
l
A
u
L
i

1. N
e
2. A

3. R

4. W

5. G
a
330 MODULE 9 BASIC TH. & FIN. REP.: D. FINANCIAL STATEMENTS

(A) (B) (C) (D) (E) (F) (G) (H) (I)
6. Changing from the gross profit method for determining year­end inventory balances to dollar value LIFO. 
7. Accounting for existing construction contracts is changed from completed contract to percentage­of­
completion. 
8. The effects of a change in estimate and a change in principle are inseparable for the same event. 
9. The excess of cash paid over the carrying value to extinguish bonds. 
10. Income or loss of the component for the period of disposal included in this calculation. 
Explanation of solutions
1. (G) The acquisition of assets does not necessitate that the same method of depreciation be used for 
the newly acquired assets as for the existing assets. Therefore, no change in accounting principle 
has occurred because the previously recorded assets will continue to be depreciated using the 
straight­line method. 
2. (A)  Accounting for the acquisition of a 100% owned subsidiary is properly accounted for as a change in
reporting entity. 
3. (B) All corrections of errors should be treated as a restatement of the prior period financial statements. 
This requires prompt recording of the error in the year in which the error was discovered, and 
reporting the effects of the error in the financial statements as an adjustment to the carrying value of 
the assets and liabilities as of the beginning of the first period presented with an offsetting adjustment 
to the opening balance of retained earnings for that period. Financial statements for each individual 
period are adjusted to reflect the correction of the period­specific effects of the error. 
4. (H) For an item to qualify as an extraordinary item it must be both unusual in nature and infrequent
in occurrence. Clearly, the write­down of inventory due to obsolescence does not qualify for 
extraordinary treatment; therefore, it should be disclosed separately in the income statement before 
extraordinary items. 
5. (I) Gains or losses on the disposal of net assets is included in the calculation of gain or loss on 
disposal of discontinued operations. Therefore, the total gain or loss on disposal should consist of 
the following two calculations: the net asset gain or loss mentioned above and the income or loss 
from operations from the measurement date to the disposal date (the phase­out period). 
6. (B) The use of the gross profit method to determine year­end inventory balances is not proper
GAAP. Therefore, if a situation such as this existed, it should be treated as a correction of an error.
7. (C) The change from one generally accepted accounting principle to another is considered a change in
accounting princi­ple. Since both the completed contract method and the percentage­of­completion 
methods are recognized GAAP, the change from one to the other is properly accounted for as a 
change in accounting principle, and accounted for on a retrospective basis. 
8. (D) When it is impossible to determine whether a change in accounting principle or a change in estimate has
occurred, the change should be considered as a change in estimate. 
9. (H)  Gains or losses on extinguishment of debt are no longer treated as extraordinary items. 
10. (F) Income or loss from operations is properly included in the calculation of the gain or loss on the 
discontinued operations. Therefore, this calculation includes both the income or loss from operations 
and the gain or loss on the disposal of net assets. The gain or loss on the disposal should also be 
disclosed parenthetically or in the notes to the financial statements. 

Task­Based Simulation 10
Multistep Income
Statement Authoritative
Literature Help

American Horse Company
INCOME STATEMENT
For the Year Ended December 31, 2010

Sales $25,000,000
Less cost of goods sold (17,000,000)
Gross profit $ 8,000,000
Less selling and administrative expenses (4,700,000)
Income from operations $ 3,300,000
Other revenue and gains
Interest revenue $ 70,000
Gain on the sale of investments 110,000 $180,000
Other expenses and losses
MODULE 9  BASIC TH. & FIN. REP.: D. FINANCIAL STATEMENTS 331

Write­off of goodwill (820,000)
Income from continuing operations before income taxes $2,660,000
Income taxes (905,000)
Income from continuing operations $1,755,000
Discontinued operations
Loss on discontinued wholesale division, including loss
on disposal of $200,000 $815,000
Income tax benefit (285,000)
Loss on discontinued operation (net of tax) (530,000)
Income before extraordinary item $1,225,000
Extraordinary loss from flood damage, net of tax (390,000)
Net income $  835,000

Earnings per share
a
Income from continuing operations $5.62
Discontinued operations: (1.77)
b
Income before extraordinary item 3.85
Extraordinary loss (net of tax) (1.30)
c
Net income $2.55
   

a $1,755,000 – $70,000
= $5.62
300,000 shares
b $1,225,000 – $70,000
= $3.85
300,000 shares
c $835,000 – $70,000
= $2.55
300,000 shares

Task­Based Simulation 11
Research
Authoritative
Literature Help

ASC 225 20 45 2

F
iC
l

L
i

1. A
n
2. T
h
3. I
n
4. A
 
5. A
 
6. A
 
7. T

8. T
(

1. (B) Changes in the market value of (trading type) marketable securities shall be included in the 
determination of incomefrom continuing operations of the period in which the change in market value occurs.
332 MODULE 9 BASIC TH. & FIN. REP.: D. FINANCIAL STATEMENTS

2. (E) The accumulated changes in the valuation (unrealized excess of cost over market value) of available­for­sale
mar­ketable equity securities shall be reported as accumulated other comprehensive income. 
3. (F) Results of operations of a component that has been or will be discontinued should be reported separately as a compo­nent
of income, after income from continuing operations but before extraordinary items. 
4. (B) A foreign subsidiary must remeasure its financial statements when its functional currency (the currency of the 
pri­mary economic environment in which the entity operates) is different from its local currency (the currency 
currently used by the entity). The effect of the remeasurement process (gain or loss) should be recognized currently in 
income. Aggregate gain or loss for the period should also be disclosed either in the financial statements or in the 
notes. 
5. (C) When translating a foreign subsidiary’s financial statements from functional currency to reporting currency, a transla­tion 
adjustment will result. The translation adjustment is not included in net income but is reported as a component of compre­
hensive income and called other comprehensive income. 
6. (D) Extraordinary items are events and transactions that are both unusual and infrequent. The earthquake in this 
problem occurred in an area previously considered subject to only minor tremors. Therefore, the earthquake would be 
considered an extraordinary event. Extraordinary items should be shown separately on the income statement, after 
income from continuing operations. 
7. (F) The question indicates that it is probable that the company will receive $1,000,000 from a pending lawsuit. Thus, 
the $1,000,000 is a gain contingency. Contingencies that may result in gains are not reflected in the accounts, as 
doing so might result in the recognition of revenue before its realization. Adequate disclosure of the gain contingency 
should be made, but care should be taken to avoid misleading implications as to the likelihood of realization. 
8. (B) The cost of research and development services performed by others on behalf of the enterprise are included in 
R&D costs. R&D costs are charged to expense when incurred and thus are included in income from continuing 
operations of that period. In addition, separate disclosure of the total R&D costs for the period should be made 
either in the financial statements or in the notes. 

Task­Based Simulation 13
Calculate Net Income
and Earnings Per Share Authoritative
Literature Help

Rap Corp.
INCOME STATEMENT
For the Year Ended December 31, 2010
Income from continuing operations before income tax $1,180,000
Income tax (448,400)
Income from continuing operations $ 731,600
Discontinued operations:
Loss from discontinued operations (including loss from
disposal of $100,000) $(190,000)
Income tax benefit 72,200
Loss from discontinued operations (net of tax) 117,800
Income before extraordinary item $ 613,800
Extraordinary item:
Gain on sale of investment 145,000
Less applicable taxes 58,000 87,000
Net income $ 700,800
Per share of common stock:
Income from continuing operations $  7.32
Discontinued operations, net of tax (1.18)
Income before extraordinary item 6.14
Extraordinary item, net of tax 0.87
Net income ($700,800/100,000) $  7.01
MODULE 10   INVENTORY 333

INVENTORY
MULTIPLE­CHOICE QUESTIONS (1­68)
1. The following information applied to Fenn, Inc. 
for2011:
Merchandise purchased for resale $400,000
Freight­in 10,000
Freight­out 5,000
Purchase returns 2,000

Fenn’s 2011 inventoriable cost was
a. $400,000 
b. $404,000 
c. $408,000 
d. $413,000 
2. On December 28, 2011, Kerr Manufacturing Co. 
pur­chased goods costing $50,000. The terms 
were FOB desti­nation. Some of the costs 
incurred in connection with the sale and delivery 
of the goods were as follows: 
Packaging for shipment $1,000
Shipping 1,500
Special handling charges 2,000
These goods were received on December 31, 2011. In 
Kerr’s December 31, 2011 balance sheet, what amount 
of cost for these goods should be included in inventory?
a. $54,500 
b. $53,500 
c. $52,000 
d. $50,000 
3. On June 1, 2011, Pitt Corp. sold merchandise with a list 
price of $5,000 to Burr on account. Pitt allowed trade dis­
counts of 30% and 20%. Credit terms were 2/15, n/40 and 
the sale was made FOB shipping point. Pitt prepaid $200 of 
delivery costs for Burr as an accommodation. On June 12, 
2011, Pitt received from Burr a remittance in full payment 
amounting to 
a. $2,744 
b. $2,940 
c. $2,944 
d. $3,140 
4. The following information was taken from Cody
Co.’s   accounting   records   for   the   year   ended
December 31, 2011: 
Decrease in raw materials inventory $  15,000
Increase in finished goods inventory 35,000
Raw material purchased 430,000
Direct labor payroll 200,000
Factory overhead 300,000
Freight­out 45,000

There was no work in process inventory at the beginning 
or end of the year. Cody’s 2011 cost of goods sold is
a. $895,000 
b. $910,000 
c. $950,000 
d. $955,000 
5. The following information pertains to Deal Corp.’s
2011 cost of goods sold:
Inventory, 12/31/10 $  90,000
2011 purchases 124,000
2011 write­off of obsolete inventory 34,000
Inventory, 12/31/11 30,000

The inventory written off became obsolete due to an unex­
pected and unusual technological advance by a competitor.
In its 2011 income statement, what amount should  b. $118,220 
Deal report as cost of goods sold? c. $123,360 
a. $218,000  d. $128,500 
b. $184,000 
c. $150,000  9. When allocating costs to inventory produced
d. $124,000  for the period, fixed overhead should be based
upon 
6. How should the following costs affect a retailer’s in­ a. The actual amounts of goods produced
ventory? during the period. 
Freight­in Interest on inventory loan b. The normal capacity of production facilities. 
a. Increase No effect c. The highest production levels in the
b. Increase Increase last three periods. 
c. No effect Increase d. The   lowest   production   level   in
d. No effect No effect the last three periods. 
7.  According   to   the   net   method,   which   of   the 10. Per the Codification, what is  considered the
followingitems   should   be   included   in   the   cost   of normal capacity of production facilities? 
inventory? a. The   average   production   over   the   previous   five­
Freight costs Purchase discounts not taken year period. 
a. Yes No b. Actual production for the period. 
b. Yes Yes c. Actual   production   for   the   period   plus   loss   of
c. No Yes capac­ity for planned maintenance. 
d. No No d. A   range   that   may   vary   based   on
8. The following information pertained to Azur Co. for  business   and   industry­specific
factors. 
theyear:
Purchases $102,800 11. How   should   unallocated   fixed   overhead
Purchase discounts 10,280 costs be treated? 
Freight in 15,420 a. Allocated to finished goods and cost of
Freight out 5,140
Beginning inventory 30,840
goods sold based on ending balances in
Ending inventory 20,560 the accounts. 
b. Allocated to raw materials, work in 
What amount should Azur report as cost of goods sold  process, and finished goods, based on the
for the year? ending balances in the accounts. 
a. $102,800 
334 MODULE 10   INVENTORY
Inventory
At base At dollar
c. Recognized   as Date year cost value LIFO
1/1/10 $90,000 $90,000
an   expense   in 2010 layer 20,000 30,000
the   period   in 2011 layer 40,000 80,000
which they are
incurred.  What was the price index used 
d. Allocated to  to compute Bach’s 2011 dollar 
value LIFO inventory layer?
work in process, 
finished goods,  a. 1.09 
and cost of  b. 1.25 
goods sold based c. 1.33 
on ending  d. 2.00 
balances in the  15. Nest   Co.   recorded   the
accounts. 
following   inventory
12. When   manufacturing information   during   the
inventory,   what   is   the month of January: 
accounting   treatment   for Total Units
abnormal freight­in costs? Units Unit cost cost on hand
a. Charge to expense for the Balance on 1/1 2,000 $1 $2,000 2,000
Purchased on 1/8 1,200 3 3,600 3,200
period.  Sold on 1/23 1,800 1,400
b. Charge   to   the   finished Purchased on 1/28 800 5 4,000 2,200
goods inventory. 
c. Charge   to   raw   materials Nest uses the LIFO method to cost
inventory.  inventory.   What   amount   should
d. Allocate   to Nest report as inventory on January
raw 31   under   each   of   the   following
materials, methods of recording inventory?
work   in Perpetual Periodic
process,   and a. $2,600 $5,400
finished b. $5,400 $2,600
goods.  c. $2,600 $2,600
d. $5,400 $5,400
13. On December 15, 2011, 
Flanagan purchased goods
costing $100,000. The 
terms were FOB shipping 
point. Costs incurred by 
Flanagan in connection 
with the purchase and 
delivery of the goods 
were as follows: 
Normal freight charges
Handling costs
Insurance on shipment
Abnormal freight charges for express shipping

The goods were received on 
December 17, 2011. What is the 
amount that Flanagan should charge
to inventory and to current period 
expense?
Inventory Current period expense
a. $3,000 $3,700
b. $5,000 $1,700
c. $5,500 $1,200
d. $6,700 $0
14. Bach Co. adopted the dollar­
value LIFO inventorymethod as of
January 1, 2011. A single 
inventory pool and an internally 
computed price index are used to 
compute Bach’s LIFO inventory 
layers. Information about Bach’s 
dollar value inventory follows:
and net realizable value. The net 
16. The weighted­average for the  realizable value less normal profit 
year inventory cost flowmethod is  margin is below the original cost. 
applicable to which of the  Under the lower of cost or market 
following inventory systems? method, the inventory item should 
be valued at 
Periodic Perpetual
a. Replacement cost. 
a. Yes Yes
b. Yes No b. Net realizable value. 
c. No Yes c. Net   realizable   value   less
d. No No normal profit margin. 
d. Original cost. 
17. During January 2011, Metro 
Co., which maintains aperpetual  21. Which of the following
inventory system, recorded the  statements   are   correct
following informa­tion pertaining  when   a   company
to its inventory: applying   the   lower   of
cost   or   market   method
Units Unit cost Total cost
Balance on 1/1/11 1,000 $1 re­ports its inventory at
Purchased on 1/7/11 600 3 replacement cost? 
Sold on 1/20/11 900 I.  The original cost is less than 
Purchased on 1/25/11 400 5
replacement cost.
Under the moving­average  AI. The net realizable value is greater
method, what amount should  than replacement cost. 
Metro report as inventory at  a. I only. 
January 31, 2011? b. II only. 
a. $2,640  c. Both I and II. 
b. $3,225  d. Neither I nor II. 
c. $3,300 
d. $3,900 
18. Based on a physical 
inventory taken on 
December 31, 2011, 
Chewy Co. determined its
chocolate inventory on a 
FIFO basis at $26,000 
with a replacement cost of
$20,000. Chewy 
estimated that, after 
further processing costs of
$12,000, the chocolate 
could be sold as finished 
candy bars for $40,000. 
Chewy’s normal profit 
margin is 10% of sales. 
Under the lower of cost or
market rule, what amount 
should Chewy report as 
chocolate inventory in its 
December 31, 2011 
balance sheet? 
a. $28,000 
b. $26,000 
c. $24,000 
d. $20,000 
19. Reporting   inventory   at
the   lower   of   cost   or
market   is   a   departure
from   the   accounting
principle of 
a. Historical cost. 
b. Consistency. 
c. Conservatism. 
d. Full disclosure. 
20. The original cost of an inventory 
item is below both replacement cost
MODULE 10   INVENTORY 335
b. Perpetual Individual item
c. Periodic Total inventory
22. The original cost of an  d. Periodic Individual item
inventory item is above the 25. Marsh Company had 150 units
re­placement cost and the  of product A on hand atJanuary 1,
net realizable value. The 
2011, costing $21 each. Purchases
replace­ment cost is below
of product A during the month of
the net realizable value 
January were as follows:
less the normal profit 
margin. As a result, under  Units Unit cost
the lower of cost or market Jan. 10 200 $22
18 250 23
method, the inventory item 28 100 24
should be reported at the 
a. Net realizable value.  A physical count on January 31, 
b. Net   realizable   value   less 2011, shows 250 units of product A
the normal profit margin.  on hand. The cost of the inventory 
c. Replacement cost.  at January 31, 2011, under the 
d. Original cost.  LIFO method is
a. $5,850 
23. On January 1, 2011, Card  b. $5,550 
Corp. signed a three­year  c. $5,350 
noncancelable purchase  d. $5,250 
contract, which allows 
Card to pur­chase up to  26. During January 2011, 
500,000 units of a  Metro Co., which 
computer part annually  maintains a perpetual 
from Hart Supply Co. at  inventory system, 
$.10 per unit and  recorded the following 
guarantees a minimum  informa­tion pertaining to 
annual purchase of  its inventory: 
100,000 units. During  Unit Total Units
2011, the part  Units cost cost on hand
unexpectedly became  Balance on 1/1/11 1,000 $1 $1,000 1,000
Purchased on 1/7/11 600 3 1,800 1,600
obsolete. Card had  Sold on 1/20/11 900 700
250,000 units of this  Purchased on 1/25/11 400 5 2,000 1,100
inventory at December 31,
2011, and believes these 
parts can be sold as scrap 
for $.02 per unit. What 
amount of probable loss 
from the purchase 
commitment should Card 
report in its 2011 income 
statement? 
a. $24,000 
b. $20,000 
c. $16,000 
d. $  8,000 
24. Thread Co. is selecting its inventory 
system in prepara­tion for its first 
year of operations. Thread intends to
use either the periodic weighted­
average method or the perpetual 
moving­average method, and to 
apply the lower of cost or market 
rule either to individual items or to 
the total inven­tory. Inventory prices
are expected to generally increase 
throughout 2011, although a few 
individual prices will de­crease. 
What inventory system should 
Thread select if it wants to maximize
the inventory carrying amount at De­
cember 31, 2011? 
Inventory method Cost or market application
a. Perpetual Total inventory
c. Decrease Decrease
Under the LIFO method, what  d. Decrease Increase
amount should Metro report as 
29.  Generally,   which   inventory
inventory at January 31, 2011?
costing   method   approxi­mates
a. $1,300 
most closely  the current cost  for
b. $2,700 
each of the follow­ing?
c. $3,900 
d. $4,100  Cost of goods sold Ending inventory
a. LIFO FIFO
27. Drew Co. uses the average cost  b. LIFO LIFO
inventory method for internal  c. FIFO FIFO
reporting purposes and LIFO for  d. FIFO LIFO
financial statement and income tax 
30. During periods of rising prices, 
reporting. At December 31, 2011, 
the in­ventory was $375,000 using  a perpetual inventorysystem would 
average cost and $320,000 us­ing  result in the same dollar amount of 
LIFO. The unadjusted credit balance ending inventory as a periodic 
in the LIFO Re­serve account on  inventory system under which of 
December 31, 2011, was $35,000.  the following inventory cost flow 
What adjusting entry should Drew  methods?
record to adjust from average cost to FIFO LIFO
LIFO at December 31, 2011?  a. Yes No
Debit b. Yes Yes
a. Cost of goods sold $55,000 c. No Yes
Inventory d. No No
b. Cost of goods sold $55,000 31.  On   January   1,   2010,   Poe
LIFO reserve
Company adopted the dollar­value
c. Cost of goods sold $20,000
Inventory LIFO   inventory   method.   Poe’s
d. Cost of goods sold $20,000 entire   inventory   con­stitutes   a
LIFO reserve single   pool.   Inventory   data   for
2010 and 2011 are as follows:
28. A company decided to change  Inventory Inventory Relevant
its inventory valuationmethod from at current at base price
Date year cost year cost index
FIFO to LIFO in a period of rising  1/1/10 $150,000 $150,000 1.00
prices. What was the result of the  12/31/10 220,000 200,000 1.10
change on ending inventory and net 12/31/11 276,000 230,000 1.20
income in the year of the change? Poe’s LIFO inventory value at 
Ending inventory Net income
December 31, 2011, is
a. Increase a. $230,000 
b. Increase b. $236,000 
336 MODULE 10   INVENTORY
cost,   and,   in
the
c. $241,000  denominator,
d. $246,000  the   ending   in­
ventory at base
32. Brock Co. adopted the  year cost. 
dollar­value LIFO  c. In   the
inventory method as of  numerator,   the
January 1, 2010. A single ending inventory
inventory pool and an  at base year cost,
internally computed price and,   the
index are used to  denominator, the
compute Brock’s LIFO  ending inventory
inventory layers.  at   current   year
Information about  cost. 
Brock’s dollar­value  d. In   the
inventory follows:  denominator,
Inventory the   average   of
At base At current the   ending   in­
Date year cost year cost
1/1/10 $40,000 $40,000 ventory  at   base
2010 layer 5,000 14,000 year cost and at
12/31/10 45,000 54,000 current   year
2011 layer 15,000 26,000
12/31/11 $60,000 $80,000
cost. 

What was Brock’s dollar­value  35. Jones Wholesalers stocks 


LIFO inventory at Decem­ber 31, a changing variety of 
2011? prod­ucts. Which 
a. $80,000  inventory costing method 
b. $74,000  will be most likely to give
c. $66,000  Jones the lowest ending 
inventory when its 
d. $60,000 
product lines are subject 
33. Estimates   of   price­level to specific price 
changes   for   specific increases? 
inventories are required for a. Specific identification. 
which   of   the   following b. Weighted­average. 
inventory methods?  c. Dollar­value LIFO. 
a. Conventional retail.  d. FIFO periodic. 
b. Dollar­value LIFO.  36. Dart   Company’s
c. Weighted­average cost.  accounting   records
d. Average cost retail.  indicated the fol­lowing
34. When the double­ information: 
extension approach to the  Inventory, 1/1/11 $  500,000
dollar­value LIFO  Purchases during 2011 2,500,000
Sales during 2011 3,200,000
inventory method is used, 
the inventory layer added  A physical inventory taken on 
in the current year is  December 31, 2011, resulted in an 
multiplied by an index  ending inventory of $575,000. 
number. Which of the  Dart’s gross profit on sales has 
following correctly states  remained constant at 25% in recent
how components are used  years. Dart suspects some 
in the calculation of this  inventory may have been taken by 
index number?  a new employee. At December 31, 
a. In   the 2011, what is the estimated cost of 
numerator,   the missing inventory?
average   of   the a. $  25,000
ending   inven­
tory at base year
cost   and   at
current   year
cost. 
b. In   the
numerator,   the
ending
inventory   at
current   year
a. $1,500,000 
b. $100,000  b. $1,590,000 
c. $175,000  c. $1,620,000 
d. $225,000  d. $1,710,000 
37. On   July   1,   2011,   Casa 39. Kew Co.’s accounts payable 
Development   Co. balance at December 31,2011, was 
purchased a tract of land $2,200,000 before considering the 
for   $1,200,000.   Casa following data:
incurred   additional   cost
of   $300,000   during   the • Goods shipped to Kew FOB 
remainder   of   2011   in shipping point on De­
cember 22, 2011, were 
preparing   the   land   for
lost in transit. The invoice 
sale.   The   tract   was cost of $40,000 was not 
subdivided   into recorded by Kew. On 
residential   lots   as January 7, 2012, Kew 
follows:  filed a $40,000 claim 
Lot class Number of lots Sales price per lot against the common 
A 100 carrier. 
B 100 • On December 27, 2011, a 
C 200 vendor authorized Kew to 
Using the relative sales value  return, for full credit, goods 
method, what amount of costs  shipped and billed at $70,000 
should be allocated to the Class A  on December 3, 2011. The 
lots? returned goods were shipped 
by Kew on December 28, 
a. $300,000 
2011. A $70,000 credit memo 
b. $375,000  was received and recorded by 
c. $600,000  Kew on January 5, 2012. 
d. $720,000  • Goods   shipped   to   Kew   FOB
destination on Decem­ 
38. Herc Co.’s inventory at 
ber 20, 2011, were received on
December 31, 2011,  January 6, 2012. The in­voice
was$1,500,000 based on a  cost was $50,000. 
physical count priced at cost, 
and before any necessary  What amount should Kew report 
adjustment for the following: as accounts payable in its 
December 31, 2011 balance 
• Merchandise costing  sheet?
$90,000, shipped FOB  a. $2,170,000 
shipping point from a 
b. $2,180,000 
vendor on December 30, 
c. $2,230,000 
2011, was re­ceived and 
recorded on January 5, 
d. $2,280,000 
2012.  40. Lewis Company’s usual 
• Goods in the shipping area sales terms are net sixty 
were excluded from in­ days, FOB shipping point. 
ventory although  Sales, net of returns and 
shipment was not made  allowances, totaled 
until Janu­ary 4, 2012. 
$2,300,000 for the year 
The goods, billed to the 
customer FOB shipping  ended December 31, 2011,
point on December 30,  before year­end 
2011, had a cost of  adjustments. Additional 
$120,000.  data are as follows: 
What amount should Herc report as
inventory in its Decem­ber 31, 
2011 balance sheet?
MODULE 10   INVENTORY 337
Holden reported the sale 
of ten freezers and 
• On December 27, 2011, Lewis remitted $8,500. The 
authorized a customer to  remittance was net of the 
return, for full credit, goods agreed 15% commission. 
shipped and billed at  What amount should 
$50,000 on December 15,  Grimm recognize as 
2011. The returned goods  consignment sales reve­
were received by Lewis on  nue for 2011? 
January 4, 2012, and a 
a. $  7,700 
$50,000 credit memo was 
issued and recorded on the  b. $  8,500 
same date.  c. $  9,800 
• Goods with an invoice  d. $10,000 
amount of $80,000 were 
billed and recorded on  43. The  following items were
January 3, 2012. The  included   in   Opal   Co.’s
goods were shipped on  inven­tory   account   at
December 30, 2011.  December 31, 2011: 
• Goods with an invoice  Merchandise out on consignment, at sales price, includ­
amount of $100,000 were  ing 40% markup on selling price $40,000
billed and recorded on  Goods purchased, in transit, shipped FOB shipping point 36,000
December 30, 2011. The  Goods held on consignment by Opal 27,000
goods were shipped on  By   what   amount   should   Opal’s
January 3, 2012.  inventory account at Decem­ber 31,
Lewis’ adjusted net sales for 2011  2011, be reduced?
should be a. $103,000 
a. $2,330,000  b. $  67,000 
b. $2,280,000  c. $  51,000 
c. $2,250,000  d. $  43,000 
d. $2,230,000  44. On   December   1,   2011,   Alt
41. On January 1, 2011, Dell,  Department Store received 
Inc. contracted with the  505 sweaters on consignment 
city of Little to provide  from Todd. Todd’s cost 
custom built desks for the  for the sweaters was $80 
city schools. The contract  each, and they were 
made Dell the city’s sole  priced to sell at $100. 
supplier and required Dell  Alt’s commission on 
to supply no less than  consigned goods is 10%. 
4,000 desks and no more  At December 31, 2011, 
than 5,500 desks per year  five sweaters remained. 
for two years. In turn,  In its Decem­ber 31, 
Little agreed to pay a fixed 2011 balance sheet, what 
price of $110 per desk.  amount should Alt report 
During 2011, Dell pro­ as payable for consigned 
duced 5,000 desks for  goods? 
Little. At December 31, 
2011, 500 of these desks 
were segregated from the 
regular inventory and were
accepted and awaiting 
pickup by Little. Little 
paid Dell $450,000 during 
2011. What amount should
Dell recognize as contract 
revenue in 2011? 
a. $450,000 
b. $495,000 
c. $550,000 
d. $605,000 
42. On October 20, 2011, 
Grimm Co. consigned 
forty freezers to Holden 
Co. for sale at $1,000 each 
and paid $800 in 
transportation costs. On 
December 30, 2011, 
inventory amounts were 
a. $49,000  essentially unchanged. 
b. $45,400  Which of the following 
c. $45,000  statements explains the 
d. $40,400  increased inventory 
turnover ratio? 
45. Southgate Co. paid the in­transit  a. Cost   of   goods   sold
insurance premium for consignment  decreased. 
goods shipped to Hendon Co., the  b. Accounts   receivable
consignee. In addition, Southgate  turnover increased. 
advanced part of the commissions  c. Total   asset   turnover
that will be due when Hendon sells  increased. 
the goods. Should Southgate include d. Gross   profit   percentage
the in­transit insurance premium and decreased. 
the advanced commissions in 
inventory costs?  50. Selected   data
Insurance premium pertaining
Advanced commissions   to   Lore   Co.
a. Yes for   the   calendar   year
b. No 2011 is as follows: 
c. Yes Net cash sales $  3,000
d. No Cost of goods sold 18,000
Inventory at beginning of year 6,000
46. Jel Co., a consignee, paid  Purchases 24,000
the freight costs for goods  Accounts receivable at beginning of year 20,000
shipped from Dale Co., a  Accounts receivable at end of year 22,000
consignor. These freight  Lore would use which of the 
costs are to be deducted  following to determine the 
from Jel’s payment to  average days’ sales in 
Dale when the con­ inventory?
signment goods are sold. 
Until Jel sells the goods, 
the freight costs should be 
included in Jel’s 
a. Cost of goods sold. 
b. Freight­out costs. 
c. Selling expenses. 
d. Accounts receivable. 
47. Heath Co.’s current ratio
is   4:1.   Which   of   the
follow­ing   transactions
would normally increase
its current ratio? 
a. Purchasing   inventory   on
account. 
b. Selling   inventory   on
account. 
c. Collecting   an   account
receivable. 
d. Purchasing   machinery   for
cash. 
48. During 2011, Rand Co. purchased 
$960,000 of inven­tory. The cost of 
goods sold for 2011 was $900,000, 
and the ending inventory at 
December 31, 2011, was $180,000. 
What was the inventory turnover for
2011? 
a. 6.4 
b. 6.0 
c. 5.3 
d. 5.0 
49. In a comparison of 2011 
to 2010, Neir Co.’s 
inventory turnover ratio 
increased substantially 
although sales and 
Numerator Denominator
a. 365 Average inventory
b. 365 Inventory turnover
c. Average inventory Sales divided by 365
338 MODULE 10   INVENTORY

d. Sales divided by 365 Inventory turnover d.   $0


Items 55 and 56 are based on the 
51. Cord Builders, Inc. has 
following data pertainingto Pell 
consistently used the percentage­of­
Co.’s construction jobs, which 
completion method of accounting for 
commenced during 2011:
construction­type contracts. During 
2010 Cord started work on a 
$9,000,000 fixed­price construction 
contract that was completed in 2012. 
Cord’s accounting records disclosed 
the following:

Cumulative contract costs incurred $3,900,000
Estimated total cost at completion 7,800,000
Project 1 Project 2
Contract price $420,000 $300,000
Costs incurred during 2011 240,000 280,000
Estimated costs to complete 120,000 40,000
Billed to customers during 2011 150,000 270,000
Received from customers during 2011 90,000 250,000
completion method of 
How much income would Cord recognizing income. 
have recognized on this  During 2011, Hansen 
contract for the year ended  started work on a 
December 31, 2011? $3,000,000 fixed­price 
a. $100,000  construction contract. 
b. $300,000  The accounting records 
c. $600,000  dis­closed the following 
d. $700,000  data for the year ended 
December 31, 2011: 
52. State Co. recognizes 
Costs incurred $  930,000
construction revenue and  Estimated cost to complete 2,170,000
expenses using the  Progress billings 1,100,000
percentage­of­completion  Collections 700,000
method. During 2010, a 
How much loss should Hansen have 
single long­term project 
was begun, which  recognized in 2011?
continued through 2011.  a. $230,000 
Information on the project  b. $100,000 
follows:  c. $  30,000 
2010
Accounts receivable from
construction contract $100,000
Construction expenses 105,000
Construction in progress 122,000
Partial billings on contract 100,000

Profit recognized from the long­
term construction contract in 2011
should be
a. $  50,000 
b. $108,000 
c. $128,000 
d. $228,000 
53. Lake Construction 
Company has 
consistently used the 
percentage­of­
completion method of 
recognizing income. 
During 2010, Lake 
entered into a fixed­price
contract to construct an 
office building for 
$10,000,000. 
Information relating to 
the contract is as 
follows: 

Percentage of completion
Estimated total cost at completion $7,500,000
Income recognized (cumulative) 500,000
Contract costs incurred during 2011 
were
a. $3,200,000 
b. $3,300,000 
c. $3,500,000 
d. $4,800,000 
54. Hansen Construction, 
Inc. has consistently 
used the percentage­of­
55. If Pell used the  Progress Income
completed contract  billings to date previously recognized
method, what amount  a. Yes No
of gross profit (loss)  b. No Yes
would Pell report in its  c. No No
2011 income statement? d. Yes Yes
a. $ (20,000)  59. The calculation of the 
b. $ 0 
income recognized in the 
c. $ 340,000 
third year of a five­year 
d. $ 420,000 
construction contract 
56. If Pell used the  accounted for using the 
percentage­of­completion  percentage­of­completion 
method, what amount of  method includes the ratio of
gross profit (loss) would  a. Total costs incurred to date
Pell report in its 2011  to total estimated costs. 
income statement?  b. Total costs incurred to date
a. $(20,000)  to total billings to date. 
b. $ 20,000  c. Cost incurred in year three
c. $ 22,500  to total estimated costs. 
d. $ 40,000  d. Costs incurred in year three
to total billings to date. 
57. Which of the following is
used   in   calculating   the 60. When should an 
in­come   recognized   in anticipated loss on a 
the fourth and final year long­term con­tract be 
of   a   contract   accounted recognized under the 
for by the percentage­of­ percentage­of­completion
completion method?  method and the 
Actual total costs completed­contract 
Income previously recognized
a. Yes method, respectively? 
b. Yes Percentage­of­completion Completed­contract
c. No a. Over life of project Contract complete
d. No b. Immediately Contract complete
c. Over life of project Immediately
58. A company used the  d. Immediately Immediately
percentage­of­completion methodof
accounting for a five­year  61. In accounting for a long­
construction contract. Which of the  term construction contractusing 
following items will the company  the percentage­of­completion 
use to calculate the income  method, the progress billings on 
recognized in the third year? contracts account is a
MODULE 10   INVENTORY 339

a. FIFO is allowed. 
a. Contra   current   asset b. Interest   costs   may   be
account.  capitalized   if   there   is   a
b. Contra   noncurrent   asset leng­thy   production   period
account.  to prepare goods for sale. 
c. Noncurrent   liability c. The   weighted­average
account.  method is acceptable. 
d. Revenue account.  d. Inventories
are   always
62.  Brady   Corporation   values   its
valued at net
inventory at the lower ofcost or net realizable
realizable   value   as   required   by value. 
IFRS.   Brady   has   the   following
information regarding its inventory: 68. Which   of   the   following
Historical cost methods   of   accounting
Estimated selling price for   in­ventory   is   not
Estimated costs to complete and sell allowed under IFRS? 
Replacement cost a. LIFO. 
What is the amount for inventory  b. Specific identification. 
that Brady should report on the  c. FIFO. 
balance sheet under the lower of  d. Weighted­average. 
cost or net realizable value method?
a. $1,000 
b. $  900 
c. $  850 
d. $  750 
63. A company determined the 
following values for its in­ventory 
as of the end of its fiscal year:
Historical cost
Current replacement cost
Net realizable value
Net realizable value less a normal profit margin
Fair value

Under IFRS, what amount should the company report as inventory on its 
balance sheet?
a. $70,000 
b. $85,000 
c. $90,000 
d. $95,000 
64. Under IFRS, which of the following inventory items are not valued at the lower
of cost or net realizable value? 
a. Manufactured inventory items. 
b. Retail inventory items. 
c. Biological inventory items. 
d. Industrial inventory items. 
65. Under IFRS, the specific identification method of ac­counting for
inventory is required for 
a. All inventory items. 
b. Inventory items which are interchangeable. 
c. Inventory items that are not interchangeable and goods that 
are produced and segregated for specific projects. 
d. Biological (agricultural) inventories. 
66. The information provided below is for an item in Harris Corporation’s
inventory at year end. Harris presents its financial statements in 
accordance with IFRS: 
Historical cost $1,200
Estimated selling price 1,300
Estimated completion and selling costs 150
Replacement cost 1,100
What should be the value of this inventory item in the com­pany’s financial 
statements?
a. $1,100 
b. $1,150 
c. $1,200 
d. $1,300 
67. Which of the following is not true about accounting for inventory
under IFRS? 
340 MODULE 10   INVENTORY

SIMULATIONS

Task­Based Simulation 1
Concepts
Authoritative
Literature Help

Indicate whether each of the following is included in the cost of inventory.
Included Not included
1. Merchandise purchased for resale
2. Freight­out
3. Direct materials
4. Sales returns
5. Packaging for shipment to customer
6. Factory overhead
7. Interest on inventory loan
8. Purchase discounts not taken
9. Freight­in
10. Direct labor

Task­Based Simulation 2
Journal Entries
Authoritative
Literature Help

Blaedon uses the periodic inventory method for inventories. Prepare the journal entries for each of the following 
transactions. Blaedon uses the gross method for recording inventory transactions.
1. On January 5, 2011, purchased $17,000 of garden tillers on account from Bestbuilt Tillers, terms 2/10, n/30,
FOB destination. Freight charges were $200. 
2. On January 10, 2011, returned garden tillers worth $2,000 to Bestbuilt Tillers due to defects. 
3. On January 14, 2011, paid for the remaining tillers purchased in 1.
4. On January 28, 2011, purchased $30,000 of lawn mowers from Lawn Giant, terms 3/10, n/30, FOB shipping point.
The freight charges were $820. 
5. On February 6, 2011, paid for the lawn mowers purchased in part 4. from Lawn Giant. 

Task­Based Simulation 3
Calculations
Authoritative
Literature Help

Situation
A client, Blaedon Co., sells lawn mowers and garden tillers. The garden tillers are purchased from Bestbuilt Tillers and
sold to customers without modification. The lawn mowers, however, are purchased from several contractors. Blaedon then
makes ongoing design refinements to the mowers before selling them to customers.
The lawn mowers cost $200. Blaedon then makes the design refinements at a cost of $85 per lawn mower. Blaedon stores 
the lawn mowers in its own warehouse and sells them directly to retailers at a list price of $500. Blaedon uses the FIFO inven­
tory method. Approximately two­thirds of new lawn mower sales involve trade­ins. For each used lawn mower traded in and 
returned to Blaedon, retailers receive a $40 allowance regardless of whether the trade­in was associated with a sale of a 2011 or
2012 model. Blaedon’s net realizable value on a used lawn mower averages $25.
At December 31, 2011, Blaedon’s inventory of new lawn mowers includes both 2011 and 2012 models. When the 2012
model was introduced in September 2011, the list price of the remaining 2011 model lawn mowers was reduced below cost. 
Blaedon is experiencing rising costs.
Blaedon has contacted your firm for advice on how to report the carrying value of inventory, the impact of the decline 
in value on the 2011 models, and the effects of using the FIFO method on their December 31, 2011 financial statements.
Assume that Blaedon had the following information regarding the garden tiller inventory:
MODULE 10 INVENTORY 341

Purchases $210,000
Purchase discounts 38,000
Purchase returns 17,500
Freight­in 12,100
Freight­out 18,000
Beginning inventory 42,900
Ending inventory 34,250
Calculate the following items:
1. Goods available for sale 
2. Costs of goods sold 

Task­Based Simulation 4
Research
Authoritative
Literature Help

Assume that you are assigned to the audit of the inventories of Litton Corporation. Research the Professional Standards 
for the section that provides guidance on the items that should be included in the cost of inventory. Enter your response in the 
answer fields below.

Task­Based Simulation 5
Calculation of
Gross Profit
Authoritative
Literature Help
Situation
London, Inc. began operation of its construction division on October 1, 2010, and entered into contracts for two separate 
projects. The Beta project contract price was $600,000 and provided for penalties of $10,000 per week for late completion. 
Although during 2011 the Beta project had been on schedule for timely completion, it was completed four weeks late in 
August 2012. The Gamma project’s original contract price was $800,000. Change orders during 2012 added $40,000 to the 
original contract price.
The following data pertains to the separate long­term construction projects in progress:
Beta Gamma
As of September 30, 2011:
Costs incurred to date $360,000 $410,000
Estimated costs to complete 40,000 410,000
Billings 315,000 440,000
Cash collections 275,000 365,000
As of September 30, 2012:
Costs incurred to date 450,000 720,000
Estimated costs to complete ­­ 180,000
Billings 560,000 710,000
Cash collections 560,000 625,000

Additional information
• London accounts for its long­term  construction contracts  using the percentage­of­completion method for financial
report­ing purposes and the completed­contract method for income tax purposes. 
• Enacted income tax rates are 25% for 2011 and 30% for future years. 
• London’s income before income taxes from all divisions, before considering revenues from long­term construction
proj­ects, was $300,000 for the year ended September 30, 2011. There were no other temporary or permanent
differences. 
Prepare a schedule showing London’s gross profit (loss) recognized for the years ended September 30, 2011, and 2012, 
un­der the percentage­of­completion method.
342 MODULE 10   INVENTORY
London Inc.
SCHEDULE OF GROSS PROFIT (LOSS)
Beta Gamma
For the Year Ended September 30, 2011:
Estimated gross profit (loss):
Contract price
Less total costs
Estimated gross profit (loss)
Percent complete:
Costs incurred to date
Total costs
Percent complete
Gross profit (loss) recognized
For the Year Ended September 30, 2012:
Estimated gross profit (loss):
Contract price
Less total costs
Estimated gross profit (loss)
Percent complete:
Costs incurred to date
Total costs
Percent complete
Gross profit (loss) recognized
Less gross profit (loss) recognized in prior year
Gross profit (loss) recognized

Task­Based Simulation 6
Financial Statement Authoritative
Disclosures Literature Help

Situation

London, Inc. began operation of its construction division on October 1, 2010, and entered into contracts for two separate 
projects. The Beta project contract price was $600,000 and provided for penalties of $10,000 per week for late completion. 
Although during 2011 the Beta project had been on schedule for timely completion, it was completed four weeks late in 
August 2012. The Gamma project’s original contract price was $800,000. Change orders during 2012 added $40,000 to the 
original contract price.
The following data pertains to the separate long­term construction projects in progress:
Beta Gamma
As of September 30, 2011:
Costs incurred to date $360,000 $410,000
Estimated costs to complete 40,000 410,000
Billings 315,000 440,000
Cash collections 275,000 365,000
As of September 30, 2012:
Costs incurred to date 450,000 720,000
Estimated costs to complete ­­ 180,000
Billings 560,000 710,000
Cash collections 560,000 625,000

Additional information
• London   accounts   for   its   long­term   construction   contracts   using   the   percentage­of­completion   method   for   financial
report­ing purposes and the completed­contract method for income tax purposes. 
• Enacted income tax rates are 25% for 2011 and 30% for future years. 
• London’s income before income taxes from all divisions, before considering revenues from long­term construction
proj­ects,  was  $300,000 for the  year ended September 30, 2011. There  were  no other temporary or  permanent
differences. 
Prepare the following schedule showing London’s balances in the following accounts at September 30, 2011, under 
the percentage­of­completion method:
• Accounts receivable 
• Costs and estimated earnings in excess of billings 
• Billings in excess of costs and estimated earnings 
MODULE 10   INVENTORY 343
London Inc.
SCHEDULE OF SELECTED BALANCE SHEET ACCOUNTS
September 30, 2011
Accounts receivable
Costs and estimated earnings in excess of billings:
Construction in progress
Less:  Billings
Costs and estimated earnings in excess of billings
Billings in excess of costs and estimated earnings
Estimated loss on contract

Task­Based Simulation 7
Research
Authoritative
Literature Help

Assume that you are assigned to the audit of Cole Construction Company. Cole constructs buildings under long­term 
contracts using the completed contract basis of accounting. Cole has determined that a loss is anticipated on its contract 
with Hale corporation. Which section of the Professional Standards provides guidance on how this situation should affect 
Cole’s financial statements? Enter your response in the answer fields below.

Task­Based Simulation 8
Inventory Concepts
Authoritative
Literature Help

Items 1 through 13 represent True or False statements concerning inventory accounting methods. Indicate whether 
eachstatement is true or false by clicking on the appropriate response.

1. T

2. T
h
3. I
n
4. A
 
5. U
n
6. T
h
7. T
h
8. U
n
9. A

10. D
u
11. I
n
12. T
h
13. U
n
344 MODULE 10   INVENTORY

Task­Based Simulation 9
Cost Flow Concepts
Authoritative
Literature Help

Below are statements describing inventory and cost flow methods. Identify which method matches each description by
clicking on the correct term and placing it in the space provided. You may use a term once, more than once, or not at all.
Terms
Specific identification
Weighted­average
Simple average
Moving­average
First­in, first­out
Last­in, first­out
Dollar­value LIFO
Gross profit

Method
1. During a period of rising prices, this method results in a higher net income. 
2. This method most closely matches the physical flow of inventory. 
3. Method that uses historical sales margins to estimate ending inventory. 
4. Method that is appropriate when there is a relatively small number of significant dollar value items
in in­ventory. 
5. Average cost must be calculated each time additional inventory is purchased. 
6. Method that averages the cost of all items on hand and purchased during the period. 
7. This method results in the lowest ending inventory in a period of rising prices. 
8. Method that uses a price index to measure changes in inventory. 
9. If used for tax purposes, this method must also be used for financial reporting purposes. 
10. The cost of goods sold balance is the same whether a perpetual or periodic inventory system is used. 

Task­Based Simulation 10
Schedule of Ending Authoritative
Inventory Literature Help

Situation

York Co. sells one product, which it purchases from various suppliers. York’s trial balance at December 31, 2011, 
in­cluded the following accounts:
Sales (33,000 units @ $16) $528,000
Sales discounts 7,500
Purchases 368,900
Purchase discounts 18,000
Freight­in 5,000
Freight­out 11,000

York Co.’s inventory purchases during 2011 were as follows:
Units Cost per unit Total cost
Beginning inventory, January 1 8,000 $8.20 $ 65,600
Purchases, quarter ended March 31 12,000 8.25 99,000
Purchases, quarter ended June 30 15,000 7.90 118,500
Purchases, quarter ended September 30 13,000 7.50 97,500
Purchases, quarter ended December 31 7,000 7.70 53,900
55,000 $434,500
Additional information
York’s accounting policy is to report inventory in its financial statements at the lower of cost or market, applied to total
in­ventory. Cost is determined under the last­in, first­out (LIFO) method.
York has determined that, at December 31, 2011, the replacement cost of its inventory was $8 per unit and the net 
realiz­able value was $8.80 per unit. York’s normal profit margin is $1.05 per unit.
From this information, complete the following schedules.
MODULE 10   INVENTORY 345

York Co.
SUPPORTING SCHEDULE OF ENDING INVENTORY
December 31, 2011
Inventory at cost (LIFO):
Cost
Units per unit   Total cost
Beginning inventory, January 1
Purchases, quarter ended March 31
Purchases, quarter ended June 30
Totals

York Co.
SCHEDULE OF COST OF GOODS SOLD
For the Year Ended December 31, 2011
Beginning inventory
Add:  Purchases
Less:  Purchase discounts
Add:  Freight­in
Goods available for sale
Less:  Ending inventory
Cost of goods sold

Task­Based Simulation 11
Research
Authoritative
Literature Help

Assume that you are assigned to the audit of Heath Corporation. Heath is considering changing its method of 
costing inventory. Which section of the Professional Standards provides guidance on the appropriate methods for 
determining inven­tory cost? Enter your response in the answer fields below.
346 MODULE 10 INVENTORY

MULTIPLE­CHOICE ANSWERS

1. c __ __ 15. b __ __ 29. a __ __ 43. d __ __ 57. a __ __


2. d __ __ 16. b __ __ 30. a __ __ 44. c __ __ 58. b __ __
3. c __ __ 17. b __ __ 31. c __ __ 45. c __ __ 59. a __ __
4. b __ __ 18. c __ __ 32. c __ __ 46. d __ __ 60. d __ __
5. c __ __ 19. a __ __ 33. b __ __ 47. b __ __ 61. a __ __
6. a __ __ 20. d __ __ 34. b __ __ 48. b __ __ 62. c __ __
7. a __ __ 21. b __ __ 35. c __ __ 49. d __ __ 63. c __ __
8. b __ __ 22. b __ __ 36. a __ __ 50. b __ __ 64. c __ __
9. b __ __ 23. c __ __ 37. c __ __ 51. a __ __ 65. c __ __
10. d __ __ 24. a __ __ 38. d __ __ 52. a __ __ 66. b __ __
11. c __ __ 25. c __ __ 39. a __ __ 53. b __ __ 67. d __ __
12. a __ __ 26. b __ __ 40. d __ __ 54. b __ __ 68. a __ __
13. c __ __ 27. d __ __ 41. c __ __ 55. a __ __ 1st:  __/68 = __%
14. d __ __ 28. c __ __ 42. d __ __ 56. b __ __ 2nd: __/68 = __%

MULTIPLE­CHOICE ANSWER EXPLANATIONS

A. Determining Inventory and Cost of Goods Sold Purchase price less returns ($400,000 – $2,000) $398,000


Freight­in 10,000
1. (c) Inventoriable costs include all costs necessary  $408,000
toprepare goods for sale. For a merchandising concern  Note that freight­out is a selling expense, not an 
these costs include the purchase price of the goods,  inventoriable cost, as the diagram below indicates.
freight­in, insurance, warehousing, and any costs 
necessary to get the goods to the point of sale (except  Customer
interest on any loans ob­tained to purchase the goods). In 
this problem, inventoriable costs total $408,000. Inventoriable costs   Point   Selling expense
 of sale (1) Raw materials purchased $430,000
Decrease in RM inventory 15,000
2. (d) When the shipping terms are FOB destination,  Raw materials used $445,000
the seller bears all costs of transporting the goods  (2) Beginning WIP $ --
to the buyer. Therefore, the seller is responsible  RM used (from above) 445,000
Direct labor 200,000
for the payment of packaging costs ($1,000), 
Factory overhead 300,000
shipping costs ($1,500), and the special handling  Cost to account for 945,000
charges ($2,000). The only amount to be included  Ending WIP --
as the buyer’s cost of the inventory purchased is  Cost of goods manuf. $945,000
the purchase price ($50,000).  (3) Cost of goods manuf. $945,000
Less: increase in FG inv. 35,000
3. (c) Purchases are always recorded net of trade dis­counts.  Cost of goods sold $910,000
When more than one trade discount is applied to a list price,
it is called a chain discount. Chain discounts are applied in  The decrease in RM inventory is added when computing RM 
steps; each discount applies to the previously dis­counted  used because RM were used in excess of those purchased. 
price. The cost, net of trade discounts, is $2,800 [$5,000 –  The increase in FG inventory is deducted when computing 
(30% × $5,000) = $3,500; and $3,500 – (20% × $3,500) =  cost of goods sold because it represents the portion of goods 
$2,800]. Payment was made within the discount period, so  manufactured which were not sold. The freight­out is 
the net purchase price is $2,744 [$2,800 – (2% × $2,800)].  irrelevant for this question because freight­out is a selling 
The remittance from Burr would also include re­ expense and therefore does not affect cost of goods sold.
imbursement of the $200 of delivery costs. Since the terms 
were FOB shipping point, Burr is responsible for paying  5. (c)    To compute cost of goods sold, the solutions
this amount, and must reimburse Pitt, who prepaid the  approach is to set up a T­account for inventory
freight. Thus, the total remittance is $2,944 ($2,744 +  Inventory
$200).  12/31/10 90,000
Purchases 124,000 34,000 Write­off
4. (b) Three computations must be performed: raw  ? Cost of goods sold
materials used, cost of goods manufactured, and  12/31/11 30,000
cost of goods sold.  Purchases increase inventory, while the write­off and cost 
of goods sold decrease inventory. Cost of goods sold can 
be computed as $150,000 using the T­account. An 
alternate solutions approach is to use the CGS 
computation

BI $ 90,000
+ Purchases 124,000
CGAS 214,000
– EI (30,000)
$34,000 recognized as inventory loss*
$184,000
$150,000 recognized as CGS

* Theoretically correct treatment. 

6. (a) The cost of inventory should include all expendi­tures 
(direct and indirect) incurred to bring an item to its ex­
isting condition and location. Freight­in charges are thus 
ap­propriately included in inventory costs. Interest cost 
shall not be capitalized for assets that are in use or ready for
their in­tended use in the earnings activities of the 
enterprise. Thus, 
MODULE 10   INVENTORY 347
prepare goods for sale. For a merchandising concern, these 
include the purchase price of the goods, freight­in, 
interest on an inventory loan should not be included in  insurance, warehousing, and any costs necessary to get the 
in­ventory (it should be expensed as incurred). goods to the point of sale. Abnormal freight and handling 
should be charged to expense of the period. Therefore, the 
7. (a) The cost of inventory should include all  normal costs for inventory are $5,500 ($3,000 + $2,000 + 
expendi­tures (direct and indirect) incurred to  $500) and the abnormal freight of $1,200 is charged to 
bring an item to its ex­isting condition and  current expense of the period. 
location. Freight charges are thus appro­priately 
included in inventory costs. Under the net 
purchase method, purchase discounts not taken 
are recorded in a Pur­chase Discounts Lost 
account. When this method is used, purchase 
discounts lost are considered a financial (i.e., 
“other”) expense, and are thus excluded from the 
cost of in­ventory. 
8. (b)   Azur should report cost of goods sold calculated
as 
Cost of goods sold (CGS) = Beg. Inventory + Net 
purchases* + Freight in – Ending Inventory
CGS = $  30,840 + $92,520** + $15,420 – $20,560
CGS = $118,220
Freight out is a selling expense and does not enter the 
cal­culation of cost of goods sold.
* Net   purchase   =   Purchases   –   Purchase   returns   and
allowances – Purchase discounts 
* ($102,800 – $10,280) 

9. (b) Fixed overhead is allocated based on the 
normal capacity of the production facilities. 
Normal capacity is the production expected to be 
achieved over a number of periods or seasons 
under normal circumstances, taking into account 
the loss of capacity resulting from planned 
maintenance. Answer (a) is incorrect, because the
actual amount of production may only be used if 
it approximates normal capacity. Answers (c) 
and (d) are incorrect because the accounting 
standards do not specify a formula to calculate 
normal capacity. 
10. (d) Normal capacity refers to a range in 
production levels that will vary based on 
business and industry­specific factors. Normal 
capacity is the production expected to be 
achieved over a number of periods or seasons 
under normal circumstances, taking into account
the loss of capacity resulting from planned 
maintenance. Answer (a) is incorrect because the
Codification does not specify a formula for 
calculating normal capacity. Answers (b) and (c)
are incor­rect because actual production may 
only be used if it ap­proximates normal capacity.
11. (c) Unallocated fixed overhead costs are 
recognized as an expense in the period in which 
they are incurred. Therefore, answers (a), (b), 
and (d) are incorrect. 
12. (a) Any abnormal costs for freight, handling 
costs, and wasted material are required to be 
treated as current period charges, and not a part
of inventory cost. Therefore, answers (b), (c), 
and (d) are incorrect. 
13. (c) Inventoriable costs include all costs necessary to 
until the next purchase. After the 1/7/11 purchase, Metro 
B. Inventory Valuation and Cost­Flow Methods owns 1,600 units (1,000 + 600) at a total cost of $2,800 
($1,000 + $1,800). Therefore, the moving­average unit 
14. (d) Ending inventory at dollar value LIFO  cost at that time is $1.75 ($2,800 ÷ 1,600 units). After the 
iscalculated as the base year inventory times the 1/20/11 sale of 900 units (at a unit of cost of $1.75), Metro 
index. Therefore, the index used can be  owns 700 units at a unit cost of $1.75 (700 × $1.75 = 
calculated as 2.00 = ($80,000 EI at dollar value  $1,225). The 1/25/11 purchase of 400 units at a total cost 
LIFO/$40,000 base year cost).  of $2,000 increases inventory to its 1/31/11 balance of 
$3,225 ($1,225 + $2,000). The new unit cost (not required)
15. (b)   The inventory valuations are calculated as fol­ 
is $2.93 ($3,225 ÷ 1,100).
lows: 
Valuations of ending inventory under LIFO perpetual B.5. Lower of Cost or Market
1,400 units at $1.00 = $1,400 18. (c) The lower of cost or market (LCM) is used 
800 units at $5.00 = 4,000 forfinancial reporting of inventories. The market value of 
Total $5,400
in­ventory is defined as the replacement cost (RC), as long 
Value of ending inventory under LIFO periodic as it is less than the ceiling (net realizable value, or NRV) 
2,000 units at $1.00 = $2,000 and more than the floor (NRV less a normal profit, or NRV 
200 units at $3.00 = 600 – NP). In this case, the amounts are
Total $2,600 Ceiling:  NRV =  $40,000 est. sell. price
B.2. Weighted­Average – $12,000 disp. cost = $28,000
Floor:  NRV – NP = $28,000 – (10% × $40,000) $24,000
RC: $20,000
16. (b) The requirement is to determine whether 
theweighted­average inventory method is applicable to a  Since RC falls below the floor, the floor (NRV – NP) is 
periodic and/or a perpetual inventory system. The  the designated market value. Once market value is 
weighted­average method computes a weighted­average  designated, LCM can be determined by simply 
unit cost of inventory for the entire period and is used with  determining the lower of cost ($26,000) or market 
periodic records. The moving­average method requires that ($24,000). Therefore, inventory is reported at $24,000.
a new unit of cost be computed each time new goods are 
19. (a) SFAC 5 establishes five different attributes 
purchased and is used with perpetual records.
onwhich assets can be measured. The attribute used 
B.4. Moving­Average should be determined by the nature of the item and the 
relevance and reliability of the attribute measured. The 
17. (b) The moving­average method requires that a  five attributes are historical cost, current cost, current 
newunit cost be computed each time goods are purchased.  market value, net realiz­
The new unit cost is used to cost all sales of inventory 
348 MODULE 10   INVENTORY
original cost (meaning statement I is 
not correct), and RC must be less than
able value, and present value.  NRV (meaning statement II is 
Historical cost is defined as the  correct) and greater than NRV – NP.
amount of cash, or its equivalent, paid
to acquire an asset. Reporting  22. (b) Inventory is priced at market 
inventory at lower of cost or market is when market valueis less than cost. 
a departure from the historical cost  Market value is defined as current 
principle as the inventory could  replace­ment cost, subject to a 
potentially be carried at the market  ceiling of net realizable value (NRV)
value if lower. Although, reporting  and a floor of net realizable value 
inventory at lower of cost or market  minus a normal profit mar­gin.
does not create a departure from 
Original Cost
conservatism as this method carries at
inventory the lowest or most  Net Realizable Value 
conservative value. The use of LCM  (NRV)
does not violate the principle of 
consistency either, as it would be  NRV –  Normal Profit
reported on this basis continually. 
Finally the use of LCM would not  Replacement Cost
violate the principle of full disclosure 
In this situation, replacement cost 
as its use would be discussed in the 
lies outside of (below) the floor and 
footnotes.
ceiling limitations. Therefore, NRV 
20. (d) Inventory is to be valued at  less a normal profit margin (the 
the lower of cost ormarket. Under  floor), will be used as the market to 
this method, market is replacement  de­termine LCM. Since original 
cost provided that replacement cost cost is greater than market, market 
is lower than net realizable value  will be used to price the inventory 
(ceiling) and higher than net  for the period.
realizable value less the normal 
profit margin (floor). The question 
does not specify whether 
replacement cost is above or below
net realizable value, but since the 
original cost is below both of these
values, that information is 
irrelevant.
NRV/RC

OC

NRV -
NP

Either NRV or RC will be 
designated as the market value of 
the inventory, and since the original 
cost is below both of these values, 
the inventory will be valued at its 
original cost. Answer (c) is incorrect
because NRV – NP represents the 
market floor. Answers (a) and (b) 
are incorrect because they are both 
above the original cost.
21. (b) Lower of cost or market 
(LCM) is used for finan­cial reporting
of inventories. The market value of 
inventory is defined as the 
replacement cost (RC) as long as it is 
less than the ceiling (net realizable 
value, or NRV) and more than the 
floor (NRV less a normal profit, or 
NRV – NP). Therefore, if inventory is
reported at RC, RC must be less than 
First­Out (LIFO)
B.6.  Losses on Purchase 
Commitments 25. (c) The requirement is to 
determine the cost of the 
23. (c) The requirement is to  1/31/11 inventory, using the
determine the amount ofprobable loss LIFO method. LIFO stands 
from the purchase commitment that  for last­in, first­out; this 
Card should report in its 2011 income means that the cost of the 
statement. When there is a decline in  units pur­chased most 
market value below the contract price recently are included in cost
at the balance sheet date and the  of goods sold. Therefore, 
contract is noncancelable, an  the 1/31/11 inventory 
unrealized loss should be recorded in  consists of the 250 units that
the period of decline and reported in  were purchased at the 
the income statement. In this case,  earliest date(s). Thus, the 
Card has a contract to purchase a  1/31/11 inventory would 
minimum of 100,000 units both in  consist of the 150 units on 
2012 and 2013 at $.10 per unit. The  hand at 1/1/11 (150 × $21 = 
$20,000 loss (200,000 × $.10) on  $3,150) plus an additional 
these obsolete units should be  100 units purchased at the 
reduced by the amount Card believes  earliest purchase date in 
is realizable from the sale of these  January (January 10; 100 × 
units. Therefore, the loss on  $22 = $2,200). The total 
purchase commitment is $16,000  value of the inventory at 
[$20,000 – (200,000 × 1/31/11 would be $5,350 
.02)]. Additionally, Card Corp. would ($3,150 + $2,200). 
need to record a loss of $20,000 ($.08 26. (b) LIFO stands for last­in, 
× 250,000) from inventory  first­out; this means that it is
obsolescence. assumed that any units sold 
24. (a) To maximize its inventory  are the units most recently 
purchased. In a perpetual 
carrying amount atDecember 31, 
system, LIFO is applied at 
2011, Thread should use the 
the time of each sale rather 
perpetual moving­average method 
than once a year as in a 
with the lower of cost or market rule 
periodic system. Using 
applied to the total inventory. First, 
LIFO, the 900 units sold on 
when using the perpetual moving­
1/20/11 would consist of the
average method, the cost of sales 
600 units purchased on 
throughout the year are determined 
1/7/11 and 300 of the 1,000 
using the average cost of purchases 
units in the 1/1/11 balance. 
up to the time of the sale. On the 
This would leave in 
other hand, under the periodic 
inventory 700 units from the
weighted­average method, the cost 
1/1/11 balance. After the 
of each item is the weighted­average 
1/25/11 purchase, inventory 
of all units purchased during the 
included those 700 units 
year. During a period of rising 
plus the 400 units purchased
prices, the perpetual moving­average
on 1/25/11. Therefore, 
method results in a lower cost of 
ending inventory is $2,700 
goods sold and a higher ending 
[(700 × $1) + (400 × $5)]. 
inventory because the cost of items 
sold throughout the year is the  27. (d) When a company uses
average of the earlier, lower prices.  LIFO for external  report­
Second, the application of the lower  ing   purposes   and   another
of cost or market rule to the total  inventory   method   for
inventory will result in a higher  internal pur­ 
ending inventory because market  poses, a LIFO Reserve account is 
values lower than cost are offset  used to reduce inventory from the 
against market values higher than  internal valuation to the LIFO 
cost. valuation. LIFO Reserve is a contra 
B.7. & B.8. First­In, First­Out  account to inventory, and is adjusted 
up or
(FIFO), and Last­In, 
MODULE 10   INVENTORY 349
is used, the first/last pur­chase 
determination is made only at the end 
down at year­end with a  of the year, based upon the actual 
corresponding increase or decrease to  chronological order of all purchases. 
Cost of Goods Sold. In this case, the  When a perpetual method is used, 
LIFO Reserve accountmust be  however, the first/last purchase 
adjusted from a balance of $35,000 to determination is made continuously 
a balance of $55,000 ($375,000 –  throughout the year. When inventory 
$320,000). Therefore, LIFO Reserve  levels get low under the perpetual 
is credited for $20,000 ($55,000 –  method, early purchase costs will 
$35,000) with a corresponding debit  often be assigned to goods sold, a 
to Cost of Goods Sold. situation that is much less likely to 
occur in a periodic system. Therefore,
28. (c) In a change to LIFO, no  in times of either rising or falling 
recognition is given to any  prices, LIFO ending inventory is 
cumulative effect associated  usually different under a periodic 
with the change because it is  system than under a perpetual system.
usually not determinable. 
Thus, the effect on ending in­
B.9. Dollar­Value LIFO
ventory and net income is the  31. (c) When using dollar­value 
result solely of current year  LIFO, the ending in­ventory at 
effects. In a period of rising  current year cost must first be 
prices, LIFO will result in a  converted to base year cost. This 
lower ending inventory  amount is given at 12/31/11 
amount than FIFO because  ($230,000), but it could be computed 
theearlier lower costs are  as follows: $276,000 ÷ 1.20 = 
assumed to remain in ending  $230,000. The next step is to 
inventory. LIFO will also  determine the incremental LIFO 
result in a lower net income  layers at base year cost. The 1/1/10 
because the more recent 
(base year) layer is $150,000, the 
2010 layer is $50,000 ($200,000 – 
higher costs are assigned to 
$150,000), and the 2011 layer is 
cost of goods sold. 
$30,000 ($230,000 – $200,000). 
29. (a) The inventory costing method  Finally, the LIFO layers are restated 
which most closely approximates the  using the price index in effect at the 
current cost for cost of goods sold is  time each layer was added.
LIFO, while the method which more  Ending inventory
Base cost at DV LIFO cost
accurately reflects ending inventory is 1/1/10 layer $150,000 × 1.00 = $150,000
FIFO. Under LIFO, the most recent  2010 layer 50,000 × 1.10 = 55,000
purchases are assumed to be the first  2011 layer 30,000 × 1.20 = 36,000
goods sold; thus, cost of goods sold  $230,000 $241,000
contains relatively current costs. On 
the other hand, since FIFO assumes 
that the goods from beginning 
inventory and the earliest purchases 
are sold first, the ending inventory is 
made up of more recent purchases 
and thus represents a more current 
value. 
30. (a) Under the FIFO method, the first 
goods purchased are considered to be 
the first goods used or sold. Ending 
inventory is thus made up of the latest
(most recent) purchases. Whenever 
the FIFO method is used, the ending 
inventory is the same whether a 
perpetual or periodic system is used. 
This is true even during periods of 
rising or falling prices because the 
inventory flow is always in 
chronological order. Under the LIFO 
method, the latest (most recent) pur­
chases are considered to be the first 
goods used or sold. Ending inventory 
is thus made up of the first (oldest) 
pur­chases. When a periodic method 
determine the appropriate 
32. (c) When using dollar­value  use of ending inventory at 
LIFO, the ending in­ventory at  current year cost and 
current year cost must first be  ending in­ventory at base 
converted to base year cost. The  year cost in calculating the 
12/31/11 inventory at base year cost  dollar­value LIFO index. 
is given as $60,000. Since the  The index number used to 
12/31/10 inventory at base year cost  convert the current year’s 
was $45,000 ($40,000 base layer and  inventory layer is 
$5,000 2010 layer), a new layer of  calculated as follows: 
$15,000 was added in 2011 ($60,000  Ending inventory 
– $45,000). This layer must be  at current year cost
Index =
restated using the 2011 price index.  Ending inventory 
The 2011 price index is computed  at base year cost
using the double­extension technique, This index indicates the relationship 
as illustrated below. between current and base year prices 
EI at year­end prices as a percentage, and when multiplied 
= $80,000 by the new layer (which is the 
EI at base year prices $60,000
increase in inventory in base year 
Therefore, the 12/31/11 inventory  dollars), it will convert the layer to 
using dollar­value LIFO is $66,000  current dollars.
as computed below.
Base cost 35. (c) During periods of rising 
Base layer $40,000 prices, the inventorycosting methods 
2010 layer 5,000 which will give Jones the lowest 
2011 layer 15,000 × 1.33 ending inventory balance are LIFO 
methods, because inventory items that
33. (b) The requirement is to  were purchased at the earliest date 
determine which inventory  (when prices were lower) will remain 
method requires estimates  in inventory and the most recently 
of price level changes for  purchased and more expensive items 
specific inventories. In  will be expensed through cost of 
accordance with the dollar­ goods sold. Any FIFO method will 
value LIFO method, the  produce a higher ending inventory 
ending inventory is first  balance during inflation since the 
converted to the base year  items purchased earliest (at lower 
cost so that the incremental  prices) will be expensed through 
layers can be determined.  CGS, while the more expensive items 
The incremental layers are  remain in inventory. Answers (a) and 
then restated using the price (b) are incorrect because neither will 
index which was in effect at give Jones a lower ending inventory 
the time each of the layers  balance than dollar­value LIFO, 
were added to the  particularly as Jones’ inventory 
inventory. Thus, the  changes (because dollar­value LIFO 
specific layers of the  allows the LIFO “layers” to be made 
ending inventory are  up of similar, not necessarily 
adjusted to the current price identical, items).
level. Therefore, answer (b)
B.10.  Gross Profit
is correct. Answers (a), (c), 
and (d) are incorrect  36. (a) The gross profit method can 
because these methods do  be used to estimatethe cost of 
not specifically adjust for  missing inventory. The first step is to
price level changes.  compute the cost of goods available 
34. (b) The requirement is to  for sale.
350 MODULE 10   INVENTORY
$120,000). 

Beginning inventory 39. (a) Before adjustment, the 


Purchases balance in the Accounts 
Cost of goods available for sale Payable account is 
The second step is to estimate cost of  $2,200,000. The $40,000 of
goods sold based on the gross profit  goods lost in transit from a 
percentage. vendor were shipped FOB 
Sales shipping point. This means
Estimated gross profit ($3,200,000 × 25%) the buyer owns the goods 
Cost of goods sold ($3,200,000 × 75%) while they were in transit; 
Note that a shortcut is to realize that  therefore, Kew should 
if gross profit is 25% of sales, cost of  record the purchase and ac­
goods sold must be 75% of sales. The counts payable in 2011. 
third step is to compute estimated  Kew, not the vendor, is 
ending inventory. ultimately responsible for 
Cost of goods available for sale the lost goods (note that 
Estimated cost of goods sold Kew, not the vendor, is 
Estimated ending inventory suing the common carrier). 
The $70,000 return that was
Since the actual count of ending 
recorded on 1/5/12 should 
inventory at December 31 was 
only $575,000, the estimated  have been recorded in 2011 
shortage in inventory is $25,000  when the return was 
($600,000 – $575,000). authorized (December 27, 
2011). Therefore, Kew 
B.14. Cost Apportionment by  should reduce 12/31/11 
Relative Sales Value Accounts Payable by 
$70,000. The $50,000 of 
37. (c) The total cost of acquiring the goods received on 1/6/12 
land and preparingit for sale  were properly recorded in 
($1,200,000 + $300,000 =  2012, since the terms were 
$1,500,000) should be allocated to  FOB destination (the buyer 
the residential lots based on their  does not own the goods 
relative sales value, as computed  until they are physically 
below. received). Therefore, no 
Lot # of Sales adjustment is necessary for 
class lots price this amount. Kew should 
A 100 × $24,000 report 12/31/11 Accounts 
B 100 × 16,000
C 200 × 10,000 Payable at $2,170,000 
($2,200,000 + $40,000 – 
Fraction allocated $70,000). 
Total cost to Class A
× 40. (d) Net sales is $2,300,000, 
$1,500,000 ($2,400/$6,000)
subject to three possible 
C. Items to Include in Inventory adjustments. The goods 
returned ($50,000) should 
38. (d) Before adjustment, the  be re­corded as a return in 
inventory based on a  2011, when Lewis 
physical count was  authorized the return. 
$1,500,000. The $90,000 of
merchandise shipped FOB 
shipping point by a vendor 
on 12/30/11 should also be 
included in Herc’s 12/31/11
inventory because Herc, the
buyer, owns the goods 
while in transit under these 
terms. The goods in the 
shipping area (cost, 
$120,000) are also owned 
by Herc because they were 
not shipped until 2012 and 
Herc still retains the risks 
of ownership until that 
point. Therefore, 12/31/11 
inventory is $1,710,000 
($1,500,000 + $90,000 + 
$10,000 (10 × $1,000). 
Since this return was not recorded  Note that the trans­
until 2012, 2011 sales must be  portation costs ($800) do 
adjusted downward. The goods  not affect sales either; one­
shipped on 12/30/11 ($80,000) were  fourth (10/40) is reflected 
recorded as a sale in 2012 Since the  in cost of goods sold and 
terms were FOB shipping point, the  three­fourths (30/40) is 
sale must be recorded in 2011 when  included in ending 
the goods were shipped; therefore,  inventory. 
2011 sales must be adjusted upward. 
The goods shipped on 1/3/12  43. (d) No adjustment is necessary for the
($100,000) should not be recorded as  goods in transit ($36,000). The goods 
a sale until 2012. Since the sale was  were shipped FOB shipping point, 
recorded in 2011, 2011 sales must be  which means the buyer (Opal) owns 
adjusted downward. Therefore,  the goods while in transit. Therefore, 
adjusted net sales for 2011 should be  Opal properly included these goods in
$2,230,000 ($2,300,000 – $50,000 +  12/31/11 inventory. The 
$80,000 – $100,000). merchandise out on consignment is 
41. (c) Generally, goods are  owned by the consignor (Opal) and 
considered sold when legaltitle to the  should be included in Opal’s 
goods passes to the buyer. In certain  inventory at cost [$40,000 – (40% ×
situations, however, the transfer of  $40,000) = $24,000]. Therefore, 
title criteria does not reflect the  inventory must be reduced by 
underlying economics of the  $16,000 for this item ($40,000 – 
situation. In this situation, although  $24,000). The goods held on con­
transfer of legal title may not have  signment ($27,000) are owned by 
occurred for the 500 segregated  the consignor, not Opal; therefore, 
desks, the economic substance of the  inventory must be reduced $27,000 
trans­action is that the seller no  for this item. The total reduction in 
longer retains the risks of owner­ship. inventory is $43,000 ($16,000 + 
Therefore, all 5,000 desks (including  $27,000).
the 500 segregated and accepted 
desks) are considered sold in 2011,  44. (c) Alt sold 500 of the consigned 
and revenue of $550,000 is  sweaters (505 – 5) at $100 each, 
recognized (5,000 × $110). Note that  resulting in total sales of $50,000 
the amount of cash collected  (500 × $100). Alt must report a 
($450,000) does not affect the  payable to Todd for this amount, less 
amount of revenue recognized in this  Alt’s commission [$50,000 – (10% × 
case. $50,000) = $45,000]. Alt does not 
owe Todd anything for the unsold 
D. Consignments sweaters until they are sold. 
42. (d) A consignor recognizes 45. (c) Inventoriable costs include all 
sales revenue from con­ costs necessary to prepare goods for 
signments when the  sale. These costs include the purchase
consignee sells the  price or manufacturing cost of the 
consigned goods to the  goods, freight, and any other costs 
ultimate customer. Sales  necessary to get the goods to the point
commissions earned by the of sale. The in­transit insurance 
consignee ($10,000 × 15%  premium would therefore be included
= $1,500) are reported as a  in inventory costs. Commissions paid 
selling expense by the  to the consignee are selling expenses 
consignor and are not  in the period the consigned goods are 
netted against sales  sold that are not required to ready the 
revenue. Therefore, sales  goods for sale. These costs, therefore,
revenue is reported at the  are not included in inventory. 
total selling price of 
MODULE 10   INVENTORY 351
inventory, divided by two. Since 
beginning inventory is not given, it 
46. (d) In a consignment, the  must be computed using the cost of 
manufacturer or wholesaleris  goods sold relationship
referred to as the consignor and the  Cost of goods sold $  900,000
dealer or retailer is referred to as the  + Ending inventory 180,000
consignee. In such an arrangement,  Cost of goods available for sale $1,080,000
– Purchases – 960,000
title to the goods remains with the 
Beginning inventory $ 120,000
consignor until they are sold to a 
third party. Jel’s payment of 
reimbursable freight costs results in 
an account receivable from Dale, 
which Jel will subtract from the sale 
proceeds it remits to Dale. An­
swers (a), (b), and (c) are incorrect 
because the consignee, Jel, generally 
does not bear any costs associated 
with the sale of consigned goods.
E. Ratios
47. (b)     The formula to compute
the current ratio is 
Current assets
Current ratio =
Current liabilities
The following entries would be 
recorded when inventory is sold on
account:
Accounts receivable Sales price
Sales of merchandise
Cost of goods sold Cost of
Inventory merchandise
Since the selling price (increase to 
AR) is normally higher than the 
cost of the merchandise sold 
(decrease to merchan­dise 
inventory) the sale would normally 
cause a net increase in current 
assets, and therefore, a net increase 
in the current ratio. When the 
existing current ratio is greater than 
one, increases of equal amounts to 
the numerator (inventory, a 
component of current assets) and 
denominator (accounts payable, a 
component of current liabilities) 
will reduce the ratio. When an 
account receivable is collected, cash
(a cur­rent asset) is increased by the
same amount that accounts 
receivable (another current asset) is 
decreased. Thus, the transaction has
no impact on the current ratio. 
When ma­chinery (a noncurrent 
asset) is purchased for cash (a 
current asset), there is a decrease in
the current ratio.
48. (b)   The formula for inventory
turnover is 
Cost of goods 
sold
Average 
inventory
Average inventory is equal to 
beginning inventory plus ending 
method is generally computed as 
Thus, if cost of goods sold increases  follows:
while inventory remains unchanged, 
then the inventory turnover ratio will
increase.
In addition, we must examine the
effects of the increase in cost of 
goods sold on the gross profit 
percentage when sales remain 
constant. Assuming the same facts as 
above, and sales of $200, we get the 
following results:
Sales 200 200
–  Cost of goods sales – 100 –  150
Gross profit 100 50
Thus, as cost of goods sold 
increases, the gross profit and the 
gross profit percentage will 
decrease.
Answer (a) is incorrect because a 
decrease in cost of goods sold will 
cause the inventory ratio to increase.
Answers (b) and (c) are incorrect 
because they are not related to 
inventory turnover.
50. (b) Average days’ sales in 
inventory measures thenumber of 
days inventory is held before sale;
it reflects on efficiency of 
inventory policies. It is computed 
using the following formula:
365
Inventory 
turnover

F. Long­Term   Construction
Contracts 
51. (a) The total expected income 
on the contract at12/31/11 is 
$900,000 ($9,000,000 – 
$8,100,000). The for­mula for 
recognizing profit under the 
percentage­of­completion method
is
Cost to date Expected
Total expected costs × profit

$6,300,000
$8,100,000 × $900,000
This result is the total profit on the 
contract in 2010 and 2011. The 2010 
profit recognized must be subtracted 
from $700,000 to determine the 2011 
profit. At 12/31/10, the total expected
income on the contract was 
$1,200,000 ($9,000,000 – 
$7,800,000). The income recognized 
in 2010 was $600,000, as computed 
below.
$3,900,000 
$7,800,000 
$1,200,000 = 
×
$600,000
Therefore, 2011 income is 
$700,000 less $600,000, or 
$100,000.
52. (a) Profit to be recognized 
using the percentage­of­completion
Cost to date × Expected – Profit recognized
Total expected cost profit in previous periods

Therefore, average inventory is  Not enough information is given in 
$150,000 [($120,000 + $180,000) ÷  this problem to perform this 
2], and inventory turnover is 6.0  computation, so 2011 profit must be 
times ($900,000 ÷ 150,000). computed indirectly. Since only 
construction expenses and profit are 
49. (d) The solutions approach is to  debited to the construction­in­
create a numericalexample that  progress (CIP account), 2010 profit 
conforms to the facts given in the  must have been $17,000 ($122,000 
question. The inventory turnover  CIP less $105,000 const. exp.). 
ratio is calculated as follows: Cumulative profit recognized by the 
Cost of goods  end of 2011 must be $67,000 
sold [$364,000 CIP less $297,000 
Average  cumulative const. exp. ($105,000 + 
inventory $192,000)]. Therefore, 2011 profit 
was $50,000 ($67,000 − $17,000).
If we assume that cost of goods sold
has increased from 100 to 150 and
average   inventory   has   remained
unchanged at 50 then the following
ratios result:
Cost of goods sold 100
= 2
Average inventory 50  
CIP
2010 Exp. 105,000
2010 Profit ? 2010 Profit = $17,000
2010 End. bal 122,000
2011 Exp. 192,000
2011 Profit ? 2011 Profit = $50,000
2011 End. bal. 364,000
352 MODULE 10   INVENTORY
Costs to date × Estimated profit  =  Gross profit to date
Total estimated costs
53. (b) Based on the 
information given, it must  Pell would recognize gross profit of 
be as­sumed that costs  $40,000 on project 1
incurred are used to  $240,000
measure the extent of  $240,000 + $120,000 × [$420,000 – ($240,000 + $120,000)] =$40,000
progress toward project 
Note that prior years’ gross profit 
completion. At 12/31/10, 
need not be subtracted from $40,000 
the project was 20% 
because the project commenced 
complete and total 
during 2011. Under both the 
estimated costs were 
percentage­of­completion method and
$7,500,000. Therefore, costs
the completed­contract method, an 
incurred as of 12/31/10 
expected loss must be recognized in 
were 20% of $7,500,000, or 
$1,500,000. At 12/31/11,  full in the period in which the 
the project was 60%  expected loss is discovered. Project 2 
complete and total  has an expected loss of ($20,000) 
estimated costs were  [$300,000 – ($280,000 + $40,000)] 
$8,000,000. Therefore, costs which must be recognized in full in 
incurred as of 12/31/11 are  2011. The net gross profit recognized 
60% of $8,000,000 or  on the two projects is $20,000 
$4,800,000. The costs  ($40,000 profit less ($20,000) loss.
incurred during 2011 were  57. (a) In the final year of a contract 
$4,800,000 less $1,500,000, accounted for bythe percentage­of­
or $3,300,000.  completion method, the percentage of
54. (b) The requirement is to  completion is 100%, since costs to 
determine the amount of  date equal total costs. Therefore, the 
formula to calculate income to be 
loss to recognize in 2011 on 
recognized in the final year is simply
a long­term, fixed­price con­ Inc
struction contract. Under  om
e
both the percentage­of­ pre
completion method and the  vio
usl
completed­contract method,  y
an expected loss on a 
contract must be recognized 
in full in the period in which
the expected loss is 
discovered. Therefore, 
Hanson must recognize a 
loss of $100,000 in 2011. 
Expected contract revenue
Expected contract costs ($930,000 + $2,170,000)
Expected loss

55. (a)       The expected income on


project 1 [$420,000 – 
($240,000 + $120,000) = $60,000] is
not recognized until the project is 
completed under the completed 
contract method. However, under the
completed contract method, an 
expected loss on a contract must be 
recognized in full in the period 
inwhich it is discovered. Project 2 
has an expected loss of ($20,000) 
[$300,000 − ($280,000 + $40,000)] 
which must be recognized 
immediately in 2011.
56. (b) Construction companies that
use   the   percentage­of­completion
method in accounting for long­term
construc­tion   contracts   usually
recognize gross profit according to
the cost­to­cost method.
liabilities as current. On the 
particular year. The formula to  balance sheet, the 
calculate current income is as  Construction in Progress 
follows: (CIP) account is netted with 
the contra account, progress 
Cost to date × Expected
Total expected cost profit
billings. If CIP exceeds 
billings, the excess is 
Progress billings do not impact the  reported as a current asset 
amount of income recog­nized. [answer (a)]. If billings 
exceed CIP, the excess is 
59. (a) The requirement is to  reported as a current 
determine the ratio to beused to  liability. Answers (b) and 
calculate income in the third year  (c) are incorrect because the
using the percentage­of­ accounts related to 
completion method. Income may  construction contracts are 
be recog­nized on a cost­to­cost  classified as current. 
basis when the percentage­of­ Answer (d) is incorrect 
completion method is used. because progress billings is 
not used as a basis for 
Cost to date × Expected
Total expected cost profit
recognizing revenues. 

Answers (b) and (d) are incorrect  G. International   Financial


because billings to date are not used  Reporting Standards (IFRS) 
as a basis for recognizing revenues. 
62. (c)    The requirement is to 
Answer (c) is incorrect as the cost­to­
cost calculation requires a cumulative calculate the amount that
calculation so changes in expected  should be presented for inventory.  
costs and expected income can be  Answer (c) is correct be­
adjusted for; thus, costs incurred in  cause the lower of cost or net realizable 
year three would be an incorrect basis value method requires
for recognition of income. net realizable value to be calculated as
the estimated selling
60. (d) An anticipated loss on a long­term price less estimated costs of completion 
contract should be recognized  and estimated costs to
immediately under both the  sell.  Therefore, the NRV is $850 
percentage­of­completion and the  ($900 – $50).  The lower of
completed­contract methods.  cost or net realizable value is 
determined by comparing the
61. (a) The requirement is to 
cost of $1,000 to the NRV of $850, 
determine the proper clas­ and using the lower
sification for the progress 
amount.  Inventory should be reported
billings on contracts account
at $850.
under the percentage­of­
completion method. In the  63. (c)    The requirement is to 
construction industry,  determine the amount that
operating cycles for  should be reported as inventory.  
construction contracts  Answer (c) is correct be­
generally exceed one year.  cause under IFRS inventory is 
Therefore, the predominant  reported at the lower of cost or
practice is to classify all  net realizable value.  Therefore, the 
contract­related assets and  amount is $90,000, which
(Contract price –
Actual costs)
r
e is the lower of $100,000 cost or 
$90,000 net realizable value.
Actual
income 64. (c)    The requirement is to 
identify the inventory items
that are not valued at the lower of cost or net realizable value
Therefore both actual total costs  under IFRS.  Answer (c) is correct 
and income  because biological inven­
previouslyrecognized are used in tory items are valued at fair value 
calculating income. less the cost to sell at the
58. (b) Under the percentage­of­ point of harvest.
completion method ofaccounting for 
long­term contracts, the cost­to­cost 
formula is used to compute the 
amount of income to be recognized in 
a
MODULE 10   INVENTORY 353

65. (c)
Th

re
qu
ire
m
en

is 
to 
id
en
tif

w
he

th

sp
ec
ifi

id
en
tif
ic
ati
on
m
et
ho

is 
re
qu
ire

un
de

IF
R
S. 
A
ns
w
er 
(c)
is 
co
rre
ct 
be
ca
us

th

sp
ec
ifi

id
en
tif
ic
ati
on
m
et
ho

is 
re
qu
ire

fo

in
ve
nt
or

ite
m

th
at 
ar

no

int
er
ch
an
ge
ab
le 
an

go
od

th
at 
ar

pr
od
uc
ed
an

se
gr
eg
at
ed
fo

sp
ec
ifi

pr
oj
ec
ts.
66. (b

T
he
re
qu
ir
e
m
en

is 
to 
de
te
r
m
in

th

va
lu

of
th

in
ve
nt
or

ite

un
de

IF
R
S.
A
ns
w
er 
(b

is 
co
rr
ec

be
ca
us

un
de

IF
R
S,
in
ve
nt
or

is 
pr
es
en
te

at 
th

lo
w
er 
of
co
st 
($
1,
20
0)
or
ne

re
ali
za
bl

va
lu

($
1,
30

se
lli
ng
pr
ic

– 
$1
50
es
ti
m
at
ed
co
m
pl
eti
on
an

se
lli
ng
co
st
s).
T
he
re
fo
re,
th

ite

sh
ou
ld 
be
va
lu
ed
at 
$1
,0
50

67. (d) 
The
requ
ire
men
t is 
to 
iden
tify 
the 
stat
eme
nt 
that 
is 
not 
corr
ect 
abo
ut 
acc
ount
ing 
for 
inve
ntor
ies 
und
er 
IFR
S. 
An­
swe
r (d)
is 
corr
ect 
bec
ause
inve
ntor
ies 
are 
not 
alw
ays 
valu
ed 
at 
net 
reali
zabl

valu
e. 
The

are 
valu
ed 
at 
the 
low
er 
of 
cost
or 
net 
reali
zabl

valu
e. 
68. (a) 
The
req
uire
me
nt 
is 
to 
ide
ntif

the 
met
hod
of 
ac­
cou
ntin

that
is 
not 
allo
wed
und
er 
IFR
S. 
Ans
wer
(a) 
is 
cor­
rect
bec
aus

the 
LIF

met
hod
is 
not 
allo
wed
und
er 
IFR
S. 
All 
of 
the 
oth
er 
met
hod

are 
allo
wed

354 MODULE 10   INVENTORY

SOLUTIONS TO SIMULATIONS

Task­Based Simulation 1
Concepts
Authoritative
Literature Help

Included Not included
1. Merchandise purchased for resale
2. Freight­out
3. Direct materials
4. Sales returns
5. Packaging for shipment to customer
6. Factory overhead
7. Interest on inventory loan
8. Purchase discounts not taken
9. Freight­in
10. Direct labor
Explanation of solutions
1. Merchandise purchased for resale is a part of inventory. 
2. Freight­out is part of selling expense. 
3. Direct materials are part of work in process, which is an inventory account
for manufacturers. 
4. Sales returns are a contra account to sales, and are not part of inventory. 
5. Packaging for shipment to customer is part of selling expenses. 
6. Factory overhead is part of work in process, which is an inventory account
for manufacturing firms. 
7. Interest on inventory loan is an operating expense. Interest cost should not be
capitalized for assets  that are  in use or ready for their intended use in the
earnings activities of the enterprise. 
8. Purchase   discounts   lost   are   considered   a   financing   expense   and   are
excluded from the cost of inventory. 
9. Freight­in   is   part   of   getting   the   goods   in   place   to   sell   and   should   be
included in inventory. 
10. Direct labor is part of work in process, and is an inventory account for
manufacturing firms. 
Task­Based Simulation 2
Journal Entries
Authoritative
Literature Help

1. On January 5, 2011, purchased $17,000 of garden tillers on
account   from   Bestbuilt   Tillers,   terms   2/10,   n/30,   FOB
destination. Freight charges were $200. 
Purchases 17,000
Accounts payable 17,000

2.  On January 10, 2011, returned garden tillers worth $2,000 to Bestbuilt 
Tillers due to defects.
Accounts payable 2,000
Purchase returns 2,000

3.  On January 14, 2011, paid for the remaining tillers purchased in 1.
Accounts payable 15,000
Purchase discounts 300*
Cash 14,700
*
 15,000 × .02 = 300

Вам также может понравиться